Uploaded by cholpon2008

rudin u osnovy matematicheskogo analiza

advertisement
Основы математического анализа
У. Рудин
Оглавление
1 Системы вещественных и комплексных
Введение . . . . . . . . . . . . . . . . . . . . .
Упорядоченные множества . . . . . . . . . . .
Поля . . . . . . . . . . . . . . . . . . . . . . . .
Поле вещественных чисел . . . . . . . . . . .
Расширенная система вещественных чисел .
Поле комплексных чисел . . . . . . . . . . . .
Евклидовы пространства . . . . . . . . . . . .
Приложение . . . . . . . . . . . . . . . . . . .
Упражнения . . . . . . . . . . . . . . . . . . .
чисел
. . . . .
. . . . .
. . . . .
. . . . .
. . . . .
. . . . .
. . . . .
. . . . .
. . . . .
.
.
.
.
.
.
.
.
.
.
.
.
.
.
.
.
.
.
.
.
.
.
.
.
.
.
.
.
.
.
.
.
.
.
.
.
.
.
.
.
.
.
.
.
.
.
.
.
.
.
.
.
.
.
.
.
.
.
.
.
.
.
.
.
.
.
.
.
.
.
.
.
.
.
.
.
.
.
.
.
.
.
.
.
.
.
.
.
.
.
.
.
.
.
.
.
.
.
.
.
.
.
.
.
.
.
.
.
.
.
.
.
.
.
.
.
.
.
.
.
.
.
.
.
.
.
.
.
.
.
.
.
.
.
.
.
.
.
.
.
.
.
.
.
.
.
.
.
.
.
.
.
.
.
.
.
.
.
.
.
.
.
.
.
.
.
.
.
.
.
.
.
.
.
.
.
.
.
.
.
.
.
.
.
.
.
.
.
.
.
.
.
.
.
.
.
.
.
3
3
4
6
8
10
10
13
14
17
2 Элементы топологии
Конечные, счетные и несчетные
Метрические пространства . . .
Компактные множества . . . .
Совершенные множества . . . .
Связные множества . . . . . . .
Упражнения . . . . . . . . . . .
.
.
.
.
.
.
.
.
.
.
.
.
.
.
.
.
.
.
.
.
.
.
.
.
.
.
.
.
.
.
.
.
.
.
.
.
.
.
.
.
.
.
.
.
.
.
.
.
.
.
.
.
.
.
.
.
.
.
.
.
.
.
.
.
.
.
.
.
.
.
.
.
.
.
.
.
.
.
.
.
.
.
.
.
.
.
.
.
.
.
.
.
.
.
.
.
.
.
.
.
.
.
.
.
.
.
.
.
.
.
.
.
.
.
.
.
.
.
.
.
.
.
.
.
.
.
.
.
.
.
.
.
.
.
.
.
.
.
.
.
.
.
.
.
.
.
.
.
.
.
.
.
.
.
.
.
.
.
.
.
.
.
.
.
.
.
.
.
19
19
23
28
31
32
32
3 Числовые последовательности и ряды
Сходящиеся последовательности . . . . . . .
Подпоследовательности . . . . . . . . . . . . .
Последовательности Коши . . . . . . . . . . .
Верхний и нижний пределы . . . . . . . . . .
Некоторые специальные последовательности
Ряды . . . . . . . . . . . . . . . . . . . . . . .
Ряды с неотрицательными членами . . . . .
Число e . . . . . . . . . . . . . . . . . . . . . .
Другие признаки сходимости . . . . . . . . .
Степенные ряды . . . . . . . . . . . . . . . . .
Суммирование по частям . . . . . . . . . . . .
Абсолютная сходимость . . . . . . . . . . . .
Сложение и умножение рядов . . . . . . . . .
Перестановки рядов . . . . . . . . . . . . . . .
Упражнения . . . . . . . . . . . . . . . . . . .
.
.
.
.
.
.
.
.
.
.
.
.
.
.
.
.
.
.
.
.
.
.
.
.
.
.
.
.
.
.
.
.
.
.
.
.
.
.
.
.
.
.
.
.
.
.
.
.
.
.
.
.
.
.
.
.
.
.
.
.
.
.
.
.
.
.
.
.
.
.
.
.
.
.
.
.
.
.
.
.
.
.
.
.
.
.
.
.
.
.
.
.
.
.
.
.
.
.
.
.
.
.
.
.
.
.
.
.
.
.
.
.
.
.
.
.
.
.
.
.
.
.
.
.
.
.
.
.
.
.
.
.
.
.
.
.
.
.
.
.
.
.
.
.
.
.
.
.
.
.
.
.
.
.
.
.
.
.
.
.
.
.
.
.
.
.
.
.
.
.
.
.
.
.
.
.
.
.
.
.
.
.
.
.
.
.
.
.
.
.
.
.
.
.
.
.
.
.
.
.
.
.
.
.
.
.
.
.
.
.
.
.
.
.
.
.
.
.
.
.
.
.
.
.
.
.
.
.
.
.
.
.
.
.
.
.
.
.
.
.
.
.
.
.
.
.
.
.
.
.
.
.
.
.
.
.
.
.
.
.
.
.
.
.
.
.
.
.
.
.
.
.
.
.
.
.
.
.
.
.
.
.
.
.
.
.
.
.
.
.
.
.
.
.
.
.
.
.
.
.
.
.
.
.
.
.
.
.
.
.
.
.
.
.
.
.
.
.
.
.
.
.
.
.
.
.
.
.
.
.
.
.
.
.
.
.
.
.
.
.
.
.
.
.
.
.
.
.
.
.
.
.
.
.
.
.
.
.
.
.
.
.
.
.
.
.
.
.
.
.
.
.
.
.
.
.
.
.
.
.
.
.
.
.
.
.
.
.
.
.
.
.
.
.
.
.
.
.
.
.
.
.
.
.
.
35
35
38
38
40
42
43
44
46
48
50
51
52
52
55
56
4 Непрерывность
Предел функции . . . . . . . . . . . . . . . . . . . .
Непрерывные функции . . . . . . . . . . . . . . . .
Непрерывность и компактность . . . . . . . . . . .
Непрерывность и связность . . . . . . . . . . . . .
Разрывы функций . . . . . . . . . . . . . . . . . . .
Монотонные функции . . . . . . . . . . . . . . . . .
Бесконечные пределы и пределы в бесконечности
Упражнения . . . . . . . . . . . . . . . . . . . . . .
.
.
.
.
.
.
.
.
.
.
.
.
.
.
.
.
.
.
.
.
.
.
.
.
.
.
.
.
.
.
.
.
.
.
.
.
.
.
.
.
.
.
.
.
.
.
.
.
.
.
.
.
.
.
.
.
.
.
.
.
.
.
.
.
.
.
.
.
.
.
.
.
.
.
.
.
.
.
.
.
.
.
.
.
.
.
.
.
.
.
.
.
.
.
.
.
.
.
.
.
.
.
.
.
.
.
.
.
.
.
.
.
.
.
.
.
.
.
.
.
.
.
.
.
.
.
.
.
.
.
.
.
.
.
.
.
.
.
.
.
.
.
.
.
.
.
.
.
.
.
.
.
.
.
.
.
.
.
.
.
.
.
.
.
.
.
.
.
.
.
.
.
.
.
.
.
.
.
.
.
.
.
.
.
.
.
.
.
.
.
.
.
61
61
62
65
68
68
69
71
72
множества
. . . . . . .
. . . . . . .
. . . . . . .
. . . . . . .
. . . . . . .
2
5 Дифференцирование
Производная вещественной функции .
Теоремы о среднем значении . . . . . .
Непрерывность производных . . . . .
Правило Лопиталя . . . . . . . . . . .
Производные высших порядков . . . .
Теорема Тейлора . . . . . . . . . . . . .
Дифференцирование векторнозначных
Упражнения . . . . . . . . . . . . . . .
.
.
.
.
.
.
.
.
.
.
.
.
.
.
.
.
.
.
.
.
.
.
.
.
.
.
.
.
.
.
.
.
.
.
.
.
.
.
.
.
.
.
.
.
.
.
.
.
.
.
.
.
.
.
.
.
.
.
.
.
.
.
.
.
.
.
.
.
.
.
.
.
.
.
.
.
.
.
.
.
.
.
.
.
.
.
.
.
.
.
.
.
.
.
.
.
.
.
.
.
.
.
.
.
.
.
.
.
.
.
.
.
.
.
.
.
.
.
.
.
.
.
.
.
.
.
.
.
.
.
.
.
.
.
.
.
.
.
.
.
.
.
.
.
.
.
.
.
.
.
.
.
.
.
.
.
.
.
.
.
.
.
.
.
.
.
.
.
.
.
.
.
.
.
.
.
.
.
.
.
.
.
.
.
.
.
.
.
.
.
.
.
.
.
.
.
.
.
.
.
75
75
77
78
79
80
80
81
83
.
.
.
.
.
.
.
.
.
.
.
.
.
.
.
.
.
.
.
.
.
.
.
.
.
.
.
.
.
.
.
.
.
.
.
.
.
.
.
.
.
.
.
.
.
.
.
.
.
.
.
.
.
.
.
.
.
.
.
.
.
.
.
.
.
.
.
.
.
.
.
.
.
.
.
.
.
.
.
.
.
.
.
.
.
.
.
.
.
.
.
.
.
.
.
.
.
.
.
.
.
.
.
.
.
.
.
.
.
.
.
.
.
.
.
.
.
.
.
.
.
.
.
.
.
.
.
.
.
.
.
.
.
.
.
.
.
.
.
.
.
.
.
.
.
.
.
.
.
.
.
.
.
.
.
.
88
88
93
98
99
99
101
7 Последовательности и ряды функций
Обсуждение главной проблемы . . . . . . . . . .
Равномерная сходимость . . . . . . . . . . . . . .
Равномерная сходимость и непрерывность . . . .
Равномерная сходимость и интегрирование . . .
Равномерная сходимость и дифференцирование
Равностепенно непрерывные семейства функций
Теорема Стоуна-Вейерштрасса . . . . . . . . . .
Упражнения . . . . . . . . . . . . . . . . . . . . .
.
.
.
.
.
.
.
.
.
.
.
.
.
.
.
.
.
.
.
.
.
.
.
.
.
.
.
.
.
.
.
.
.
.
.
.
.
.
.
.
.
.
.
.
.
.
.
.
.
.
.
.
.
.
.
.
.
.
.
.
.
.
.
.
.
.
.
.
.
.
.
.
.
.
.
.
.
.
.
.
.
.
.
.
.
.
.
.
.
.
.
.
.
.
.
.
.
.
.
.
.
.
.
.
.
.
.
.
.
.
.
.
.
.
.
.
.
.
.
.
.
.
.
.
.
.
.
.
.
.
.
.
.
.
.
.
.
.
.
.
.
.
.
.
.
.
.
.
.
.
.
.
.
.
.
.
.
.
.
.
.
.
.
.
.
.
.
.
.
.
.
.
.
.
.
.
.
.
.
.
.
.
.
.
.
.
.
.
.
.
.
.
.
.
.
.
.
.
.
.
105
105
108
109
111
112
113
116
121
8 Некоторые специальные функции
Степенные ряды . . . . . . . . . . . . . . . . . . . . . . .
Показательная и логарифмическая функции . . . . . .
Тригонометрические функции . . . . . . . . . . . . . . .
Алгебраическая замкнутость поля комплексных чисел
Ряды Фурье . . . . . . . . . . . . . . . . . . . . . . . . .
Гамма-функция . . . . . . . . . . . . . . . . . . . . . . .
Упражнения . . . . . . . . . . . . . . . . . . . . . . . . .
.
.
.
.
.
.
.
.
.
.
.
.
.
.
.
.
.
.
.
.
.
.
.
.
.
.
.
.
.
.
.
.
.
.
.
.
.
.
.
.
.
.
.
.
.
.
.
.
.
.
.
.
.
.
.
.
.
.
.
.
.
.
.
.
.
.
.
.
.
.
.
.
.
.
.
.
.
.
.
.
.
.
.
.
.
.
.
.
.
.
.
.
.
.
.
.
.
.
.
.
.
.
.
.
.
.
.
.
.
.
.
.
.
.
.
.
.
.
.
.
.
.
.
.
.
.
.
.
.
.
.
.
.
.
.
.
.
.
.
.
.
.
.
.
.
.
.
126
126
131
133
135
136
141
144
9 Функции нескольких переменных
Линейные преобразования . . . . . . . . . .
Дифференцирование . . . . . . . . . . . . .
Принцип сжимающих отображений . . . . .
Теорема об обратной функции . . . . . . . .
Теорема о неявной функции . . . . . . . . .
Теорема о ранге . . . . . . . . . . . . . . . .
Определители . . . . . . . . . . . . . . . . .
Производные высшего порядка . . . . . . .
Дифференцирование под знаком интеграла
Упражнения . . . . . . . . . . . . . . . . . .
. . . . . .
. . . . . .
. . . . . .
. . . . . .
. . . . . .
. . . . . .
функций
. . . . . .
6 Интеграл Римана-Стильтьеса
Определение и существование интеграла .
Свойства интеграла . . . . . . . . . . . . . .
Интегрирование и дифференцирование . .
Интегрирование векторнозначных функций
Спрямляемые кривые . . . . . . . . . . . . .
Упражнения . . . . . . . . . . . . . . . . . .
10 Интегрирование дифференциальных
Интегрирование . . . . . . . . . . . . . . .
Простые отображения . . . . . . . . . . .
Разбиения единицы . . . . . . . . . . . . .
Замена переменных . . . . . . . . . . . . .
Дифференциальные формы . . . . . . . .
Симплексы и цепи . . . . . . . . . . . . . .
Теорема Стокса . . . . . . . . . . . . . . .
.
.
.
.
.
.
.
.
.
.
.
.
.
.
.
.
.
.
.
.
.
.
.
.
.
.
.
.
.
.
.
.
.
.
.
.
.
.
.
.
.
.
.
.
.
.
.
.
.
.
.
.
.
.
.
.
.
.
.
.
.
.
.
.
.
.
.
.
.
.
.
.
.
.
.
.
.
.
.
.
.
.
.
.
.
.
.
.
.
.
.
.
.
.
.
.
.
.
.
.
.
.
.
.
.
.
.
.
.
.
.
.
.
.
.
.
.
.
.
.
.
.
.
.
.
.
.
.
.
.
.
.
.
.
.
.
.
.
.
.
.
.
.
.
.
.
.
.
.
.
.
.
.
.
.
.
.
.
.
.
.
.
.
.
.
.
.
.
.
.
.
.
.
.
.
.
.
.
.
.
.
.
.
.
.
.
.
.
.
.
.
.
.
.
.
.
.
.
.
.
.
.
.
.
.
.
.
.
.
.
.
.
.
.
.
.
.
.
.
.
.
.
.
.
.
.
.
.
.
.
.
.
.
.
.
.
.
.
.
.
.
.
.
.
.
.
.
.
.
.
.
.
.
.
.
.
.
.
.
.
.
.
150
150
155
162
163
165
168
171
173
174
176
форм
. . . . .
. . . . .
. . . . .
. . . . .
. . . . .
. . . . .
. . . . .
.
.
.
.
.
.
.
.
.
.
.
.
.
.
.
.
.
.
.
.
.
.
.
.
.
.
.
.
.
.
.
.
.
.
.
.
.
.
.
.
.
.
.
.
.
.
.
.
.
.
.
.
.
.
.
.
.
.
.
.
.
.
.
.
.
.
.
.
.
.
.
.
.
.
.
.
.
.
.
.
.
.
.
.
.
.
.
.
.
.
.
.
.
.
.
.
.
.
.
.
.
.
.
.
.
.
.
.
.
.
.
.
.
.
.
.
.
.
.
.
.
.
.
.
.
.
.
.
.
.
.
.
.
.
.
.
.
.
.
.
.
.
.
.
.
.
.
.
.
.
.
.
.
.
.
.
.
.
.
.
.
.
.
.
.
.
.
.
181
181
183
185
186
187
196
201
.
.
.
.
.
.
.
.
.
.
.
.
.
.
.
.
.
.
.
.
3
.
.
.
.
.
.
.
.
.
.
Замкнутые формы и точные формы . . . . . . . . . . . . . . . . . . . . . . . . . . . . . . . . 203
Векторный анализ . . . . . . . . . . . . . . . . . . . . . . . . . . . . . . . . . . . . . . . . . . . 207
Упражнения . . . . . . . . . . . . . . . . . . . . . . . . . . . . . . . . . . . . . . . . . . . . . . 213
11 Теория Лебега
Функции множеств . . . . . . . . . . . .
Построение меры Лебега . . . . . . . . .
Пространства с мерой . . . . . . . . . . .
Измеримые функции . . . . . . . . . . .
Простые функции . . . . . . . . . . . . .
Интегрирование . . . . . . . . . . . . . .
Сравнение с интегралом Римана . . . .
Интегрирование комплексных функций
Функции класса ℒ 2 . . . . . . . . . . . .
Упражнения . . . . . . . . . . . . . . . .
.
.
.
.
.
.
.
.
.
.
.
.
.
.
.
.
.
.
.
.
.
.
.
.
.
.
.
.
.
.
.
.
.
.
.
.
.
.
.
.
4
.
.
.
.
.
.
.
.
.
.
.
.
.
.
.
.
.
.
.
.
.
.
.
.
.
.
.
.
.
.
.
.
.
.
.
.
.
.
.
.
.
.
.
.
.
.
.
.
.
.
.
.
.
.
.
.
.
.
.
.
.
.
.
.
.
.
.
.
.
.
.
.
.
.
.
.
.
.
.
.
.
.
.
.
.
.
.
.
.
.
.
.
.
.
.
.
.
.
.
.
.
.
.
.
.
.
.
.
.
.
.
.
.
.
.
.
.
.
.
.
.
.
.
.
.
.
.
.
.
.
.
.
.
.
.
.
.
.
.
.
.
.
.
.
.
.
.
.
.
.
.
.
.
.
.
.
.
.
.
.
.
.
.
.
.
.
.
.
.
.
.
.
.
.
.
.
.
.
.
.
.
.
.
.
.
.
.
.
.
.
.
.
.
.
.
.
.
.
.
.
.
.
.
.
.
.
.
.
.
.
.
.
.
.
.
.
.
.
.
.
.
.
.
.
.
.
.
.
.
.
.
.
.
.
.
.
.
.
.
.
.
.
.
.
.
.
.
.
.
.
.
.
.
.
.
.
.
.
.
.
221
221
223
228
229
231
231
238
239
240
245
Глава 1
Системы вещественных и
комплексных чисел
Введение
Изучение основных понятий анализа (таких, как сходимость, непрерывность, дифференцирование
и интегрирование) должно основываться на точно определенном понятии числа. Мы, однако, не
будем вступать в обсуждение аксиом, которым подчиняется арифметика целых чисел, а будем
предполагать знакомство читателя с рациональными числами (т. е. числами вида 𝑚/𝑛, где 𝑚 и 𝑛 —
целые, 𝑛 ≠ 0).
Система рациональных чисел обладает многими недостатками и как поле, и как упорядоченное
множество (эти понятия будут определены в пп. 1.6 и 1.12). Например, не существует рационального числа 𝑝, такого, что 𝑝2 = 2 (мы это вскоре докажем). Это делает необходимым введение так называемых «иррациональных чисел», которые часто записываются в виде бесконечных десятичных
разложений, причем соответствующие конечные десятичные дроби считаются их приближениями.
Так, последовательность
1; 1,4; 1,41; 1,414; 1,4142; …
«стремится к √2». Но до тех пор, пока мы не определили иррациональное число √2, остается
открытым вопрос: к чему же все-таки «стремится» эта последовательность?
Мы сможем ответить на этот вопрос, когда построим так называемую «систему вещественных
чисел».
1.1 Пример. Сначала покажем, что никакое рациональное число 𝑝 не удовлетворяет уравнению
𝑝2 = 2.
(1)
Если бы такое 𝑝 существовало, мы могли бы записать его в виде 𝑝 = 𝑚/𝑛, где 𝑚, 𝑛 — целые, причем
хотя бы одно из них нечетно. Подставляя в уравнение (2), получаем
𝑚2 = 2𝑛2 .
(2)
Это показывает, что 𝑚2 — четное число. Значит, 𝑚 четно (если бы 𝑚 было нечетным, то и 𝑚2 было
бы нечетным) и, следовательно, 𝑚2 делится на 4. Поэтому правая часть равенства (2) делится на
4, так что 𝑛2 четно, откуда следует, что и 𝑛 четно.
Исследуем теперь подробнее эту ситуацию. Пусть 𝐴 — множество всех положительных рациональных 𝑝, таких, что 𝑝2 < 2, и пусть множество 𝐵 состоит из всех положительных рациональных
𝑝, таких, что 𝑝2 > 2. Мы покажем, что 𝐴 не содержит наибольшего числа, а 𝐵 не содержит
наименьшего.
Точнее, для любого 𝑝 из 𝐴 можно найти рациональное число 𝑞 из 𝐴, такое, что 𝑝 < 𝑞, и для
любого 𝑝 из 𝐵 мы можем найти рациональное число 𝑞 из 𝐵, такое, что 𝑞 < 𝑝.
Для этого мы свяжем с каждым рациональным числом 𝑝 > 0 число
(3)
𝑞 =𝑝−
𝑝2 − 2 2𝑝 + 2
=
.
𝑝+2
𝑝+2
5
Тогда
(4)
𝑞2 − 2 =
2(𝑝2 − 2)
.
(𝑝 + 2)2
Если 𝑝 принадлежит 𝐴, то 𝑝2 − 2 < 0, из (3) следует, что 𝑞 > 𝑝, а из (4) следует, что 𝑞 2 < 2.
Следовательно, 𝑞 принадлежит 𝐴.
Если 𝑝 принадлежит 𝐵, то 𝑝2 − 2 > 0, из (3) следует, что 0 < 𝑞 < 𝑝, а из (4) следует, что 𝑞 2 > 2.
Следовательно, 𝑞 принадлежит 𝐵.
1.2 Замечание. Цель приведенного выше рассуждения — показать, что в системе рациональных
чисел имеются некоторые пробелы, несмотря на то, что между любыми двумя рациональными
числами находится третье: если 𝑟 < 𝑠, то 𝑟 < (𝑟 + 𝑠)/2 < 𝑠. Система вещественных чисел заполняет
эти пробелы. Именно по этой причине она играет фундаментальную роль в анализе.
Чтобы разъяснить структуру систем вещественных и комплексных чисел, мы начнем с краткого
обсуждения общих понятий упорядоченного множества и поля.
Далее следует стандартная терминология теории множеств, которая будет использоваться в
книге.
1.3 Определения. Если 𝐴 — множество (элементами которого могут быть числа или любые другие объекты), то запись 𝑥 ∈ 𝐴 означает, что 𝑥 принадлежит (или является элементом) 𝐴.
Если 𝑥 не является элементом 𝐴, мы будем писать 𝑥 ∉ 𝐴.
Множество, не содержащее ни одного элемента, мы будем называть пустым множеством. Если
множество содержит хотя бы один элемент, то оно называется непустым.
Мы будем говорить, что множество 𝐴 есть подмножество множества 𝐵, и писать 𝐴 ⊂ 𝐵 или
𝐵 ⊃ 𝐴, если каждый элемент множества 𝐴 является элементом множества 𝐵. Если, кроме того, в 𝐵
имеется элемент, не принадлежащий 𝐴, то 𝐴 называется собственным подмножеством множества
𝐵. В частности, 𝐴 ⊂ 𝐴, каково бы ни было множество 𝐴.
Если 𝐴 ⊂ 𝐵 и 𝐵 ⊂ 𝐴, то мы будем писать 𝐴 = 𝐵. В противном случае 𝐴 ≠ 𝐵.
1.4 Определение. В гл. 1 множество рациональных чисел будет обозначаться через 𝑄.
Упорядоченные множества
1.5 Определение. Пусть 𝑆 — множество. Порядком на 𝑆 называется отношение, обозначаемое
знаком <, обладающее двумя свойствами:
(i) Если 𝑥 ∈ 𝑆 и 𝑦 ∈ 𝑆, то верно ровно одно из утверждений
𝑥 < 𝑦,
𝑥 = 𝑦,
𝑦 < 𝑥.
(ii) Если 𝑥, 𝑦, 𝑧 ∈ 𝑆, 𝑥 < 𝑦 и 𝑦 < 𝑧, то 𝑥 < 𝑧.
Утверждение 𝑥 < 𝑦 можно читать как «𝑥 меньше 𝑦» или «𝑥 предшествует 𝑦».
Вместо 𝑥 < 𝑦 часто удобно писать 𝑦 > 𝑥.
Символ 𝑥 ≤ 𝑦 означает, что 𝑥 < 𝑦 или 𝑥 = 𝑦, без уточнения, какое именно из утверждений верно.
Другими словами, 𝑥 ≤ 𝑦 — отрицание 𝑥 > 𝑦.
1.6 Определение. Упорядоченным множеством называется множество 𝑆, на котором определен
порядок.
Например, 𝑄 становится упорядоченным множеством, если определить 𝑟 < 𝑠 как «𝑠 − 𝑟 — положительное рациональное число».
1.7 Определение. Пусть 𝑆 — упорядоченное множество, а 𝐸 ⊂ 𝑆. Если существует 𝛽 ∈ 𝑆, такое,
что 𝑥 ≤ 𝛽 для всех 𝑥 ∈ 𝐸, то мы будем говорить, что 𝐸 ограничено сверху, а 𝛽 будем называть
верхней границей множества 𝐸.
Нижние границы определяются аналогичным образом (с заменой ≤ на ≥).
1.8 Определение. Пусть 𝑆 — упорядоченное множество, 𝐸 ⊂ 𝑆 и 𝐸 ограничено сверху. Предположим, что существует 𝛼 ∈ 𝑆, обладающее следующими свойствами:
6
(i) 𝛼 является верхней границей множества 𝐸;
(ii) если 𝛾 < 𝛼, то 𝛾 не является верхней границей множества 𝐸.
Тогда 𝛼 называется верхней гранью (точной верхней границей) множества 𝐸 (как следует из
(ii), существует не более чем одно такое 𝛼). Мы будем писать
𝛼 = sup 𝐸.
Нижняя грань (точная нижняя граница) ограниченного снизу множества 𝐸 определяется аналогично: утверждение
𝛼 = inf 𝐸
означает, что 𝛼 — нижняя граница множества 𝐸 и ни одно 𝛽, такое, что 𝛽 > 𝛼, не является нижней
границей множества 𝐸.
1.9 Примеры.
(a) Рассмотрим множества 𝐴 и 𝐵 из примера 1.1 как подмножества упорядоченного множества 𝑄.
Множество 𝐴 ограничено сверху. Верхние границы множества 𝐴 — это в точности элементы
𝐵. Так как в 𝐵 нет наименьшего элемента, 𝐴 не имеет верхней грани в 𝑄.
По той же причине 𝐵 ограничено снизу: множество всех нижних границ 𝐵 состоит из 𝐴 и
всех 𝑟 ∈ 𝑄, таких, что 𝑟 ≤ 0. Так как в 𝐴 нет наибольшего элемента, 𝐵 не имеет нижней
грани в 𝑄.
(b) Если существует 𝛼 = sup 𝐸, то 𝛼 может как быть, так и не быть элементом 𝐸. Например,
пусть 𝐸1 — множество всех 𝑟 ∈ 𝑄, таких, что 𝑟 < 0, а 𝐸2 — множество всех 𝑟 ∈ 𝑄, таких, что
𝑟 ≤ 0. Тогда
sup 𝐸1 = sup 𝐸2 = 0,
причем 0 ∉ 𝐸1 , 0 ∈ 𝐸2 .
(c) Пусть 𝐸 состоит из всех чисел 1/𝑛, где 𝑛 = 1, 2, 3, … . Тогда sup 𝐸 = 1 и принадлежит 𝐸, а
inf 𝐸 = 0 и не принадлежит 𝐸.
1.10 Определение. Говорят, что упорядоченное множество 𝑆 обладает свойством верхней грани,
если выполняется следующее:
Если 𝐸 ⊂ 𝑆, 𝐸 непусто и 𝐸 ограничено сверху, то sup 𝐸 существует в 𝑆.
Пример 1.9(a) показывает, что 𝑄 не обладает свойством верхней грани.
Сейчас мы покажем, что есть тесная свезь между нижними и верхними гранями, и упорядоченное множество со свойством верхней грани также обладает свойством нижней грани.
1.11 Теорема. Пусть 𝑆 — упорядоченное множество со свойством верхней грани, 𝐵 ⊂ 𝑆, 𝐵
непусто и 𝐵 ограничено снизу. Пусть 𝐿 — множество нижних границ множества 𝐵. Тогда
𝛼 = sup 𝐿
существует в 𝑆, и 𝛼 = inf 𝐵.
Это означает, что inf 𝐵 существует в 𝑆.
Доказательство. 𝐿 непусто, так как 𝐵 ограничено снизу. Так как 𝐿 состоит в точности из тех
𝑦 ∈ 𝑆, для которых выполняется неравенство 𝑦 ≤ 𝑥 при всех 𝑥 ∈ 𝐵, мы видим, что все 𝑥 ∈ 𝐵
являются верхними границами множества 𝐿. Это означает, что 𝐿 ограничено сверху. Из свойства
верхней грани множества 𝑆 следует, что 𝐿 имеет верхнюю грань в 𝑆, обозначим ее 𝛼.
Если 𝛾 < 𝛼, то (по определению 1.8) 𝛾 не является верхней границей множества 𝐿, поэтому
𝛾 ∉ 𝐵. Следовательно, 𝛼 ≤ 𝑥 для всех 𝑥 ∈ 𝐵. Это означает, что 𝛼 ∈ 𝐿.
Если 𝛼 < 𝛽, то 𝛽 ∉ 𝐿, так как 𝛼 — верхняя граница множества 𝐿.
Мы показали, что 𝛼 ∈ 𝐿 и 𝛽 ∉ 𝐿 при 𝛽 > 𝛼. Другими словами, 𝛼 — нижняя граница множества
𝐵, а если 𝛽 > 𝛼, то 𝛽 — нет. Это означает, что 𝛼 = inf 𝐵.
7
Поля
1.12 Определение. Поле — множество 𝐹 с двумя операциями, называемыми сложением и умножением, которые удовлетворяют следующим так называемым «аксиомам поля» (A), (M) и (D):
(A) Аксиомы сложения
(A1)
(A2)
(A3)
(A4)
(A5)
Если 𝑥 ∈ 𝐹 и 𝑦 ∈ 𝐹 , то их сумма 𝑥 + 𝑦 принадлежит 𝐹 .
Сложение коммутативно: 𝑥 + 𝑦 = 𝑦 + 𝑥 для всех 𝑥, 𝑦 ∈ 𝐹 .
Сложение ассоциативно: (𝑥 + 𝑦) + 𝑧 = 𝑥 + (𝑦 + 𝑧) для всех 𝑥, 𝑦, 𝑧 ∈ 𝐹 .
𝐹 содержит элемент 0, такой, что 0 + 𝑥 = 𝑥 для каждого 𝑥 ∈ 𝐹 .
Каждому 𝑥 ∈ 𝐹 соответствует элемент −𝑥 ∈ 𝐹 , такой, что
𝑥 + (−𝑥) = 0.
(M) Аксиомы умножения
(M1)
(M2)
(M3)
(M4)
(M5)
Если 𝑥 ∈ 𝐹 и 𝑦 ∈ 𝐹 , то их произведение 𝑥𝑦 принадлежит 𝐹 .
Умножение коммутативно: 𝑥𝑦 = 𝑦𝑥 для всех 𝑥, 𝑦 ∈ 𝐹 .
Умножение ассоциативно: (𝑥𝑦)𝑧 = 𝑥(𝑦𝑧) для всех 𝑥, 𝑦, 𝑧 ∈ 𝐹 .
𝐹 содержит элемент 1 ≠ 0, такой, что 1𝑥 = 𝑥 для каждого 𝑥 ∈ 𝐹 .
Если 𝑥 ∈ 𝐹 и 𝑥 ≠ 0, то существует элемент 1/𝑥 ∈ 𝐹 , такой, что
𝑥 ⋅ (1/𝑥) = 1.
(D) Закон дистрибутивности
𝑥(𝑦 + 𝑧) = 𝑥𝑦 + 𝑥𝑧
выполняется для всех 𝑥, 𝑦, 𝑧 ∈ 𝐹 .
1.13 Замечания.
(a) Обычно (в любом поле) используется запись
𝑥
𝑥 − 𝑦, , 𝑥 + 𝑦 + 𝑧, 𝑥𝑦𝑧, 𝑥2 , 𝑥3 , 2𝑥, 3𝑥, …
𝑦
вместо
1
𝑥 + (−𝑦), 𝑥 ⋅ ( ), (𝑥 + 𝑦) + 𝑧, (𝑥𝑦)𝑧, 𝑥𝑥, 𝑥𝑥𝑥, 𝑥 + 𝑥, 𝑥 + 𝑥 + 𝑥, … .
𝑦
(b) Аксиомы поля очевидным образом выполняются на множестве рациональных чисел 𝑄, если
сложение и умножение имеют их обычный смысл. Следовательно, 𝑄 — поле.
(c) Хотя в нашу задачу не входит детальное изучение полей (и любых других алгебраических
структур), не лишним будет доказать, что некоторые известные свойства множества 𝑄 являются следствиями аксиом поля; после этого не будет необходимости повторять эти доказательства для вещественных и комплексных чисел.
1.14 Предложение. Из аксиом сложения вытекают следующие утверждения.
(a)
(b)
(c)
(d)
Если 𝑥 + 𝑦 = 𝑥 + 𝑧, то 𝑦 = 𝑧.
Если 𝑥 + 𝑦 = 𝑥, то 𝑦 = 0.
Если 𝑥 + 𝑦 = 0, то 𝑦 = −𝑥.
−(−𝑥) = 𝑥.
Утверждение (a) — правило сокращения. Заметим, что (b) говорит о единственности элемента,
существование которого постулируется в (A4), а (c) говорит о том же для (A5).
Доказательство. Из 𝑥 + 𝑦 = 𝑥 + 𝑧 с помощью аксиом (A) следует
𝑦 = 0 + 𝑦 = (−𝑥 + 𝑥) + 𝑦 = −𝑥 + (𝑥 + 𝑦)
= −𝑥 + (𝑥 + 𝑧) = (−𝑥 + 𝑥) + 𝑧 = 0 + 𝑧 = 𝑧.
Тем самым доказано (a). Чтобы получить (b), положим 𝑧 = 0 в (a). Чтобы получить (c), положим
𝑧 = −𝑥 в (a).
Так как −𝑥 + 𝑥 = 0, из (c) подстановкой −𝑥 вместо 𝑥 получается (d).
8
1.15 Предложение. Из аксиом умножения вытекают следующие утверждения.
(a)
(b)
(c)
(d)
Если
Если
Если
Если
𝑥 ≠ 0 и 𝑥𝑦 = 𝑥𝑧, то 𝑦 = 𝑧.
𝑥 ≠ 0 и 𝑥𝑦 = 𝑥, то 𝑦 = 1.
𝑥 ≠ 0 и 𝑥𝑦 = 1, то 𝑦 = 1/𝑥.
𝑥 ≠ 0, то 1/(1/𝑥) = 𝑥.
Доказательство аналогично предложению 1.14, поэтому мы опускаем его.
1.16 Предложение. Из аксиом поля вытекают следующие утверждения для любых 𝑥, 𝑦, 𝑧 ∈ 𝐹 .
(a)
(b)
(c)
(d)
0𝑥 = 0.
Если 𝑥 ≠ 0 и 𝑦 ≠ 0, то 𝑥𝑦 ≠ 0.
(−𝑥)𝑦 = −(𝑥𝑦) = 𝑥(−𝑦).
(−𝑥)(−𝑦) = 𝑥𝑦.
Доказательство. 0𝑥 + 0𝑥 = (0 + 0)𝑥 = 0𝑥. Далее из 1.14(b) следует, что 0𝑥 = 0, и (a) доказано.
Далее, допустим, что 𝑥 ≠ 0, 𝑦 ≠ 0, но 𝑥𝑦 = 0. Тогда (a) дает
1=
1
1
1
1
𝑥𝑦 =
0 = 0,
(𝑦) (𝑥)
(𝑦) (𝑥)
противоречие. Тем самым доказано (b).
Первое равенство в (c) следует из
(−𝑥)𝑦 + 𝑥𝑦 = (−𝑥 + 𝑥)𝑦 = 0𝑦 = 0
и 1.14(c); вторая половина (c) доказывается аналогично. Наконец,
(−𝑥)(−𝑦) = −[𝑥(−𝑦)] = −[−(𝑥𝑦)] = 𝑥𝑦,
что следует из (c) и 1.14(d).
1.17 Определение. Упорядоченным полем называется поле 𝐹 , являющееся также упорядоченным
множеством, причем
(i) 𝑥 + 𝑦 < 𝑥 + 𝑧, если 𝑥, 𝑦, 𝑧 ∈ 𝐹 и 𝑦 < 𝑧,
(ii) 𝑥𝑦 > 0, если 𝑥 ∈ 𝐹 , 𝑦 ∈ 𝐹 , 𝑥 > 0 и 𝑦 > 0.
Если 𝑥 > 0, 𝑥 называют положительным, если 𝑥 < 0, то отрицательным.
Например, 𝑄 является упорядоченным полем.
В любом упорядоченном поле применимы все обычные законы преобразования неравенств:
умножение на положительную (отрицательную) величину сохраняет (меняет) знак неравенства,
квадрат всегда неотрицателен и т. д. Следующее предложение перечисляет некоторые из них.
1.18 Предложение. В любом упорядоченном поле верны следующие утверждения.
(a)
(b)
(c)
(d)
(e)
Если
Если
Если
Если
Если
𝑥 > 0, то −𝑥 < 0 и наоборот.
𝑥 > 0 и 𝑦 < 𝑧, то 𝑥𝑦 < 𝑥𝑧.
𝑥 < 0 и 𝑦 < 𝑧, то 𝑥𝑦 > 𝑥𝑧.
𝑥 ≠ 0, то 𝑥2 > 0. В частности, 1 > 0.
0 < 𝑥 < 𝑦, то 0 < 1/𝑦 < 1/𝑥.
Доказательство.
(a) Если 𝑥 > 0, то 0 = −𝑥 + 𝑥 > −𝑥 + 0, то есть −𝑥 < 0. Если 𝑥 < 0, то 0 = −𝑥 + 𝑥 < −𝑥 + 0, то есть
−𝑥 > 0. Тем самым доказано (a).
(b) Так как 𝑧 > 𝑦, 𝑧 − 𝑦 > 𝑦 − 𝑦 = 0, значит, 𝑥(𝑧 − 𝑦) > 0, из чего следует
𝑥𝑧 = 𝑥(𝑧 − 𝑦) + 𝑥𝑦 > 0 + 𝑥𝑦 = 𝑥𝑦.
9
(c) Из (a), (b) и 1.16(c) следует, что
−[𝑥(𝑧 − 𝑦)] = (−𝑥)(𝑧 − 𝑦) > 0,
так что 𝑥(𝑧 − 𝑦) < 0, значит, 𝑥𝑧 < 𝑥𝑦.
(d) Если 𝑥 > 0, из части (ii) определения 1.17 следует, что 𝑥2 > 0. Если 𝑥 < 0, то −𝑥 > 0, значит,
(−𝑥)2 > 0. Но по предложению 1.16(d) 𝑥2 = (−𝑥)2 . Так как 1 = 12 , 1 > 0.
(e) Если 𝑦 > 0 и 𝑣 ≤ 0, то 𝑦𝑣 ≤ 0. Но 𝑦 ⋅ (1/𝑦) = 1 > 0. Значит, 1/𝑦 > 0. Аналогично 1/𝑥 > 0. Умножая
обе части неравенства 𝑥 < 0 на положительную величину (1/𝑥)(1/𝑦), мы получаем 1/𝑦 < 1/𝑥.
Поле вещественных чисел
Теперь мы сформулируем теорему существования, которая является ядром данной главы.
1.19 Теорема. Существует упорядоченное поле 𝑅, обладающее свойством верхней грани.
Кроме того, 𝑅 содержит 𝑄 как подполе.
Второе утверждение означает, что 𝑄 ⊂ 𝑅, операции сложения и умножения в 𝑅, применяемые
к элементам поля 𝑄, согласуются с обычными операциями над рациональными числами, и что
положительные рациональные числа являются положительными элементами поля 𝑅.
Элементы поля 𝑅 называются вещественными числами.
Доказательство теоремы 1.19 достаточно длинное и утомительное, поэтому оно вынесено в приложение к гл. 1. Оно непосредственно строит 𝑅 из 𝑄.
Следующая теорема почти без усилий может быть получена из этого построения, но мы предпочитаем получить ее как следствие теоремы 1.19, так как это хорошо иллюстрирует применение
свойства верхней грани.
1.20 Теорема.
(a) Если 𝑥 ∈ 𝑅, 𝑦 ∈ 𝑅 и 𝑥 > 0, то существует положительное целое число 𝑛, такое, что 𝑛𝑥 > 𝑦.
(b) Если 𝑥 ∈ 𝑅, 𝑦 ∈ 𝑅 и 𝑥 < 𝑦, то существует 𝑝 ∈ 𝑄, такое, что 𝑥 < 𝑝 < 𝑦.
Свойство (a) обычно называется архимедовостью поля 𝑅. Свойство (b) говорит, что 𝑄 плотно
в 𝑅: между двумя любыми вещественными числами есть рациональное.
Доказательство.
(a) Пусть 𝐴 — множество всех 𝑛𝑥, где 𝑛 пробегает положительные целые числа. Если бы (a) не
выполнялось, 𝑦 было бы верхней границей множества 𝐴. Но тогда 𝐴 имело бы верхнюю грань
в 𝑅. Обозначим 𝛼 = sup 𝐴. Так как 𝑥 > 0, 𝛼 − 𝑥 < 𝛼, и 𝛼 − 𝑥 не является верхней границей
множества 𝐴. Следовательно, 𝛼 − 𝑥 < 𝑚𝑥 для некоторого положительного целого числа 𝑚. Но
тогда 𝛼 < (𝑚 + 1)𝑥 ∈ 𝐴, что невозможно, так как 𝛼 — верхняя граница множества 𝐴.
(b) Так как 𝑥 < 𝑦, 𝑦 − 𝑥 > 0, и (a) дает положительное целое 𝑛, такое, что
𝑛(𝑦 − 𝑥) > 1.
Снова применяя (a), получаем положительные целые 𝑚1 и 𝑚2 , такие, что 𝑚1 > 𝑛𝑥, 𝑚2 > −𝑛𝑥.
Тогда
−𝑚2 < 𝑛𝑥 < 𝑚1 .
Следовательно, существует целое 𝑚 (−𝑚2 ≤ 𝑚 ≤ 𝑚1 ), такое, что
𝑚 − 1 ≤ 𝑛𝑥 < 𝑚.
Объединяя эти неравенства, получаем
𝑛𝑥 < 𝑚 ≤ 1 + 𝑛𝑥 < 𝑛𝑦.
Так как 𝑛 > 0, из этого следует
𝑥<
Положим 𝑝 = 𝑚/𝑛, и (b) доказано.
10
𝑚
< 𝑦.
𝑛
Теперь мы докажем существование корней 𝑛-ой степени из положительных вещественных чисел. Тем самым будет показано, как в 𝑅 можно преодолеть трудности, отмеченные во введении
(иррациональность √2).
1.21 Теорема. Для каждого вещественного 𝑥 > 0 и каждого целого 𝑛 > 0 существует одно и
только одно вещественное 𝑦 > 0, такое, что 𝑦𝑛 = 𝑥.
𝑛
Это число 𝑦 записывается как √
𝑥 или 𝑥1/𝑛 .
Доказательство. Единственность 𝑦 очевидна, так как из 0 < 𝑦1 < 𝑦2 следует 𝑦𝑛1 < 𝑦𝑛2 .
Пусть 𝐸 — множество, состоящее из всех положительных вещественных чисел 𝑡, таких, что
𝑡𝑛 < 𝑥.
Если 𝑡 = 𝑥/(1 + 𝑥), то 0 ≤ 𝑡 < 1. Значит, 𝑡𝑛 ≤ 𝑡 < 𝑥. Таким образом, 𝑡 ∈ 𝐸, и 𝐸 непусто.
Если 𝑡 > 1 + 𝑥, то 𝑡𝑛 ≥ 𝑡 > 𝑥, так что 𝑡 ∉ 𝐸. Таким образом, 1 + 𝑥 является верхней границей
множества 𝐸.
Из теоремы 1.19 следует существование
𝑦 = sup 𝐸.
Чтобы доказать, что 𝑦𝑛 = 𝑥, мы покажем, что любое из неравенств 𝑦𝑛 < 𝑥 и 𝑦𝑛 > 𝑥 приводит к
противоречию.
Из тождества 𝑏𝑛 − 𝑎𝑛 = (𝑏 − 𝑎)(𝑏𝑛−1 + 𝑏𝑛−2 𝑎 + ⋯ + 𝑎𝑛−1 ) следует неравенство
𝑏𝑛 − 𝑎𝑛 < (𝑏 − 𝑎)𝑛𝑏𝑛−1 ,
если 0 < 𝑎 < 𝑏.
Допустим, что 𝑦𝑛 < 𝑥. Выберем ℎ таким, что 0 < ℎ < 1 и
ℎ<
𝑥 − 𝑦𝑛
.
𝑛(𝑦 + 1)𝑛−1
Положим 𝑎 = 𝑦, 𝑏 = 𝑦 + ℎ. Тогда
(𝑦 + ℎ)𝑛 − 𝑦𝑛 < ℎ𝑛(𝑦 + ℎ)𝑛−1 < ℎ𝑛(𝑦 + 1)𝑛−1 < 𝑥 − 𝑦𝑛 .
Таким образом, (𝑦 + ℎ)𝑛 < 𝑥, и 𝑦 + ℎ ∈ 𝐸. Так как 𝑦 + ℎ > 𝑦, это противоречит тому, что 𝑦 — верхняя
граница множества 𝐸.
Допустим, что 𝑦𝑛 > 𝑥. Положим
𝑦𝑛 − 𝑥
𝑘=
.
𝑛𝑦𝑛−1
Тогда 0 < 𝑘 < 𝑦. Если 𝑡 ≥ 𝑦 − 𝑘, мы получим, что
𝑦𝑛 − 𝑡𝑛 ≤ 𝑦𝑛 − (𝑦 − 𝑘)𝑛 ≤ 𝑘𝑛𝑦𝑛−1 = 𝑦𝑛 − 𝑥.
Таким образом, 𝑡𝑛 > 𝑥, и 𝑡 ∉ 𝐸. Следовательно, 𝑦 − 𝑘 — верхняя граница множества 𝐸. Но 𝑦 − 𝑘 < 𝑦,
что противоречит тому, что 𝑦 — верхняя грань множества 𝐸.
Следовательно, 𝑦𝑛 = 𝑥, и теорема доказана.
Следствие. Если 𝑎 и 𝑏 — положительные вещественные числа, а 𝑛 — положительное целое число, то
(𝑎𝑏)1/𝑛 = 𝑎1/𝑛 𝑏1/𝑛 .
Доказательство. Положим 𝛼 = 𝑎1/𝑛 , 𝛽 = 𝑏1/𝑛 . Тогда
𝑎𝑏 = 𝛼 𝑛 𝛽 𝑛 = (𝛼𝛽)𝑛 ,
так как умножение коммутативно (аксиома (M2) в определении 1.12). Тогда условие единственности
теоремы 1.21 показывает, что
(𝑎𝑏)1/𝑛 = 𝛼𝛽 = 𝑎1/𝑛 𝑏1/𝑛 .
11
1.22 Десятичные дроби. В заключение этого раздела укажем на соотношение между вещественными числами и десятичными дробями.
Пусть 𝑥 > 0 — вещественное число. Пусть 𝑛0 — наибольшее целое число, такое, что 𝑛0 ≤ 𝑥 (заметим, что существование 𝑛0 опирается на архимедовость поля 𝑅). Если 𝑛0 , 𝑛1 , … , 𝑛𝑘−1 уже выбраны,
то пусть 𝑛𝑘 — наибольшее целое число, такое, что
𝑛
𝑛
𝑛0 + 1 + ⋯ + 𝑘𝑘 ≤ 𝑥.
10
10
Пусть 𝐸 — множество чисел
𝑛
𝑛
(5)
𝑛0 + 1 + ⋯ + 𝑘𝑘
(𝑘 = 0, 1, 2, … ).
10
10
Тогда 𝑥 = sup 𝐸. Десятичное разложение 𝑥 имеет вид
(6)
𝑛0 .𝑛1 𝑛2 𝑛3 … .
Обратно, для любой бесконечной десятичной дроби (6) множество чисел (5) ограничено сверху,
и (6) является десятичным разложением числа sup 𝐸 1 .
Мы не обсуждаем подробно десятичные дроби, так как никогда не будем ими пользоваться.
Расширенная система вещественных чисел
1.23 Определение. Расширенная система вещественных чисел состоит из поля вещественных
чисел 𝑅 и двух символов +∞ и −∞. Мы сохраняем порядок на 𝑅 и определяем
−∞ < 𝑥 < +∞
для всех 𝑥 ∈ 𝑅.
Ясно, что +∞ является верхней границей любого подмножества расширенной системы вещественных чисел, и любое непустое подмножество имеет верхнюю грань. Например, если 𝐸 — непустое множество вещественных чисел, не ограниченное сверху в 𝑅, то в расширенной системе вещественных чисел sup 𝐸 = +∞.
Те же замечания справедливы для нижних границ.
Расширенная система вещественных чисел не образует поле, но удобно принять следующие
соглашения:
(a) Если 𝑥 — вещественное число, то
𝑥 + ∞ = +∞,
𝑥 − ∞ = −∞,
𝑥
𝑥
=
= 0.
+∞ −∞
(b) Если 𝑥 > 0, то 𝑥 ⋅ (+∞) = +∞, 𝑥 ⋅ (−∞) = −∞.
(c) Если 𝑥 < 0, то 𝑥 ⋅ (+∞) = −∞, 𝑥 ⋅ (−∞) = +∞.
Если необходимо подчеркнуть различие между вещественными числами, с одной стороны, и
символами +∞ и −∞, с другой, то первые называют конечными.
Поле комплексных чисел
1.24 Определение. Комплексным числом называется упорядоченная пара вещественных чисел
(𝑎, 𝑏). «Упорядоченная» означает, что (𝑎, 𝑏) и (𝑏, 𝑎) считаются различными, если 𝑎 ≠ 𝑏.
Пусть 𝑥 = (𝑎, 𝑏) и 𝑦 = (𝑐, 𝑑) — два комплексных числа. Мы говорим, что 𝑥 = 𝑦, тогда и только
тогда, когда 𝑎 = 𝑐 и 𝑏 = 𝑑 (заметим, что это определение не полностью избыточно; сравните
с определением равенства рациональных чисел, представленных в виде отношения целых). Мы
определяем
𝑥 + 𝑦 = (𝑎 + 𝑐, 𝑏 + 𝑑),
𝑥𝑦 = (𝑎𝑐 − 𝑏𝑑, 𝑎𝑑 + 𝑏𝑐).
1
Это не совсем верно: для бесконечной десятичной дроби 0,999 … десятичным разложением числа sup 𝐸 является
1,000 … . — Прим. перев.
12
1.25 Теорема. Данные определения сложения и умножения превращают множество комплексных чисел в поле, причем (0, 0) и (1, 0) играют роль 0 и 1.
Доказательство. Мы просто проверим аксиомы поля, перечисленные в определении 1.12 (конечно, мы будем при этом пользоваться свойствами поля 𝑅).
Пусть 𝑥 = (𝑎, 𝑏), 𝑦 = (𝑐, 𝑑), 𝑧 = (𝑒, 𝑓 ).
(A1) Очевидно.
(A2) 𝑥 + 𝑦 = (𝑎 + 𝑐, 𝑏 + 𝑑) = (𝑐 + 𝑎, 𝑑 + 𝑏) = 𝑦 + 𝑥.
(A3) (𝑥 + 𝑦) + 𝑧 = (𝑎 + 𝑐, 𝑏 + 𝑑) + (𝑒, 𝑓 )
= (𝑎 + 𝑐 + 𝑒, 𝑏 + 𝑑 + 𝑓 )
(A4)
(A5)
(M1)
(M2)
(M3)
= (𝑎 + 𝑏) + (𝑐 + 𝑒, 𝑑 + 𝑓 ) = 𝑥 + (𝑦 + 𝑧).
𝑥 + 0 = (𝑎, 𝑏) + (0, 0) = (𝑎, 𝑏) = 𝑥.
Положим −𝑥 = (−𝑎, −𝑏). Тогда 𝑥 + (−𝑥) = (0, 0) = 0.
Очевидно.
𝑥𝑦 = (𝑎𝑐 − 𝑏𝑑, 𝑎𝑏 + 𝑏𝑐) = (𝑐𝑎 − 𝑑𝑏, 𝑑𝑎 + 𝑐𝑏) = 𝑦𝑥.
(𝑥𝑦)𝑧 = (𝑎𝑐 − 𝑏𝑑, 𝑎𝑑 + 𝑏𝑐)(𝑒, 𝑓 )
= (𝑎𝑐𝑒 − 𝑏𝑑𝑒 − 𝑎𝑑𝑓 − 𝑏𝑐𝑓 , 𝑎𝑐𝑓 − 𝑏𝑑𝑓 + 𝑎𝑑𝑒 + 𝑏𝑐𝑒)
= (𝑎, 𝑏)(𝑐𝑒 − 𝑑𝑓 , 𝑐𝑓 + 𝑑𝑒) = 𝑥(𝑦𝑧).
(M4) 1𝑥 = (1, 0)(𝑎, 𝑏) = (𝑎, 𝑏) = 𝑥.
(M5) Если 𝑥 ≠ 0, то (𝑎, 𝑏) ≠ (0, 0), что означает, что хотя бы одно из вещественных чисел 𝑎 и 𝑏
отлично от 0. Следовательно, по предложению 1.18(d) 𝑎2 +𝑏2 > 0, и мы можем определить
1
𝑎
−𝑏
=
,
.
𝑥 ( 𝑎2 + 𝑏2 𝑎2 + 𝑏2 )
Тогда
𝑥⋅
𝑎
−𝑏
1
= (𝑎, 𝑏)
,
= (1, 0) = 1.
( 𝑎2 + 𝑏2 𝑎2 + 𝑏2 )
𝑥
(D) 𝑥(𝑦 + 𝑧) = (𝑎, 𝑏)(𝑐 + 𝑒, 𝑑 + 𝑓 )
= (𝑎𝑐 + 𝑎𝑒 − 𝑏𝑑 − 𝑏𝑓 , 𝑎𝑑 + 𝑎𝑓 + 𝑏𝑐 + 𝑏𝑒)
= (𝑎𝑐 − 𝑏𝑑, 𝑎𝑑 + 𝑏𝑐) + (𝑎𝑒 − 𝑏𝑓 , 𝑎𝑓 + 𝑏𝑒)
= 𝑥𝑦 + 𝑥𝑧.
1.26 Теорема. Для любых вещественных чисел 𝑎 и 𝑏 имеем
(𝑎, 0) + (𝑏, 0) = (𝑎 + 𝑏, 0),
(𝑎, 0)(𝑏, 0) = (𝑎𝑏, 0).
Доказательство тривиально.
Теорема 1.26 показывает, что комплексные числа вида (𝑎, 0) обладают теми же арифметическими свойствами, что и соответствующие вещественные числа 𝑎. Мы можем поэтому отождествить
(𝑎, 0) с 𝑎. Это отождествление дает нам поле вещественных чисел как подполе поля комплексных
чисел.
Читатель, вероятно, заметил, что мы построили комплексные числа, не вводя таинственного
квадратного корня из −1. Теперь мы покажем, что обозначение (𝑎, 𝑏) равносильно более привычному
𝑎 + 𝑏𝑖.
1.27 Определение. 𝑖 = (0, 1).
1.28 Теорема. 𝑖2 = −1.
Доказательство. 𝑖2 = (0, 1)(0, 1) = (−1, 0) = −1.
1.29 Теорема. Если 𝑎 и 𝑏 — вещественные числа, то (𝑎, 𝑏) = 𝑎 + 𝑏𝑖.
13
Доказательство.
𝑎 + 𝑏𝑖 = (𝑎, 0) + (𝑏, 0)(0, 1)
= (𝑎, 0) + (0, 𝑏) = (𝑎, 𝑏).
1.30 Определение. Если 𝑎 и 𝑏 — вещественные числа и 𝑧 = 𝑎 + 𝑏𝑖, то комплексное число 𝑧 ̄ = 𝑎 − 𝑏𝑖
называется сопряженным с 𝑧. Числа 𝑎 и 𝑏 называются соответственно вещественной и мнимой
частями числа 𝑧.
Мы иногда будем писать
𝑎 = Re(𝑧),
𝑏 = Im(𝑧).
1.31 Теорема. Если 𝑧 и 𝑤 — комплексные числа, то
(a)
(b)
(c)
(d)
𝑧 + 𝑤 = 𝑧̄ + 𝑤,̄
𝑧𝑤 = 𝑧 ̄ ⋅ 𝑤,̄
𝑧 + 𝑧 ̄ = 2 Re(𝑧), 𝑧 − 𝑧̄ = 2𝑖 Im(𝑧),
𝑧𝑧 ̄ вещественно и положительно (за исключением случая 𝑧 = 0).
Доказательство. (a), (b) и (c) почти тривиальны. Чтобы доказать (d), запишем 𝑧 = 𝑎 + 𝑏𝑖 и
заметим, что 𝑧𝑧̄ = 𝑎2 + 𝑏2 .
1.32 Определение. Если 𝑧 — комплексное число, его абсолютной величиной называется неотрицательный квадратный корень из 𝑧𝑧,̄ то есть |𝑧| = (𝑧𝑧)̄ 1/2 .
Существование (и единственность) |𝑧| следует из теоремы 1.21 и части (d) теоремы 1.31.
Заметим, что, если 𝑥 вещественно, то 𝑥̄ = 𝑥, значит, |𝑥| = √𝑥2 . Следовательно, |𝑥| = 𝑥 при 𝑥 ≥ 0
и |𝑥| = −𝑥 при 𝑥 < 0.
1.33 Теорема. Пусть 𝑧 и 𝑤 — комплексные числа. Тогда
(a)
(b)
(c)
(d)
(e)
|𝑧| > 0, за исключением 𝑧 = 0, |0| = 0,
|𝑧|̄ = |𝑧|,
|𝑧𝑤| = |𝑧||𝑤|,
| Re 𝑧| ≤ |𝑧|,
|𝑧 + 𝑤| ≤ |𝑧| + |𝑤|.
Доказательство. (a) и (b) тривиальны. Положим 𝑧 = 𝑎 + 𝑏𝑖, 𝑤 = 𝑐 + 𝑑𝑖, где 𝑎, 𝑏, 𝑐, 𝑑 вещественны.
Тогда
|𝑧𝑤|2 = (𝑎𝑐 − 𝑏𝑑)2 + (𝑎𝑑 + 𝑏𝑐)2 = (𝑎2 + 𝑏2 )(𝑐 2 + 𝑑 2 ) = |𝑧|2 |𝑤|2
или |𝑧𝑤|2 = (|𝑧||𝑤|)2 . Теперь (c) следует из условия единственности в теореме 1.21.
Чтобы доказать (d), заметим, что 𝑎2 ≤ 𝑎2 + 𝑏2 , следовательно,
|𝑎| = √𝑎2 ≤ √𝑎2 + 𝑏2 .
̄ СледоваЧтобы доказать (e), заметим, что 𝑧𝑤
̄ сопряжено с 𝑧𝑤,̄ поэтому 𝑧𝑤̄ + 𝑧𝑤
̄ = 2 Re(𝑧𝑤).
тельно,
|𝑧 + 𝑤|2 = (𝑧 + 𝑤)(𝑧 ̄ + 𝑤)̄ = 𝑧𝑧 ̄ + 𝑧𝑤̄ + 𝑧𝑤
̄ + 𝑤𝑤̄
= |𝑧|2 + 2 Re(𝑧𝑤)̄ + |𝑤|2
≤ |𝑧|2 + 2|𝑧𝑤|̄ + |𝑤|2
= |𝑧|2 + 2|𝑧||𝑤| + |𝑤|2 = (|𝑧| + |𝑤|)2 .
После извлечения квадратного корня из обеих частей получаем (e).
1.34 Обозначение. Если 𝑥1 , … , 𝑥𝑛 — комплексные числа, то мы будем писать
𝑛
𝑥1 + 𝑥 2 + ⋯ + 𝑥 𝑛 =
∑
𝑗=1
14
𝑥𝑗 .
Мы закончим этот раздел важным неравенством, известным под названием неравенства Шварца.
1.35 Теорема. Если 𝑎1 , … , 𝑎𝑛 и 𝑏1 , … , 𝑏𝑛 — комплексные числа, то
𝑛
𝑛
| 𝑛
|2
2
| 𝑎𝑗 𝑏̄𝑗 | ≤
|𝑎 |
|𝑏 |2 .
∑ 𝑗 ∑ 𝑗
||∑
||
𝑗−1
𝑗=1
𝑗=1
Доказательство. Положим 𝐴 = ∑ |𝑎𝑗 |2 , 𝐵 = ∑ |𝑏𝑗 |2 , 𝐶 = ∑ 𝑎𝑗 𝑏̄𝑗 (во всех суммах в доказательстве
𝑗 пробегает значения 1, … , 𝑛). Если 𝐵 = 0, то 𝑏1 = ⋯ = 𝑏𝑛 = 0, и заключение тривиально. Допустим
поэтому, что 𝐵 > 0. По теореме 1.31 имеем
∑
|𝐵𝑎𝑗 − 𝐶𝑏𝑗 |2 =
∑
= 𝐵2
(𝐵𝑎𝑗 − 𝐶𝑏𝑗 )(𝐵 𝑎𝑗̄ − 𝐶𝑏𝑗 )
|𝑎 |2 − 𝐵 𝐶 ̄
𝑎 𝑏̄ − 𝐵𝐶
𝑎 ̄ 𝑏 + |𝐶|2
|𝑏 |2
∑ 𝑗
∑ 𝑗 𝑗
∑ 𝑗 𝑗
∑ 𝑗
= 𝐵 2 𝐴 − 𝐵|𝐶|2
= 𝐵(𝐴𝐵 − |𝐶|2 ).
Так как все слагаемые в первой сумме неотрицательны, мы видим, что
𝐵(𝐴𝐵 − |𝐶|2 ) ≥ 0.
Поскольку 𝐵 > 0, отсюда следует, что 𝐴𝐵 − |𝐶|2 ≥ 0. Это и есть требуемое неравенство.
Евклидовы пространства
1.36 Определения. Для каждого положительного целого 𝑘 обозначим через 𝑅𝑘 множество всех
упорядоченных последовательностей длины 𝑘
x = (𝑥1 , 𝑥2 , … , 𝑥𝑘 ),
где 𝑥1 , … , 𝑥𝑘 — вещественные числа, называемые координатами элемента x. Элементы множества
𝑅𝑘 называются точками или векторами, особенно при 𝑘 > 1. Мы будем обозначать векторы буквами,
набранными жирным шрифтом. Если y = (𝑦1 , … , 𝑦𝑘 ) и 𝛼 — вещественное число, положим
x + y = (𝑥1 + 𝑦1 , … , 𝑥𝑘 + 𝑦𝑘 )
𝛼x = (𝛼𝑥1 , … , 𝛼𝑥𝑘 ),
так что x + y ∈ 𝑅𝑘 и 𝛼x ∈ 𝑅𝑘 . Тем самым определено сложение векторов, а также умножение вектора на вещественное число (скаляр). Эти две операции подчиняются законам коммутативности,
ассоциативности и дистрибутивности (доказательство тривиально, так как аналогичным законам
подчиняются вещественные числа) и превращают 𝑅𝑘 в векторное пространство над полем вещественных чисел. Нулевой элемент множества 𝑅𝑘 (иногда называемый началом или нулевым
вектором) — точка 0, все координаты которой равны 0.
Мы определим еще так называемое скалярное (или внутреннее) произведение векторов x и y
как
𝑘
x⋅y=
∑
𝑥𝑖 𝑦𝑖
𝑖=1
и норму вектора x как
|x| = (x ⋅ x)1/2
⎛ 𝑘
⎞
= ⎜ 𝑥2𝑖 ⎟
⎜∑
⎟
⎝ 1
⎠
1/2
.
Определенная таким образом структура (векторное пространство 𝑅𝑘 со скалярным произведением и нормой) называется евклидовым 𝑘-мерным пространством.
15
1.37 Теорема. Пусть x, y, z ∈ 𝑅𝑘 , а 𝛼 — вещественное число. Тогда
(a)
(b)
(c)
(d)
(e)
(f )
|x| ≥ 0;
|x| = 0 тогда и только тогда, когда x = 0;
|𝛼x| = |𝛼||x|;
|x ⋅ y| ≤ |x||y|;
|x + y| ≤ |x| + |y|;
|x − z| ≤ |x − y| + |y − z|.
Доказательство. (a), (b) и (c) очевидны, а (d) следует непосредственно из неравенства Шварца.
В силу (d) имеем
|x + y|2 = (x + y) ⋅ (x + y)
= x ⋅ x + 2x ⋅ y + y ⋅ y
≤ |x|2 + 2|x||y| + |y|2
= (|x| + |y|)2 ,
что доказывает (e). Наконец, (f) следует из (e), если заменить x на x − y и y на y − z.
1.38 Замечания. Теорема 1.37(a), (b) и (f) позволит нам (см. гл. 2) рассматривать 𝑅𝑘 как метрическое пространство.
𝑅1 (множество вещественных чисел) обычно называют прямой или вещественной прямой. Аналогичным образом, 𝑅2 называют плоскостью или комплексной плоскостью (ср. определения 1.24 и
1.36). В этих двух случаях норма — это просто абсолютная величина соответствующего вещественного или комплексного числа.
Приложение
В этом приложении будет доказана теорема 1.19 путем построения поля 𝑅 из 𝑄. Мы разделим
построение на несколько шагов.
Шаг 1. Элементами множества 𝑅 будут определенные подмножества множества 𝑄, называемые
сечениями. По определению, сечение — любое множество 𝛼 ⊂ 𝑄 со следующими тремя свойствами.
(I) 𝛼 непусто и 𝛼 ≠ 𝑄.
(II) Если 𝑝 ∈ 𝛼, 𝑞 ∈ 𝑄 и 𝑞 < 𝑝, то 𝑞 ∈ 𝛼.
(III) Если 𝑝 ∈ 𝛼, то 𝑝 < 𝑟 для некоторого 𝑟 ∈ 𝛼.
Буквы 𝑝, 𝑞, 𝑟, … будут всегда обозначать рациональные числа, а 𝛼, 𝛽, 𝛾, … — сечения.
Заметим, что (III) просто говорит о том, что 𝛼 не имеет наибольшего элемента; из (II) следуют
два факта, которые мы будем свободно использовать:
Если 𝑝 ∈ 𝛼 и 𝑞 ∉ 𝛼, то 𝑝 < 𝑞.
Если 𝑟 ∉ 𝛼 и 𝑟 < 𝑠, то 𝑠 ∉ 𝛼.
Шаг 2. Определим 𝛼 < 𝛽 как «𝛼 — собственное подмножество множества 𝛽».
Проверим, что это удовлетворяет требованиям определения 1.5.
Если 𝛼 < 𝛽 и 𝛽 < 𝛾, то ясно, что 𝛼 < 𝛾 (собственное подмножество собственного подмножества
является собственным подмножеством). Кроме того, ясно, что для любой пары 𝛼, 𝛽 выполняется
самое большее одно из трех соотношений
𝛼 < 𝛽,
𝛼 = 𝛽,
𝛽 < 𝛼.
Чтобы показать, что выполняется хотя бы одно, допустим, что первые два не выполняются. Тогда
𝛼 не является подмножеством множества 𝛽. Следовательно, существует 𝑝 ∈ 𝛼, для которого 𝑝 ∉ 𝛽.
Если 𝑞 ∈ 𝛽, то 𝑞 < 𝑝 (так как 𝑝 ∉ 𝛽), следовательно, 𝑞 ∈ 𝛼 вследствие (II). Значит, 𝛽 ⊂ 𝛼. Так как
𝛽 ≠ 𝛼, мы приходим к выводу, что 𝛽 < 𝛼.
Таким образом, 𝑅 становится упорядоченным множеством.
16
Шаг 3. Упорядоченное множество 𝑅 обладает свойством верхней грани.
Для доказательства возьмем 𝐴 — непустое подмножество множества 𝑅, и пусть 𝛽 ∈ 𝑅 — верхняя
граница множества 𝐴. Обозначим через 𝛾 объединение всех 𝛼 ∈ 𝐴. Другими словами, 𝑝 ∈ 𝛾 тогда и
только тогда, когда 𝑝 ∈ 𝛼 для некоторого 𝛼 ∈ 𝐴. Мы докажем, что 𝛾 ∈ 𝑅 и что 𝛾 = sup 𝐴.
Так как 𝐴 непусто, существует 𝛼0 ∈ 𝐴. Это 𝛼0 непусто. Так как 𝛼0 ⊂ 𝛾, 𝛾 непусто. Далее, 𝛾 ⊂ 𝛽
(так как 𝛼 ⊂ 𝛽 для всех 𝛼 ∈ 𝐴), и поэтому 𝛾 ≠ 𝑄. Следовательно, 𝛾 удовлетворяет свойству (I).
Чтобы доказать (II) и (III), выберем 𝑝 ∈ 𝛾. Тогда 𝑝 ∈ 𝛼1 для некоторого 𝛼1 ∈ 𝐴. Если 𝑞 < 𝑝, то
𝑞 ∈ 𝛼1 , значит, 𝑞 ∈ 𝛾; утверждение (II) доказано. Если 𝑟 ∈ 𝛼1 выбрано так, что 𝑟 > 𝑝, мы видим, что
𝑟 ∈ 𝛾 (так как 𝛼1 ⊂ 𝛾), и, следовательно, 𝛾 удовлетворяет (III).
Итак, 𝛾 ∈ 𝑅.
Ясно, что 𝛼 ≤ 𝛾 для любого 𝛼 ∈ 𝐴.
Пусть 𝛿 < 𝛾. Тогда существует 𝑠 ∈ 𝛾, такое, что 𝑠 ∉ 𝛿. Так как 𝑠 ∈ 𝛾, 𝑠 ∈ 𝛼 для некоторого 𝛼 ∈ 𝐴.
Значит, 𝛿 < 𝛼, и 𝛿 не является верхней границей множества 𝐴.
Это дает желаемый результат: 𝛾 = sup 𝐴.
Шаг 4. Если 𝛼 ∈ 𝑅 и 𝛽 ∈ 𝑅, мы определяем 𝛼 + 𝛽 как множество всех сумм вида 𝑟 + 𝑠, где 𝑟 ∈ 𝛼 и
𝑠 ∈ 𝛽.
Обозначим 0∗ множество всех отрицательных рациональных чисел. Ясно, что 0∗ является сечением. Мы проверим, что в 𝑅 выполняются аксиомы сложения (см. определение 1.12) с 0∗ в роли
0.
(A1) Мы должны показать, что 𝛼 + 𝛽 — сечение. Ясно, что 𝛼 + 𝛽 — непустое подмножество
множества 𝑄. Возьмем 𝑟′ ∉ 𝛼, 𝑠′ ∉ 𝛽. Тогда 𝑟′ + 𝑠′ > 𝑟 + 𝑠 при любом выборе 𝑟 ∈ 𝛼, 𝑠 ∈ 𝛽.
Значит, 𝑟′ + 𝑠′ ∉ 𝛼 + 𝛽. Следовательно, 𝛼 + 𝛽 удовлетворяет свойству (I).
Выберем 𝑝 ∈ 𝛼 + 𝛽. Тогда 𝑝 = 𝑟 + 𝑠, где 𝑟 ∈ 𝛼, 𝑠 ∈ 𝛽. Если 𝑞 < 𝑝, то 𝑞 − 𝑠 < 𝑟, так что
𝑞 − 𝑠 ∈ 𝛼, и 𝑞 = (𝑞 − 𝑠) + 𝑠 ∈ 𝛼 + 𝛽. Так выполняется (II). Выберем 𝑡 ∈ 𝛼, так, что 𝑡 > 𝑟.
Тогда 𝑝 < 𝑡 + 𝑠 и 𝑡 + 𝑠 ∈ 𝛼 + 𝛽. Так выполняется (III).
(A2) 𝛼 + 𝛽 — множество всех 𝑟 + 𝑠, где 𝑟 ∈ 𝛼, 𝑠 ∈ 𝛽. По тому же определению, 𝛽 + 𝛼 — множество
всех 𝑠 + 𝑟. Так как 𝑟 + 𝑠 = 𝑠 + 𝑟 для всех 𝑟 ∈ 𝑄, 𝑠 ∈ 𝑄, мы имеем 𝛼 + 𝛽 = 𝛽 + 𝛼.
(A3) Как и выше, это следует из закона ассоциативности в поле 𝑄.
(A4) Если 𝑟 ∈ 𝛼 и 𝑠 ∈ 0∗ , то 𝑟+𝑠 < 𝑟, значит, 𝑟+𝑠 ∈ 𝛼. Следовательно, 𝛼+0∗ ⊂ 𝛼. Чтобы получить
обратное включение, выберем 𝑝 ∈ 𝛼 и 𝑟 ∈ 𝛼, 𝑟 > 𝑝. Тогда 𝑝 − 𝑟 ∈ 0∗ , и 𝑝 = 𝑟 + (𝑝 − 𝑟) ∈ 𝛼 + 0∗ .
Значит, 𝛼 ⊂ 𝛼 + 0∗ . Мы приходим к выводу, что 𝛼 + 0∗ = 𝛼.
(A5) Зафиксируем 𝛼 ∈ 𝑅. Пусть 𝛽 — множество всех 𝑝 со следующим свойством:
Существует 𝑟 > 0, такое, что −𝑝 − 𝑟 ∉ 𝛼.
Другими словами, некоторое рациональное число, меньшее, чем −𝑝, не попадает в 𝛼.
Мы покажем, что 𝛽 ∈ 𝑅 и что 𝛼 + 𝛽 = 0∗ .
Если 𝑠 ∉ 𝛼 и 𝑝 = −𝑠 − 1, то −𝑝 − 1 ∉ 𝛼 и 𝑝 ∈ 𝛽. Значит, 𝛽 непусто. Если 𝑞 ∈ 𝛼, то −𝑞 ∉ 𝛽.
Значит, 𝛽 ≠ 𝑄. Следовательно, 𝛽 удовлетворяет (I).
Выберем 𝑝 ∈ 𝛽 и 𝑟 > 0, так, что −𝑝 − 𝑟 ∉ 𝛼. Если 𝑞 < 𝑝, то −𝑞 − 𝑟 > −𝑝 − 𝑟, значит,
−𝑞 − 𝑟 ∉ 𝛼. Следовательно, 𝑞 ∈ 𝛽, и выполняется (II). Положим 𝑡 = 𝑝 + (𝑟/2). Тогда 𝑡 > 𝑝,
и −𝑡 − (𝑟/2) = −𝑝 − 𝑟 ∉ 𝛼, так что 𝑡 ∈ 𝛽. Следовательно, 𝛽 удовлетворяет (III).
Мы доказали, что 𝛽 ∈ 𝑅.
Если 𝑟 ∈ 𝛼 и 𝑠 ∈ 𝛽, то −𝑠 ∉ 𝛼, значит, 𝑟 < −𝑠, 𝑟 + 𝑠 < 0. Следовательно, 𝛼 + 𝛽 ⊂ 0∗ .
Чтобы доказать обратное включение, выберем 𝑣 ∈ 0∗ , положим 𝑤 = −𝑣/2. Тогда 𝑤 > 0,
и существует целое 𝑛, такое, что 𝑛𝑤 ∈ 𝛼, но (𝑛 + 1)𝑤 ∉ 𝛼 (заметим, что это опирается на
архимедовость поля 𝑄!). Положим 𝑝 = −(𝑛 + 2)𝑤. Тогда 𝑝 ∈ 𝛽, так как −𝑝 − 𝑤 ∉ 𝛼, и
𝑣 = 𝑛𝑤 + 𝑝 ∈ 𝛼 + 𝛽.
Следовательно, 0∗ ⊂ 𝛼 + 𝛽.
Мы приходим к выводу, что 𝛼 + 𝛽 = 0∗ .
Такое 𝛽, естественно, будет обозначаться −𝛼.
Шаг 5. Из того, что определенное на шаге 4 сложение удовлетворяет аксиомам (A) определения
1.12, следует, что в поле 𝑅 справедливо предложение 1.14, и мы можем доказать одно из требований
определения 1.17:
17
Если 𝛼, 𝛽, 𝛾 ∈ 𝑅 и 𝛽 < 𝛾, то 𝛼 + 𝛽 < 𝛼 + 𝛾.
Действительно, из определения сложения в поле 𝑅 очевидно, что 𝛼 + 𝛽 ⊂ 𝛼 + 𝛾; если бы выполнялось 𝛼 + 𝛽 = 𝛼 + 𝛾, по правилу сокращения (предложение 1.14) получилось бы 𝛽 = 𝛾.
Из вышесказанного также следует, что 𝛼 > 0∗ тогда и только тогда, когда −𝛼 < 0∗ .
Шаг 6. Умножение немного сложнее сложения, потому что произведение отрицательных рациональных чисел положительно. По этой причине сначала мы ограничимся 𝑅+ , множеством всех
𝛼 ∈ 𝑅, для которых 𝛼 > 0∗ .
Если 𝛼 ∈ 𝑅+ и 𝛽 ∈ 𝑅+ , мы определяем 𝛼𝛽 как множество всех 𝑝, таких, что 𝑝 ≤ 𝑟𝑠 для некоторых
𝑟 ∈ 𝛼, 𝑠 ∈ 𝛽, 𝑟 > 0, 𝑠 > 0.
Обозначим 1∗ множество всех 𝑞 < 1.
Тогда выполняются аксиомы (M) и (D) определения 1.12 для 𝑅+ в качестве 𝐹 и с 1∗ в роли 1.
Доказательства настолько похожи на детально выписанные доказательства в шаге 4, что мы
опускаем их.
Заметим, в частности, что выполняется второе требование определения 1.17: если 𝛼 > 0∗ и
𝛽 > 0∗ , то 𝛼𝛽 > 0∗ .
Шаг 7. Мы завершаем определение умножения, положив 𝛼0∗ = 0∗ 𝛼 = 0∗ и
⎧(−𝛼)(−𝛽),
⎪
𝛼𝛽 = ⎨−[(−𝛼)𝛽],
⎪−[𝛼 ⋅ (−𝛽)],
⎩
если 𝛼 < 0∗ , 𝛽 < 0∗ ,
если 𝛼 < 0∗ , 𝛽 > 0∗ ,
если 𝛼 > 0∗ , 𝛽 < 0∗ .
Произведения в правой части были определены на шаге 6.
На основе доказательства (на шаге 6), что в 𝑅+ выполняются аксиомы (M), очень просто доказать их справедливость в 𝑅 с помощью многократного применения тождества 𝛾 = −(−𝛾), являющегося частью предложения 1.14 (см. шаг 5).
Доказательство закона дистрибутивности
𝛼(𝛽 + 𝛾) = 𝛼𝛽 + 𝛼𝛾
разбивается на случаи. Например, допустим, что 𝛼 > 0∗ , 𝛽 < 0∗ , 𝛽 + 𝛾 > 0∗ . Тогда 𝛾 = (𝛽 + 𝛾) + (−𝛽),
и (так как мы уже знаем, что в 𝑅+ выполняется закон дистрибутивности)
𝛼𝛾 = 𝛼(𝛽 + 𝛾) + 𝛼 ⋅ (−𝛽).
Но 𝛼 ⋅ (−𝛽) = −(𝛼𝛽). Тем самым
𝛼𝛽 + 𝛼𝛾 = 𝛼(𝛽 + 𝛾).
Остальные случаи доказываются аналогичным образом.
Мы завершили доказательство того, что 𝑅 является упорядоченным полем с свойством верхней грани.
Шаг 8. Сопоставим каждому 𝑟 ∈ 𝑄 множество 𝑟∗ , состоящее из всех 𝑝 ∈ 𝑄, таких, что 𝑝 < 𝑟. Ясно,
что каждое такое 𝑟∗ является сечением, то есть 𝑟∗ ∈ 𝑅. Эти сечения удовлетворяют следующим
соотношениям:
(a) 𝑟∗ + 𝑠∗ = (𝑟 + 𝑠)∗ ,
(b) 𝑟∗ 𝑠∗ = (𝑟𝑠)∗ ,
(c) 𝑟∗ < 𝑠∗ тогда и только тогда, когда 𝑟 < 𝑠.
Чтобы доказать (a), выберем 𝑝 ∈ 𝑟∗ + 𝑠∗ . Тогда 𝑝 = 𝑢 + 𝑣, где 𝑢 < 𝑟, 𝑣 < 𝑠. Значит, 𝑝 < 𝑟 + 𝑠, что
означает 𝑝 ∈ (𝑟 + 𝑠)∗ .
Обратно, пусть 𝑝 ∈ (𝑟 + 𝑠)∗ . Тогда 𝑝 < 𝑟 + 𝑠. Выберем 𝑡 таким, что 2𝑡 = 𝑟 + 𝑠 − 𝑝, положим
𝑟′ = 𝑟 − 𝑡, 𝑠′ = 𝑠 − 𝑡.
Тогда 𝑟′ ∈ 𝑟∗ , 𝑠′ ∈ 𝑠∗ и 𝑝 = 𝑟′ + 𝑠′ , поэтому 𝑝 ∈ 𝑟∗ + 𝑠∗ .
Утверждение (a) доказано. Доказательство (b) аналогично.
Если 𝑟 < 𝑠, то 𝑟 ∈ 𝑠∗ , но 𝑟 ∉ 𝑟∗ ; следовательно, 𝑟∗ < 𝑠∗ .
Если 𝑟∗ < 𝑠∗ , то существует 𝑝 ∈ 𝑠∗ , такое, что 𝑝 ∉ 𝑟∗ . Следовательно, 𝑟 ≤ 𝑝 < 𝑠, что означает
𝑟 < 𝑠.
Утверждение (c) доказано.
18
Шаг 9. На шаге 8 мы убедились, что замена рациональных чисел 𝑟 соответствующими «рациональными сечениями» 𝑟∗ ∈ 𝑅 сохраняет сумму, произведение и порядок. Этот факт можно выразить,
сказав, что упорядоченное поле 𝑄 изоморфно упорядоченному полю 𝑄∗ , элементами которого являются рациональные сечения. Конечно, 𝑟∗ совсем не то же самое, что 𝑟, но интересующие нас
свойства (арифметические и порядковые) совпадают в обоих полях.
Это отождествление 𝑄 и 𝑄∗ позволяет нам рассматривать 𝑄 как подполе поля 𝑅.
Вторая часть теоремы 1.19 должна пониматься в смысле этого отождествления. Заметим, что
то же самое происходит, когда вещественные числа рассматриваются как подполе поля комплексных чисел, а также на гораздо более элементарном уровне, когда целые числа отождествляются с
некоторым подмножеством множества 𝑄.
Фактически, хотя мы не доказываем это, любые два упорядоченных поля со свойством верхней
грани изоморфны. Поэтому первая часть 1.19 полностью характеризует поле вещественных чисел
𝑅.
Книги Ландау и Терстона в списке литературы полностью посвящены числовым системам. Глава 1 книги Кноппа содержит более неторопливое описание построения поля 𝑅 из 𝑄. Еще одно
построение, в котором каждое вещественное число определяется как класс эквивалентности последовательностей Коши рациональных чисел (см. гл. 3), приводится в разд. 5 книги Хьюитта и
Стромберга.
Использованные нами сечения в 𝑄 были предложены Дедекиндом. Построение поля 𝑅 из 𝑄 с
помощью последовательностей Коши принадлежит Кантору. И Кантор, и Дедекинд опубликовали
свои построения в 1872 году.
Упражнения
1.
2.
3.
4.
Если 𝑟 рационально (𝑟 ≠ 0), а 𝑥 иррационально, доказать, что 𝑟 + 𝑥 и 𝑟𝑥 иррациональны.
Доказать, что не существует рационального числа, квадрат которого равен 12.
Доказать предложение 1.15.
Пусть 𝐸 — непустое подмножество упорядоченного множества, 𝛼 — нижняя граница множества 𝐸 и 𝛽 —
верхняя граница множества 𝐸. Доказать, что 𝛼 ≤ 𝛽.
5. Пусть 𝐴 — непустое ограниченное снизу множество вещественных чисел. Пусть −𝐴 — множество всех
чисел −𝑥, где 𝑥 ∈ 𝐴. Доказать, что
inf 𝐴 = − sup(−𝐴).
6. Зафиксируем 𝑏 > 1.
(a) Если 𝑚, 𝑛, 𝑝, 𝑞 — целые, 𝑛 > 0, 𝑞 > 0 и 𝑟 = 𝑚/𝑛 = 𝑝/𝑞, доказать, что
(𝑏𝑚 )1/𝑛 = (𝑏𝑝 )1/𝑞 .
Следовательно, имеет смысл определить 𝑏𝑟 = (𝑏𝑚 )1/𝑛 .
(b) Доказать, что 𝑏𝑟+𝑠 = 𝑏𝑟 𝑏𝑠 , если 𝑟 и 𝑠 рациональны.
(c) Если 𝑥 вещественно, определить 𝐵(𝑥) как множество всех чисел 𝑏𝑡 , где 𝑡 рационально и 𝑡 ≤ 𝑥.
Доказать, что
𝑏𝑟 = sup 𝐵(𝑟),
если 𝑟 рационально. Следовательно, имеет смысл определить
𝑏𝑥 = sup 𝐵(𝑥)
для всех вещественных 𝑥.
(d) Доказать, что 𝑏𝑥+𝑦 = 𝑏𝑥 𝑏𝑦 для всех вещественных 𝑥 и 𝑦.
7. Зафиксируем 𝑏 > 1, 𝑦 > 0. Доказать, что существует одно и только одно вещественное число 𝑥, для
которого 𝑏𝑥 = 𝑦, пользуясь следующим планом (это 𝑥 называется логарифмом 𝑦 по основанию 𝑏).
Для любого положительного целого 𝑛 выполняется 𝑏𝑛 − 1 ≥ 𝑛(𝑏 − 1).
Следовательно, 𝑏 − 1 ≥ 𝑛(𝑏1/𝑛 − 1).
Если 𝑡 > 1 и 𝑛 > (𝑏 − 1)/(𝑡 − 1), то 𝑏1/𝑛 < 𝑡.
Если 𝑤 таково, что 𝑏𝑤 < 𝑦, то 𝑏𝑤+(1/𝑛) < 𝑦 при достаточно большом 𝑛; чтобы убедиться в этом,
применить часть (c) для 𝑡 = 𝑦 ⋅ 𝑏−𝑤 .
(e) Если 𝑏𝑤 > 𝑦, то 𝑏𝑤−(1/𝑛) > 𝑦 при достаточно большом 𝑛.
(f) Пусть 𝐴 — множество всех 𝑤, таких, что 𝑏𝑤 < 𝑦. Показать, что 𝑥 = sup 𝐴 удовлетворяет 𝑏𝑥 = 𝑦.
(a)
(b)
(c)
(d)
19
(g) Доказать, что такое 𝑥 только одно.
8. Доказать, что в поле комплексных чисел не может быть определено отношение порядка, превращающее
его в упорядоченное поле. Указание: −1 является квадратом.
9. Пусть 𝑧 = 𝑎 + 𝑏𝑖, 𝑤 = 𝑐 + 𝑑𝑖. Будем считать, что 𝑧 < 𝑤, если 𝑎 < 𝑐, либо если 𝑎 = 𝑐 и 𝑏 < 𝑑. Доказать,
что это превращает множество комплексных чисел в упорядоченное множество (такой тип отношения
порядка по очевидным причинам называется словарным или лексикографическим порядком). Обладает
ли это упорядоченное множество свойством верхней грани?
10. Пусть 𝑧 = 𝑎 + 𝑏𝑖, 𝑤 = 𝑢 + 𝑖𝑣,
1/2
1/2
|𝑤| + 𝑢
|𝑤| − 𝑢
𝑎=
,
𝑏=
.
( 2 )
( 2 )
Доказать, что 𝑧2 = 𝑤, если 𝑣 ≥ 0, и (𝑧)̄ 2 = 𝑤, если 𝑣 ≤ 0. Сделать вывод, что каждое комплексное число
(за одним исключением!) имеет два комплексных квадратных корня.
11. Если 𝑧 — комплексное число, доказать, что существуют 𝑟 ≥ 0 и комплексное число 𝑤 с |𝑤| = 1, такие,
что 𝑧 = 𝑟𝑤. Однозначно ли 𝑤 и 𝑟 определяются по 𝑧?
12. Если 𝑧1 , … , 𝑧𝑛 — комплексные числа, доказать, что
|𝑧1 + 𝑧2 + ⋯ + 𝑧𝑛 | ≤ |𝑧1 | + |𝑧2 | + ⋯ + |𝑧𝑛 |.
13. Если 𝑥 и 𝑦 — комплексные числа, доказать, что
||𝑥| − |𝑦|| ≤ |𝑥 − 𝑦|.
14. Если 𝑧 — такое комплексное число, что |𝑧| = 1, то есть 𝑧𝑧 ̄ = 1, вычислить
|1 + 𝑧|2 + |1 − 𝑧|2 .
15. При каких условиях неравенство Шварца превращается в равенство?
16. Пусть 𝑘 ≥ 3, x, y ∈ 𝑅𝑘 , |x − y| = 𝑑 > 0 и 𝑟 > 0. Доказать:
(a) Если 2𝑟 > 𝑑, существует бесконечно много z ∈ 𝑅𝑘 , таких, что
|z − x| = |z − y| = 𝑟.
(b) Если 2𝑟 = 𝑑, существует ровно одно такое z.
(c) Если 2𝑟 < 𝑑, таких z не существует.
Как следует изменить эти утверждения при 𝑘, равном 2 или 1?
17. Доказать, что
|x + y|2 + |x − y|2 = 2|x|2 + 2|y|2 ,
если x ∈ 𝑅𝑘 и y ∈ 𝑅𝑘 . Истолковать это геометрически, как некоторое утверждение о параллелограммах.
18. Если 𝑘 ≥ 2 и x ∈ 𝑅𝑘 , доказать, что существует y ∈ 𝑅𝑘 , такое, что y ≠ 0, но x ⋅ y = 0. Остается ли это
истинным при 𝑘 = 1?
19. Пусть a ∈ 𝑅𝑘 , b ∈ 𝑅𝑘 . Найти c ∈ 𝑅𝑘 и 𝑟 > 0, такие, что
|x − a| = 2|x − b|
тогда и только тогда, когда |x − c| = 𝑟.
(Решение: 3c = 4b − a, 3𝑟 = 2|b − a|.)
20. Предположим, что в определении сечения было опущено свойство (III). Сохранить определения порядка
и сложения. Показать, что получающееся упорядоченное множество обладает свойством верхней грани,
сложение удовлетворяет аксиомам от (A1) до (A4) (со слегка другим нулевым элементом!), но (A5)
нарушается.
20
Глава 2
Элементы топологии
Конечные, счетные и несчетные множества
Мы начнем этот раздел с определения понятия функции.
2.1 Определение. Рассмотрим два множества 𝐴 и 𝐵, элементами которых могут быть любые
объекты, и предположим, что каждому элементу 𝑥 множества 𝐴 некоторым образом поставлен в
соответствие элемент множества 𝐵, который мы обозначим через 𝑓 (𝑥). Тогда 𝑓 называется функцией из 𝐴 в 𝐵 (или отображением 𝐴 в 𝐵). Множество 𝐴 называется областью определения 𝑓
(мы будем говорить также, что 𝑓 определена на 𝐴), а элементы 𝑓 (𝑥) называются значениями 𝑓 .
Множество всех значений функции 𝑓 называется ее множеством значений.
2.2 Определение. Пусть 𝐴 и 𝐵 — два множества, и пусть 𝑓 — отображение 𝐴 в 𝐵. Если 𝐸 ⊂ 𝐴, то
𝑓 (𝐸) определяется как множество всех элементов 𝑓 (𝑥) для 𝑥 ∈ 𝐸. Мы будем называть 𝑓 (𝐸) образом
𝐸 при отображении 𝑓 . В этих обозначениях 𝑓 (𝐴) — это множество значений 𝑓 . Ясно, что 𝑓 (𝐴) ⊂ 𝐵.
Если 𝑓 (𝐴) = 𝐵, то мы будем говорить, что 𝑓 отображает 𝐴 на 𝐵 (заметим, что в соответствии с
этим словоупотреблением на — частный случай в).
Если 𝐸 ⊂ 𝐵, то 𝑓 −1 (𝐸) обозначает множество всех 𝑥 ∈ 𝐴, таких, что 𝑓 (𝑥) ∈ 𝐸. Мы будем
называть 𝑓 −1 (𝐸) прообразом 𝐸 при отображении 𝑓 . Если 𝑦 ∈ 𝐵, то 𝑓 −1 (𝑦) — это множество всех
𝑥 ∈ 𝐴, таких, что 𝑓 (𝑥) = 𝑦. Если при каждом 𝑦 ∈ 𝐵 множество 𝑓 −1 (𝑦) состоит не более чем из одного
элемента 𝐴, то 𝑓 называется взаимно однозначным отображением 𝐴 в 𝐵. Это можно выразить
следующим образом: 𝑓 является взаимно однозначным отображением 𝐴 в 𝐵, если 𝑓 (𝑥1 ) ≠ 𝑓 (𝑥2 )
каждый раз, когда 𝑥1 ≠ 𝑥2 , 𝑥1 ∈ 𝐴, 𝑥2 ∈ 𝐴.
(Запись 𝑥1 ≠ 𝑥2 означает, что 𝑥1 и 𝑥2 — различные элементы; в противном случае мы пишем
𝑥1 = 𝑥2 .)
2.3 Определение. Если существует взаимно однозначное отображение 𝐴 на 𝐵, то мы будем говорить, что между 𝐴 и 𝐵 может быть установлено взаимно однозначное соответствие, или что 𝐴 и
𝐵 имеют одно и то же кардинальное число, или, короче, что 𝐴 и 𝐵 эквивалентны, и будем писать
𝐴 ∼ 𝐵. Это отношение, очевидно, обладает следующими свойствами:
рефлексивность: 𝐴 ∼ 𝐴;
симметричность: если 𝐴 ∼ 𝐵, то 𝐵 ∼ 𝐴;
транзитивность: если 𝐴 ∼ 𝐵 и 𝐵 ∼ 𝐶, то 𝐴 ∼ 𝐶.
Всякое отношение, обладающее этими тремя свойствами, называется отношением эквивалентности.
2.4 Определение. Для любого положительного целого 𝑛 пусть 𝐽𝑛 — множество, элементами которого являются целые числа 1, … , 𝑛; пусть 𝐽 — множество, состоящее из всех положительных целых
чисел. Для любого множества 𝐴 мы будем говорить, что
(a) 𝐴 конечно, если 𝐴 ∼ 𝐽𝑛 при некотором 𝑛 (пустое множество также считается конечным);
(b) 𝐴 бесконечно, если 𝐴 не является конечным;
(c) 𝐴 счетно, если 𝐴 ∼ 𝐽 ;
21
(d) 𝐴 несчетно, если 𝐴 не конечно и не счетно;
(e) 𝐴 не более чем счетно, если 𝐴 или конечно, или счетно.
Для конечных множеств 𝐴 и 𝐵 очевидно, что 𝐴 ∼ 𝐵 тогда и только тогда, когда 𝐴 и 𝐵 содержат
одно и то же число элементов. Однако для бесконечных множеств смысл слов «содержат одно и то
же число элементов» становится очень туманным, в то время как понятие взаимно однозначного
соответствия сохраняет ясность.
2.5 Пример. Пусть 𝐴 — множество всех целых чисел. Тогда 𝐴 — счетное множество. Действительно, рассмотрим следующее расположение множеств 𝐴 и 𝐽 :
𝐴∶
0, 1, −1, 2, −2, 3, −3, …
𝐽∶
1, 2, 3, 4, 5, 6, 7, …
В этом примере мы можем даже указать в явном виде формулу функции 𝑓 из 𝐽 в 𝐴, устанавливающую взаимно однозначное соответствие:
𝑓 (𝑛) =
𝑛
2
𝑛−1
{− 2
(𝑛 четно),
(𝑛 нечетно).
2.6 Замечание. Конечное множество не может быть эквивалентно своему собственному подмножеству. Однако для бесконечных множеств это возможно, как показывает пример 2.5, в котором
𝐽 — собственное подмножество множества 𝐴.
В действительности мы могли бы заменить определение 2.4(b) следующей формулировкой: 𝐴
бесконечно, если 𝐴 эквивалентно своему собственному подмножеству.
2.7 Определение. Последовательностью мы будем называть функцию 𝑓 , определенную на множестве 𝐽 всех положительных целых чисел. Если 𝑓 (𝑛) = 𝑥𝑛 при 𝑛 ∈ 𝐽 , то принято обозначать
последовательность 𝑓 символом {𝑥𝑛 } или писать 𝑥1 , 𝑥2 , 𝑥3 , … . Значения функции 𝑓 , т. е. элементы
𝑥𝑛 , называются членами последовательности. Если 𝐴 — некоторое множество и если 𝑥𝑛 ∈ 𝐴 при
всех 𝑛 ∈ 𝐽 , то {𝑥𝑛 } называется последовательностью в 𝐴, или последовательностью элементов
множества 𝐴.
Заметим, что члены 𝑥1 , 𝑥2 , 𝑥3 , … последовательности не обязаны быть различными.
Ввиду того что всякое счетное множество служит множеством значений некоторой определенной на 𝐽 взаимно однозначной функции, мы можем рассматривать всякое счетное множество как
множество значений некоторой последовательности с различными членами. Допуская некоторую
вольность речи, говорят, что элементы любого счетного множества можно «расположить в последовательность».
Иногда удобно заменить в этом определении 𝐽 множеством всех неотрицательных целых чисел,
т. е. начинать с 0, а не с 1.
2.8 Теорема. Всякое бесконечное подмножество счетного множества 𝐴 счетно.
Доказательство. Предположим, что 𝐸 ⊂ 𝐴 и 𝐸 бесконечно. Расположим элементы 𝑥 множества 𝐴
в последовательность {𝑥𝑛 } с различными членами. Построим последовательность {𝑛𝑘 } следующим
образом.
Пусть 𝑛1 — наименьшее целое положительное число, такое, что 𝑥𝑛1 ∈ 𝐸. Если 𝑛1 , … , 𝑛𝑘−1 (𝑘 =
2, 3, 4, … ) уже выбраны, то пусть 𝑛𝑘 — наименьшее целое число, большее 𝑛𝑘−1 и такое, что 𝑥𝑛𝑘 ∈ 𝐸.
Полагая 𝑓 (𝑘) = 𝑥𝑛𝑘 (𝑘 = 1, 2, 3, … ), мы получим взаимно однозначное соответствие между 𝐸 и
𝐽.
Эта теорема показывает, что, грубо говоря, счетные множества представляют «наименьшую»
бесконечность: никакое несчетное множество не может быть подмножеством счетного.
2.9 Определение. Пусть 𝐴 и Ω — множества; предположим, что каждому элементу 𝛼 множества
𝐴 сопоставлено некоторое подмножество множества Ω, которое мы обозначим через 𝐸𝛼 .
Множество, элементами которого служат множества 𝐸𝛼 , будет обозначаться через {𝐸𝛼 }. Вместо
того чтобы говорить о множестве множеств, мы иногда будем говорить о наборе множеств или о
семействе множеств.
22
Объединение множеств 𝐸𝛼 определяется как множество 𝑆, такое, что 𝑥 ∈ 𝑆 тогда и только тогда,
когда 𝑥 ∈ 𝐸𝛼 хотя бы при одном 𝛼 ∈ 𝐴. Мы будем пользоваться обозначением
(1)
𝑆=
⋃
𝐸𝛼 .
𝛼∈𝐴
Если 𝐴 состоит из целых чисел 1, … , 𝑛, то обычно пишут
𝑛
(2)
𝑆=
⋃
𝐸𝑚
𝑚=1
или
(3)
𝑆 = 𝐸 1 ∪ 𝐸2 ∪ ⋯ ∪ 𝐸 𝑛 .
Если 𝐴 — множество всех положительных целых чисел, то обычное обозначение таково:
∞
(4)
𝑆=
⋃
𝐸𝑚 .
𝑚=1
Символ ∞ в (4) указывает только, что берется объединение счетного семейства множеств. Его
не следует смешивать с символами +∞ и −∞, введенными в определении 1.23.
Пересечение множеств 𝐸𝛼 определяется как множество 𝑃 , такое, что 𝑥 ∈ 𝑃 тогда и только тогда,
когда 𝑥 ∈ 𝐸𝛼 при всех 𝛼 ∈ 𝐴. Мы будем пользоваться обозначением
(5)
𝑃 =
⋂
𝐸𝛼 ,
𝛼∈𝐴
или
𝑛
(6)
𝑃 =
⋂
𝐸𝑚 = 𝐸1 ∩ 𝐸2 ∩ ⋯ ∩ 𝐸𝑛 ,
𝑚=1
или
∞
(7)
𝑃 =
⋂
𝐸𝑚 ,
𝑚=1
как для объединений. Если 𝐴 ∩ 𝐵 не пусто, то мы будем говорить, что 𝐴 и 𝐵 пересекаются, в
противном случае — что они не пересекаются.
2.10 Примеры.
(a) Допустим, что 𝐸1 состоит из 1, 2, 3, а 𝐸2 — из 2, 3, 4. Тогда 𝐸1 ∪ 𝐸2 состоит из 1, 2, 3, 4, в то
время как 𝐸1 ∩ 𝐸2 — из 2, 3.
(b) Пусть 𝐴 — множество всех вещественных чисел 𝑥, таких, что 0 < 𝑥 ≤ 1. Для любого 𝑥 ∈ 𝐴
пусть 𝐸𝑥 — множество всех вещественных чисел 𝑦, таких, что 0 < 𝑦 < 𝑥. Тогда
(i) 𝐸𝑥 ⊂ 𝐸𝑧 тогда и только тогда, когда 0 < 𝑥 ≤ 𝑧 ≤ 1;
(ii) ⋃𝑥∈𝐴 𝐸𝑥 = 𝐸1 ;
(iii) ⋂𝑥∈𝐴 𝐸𝑥 пусто.
Утверждения (i) и (ii) очевидны. Чтобы доказать (iii), заметим, что при любом 𝑦 > 0 имеем
𝑦 ∉ 𝐸𝑥 , если 𝑥 < 𝑦. Значит, 𝑦 ∉ ⋂𝑥∈𝐴 𝐸𝑥 .
2.11 Замечания. Многие свойства объединений и пересечений совершенно аналогичны свойствам
сумм и произведений. В действительности слова «сумма» и «произведение» иногда употребляют в
этом смысле и вместо ⋃ и ⋂ пишут ∑ и ∏.
Законы коммутативности и ассоциативности тривиально выполняются:
(8)
𝐴 ∪ 𝐵 = 𝐵 ∪ 𝐴;
𝐴 ∩ 𝐵 = 𝐵 ∩ 𝐴;
(9)
(𝐴 ∪ 𝐵) ∪ 𝐶 = 𝐴 ∪ (𝐵 ∪ 𝐶);
(𝐴 ∩ 𝐵) ∩ 𝐶 = 𝐴 ∩ (𝐵 ∩ 𝐶).
23
Этим оправдано отсутствие скобок в (3) и (6).
Закон дистрибутивности также выполняется:
(10)
𝐴 ∩ (𝐵 ∪ 𝐶) = (𝐴 ∩ 𝐵) ∪ (𝐴 ∩ 𝐶).
Чтобы доказать это, обозначим левую и правую части равенства (10) соответственно через 𝐸 и 𝐹 .
Допустим, что 𝑥 ∈ 𝐸. Тогда 𝑥 ∈ 𝐴 и 𝑥 ∈ 𝐵 ∪ 𝐶, т. е. 𝑥 ∈ 𝐵 или 𝑥 ∈ 𝐶 (или и то и другое). Значит,
𝑥 ∈ 𝐴 ∩ 𝐵 или 𝑥 ∈ 𝐴 ∩ 𝐶, так что 𝑥 ∈ 𝐹 . Таким образом, 𝐸 ⊂ 𝐹 .
Предположим теперь, что 𝑥 ∈ 𝐹 . Тогда 𝑥 ∈ 𝐴 ∩ 𝐵 или 𝑥 ∈ 𝐴 ∩ 𝐶. Таким образом, 𝑥 ∈ 𝐴 и
𝑥 ∈ 𝐵 ∪ 𝐶. Значит, 𝑥 ∈ 𝐴 ∩ (𝐵 ∪ 𝐶), так что 𝐹 ⊂ 𝐸.
Следовательно, 𝐸 = 𝐹 .
Перечислим еще несколько легко проверяемых соотношений:
(11)
𝐴 ⊂ 𝐴 ∪ 𝐵,
(12)
𝐴 ∩ 𝐵 ⊂ 𝐴.
Если 0 обозначает пустое множество, то
(13)
𝐴 ∪ 0 = 𝐴,
𝐴 ∩ 0 = 0.
𝐴 ∪ 𝐵 = 𝐵,
𝐴 ∩ 𝐵 = 𝐴.
Если 𝐴 ⊂ 𝐵, то
(14)
2.12 Теорема. Пусть {𝐸𝑛 }, 𝑛 = 1, 2, 3, … , — последовательность счетных множеств; положим
∞
(15)
𝑆=
⋃
𝐸𝑛 .
𝑛=1
Тогда 𝑆 счетно.
Доказательство. Расположим каждое множество 𝐸𝑛 в последовательность {𝑥𝑛𝑘 }, 𝑘 = 1, 2, 3, … , и
рассмотрим бесконечную таблицу
𝑥11
𝑥13
𝑥14
…
𝑥22
𝑥23
x<
x<
x
x
xx
xx
𝑥32
𝑥33
𝑥31
<
xx
x
x
𝑥41
𝑥42
𝑥43
𝑥24
…
𝑥34
…
𝑥44
…
…
…
…
<
xx
xx
𝑥12
<
xx
xx
𝑥21
(16)
…
<
xx
xx
…
в которой элементы 𝐸𝑛 образуют 𝑛-ую строку. Эта таблица содержит все элементы множества 𝑆.
Как указывают стрелки, эти элементы можно расположить в последовательность
(17)
𝑥11 ; 𝑥21 , 𝑥12 ; 𝑥31 , 𝑥22 , 𝑥13 ; 𝑥41 , 𝑥32 , 𝑥23 , 𝑥14 ; … .
Если какие-нибудь два множества 𝐸𝑛 имеют общие элементы, то они появятся в (17) более чем один
раз. Значит, существует подмножество 𝑇 множества всех положительных целых чисел, такое, что
𝑆 ∼ 𝑇 , откуда следует, что 𝑆 не более чем счетно (теорема 2.8). Поскольку 𝐸1 ⊂ 𝑆 и 𝐸1 бесконечно,
то и 𝑆 бесконечно, и поэтому счетно.
Следствие. Допустим, что 𝐴 не более чем счетно и что при каждом 𝛼 ∈ 𝐴 множество 𝐵𝛼 не
более чем счетно. Положим
𝑇 =
𝐵 .
⋃ 𝛼
𝛼∈𝐴
Тогда 𝑇 не более чем счетно.
24
Действительно, 𝑇 эквивалентно некоторому подмножеству множества (15).
2.13 Теорема. Пусть 𝐴 — счетное множество, и пусть 𝐵𝑛 — множество всех наборов (𝑎1 , … , 𝑎𝑛 )
из 𝑛 членов, где 𝑎𝑘 ∈ 𝐴 (𝑘 = 1, … , 𝑛) и элементы 𝑎1 , … , 𝑎𝑛 не обязательно различны. Тогда 𝐵𝑛
счетно.
Доказательство. То, что 𝐵1 счетно, очевидно, так как 𝐵1 = 𝐴. Допустим, что 𝐵𝑛−1 счетно (𝑛 =
2, 3, 4, … ). Элементы 𝐵𝑛 имеют вид
(18)
(𝑏, 𝑎)
(𝑏 ∈ 𝐵𝑛−1 , 𝑎 ∈ 𝐴).
При каждом фиксированном 𝑏 множество пар (𝑏, 𝑎) эквивалентно 𝐴 и, значит, счетно. Таким образом, 𝐵𝑛 — объединение счетного множества счетных множеств. По теореме 2.12 𝐵𝑛 счетно.
Утверждение теоремы доказано по индукции.
Следствие. Множество всех рациональных чисел счетно.
Доказательство. Применим теорему 2.13, взяв 𝑛 = 2 и заметив, что каждое рациональное число
𝑟 имеет вид 𝑏/𝑎, где 𝑎 и 𝑏 — целые числа. Множество пар (𝑎, 𝑏), а поэтому и множество дробей 𝑏/𝑎,
счетно.
На самом деле даже множество всех алгебраических чисел счетно (см. упражнение 2).
Следующая теорема показывает, однако, что не все бесконечные множества счетны.
2.14 Теорема. Пусть 𝐴 — множество всех последовательностей, элементы которых — цифры
0 и 1. Множество 𝐴 несчетно.
Элементами 𝐴 служат последовательности 1, 0, 0, 1, 0, 1, 1, 1, … .
Доказательство. Пусть 𝐸 — счетное подмножество множества 𝐴, и пусть 𝐸 состоит из последовательностей 𝑠1 , 𝑠2 , 𝑠3 , … . Построим последовательность 𝑠 следующим образом. Если 𝑛-ая цифра в
𝑠𝑛 равна 1, то пусть 𝑛-ая цифра в 𝑠 равна 0, и наоборот. Тогда последовательность 𝑠 отличается от
каждого элемента множества 𝐸 хотя бы одним членом, значит, 𝑠 ∉ 𝐸. Но очевидно, что 𝑠 ∈ 𝐴, так
что 𝐸 — собственное подмножество множества 𝐴.
Мы показали, что каждое счетное подмножество множества 𝐴 является собственным подмножеством. Следовательно, 𝐴 несчетно (потому что в противном случае 𝐴 было бы своим собственным
подмножеством, что невозможно).
Идею приведенного доказательства впервые использовал Кантор, и она называется канторовским диагональным процессом, потому что если последовательности 𝑠1 , 𝑠2 , 𝑠3 , … расположить в
виде таблицы типа (16), то при построении новой последовательности будут учитываться элементы диагонали.
Читатели, знакомые с двоичным представлением вещественных чисел (по основанию 2 вместо
10), заметят, что из теоремы 2.14 следует, что множество всех вещественных чисел несчетно1 .
Второе доказательство этого факта мы дадим в теореме 2.43.
Метрические пространства
2.15 Определение. Говорят, что множество 𝑋, элементы которого мы будем называть точками, есть метрическое пространство, если любым двум точкам 𝑝 и 𝑞 множества 𝑋 соответствует
вещественное число 𝑑(𝑝, 𝑞), называемое расстоянием от 𝑝 до 𝑞, такое, что
(a) 𝑑(𝑝, 𝑞) > 0, если 𝑝 ≠ 𝑞; 𝑑(𝑝, 𝑝) = 0;
(b) 𝑑(𝑝, 𝑞) = 𝑑(𝑞, 𝑝);
(c) 𝑑(𝑝, 𝑞) ≤ 𝑑(𝑝, 𝑟) + 𝑑(𝑟, 𝑞) при любом 𝑟 ∈ 𝑋.
Любая функция, удовлетворяющая этим трем свойствам, называется функцией расстояния или
метрикой.
1
Из-за того, что некоторые числа имеют два двоичных представления (0,111 … = 1,000 … ), доказательство по основанию
2 некорректно — может оказаться, что 𝑠 ∈ 𝐸. Но его можно легко исправить, перейдя к основанию 4, то есть использовав
по два двоичных разряда у каждого 𝑠𝑖 . — Прим. перев.
25
2.16 Примеры. Самыми важными примерами метрических пространств с нашей точки зрения
служат евклидовы пространства 𝑅𝑘 , особенно 𝑅1 (вещественная прямая) и 𝑅2 (комплексная плоскость); расстояние в 𝑅𝑘 определяется так:
(19)
𝑑(x, y) = |x − y|
(x, y ∈ 𝑅𝑘 ).
По теореме 1.37 метрика (19) удовлетворяет условиям определения 2.15.
Важно заметить, что каждое подмножество 𝑌 метрического пространства 𝑋 в свою очередь
является метрическим пространством с той же самой функцией расстояния. В самом деле, ясно,
что если условия (a)–(c) определения 2.15 выполнены для 𝑝, 𝑞, 𝑟 ∈ 𝑋, то они выполнены и для 𝑝, 𝑞, 𝑟,
лежащих в 𝑌 .
Таким образом, каждое подмножество евклидова пространства — метрическое пространство.
Другими примерами служат пространства 𝒞 (𝐾) и ℒ 2 (𝜇), рассматриваемые соответственно в гл. 7
и 11.
2.17 Определение. Под интервалом (𝑎, 𝑏) мы будем понимать множество всех вещественных
чисел 𝑥, таких, что 𝑎 < 𝑥 < 𝑏.
Под сегментом [𝑎, 𝑏] мы будем понимать множество всех вещественных чисел 𝑥, таких, что
𝑎 ≤ 𝑥 ≤ 𝑏.
Иногда мы будем встречаться с полуинтервалами [𝑎, 𝑏) и (𝑎, 𝑏]; первый состоит из всех 𝑥, таких,
что 𝑎 ≤ 𝑥 < 𝑏, второй — из всех 𝑥, таких, что 𝑎 < 𝑥 ≤ 𝑏.
Если 𝑎𝑖 < 𝑏𝑖 при 𝑖 = 1, … , 𝑘, то множество всех точек x = (𝑥1 , … , 𝑥𝑘 ) в 𝑅𝑘 , координаты которых
удовлетворяют неравенствам 𝑎𝑖 ≤ 𝑥𝑖 ≤ 𝑏𝑖 (1 ≤ 𝑖 ≤ 𝑘), называется 𝑘-мерной клеткой. Таким образом,
одномерная клетка — это сегмент, двумерная клетка — прямоугольник и т. д.
Если x ∈ 𝑅𝑘 и 𝑟 > 0, то открытый (или замкнутый) шар 𝐵 с центром в x радиусом 𝑟 есть по
определению множество всех y ∈ 𝑅𝑘 , таких, что |y − x| < 𝑟 (или |y − x| ≤ 𝑟).
Назовем множество 𝐸 ⊂ 𝑅𝑘 выпуклым, если
𝜆x + (1 − 𝜆)y ∈ 𝐸
при любых x ∈ 𝐸, y ∈ 𝐸 и 0 < 𝜆 < 1.
Например, шары выпуклы, ибо если |y − x| < 𝑟, |z − x| < 𝑟 и 0 < 𝜆 < 1, то
|𝜆y + (1 − 𝜆)z − x| = |𝜆(y − x) + (1 − 𝜆)(z − x)|
≤ 𝜆|y − x| + (1 − 𝜆)|z − x| < 𝜆𝑟 + (1 − 𝜆)𝑟
= 𝑟.
То же доказательство применимо и к замкнутым шарам. Легко видеть также, что 𝑘-мерные клетки
выпуклы.
2.18 Определение. Пусть 𝑋 — метрическое пространство. Все упоминаемые ниже точки и множества следует считать элементами и подмножествами пространства 𝑋.
(a) Окрестностью точки 𝑝 называется множество 𝑁𝑟 (𝑝), состоящее из всех 𝑞, таких, что 𝑑(𝑝, 𝑞) < 𝑟
для какого-нибудь 𝑟 > 0. Число 𝑟 называется радиусом окрестности 𝑁𝑟 (𝑝).
(b) Точка 𝑝 называется предельной точкой множества 𝐸, если каждая окрестность 𝑝 содержит
точку 𝑞 ≠ 𝑝, такую, что 𝑞 ∈ 𝐸.
(c) Если 𝑝 ∈ 𝐸 и 𝑝 не является предельной точкой множества 𝐸, то 𝑝 называется изолированной
точкой множества 𝐸.
(d) 𝐸 замкнуто, если каждая предельная точка множества 𝐸 принадлежит 𝐸.
(e) Точка 𝑝 называется внутренней точкой множества 𝐸, если она имеет окрестность 𝑁, такую,
что 𝑁 ⊂ 𝐸.
(f) 𝐸 открыто, если каждая точка множества 𝐸 является его внутренней точкой.
(g) Дополнением множества 𝐸 (обозначается 𝐸 𝑐 ) называется множество всех точек 𝑝 ∈ 𝑋, таких,
что 𝑝 ∉ 𝐸.
(h) 𝐸 совершенно, если оно замкнуто и если каждая точка множества 𝐸 является его предельной
точкой.
(i) 𝐸 ограничено, если существуют вещественное число 𝑀 и точка 𝑞 ∈ 𝑋, такие, что 𝑑(𝑝, 𝑞) < 𝑀
при всех 𝑝 ∈ 𝐸.
26
(j) 𝐸 всюду плотно в 𝑋, если каждая точка множества 𝑋 либо является предельной точкой
множества 𝐸, либо принадлежит 𝐸 (либо и то, и другое).
Отметим, что окрестностями в 𝑅1 служат интервалы, в то время как окрестности в 𝑅2 — это
внутренности кругов.
2.19 Теорема. Всякая окрестность является открытым множеством.
Доказательство. Рассмотрим окрестность 𝐸 = 𝑁𝑟 (𝑝); пусть 𝑞 — какая-нибудь точка множества
𝐸. Тогда существует положительное вещественное число ℎ, такое, что
𝑑(𝑝, 𝑞) = 𝑟 − ℎ.
Для всех точек 𝑠, таких, что 𝑑(𝑞, 𝑠) < ℎ, мы имеем
𝑑(𝑝, 𝑠) ≤ 𝑑(𝑝, 𝑞) + 𝑑(𝑞, 𝑠) < 𝑟 − ℎ + ℎ < 𝑟,
так что 𝑠 ∈ 𝐸. Таким образом, 𝑞 — внутренняя точка множества 𝐸.
2.20 Теорема. Если 𝑝 — предельная точка множества 𝐸, то любая окрестность точки 𝑝 содержит бесконечно много точек множества 𝐸.
Доказательство. Предположим, что существует окрестность 𝑁 точки 𝑝, содержащая только конечное число точек множества 𝐸. Пусть 𝑞1 , … , 𝑞𝑛 — те точки множества 𝑁 ∩ 𝐸, которые не совпадают с 𝑝. Положим
𝑟 = min 𝑑(𝑝, 𝑞𝑚 )
1≤𝑚≤𝑛
(так мы обозначаем наименьшее из чисел 𝑑(𝑝, 𝑞1 ), … , 𝑑(𝑝, 𝑞𝑛 )). Ясно, что минимум конечного множества положительных чисел положителен, так что 𝑟 > 0.
Окрестность 𝑁𝑟 (𝑝) не содержит ни одной точки 𝑞 множества 𝐸, такой, что 𝑞 ≠ 𝑝, поэтому 𝑝 не
является предельной точкой множества 𝐸. Это противоречие и доказывает теорему.
Следствие. Конечное множество точек не имеет предельных точек.
2.21 Примеры. Рассмотрим следующие подмножества пространства 𝑅2 .
(a)
(b)
(c)
(d)
(e)
Множество всех комплексных 𝑧, таких, что |𝑧| < 1.
Множество всех комплексных 𝑧, таких, что |𝑧| ≤ 1.
Непустое конечное множество.
Множество всех целых чисел.
Множество, состоящее из чисел 1/𝑛 (𝑛 = 1, 2, 3, … ). Отметим, что это множество 𝐸 имеет предельную точку (а именно 𝑧 = 0), но никакая точка множества 𝐸 не является его предельной
точкой; мы хотим подчеркнуть разницу между наличием предельной точки и принадлежностью ее множеству.
(f) Множество всех комплексных чисел (т. е. 𝑅2 ).
(g) Интервал (𝑎, 𝑏).
Отметим, что множества (d), (e) и (g) можно рассматривать и как подмножества пространства
𝑅1 .
Некоторые свойства этих множеств перечислены в следующей таблице:
(a)
(b)
(c)
(d)
(e)
(f)
(g)
Замкнуто
Нет
Да
Да
Да
Нет
Да
Нет
Открыто
Да
Нет
Нет
Нет
Нет
Да
Совершенно
Нет
Да
Нет
Нет
Нет
Да
Нет
27
Ограничено
Да
Да
Да
Нет
Да
Нет
Да
В строке (g) мы не заполнили вторую клетку. Причина этого в том, что интервал (𝑎, 𝑏) не есть
открытое множество, если рассматривать его как подмножество пространства 𝑅2 , но он является
открытым подмножеством пространства 𝑅1 .
2.22 Теорема. Пусть {𝐸𝛼 } — семейство (конечное или бесконечное) множеств 𝐸𝛼 . Тогда
𝑐
(20)
(⋃
𝛼
𝐸𝛼
)
=
⋂
(𝐸𝛼𝑐 ).
𝛼
Доказательство. Пусть 𝐴 и 𝐵 — левая и правая части равенства (20). Если 𝑥 ∈ 𝐴, то 𝑥 ∉ ⋃𝛼 𝐸𝛼 ,
значит, 𝑥 ∉ 𝐸𝛼 при всех 𝛼, поэтому 𝑥 ∈ 𝐸𝛼𝑐 при каждом 𝛼, так что 𝑥 ∈ ⋂ 𝐸𝛼𝑐 . Таким образом, 𝐴 ⊂ 𝐵.
Обратно, если 𝑥 ∈ 𝐵, то 𝑥 ∈ 𝐸𝛼𝑐 при каждом 𝛼, значит, 𝑥 ∉ 𝐸𝛼 при всех 𝛼, поэтому 𝑥 ∉ ⋃𝛼 𝐸𝛼 ,
так что 𝑥 ∈ (⋃𝛼 𝐸𝛼 )𝑐 . Таким образом, 𝐵 ⊂ 𝐴.
Следовательно, 𝐴 = 𝐵.
2.23 Теорема. Множество 𝐸 открыто тогда и только тогда, когда его дополнение замкнуто.
Доказательство. Сначала предположим, что 𝐸 𝑐 замкнуто. Выберем 𝑥 ∈ 𝐸. Тогда 𝑥 ∉ 𝐸 𝑐 и 𝑥 не
является предельной точкой множества 𝐸 𝑐 . Значит, существует окрестность 𝑁 точки 𝑥, такая, что
𝐸 𝑐 ∩ 𝑁 пусто, т. е. 𝑁 ⊂ 𝐸. Таким образом, 𝑥 — внутренняя точка множества 𝐸, и 𝐸 открыто.
Теперь предположим, что 𝐸 открыто. Пусть 𝑥 — предельная точка множества 𝐸 𝑐 . Тогда каждая
окрестность точки 𝑥 содержит некоторую точку множества 𝐸 𝑐 , так что 𝑥 не является внутренней
точкой множества 𝐸. Поскольку 𝐸 открыто, это значит, что 𝑥 ∈ 𝐸 𝑐 . Следовательно, 𝐸 𝑐 замкнуто.
Следствие. Множество 𝐹 замкнуто тогда и только тогда, когда его дополнение открыто.
2.24 Теорема.
(a) Для любого семейства {𝐺𝛼 } открытых множеств множество ⋃𝛼 𝐺𝛼 открыто.
(b) Для любого семейства {𝐹𝛼 } замкнутых множеств множество ⋂𝛼 𝐹𝛼 замкнуто.
(c) Для любого конечного семейства 𝐺1 , … , 𝐺𝑛 открытых множеств множество ⋂𝑛𝑖=1 𝐺𝑖 открыто.
(d) Для любого конечного семейства 𝐹1 , … , 𝐹𝑛 замкнутых множеств множество ⋃𝑛𝑖=1 𝐹𝑖 замкнуто.
Доказательство. Положим 𝐺 = ⋃𝛼 𝐺𝛼 . Если 𝑥 ∈ 𝐺, то 𝑥 ∈ 𝐺𝛼 при некотором 𝛼. Так как 𝑥 —
внутренняя точка множества 𝐺𝛼 , то 𝑥 — внутренняя точка множества 𝐺, и 𝐺 открыто. Утверждение
(a) доказано.
По теореме 2.22 имеем
𝑐
(21)
(⋂
𝛼
𝐹𝛼
)
=
⋃
(𝐹𝛼𝑐 ),
𝛼
а по теореме 2.23 множества 𝐹𝛼𝑐 открыты. Значит, из (a) следует, что (21) открыто, так что ⋂𝛼 𝐹𝛼
замкнуто.
Теперь положим 𝐻 = ⋂𝑛𝑖=1 𝐺𝑖 . Для любого 𝑥 ∈ 𝐻 существует окрестность 𝑁𝑖 точки 𝑥 радиуса
𝑟𝑖 , такая, что 𝑁𝑖 ⊂ 𝐺𝑖 (𝑖 = 1, … , 𝑛). Положим
𝑟 = min(𝑟1 , … , 𝑟𝑛 ),
и пусть 𝑁 — окрестность точки 𝑥 радиуса 𝑟. Тогда 𝑁 ⊂ 𝐺𝑖 при 𝑖 = 1, … , 𝑛, так что 𝑁 ⊂ 𝐻, и 𝐻
открыто.
Переходя к дополнениям, мы выведем (d) из (c):
𝑐
𝑛
(⋃
𝑖=1
𝐹𝑖
)
𝑛
=
(𝐹 𝑐 ).
⋂ 𝑖
𝑖=1
28
2.25 Примеры. В утверждениях (c) и (d) предыдущей теоремы конечность семейств существенна.
Действительно, пусть 𝐺𝑛 — интервал (− 1𝑛 , 1𝑛 ) (𝑛 = 1, 2, 3, … ). Тогда 𝐺𝑛 — открытое подмножество
прямой 𝑅1 . Положим 𝐺 = ⋂∞
𝑛=1 𝐺𝑛 . Тогда 𝐺 состоит из единственной точки (а именно 𝑥 = 0) и
поэтому не является открытым подмножеством прямой 𝑅1 .
Таким образом, пересечение бесконечного семейства открытых множеств не обязано быть открытым. Подобным образом, объединение бесконечного семейства замкнутых множеств не обязано
быть замкнутым.
2.26 Определение. Если 𝑋 — метрическое пространство, 𝐸 ⊂ 𝑋, а 𝐸 ′ означает множество всех
предельных точек множества 𝐸 в 𝑋, то замыканием множества 𝐸 называется множество 𝐸̄ = 𝐸∪𝐸 ′ .
2.27 Теорема. Если 𝑋 — метрическое пространство и 𝐸 ⊂ 𝑋, то
(a) 𝐸̄ замкнуто;
(b) 𝐸 = 𝐸̄ тогда и только тогда, когда 𝐸 замкнуто;
(c) 𝐸̄ ⊂ 𝐹 для каждого замкнутого множества 𝐹 ⊂ 𝑋, такого, что 𝐸 ⊂ 𝐹 .
Из (a) и (c) следует, что 𝐸̄ — наименьшее замкнутое подмножество пространства 𝑋, содержащее
𝐸.
Доказательство.
(a) Если 𝑝 ∈ 𝑋 и 𝑝 ∉ 𝐸,̄ то 𝑝 не является ни точкой 𝐸, ни предельной точкой 𝐸. Значит, 𝑝
имеет окрестность, не пересекающую 𝐸. Следовательно, дополнение 𝐸̄ открыто. Значит, 𝐸̄
замкнуто.
(b) Если 𝐸 = 𝐸,̄ то из (a) следует, что 𝐸 замкнуто. Если 𝐸 замкнуто, то 𝐸 ′ ⊂ 𝐸 (по определениям
2.18(d) и 2.26), значит, 𝐸̄ = 𝐸.
(c) Если 𝐹 замкнуто и 𝐹 ⊃ 𝐸, то 𝐹 ⊃ 𝐹 ′ , значит, 𝐹 ⊃ 𝐸 ′ . Следовательно, 𝐹 ⊃ 𝐸.̄
2.28 Теорема. Пусть 𝐸 — непустое множество вещественных чисел, ограниченное сверху. Пусть
𝑦 = sup 𝐸. Тогда 𝑦 ∈ 𝐸.̄ Следовательно, 𝑦 ∈ 𝐸, если 𝐸 замкнуто.
Сравните это утверждение с примерами 1.9.
Доказательство. Если 𝑦 ∈ 𝐸, то 𝑦 ∈ 𝐸.̄ Допустим, что 𝑦 ∉ 𝐸. Для каждого ℎ > 0 существует
точка 𝑥 ∈ 𝐸, такая, что 𝑦 − ℎ < 𝑥 < 𝑦, так как иначе 𝑦 − ℎ было бы верхней границей множества 𝐸.
Таким образом, 𝑦 — предельная точка множества 𝐸. Значит, 𝑦 ∈ 𝐸.̄
2.29 Замечание. Допустим, что 𝐸 ⊂ 𝑌 ⊂ 𝑋, где 𝑋 — метрическое пространство. То, что 𝐸 —
открытое подмножество пространства 𝑋, означает, что с каждой точкой 𝑝 ∈ 𝐸 связано положительное число 𝑟, для которого из условий 𝑑(𝑝, 𝑞) < 𝑟, 𝑞 ∈ 𝑋 следует 𝑞 ∈ 𝐸. Но мы уже заметили
(п. 2.16), что 𝑌 — тоже метрическое пространство, так что наши определения с таким же успехом
могут проводиться внутри 𝑌 . Для полной точности мы будем говорить, что 𝐸 открыто относительно 𝑌 , если каждой точке 𝑝 ∈ 𝐸 отвечает 𝑟 > 0, такое, что 𝑞 ∈ 𝐸, если 𝑑(𝑝, 𝑞) < 𝑟 и 𝑞 ∈ 𝑌 . Пример
2.21(g) показывает, что множество может быть открытым относительно 𝑌 , не будучи открытым
подмножеством пространства 𝑋. Однако между этими понятиями имеется простое соотношение,
которое мы сейчас установим.
2.30 Теорема. Пусть 𝑌 ⊂ 𝑋. Подмножество 𝐸 множества 𝑌 открыто относительно 𝑌 тогда
и только тогда, когда 𝐸 = 𝑌 ∩ 𝐺 для некоторого открытого подмножества 𝐺 пространства 𝑋.
Доказательство. Допустим, что 𝐸 открыто относительно 𝑌 . Для каждого 𝑝 ∈ 𝐸 найдется положительное число 𝑟𝑝 , такое, что из условий 𝑑(𝑝, 𝑞) < 𝑟𝑝 , 𝑞 ∈ 𝑌 следует, что 𝑞 ∈ 𝐸. Пусть 𝑉𝑝 —
множество всех 𝑞 ∈ 𝑋, таких, что 𝑑(𝑝, 𝑞) < 𝑟𝑝 ; положим
𝐺=
⋃
𝑉𝑝 .
𝑝∈𝐸
Тогда 𝐺 — открытое подмножество пространства 𝑋 по теоремам 2.19 и 2.24.
Поскольку 𝑝 ∈ 𝑉𝑝 при всех 𝑝 ∈ 𝐸, ясно, что 𝐸 ⊂ 𝐺 ∩ 𝑌 .
29
Согласно нашему выбору окрестности 𝑉𝑝 , имеем 𝑉𝑝 ∩𝑌 ⊂ 𝐸 при каждом 𝑝 ∈ 𝐸, так что 𝐺∩𝑌 ⊂ 𝐸.
Таким образом, 𝐸 = 𝐺 ∩ 𝑌 , и половина теоремы доказана.
Обратно, если 𝐺 открыто в 𝑋 и 𝐸 = 𝐺 ∩ 𝑌 , то каждая точка 𝑝 ∈ 𝐸 имеет окрестность 𝑉𝑝 ⊂ 𝐺.
Тогда 𝑉𝑝 ∩ 𝑌 ⊂ 𝐸, так что множество 𝐸 открыто относительно 𝑌 .
Компактные множества
2.31 Определение. Открытым покрытием множества 𝐸 в метрическом пространстве 𝑋 мы
будем называть семейство {𝐺𝛼 } открытых подмножеств пространства 𝑋, такое, что 𝐸 ⊂ ⋃𝛼 𝐺𝛼 .
2.32 Определение. Подмножество 𝐾 метрического пространства 𝑋 называется компактным,
если каждое открытое покрытие множества 𝐾 содержит конечное подпокрытие.
Говоря точнее, требование состоит в том, что если {𝐺𝛼 } — открытое покрытие множества 𝐾, то
имеется конечное число индексов 𝛼1 , … , 𝛼𝑛 , таких, что
𝐾 ⊂ 𝐺𝛼1 ∪ ⋯ ∪ 𝐺𝛼𝑛 .
Понятие компактности имеет большое значение в анализе, особенно в связи с непрерывностью
(гл. 4).
Ясно, что каждое конечное множество компактно. Существование широкого класса бесконечных
компактных множеств в 𝑅𝑘 будет следовать из теоремы 2.41.
Мы заметили ранее (в п. 2.29), что если 𝐸 ⊂ 𝑌 ⊂ 𝑋, то множество 𝐸 может быть открытым
относительно 𝑌 , не будучи открытым относительно 𝑋. Свойство множества 𝐸 быть открытым
зависит, таким образом, от пространства, в которое 𝐸 погружено. То же верно и в отношении
свойства множества быть замкнутым.
Однако, как мы увидим, компактность — более удобное понятие. Чтобы сформулировать следующую теорему, мы будем говорить временно, что 𝐾 компактно относительно 𝑋, если выполнены
требования определения 2.32.
2.33 Теорема. Допустим, что 𝐾 ⊂ 𝑌 ⊂ 𝑋. Тогда 𝐾 компактно относительно 𝑋 тогда и только
тогда, когда 𝐾 компактно относительно 𝑌 .
В силу этой теоремы мы сможем во многих ситуациях рассматривать компактные множества
как метрические пространства сами по себе, не обращая никакого внимания на объемлющее пространство. В частности, хотя почти бессмысленно говорить об открытых пространствах или о
замкнутых пространствах (каждое метрическое пространство 𝑋 служит открытым подмножеством самого себя и замкнутым подмножеством самого себя), имеет смысл говорить о компактных
метрических пространствах.
Доказательство. Предположим, что 𝐾 компактно относительно 𝑋; пусть {𝑉𝛼 } — семейство множеств, открытых относительно 𝑌 , такое, что 𝐾 ⊂ ⋃𝛼 𝑉𝛼 . По теореме 2.30 для всех 𝛼 существуют
множества 𝐺𝛼 , открытые относительно 𝑋, такие, что 𝑉𝛼 = 𝑌 ∩ 𝐺𝛼 ; поскольку 𝐾 компактно относительно 𝑋, мы имеем
(22)
𝐾 ⊂ 𝐺𝛼1 ∪ ⋯ ∪ 𝐺𝛼𝑛
при некотором выборе конечного числа индексов 𝛼1 , … , 𝛼𝑛 . Так как 𝐾 ⊂ 𝑌 , то из (22) следует, что
(23)
𝐾 ⊂ 𝑉𝛼1 ∪ ⋯ ∪ 𝑉𝛼𝑛 .
Тем самым доказано, что 𝐾 компактно относительно 𝑌 .
Обратно, допустим, что 𝐾 компактно относительно 𝑌 . Пусть {𝐺𝛼 } — семейство открытых подмножеств пространства 𝑋, покрывающее 𝐾. Положим 𝑉𝛼 = 𝑌 ∩ 𝐺𝛼 . Тогда (23) будет выполнено
при некотором выборе 𝛼1 , … , 𝛼𝑛 ; так как 𝑉𝛼 ⊂ 𝐺𝛼 , то (22) следует из (23).
Доказательство закончено.
2.34 Теорема. Компактные подмножества метрических пространств замкнуты.
30
Доказательство. Пусть 𝐾 — компактное подмножество метрического пространства 𝑋. Мы докажем, что дополнение множества 𝐾 — открытое подмножество пространства 𝑋.
Предположим, что 𝑝 ∈ 𝑋, 𝑝 ∉ 𝐾. Если 𝑞 ∈ 𝐾, то пусть 𝑉𝑞 и 𝑊𝑞 — окрестности соответственно
точек 𝑝 и 𝑞 радиуса, меньшего 21 𝑑(𝑝, 𝑞) (см. определение 2.18(a)). Ввиду того что 𝐾 компактно,
найдется конечный набор точек 𝑞1 , … , 𝑞𝑛 , принадлежащих 𝐾, таких, что
𝐾 ⊂ 𝑊𝑞1 ∪ ⋯ ∪ 𝑊𝑞𝑛 = 𝑊 .
Если 𝑉 = 𝑉𝑞1 ∩ ⋯ ∩ 𝑉𝑞𝑛 , то 𝑉 — окрестность точки 𝑝, не пересекающаяся с 𝑊 . Значит, 𝑉 ⊂ 𝐾 𝑐 , так
что 𝑝 — внутренняя точка множества 𝐾 𝑐 . Теорема доказана.
2.35 Теорема. Замкнутые подмножества компактных множеств компактны.
Доказательство. Допустим, что 𝐹 ⊂ 𝐾 ⊂ 𝑋, 𝐹 замкнуто (относительно 𝑋), а 𝐾 компактно. Пусть
{𝑉𝛼 } — открытое покрытие множества 𝐹 . Если присоединить 𝐹 𝑐 к {𝑉𝛼 }, то мы получим открытое
покрытие Ω множества 𝐾. Поскольку 𝐾 компактно, существует конечное подсемейство Φ семейства
Ω, покрывающее 𝐾, а следовательно, и 𝐹 . Если 𝐹 𝑐 входит в Φ, то мы можем удалить его из Φ и
все равно получить открытое покрытие множества 𝐹 . Таким образом, мы показали, что конечное
подсемейство покрытия {𝑉𝛼 } покрывает 𝐹 .
Следствие. Если 𝐹 замкнуто, а 𝐾 компактно, то 𝐹 ∩ 𝐾 компактно.
Доказательство. Теоремы 2.24(b) и 2.34 показывают, что 𝐹 ∩ 𝐾 замкнуто, а так как 𝐹 ∩ 𝐾 ⊂ 𝐾,
то 𝐹 ∩ 𝐾 компактно по теореме 2.35.
2.36 Теорема. Если {𝐾𝛼 } — семейство компактных подмножеств метрического пространства
𝑋, такое, что пересечение любого конечного подсемейства семейства {𝐾𝛼 } непусто, то и ⋂ 𝐾𝛼
непусто.
Доказательство. Зафиксируем множество 𝐾1 из семейства {𝐾𝛼 } и положим 𝐺𝛼 =𝐾𝛼𝑐 . Предположим, что в 𝐾1 нет такой точки, которая принадлежала бы всем 𝐾𝛼 . Тогда множества 𝐺𝛼 образуют открытое покрытие множества 𝐾1 ; так как 𝐾1 компактно, найдется конечный набор индексов
𝛼1 , … , 𝛼𝑛 , такой, что 𝐾1 ⊂ 𝐺𝛼1 ∪ ⋯ ∪ 𝐺𝛼𝑛 . Но это означает, что
𝐾1 ∩ 𝐾𝛼1 ∩ ⋯ ∩ 𝐾𝛼𝑛
пусто, что противоречит условию теоремы.
Следствие. Если {𝐾𝑛 } — последовательность непустых компактных множеств, такая, что
𝐾𝑛 ⊃ 𝐾𝑛+1 (𝑛 = 1, 2, 3, … ), то ⋂∞
1 𝐾𝑛 непусто.
2.37 Теорема. Если 𝐸 — бесконечное подмножество компактного множества 𝐾, то 𝐸 имеет
предельную точку в 𝐾.
Доказательство. Если бы никакая точка множества 𝐾 не была предельной точкой множества 𝐸,
то каждая точка 𝑞 ∈ 𝐾 имела бы окрестность 𝑉𝑞 , содержащую не более одной точки множества
𝐸 (а именно 𝑞, если 𝑞 ∈ 𝐸). Ясно, что никакое конечное подсемейство семейства {𝑉𝑞 } не может
покрыть множество 𝐸; то же верно и для 𝐾, так как 𝐸 ⊂ 𝐾. Но это противоречит компактности
множества 𝐾.
2.38 Теорема. Если {𝐼𝑛 } — последовательность сегментов в 𝑅1 , такая, что 𝐼𝑛 ⊃ 𝐼𝑛+1 (𝑛 =
1, 2, 3, … ), то ⋂∞
1 𝐼𝑛 непусто.
Доказательство. Если 𝐼𝑛 = [𝑎𝑛 , 𝑏𝑛 ], то пусть 𝐸 есть множество всех 𝑎𝑛 . Тогда 𝐸 непусто и ограничено сверху (числом 𝑏1 ). Пусть 𝑥 = sup 𝐸. Если 𝑚 и 𝑛 — положительные целые числа, то
𝑎𝑛 ≤ 𝑎𝑚+𝑛 ≤ 𝑏𝑚+𝑛 ≤ 𝑏𝑚 ,
так что 𝑥 ≤ 𝑏𝑚 при каждом 𝑚. Поскольку очевидно, что 𝑎𝑚 ≤ 𝑥, то мы видим, что 𝑥 ∈ 𝐼𝑚 при
𝑚 = 1, 2, 3, … .
31
2.39 Теорема. Пусть 𝑘 — положительное целое число. Если {𝐼𝑛 } — последовательность 𝑘-мерных
клеток, такая, что 𝐼𝑛 ⊃ 𝐼𝑛+1 (𝑛 = 1, 2, 3, … ), то ⋂∞
1 𝐼𝑛 непусто.
Доказательство. Пусть 𝐼𝑛 состоит из всех точек x = (𝑥1 , … , 𝑥𝑘 ), таких, что
𝑎𝑛,𝑗 ≤ 𝑥𝑗 ≤ 𝑏𝑛,𝑗
(1 ≤ 𝑗 ≤ 𝑘; 𝑛 = 1, 2, 3, ⋯ ),
и пусть 𝐼𝑛,𝑗 = [𝑎𝑛,𝑗 , 𝑏𝑛,𝑗 ]. При каждом 𝑗 последовательность {𝐼𝑛,𝑗 } удовлетворяет предположениям
теоремы 2.38. Значит, существуют вещественные числа 𝑥∗𝑗 (1 ≤ 𝑗 ≤ 𝑘), такие, что
𝑎𝑛,𝑗 ≤ 𝑥∗𝑗 ≤ 𝑏𝑛,𝑗
(1 ≤ 𝑗 ≤ 𝑘; 𝑛 = 1, 2, 3, ⋯ ).
Полагая x∗ = (𝑥∗1 , ⋯ , 𝑥∗𝑘 ), мы видим, что x∗ ∈ 𝐼𝑛 при 𝑛 = 1, 2, 3, … . Теорема доказана.
2.40 Теорема. Любая 𝑘-мерная клетка компактна.
Доказательство. Пусть 𝐼 — 𝑘-мерная клетка, состоящая из всех точек x = (𝑥1 , … , 𝑥𝑘 ), таких, что
𝑎𝑗 ≤ 𝑥𝑗 ≤ 𝑏𝑗 (1 ≤ 𝑗 ≤ 𝑘). Положим
1/2
𝑘
⎧
⎫
⎪
⎪
𝛿 = ⎨ (𝑏𝑗 − 𝑎𝑗 )2 ⎬
∑
⎪
⎪
⎩ 1
⎭
.
Тогда |x − y| ≤ 𝛿, если x ∈ 𝐼, y ∈ 𝐼.
Чтобы получить противоречие, допустим, что существует открытое покрытие {𝐺𝛼 } множества
𝐼, не содержащее никакого конечного подпокрытия множества 𝐼. Положим 𝑐𝑗 = (𝑎𝑗 +𝑏𝑗 )/2. Сегменты
[𝑎𝑗 , 𝑐𝑗 ] и [𝑐𝑗 , 𝑏𝑗 ] определяют тогда 2𝑘 𝑘-мерных клеток 𝑄𝑖 , объединение которых есть 𝐼. Хотя бы одно
из этих множеств 𝑄𝑖 (обозначим его 𝐼1 ) не может быть покрыто никаким конечным подсемейством
семейства {𝐺𝛼 } (в противном случае так была бы покрыта вся клетка 𝐼). Теперь мы разобъем
𝐼1 и продолжим этот процесс. Мы получим последовательность {𝐼𝑛 }, обладающую следующими
свойствами:
(a) 𝐼 ⊃ 𝐼1 ⊃ 𝐼2 ⊃ 𝐼3 ⊃ ⋯ ;
(b) 𝐼𝑛 не покрывается никаким конечным подсемейством семейства {𝐺𝛼 };
(c) если x ∈ 𝐼𝑛 и y ∈ 𝐼𝑛 , то |x − y| ≤ 2−𝑛 𝛿.
Из (a) и теоремы 2.39 следует, что существует точка x∗ , принадлежащая всем 𝐼𝑛 . При некотором
𝛼 имеем x∗ ∈ 𝐺𝛼 . Поскольку 𝐺𝛼 открыто, существует 𝑟 > 0, такое, что из неравенства |y − x∗ | < 𝑟
следует y ∈ 𝐺𝛼 . Если 𝑛 столь велико, что 2−𝑛 𝛿 < 𝑟 (такое 𝑛 существует, ибо иначе 2𝑛 ≤ 𝛿/𝑟 для всех
положительных целых 𝑛, что невозможно, так как 𝑅 архимедово), то из (c) следует, что 𝐼𝑛 ⊂ 𝐺𝛼 , а
это противоречит (b).
Доказательство закончено.
Утверждение об эквивалентности свойств (a) и (b) в следующей теореме известно под названием
теоремы Гейне-Бореля.
2.41 Теорема. Если множество 𝐸 в 𝑅𝑘 обладает одним из трех следующих свойств, то оно
обладает и двумя другими:
(a) 𝐸 замкнуто и ограничено;
(b) 𝐸 компактно;
(c) каждое бесконечное подмножество множества 𝐸 имеет предельную точку в 𝐸.
Доказательство. Если (a) выполнено, то 𝐸 ⊂ 𝐼 для некоторой 𝑘-мерной клетки 𝐼, и (b) следует
из теорем 2.40 и 2.35. Теорема 2.37 показывает, что (b) влечет за собой (c). Остается доказать, что
из (c) следует (a).
Если 𝐸 не ограничено, то оно содержит точки x𝑛 , такие, что
|x𝑛 | > 𝑛
(𝑛 = 1, 2, 3, … ).
Множество 𝑆, состоящее из этих точек x𝑛 , бесконечно и, очевидно, не имеет предельных точек в
𝑅𝑘 и тем более в 𝐸. Таким образом, из (c) следует, что 𝐸 ограничено.
32
Если 𝐸 не замкнуто, то имеется точка x0 ∈ 𝑅𝑘 , являющаяся предельной точкой множества 𝐸,
но не принадлежащая 𝐸. При 𝑛 = 1, 2, 3, … имеются точки x𝑛 ∈ 𝐸, такие, что |x𝑛 − x0 | < 1/𝑛.
Пусть 𝑆 — множество этих точек x𝑛 . Тогда 𝑆 бесконечно (в противном случае |x𝑛 − x0 | имело бы
постоянное положительное значение для бесконечно многих 𝑛), x0 — предельная точка множества
𝑆 и 𝑆 не имеет других предельных точек в 𝑅𝑘 . Действительно, если y ∈ 𝑅𝑘 , y ≠ x0 , то
|x𝑛 − y| ≥ |x0 − y| − |x𝑛 − x0 |
1 1
≥ |x0 − y| − ≥ |x0 − y|
𝑛 2
для всех 𝑛, кроме конечного числа; это показывает, что y не является предельной точкой множества
𝑆 (теорема 2.20).
Таким образом, 𝑆 не имеет предельных точек в 𝐸; значит, 𝐸 замкнуто, если выполнено (c).
Следует отметить в связи с этой теоремой, что (b) и (c) эквивалентны в любом метрическом
пространстве (упражнение 26), но что из (a) в общем случае не следуют (b) и (c). Пример приведен
в упражнении 16, также примером является пространство ℒ 2 , которое обсуждается в гл. 11.
2.42 Теорема (Вейерштрасс). Всякое ограниченное бесконечное подмножество пространства 𝑅𝑘
имеет предельную точку в 𝑅𝑘 .
Доказательство. Будучи ограниченным, множество 𝐸, о котором идет речь, является подмножеством некоторой 𝑘-мерной клетки 𝐼 ⊂ 𝑅𝑘 . По теореме 2.40 𝐼 компактно, поэтому, согласно теореме
2.37, 𝐸 имеет в 𝐼 предельную точку.
Совершенные множества
2.43 Теорема. Пусть 𝑃 — непустое совершенное множество в 𝑅𝑘 . Тогда 𝑃 несчетно.
Доказательство. Поскольку 𝑃 имеет предельные точки, оно должно быть бесконечным. Допустим, что 𝑃 счетно, и обозначим его точки через x1 , x2 , x3 , … . Мы построим последовательность
окрестностей {𝑉𝑛 } следующим образом.
Пусть 𝑉1 — какая-нибудь окрестность точки x1 . Если 𝑉1 состоит из всех y ∈ 𝑅𝑘 , таких, что
|y − x1 | < 𝑟, то ее замыкание 𝑉1̄ — множество всех y ∈ 𝑅𝑘 , таких, что |y − x1 | ≤ 𝑟.
Допустим, что окрестность 𝑉𝑛 построена так, что 𝑉𝑛 ∩ 𝑃 непусто. Поскольку каждая точка
̄ ⊂ 𝑉𝑛 ,
множества 𝑃 является его предельной точкой, существует окрестность 𝑉𝑛+1 , такая, что (i) 𝑉𝑛+1
̄
(ii) x𝑛 ∉ 𝑉𝑛+1 , (iii) 𝑉𝑛+1 ∩ 𝑃 непусто. В силу (iii), 𝑉𝑛+1 удовлетворяет предположению индукции, и
построение может быть продолжено.
Положим 𝐾𝑛 = 𝑉𝑛̄ ∩ 𝑃 . Будучи замкнутым и ограниченным, 𝑉𝑛̄ компактно. Так как x𝑛 ∉ 𝐾𝑛+1 ,
∞
ни одна точка 𝑃 не лежит в ⋂∞
1 𝐾𝑛 . Так как 𝐾𝑛 ⊂ 𝑃 , отсюда следует, что ⋂1 𝐾𝑛 пусто. Но каждое
множество 𝐾𝑛 непусто в силу (iii), и 𝐾𝑛 ⊃ 𝐾𝑛+1 в силу (i); это противоречит следствию из теоремы
2.36.
Следствие. Каждый сегмент [𝑎, 𝑏] (𝑎 < 𝑏) несчетен. В частности, множество всех вещественных чисел несчетно.
2.44 Канторово множество. Множество, к построению которого мы переходим, показывает, что
в 𝑅1 существуют совершенные множества, не содержащие никакого интервала.
Пусть 𝐸0 — сегмент [0, 1]. Удалим интервал ( 31 , 23 ), и пусть 𝐸1 — объединение сегментов
1
2
[0, 3 ] , [ 3 , 1] .
Удалим средние трети этих сегментов, и пусть 𝐸2 — объединение сегментов
1
2 3
6 7
8
[0, 9 ] , [ 9 , 9 ] , [ 9 , 9 ] , [ 9 , 1] .
Продолжая таким образом, мы получим последовательность компактных множеств 𝐸 𝑛 , таких, что
33
(a) 𝐸1 ⊃ 𝐸2 ⊃ 𝐸3 ⊃ ⋯ ;
(b) 𝐸𝑛 есть объединение 2𝑛 сегментов, длина каждого из которых равна 3−𝑛 .
Множество
∞
𝑃 =
⋂
𝐸𝑛
𝑛=1
называется множеством Кантора. Очевидно, что 𝑃 компактно, и теорема 2.36 показывает, что 𝑃
непусто.
Никакой интервал вида
(24)
3𝑘 + 1 3𝑘 + 2
( 3𝑚 , 3𝑚 ) ,
где 𝑘 и 𝑚 — положительные целые числа, не имеет общих точек с 𝑃 . Значит, 𝑃 не содержит никакого
интервала, ибо каждый интервал (𝛼, 𝛽) содержит интервал вида (24), если
3−𝑚 <
𝛽−𝛼
.
6
Чтобы показать, что 𝑃 совершенно, достаточно показать, что 𝑃 не содержит изолированных
точек. Пусть 𝑥 ∈ 𝑃 и пусть 𝑆 — какой-нибудь интервал, содержащий 𝑥. Пусть 𝐼𝑛 — сегмент множества 𝐸𝑛 , содержащий 𝑥. Выберем достаточно большое 𝑛, чтобы 𝐼𝑛 ⊂ 𝑆. Пусть 𝑥𝑛 — тот конец
сегмента 𝐼𝑛 , для которого 𝑥𝑛 ≠ 𝑥.
Из построения множества 𝑃 следует, что 𝑥𝑛 ∈ 𝑃 . Значит, 𝑥 есть предельная точка множества
𝑃 , и 𝑃 совершенно.
Одно из самых интересных свойств множества Кантора состоит в том, что оно доставляет нам
пример несчетного множества меры нуль (понятие меры будет обсуждаться а гл. 11).
Связные множества
2.45 Определение. Два подмножества 𝐴 и 𝐵 метрического пространства 𝑋 называются отделенными друг от друга, если как 𝐴 ∩ 𝐵,̄ так и 𝐴̄ ∩ 𝐵 пусты, т. е. ни одна точка 𝐴 не лежит в замыкании
множества 𝐵 и ни одна точка 𝐵 не лежит в замыкании множества 𝐴.
Множество 𝐸 ⊂ 𝑋 называется связным, если 𝐸 не является объединением двух непустых отделенных друг от друга множеств.
2.46 Замечание. Отделенные друг от друга множества, конечно, не пересекаются, но непересекающиеся множества не обязаны быть отделенными друг от друга. Например, сегмент [0, 1] и
интервал (1, 2) не являются отделенными друг от друга, так как 1 — предельная точка интервала
(1, 2). Однако интервалы (0, 1) и (1, 2) являются отделенными друг от друга.
Связные подмножества прямой имеют особенно простую структуру.
2.47 Теорема. Подмножество 𝐸 вещественной прямой 𝑅1 связно тогда и только тогда, когда
оно обладает следующим свойством: если 𝑥 ∈ 𝐸, 𝑦 ∈ 𝐸 и 𝑥 < 𝑧 < 𝑦, то 𝑧 ∈ 𝐸.
Доказательство. Если существуют 𝑥 ∈ 𝐸, 𝑦 ∈ 𝐸 и некоторое 𝑧 ∈ (𝑥, 𝑦), такое, что 𝑧 ∉ 𝐸, то
𝐸 = 𝐴𝑧 ∪ 𝐵𝑧 , где
𝐴𝑧 = 𝐸 ∩ (−∞, 𝑧),
𝐵𝑧 = 𝐸 ∩ (𝑧, ∞).
Так как 𝑥 ∈ 𝐴𝑧 и 𝑦 ∈ 𝐵𝑧 , 𝐴 и 𝐵 непусты. Так как 𝐴𝑧 ⊂ (−∞, 𝑧) и 𝐵𝑧 ⊂ (𝑧, ∞), они отделены друг от
друга. Следовательно, 𝐸 не связно.
Чтобы доказать обратное, допустим, что 𝐸 не связно. Тогда существуют непустые отделенные
друг от друга множества 𝐴 и 𝐵, такие, что 𝐴 ∪ 𝐵 = 𝐸. Выберем 𝑥 ∈ 𝐴, 𝑦 ∈ 𝐵, и предположим (без
потери общности), что 𝑥 < 𝑦. Обозначим
𝑧 = sup(𝐴 ∩ [𝑥, 𝑦]).
По теореме 2.28 𝑧 ∈ 𝐴,̄ следовательно, 𝑧 ∉ 𝐵. В частности, 𝑥 ≤ 𝑧 < 𝑦.
Если 𝑧 ∉ 𝐴, то 𝑥 < 𝑧 < 𝑦 и 𝑧 ∉ 𝐸.
Если 𝑧 ∈ 𝐴, то 𝑧 ∉ 𝐵,̄ следовательно, существует 𝑧1 , такое, что 𝑧 < 𝑧1 < 𝑦 и 𝑧1 ∉ 𝐵. Тогда
𝑥 < 𝑧1 < 𝑦 и 𝑧1 ∉ 𝐸.
34
Упражнения
1. Доказать, что пустое множество является подмножеством любого множества.
2. Комплексное число 𝑧 называет алгебраическим, если существуют целые числа 𝑎0 , … , 𝑎𝑛 , не равные одновременно нулю, для которых
𝑎0 𝑧𝑛 + 𝑎1 𝑧𝑛−1 + ⋯ + 𝑎𝑛−1 𝑧 + 𝑎𝑛 = 0.
Доказать, что множество всех алгебраических чисел счетно. Указание. При каждом положительном
целом 𝑁 существует только конечное число таких уравнений, что
𝑛 + |𝑎0 | + |𝑎1 | + ⋯ + |𝑎𝑛 | = 𝑁.
3.
4.
5.
6.
Доказать, что существуют вещественнные числа, не являющиеся алгебраическими.
Является ли множество всех иррациональных вещественных чисел счетным?
Построить ограниченное множество вещественных чисел, имеющее ровно три предельные точки.
Пусть 𝐸 ′ — множество всех предельных точек некоторого множества 𝐸. Доказать, что 𝐸 ′ замкнуто.
Доказать, что 𝐸 и 𝐸̄ имеют одни и те же предельные точки (напомним, что 𝐸̄ = 𝐸 ∪ 𝐸 ′ ). Всегда ли 𝐸 и
𝐸 ′ имеют одни и те же предельные точки?
7. Пусть 𝐴1 , 𝐴2 , 𝐴3 , … — подмножества метрического пространства.
𝑛
𝑛
(a) Если 𝐵𝑛 = ⋃𝑖=1 𝐴𝑖 , доказать, что 𝐵𝑛̄ = ⋃𝑖=1 𝐴𝑖̄ при 𝑛 = 1, 2, 3, … .
∞
∞
̄
(b) Если 𝐵 = ⋃𝑖=1 𝐴𝑖 , доказать, что 𝐵 ⊃ ⋃𝑖=1 𝐴𝑖̄ .
Показать на примере, что это включение может быть собственным.
8. Каждая ли точка произвольного открытого множества 𝐸 ⊂ 𝑅2 является предельной точкой 𝐸? Ответить
на тот же вопрос для замкнутых множеств в 𝑅2 .
9. Обозначим 𝐸 ∘ множество всех внутренних точек множества 𝐸 (см. определение 2.18(e); 𝐸 ∘ называется
внутренностью 𝐸).
(a)
(b)
(c)
(d)
(e)
(f)
Доказать, что 𝐸 ∘ всегда открыто.
Доказать, что 𝐸 открыто тогда и только тогда, когда 𝐸 ∘ = 𝐸.
Если 𝐺 ⊂ 𝐸 и 𝐺 открыто, доказать, что 𝐺 ⊂ 𝐸 ∘ .
Доказать, что дополнение множества 𝐸 ∘ — это замыкание дополнения множества 𝐸.
Всегда ли 𝐸 и 𝐸̄ имеют равные внутренности?
Всегда ли 𝐸 и 𝐸 ∘ имеют равные замыкания?
10. Пусть 𝑋 — бесконечное множество. Для 𝑝 ∈ 𝑋 и 𝑞 ∈ 𝑋 определим
𝑑(𝑝, 𝑞) =
1 (если 𝑝 ≠ 𝑞),
{0 (если 𝑝 = 𝑞).
Доказать, что это метрика. Какие подмножества получающегося метрического пространства открыты?
Замкнуты? Компактны?
11. Для 𝑥 ∈ 𝑅1 и 𝑦 ∈ 𝑅1 обозначим
𝑑1 (𝑥, 𝑦) = (𝑥 − 𝑦)2 ,
𝑑2 (𝑥, 𝑦) = √|𝑥 − 𝑦|,
𝑑3 (𝑥, 𝑦) = |𝑥2 − 𝑦2 |,
𝑑4 (𝑥, 𝑦) = |𝑥 − 2𝑦|,
𝑑5 (𝑥, 𝑦) =
|𝑥 − 𝑦|
.
1 + |𝑥 − 𝑦|
Определите для каждой из этих функцией, является ли она метрикой.
12. Пусть 𝐾 ⊂ 𝑅1 состоит из 0 и чисел 1/𝑛 при 𝑛 = 1, 2, 3, … . Доказать, что 𝐾 компактно, напрямую из
определения (не используя теорему Гейне—Бореля).
13. Построить компактное множество вещественных чисел, множество предельных точек которого счетно.
14. Дать пример открытого покрытия интервала (0, 1), которое не содержит конечного подпокрытия.
15. Показать, что теорема 2.36 и ее следствие становятся неверными (например, в 𝑅1 ), если слово «компактное» заменить словом «замкнутое» или «ограниченное».
16. Рассмотрим множество всех рациональных чисел 𝑄 как метрическое пространство с метрикой 𝑑(𝑝, 𝑞) =
|𝑝 − 𝑞|. Пусть 𝐸 — множество всех 𝑝 ∈ 𝑄, таких, что 2 < 𝑝2 < 3. Показать, что 𝐸 замкнуто и ограничено
в 𝑄, но не компактно. Является ли 𝐸 открытым в 𝑄?
17. Пусть 𝐸 — множество всех 𝑥 ∈ [0, 1], десятичное разложение которых содержит только цифры 4 и 7.
Является ли 𝐸 счетным? Всюду плотным в [0, 1]? Компактным? Совершенным?
35
18. Существует ли непустое совершенное подмножество 𝑅1 , не содержащее ни одного рационального числа?
19. (a) Если 𝐴 и 𝐵 — непересекающиеся замкнутые множества в некотором метрическом пространстве 𝑋,
доказать, что они отделены друг от друга.
(b) Доказать то же самое для непересекающихся открытых множеств.
(c) Зафиксируем 𝑝 ∈ 𝑋, 𝛿 > 0, определим 𝐴 как множество всех 𝑞 ∈ 𝑋, для которых 𝑑(𝑝, 𝑞) < 𝛿,
определим 𝐵 подобным образом, но со знаком > вместо <. Доказать, что 𝐴 и 𝐵 отделены друг от
друга.
(d) Доказать, что любое связное метрическое пространство, в котором есть хотя бы две точки, несчетно. Указание. Используйте (c).
20. Всегда ли замыкание и внутренность связного множества связны (посмотреть на подмножества 𝑅2 )?
21. Пусть 𝐴 и 𝐵 — отделенные друг от друга подмножества некоторого 𝑅𝑘 , возьмем a ∈ 𝐴, b ∈ 𝐵 и определим
p(𝑡) = (1 − 𝑡)a + 𝑡b
для 𝑡 ∈ 𝑅1 . Положим 𝐴0 = p−1 (𝐴), 𝐵0 = p−1 (𝐵) (тем самым 𝑡 ∈ 𝐴0 тогда и только тогда, когда p(𝑡) ∈ 𝐴).
(a) Доказать, что 𝐴0 и 𝐵0 являются отделенными друг от друга подмножествами прямой 𝑅1 .
(b) Доказать, что существует 𝑡0 ∈ (0, 1), такое, что p(𝑡0 ) ∉ 𝐴 ∪ 𝐵.
(c) Доказать, что каждое выпуклое множество пространства 𝑅𝑘 связно.
22. Метрическое пространство называется сепарабельным, если оно содержит счетное всюду плотное подмножество. Показать, что 𝑅𝑘 сепарабельно. Указание. Рассмотрите множество точек, все координаты
которых рациональны.
23. Семейство {𝑉𝛼 } открытых подмножеств пространства 𝑋 называется базой пространства 𝑋, если верно
следующее: для каждого 𝑥 ∈ 𝑋 и каждого открытого множества 𝐺 ⊂ 𝑋, такого, что 𝑥 ∈ 𝐺, имеем
𝑥 ∈ 𝑉𝛼 ⊂ 𝐺 при некотором 𝛼. Иными словами, каждое открытое множество в 𝑋 есть объединение
некоторого подсемейства семейства {𝑉𝛼 }.
Доказать, что каждое сепарабельное метрическое пространство имеет счетную базу. Указание. Возьмите все окрестности с рациональными радиусами и центрами в некотором счетном всюду плотном
подмножестве пространства 𝑋.
24. Пусть 𝑋 — метрическое пространство, в котором каждое бесконечное подмножество имеет предельную
точку. Доказать, что 𝑋 сепарабельно. Указание. Зафиксируйте 𝛿 > 0 и выберите 𝑥1 ∈ 𝑋. Выбрав
𝑥1 , … , 𝑥𝑗 ∈ 𝑋, выберите, если это возможно, 𝑥𝑗+1 ∈ 𝑋, такое, что 𝑑(𝑥𝑖 , 𝑥𝑗+1 ) ≥ 𝛿 для 𝑖 = 1, … , 𝑗. Покажите,
что этот процесс должен закончиться после конечного числа шагов и что поэтому 𝑋 можно покрыть
конечным числом окрестностей радиуса 𝛿. Возьмите 𝛿 = 1/𝑛 (𝑛 = 1, 2, 3, … ) и рассмотрите центры
соответствующих окружностей.
25. Доказать, что любое компактное метрическое пространство 𝐾 имеет счетную базу и потому сепарабельно. Указание. Для каждого положительного целого 𝑛 существует конечное число окрестностей радиуса
1/𝑛, объединение которых покрывает 𝐾.
26. Пусть 𝑋 — метрическое пространство, в котором каждое бесконечное подмножество имеет предельную
точку. Доказать, что 𝑋 компактно. Указание. Согласно упражнениям 23 и 24, 𝑋 имеет счетную базу. Следовательно, каждое открытое покрытие пространства 𝑋 содержит счетное подпокрытие {𝐺𝑛 },
𝑛 = 1, 2, 3, … . Если никакое конечное подсемейство семейства {𝐺𝑛 } не покрывает 𝑋, то дополнение 𝐹𝑛
множества 𝐺1 ∪ 𝐺2 ∪ ⋯ ∪ 𝐺𝑛 непусто при каждом 𝑛, но ⋂ 𝐹𝑛 пусто. Пусть 𝐸 — множество, содержащее
по точке из каждого 𝐹𝑛 . Рассмотрите предельную точку множества 𝐸 и получите противоречие.
27. Назовем точку 𝑝 в метрическом пространстве 𝑋 точкой конденсации множества 𝐸 ⊂ 𝑋, если каждая
окрестность точки 𝑝 содержит несчетное число точек множества 𝐸.
Пусть 𝐸 ⊂ 𝑅𝑘 , 𝐸 несчетно, и 𝑃 — множество всех точек конденсации множества 𝐸. Доказать, что 𝑃
совершенно и что не более чем счетное число точек множества 𝐸 не содержится в 𝑃 . Другими словами,
показать, что 𝑃 𝑐 ∩ 𝐸 не более чем счетно. Указание. Пусть {𝑉𝑛 } — счетная база пространства 𝑅𝑘 , 𝑊 —
объединение тех 𝑉𝑛 , для которых 𝐸 ∩ 𝑉𝑛 не более чем счетно. Покажите, что 𝑃 = 𝑊 𝑐 .
28. Доказать, что каждое замкнутое множество в сепарабельном метрическом пространстве есть объединение совершенного множества (может быть, пустого) и некоторого не более чем счетного множества.
(Следствие: каждое счетное замкнутое множество в 𝑅𝑘 имеет изолированные точки.) Указание. Используйте упражнение 27.
29. Доказать, что каждое открытое множество в 𝑅1 есть объединение не более чем счетного семейства
попарно непересекающихся интервалов. Указание. Используйте упражнение 22.
30. Следуя доказательству теоремы 2.43, получить такой результат:
∞
Если 𝑅𝑘 = ⋃1 𝐹𝑛 , где каждое 𝐹𝑛 — замкнутое подмножество пространства 𝑅𝑘 , то хотя бы одно 𝐹𝑛 имеет
непустую внутренность.
Эквивалентное утверждение: Если 𝐺𝑛 — всюду плотное открытое подмножество пространства 𝑅𝑘 при
∞
𝑛 = 1, 2, 3, … , то ⋂1 𝐺𝑛 непусто (на самом деле оно всюду плотно в 𝑅𝑘 ).
(Это частный случай теоремы Бэра; см. общий случай в упражнении 22, гл. 3.)
36
Глава 3
Числовые последовательности и
ряды
Как указывает название, в этой главе мы будем иметь дело главным образом с последовательностями и рядами комплексных чисел. Однако основные факты, связанные со сходимостью, столь
же легко объяснить и в более общей ситуации. Первые три раздела будут поэтому посвящены
последовательностям в евклидовых пространствах или даже в метрических пространствах.
Сходящиеся последовательности
3.1 Определение. Последовательность {𝑝𝑛 } в метрическом пространстве 𝑋 называется сходящейся, если существует точка 𝑝 ∈ 𝑋, обладающая следующим свойством: для каждого 𝜀 > 0 существует
целое число 𝑁, такое, что при 𝑛 ≥ 𝑁 имеем 𝑑(𝑝𝑛 , 𝑝) < 𝜀 (здесь 𝑑 обозначает расстояние в 𝑋).
В этом случае мы будем говорить также, что последовательность {𝑝𝑛 } сходится к 𝑝 или что 𝑝 —
предел последовательности {𝑝𝑛 } (см. теорему 3.2(b)), и будем писать 𝑝𝑛 → 𝑝 или
lim 𝑝
𝑛→∞ 𝑛
= 𝑝.
Если последовательность {𝑝𝑛 } не сходится, то говорят, что она расходится.
Полезно отметить, что наше определение «сходящейся последовательности» зависит не только
от {𝑝𝑛 }, но и от 𝑋; например, последовательность {1/𝑛} сходится в 𝑅1 (к 0), но не сходится во
множестве всех положительных вещественных чисел (когда 𝑑(𝑥, 𝑦) = |𝑥 − 𝑦|). В тех случаях, когда
возможна путаница, мы будем более точными и будем говорить отчетливо «сходится в 𝑋» вместо
«сходится».
Напомним, что множество всех точек 𝑝𝑛 (𝑛 = 1, 2, 3, … ) есть множество значений последовательности {𝑝𝑛 }. Множество значений последовательности может быть конечным или бесконечным.
Последовательность называется ограниченной, если множество ее значений ограничено.
Для примера рассмотрим следующие последовательности комплексных чисел (при этом 𝑋 =
𝑅2 ):
(a) Если 𝑠𝑛 = 1/𝑛, то lim𝑛→∞ 𝑠𝑛 = 0; множество значений бесконечно, а последовательность ограничена.
(b) Если 𝑠𝑛 = 𝑛2 , то последовательность {𝑠𝑛 } не ограничена, расходится, а множество ее значений
бесконечно.
(c) Если 𝑠𝑛 = 1 + [(−1)𝑛 /𝑛], то последовательность {𝑠𝑛 } сходится к 1, ограничена, а множество ее
значений бесконечно.
(d) Если 𝑠𝑛 = 𝑖𝑛 , то последовательность {𝑠𝑛 } расходится, ограничена, а множество ее значений
конечно.
(e) Если 𝑠𝑛 = 1 (𝑛 = 1, 2, 3, … ), то {𝑠𝑛 } сходится к 1, ограничена, а множество ее значений конечно.
Сформулируем теперь некоторые важные свойства сходящихся последовательностей в метрических пространствах.
37
3.2 Теорема. Пусть {𝑝𝑛 } — последовательность в метрическом пространстве 𝑋.
(a) {𝑝𝑛 } сходится к 𝑝 ∈ 𝑋 тогда и только тогда, когда каждая окрестность точки 𝑝 содержит
𝑝𝑛 для всех 𝑛, за исключением конечного числа.
(b) Если 𝑝 ∈ 𝑋, 𝑝′ ∈ 𝑋 и {𝑝𝑛 } сходится к 𝑝 и к 𝑝′ , то 𝑝 = 𝑝′ .
(c) Если {𝑝𝑛 } сходится, то {𝑝𝑛 } ограничена.
(d) Если 𝐸 ⊂ 𝑋 и если 𝑝 — предельная точка множества 𝐸, то существует последовательность {𝑝𝑛 } в 𝐸, такая, что 𝑝 = lim 𝑝𝑛 .
𝑛→∞
Доказательство.
(a) Допустим, что 𝑝𝑛 → 𝑝, и пусть 𝑉 — окрестность точки 𝑝. Для некоторого 𝜀 > 0 из условий
𝑑(𝑝, 𝑞) < 𝜀, 𝑞 ∈ 𝑋 следует, что 𝑞 ∈ 𝑉 . Этому 𝜀 соответствует 𝑁, такое, что из 𝑛 ≥ 𝑁 следует
𝑑(𝑝𝑛 , 𝑝) < 𝜀. Таким образом, 𝑛 ≥ 𝑁 влечет за собой 𝑝𝑛 ∈ 𝑉 .
Обратно, допустим, что каждая окрестность точки 𝑝 содержит все 𝑝𝑛 , кроме конечного их
числа. Зафиксируем 𝜀 > 0, и пусть 𝑉 — множество всех 𝑞 ∈ 𝑋, таких, что 𝑑(𝑝, 𝑞) < 𝜀. По
предположению существует 𝑁 (соответствующее этому 𝑉 ), такое, что 𝑝𝑛 ∈ 𝑉 , если 𝑛 ≥ 𝑁.
Таким образом, 𝑑(𝑝𝑛 , 𝑝) < 𝜀, если 𝑛 ≥ 𝑁; значит, 𝑝𝑛 → 𝑝.
(b) Пусть задано 𝜀 > 0. Существуют целые числа 𝑁, 𝑁 ′ , такие, что
𝜀
при 𝑛 ≥ 𝑁 имеем 𝑑(𝑝𝑛 , 𝑝) < ,
2
𝜀
при 𝑛 ≥ 𝑁 ′ имеем 𝑑(𝑝𝑛 , 𝑝′ ) < .
2
Значит, если 𝑛 ≥ max(𝑁, 𝑁 ′ ), то
𝑑(𝑝, 𝑝′ ) ≤ 𝑑(𝑝, 𝑝𝑛 ) + 𝑑(𝑝𝑛 , 𝑝′ ) < 𝜀.
Поскольку 𝜀 было произвольным, мы заключаем, что 𝑑(𝑝, 𝑝′ ) = 0.
(c) Допустим, что 𝑝𝑛 → 𝑝. Существует целое 𝑁, такое, что при 𝑛 > 𝑁 имеем 𝑑(𝑝𝑛 , 𝑝) < 1. Положим
𝑟 = max{1, 𝑑(𝑝1 , 𝑝), … , 𝑑(𝑝𝑁 , 𝑝)}.
Тогда 𝑑(𝑝𝑛 , 𝑝) ≤ 𝑟 при 𝑛 = 1, 2, 3, … .
(d) Для каждого положительного целого 𝑛 существует точка 𝑝𝑛 ∈ 𝐸 такая, чо 𝑑(𝑝𝑛 , 𝑝) < 1/𝑛. Для
данного 𝜀 > 0 выберем 𝑁 так, что 𝑁𝜀 > 1. Если 𝑛 > 𝑁, то 𝑑(𝑝𝑛 , 𝑝) < 𝜀. Значит, 𝑝𝑛 → 𝑝.
Доказательство закончено.
Для последовательностей в 𝑅𝑘 мы можем изучать соотношения между сходимостью, с одной
стороны, и алгебраическими операциями, с другой. Сначала мы рассмотрим последовательности
комплексных чисел.
3.3 Теорема. Допустим, что {𝑠𝑛 }, {𝑡𝑛 } — последовательности комплексных чисел и lim𝑛→∞ 𝑠𝑛 =
𝑠, lim𝑛→∞ 𝑡𝑛 = 𝑡. Тогда
(a) lim (𝑠𝑛 + 𝑡𝑛 ) = 𝑠 + 𝑡;
𝑛→∞
(b) lim 𝑐𝑠𝑛 = 𝑐𝑠, lim (𝑐 + 𝑠𝑛 ) = 𝑐 + 𝑠 для любого числа 𝑐;
𝑛→∞
𝑛→∞
(c) lim 𝑠𝑛 𝑡𝑛 = 𝑠𝑡;
𝑛→∞
(d) lim
1
𝑛→∞ 𝑠𝑛
=
1
, если только 𝑠𝑛 ≠ 0 (𝑛 = 1, 2, 3, … ) и 𝑠 ≠ 0.
𝑠
Доказательство.
(a) Для данного 𝜀 > 0 существуют целые 𝑁1 , 𝑁2 , такие, что
𝜀
,
2
𝜀
при 𝑛 ≥ 𝑁2 имеем |𝑡𝑛 − 𝑡| < .
2
Если 𝑁 = max(𝑁1 , 𝑁2 ), то при 𝑛 ≥ 𝑁 получим
при 𝑛 ≥ 𝑁1 имеем |𝑠𝑛 − 𝑠| <
|(𝑠𝑛 + 𝑡𝑛 ) − (𝑠 + 𝑡)| ≤ |𝑠𝑛 − 𝑠| + |𝑡𝑛 − 𝑡| < 𝜀.
Тем самым (a) доказано. Доказательство (b) тривиально.
38
(c) Воспользуемся тождеством
(1)
𝑠𝑛 𝑡𝑛 − 𝑠𝑡 = (𝑠𝑛 − 𝑠)(𝑡𝑛 − 𝑡) + 𝑠(𝑡𝑛 − 𝑡) + 𝑡(𝑠𝑛 − 𝑠).
Для данного 𝜀 > 0 существуют целые 𝑁1 , 𝑁2 , такие, что
при 𝑛 ≥ 𝑁1 имеем |𝑠𝑛 − 𝑠| < √𝜀,
при 𝑛 ≥ 𝑁2 имеем |𝑡𝑛 − 𝑡| < √𝜀.
Если мы возьмем 𝑁 = max(𝑁1 , 𝑁2 ), то при 𝑛 ≥ 𝑁 получим
|(𝑠𝑛 − 𝑠)(𝑡𝑛 − 𝑡)| < 𝜀,
откуда
lim (𝑠𝑛 − 𝑠)(𝑡𝑛 − 𝑡) = 0.
𝑛→∞
Применив теперь (a) и (b) к тождеству (1), мы заключаем, что
lim (𝑠𝑛 𝑡𝑛 − 𝑠𝑡) = 0.
𝑛→∞
(d) Выбрав 𝑚 так, что |𝑠𝑛 − 𝑠| < 12 |𝑠| при 𝑛 ≥ 𝑚, мы видим, что
|𝑠𝑛 | > 12 |𝑠|
(𝑛 ≥ 𝑚).
Для данного 𝜀 > 0 существует целое 𝑁 > 𝑚, такое, что при 𝑛 ≥ 𝑁 имеем
|𝑠𝑛 − 𝑠| < 21 |𝑠|2 𝜀.
Отсюда при 𝑛 ≥ 𝑁 получаем
𝑠 −𝑠
1
1
2
−
= 𝑛
<
|𝑠 − 𝑠| < 𝜀.
| 𝑠𝑛 𝑠 | | 𝑠𝑛 𝑠 | |𝑠|2 𝑛
3.4 Теорема.
(a) Допустим, что x𝑛 ∈ 𝑅𝑘 (𝑛 = 1, 2, 3, … ) и
x𝑛 = (𝛼1,𝑛 , … , 𝛼𝑘,𝑛 ).
Тогда {x𝑛 } сходится к x = (𝛼1 , … , 𝛼𝑘 ) тогда и только тогда, когда
(2)
lim 𝛼
𝑛→∞ 𝑗,𝑛
= 𝛼𝑗
(1 ≤ 𝑗 ≤ 𝑘).
(b) Допустим, что {x𝑛 }, {y𝑛 } — последовательности в 𝑅𝑘 , {𝛽𝑛 } — последовательность вещественных чисел и x𝑛 → x, y𝑛 → y, 𝛽𝑛 → 𝛽. Тогда
lim (x𝑛 + y𝑛 ) = x + y,
lim x𝑛 ⋅ y𝑛 = x ⋅ y,
𝑛→∞
lim 𝛽 x
𝑛→∞ 𝑛 𝑛
𝑛→∞
= 𝛽x.
Доказательство.
(a) Если x𝑛 → x, то неравенства
|𝛼𝑗,𝑛 − 𝛼𝑗 | ≤ |x𝑛 − x|,
вытекающие непосредственно из определения нормы в 𝑅𝑘 , показывают, что (2) выполняется.
Обратно, если (2) выполнено, то каждому 𝜀 > 0 соответствует целое 𝑁, такое, что при 𝑛 ≥ 𝑁
имеем
𝜀
|𝛼𝑗,𝑛 − 𝛼𝑗 | <
(1 ≤ 𝑗 ≤ 𝑘).
√𝑘
Значит, при 𝑛 ≥ 𝑁 получаем
1/2
𝑘
⎧
⎫
⎪
⎪
|x𝑛 − x| = ⎨ |𝛼𝑗,𝑛 − 𝛼𝑗 |2 ⎬
∑
⎪
⎪
⎩ 𝑗=1
⎭
откуда x𝑛 → x. Тем самым (a) доказано.
Часть (b) следует из (a) и теоремы 3.3.
39
< 𝜀,
Подпоследовательности
3.5 Определение. Пусть задана последовательность {𝑝𝑛 }. Рассмотрим последовательность {𝑛𝑘 }
положительных целых чисел, такую, что 𝑛1 < 𝑛2 < 𝑛3 < ⋯ . Тогда последовательность {𝑝𝑛𝑖 } называется подпоследовательностью последовательности {𝑝𝑛 }. Если последовательность {𝑝𝑛𝑖 } сходится,
то ее предел называется частичным пределом последовательности {𝑝𝑛 }.
Ясно, что последовательность {𝑝𝑛 } сходится к 𝑝 тогда и только тогда, когда всякая ее подпоследовательность сходится к 𝑝. Мы предоставляем читателю провести детальное доказательство.
3.6 Теорема.
(a) Если {𝑝𝑛 } — последовательность в компактном метрическом пространстве 𝑋, то она содержит подпоследовательность, сходящуюся к точке пространства 𝑋.
(b) Всякая ограниченная последовательность в 𝑅𝑘 содержит сходящуюся подпоследовательность.
Доказательство.
(a) Пусть 𝐸 — множество значений последовательности {𝑝𝑛 }. Если 𝐸 конечно, то существуют
𝑝 ∈ 𝐸 и последовательность {𝑛𝑖 } (𝑛1 < 𝑛2 < 𝑛3 < ⋯ ), такие, что
𝑝𝑛1 = 𝑝𝑛2 = ⋯ = 𝑝.
Полученная таким образом подпоследовательность {𝑝𝑛𝑖 }, очевидно, сходится к 𝑝.
Если 𝐸 бесконечно, то по теореме 2.37 оно имеет предельную точку 𝑝 ∈ 𝑋. Выберем 𝑛1 так, что
𝑑(𝑝, 𝑝𝑛1 ) < 1. Выбрав 𝑛1 , … , 𝑛𝑖−1 , мы получим по теореме 2.20, что существует целое 𝑛𝑖 > 𝑛𝑖−1 ,
такое, что 𝑑(𝑝, 𝑝𝑛𝑖 ) < 1/𝑖. Тогда {𝑝𝑛𝑖 } сходится к 𝑝.
(b) Это следует из (a), так как по теореме 2.41 каждое ограниченное подмножество пространства
𝑅𝑘 лежит в компактном подмножестве пространства 𝑅𝑘 .
3.7 Теорема. Частичные пределы последовательности {𝑝𝑛 } в метрическом пространстве 𝑋
образуют замкнутое подмножество пространства 𝑋.
Доказательство. Пусть 𝐸 ∗ — множество всех частичных пределов последовательности {𝑝𝑛 }, а
𝑞 — предельная точка множества 𝐸 ∗ . Мы должны показать, что 𝑞 ∈ 𝐸 ∗ .
Выберем 𝑛1 так, чтобы 𝑝𝑛1 ≠ 𝑞 (если такого 𝑛1 не существует, то 𝐸 ∗ состоит из одной точки, и
доказывать нечего). Положим 𝛿 = 𝑑(𝑞, 𝑝𝑛1 ). Пусть 𝑛1 , … , 𝑛𝑖−1 уже выбраны. Так как 𝑞 — предельная
точка множества 𝐸 ∗ , существует 𝑥 ∈ 𝐸 ∗ , такое, что 𝑑(𝑥, 𝑞) < 2−𝑖 𝛿. Так как 𝑥 ∈ 𝐸 ∗ , существует
𝑛𝑖 > 𝑛𝑖−1 , такое, что 𝑑(𝑥, 𝑝𝑛𝑖 ) < 2−𝑖 𝛿. Таким образом,
𝑑(𝑞, 𝑝𝑛𝑖 ) ≤ 21−𝑖 𝛿
при 𝑖 = 1, 2, 3, … . Это означает, что {𝑝𝑛𝑖 } сходится к 𝑞. Значит, 𝑞 ∈ 𝐸 ∗ .
Последовательности Коши
3.8 Определение. Последовательность {𝑝𝑛 } в метрическом пространстве 𝑋 называется последовательностью Коши (фундаментальной последовательностью, последовательностью, сходящейся
в себе), если для любого 𝜀 > 0 существует целое 𝑁, такое, что 𝑑(𝑝𝑛 , 𝑝𝑚 ) < 𝜀 при 𝑛 ≥ 𝑁 и 𝑚 ≥ 𝑁.
При рассмотрении последовательностей Коши, а также в других ситуациях, которые возникнут
позднее, окажется полезным следующее геометрическое понятие.
3.9 Определение. Пусть 𝐸 — непустое подмножество метрического пространства 𝑋, и пусть 𝑆 —
множество всех вещественных чисел вида 𝑑(𝑝, 𝑞), где 𝑝 ∈ 𝐸 и 𝑞 ∈ 𝐸. Диаметром множества 𝐸
называется число sup 𝑆. Это число обозначается diam 𝐸.
Если {𝑝𝑛 } — последовательность в 𝑋, а 𝐸𝑁 состоит из точек 𝑝𝑁 , 𝑝𝑁+1 , 𝑝𝑁+2 , … , то из двух последних определений ясно, что {𝑝𝑛 } — последовательность Коши тогда и только тогда, когда
lim diam 𝐸𝑁 = 0.
𝑁→∞
40
3.10 Теорема.
(a) Если 𝐸̄ — замыкание множества 𝐸 в метрическом пространстве 𝑋, то
diam 𝐸̄ = diam 𝐸.
(b) Если {𝐾𝑛 } — последовательность компактных множеств в 𝑋, такая, что 𝐾𝑛 ⊃ 𝐾𝑛+1 (𝑛 =
1, 2, 3, … ), и если
lim diam 𝐾𝑛 = 0,
𝑛→∞
то
⋂∞
1 𝐾𝑛
состоит ровно из одной точки.
Доказательство.
(a) Так как 𝐸 ⊂ 𝐸,̄ ясно, что
diam 𝐸 ≤ diam 𝐸.̄
Зафиксируем 𝜀 > 0 и выберем 𝑝 ∈ 𝐸,̄ 𝑞 ∈ 𝐸.̄ По определению множества 𝐸,̄ в 𝐸 содержатся
точки 𝑝′ , 𝑞 ′ , такие, что 𝑑(𝑝, 𝑝′ ) < 𝜀, 𝑑(𝑞, 𝑞 ′ ) < 𝜀. Значит,
𝑑(𝑝, 𝑞) ≤ 𝑑(𝑝, 𝑝′ ) + 𝑑(𝑝′ , 𝑞 ′ ) + 𝑑(𝑞 ′ , 𝑞)
< 2𝜀 + 𝑑(𝑝′ , 𝑞 ′ ) ≤ 2𝜀 + diam 𝐸.
Следовательно,
diam 𝐸̄ ≤ 2𝜀 + diam 𝐸,
и так как 𝜀 произвольно, (a) доказано.
(b) Положим 𝐾 = ⋂∞
𝑛 𝐾𝑛 . По теореме 2.36 𝐾 непусто. Если 𝐾 содержит более одной точки, то
diam 𝐾 > 0. Но при каждом 𝑛 мы имеем 𝐾𝑛 ⊃ 𝐾, так что diam 𝐾𝑛 ≥ diam 𝐾. Это противоречит
предположению, что diam 𝐾𝑛 → 0.
3.11 Теорема.
(a) Всякая сходящаяся последовательность в метрическом пространстве 𝑋 является последовательностью Коши.
(b) Если 𝑋 — компактное метрическое пространство, а {𝑝𝑛 } — последовательность Коши в 𝑋,
то {𝑝𝑛 } сходится к некоторой точке пространства 𝑋.
(c) Всякая последовательность Коши в 𝑅𝑘 сходится.
Замечание. Разница между определением сходящейся последовательности и определением последовательности Коши состоит в том, что в первое определение в явном виде входит предел, в
то время как во второе определение он не входит. Таким образом, теорема 3.11(b) позволяет нам
решить, сходится или нет данная последовательность, даже если мы не знаем предела, к которому
она может сходиться.
То (содержащееся в теореме 3.11) утверждение, что последовательность в 𝑅𝑘 сходится тогда и
только тогда, когда она является последовательностью Коши, обычно называют критерием сходимости Коши.
Доказательство.
(a) Если 𝑝𝑛 → 𝑝 и 𝜀 > 0, то существует целое 𝑁, такое, что 𝑑(𝑝, 𝑝𝑛 ) < 𝜀 при всех 𝑛 ≥ 𝑁. Значит,
𝑑(𝑝𝑛 , 𝑝𝑚 ) ≤ 𝑑(𝑝𝑛 , 𝑝) + 𝑑(𝑝, 𝑝𝑚 ) < 2𝜀,
если 𝑛 ≥ 𝑁 и 𝑚 ≥ 𝑁. Следовательно, {𝑝𝑛 } — последовательность Коши.
(b) Допустим, что {𝑝𝑛 } — последовательность Коши в компактном пространстве 𝑋. Для 𝑁 =
1, 2, 3, … пусть 𝐸𝑁 — множество, состоящее из 𝑝𝑁 , 𝑝𝑁+1 , 𝑝𝑁+2 , … . Тогда
(3)
̄ =0
lim diam 𝐸𝑁
𝑁→∞
по определению 3.9 и теореме 3.10(a). Будучи замкнутым подмножеством компактного про̄ компактно (по теореме 2.35). Кроме того, 𝐸𝑁 ⊃ 𝐸𝑁+1 , так что
странства 𝑋, каждое 𝐸𝑁
̄
̄
𝐸𝑁 ⊃ 𝐸𝑁+1 .
41
̄ .
По теореме 3.10(b) существует единственная точка 𝑝 ∈ 𝑋, принадлежащая каждому 𝐸𝑁
̄
Пусть задано 𝜀 > 0. В силу (3) имеется целое 𝑁0 , такое, что diam 𝐸𝑁 < 𝜀, если 𝑁 ≥ 𝑁0 .
̄ , это значит, что 𝑑(𝑝, 𝑞) < 𝜀 при всех 𝑞 ∈ 𝐸𝑁
̄ , а поэтому и при всех 𝑞 ∈ 𝐸𝑁 .
Поскольку 𝑝 ∈ 𝐸𝑁
Другими словами, 𝑑(𝑝, 𝑝𝑛 ) < 𝜀 при 𝑛 ≥ 𝑁0 . Но это означает в точности, что 𝑝𝑛 → 𝑝.
(c) Пусть {x𝑛 } — последовательность Коши в 𝑅𝑘 . Определим 𝐸𝑛 как в (b), используя x𝑖 вместо
𝑝𝑖 . Найдется 𝑁, для которого diam 𝐸𝑁 < 1. Множество значений последовательности {x𝑛 } —
объединение множества 𝐸𝑛 и конечного множества {x1 , … , x𝑁−1 }. Следовательно, последовательность {x𝑛 } ограничена. Так как любое ограниченное подмножество пространства 𝑅𝑘
имеет компактное замыкание в 𝑅𝑘 (по теореме 2.41), (c) следует из (b).
3.12 Определение. Метрическое пространство, в котором каждая последовательность Коши сходится, называется полным.
С учетом этого определения теорему 3.11 можно сформулировать так: все компактные метрические пространства и все евклидовы пространства являются полными. Из теоремы 3.11 следует
также, что любое замкнутое подмножество 𝐸 полного метрического пространства 𝑋 является
полным (каждая последовательность Коши в 𝐸 является последовательностью Коши в 𝑋, значит, она сходится к некоторой точке 𝑝 ∈ 𝑋, причем на самом деле 𝑝 ∈ 𝐸, так как 𝐸 замкнуто).
Примером неполного метрического пространства служит пространство всех рациональных чисел с
расстоянием 𝑑(𝑥, 𝑦) = |𝑥 − 𝑦|.
Теорема 3.2(c) и пример (d) из определения 3.1 показывают, что сходящиеся последовательности
ограничены, но ограниченные последовательности в 𝑅𝑘 не обязательно сходятся. Однако имеется
один важный случай, когда сходимость действительно равносильна ограниченности. Так обстоит
дело с монотонными последовательностями в 𝑅1 .
3.13 Определение. Последовательность {𝑠𝑛 } вещественных чисел называется
(a) монотонно возрастающей, если 𝑠𝑛 ≤ 𝑠𝑛+1 (𝑛 = 1, 2, 3, … );
(b) монотонно убывающей, если 𝑠𝑛 ≥ 𝑠𝑛+1 (𝑛 = 1, 2, 3, … ).
Класс монотонных последовательностей состоит из возрастающих и убывающих последовательностей.
3.14 Теорема. Монотонная последовательность {𝑠𝑛 } сходится в том и только в том случае,
когда она ограничена.
Доказательство. Допустим, что 𝑠𝑛 ≤ 𝑠𝑛+1 (в другом случае доказательство аналогично). Пусть
𝐸 — множество значений последовательности {𝑠𝑛 }. Если {𝑠𝑛 } ограничена, то пусть 𝑠 — верхняя
грань множества 𝐸. Тогда
(𝑛 = 1, 2, 3, … ).
𝑠𝑛 ≤ 𝑠
Для любого 𝜀 > 0 существует целое 𝑁, такое, что
𝑠 − 𝜀 < 𝑠𝑁 ≤ 𝑠,
так как иначе 𝑠 − 𝜀 было бы верхней границей множества 𝐸. Поскольку {𝑠𝑛 } возрастает, при 𝑛 ≥ 𝑁
имеем
𝑠 − 𝜀 < 𝑠𝑛 ≤ 𝑠,
откуда следует, что {𝑠𝑛 } сходится (к 𝑠).
Обратное следует из теоремы 3.2(c).
Верхний и нижний пределы
3.15 Определение. Пусть {𝑠𝑛 } — последовательность вещественных чисел, обладающая следующим свойством: для любого вещественного 𝑀 существует целое 𝑁, такое, что при 𝑛 ≥ 𝑁 мы имеем
𝑠𝑛 ≥ 𝑀. Тогда мы пишем
𝑠𝑛 → +∞.
Аналогичным образом, если для любого вещественного 𝑀 существует целое 𝑁, такое, что при
𝑛 ≥ 𝑁 мы имеем 𝑠𝑛 ≤ 𝑀, то мы пишем
𝑠𝑛 → −∞.
42
Следует отметить, что мы теперь используем символ → (введенный в определении 3.1) для
некоторых типов расходящихся последовательностей, так же как и для сходящихся последовательностей, но что определения сходимости и предела, данные в определении 3.1, никоим образом не
меняются.
3.16 Определение. Пусть {𝑠𝑛 } — последовательность вещественных чисел. Пусть 𝐸 — множество чисел 𝑥 (в расширенной системе вещественных чисел), таких, что 𝑠𝑛𝑘 → 𝑥 для некоторой
подпоследовательности {𝑠𝑛𝑘 }. Это множество 𝐸 содержит все частичные пределы, определенные в
определении 3.5, и, возможно, числа +∞, −∞.
Вспомним теперь определения 1.8 и 1.23 и положим
𝑠∗ = sup 𝐸,
𝑠∗ = inf 𝐸.
Числа 𝑠∗ , 𝑠∗ называются верхним и нижним пределами последовательности {𝑠𝑛 }; мы используем
обозначения
lim 𝑠𝑛 = 𝑠∗ ,
lim 𝑠𝑛 = 𝑠∗ .
𝑛→∞
𝑛→∞
3.17 Теорема. Пусть {𝑠𝑛 } — последовательность вещественных чисел. Пусть 𝐸 и 𝑠∗ имеют
тот же смысл, что и в определении 3.16. Тогда 𝑠∗ обладает следующими двумя свойствами:
(a) 𝑠∗ ∈ 𝐸;
(b) если 𝑥 > 𝑠∗ , то существует целое 𝑁, такое, что при 𝑛 ≥ 𝑁 имеем 𝑠𝑛 < 𝑥.
Более того, 𝑠∗ — единственное число, обладающее свойствами (a) и (b).
Конечно, аналогичный результат верен для 𝑠∗ .
Доказательство.
(a) Если 𝑠∗ = +∞, то 𝐸 не ограничено сверху; значит, последовательность {𝑠𝑛 } не ограничена
сверху и существует подпоследовательность {𝑠𝑛𝑘 }, такая, что 𝑠𝑛𝑘 → +∞.
Если 𝑠∗ — вещественное число, то 𝐸 ограничено сверху и существует по крайней мере один
частичный предел, поэтому (a) следует из теорем 3.7 и 2.28.
Если 𝑠∗ = −∞, то 𝐸 содержит только один элемент, а именно −∞, и не существует ни одного частичного предела. Значит, для любого вещественного 𝑀 неравенство 𝑠𝑛 > 𝑀 может
выполняться не более чем для для конечного числа значений 𝑛, так что 𝑠𝑛 → −∞.
Тем самым (a) установлено во всех случаях.
(b) Допустим, что существует число 𝑥 > 𝑠∗ , такое, что 𝑠𝑛 ≥ 𝑥 для бесконечного множества значений 𝑛. В этом случае существует число 𝑦 ∈ 𝐸, такое, что 𝑦 ≥ 𝑥 > 𝑠∗ , а это противоречит
определению 𝑠∗ .
Таким образом, 𝑠∗ удовлетворяет (a) и (b).
Для доказательства единственности допустим, что существуют два числа 𝑝 и 𝑞, удовлетворяющие (a) и (b), и допустим, что 𝑝 < 𝑞. Выберем 𝑥 таким, что 𝑝 < 𝑥 < 𝑞. Так как 𝑝 удовлетворяет
(b), имеем 𝑠𝑛 < 𝑥 при 𝑛 ≥ 𝑁. Но тогда 𝑞 не может удовлетворять (a).
3.18 Примеры.
(a) Пусть {𝑠𝑛 } — последовательность, содержащая все рациональные числа. Тогда каждое вещественное число является частичным пределом и
lim 𝑠
𝑛→∞ 𝑛
= +∞,
lim 𝑠𝑛 = −∞.
𝑛→∞
(b) Пусть 𝑠𝑛 = (−1)𝑛 /[1 + (1/𝑛)]. Тогда
lim 𝑠
𝑛→∞ 𝑛
= 1,
lim 𝑠𝑛 = −1.
𝑛→∞
43
(c) Для последовательности {𝑠𝑛 } вещественных чисел lim 𝑠𝑛 = 𝑠 тогда и только тогда, когда
𝑛→∞
lim 𝑠
𝑛→∞ 𝑛
= lim 𝑠𝑛 = 𝑠.
𝑛→∞
Мы закончим этот раздел одной полезной теоремой, доказательство которой совсем тривиально.
3.19 Теорема. Если 𝑠𝑛 ≤ 𝑡𝑛 при 𝑛 ≥ 𝑁, где 𝑁 фиксировано, то
lim 𝑠𝑛 ≤ lim 𝑡𝑛 ,
𝑛→∞
lim 𝑠
𝑛→∞ 𝑛
𝑛→∞
≤ lim 𝑡𝑛 .
𝑛→∞
Некоторые специальные последовательности
Теперь мы вычислим пределы некоторых часто встречающихся последовательностей. Все доказательства будут основаны на следующем замечании: если 0 ≤ 𝑥𝑛 ≤ 𝑠𝑛 при 𝑛 ≥ 𝑁, где 𝑁 — некоторое
фиксированное число, и если 𝑠𝑛 → 0, то 𝑥𝑛 → 0.
3.20 Теорема.
1
= 0.
𝑛𝑝
𝑛
(b) Если 𝑝 > 0, то lim √
𝑝 = 1.
(a) Если 𝑝 > 0, то lim
𝑛→∞
𝑛→∞
𝑛
(c) lim √𝑛 = 1.
𝑛→∞
𝑛𝛼
= 0.
𝑛→∞ (1 + 𝑝)𝑛
(d) Если 𝑝 > 0 и 𝛼 — вещественное число, то lim
(e) Если |𝑥| < 1, то lim 𝑥𝑛 = 0.
𝑛→∞
Доказательство.
(a) Возьмем 𝑛 > (1/𝜀)1/𝑝 (заметим, что здесь используется архимедовость системы вещественных
чисел).
𝑛
(b) Если 𝑝 > 1, то положим 𝑥𝑛 = √
𝑝 − 1. Тогда 𝑥𝑛 > 0 и, согласно биномиальной теореме,
1 + 𝑛𝑥𝑛 ≤ (1 + 𝑥𝑛 )𝑛 = 𝑝,
так что
𝑝−1
.
𝑛
Значит, 𝑥𝑛 → 0. Если 𝑝 = 1, то (b) тривиально, а если 0 < 𝑝 < 1, то результат получается
переходом к обратным величинам.
𝑛
𝑛 − 1. Тогда 𝑥𝑛 ≥ 0 и, согласно биномиальной теореме,
(c) Положим 𝑥𝑛 = √
0 < 𝑥𝑛 ≤
𝑛 = (1 + 𝑥𝑛 )𝑛 ≥
𝑛(𝑛 − 1) 2
𝑥𝑛 .
2
Значит,
2
(𝑛 ≥ 2).
0 ≤ 𝑥𝑛 ≤ √
𝑛−1
(d) Пусть 𝑘 — такое целое число, что 𝑘 > 𝛼, 𝑘 > 0. Для 𝑛 > 2𝑘 имеем
(1 + 𝑝)𝑛 >
𝑛 𝑘 𝑛(𝑛 − 1) ⋯ (𝑛 − 𝑘 + 1) 𝑘 𝑛𝑘 𝑝𝑘
𝑝 =
𝑝 > 𝑘 .
(𝑘 )
𝑘!
2 𝑘!
Значит,
0<
𝑛𝛼
2𝑘 𝑘! 𝛼−𝑘
<
𝑛
(1 + 𝑝)𝑛
𝑝𝑘
Поскольку 𝛼 − 𝑘 < 0, имеем 𝑛𝛼−𝑘 → 0 в силу (a).
(e) Возьмем 𝛼 = 0 в (d).
44
(𝑛 > 2𝑘).
Ряды
В остальной части этой главы все рассматриваемые последовательности и ряды будут комплекснозначными, если явно не оговорено противное. В упражнении 15 предлагается распространить
некоторые из следующих ниже теорем на ряды с членами из 𝑅𝑘 .
3.21 Определение. Пусть задана последовательность {𝑎𝑛 }. Мы будем использовать обозначение
𝑞
∑
𝑎𝑛
(𝑝 ≤ 𝑞)
𝑛=𝑝
для суммы 𝑎𝑝 + 𝑎𝑝+1 + ⋯ + 𝑎𝑞 . Последовательности {𝑎𝑛 } мы сопоставим последовательность {𝑠𝑛 }, где
𝑛
𝑠𝑛 =
∑
𝑎𝑘 .
𝑘=1
Мы также будем использовать для {𝑠𝑛 } символическое выражение
𝑎1 + 𝑎2 + 𝑎3 + ⋯
или, короче,
∞
(4)
∑
𝑎𝑛 .
𝑛=1
Символ (4) мы будем называть бесконечным рядом или просто рядом. Числа 𝑠𝑛 называются
частными суммами этого ряда. Если последовательность {𝑠𝑛 } сходится к 𝑠, то мы будем говорить,
что ряд сходится, и будем писать
∞
∑
𝑎𝑛 = 𝑠.
𝑛=1
Число 𝑠 называется суммой этого ряда, но необходимо ясно понимать, что 𝑠 является пределом
последовательности сумм, а не получается простым сложением.
Если последовательность {𝑠𝑛 } расходится, то говорят, что ряд расходится.
Иногда для удобства обозначений мы будем рассматривать ряды вида
∞
(5)
∑
𝑎𝑛 .
𝑛=0
Часто, если это не вызывает двусмысленности или разница несущественна, мы будем вместо (4)
или (5) писать просто ∑ 𝑎𝑛 .
Ясно, что каждую теорему о последовательностях можно сформулировать на языке рядов (полагая 𝑎1 = 𝑠1 и 𝑎𝑛 = 𝑠𝑛 − 𝑠𝑛−1 при 𝑛 > 1) и обратно. Но тем не менее полезно различать эти понятия.
Критерий Коши (теорему 3.11) можно сформулировать в следующем виде.
3.22 Теорема. Ряд ∑ 𝑎𝑛 сходится тогда и только тогда, когда для любого 𝜀 > 0 существует
целое 𝑁, такое, что
𝑚
(6)
∑
|𝑘=𝑛
𝑎𝑘 ≤ 𝜀,
|
если 𝑚 ≥ 𝑛 ≥ 𝑁.
В частности, взяв 𝑚 = 𝑛, из (6) получаем
|𝑎𝑛 | ≤ 𝜀
(𝑛 ≥ 𝑁).
Иными словами, справедлива следующая теорема.
45
3.23 Теорема. Если ряд ∑ 𝑎𝑛 сходится, то lim𝑛→∞ 𝑎𝑛 = 0.
Однако условие 𝑎𝑛 → 0 недостаточно для того, чтобы обеспечить сходимость ряда ∑ 𝑎𝑛 . Например, ряд
∞
1
∑𝑛
𝑛=1
расходится; за доказательством мы отсылаем к теореме 3.28.
Теорема 3.14 о монотонных последовательностях также имеет очевидный аналог для рядов.
3.24 Теорема. Ряд неотрицательных1 членов сходится тогда и только тогда, когда его частные
суммы образуют ограниченную последовательность.
Теперь мы обратимся к признаку сходимости другой природы — к так называемому «признаку
сравнения».
3.25 Теорема.
(a) Если |𝑎𝑛 | ≤ 𝑐𝑛 при 𝑛 ≥ 𝑁0 , где 𝑁0 — некоторое фиксированное целое, и если ряд ∑ 𝑐𝑛 сходится,
то и ряд ∑ 𝑎𝑛 сходится.
(b) Если 𝑎𝑛 ≥ 𝑑𝑛 ≥ 0 при 𝑛 ≥ 𝑁0 и если ряд ∑ 𝑑𝑛 расходится, то и ряд ∑ 𝑎𝑛 расходится.
Заметим, что (b) применимо только к рядам с неотрицательными членами 𝑎𝑛 .
Доказательство. Согласно критерию Коши, для данного 𝜀 > 0 существует 𝑁 ≥ 𝑁0 , такое, что
при 𝑚 ≥ 𝑛 ≥ 𝑁 имеем
𝑚
∑
𝑐𝑘 ≤ 𝜀.
𝑘=𝑛
Значит,
𝑚
𝑚
𝑚
𝑎 ≤
|𝑎 | ≤
𝑐 ≤ 𝜀,
∑ 𝑘| ∑ 𝑘
∑ 𝑘
|𝑘=𝑛
𝑘=𝑛
𝑘=𝑛
и (a) доказано.
Далее, (b) следует из (a), так как если ряд ∑ 𝑎𝑛 сходится, то и ряд ∑ 𝑑𝑛 должен сходиться
(заметим, что (b) следует также из теоремы 3.24).
Этот признак сравнения очень полезен; чтобы успешно применять его, мы должны освоиться с
некоторым набором рядов с неотрицательными членами, заведомо сходящихся или расходящихся.
Ряды с неотрицательными членами
Простейшим из всех таких рядов, по-видимому, является геометрическая прогрессия.
3.26 Теорема. Если 0 ≤ 𝑥 < 1, то
∞
∑
𝑥𝑛 =
𝑛=0
1
.
1−𝑥
Если 𝑥 ≥ 1, то этот ряд расходится.
Доказательство. Если 𝑥 ≠ 1, то
𝑛
𝑠𝑛 =
∑
𝑥𝑘 =
𝑘=0
1 − 𝑥𝑛+1
.
1−𝑥
Устремляя 𝑛 к ∞, мы получим требуемый результат. При 𝑥 = 1 получается ряд
1+1+1+⋯ ,
который, очевидно, расходится.
1
Выражение «неотрицательный» всегда относится к вещественным числам.
46
Во многих случаях, встречающихся в приложениях, члены ряда монотонно убывают. Поэтому
следующая теорема Коши представляет особый интерес. Поразительная особенность утверждения
теоремы состоит в том, что довольно «редкая» подпоследовательность последовательности {𝑎𝑛 }
определяет сходимость или расходимость ряда ∑ 𝑎𝑛 .
3.27 Теорема. Допустим, что 𝑎1 ≥ 𝑎2 ≥ 𝑎3 ≥ ⋯ ≥ 0. Тогда ряд ∑∞
𝑛=1 𝑎𝑛 сходится в том и только
в том случае, когда сходится ряд
∞
(7)
∑
2𝑘 𝑎2𝑘 = 𝑎1 + 2𝑎2 + 4𝑎4 + 8𝑎8 + ⋯ .
𝑘=0
Доказательство. По теореме 3.24 достаточно установить ограниченность частных сумм. Пусть
𝑠𝑛 = 𝑎 1 + 𝑎 2 + ⋯ + 𝑎 𝑛 ,
𝑡𝑘 = 𝑎1 + 2𝑎2 + ⋯ + 2𝑘 𝑎2𝑘 .
При 𝑛 < 2𝑘 имеем
𝑠𝑛 ≤ 𝑎1 + (𝑎2 + 𝑎3 ) + ⋯ + (𝑎2𝑘 + ⋯ + 𝑎2𝑘+1 −1 )
≤ 𝑎1 + 2𝑎2 + ⋯ + 2𝑘 𝑎2𝑘
= 𝑡𝑘 ,
так что
(8)
𝑠𝑛 ≤ 𝑡𝑘 .
С другой стороны, при 𝑛 > 2𝑘 имеем
𝑠𝑛 ≥ 𝑎1 + 𝑎2 + (𝑎3 + 𝑎4 ) + ⋯ + (𝑎2𝑘−1 +1 + ⋯ + 𝑎2𝑘 )
≥ 12 𝑎1 + 𝑎2 + 2𝑎4 + ⋯ + 2𝑘−1 𝑎2𝑘
= 21 𝑡𝑘 ,
так что
(9)
2𝑠𝑛 ≥ 𝑡𝑘 .
В силу (8) и (9) последовательности {𝑠𝑛 } и {𝑡𝑛 } или обе ограничены, или обе не ограничены.
Доказательство закончено.
3.28 Теорема. Ряд ∑
1
сходится, если 𝑝 > 1, и расходится, если 𝑝 ≤ 1.
𝑛𝑝
Доказательство. Если 𝑝 ≤ 0, то расходимость следует из теоремы 3.23. Если 𝑝 > 0, то применима
теорема 3.27, и мы приходим к ряду
∞
∑
𝑘=0
∞
2𝑘 ⋅
1
2(1−𝑝)𝑘 .
=
∑
2𝑘𝑝 𝑘=0
Но 21−𝑝 < 1 тогда и только тогда, когда 1−𝑝 < 0, и теорема вытекает из сравнения с геометрической
прогрессией (следует положить 𝑥 = 21−𝑝 в теореме 3.26).
В качестве еще одного приложения теоремы 3.27 будет доказана следующая теорема.
3.29 Теорема. Если 𝑝 > 1, то ряд
∞
(10)
1
∑ 𝑛(log 𝑛)𝑝
𝑛=2
сходится; если 𝑝 ≤ 1, то этот ряд расходится.
47
Замечание. Символ log 𝑛 обозначает логарифм 𝑛 по основанию 𝑒 (ср. упражнение 7, гл. 1); число
𝑒 будет вскоре определено (см. определение 3.30). Мы начинаем ряд с 𝑛 = 2, так как log 1 = 0.
Доказательство. Из монотонности логарифмической функции (которая более подробно будет
рассматриваться в гл. 8) следует, что последовательность {log 𝑛} возрастает. Значит, последовательность {1/𝑛 log 𝑛} убывает, и мы можем применить теорему 3.27 к (10); это приводит к ряду
∞
(11)
∑
∞
2𝑘 ⋅
𝑘=1
∞
1
1
1
1
=
=
,
𝑝
𝑝
𝑝
𝑘
𝑘
𝑝
∑
∑
(𝑘
log
2)
(log
2)
𝑘
2 (log 2 )
𝑘=1
𝑘=1
и теорема 3.29 следует из теоремы 3.28.
Эту процедуру, очевидно, можно продолжить. Например, ряд
∞
1
∑ 𝑛 log 𝑛 log log 𝑛
(12)
𝑛=3
расходится, тогда как ряд
∞
1
∑ 𝑛 log 𝑛(log log 𝑛)2
(13)
𝑛=3
сходится.
Можно заметить, что члены ряда (12) очень мало отличаются от членов ряда (13). Однако
один ряд сходится, а другой расходится. Продолжая процесс, который привел нас от теоремы 3.28
к теореме 3.29, а затем к (12) и (13), мы получим пары сходящихся и расходящихся рядов, члены
которых отличаются даже меньше, чем члены рядов (12) и (13). Можно было бы предположить,
что имеется некое предельное положение, «граница», по одну сторону которой лежат все сходящиеся, а по другую — все расходящиеся ряды, по крайней мере пока речь идет о рядах с монотонно
убывающими членами. Конечно, это понятие «границы» совсем неясное. Однако мы хотим отметить следующее: как бы мы ни уточнили это понятие, такое предположение окажется неверным.
Упражнения 11(b) и 12(b) могут служить иллюстрациями.
Мы не хотим вдаваться глубже в подобные вопросы теории сходимости и отсылаем читателя к
главе IX, в особенности разделу 41, книги Кноппа «Theory and Application of Infinite Series».
Число e
∞
3.30 Определение. 𝑒 =
1
.
∑ 𝑛!
𝑛=0
Здесь 𝑛! = 1 ⋅ 2 ⋅ 3 ⋯ 𝑛, если 𝑛 ≥ 1, и 0! = 1.
Так как
1
1
1
+
+⋯+
1⋅2 1⋅2⋅3
1 ⋅ 2⋯𝑛
1
1
1
< 1 + 1 + + 2 + ⋯ + 𝑛−1 < 3,
2 2
2
𝑠𝑛 = 1 + 1 +
этот ряд сходится, поэтому определение имеет смысл. На самом деле указанный ряд сходится очень
быстро, и это позволяет нам вычислить 𝑒 с большой точностью.
Интересно отметить, что 𝑒 можно определить также при помощи другого предельного перехода;
доказательство служит хорошей иллюстрацией того, как следует оперировать с пределами.
1 𝑛
3.31 Теорема. lim (1 + ) = 𝑒.
𝑛→∞
𝑛
48
Доказательство. Пусть
𝑛
𝑠𝑛 =
1 𝑛
𝑡 𝑛 = (1 + ) .
𝑛
1
,
∑ 𝑘!
𝑘=0
По биномиальной теореме
𝑡𝑛 = 1 + 1 +
1
1
1
1
2
1
1
2
𝑛−1
1 − ) + (1 − ) ( 1 − ) + ⋯ + (1 − ) ( 1 − ) ⋯ (1 −
.
2! (
𝑛
3!
𝑛
𝑛
𝑛!
𝑛
𝑛
𝑛 )
Значит, 𝑡𝑛 ≤ 𝑠𝑛 , так что
(14)
lim 𝑡
𝑛→∞ 𝑛
≤𝑒
по теореме 3.19. Далее, если 𝑛 ≥ 𝑚, то
1
1
1
𝑚−1
1
1− )+⋯+
1 − ) ⋯ (1 −
.
2! (
𝑛
𝑚! (
𝑛
𝑛 )
Положим 𝑛 → ∞, оставляя 𝑚 фиксированным. Получим
𝑡𝑛 ≥ 1 + 1 +
lim 𝑡𝑛 ≥ 1 + 1 +
𝑛→∞
1
1
+⋯+
,
2!
𝑚!
так что
𝑠𝑚 ≤ lim 𝑡𝑛 .
𝑛→∞
Устремляя 𝑚 к ∞, мы окончательно получим
(15)
𝑒 ≤ lim 𝑡𝑛 .
𝑛→∞
Утверждение теоремы следует из (14) и (15).
Скорость, с которой сходятся ряд ∑ 𝑛!1 , можно оценить так: если 𝑠𝑛 обозначает то же, что и
выше, то
1
1
1
+
+
+⋯
(𝑛 + 1)! (𝑛 + 2)! (𝑛 + 3)!
1
1
1
1
+⋯ =
<
1+
+
,
2
}
{
(𝑛 + 1)!
𝑛 + 1 (𝑛 + 1)
𝑛!𝑛
𝑒 − 𝑠𝑛 =
так что
(16)
0 < 𝑒 − 𝑠𝑛 <
1
.
𝑛!𝑛
Таким образом, 𝑠10 приближает число 𝑒 с ошибкой, меньшей 10−7 . Неравенство (16) представляет
и теоретический интерес, так как оно позволяет очень легко доказать иррациональность числа 𝑒.
3.32 Теорема. Число 𝑒 иррационально.
Доказательство. Допустим, что 𝑒 рационально. Тогда 𝑒 = 𝑝/𝑞, где 𝑝 и 𝑞 — положительные целые
числа. В силу (16)
(17)
0 < 𝑞!(𝑒 − 𝑠𝑞 ) <
1
.
𝑞
Согласно предположению, 𝑞!𝑒 целое. Поскольку
1
1
𝑞!𝑠𝑞 = 𝑞! 1 + 1 +
+⋯+
(
2!
𝑞! )
целое, мы видим, что 𝑞!(𝑒 − 𝑠𝑞 ) также целое.
Так как 𝑞 ≥ 1, из (17) следует существование целого числа между 0 и 1. Таким образом, мы
добились противоречия.
На самом деле 𝑒 не является даже алгебраическим числом. Простое доказательство этого факта
можно найти на стр. 25 книги Нивена или стр. 176 книги Херстейна.
49
Другие признаки сходимости
𝑛
3.33 Теорема (признак Коши). Пусть задан ряд ∑ 𝑎𝑛 . Положим 𝛼 = lim √|𝑎𝑛 |. Тогда
𝑛→∞
(a) если 𝛼 < 1, то ряд ∑ 𝑎𝑛 сходится;
(b) если 𝛼 > 1, то ряд ∑ 𝑎𝑛 расходится;
(c) если 𝛼 = 1, то признак не дает ответа.
Доказательство. Если 𝛼 < 1, то можно выбрать 𝛽 так, что 𝛼 < 𝛽 < 1, а целое число 𝑁 — так, что
𝑛
√|𝑎𝑛 | < 𝛽
при 𝑛 ≥ 𝑁 (по теореме 3.17(b)). Иначе говоря, при 𝑛 ≥ 𝑁 имеем
|𝑎𝑛 | < 𝛽 𝑛 .
Так как 0 < 𝛽 < 1, ряд ∑ 𝛽 𝑛 сходится. Сходимость ряда ∑ 𝑎𝑛 следует теперь из признака сравнения.
Если 𝛼 > 1, то, снова по теореме 3.17, существует последовательность {𝑛𝑘 }, такая, что
𝑛𝑘
√|𝑎𝑛𝑘 | → 𝛼.
Значит, |𝑎𝑛 | > 1 для бесконечного множества значений 𝑛, так что условие 𝑎𝑛 → 0, необходимое для
сходимости ряда ∑ 𝑎𝑛 , не выполнено (теорема 3.23).
Чтобы доказать (c), рассмотрим ряды
1
1
,
.
∑ 𝑛 ∑ 𝑛2
Для каждого из этих рядов 𝛼 = 1, но первый расходится, а второй сходится.
3.34 Теорема (признак Даламбера). Ряд ∑ 𝑎𝑛
𝑎𝑛+1
< 1,
𝑛→∞ | 𝑎𝑛 |
𝑎𝑛+1
(b) расходится, если
≥ 1 при 𝑛 ≥ 𝑛0 , где 𝑛0 — некоторое фиксированное целое число.
| 𝑎𝑛 |
(a) сходится, если lim
Доказательство. Если условие (a) выполнено, то можно найти 𝛽 < 1 и целое 𝑁, такие, что
𝑎𝑛+1
<𝛽
| 𝑎𝑛 |
при 𝑛 ≥ 𝑁. В частности,
|𝑎𝑁+1 | < 𝛽|𝑎𝑁 |,
|𝑎𝑁+2 | < 𝛽|𝑎𝑁+1 | < 𝛽 2 |𝑎𝑁 |,
…
|𝑎𝑁+𝑝 | < 𝛽 𝑝 |𝑎𝑁 |.
Это значит, что
|𝑎𝑛 | < |𝑎𝑁 |𝛽 −𝑁 ⋅ 𝛽 𝑛
при 𝑛 ≥ 𝑁, и (a) следует из признака сравнения, так как ряд ∑ 𝛽 𝑛 сходится.
Если |𝑎𝑛+1 | ≥ |𝑎𝑛 | при 𝑛 ≥ 𝑛0 , то легко видеть, что условие 𝑎𝑛 → 0 не выполнено, откуда и следует
(b).
Замечание. Знание того, что lim 𝑎𝑛+1 /𝑎𝑛 = 1, ничего не говорит о сходимости ∑ 𝑎𝑛 . Это показывают ряды ∑ 1/𝑛 и ∑ 1/𝑛2 .
3.35 Примеры.
50
(a) Рассмотрим ряд
1
1
1
1
1
1
1 1
+ +
+
+
+
+
+
+⋯ ,
2 3 22 32 23 33 24 34
для которого
𝑎𝑛+1
2 𝑛
= lim ( ) = 0,
𝑛→∞ 3
𝑛→∞ 𝑎𝑛
lim
2𝑛
1
1
𝑛
lim √
𝑎𝑛 = lim √ 𝑛 =
,
𝑛→∞
3
𝑛→∞
√3
2𝑛
1
1
𝑛
lim √
𝑎𝑛 = lim √ 𝑛 =
,
𝑛→∞
2
√2
𝑎𝑛+1
1 3 𝑛
= lim ( ) = +∞.
lim
𝑛→∞ 2 2
𝑛→∞ 𝑎𝑛
𝑛→∞
Признак Коши указывает на сходимость; признак Даламбера не дает ответа.
(b) То же самое верно в отношении ряда
1
1 1
1
1
1
1
+1+ + +
+
+
+
+⋯ ,
2
8 4 32 16 128 64
где
𝑎𝑛+1
1
= ,
𝑎
8
𝑛→∞
𝑛
𝑎𝑛+1
lim
= 2,
𝑛→∞ 𝑎𝑛
lim
но
𝑛
𝑎𝑛 = 21 .
lim √
𝑛→∞
3.36 Замечания. Признак Даламбера, как правило, легче применять, чем признак Коши, так
как обычно легче вычислять частные, чем корни 𝑛-ой степени. Однако признак Коши сильнее в
следующем смысле: когда признак Даламбера указывает на сходимость, то и признак Коши тоже
указывает на сходимость; если же признак Коши не дает ответа, то и признак Даламбера тоже не
дает ответа. Это следует из теоремы 3.37 и иллюстрируется приведенными выше примерами.
Ни один из этих двух признаков не является особенно тонким в отношении расходимости. В
обоих расходимость выводится из того, что 𝑎𝑛 не стремится к нулю при 𝑛 → ∞.
3.37 Теорема. Для любой последовательности {𝑐𝑛 } положительных чисел имеем
𝑐𝑛+1
𝑛
≤ lim √
𝑐𝑛 ,
𝑛→∞ 𝑐𝑛
𝑛→∞
𝑐𝑛+1
𝑛
.
lim √
𝑐𝑛 ≤ lim
𝑛→∞
𝑛→∞ 𝑐𝑛
lim
Доказательство. Мы докажем второе неравенство; доказательство первого совершенно аналогично. Положим
𝑐𝑛+1
𝛼 = lim
.
𝑛→∞ 𝑐𝑛
Если 𝛼 = +∞, то доказывать нечего. Если 𝛼 конечно, то выберем 𝛽 > 𝛼. Существует целое 𝑁, такое,
что
𝑐𝑛+1
≤𝛽
𝑐𝑛
при 𝑛 ≥ 𝑁. В частности, для любого 𝑝 > 0
𝑐𝑁+𝑘+1 ≤ 𝛽𝑐𝑁+𝑘
(𝑘 = 0, 1, … , 𝑝 − 1).
51
Перемножая эти неравенства, мы получаем
𝑐𝑁+𝑝 ≤ 𝛽 𝑝 𝑐𝑁 ,
или
𝑐𝑛 ≤ 𝑐𝑁 𝛽 −𝑁 ⋅ 𝛽 𝑛
(𝑛 ≥ 𝑁).
Значит,
𝑛
𝑛
𝑐𝑛 ≤ √𝑐𝑁 𝛽 −𝑁 ⋅ 𝛽,
√
так что
𝑛
lim √
𝑐𝑛 ≤ 𝛽
(18)
𝑛→∞
по теореме 3.20(b). Поскольку (18) верно при любом 𝛽 > 𝛼, имеем
𝑛
lim √
𝑐𝑛 ≤ 𝛼.
𝑛→∞
Степенные ряды
3.38 Определение. Пусть задана последовательность комплексных чисел {𝑐𝑛 }. Ряд
∞
(19)
∑
𝑐𝑛 𝑧𝑛
𝑛=0
называется степенным рядом. Числа 𝑐𝑛 называются коэффициентами этого ряда; 𝑧 — комплексное
число.
Вообще говоря, этот ряд сходится или расходится в зависимости от выбора числа 𝑧. Точнее, с
каждым степенным рядом связан круг, так называемый круг сходимости, такой, что ряд (19) сходится, если 𝑧 лежит внутри этого круга, и расходится, если 𝑧 находится вне круга (чтобы охватить
все случаи, мы должны рассматривать плоскость как внутренность круга бесконечного радиуса,
а точку — как круг нулевого радиуса). Поведение ряда на окружности круга сходимости может
быть гораздо более разнообразным, и его нельзя так просто описать.
3.39 Теорема. Пусть задан степенной ряд ∑ 𝑐𝑛 𝑧𝑛 . Положим
𝑛
𝛼 = lim √|𝑐𝑛 |,
𝑅=
𝑛→∞
1
𝛼
(если 𝛼 = 0, то 𝑅 = +∞; если 𝛼 = +∞, то 𝑅 = 0). Тогда ряд ∑ 𝑐𝑛 𝑧𝑛 сходится, если |𝑧| < 𝑅, и
расходится, если |𝑧| > 𝑅.
Доказательство. Положим 𝑎𝑛 = 𝑐𝑛 𝑧𝑛 и применим признак Коши:
𝑛
𝑛
lim √|𝑎𝑛 | = |𝑧| lim √|𝑐𝑛 | =
𝑛→∞
𝑛→∞
|𝑧|
.
𝑅
Замечание. Число 𝑅 называется радиусом сходимости ряда ∑ 𝑐𝑛 𝑧𝑛 .
3.40 Примеры.
(a) Ряд ∑ 𝑛𝑛 𝑧𝑛 имеет 𝑅 = 0.
𝑧𝑛
(b) Ряд ∑
имеет 𝑅 = +∞ (в этом случае проще применить признак Даламбера, чем признак
𝑛!
Коши).
(c) Ряд ∑ 𝑧𝑛 имеет 𝑅 = 1. Если |𝑧| = 1, то ряд расходится, так как последовательность {𝑧𝑛 } не
стремится к 0 при 𝑛 → ∞.
𝑧𝑛
имеет 𝑅 = 1. Он расходится при 𝑧 = 1 и сходится для всех остальных 𝑧, для которых
(d) Ряд ∑
𝑛
|𝑧| = 1 (последнее утверждение будет доказано в теореме 3.44).
𝑧𝑛
(e) Ряд ∑ 2 имеет 𝑅 = 1. Он сходится для всех 𝑧, для которых |𝑧| = 1, согласно признаку
𝑛
сравнения, так как |𝑧𝑛 /𝑛2 | = 1/𝑛2 .
52
Суммирование по частям
3.41 Теорема. Пусть даны две последовательности {𝑎𝑛 }, {𝑏𝑛 }. Положим
𝑛
𝐴𝑛 =
∑
𝑎𝑘
𝑘=0
при 𝑛 ≥ 0; 𝐴−1 = 0. Тогда, если 0 ≤ 𝑝 ≤ 𝑞, то
𝑞
(20)
∑
𝑞−1
𝑎𝑛 𝑏𝑛 =
𝑛=𝑝
∑
𝐴𝑛 (𝑏𝑛 − 𝑏𝑛+1 ) + 𝐴𝑞 𝑏𝑞 − 𝐴𝑝−1 𝑏𝑝 .
𝑛=𝑝
Доказательство. Имеем
𝑞
∑
𝑞
𝑎𝑛 𝑏𝑛 =
𝑛=𝑝
𝑞
∑
(𝐴𝑛 − 𝐴𝑛−1 )𝑏𝑛 =
𝑛=𝑝
∑
𝑞−1
𝐴𝑛 𝑏𝑛 −
𝑛=𝑝
∑
𝐴𝑛 𝑏𝑛+1 ,
𝑛=𝑝−1
и последнее выражение справа, очевидно, равно правой части равенства (20).
Формула (20), так называемая формула суммирования по частям, полезна при исследовании
рядов вида ∑ 𝑎𝑛 𝑏𝑛 , в особенности если последовательность {𝑏𝑛 } монотонна. Сейчас мы дадим ее
приложения.
3.42 Теорема. Допустим, что
(a) частные суммы 𝐴𝑛 ряда ∑ 𝑎𝑛 образуют ограниченную последовательность;
(b) 𝑏0 ≥ 𝑏1 ≥ 𝑏2 ≥ ⋯ ;
(c) lim 𝑏𝑛 = 0.
𝑛→∞
Тогда ряд ∑ 𝑎𝑛 𝑏𝑛 сходится.
Доказательство. Выберем 𝑀 так, что |𝐴𝑛 | ≤ 𝑀 при всех 𝑛. Для данного 𝜀 > 0 существует целое
𝑁, такое, что 𝑏𝑁 ≤ (𝜀/2𝑀). При 𝑁 ≤ 𝑝 ≤ 𝑞 имеем
|𝑞−1
|
𝑎𝑛 𝑏𝑛 = | 𝐴𝑛 (𝑏𝑛 − 𝑏𝑛+1 ) + 𝐴𝑞 𝑏𝑞 − 𝐴𝑝−1 𝑏𝑝 |
||
|∑
| ||∑
𝑛=𝑝
𝑛=𝑝
𝑞
|𝑞−1
|
≤ 𝑀 | (𝑏𝑛 − 𝑏𝑛+1 ) + 𝑏𝑞 + 𝑏𝑝 |
||∑
||
𝑛=𝑝
= 2𝑀𝑏𝑝 ≤ 2𝑀𝑏𝑁 ≤ 𝜀.
Сходимость следует теперь из критерия Коши. Заметим, что первое неравенство в написанной
выше цепочке основано, конечно, на том, что 𝑏𝑛 − 𝑏𝑛+1 ≥ 0.
3.43 Теорема. Допустим, что
(a) |𝑐1 | ≥ |𝑐2 | ≥ |𝑐3 | ≥ ⋯ ;
(b) 𝑐2𝑚−1 ≥ 0, 𝑐2𝑚 ≤ 0
(𝑚 = 1, 2, 3, … );
(c) lim𝑛→∞ 𝑐𝑛 = 0.
Тогда ряд ∑ 𝑐𝑛 сходится.
Ряд, для которого выполнено (b), называется «знакопеременным рядом»; сформулированная
теорема была известна Лейбницу.
Доказательство. Применим теорему 3.42, положив 𝑎𝑛 = (−1)𝑛+1 , 𝑏𝑛 = |𝑐𝑛 |.
53
3.44 Теорема. Допустим, что радиус сходимости ряда ∑ 𝑐𝑛 𝑧𝑛 равен 1 и 𝑐0 ≥ 𝑐1 ≥ 𝑐2 ≥ ⋯ ,
lim𝑛→∞ 𝑐𝑛 = 0. Тогда ряд ∑ 𝑐𝑛 𝑧𝑛 сходится в каждой точке окружности |𝑧| = 1, за исключением,
возможно, точки 𝑧 = 1.
Доказательство. Положим 𝑎𝑛 = 𝑧𝑛 , 𝑏𝑛 = 𝑐𝑛 . Тогда выполнены условия теоремы 3.42, так как
𝑛
|𝐴𝑛 | =
1 − 𝑧𝑛+1
2
𝑧𝑚 =
≤
,
∑ | | 1 − 𝑧 | |1 − 𝑧|
|𝑚=0
если |𝑧| = 1, 𝑧 ≠ 1.
Абсолютная сходимость
Говорят, что ряд ∑ 𝑎𝑛 сходится абсолютно, если сходится ряд ∑ |𝑎𝑛 |.
3.45 Теорема. Если ряд ∑ 𝑎𝑛 сходится абсолютно, то он сходится.
Доказательство. Утверждение следует из неравенства
𝑚
𝑚
𝑎 ≤
|𝑎 |
∑ 𝑘| ∑ 𝑘
|𝑘=𝑛
𝑘=𝑛
и из критерия Коши.
3.46 Замечания. Для рядов с положительными членами абсолютная сходимость — то же самое,
что сходимость.
Если ряд ∑ 𝑎𝑛 сходится, а ряд ∑ |𝑎𝑛 | расходится, то говорят, что ряд ∑ 𝑎𝑛 сходится неабсолютно.
Например, ряд
(−1)𝑛
∑ 𝑛
сходится неабсолютно (теорема 3.43).
Признак сравнения, так же как и признаки Даламбера и Коши, на самом деле есть признак
абсолютной сходимости, и поэтому не может дать никакой информации о неабсолютно сходящихся
рядах. Для работы с последними иногда можно пользоваться суммированием по частям. В частности, степенной ряд сходится абсолютно внутри круга сходимости.
Мы увидим, что с абсолютно сходящимися рядами можно обращаться практически так же, как
с конечными суммами. Мы можем перемножать их почленно и переставлять слагаемые, не меняя
суммы. Но для неабсолютно сходящихся рядов это уже неверно, и при действиях с ними нужно
больше осторожности.
Сложение и умножение рядов
3.47 Теорема. Если ∑ 𝑎𝑛 = 𝐴 и ∑ 𝑏𝑛 = 𝐵, то ∑(𝑎𝑛 + 𝑏𝑛 ) = 𝐴 + 𝐵 и ∑ 𝑐𝑎𝑛 = 𝑐𝐴 при любом 𝑐.
Доказательство. Пусть
𝑛
𝑛
𝐴𝑛 =
∑
𝑎𝑘 ,
𝐵𝑛 =
∑
𝑏𝑘 .
𝑘=0
𝑘=0
Тогда
𝑛
𝐴𝑛 + 𝐵 𝑛 =
∑
(𝑎𝑘 + 𝑏𝑘 ).
𝑘=0
Так как lim𝑛→∞ 𝐴𝑛 = 𝐴 и lim𝑛→∞ 𝐵𝑛 = 𝐵, ясно, что
lim (𝐴𝑛 + 𝐵𝑛 ) = 𝐴 + 𝐵.
𝑛→∞
Доказательство второго утверждения даже проще.
54
Таким образом, два сходящихся ряда можно сложить почленно, и полученный ряд будет сходиться к сумме этих двух рядов. Положение усложняется при рассмотрении произведения двух
рядов. Сначала мы должны определить произведение. Это можно сделать разными способами; мы
будем рассматривать так называемое «произведение Коши».
3.48 Определение. Пусть заданы ряды ∑ 𝑎𝑛 и ∑ 𝑏𝑛 . Положим
𝑛
𝑐𝑛 =
∑
𝑎𝑘 𝑏𝑛−𝑘
(𝑛 = 0, 1, 2, … )
𝑘=0
и назовем ряд ∑ 𝑐𝑛 произведением двух данных рядов.
Это определение можно объяснить следующим образом. Если, взяв два степенных ряда ∑ 𝑎𝑛 𝑧𝑛
и ∑ 𝑏𝑛 𝑧𝑛 , мы перемножим их почленно и приведем подобные члены при одинаковых степенях 𝑧, то
мы получим
∞
∞
∑
𝑎𝑛 𝑧𝑛 ⋅
𝑛=0
∑
𝑏𝑛 𝑧𝑛 = (𝑎0 + 𝑎1 𝑧 + 𝑎2 𝑧2 + ⋯ )(𝑏0 + 𝑏1 𝑧 + 𝑏2 𝑧2 + ⋯ )
𝑛=0
= 𝑎0 𝑏0 + (𝑎0 𝑏1 + 𝑎1 𝑏0 )𝑧 + (𝑎0 𝑏2 + 𝑎1 𝑏1 + 𝑎2 𝑏0 )𝑧2 + ⋯
= 𝑐0 + 𝑐1 𝑧 + 𝑐2 𝑧2 + ⋯ .
Полагая 𝑧 = 1, мы приходим к данному выше определению.
3.49 Пример. Если
𝑛
𝐴𝑛 =
𝑛
𝑎𝑘 ,
∑
𝐵𝑛 =
𝑘=0
∑
𝑛
𝑏𝑘 ,
𝐶𝑛 =
𝑘=0
∑
𝑐𝑘
𝑘=0
и 𝐴𝑛 → 𝐴, 𝐵𝑛 → 𝐵, то совсем не ясно, будет ли последовательность {𝐶𝑛 } сходиться к 𝐴𝐵, так как
неверно, что 𝐶𝑛 = 𝐴𝑛 𝐵𝑛 . Зависимость {𝐶𝑛 } от {𝐴𝑛 } и {𝐵𝑛 } довольно сложна (см. доказательство
теоремы 3.50). Сейчас мы покажем, что произведение двух сходящихся рядов может на самом деле
расходиться.
Ряд
∞
(−1)𝑛
1
1
1
=1−
+
−
+⋯
∑ √𝑛
√2 √3 √4
+1
𝑛=0
сходится (теорема 3.43). Составив произведение этого ряда с самим собой, мы получим
∞
∑
𝑐𝑛 = 1 −
𝑛=0
1
( √2
+
1
√2 )
+
1
( √3
1
+
+
√2√2
так что
1
√3 )
−
1
( √4
1
+
√3√2
1
+
√2√3
+
1
√4 )
+⋯ ,
𝑛
𝑐𝑛 = (−1)𝑛
1
.
∑ √(𝑛 − 𝑘 + 1)(𝑘 + 1)
𝑘=0
Так как
2
2
2
𝑛
𝑛
𝑛
(𝑛 − 𝑘 + 1)(𝑘 + 1) = ( + 1) − ( − 𝑘) ≤ ( + 1) ,
2
2
2
то мы имеем
𝑛
|𝑐𝑛 | ≥
2(𝑛 + 1)
2
=
,
∑𝑛+2
𝑛+2
𝑘=0
так что условие 𝑐𝑛 → 0, необходимое для сходимости ряда ∑ 𝑐𝑛 , не выполнено.
В связи со следующей теоремой, принадлежащей Мертенсу, заметим, что мы рассматривали в
этом примере произведение двух неабсолютно сходящихся рядов.
3.50 Теорема. Допустим, что
55
∞
(a) ряд
∑
𝑎𝑛 сходится абсолютно,
𝑛=0
∞
(b)
∑
𝑎𝑛 = 𝐴,
∑
𝑏𝑛 = 𝐵,
𝑛=0
∞
(c)
𝑛=0
𝑛
(d) 𝑐𝑛 =
∑
𝑎𝑘 𝑏𝑛−𝑘
(𝑛 = 0, 1, 2, ⋯ ).
𝑘=0
Тогда
∞
∑
𝑐𝑛 = 𝐴𝐵.
𝑛=0
Это значит, что произведение двух сходящихся рядов сходится и сумма равна произведению
сумм, если хотя бы один из этих двух рядов сходится абсолютно.
Доказательство. Положим
𝑛
𝐴𝑛 =
𝑛
∑
𝑘=0
𝑎𝑘 ,
𝐵𝑛 =
𝑛
∑
𝑏𝑘 ,
𝐶𝑛 =
𝑘=0
∑
𝑐𝑘 ,
𝛽𝑛 = 𝐵𝑛 − 𝐵.
𝑘=0
Тогда
𝐶𝑛 = 𝑎0 𝑏0 + (𝑎0 𝑏1 + 𝑎1 𝑏0 ) + ⋯ + (𝑎0 𝑏𝑛 + 𝑎1 𝑏𝑛−1 + ⋯ + 𝑎𝑛 𝑏0 )
= 𝑎0 𝐵𝑛 + 𝑎1 𝐵𝑛−1 + ⋯ + 𝑎𝑛 𝐵0
= 𝑎0 (𝐵 + 𝛽𝑛 ) + 𝑎1 (𝐵 + 𝛽𝑛−1 ) + ⋯ + 𝑎𝑛 (𝐵 + 𝛽0 )
= 𝐴𝑛 𝐵 + 𝑎0 𝛽𝑛 + 𝑎1 𝛽𝑛−1 + ⋯ + 𝑎𝑛 𝛽0 .
Положим
𝛾𝑛 = 𝑎0 𝛽𝑛 + 𝑎1 𝛽𝑛−1 + ⋯ + 𝑎𝑛 𝛽0 .
Мы хотим показать, что 𝐶𝑛 → 𝐴𝐵. Так как 𝐴𝑛 𝐵 → 𝐴𝐵, достаточно показать, что
(21)
lim 𝛾
𝑛→∞ 𝑛
Положим
= 0.
∞
𝛼=
∑
|𝑎𝑛 |
𝑛=0
(именно здесь мы используем (a)). Пусть задано 𝜀 > 0. В силу (c) 𝛽𝑛 → 0. Значит, можно выбрать
𝑁 так, что |𝛽𝑛 | ≤ 𝜀 при 𝑛 ≥ 𝑁, и в этом случае
|𝛾𝑛 | ≤ |𝛽0 𝑎𝑛 + ⋯ + 𝛽𝑁 𝑎𝑛−𝑁 | + |𝛽𝑁+1 𝑎𝑛−𝑁−1 + ⋯ + 𝛽𝑛 𝑎0 |
≤ |𝛽0 𝑎𝑛 + ⋯ + 𝛽𝑁 𝑎𝑛−𝑁 | + 𝜀𝛼.
Оставляя 𝑁 фиксированным и устремляя 𝑛 к ∞, мы получим
lim |𝛾𝑛 | ≤ 𝜀𝛼,
𝑛→∞
так как 𝑎𝑘 → 0 при 𝑘 → ∞. Так как 𝜀 произвольно, отсюда следует (21).
Можно задать другой вопрос: обязательно ли сумма ряда ∑ 𝑐𝑛 , в том случае, когда он сходится,
равна 𝐴𝐵? Абель показал, что ответ на этот вопрос утвердительный.
3.51 Теорема. Если ряды ∑ 𝑎𝑛 , ∑ 𝑏𝑛 , ∑ 𝑐𝑛 сходятся соответственно к 𝐴, 𝐵, 𝐶 и 𝑐𝑛 = 𝑎0 𝑏𝑛 +⋯+𝑎𝑛 𝑏0 ,
то 𝐶 = 𝐴𝐵.
Здесь не делается никаких предположений об абсолютной сходимости. Мы дадим простое доказательство (основанное на непрерывности степенных рядов) после теоремы 8.2.
56
Перестановки рядов
3.52 Определение. Пусть {𝑘𝑛 }, 𝑛 = 1, 2, 3, … , — последовательность, в которой каждое положительное целое число встречается один и только один раз (т. е. {𝑘𝑛 } — взаимно однозначное отображение 𝐽 на 𝐽 , в обозначениях определения 2.2). Положим
𝑎′𝑛 = 𝑎𝑘𝑛
(𝑛 = 1, 2, 3, … );
мы будем говорить, что ряд ∑ 𝑎′𝑛 является перестановкой ряда ∑ 𝑎𝑛 .
Если {𝑠𝑛 }, {𝑠′𝑛 } — последовательности частных сумм рядов ∑ 𝑎𝑛 , ∑ 𝑎′𝑛 , то легко видеть, что,
вообще говоря, эти две последовательности составлены из совершенно разных чисел. Мы, таким
образом, приходим к задаче выяснения, при каких условиях все перестановки сходящегося ряда
сходятся и должны ли совпадать их суммы.
3.53 Пример. Рассмотрим сходящийся ряд
(22)
1−
1
2
+
1
3
−
1
4
+
1
5
−
1
6
+⋯
и одну из его перестановок
(23)
1+
1
3
−
1
2
+
1
5
+
1
7
−
1
4
+
1
9
+
1
11
−
1
6
+⋯ ,
в которой за двумя положительными членами ряда всегда следует отрицательный. Если 𝑠 — сумма
ряда (22), то
𝑠 < 1 − 12 + 13 = 65 .
Так как
1
1
1
+
−
>0
4𝑘 − 3 4𝑘 − 1 2𝑘
при 𝑘 ≥ 1, то мы видим, что 𝑠′3 < 𝑠′6 < 𝑠′9 < ⋯ , где 𝑠′𝑛 есть 𝑛-ая частная сумма ряда (23). Значит,
lim 𝑠′
𝑛→∞ 𝑛
> 𝑠′3 = 56 ,
так что ряд (23) наверняка не сходится к 𝑠 (мы предоставляем читателю проверку того, что ряд
(23) тем не менее сходится).
Этот пример иллюстрирует следующую теорему, принадлежащую Риману.
3.54 Теорема. Пусть ∑ 𝑎𝑛 — неабсолютно сходящийся ряд вещественных чисел, и пусть
−∞ ≤ 𝛼 ≤ 𝛽 ≤ ∞.
Тогда существует перестановка ∑ 𝑎′𝑛 с частными суммами 𝑠′𝑛 , такая, что
lim 𝑠′𝑛 = 𝛼,
(24)
lim 𝑠′
𝑛→∞ 𝑛
𝑛→∞
= 𝛽.
Доказательство. Пусть
𝑝𝑛 =
|𝑎𝑛 | + 𝑎𝑛
,
2
𝑞𝑛 =
|𝑎𝑛 | − 𝑎𝑛
2
(𝑛 = 1, 2, 3, … ).
Тогда 𝑝𝑛 − 𝑞𝑛 = 𝑎𝑛 , 𝑝𝑛 + 𝑞𝑛 = |𝑎𝑛 |, 𝑝𝑛 ≥ 0, 𝑞𝑛 ≥ 0. Ряды ∑ 𝑝𝑛 , ∑ 𝑞𝑛 оба должны расходиться.
Действительно, если бы оба эти ряда сходились, то и ряд
(𝑝 + 𝑞𝑛 ) =
|𝑎 |
∑ 𝑛
∑ 𝑛
сходился бы, вопреки предположению. Так как
𝑁
∑
𝑛=1
𝑁
𝑎𝑛 =
𝑁
𝑁
(𝑝 − 𝑞𝑛 ) =
𝑝 −
𝑞 ,
∑ 𝑛
∑ 𝑛 ∑ 𝑛
𝑛=1
𝑛=1
57
𝑛=1
расходимость ряда ∑ 𝑝𝑛 и сходимость ряда ∑ 𝑞𝑛 (или наоборот) влечет за собой расходимость ряда
∑ 𝑎𝑛 , что снова противоречит предположению.
Обозначим теперь через 𝑃1 , 𝑃2 , 𝑃3 , … неотрицательные члены ряда ∑ 𝑎𝑛 в том порядке, в каком
они встречаются, и пусть 𝑄1 , 𝑄2 , 𝑄3 , … — абсолютные величины отрицательных членов ряда ∑ 𝑎𝑛
также в их естественном порядке.
Ряды ∑ 𝑃𝑛 , ∑ 𝑄𝑛 отличаются от рядов ∑ 𝑝𝑛 , ∑ 𝑞𝑛 только нулевыми членами и поэтому расходятся.
Мы построим такие последовательности {𝑚𝑛 }, {𝑘𝑛 }, что ряд
(25)
𝑃1 + ⋯ + 𝑃𝑚1 − 𝑄1 − ⋯ − 𝑄𝑘1 + 𝑃𝑚1 +1 + ⋯ + 𝑃𝑚2 − 𝑄𝑘1 +1 − ⋯ − 𝑄𝑘2 + ⋯ ,
очевидно, являющийся перестановкой ряда ∑ 𝑎𝑛 , удовлетворяет условию (24).
Выберем последовательности вещественных чисел {𝛼𝑛 }, {𝛽𝑛 } так, что 𝛼𝑛 → 𝛼, 𝛽𝑛 → 𝛽, 𝛼𝑛 < 𝛽𝑛 ,
𝛽1 > 0.
Пусть 𝑚1 , 𝑘1 — наименьшие из таких целых чисел, что
𝑃1 + ⋯ + 𝑃𝑚1 > 𝛽1 ,
𝑃1 + ⋯ + 𝑃𝑚1 − 𝑄1 − ⋯ − 𝑄𝑘1 < 𝛼1 ;
пусть 𝑚2 , 𝑘2 — наименьшие из таких целых чисел, что
𝑃1 + ⋯ + 𝑃𝑚1 − 𝑄1 − ⋯ − 𝑄𝑘1 + 𝑃𝑚1 +1 + ⋯ + 𝑃𝑚2 > 𝛽2 ,
𝑃1 + ⋯ + 𝑃𝑚1 − 𝑄1 − ⋯ − 𝑄𝑘1 + 𝑃𝑚1 +1 + ⋯ + 𝑃𝑚2 − 𝑄𝑘1 +1 − ⋯ − 𝑄𝑘2 < 𝛼2
и т. д. Мы можем продолжить этот выбор, так как ряды ∑ 𝑃𝑛 и ∑ 𝑄𝑛 расходятся.
Если через 𝑥𝑛 , 𝑦𝑛 обозначить частные суммы ряда (25), последние члены которых 𝑃𝑚𝑛 , −𝑄𝑘𝑛 , то
|𝑥𝑛 − 𝛽𝑛 | ≤ 𝑃𝑚𝑛 ,
|𝑦𝑛 − 𝛼𝑛 | ≤ 𝑄𝑘𝑛 .
Так как 𝑃𝑛 → 0 и 𝑄𝑛 → 0 при 𝑛 → ∞, мы видим, что 𝑥𝑛 → 𝛽, 𝑦𝑛 → 𝛼.
Наконец, ясно, что никакое число, меньшее чем 𝛼 или большее чем 𝛽, не может быть частичным
пределом последовательности частных сумм ряда (25).
3.55 Теорема. Если ∑ 𝑎𝑛 — ряд комплексных чисел, сходящийся абсолютно, то все его перестановки сходятся, причем к одной и той же сумме.
Доказательство. Пусть ∑ 𝑎′𝑛 — перестановка, обладающая частными суммами 𝑠′𝑛 . Для данного
𝜀 > 0 существует целое 𝑁, такое, что при 𝑚 ≥ 𝑛 ≥ 𝑁 имеем
𝑚
(26)
∑
|𝑎𝑖 | ≤ 𝜀.
𝑖=𝑛
Выберем теперь 𝑝 так, чтобы все целые числа 1, 2, … , 𝑁 содержались во множестве 𝑘1 , 𝑘2 , … , 𝑘𝑝
(мы используем обозначения из определения 3.52). Тогда при 𝑛 > 𝑝 числа 𝑎1 , … , 𝑎𝑁 уничтожаются
в разности 𝑠𝑛 − 𝑠′𝑛 , так что |𝑠𝑛 − 𝑠′𝑛 | ≤ 𝜀 в силу (26). Значит, последовательность {𝑠′𝑛 } сходится к той
же сумме, что и {𝑠𝑛 }.
Упражнения
1. Доказать, что из сходимости последовательности {𝑠𝑛 } следует сходимость последовательности {|𝑠𝑛 |}.
Верно ли обратное?
2. Вычислить lim (√𝑛2 + 𝑛 − 𝑛).
𝑛→∞
3. Пусть 𝑠1 = √2 и
𝑠𝑛+1 = √2 + √𝑠𝑛
(𝑛 = 1, 2, 3, … ).
Доказать, что последовательность {𝑠𝑛 } сходится и что 𝑠𝑛 < 2 при 𝑛 = 1, 2, 3, … .
58
4. Найти верхний и нижний пределы последовательности {𝑠𝑛 }, определенной следующим образом:
𝑠1 = 0;
𝑠2𝑚 =
𝑠2𝑚−1
;
2
𝑠2𝑚+1 =
1
+ 𝑠2𝑚 .
2
5. Доказать, что для любых последовательностей вещественных чисел {𝑎𝑛 }, {𝑏𝑛 } выполняется
lim (𝑎𝑛 + 𝑏𝑛 ) ≤ lim 𝑎𝑛 + lim 𝑏𝑛 ,
𝑛→∞
𝑛→∞
𝑛→∞
если только сумма в правой части не имеет вид ∞ − ∞.
6. Исследовать поведение (сходимость или расходимость) ряда ∑ 𝑎𝑛 , если
(a) 𝑎𝑛 = √𝑛 + 1 − √𝑛;
√𝑛 + 1 − √𝑛
(b) 𝑎𝑛 =
;
𝑛
𝑛
𝑛
(c) 𝑎𝑛 = (√𝑛 − 1) ;
1
(d) 𝑎𝑛 =
для комплексных значений 𝑧.
1 + 𝑧𝑛
7. Доказать, что из сходимости ряда ∑ 𝑎𝑛 следует сходимость ряда
√𝑎𝑛
,
∑ 𝑛
если 𝑎𝑛 ≥ 0.
8. Если ряд ∑ 𝑎𝑛 сходится и если {𝑏𝑛 } — монотонная и ограниченная последовательность, доказать, что
ряд ∑ 𝑎𝑛 𝑏𝑛 сходится.
9. Найти радиус сходимости каждого из следующих степенных рядов:
(a) ∑ 𝑛3 𝑧𝑛 ;
2𝑛
(b) ∑ 𝑧𝑛 ;
𝑛!
2𝑛 𝑛
(c) ∑ 2 𝑧 ;
𝑛
𝑛3
(d) ∑ 𝑛 𝑧𝑛 .
3
10. Допустим, что коэффициенты степенного ряда ∑ 𝑎𝑛 𝑧𝑛 — целые числа, среди которых бесконечно много
отличных от нуля. Доказать, что радиус сходимости не превышает 1.
11. Допустим, что 𝑎𝑛 > 0, 𝑠𝑛 = 𝑎1 + ⋯ + 𝑎𝑛 и ряд ∑ 𝑎𝑛 расходится.
𝑎𝑛
(a) Доказать, что ряд ∑
расходится.
1 + 𝑎𝑛
(b) Доказать, что
𝑎𝑁+1
𝑎𝑁+𝑘
𝑠
+⋯+
≥1− 𝑁
𝑠𝑁+1
𝑠𝑁+𝑘
𝑠𝑁+𝑘
𝑎
и сделать вывод, что ряд ∑ 𝑛 расходится.
𝑠𝑛
(c) Доказать, что
𝑎𝑛
1
1
≤
−
𝑠𝑛−1 𝑠𝑛
𝑠2𝑛
𝑎
и сделать вывод, что ряд ∑ 2𝑛 сходится.
𝑠𝑛
(d) Что можно сказать о рядах
𝑎𝑛
𝑎𝑛
и
?
∑ 1 + 𝑛2 𝑎𝑛
∑ 1 + 𝑛𝑎𝑛
12. Допустим, что 𝑎𝑛 > 0 и ряд ∑ 𝑎𝑛 сходится. Положим
∞
𝑟𝑛 =
∑
𝑎𝑚 .
𝑚=𝑛
(a) Доказать, что
𝑎𝑚
𝑎
𝑟
+⋯+ 𝑛 >1− 𝑛
𝑟𝑚
𝑟𝑛
𝑟𝑚
𝑎
при 𝑚 < 𝑛 и сделать вывод, что ряд ∑ 𝑛 расходится.
𝑟𝑛
59
(b) Доказать, что
𝑎𝑛
√𝑟𝑛
и сделать вывод, что ряд ∑
< 2(√𝑟𝑛 − √𝑟𝑛+1 )
𝑎𝑛
сходится.
√𝑟𝑛
13. Доказать, что произведение Коши двух абсолютно сходящихся рядов сходится абсолютно.
14. Пусть {𝑠𝑛 } — последовательность комплексных чисел. Рассмотрим средние арифметические
𝜎𝑛 =
(a)
(b)
(c)
(d)
𝑠0 + 𝑠1 + ⋯ + 𝑠𝑛
𝑛+1
(𝑛 = 0, 1, 2, … ).
Если lim 𝑠𝑛 = 𝑠, доказать, что lim 𝜎𝑛 = 𝑠.
Построить последовательность {𝑠𝑛 }, которая расходится, хотя lim 𝜎𝑛 = 0.
Возможно ли, что 𝑠𝑛 > 0 для всех 𝑛 и lim 𝑠𝑛 = ∞, хотя lim 𝜎𝑛 = 0?
Пусть 𝑎𝑛 = 𝑠𝑛 − 𝑠𝑛−1 при 𝑛 ≥ 1. Показать, что
𝑛
𝑠𝑛 − 𝜎 𝑛 =
1
𝑘𝑎𝑘 .
𝑛+1 ∑
𝑘=1
Предположим, что lim(𝑛𝑎𝑛 ) = 0 и что последовательность {𝜎𝑛 } сходится. Доказать, что последовательность {𝑠𝑛 } сходится (это дает результат, обратный (a), при дополнительном условии 𝑛𝑎𝑛 → 0).
(e) Вывести последнее заключение из более слабого предположения. Пусть 𝑀 < ∞, |𝑛𝑎𝑛 | ≤ 𝑀 для
всех 𝑛 и lim 𝜎𝑛 = 𝜎. Доказать, что lim 𝑠𝑛 = 𝜎, пользуясь следующим планом.
Если 𝑚 < 𝑛, то
𝑛
𝑚+1
1
𝑠𝑛 − 𝜎𝑛 =
(𝜎𝑛 − 𝜎𝑚 ) +
(𝑠 − 𝑠𝑖 ).
𝑛−𝑚
𝑛−𝑚 ∑ 𝑛
𝑖=𝑚+1
Для этих 𝑖 выполняется
(𝑛 − 𝑖)𝑀
(𝑛 − 𝑚 − 1)𝑀
≤
.
𝑖+1
𝑚+2
Зафиксируем 𝜀 > 0 и поставим в соответствие каждому 𝑛 целое число 𝑚, удовлетворяющее неравенству
𝑛−𝜀
< 𝑚 + 1.
𝑚≤
1+𝜀
Тогда (𝑚 + 1)/(𝑛 − 𝑚) ≤ 1/𝜀 и |𝑠𝑛 − 𝑠𝑖 | < 𝑀𝜀. Следовательно,
|𝑠𝑛 − 𝑠𝑖 | ≤
lim |𝑠𝑛 − 𝜎| ≤ 𝑀𝜀.
𝑛→∞
Так как 𝜀 было произвольным, lim 𝑠𝑛 = 𝜎.
15. Определение 3.21 можно распространить на тот случай, когда 𝑎𝑛 принадлежат некоторому фиксированному пространству 𝑅𝑘 . Абсолютная сходимость определяется как сходимость ряда ∑ |a𝑛 |. Показать,
что теоремы 3.22, 3.23, 3.25(a), 3.33, 3.34, 3.42, 3.45, 3.47 и 3.55 остаются верными в этом более общем
случае (доказательства требуют лишь незначительных изменений).
16. Зафиксируем положительное 𝛼. Выберем 𝑥1 > √𝛼 и определим 𝑥2 , 𝑥3 , 𝑥4 , … рекуррентной формулой
𝑥𝑛+1 =
1
𝛼
𝑥 +
.
2 ( 𝑛 𝑥𝑛 )
(a) Доказать, что последовательность {𝑥𝑛 } монотонно убывает и что lim 𝑥𝑛 = √𝛼.
(b) Положим 𝜀𝑛 = 𝑥𝑛 − √𝛼. Показать, что
𝜀𝑛+1 =
𝜀2𝑛
𝜀2
< 𝑛 ,
2𝑥𝑛
2√𝛼
так что, обозначая 𝛽 = 2√𝛼, имеем
𝜀𝑛+1 < 𝛽
𝜀1 2
(𝛽)
𝑛
(𝑛 = 1, 2, 3, … ).
(c) Это хороший алгоритм вычисления квадратных корней, так как рекуррентная формула проста,
1
а сходимость исключительно быстрая. Например, при 𝛼 = 3 и 𝑥1 = 2 показать, что 𝜀1 /𝛽 < 10
и,
следовательно,
𝜀5 < 4 ⋅ 10−16 ,
𝜀6 < 4 ⋅ 10−32 .
60
17. Зафиксируем 𝛼 > 1. Возьмем 𝑥1 > √𝛼 и определим
𝑥𝑛+1 =
(a)
(b)
(c)
(d)
𝛼 + 𝑥𝑛
𝛼 − 𝑥2𝑛
= 𝑥𝑛 +
.
1 + 𝑥𝑛
1 + 𝑥𝑛
Доказать, что 𝑥1 > 𝑥3 > 𝑥5 > ⋯ .
Доказать, что 𝑥2 < 𝑥4 < 𝑥6 < ⋯ .
Доказать, что lim 𝑥𝑛 = √𝛼.
Сравнить скорость сходимости данного процесса с аналогичным процессом в упражнении 16.
18. Заменить рекуррентную формулу в упражнении 16 на
𝑥𝑛+1 =
𝑝−1
𝛼 −𝑝+1
𝑥 + 𝑥𝑛 ,
𝑝 𝑛 𝑝
где 𝑝 — фиксированное положительное целое число, и описать поведение получающейся последовательности {𝑥𝑛 }.
19. Поставим в соответствие каждой последовательности 𝑎 = {𝛼𝑛 }, в которой 𝛼𝑛 равны либо 0, либо 2,
вещественное число
∞
𝛼𝑛
.
𝑥(𝑎) =
∑ 3𝑛
𝑛=1
Доказать, что множество всех 𝑥(𝑎) в точности совпадает с множеством Кантора, описанным в пункте
2.44.
20. Пусть {𝑝𝑛 } — последовательность Коши в метрическом пространстве 𝑋, и какая-нибудь ее подпоследовательность {𝑝𝑛𝑖 } сходится к точке 𝑝 ∈ 𝑋. Доказать, что вся последовательность {𝑝𝑛 } сходится к 𝑝.
21. Доказать следующий аналог теоремы 3.10(b): если {𝐸𝑛 } — последовательность замкнутых непустых и
ограниченных множеств в полном метрическом пространстве 𝑋, если 𝐸𝑛 ⊃ 𝐸𝑛+1 и если
lim diam 𝐸𝑛 = 0,
𝑛→∞
∞
то ⋂1 𝐸𝑛 состоит ровно из одной точки.
22. Пусть 𝑋 — непустое полное метрическое пространство, а {𝐺𝑛 } — последовательность всюду плотных от∞
крытых подмножеств пространства 𝑋. Доказать теорему Бэра, состоящую в том, что ⋂1 𝐺𝑛 непусто (на
самом деле оно всюду плотно в 𝑋). Указание: Найти стягивающуюся последовательность окрестностей
𝐸𝑛 , таких, что 𝐸𝑛̄ ⊂ 𝐺𝑛 , и применить упражнение 21.
23. Пусть {𝑝𝑛 } и {𝑞𝑛 } — последовательности Коши в метрическом пространстве 𝑋. Показать, что последовательность {𝑑(𝑝𝑛 , 𝑞𝑛 )} сходится. Указание: Для любых 𝑚, 𝑛 имеем
𝑑(𝑝𝑛 , 𝑞𝑛 ) ≤ 𝑑(𝑝𝑛 , 𝑝𝑚 ) + 𝑑(𝑝𝑚 , 𝑞𝑚 ) + 𝑑(𝑞𝑚 , 𝑞𝑛 );
следовательно,
|𝑑(𝑝𝑛 , 𝑞𝑛 ) − 𝑑(𝑝𝑚 , 𝑞𝑚 )|
мало, если 𝑚, 𝑛 велики.
24. Пусть 𝑋 — метрическое пространство.
(a) Назовем две последовательности Коши {𝑝𝑛 }, {𝑞𝑛 } в 𝑋 эквивалентными, если
lim 𝑑(𝑝𝑛 , 𝑞𝑛 ) = 0.
𝑛→∞
Доказать, что этим определено отношение эквивалентности.
(b) Пусть 𝑋 ∗ — множество всех полученных таким образом классов эквивалентности. Если 𝑃 ∈ 𝑋 ∗ ,
𝑄 ∈ 𝑋 ∗ , {𝑝𝑛 } ∈ 𝑃 , {𝑞𝑛 } ∈ 𝑄, определим
Δ(𝑃 , 𝑄) = lim 𝑑(𝑝𝑛 , 𝑞𝑛 );
𝑛→∞
этот предел существует в силу упражнения 23. Показать, что число Δ(𝑃 , 𝑄) не изменится, если
{𝑝𝑛 } и {𝑞𝑛 } заменить эквивалентными последовательностями, и что тем самым Δ — расстояние на
𝑋∗.
(c) Доказать, что полученное метрическое пространство 𝑋 ∗ полно.
(d) Для каждого 𝑝 ∈ 𝑋 возьмем последовательность Коши, все члены которой равны 𝑝; пусть 𝑃𝑝 —
элемент множества 𝑋 ∗ , содержащий эту последовательность. Доказать, что
Δ(𝑃𝑝 , 𝑃𝑞 ) = 𝑑(𝑝, 𝑞)
при всех 𝑝, 𝑞 ∈ 𝑋. Иными словами, отображение 𝜑, заданное как 𝜑(𝑝) = 𝑃𝑝 , есть изометрия (т.е.
отображение, сохраняющее расстояния) пространства 𝑋 в 𝑋 ∗ .
61
(e) Доказать, что 𝜑(𝑋) всюду плотно в 𝑋 ∗ и что 𝜑(𝑋) = 𝑋 ∗ , если 𝑋 полно. В силу (d) можно отождествить 𝑋 с 𝜑(𝑋) и, таким образом, считать, что 𝑋 погружено в полное метрическое пространство
𝑋 ∗ . Мы будем называть 𝑋 ∗ пополнением пространства 𝑋.
25. Пусть 𝑋 — метрическое пространство, точками которого являются рациональные числа, с метрикой
𝑑(𝑥, 𝑦) = |𝑥 − 𝑦|. Что служит пополнением этого пространства (ср. с упражнением 24)?
62
Глава 4
Непрерывность
Понятие функции и связанная с ним терминология были введены в определениях 2.1 и 2.2. Хотя в
последующих главах мы будем в основном интересоваться вещественными и комплексными функциями (т. е. функциями, значения которых — вещественные или комплексные числа), мы будем
рассматривать также векторнозначные функции (т. е. функции со значениями в 𝑅𝑘 ) и функции со
значениями в любом метрическом пространстве. Теоремы, которые мы будем доказывать в этой
общей обстановке, ничуть не стали бы более легкими, если бы мы ограничились, например, вещественными функциями; наоборот, на самом деле картина упростится и прояснится, если мы
отбросим излишние предположения и будем формулировать и доказывать теоремы в разумной
общности.
Наши функции будут определены в метрических пространствах, выбор которых будет надлежащим образом уточняться в различных примерах.
Предел функции
4.1 Определение. Пусть 𝑋 и 𝑌 — метрические пространства; пусть 𝐸 ⊂ 𝑋, 𝑓 отображает 𝐸 в 𝑌 ,
а 𝑝 — предельная точка множества 𝐸. Мы будем писать «𝑓 (𝑥) → 𝑞 при 𝑥 → 𝑝», или
(1)
lim 𝑓 (𝑥) = 𝑞,
𝑥→𝑝
если существует точка 𝑞 ∈ 𝑌 , обладающая следующим свойством: для любого 𝜀 > 0 существует
𝛿 > 0, такое, что
(2)
𝑑𝑌 (𝑓 (𝑥), 𝑞) < 𝜀
для любых точек 𝑥 ∈ 𝐸, для которых
(3)
0 < 𝑑𝑋 (𝑥, 𝑝) < 𝛿.
Символы 𝑑𝑋 и 𝑑𝑌 относятся соответственно к расстояниям в пространствах 𝑋 и 𝑌 .
Если 𝑋 и (или) 𝑌 заменяются вещественной прямой, комплексной плоскостью или каким-нибудь
евклидовым пространством 𝑅𝑘 , то расстояния 𝑑𝑋 , 𝑑𝑌 , конечно, заменяются абсолютными величинами или нормами разностей (см. п. 2.16).
Следует отметить, что 𝑝 ∈ 𝑋, но точка 𝑝 не обязана принадлежать множеству 𝐸 в предыдущем
определении. Более того, даже если 𝑝 ∈ 𝐸, то вполне возможно, что 𝑓 (𝑝) ≠ lim𝑥→𝑝 𝑓 (𝑥).
Этому определению можно придать другую форму, высказав его в терминах последовательностей.
4.2 Теорема. Пусть 𝑋, 𝑌 , 𝐸, 𝑓 и 𝑝 — те же, что и в определении 4.1. Тогда
(4)
lim 𝑓 (𝑥) = 𝑞
𝑥→𝑝
в том и только в том случае, когда
(5)
lim 𝑓 (𝑝𝑛 ) = 𝑞
𝑛→∞
63
для любой последовательности {𝑝𝑛 } в 𝐸, такой, что
(6)
lim 𝑝
𝑛→∞ 𝑛
𝑝𝑛 ≠ 𝑝,
= 𝑝.
Доказательство. Допустим, что выполнено (4). Выберем последовательность {𝑝𝑛 } в 𝐸, удовлетворяющую (6). Пусть дано 𝜀 > 0. Тогда существует 𝛿 > 0, такое, что 𝑑𝑌 (𝑓 (𝑥), 𝑞) < 𝜀, если 𝑥 ∈ 𝐸 и
0 < 𝑑𝑋 (𝑥, 𝑝) < 𝛿. Кроме того, существует 𝑁, такое, что при 𝑛 > 𝑁 имеем 0 < 𝑑𝑋 (𝑝𝑛 , 𝑝) < 𝛿. Таким
образом, при 𝑛 > 𝑁 имеем 𝑑𝑌 (𝑓 (𝑝𝑛 ), 𝑞) < 𝜀, откуда следует, что выполнено (5).
Обратно, допустим, что (4) неверно. Тогда существует 𝜀 > 0, такое, что для каждого 𝛿 > 0
найдется точка 𝑥 ∈ 𝐸 (зависящая от 𝛿), для которой 𝑑𝑌 (𝑓 (𝑥), 𝑞) ≥ 𝜀, но 0 < 𝑑𝑋 (𝑥, 𝑝) < 𝛿. Выбирая
𝛿𝑛 = 1/𝑛 (𝑛 = 1, 2, 3, … ), мы найдем последовательность в 𝐸, удовлетворяющую (6), для которой (5)
не выполняется.
Следствие. Если 𝑓 имеет предел в 𝑝, то этот предел единственный.
Это следует из теорем 3.2(b) и 4.2.
4.3 Определение. Пусть на 𝐸 определены две комплексные функции 𝑓 и 𝑔. Символом 𝑓 + 𝑔
обозначается функция, сопоставляющая каждой точке 𝑥 множества 𝐸 число 𝑓 (𝑥) + 𝑔(𝑥). Подобным
образом мы определим разность 𝑓 − 𝑔, произведение 𝑓 𝑔 и отношение 𝑓 /𝑔 двух функций, имея в
виду, что отношение определено лишь в тех точках 𝑥 множества 𝐸, в которых 𝑔(𝑥) ≠ 0. Если 𝑓
сопоставляет каждой точке 𝑥 множества 𝐸 одно и то же число 𝑐, то 𝑓 называется постоянной функцией, или просто постоянной, и мы будем писать в этом случае 𝑓 = 𝑐. Если 𝑓 и 𝑔 — вещественные
функции и если 𝑓 (𝑥) ≥ 𝑔(𝑥) при каждом 𝑥 ∈ 𝐸, то мы иногда для краткости будем писать 𝑓 ≥ 𝑔.
Аналогично, если f и g отображают 𝐸 в 𝑅𝑘 , то мы определим f + g и f ⋅ g равенствами
(f + g)(𝑥) = f(𝑥) + g(𝑥),
(f ⋅ g)(𝑥) = f(𝑥) ⋅ g(𝑥),
и если 𝜆 — вещественное число, то (𝜆f)(𝑥) = 𝜆f(𝑥).
4.4 Теорема. Пусть 𝑋 — метрическое пространство, 𝐸 ⊂ 𝑋, 𝑝 — предельная точка множества
𝐸, 𝑓 и 𝑔 — комплексные функции на 𝐸 и
lim 𝑓 (𝑥) = 𝐴,
lim 𝑔(𝑥) = 𝐵.
𝑥→𝑝
𝑥→𝑝
Тогда
(a) lim (𝑓 + 𝑔)(𝑥) = 𝐴 + 𝐵;
𝑥→𝑝
(b) lim (𝑓 𝑔)(𝑥) = 𝐴𝐵;
𝑥→𝑝
(c) lim
𝑓
𝑥→𝑝 ( 𝑔
)
(𝑥) =
𝐴
, если 𝐵 ≠ 0.
𝐵
Доказательство. Эти утверждения, в силу теоремы 4.2, немедленно следуют из аналогичных
свойств последовательностей (теорема 3.3).
Замечание. Если f и g отображают 𝐸 в 𝑅𝑘 , то (a) остается верным, а (b) принимает вид
(b’) lim (f ⋅ g)(𝑥) = A ⋅ B
𝑥→𝑝
(ср. с теоремой 3.4).
Непрерывные функции
4.5 Определение. Пусть 𝑋 и 𝑌 — метрические пространства, 𝐸 ⊂ 𝑋, 𝑝 ∈ 𝐸 и 𝑓 отображает 𝐸 в
𝑌 . Тогда функция 𝑓 называется непрерывной в 𝑝, если для любого 𝜀 > 0 существует 𝛿 > 0, такое,
что
𝑑𝑌 (𝑓 (𝑥), 𝑓 (𝑝)) < 𝜀
64
для всех точек 𝑥 ∈ 𝐸, для которых 𝑑𝑋 (𝑥, 𝑝) < 𝛿.
Если 𝑓 непрерывна в каждой точке множества 𝐸, то 𝑓 называется непрерывной на 𝐸.
Для того чтобы быть непрерывной в точке 𝑝, 𝑓 должна быть определена в этой точке (ср. это
с замечанием, следующим за определением 4.1).
Если 𝑝 — изолированная точка множества 𝐸, то из нашего определения следует, что каждая
функция 𝑓 , область определения которой включает 𝐸, непрерывна в 𝑝. Действительно, каково бы
ни было 𝜀 > 0, можно указать 𝛿 > 0, такое, что единственной точкой 𝑥 ∈ 𝐸, для которой 𝑑𝑋 (𝑥, 𝑝) < 𝛿,
окажется 𝑥 = 𝑝; тогда
𝑑𝑌 (𝑓 (𝑥), 𝑓 (𝑝)) = 0 < 𝜀.
4.6 Теорема. В ситуации, описанной в определении 4.5, предположим, что 𝑝 — предельная точка
множества 𝐸. Функция 𝑓 непрерывна в точке 𝑝 тогда и только тогда, когда lim𝑥→𝑝 𝑓 (𝑥) = 𝑓 (𝑝).
Доказательство. Это становится ясным, если сравнить определения 4.1 и 4.5.
Теперь перейдем к сложным функциям. Краткая формулировка следующей теоремы такова:
непрерывная функция от непрерывной функции непрерывна.
4.7 Теорема. Пусть 𝑋, 𝑌 , 𝑍 — метрические пространства, 𝐸 ⊂ 𝑋, 𝑓 отображает 𝐸 в 𝑌 , 𝑔
отображает множество значений 𝑓 , а именно 𝑓 (𝐸), в 𝑍 и ℎ — отображение 𝐸 в 𝑍, определенное
равенством
ℎ(𝑥) = 𝑔(𝑓 (𝑥))
(𝑥 ∈ 𝐸).
Если 𝑓 непрерывно в точке 𝑝 ∈ 𝐸, а 𝑔 непрерывно в точке 𝑓 (𝑝), то ℎ непрерывно в 𝑝.
Эта функция ℎ называется сложной функцией или композицией 𝑓 и 𝑔. В этой связи часто
используется обозначение
ℎ = 𝑔 ∘ 𝑓.
Доказательство. Пусть дано 𝜀 > 0. Поскольку 𝑔 непрерывно в 𝑓 (𝑝), существует 𝜂 > 0, такое, что
𝑑𝑍 (𝑔(𝑦), 𝑔(𝑓 (𝑝))) < 𝜀, если 𝑑𝑌 (𝑦, 𝑓 (𝑝)) < 𝜂 и 𝑦 ∈ 𝑓 (𝐸).
Поскольку 𝑓 непрерывно в 𝑝, существует 𝛿 > 0, такое, что
𝑑𝑌 (𝑓 (𝑥), 𝑓 (𝑝)) < 𝜂, если 𝑑𝑋 (𝑥, 𝑝) < 𝛿 и 𝑥 ∈ 𝐸.
Следовательно,
𝑑𝑍 (ℎ(𝑥), ℎ(𝑝)) = 𝑑𝑍 (𝑔(𝑓 (𝑥)), 𝑔(𝑓 (𝑝))) < 𝜀,
если 𝑑𝑋 (𝑥, 𝑝) < 𝛿 и 𝑥 ∈ 𝐸. Таким образом, ℎ непрерывно в 𝑝.
4.8 Теорема. Отображение 𝑓 метрического пространства 𝑋 в метрическое пространство 𝑌
непрерывно на 𝑋 тогда и только тогда, когда 𝑓 −1 (𝑉 ) открыто в 𝑋 для каждого открытого
множества 𝑉 пространства 𝑌 .
(Прообразы были определены в п. 2.2.) Это очень полезное свойство, характеризующее непрерывность.
Доказательство. Допустим, что 𝑓 непрерывно на 𝑋; пусть 𝑉 — открытое множество в 𝑌 . Мы
должны показать, что каждая точка множества 𝑓 −1 (𝑉 ) является его внутренней точкой. Итак,
пусть 𝑝 ∈ 𝑋 и 𝑓 (𝑝) ∈ 𝑉 . Поскольку 𝑉 открыто, существует 𝜀 > 0, такое, что 𝑦 ∈ 𝑉 , если 𝑑𝑌 (𝑓 (𝑝), 𝑦) <
𝜀, а так как 𝑓 непрерывно в 𝑝, то существует 𝛿 > 0, такое, что 𝑑𝑌 (𝑓 (𝑥), 𝑓 (𝑝)) < 𝜀, если 𝑑𝑋 (𝑥, 𝑝) < 𝛿.
Таким образом, 𝑥 ∈ 𝑓 −1 (𝑉 ), как только 𝑑𝑋 (𝑥, 𝑝) < 𝛿.
Обратно, допустим, что 𝑓 −1 (𝑉 ) открыто в 𝑋, каково бы ни было открытое множество 𝑉 в 𝑌 .
Зафиксируем 𝑝 ∈ 𝑋 и 𝜀 > 0, и пусть 𝑉 — множество всех 𝑦 ∈ 𝑌 , таких, что 𝑑𝑌 (𝑦, 𝑓 (𝑝)) < 𝜀. Тогда 𝑉
открыто, а поэтому и 𝑓 −1 (𝑉 ) открыто; значит, существует 𝛿 > 0, такое, что 𝑥 ∈ 𝑓 −1 (𝑉 ), как только
𝑑𝑋 (𝑝, 𝑥) < 𝛿. Но если 𝑥 ∈ 𝑓 −1 (𝑉 ), то 𝑓 (𝑥) ∈ 𝑉 , так что 𝑑𝑌 (𝑓 (𝑥), 𝑓 (𝑝)) < 𝜀.
Доказательство закончено.
65
Следствие. Отображение 𝑓 метрического пространства 𝑋 в метрическое пространство 𝑌
непрерывно тогда и только тогда, когда 𝑓 −1 (𝐶) замкнуто в 𝑋 для каждого замкнутого множества 𝐶 пространства 𝑌 .
Это следует из теоремы, так как множество замкнуто тогда и только тогда, когда его дополнение открыто, и так как 𝑓 −1 (𝐸 𝑐 ) = [𝑓 −1 (𝐸)]𝑐 для каждого 𝐸 ⊂ 𝑌 .
Обратимся теперь к комплекснозначным и векторнозначным функциям и к функциям, определенным на подмножествах пространства 𝑅𝑘 .
4.9 Теорема. Пусть 𝑓 и 𝑔 — комплексные непрерывные функции на метрическом пространстве
𝑋. Тогда 𝑓 + 𝑔, 𝑓 𝑔 и 𝑓 /𝑔 непрерывны на 𝑋.
В последнем случае мы должны, конечно, предполагать, что 𝑔(𝑥) ≠ 0 при всех 𝑥 ∈ 𝑋.
Доказательство. Для изолированных точек доказывать нечего. Для предельных точек утверждение следует из теорем 4.4 и 4.6.
4.10 Теорема.
(a) Пусть 𝑓1 , … , 𝑓𝑘 — вещественные функции на метрическом пространстве 𝑋, и пусть f —
отображение пространства 𝑋 в 𝑅𝑘 , определенное равенством
(7)
f(𝑥) = (𝑓1 (𝑥), … , 𝑓𝑘 (𝑥))
(𝑥 ∈ 𝑋).
Отображение f непрерывно тогда и только тогда, когда каждая из функций 𝑓1 , … , 𝑓𝑘 непрерывна.
(b) Если f и g — непрерывные отображения пространства 𝑋 в 𝑅𝑘 , то f + g и f ⋅ g непрерывны
на 𝑋.
Функции 𝑓1 , … , 𝑓𝑘 называются компонентами отображения f. Отметим, что f+g — отображение
в 𝑅𝑘 , тогда как f ⋅ g — вещественная функция на 𝑋.
Доказательство. Часть (a) следует из неравенств
1
𝑘
2
⎧
⎫
⎪
⎪
|𝑓𝑗 (𝑥) − 𝑓𝑗 (𝑦)| ≤ |f(𝑥) − f(𝑦)| = ⎨ |𝑓𝑖 (𝑥) − 𝑓𝑖 (𝑦)|2 ⎬
∑
⎪
⎪
⎩ 𝑖=1
⎭
при 𝑗 = 1, … , 𝑘. Часть (b) следует из (a) и теоремы 4.9.
4.11 Примеры. Если 𝑥1 , … , 𝑥𝑘 — координаты точки x ∈ 𝑅𝑘 , то функции 𝜑𝑖 , определенные равенствами
(8)
(x ∈ 𝑅𝑘 ),
𝜑𝑖 (x) = 𝑥𝑖
непрерывны на 𝑅𝑘 , так как неравенство
|𝜑𝑖 (x) − 𝜑𝑖 (y)| ≤ |x − y|
показывает, что в определении 4.5 можно положить 𝛿 = 𝜀. Функции 𝜑𝑖 иногда называются координатными функциями.
Повторное применение теоремы 4.9 показывает, что каждый одночлен
𝑛
𝑛
𝑛
𝑥11 𝑥22 ⋯ 𝑥𝑘𝑘 ,
(9)
где 𝑛1 , … , 𝑛𝑘 — неотрицательные целые числа, непрерывен на 𝑅𝑘 . То же верно в отношении функций, отличающихся от (9) постоянным множителем, так как постоянные, очевидно, непрерывны.
Следовательно, каждый многочлен 𝑃 , заданный равенством
(10)
𝑃 (x) =
𝑛
∑
𝑛
𝑐𝑛1 …𝑛𝑘 𝑥11 ⋯ 𝑥𝑘𝑘
66
(x ∈ 𝑅𝑘 ),
непрерывен на 𝑅𝑘 . Здесь коэффициенты 𝑐𝑛1 …𝑛𝑘 — комплексные числа, 𝑛1 , … , 𝑛𝑘 — неотрицательные
целые числа, а сумма содержит конечное число слагаемых.
Далее, всякая рациональная функция от 𝑥1 , … , 𝑥𝑘 , т. е. каждое отношение двух многочленов
вида (10), непрерывна на 𝑅𝑘 всюду, где знаменатель отличен от нуля.
Из неравенства треугольника легко получаем, что
(11)
(x, y ∈ 𝑅𝑘 ).
||x| − |y|| ≤ |x − y|
Значит, отображение x → |x| является непрерывной вещественной функцией на 𝑅𝑘 .
Если f — непрерывное отображение метрического пространства 𝑋 в 𝑅𝑘 и если 𝜑 определено на
𝑋 равенством 𝜑(𝑝) = |f(𝑝)|, то из теоремы 4.7 следует, что 𝜑 — непрерывная вещественная функция
на 𝑋.
4.12 Замечание. Мы определили понятие непрерывности для функций, заданных на подмножестве 𝐸 метрического пространства 𝑋. Однако дополнение множества 𝐸 в 𝑋 не играет никакой
роли в этом определении (заметим, что для пределов функций ситуация несколько отличалась).
Тем самым мы не теряем ничего интересного, выбросив дополнение области определения функции
𝑓 . Это означает, что мы можем говорить о непрерывных отображениях одного метрического пространства в другое, а не об отображениях подмножеств. Это упрощает условия и доказательства
некоторых теорем. Мы уже воспользовались этим в теоремах 4.8 – 4.10 и будем поступать так же
в разделе, посвященном компактности.
Непрерывность и компактность
4.13 Определение. Отображение f множества 𝐸 в 𝑅𝑘 называется ограниченным, если существует
вещественное число 𝑀, такое, что |f(𝑥)| ≤ 𝑀 для всех 𝑥 ∈ 𝐸.
4.14 Теорема. Пусть 𝑓 — непрерывное отображение компактного метрического пространства
𝑋 в метрическое пространство 𝑌 . Тогда 𝑓 (𝑋) компактно.
Доказательство. Пусть {𝑉𝛼 } — открытое покритие множества 𝑓 (𝑋). Поскольку 𝑓 непрерывно,
теорема 4.8 показывает, что все множества 𝑓 −1 (𝑉𝛼 ) открыты. Так как 𝑋 компактно, имеется конечный набор индексов, которые мы обозначим 𝛼1 , … , 𝛼𝑛 , такой, что
(12)
𝑋 ⊂ 𝑓 −1 (𝑉𝛼1 ) ∪ ⋯ ∪ 𝑓 −1 (𝑉𝛼𝑛 ).
Поскольку 𝑓 (𝑓 −1 (𝐸)) ⊂ 𝐸 при каждом 𝐸 ⊂ 𝑌 , из (12) следует, что
(13)
𝑓 (𝑋) ⊂ 𝑉𝛼1 ∪ ⋯ ∪ 𝑉𝛼𝑛 .
Доказательство закончено.
Замечание. Мы воспользовались соотношением 𝑓 (𝑓 −1 (𝐸)) ⊂ 𝐸, которое верно при 𝐸 ⊂ 𝑌 . Если
𝐸 ⊂ 𝑋, то 𝑓 −1 (𝑓 (𝐸)) ⊃ 𝐸; равенства может не быть ни в одном из случаев.
Выведем теперь некоторые следствия из теоремы 4.14.
4.15 Теорема. Если f — непрерывное отображение компактного метрического пространства 𝑋
в 𝑅𝑘 , то f(𝑋) замкнуто и ограничено. Таким образом, f ограничено.
Это следует из теоремы 2.41. Этот результат особенно важен тогда, когда 𝑓 — вещественная
функция.
4.16 Теорема. Пусть 𝑓 — непрерывная вещественная функция на компактном метрическом
пространстве 𝑀 и
(14)
𝑀 = sup 𝑓 (𝑝),
𝑚 = inf 𝑓 (𝑝).
𝑝∈𝑋
𝑝∈𝑋
Тогда существуют точки 𝑝, 𝑞 ∈ 𝑋, такие, что 𝑓 (𝑝) = 𝑀 и 𝑓 (𝑞) = 𝑚.
67
Обозначения в (14) имеют следующий смысл: 𝑀 — верхняя грань множества всех чисел 𝑓 (𝑝),
когда 𝑝 пробегает 𝑋, а 𝑚 — нижняя грань этого числового множества.
Утверждение теоремы можно сформулировать и так: существуют точки 𝑝 и 𝑞 в 𝑋, такие,
что 𝑓 (𝑞) ≤ 𝑓 (𝑥) ≤ 𝑓 (𝑝) для всех 𝑥 ∈ 𝑋, т. е. 𝑓 достигает своего максимума (в 𝑝) и минимума (в 𝑞).
Доказательство. По теореме 4.15 𝑓 (𝑋) — замкнутое и ограниченное множество вещественных
чисел; значит, 𝑓 (𝑋) содержит
𝑀 = sup 𝑓 (𝑋)
и
𝑚 = inf 𝑓 (𝑋)
по теореме 2.28.
4.17 Теорема. Пусть 𝑓 — непрерывное взаимно однозначное отображение компактного метрического пространства 𝑋 на метрическое пространство 𝑌 . Тогда обратное отображение 𝑓 −1 ,
определенное на 𝑌 равенством
𝑓 −1 (𝑓 (𝑥)) = 𝑥
(𝑥 ∈ 𝑋),
есть непрерывное отображение пространства 𝑌 на 𝑋.
Доказательство. Применяя теорему 4.8 к 𝑓 −1 вместо 𝑓 , мы видим, что достаточно доказать, что
𝑓 (𝑉 ) — открытое множество в 𝑌 для каждого открытого в 𝑋 множества 𝑉 . Зафиксируем такое
множество 𝑉 .
Дополнение 𝑉 𝑐 множества 𝑉 замкнуто в 𝑋 и, следовательно, компактно (теорема 2.35); значит,
𝑓 (𝑉 𝑐 ) — компактное подмножество множества 𝑌 (теорема 4.14); поэтому оно замкнуто в 𝑌 (теорема 2.34). Поскольку 𝑓 взаимно однозначно отображает 𝑋 на 𝑌 , 𝑓 (𝑉 ) совпадает с дополнением
множества 𝑓 (𝑉 𝑐 ). Значит, 𝑓 (𝑉 ) открыто.
4.18 Определение. Пусть 𝑓 — отображение метрического пространства 𝑋 в метрическое пространство 𝑌 . Мы будем говорить, что 𝑓 равномерно непрерывно на 𝑋, если для каждого 𝜀 > 0
существует 𝛿 > 0, такое, что
(15)
𝑑𝑌 (𝑓 (𝑝), 𝑓 (𝑞)) < 𝜀
для всех 𝑝 и 𝑞 из 𝑋, для которых 𝑑𝑋 (𝑝, 𝑞) < 𝛿.
Посмотрим, чем отличаются понятия непрерывности и равномерной непрерывности. Во-первых,
равномерная непрерывность есть свойство функции на множестве, тогда как непрерывность может
быть определена в одной точке. Бессмысленно спрашивать, является ли данная функция равномерно непрерывной в некоторой точке. Во-вторых, если 𝑓 непрерывна на 𝑋, то для каждого 𝜀 > 0
и для каждой точки 𝑝 множества 𝑋 можно найти число 𝛿 > 0, обладающее свойством, указанным
в определении 4.5. Это 𝛿 зависит от 𝜀 и от 𝑝. Если же 𝑓 равномерно непрерывна на 𝑋, то для
каждого 𝜀 > 0 можно найти одно число 𝛿 > 0, которое годится для всех точек 𝑝 множества 𝑋.
Очевидно, каждая равномерно непрерывная функция непрерывна. В случае компактных множеств эти два понятия, как показывает следующая теорема, равносильны.
4.19 Теорема. Пусть 𝑓 — непрерывное отображение компактного метрического пространства
𝑋 в метрическое пространство 𝑌 . Тогда 𝑓 равномерно непрерывно на 𝑋.
Доказательство. Пусть дано 𝜀 > 0. Поскольку 𝑓 непрерывно, каждой точке 𝑝 ∈ 𝑋 можно сопоставить положительное число 𝜑(𝑝), такое, что
(16)
если 𝑞 ∈ 𝑋, 𝑑𝑋 (𝑝, 𝑞) < 𝜑(𝑝),
то 𝑑𝑌 (𝑓 (𝑝), 𝑓 (𝑞)) <
𝜀
.
2
Пусть 𝐽 (𝑝) — множество всех 𝑞 ∈ 𝑋, для которых
(17)
𝑑𝑋 (𝑝, 𝑞) < 12 𝜑(𝑝).
Поскольку 𝑝 ∈ 𝐽 (𝑝), семейство всех множеств 𝐽 (𝑝) образует открытое покрытие пространства 𝑋, а
поскольку 𝑋 компактно, в нем найдется конечное множество точек 𝑝1 , … , 𝑝𝑛 , такое, что
(18)
𝑋 ⊂ 𝐽 (𝑝1 ) ∪ ⋯ ∪ 𝐽 (𝑝𝑛 ).
68
Положим
(19)
𝛿=
1
2
min[𝜑(𝑝1 ), … , 𝜑(𝑝𝑛 )].
Тогда 𝛿 > 0. (Именно здесь существенна конечность покрытия, упоминаемого в определении компактности. Минимум конечного числа положительных чисел положителен, тогда как нижняя грань
бесконечного множества положительных чисел вполне может оказаться равной нулю.)
Пусть теперь 𝑞 и 𝑝 — точки множества 𝑋, такие, что 𝑑𝑋 (𝑝, 𝑞) < 𝛿. Согласно (18), существует
целое 𝑚, 1 ≤ 𝑚 ≤ 𝑛, такое, что 𝑝 ∈ 𝐽 (𝑝𝑚 ). Значит,
𝑑𝑋 (𝑝, 𝑝𝑚 ) < 21 𝜑(𝑝𝑚 )
(20)
и, кроме того,
𝑑𝑋 (𝑞, 𝑝𝑚 ) ≤ 𝑑𝑋 (𝑝, 𝑞) + 𝑑𝑋 (𝑝, 𝑝𝑚 ) < 𝛿 + 12 𝜑(𝑝𝑚 ) ≤ 𝜑(𝑝𝑚 ).
Наконец, из (16) следует, что тогда
𝑑𝑌 (𝑓 (𝑝), 𝑓 (𝑞)) ≤ 𝑑𝑌 (𝑓 (𝑝), 𝑓 (𝑝𝑚 )) + 𝑑𝑌 (𝑓 (𝑞), 𝑓 (𝑝𝑚 )) < 𝜀.
Доказательство закончено.
Другое доказательство намечено в упражнении 10.
Теперь мы перейдем к доказательству того, что компактность существенна в предположениях
теорем 4.14, 4.15, 4.16 и 4.19.
4.20 Теорема. Пусть 𝐸 — некомпактное множество в 𝑅1 . Тогда
(a) существует непрерывная на 𝐸 функция, не являющаяся ограниченной;
(b) существует непрерывная и ограниченная на 𝐸 функция, не имеющая максимума.
Если, кроме того, 𝐸 ограничено, то
(c) существует непрерывная на 𝐸 функция, не являющаяся равномерно непрерывной.
Доказательство. Допустим сначала, что 𝐸 ограничено, так что существует предельная точка 𝑥0
множества 𝐸, не содержащаяся в 𝐸. Рассмотрим функцию
(21)
𝑓 (𝑥) =
1
𝑥 − 𝑥0
(𝑥 ∈ 𝐸).
Она непрерывна на 𝐸 (теорема 4.9), но, очевидно, не ограничена. Убедимся в том, что функция
(21) не является равномерно непрерывной. Пусть 𝜀 > 0 и 𝛿 > 0 произвольны. Выберем точку 𝑥 ∈ 𝐸
такую, что |𝑥 − 𝑥0 | < 𝛿. Взяв 𝑡 достаточно близким к 𝑥0 , мы можем сделать разность |𝑓 (𝑡) − 𝑓 (𝑥)|
больше 𝜀, хотя |𝑡 − 𝑥| < 𝛿. Поскольку это верно при любом 𝛿 > 0, 𝑓 не является равномерно
непрерывной на 𝐸.
Функция 𝑔, заданная равенством
(22)
𝑔(𝑥) =
1
1 + (𝑥 − 𝑥0 )2
(𝑥 ∈ 𝐸),
непрерывна на 𝐸 и ограничена, так как 0 < 𝑔(𝑥) < 1. Ясно, что
sup 𝑔(𝑥) = 1,
𝑥∈𝐸
тогда как 𝑔(𝑥) < 1 при всех 𝑥 ∈ 𝐸. Таким образом, 𝑔 не имеет максимума на 𝐸.
Доказав теорему для ограниченных множеств 𝐸, предположим, что 𝐸 не ограничено. Тогда (a)
выполняется для 𝑓 (𝑥) = 𝑥, а (b) — для функции
(23)
ℎ(𝑥) =
𝑥2
1 + 𝑥2
(𝑥 ∈ 𝐸),
так как
sup ℎ(𝑥) = 1
𝑥∈𝐸
и ℎ(𝑥) < 1 при всех 𝑥 ∈ 𝐸.
Утверждение (c) неверно, если не предполагать, что 𝐸 ограничено. Действительно, пусть 𝐸 —
множество всех целых чисел. Тогда любая функция, заданная на 𝐸, равномерно непрерывна на 𝐸.
Чтобы убедиться в этом, нужно всего лишь взять 𝛿 < 1 в определении 4.18.
69
В заключение этого раздела мы покажем, что компактность существенна также и в теореме
4.17.
4.21 Пример. Пусть 𝑋 — полуинтервал [0, 2𝜋) вещественной оси и пусть f — отображение множества 𝑋 на окружность 𝑌 , состоящую из всех точек, удаленных от начала на расстояние 1, заданное
равенством
(24)
f(𝑡) = (cos 𝑡, sin 𝑡)
(0 ≤ 𝑡 < 2𝜋).
Непрерывность тригонометрических функций (косинуса и синуса), так же как и их периодичность,
будут установлены в гл. 8. Из этих фактов следует, что f — непрерывное взаимно однозначное
отображение 𝑋 на 𝑌 .
Однако обратное отображение (которое существует, так как f взаимно однозначно отображает
𝑋 на 𝑌 ) не непрерывно в точке (1, 0) = f(0). Конечно, в этом примере 𝑋 не компактно. (Любопытно,
что f−1 не непрерывно, несмотря на то, что 𝑌 компактно!)
Непрерывность и связность
4.22 Теорема. Если 𝑓 — непрерывное отображение метрического пространства 𝑋 в метрическое пространство 𝑌 , а 𝐸 — связное подмножество пространства 𝑋, то 𝑓 (𝐸) связно.
Доказательство. Предположим обратное, что 𝑓 (𝐸) = 𝐴 ∪ 𝐵, где 𝐴 и 𝐵 — непустые отделенные
друг от друга подмножества множества 𝑌 . Положим 𝐺 = 𝐸 ∩ 𝑓 −1 (𝐴), 𝐻 = 𝐸 ∩ 𝑓 −1 (𝐵).
Тогда 𝐸 = 𝐺 ∪ 𝐻, и 𝐺 и 𝐻 оба непусты.
̄ последнее множество замкнуто,
Так как 𝐴 ⊂ 𝐴̄ (замыкание множества 𝐴), имеем 𝐺 ⊂ 𝑓 −1 (𝐴);
̄ Из этого следует, что 𝑓 (𝐺)̄ ⊂ 𝐴.̄ Так как 𝑓 (𝐻) = 𝐵 и
так как 𝑓 непрерывно; значит, 𝐺̄ ⊂ 𝑓 −1 (𝐴).
𝐴̄ ∩ 𝐵 пусто, заключаем, что 𝐺̄ ∩ 𝐻 пусто.
Аналогичное рассуждение показывает, что 𝐺 ∩ 𝐻̄ пусто. Следовательно, 𝐺 и 𝐻 отделены друг
от друга, но это невозможно, если 𝐸 связно.
4.23 Теорема. Пусть 𝑓 — непрерывная вещественная функция на сегменте [𝑎, 𝑏]. Если 𝑓 (𝑎) <
𝑓 (𝑏) и если 𝑐 — такое число, что 𝑓 (𝑎) < 𝑐 < 𝑓 (𝑏), то существует точка 𝑥 ∈ (𝑎, 𝑏), такая, что
𝑓 (𝑥) = 𝑐.
Аналогичное утверждение справедливо, разумеется, и тогда, когда 𝑓 (𝑎) > 𝑓 (𝑏). Грубо говоря,
эта теорема означает, что непрерывная вещественная функция принимает на сегменте все промежуточные значения.
Доказательство. По теореме 2.47 сегмент [𝑎, 𝑏] связен. Значит, по теореме 4.22, 𝑓 ([𝑎, 𝑏]) — связное
подмножество пространства 𝑅1 , и остается еще раз сослаться на теорему 2.47, чтобы утверждение
было доказано.
4.24 Замечание. На первый взгляд может показаться, что верна теорема, обратная к теореме
4.23. Иначе говоря, можно подумать, что если для любых двух точек 𝑥1 < 𝑥2 и для любого числа 𝑐,
лежащего между 𝑓 (𝑥1 ) и 𝑓 (𝑥2 ), найдется точка 𝑥 ∈ (𝑥1 , 𝑥2 ), такая, что 𝑓 (𝑥) = 𝑐, то 𝑓 должна быть
непрерывной.
Однако пример 4.27(d) показывает, что это не так.
Разрывы функций
Если 𝑥 — точка из области определения функции 𝑓 , в которой эта функция не является непрерывной, то мы будем говорить, что 𝑓 разрывна в 𝑥 или что 𝑓 имеет разрыв в 𝑥. В случае, когда
𝑓 определена на сегменте или на интервале, удобно разделить разрывы на два типа. Прежде чем
произвести эту классификацию, мы должны определить правосторонний и левосторонний пределы
функции 𝑓 в 𝑥, которые мы обозначим соответственно через 𝑓 (𝑥+) и 𝑓 (𝑥−).
70
4.25 Определение. Пусть 𝑓 определена на (𝑎, 𝑏). Рассмотрим любую точку 𝑥, такую, что 𝑎 ≤ 𝑥 < 𝑏.
Мы будем писать
𝑓 (𝑥+) = 𝑞,
если 𝑓 (𝑡𝑛 ) → 𝑞 при 𝑛 → ∞ для всех последовательностей {𝑡𝑛 } в (𝑥, 𝑏), таких, что 𝑡𝑛 → 𝑥. Чтобы
получить определение предела 𝑓 (𝑥−) при 𝑎 < 𝑥 ≤ 𝑏, мы ограничимся последовательностями {𝑡𝑛 } в
(𝑎, 𝑥).
Ясно, что в любой точке 𝑥 интервала (𝑎, 𝑏) предел lim 𝑓 (𝑡) существует тогда и только тогда,
𝑡→𝑥
когда
𝑓 (𝑥+) = 𝑓 (𝑥−) = lim 𝑓 (𝑡).
𝑡→𝑥
4.26 Определение. Пусть 𝑓 определена на (𝑎, 𝑏). Если 𝑓 разрывна в точке 𝑥 и если 𝑓 (𝑥+) и 𝑓 (𝑥−)
существуют, то говорят, что 𝑓 имеет в 𝑥 разрыв первого рода, или простой разрыв. В противном
случае разрыв называется разрывом второго рода.
Возможны две разновидности простых разрывов: либо 𝑓 (𝑥+) ≠ 𝑓 (𝑥−) (в этом случае значение
𝑓 (𝑥) несущественно), либо 𝑓 (𝑥+) = 𝑓 (𝑥−) ≠ 𝑓 (𝑥).
4.27 Примеры.
(a) Положим
𝑓 (𝑥) =
1
{0
(𝑥 рационально),
(𝑥 иррационально).
Тогда 𝑓 имеет разрывы второго рода во всех точках 𝑥, так как ни 𝑓 (𝑥+), ни 𝑓 (𝑥−) не существуют.
(b) Положим
𝑓 (𝑥) =
𝑥 (𝑥 рационально),
{0 (𝑥 иррационально).
Тогда 𝑓 непрерывна в 𝑥 = 0 и имеет разрывы второго рода во всех остальных точках.
(c) Положим
⎧𝑥 + 2
⎪
𝑓 (𝑥) = ⎨−𝑥 − 2
⎪𝑥 + 2
⎩
(−3 < 𝑥 < −2),
(−2 ≤ 𝑥 < 0),
(0 ≤ 𝑥 < 1).
Тогда 𝑓 имеет простой разрыв в 𝑥 = 0 и непрерывна во всех остальных точках интервала
(−3, 1).
(d) Положим
𝑓 (𝑥) =
sin 𝑥1
{0
(𝑥 ≠ 0),
(𝑥 = 0).
Поскольку ни 𝑓 (0+), ни 𝑓 (0−) не существуют, 𝑓 имеет разрыв второго рода в 𝑥 = 0. Мы еще
не доказали, что sin 𝑥 — непрерывная функция. Если считать это утверждение верным, то из
теоремы 4.7 следует, что 𝑓 непрерывна в каждой точке 𝑥 ≠ 0.
Монотонные функции
Теперь мы изучим функции, которые нигде не убывают (или нигде не возрастают) на данном
интервале.
4.28 Определение. Пусть 𝑓 — вещественная функция на (𝑎, 𝑏). Говорят, что 𝑓 монотонно возрастает на (𝑎, 𝑏), если при 𝑎 < 𝑥 < 𝑦 < 𝑏 имеем 𝑓 (𝑥) ≤ 𝑓 (𝑦). Обращая последнее неравенство,
мы получим определение монотонно убывающей функции. Класс монотонных функций состоит из
убывающих и из возрастающих функций.
71
4.29 Теорема. Пусть 𝑓 монотонно возрастает на (𝑎, 𝑏). Тогда 𝑓 (𝑥+) и 𝑓 (𝑥−) существуют в
каждой точке 𝑥 интервала (𝑎, 𝑏). Точнее,
(25)
sup 𝑓 (𝑡) = 𝑓 (𝑥−) ≤ 𝑓 (𝑥) ≤ 𝑓 (𝑥+) = inf 𝑓 (𝑡).
𝑥<𝑡<𝑏
𝑎<𝑡<𝑥
Кроме того, если 𝑎 < 𝑥 < 𝑦 < 𝑏, то
(26)
𝑓 (𝑥+) ≤ 𝑓 (𝑦−).
Аналогичные результаты, очевидно, верны и для монотонно убывающих функций.
Доказательство. По предположению, множество чисел 𝑓 (𝑡), где 𝑎 < 𝑡 < 𝑥, ограничено сверху
числом 𝑓 (𝑥) и потому имеет верхнюю грань, которую мы обозначим через 𝐴. Очевидно, что 𝐴 ≤
𝑓 (𝑥). Мы должны показать, что 𝐴 = 𝑓 (𝑥−).
Пусть задано 𝜀 > 0. Из определения числа 𝐴 как верхней грани следует, что существует 𝛿 > 0,
такое, что 𝑎 < 𝑥 − 𝛿 < 𝑥 и
(27)
𝐴 − 𝜀 < 𝑓 (𝑥 − 𝛿) ≤ 𝐴.
Поскольку 𝑓 монотонна, имеем
(28)
𝑓 (𝑥 − 𝛿) ≤ 𝑓 (𝑡) ≤ 𝐴
(𝑥 − 𝛿 < 𝑡 < 𝑥).
Комбинируя (27) и (28), мы получаем
|𝑓 (𝑡) − 𝐴| < 𝜀
(𝑥 − 𝛿 < 𝑡 < 𝑥).
Значит, 𝑓 (𝑥−) = 𝐴.
Вторая часть неравенства (25) доказывается точно таким же способом.
Далее, если 𝑎 < 𝑥 < 𝑦 < 𝑏, то из (25) следует, что
(29)
𝑓 (𝑥+) = inf 𝑓 (𝑡) = inf 𝑓 (𝑡).
𝑥<𝑡<𝑦
𝑥<𝑡<𝑏
Последнее равенство получится, если применить (25) к (𝑎, 𝑦) вместо (𝑎, 𝑏). Подобным же образом
(30)
𝑓 (𝑦−) = sup 𝑓 (𝑡) = sup 𝑓 (𝑡).
𝑎<𝑡<𝑦
𝑥<𝑡<𝑦
Сравнение равенств (29) и (30) дает (26).
Следствие. Монотонная функция не имеет разрывов второго рода.
Из этого следствия вытекает, что монотонная функция имеет разрывы не более чем в счетном множестве точек. Вместо того чтобы сослаться на общую теорему, доказательство которой
намечено в упражнении 17, мы дадим здесь простое доказательство, применимое к монотонным
функциям.
4.30 Теорема. Пусть 𝑓 монотонна на (𝑎, 𝑏). Тогда множество точек интервала (𝑎, 𝑏), в которых
𝑓 разрывна, не более чем счетно.
Доказательство. Допустим для определенности, что 𝑓 возрастает, и пусть 𝐸 — множество точек,
в которых 𝑓 разрывна.
Каждой точке 𝑥 множества 𝐸 сопоставим рациональное число 𝑟(𝑥), такое, что
𝑓 (𝑥−) < 𝑟(𝑥) < 𝑓 (𝑥+).
Так как при 𝑥1 < 𝑥2 мы имеем 𝑓 (𝑥1 +) ≤ 𝑓 (𝑥2 −), ясно, что 𝑟(𝑥1 ) ≠ 𝑟(𝑥2 ), если 𝑥1 ≠ 𝑥2 .
Таким образом, установлено взаимно однозначное соответствие между множеством 𝐸 и подмножеством множества всех рациональных чисел. Это последнее множество, как мы знаем, счетно.
72
4.31 Замечание. Нужно отметить, что точки разрыва монотонной функции не обязаны быть
изолированными. В самом деле, для любого счетного подмножества 𝐸 интервала (𝑎, 𝑏), которое
может быть даже всюду плотным, можно построить функцию 𝑓 , монотонную на (𝑎, 𝑏), имеющую
разрыв в каждой точке множества 𝐸 и непрерывную во всех остальных точках интервала (𝑎, 𝑏).
Чтобы доказать это, расположим точки множества 𝐸 в последовательность {𝑥𝑛 }, 𝑛 = 1, 2, 3, … .
Пусть {𝑐𝑛 } — последовательность положительных чисел, такая, что ряд ∑ 𝑐𝑛 сходится. Положим
(31)
𝑓 (𝑥) =
∑
𝑐𝑛
(𝑎 < 𝑥 < 𝑏).
𝑥𝑛 <𝑥
В суммировании участвуют все индексы 𝑛, для которых 𝑥𝑛 < 𝑥. Если слева от 𝑥 нет точек 𝑥𝑛 ,
то сумма пуста и, следуя обычному соглашению, мы считаем ее равной нулю. Поскольку ряд (31)
сходится абсолютно, порядок, в котором выписываются его члены, несуществен.
Мы предоставляем читателю проверку следующих свойств функции 𝑓 :
(a) 𝑓 монотонно возрастает на (𝑎, 𝑏);
(b) 𝑓 разрывна в каждой точке множества 𝐸; точнее,
𝑓 (𝑥𝑛 +) − 𝑓 (𝑥𝑛 −) = 𝑐𝑛 ;
(c) 𝑓 непрерывна во всех остальных точках интервала (𝑎, 𝑏).
Более того, нетрудно видеть, что 𝑓 (𝑥−) = 𝑓 (𝑥) во всех точках интервала (𝑎, 𝑏). Если функция
удовлетворяет этому условию, то мы будем говорить, что 𝑓 непрерывна слева. Если бы в (31)
суммирование производилось по всем индексам 𝑛, для которых 𝑥𝑛 ≤ 𝑥, то мы имели бы 𝑓 (𝑥+) = 𝑓 (𝑥)
во всех точках интервала (𝑎, 𝑏), т. е. 𝑓 была бы непрерывной справа.
Функции такого типа можно строить и другим методом; пример читатель найдет в теореме 6.16.
Бесконечные пределы и пределы в бесконечности
Чтобы иметь возможность действовать в расширенной системе вещественных чисел, мы расширим
рамки определения 4.1, сформулировав его в терминах окрестностей.
Для любого вещественного числа 𝑥 мы уже определили окрестность 𝑥 как любой интервал вида
(𝑥 − 𝛿, 𝑥 + 𝛿).
4.32 Определение. При любом вещественном 𝑐 множество всех вещественных чисел 𝑥, таких, что
𝑥 > 𝑐, называется окрестностью точки +∞ и обозначается (𝑐, +∞). Аналогично, множество (−∞, 𝑐)
называется окрестностью точки −∞.
4.33 Определение. Пусть вещественная функция 𝑓 определена на 𝐸 ⊂ 𝑅. Мы будем говорить,
что
𝑓 (𝑡) → 𝐴 при 𝑡 → 𝑥,
где 𝐴 и 𝑥 принадлежат расширенной системе вещественных чисел, если для любой окрестности 𝑈
точки 𝐴 существует окрестность 𝑉 точки 𝑥, такая, что 𝑉 ∩ 𝐸 непусто и 𝑓 (𝑡) ∈ 𝑈 при всех 𝑡 ∈ 𝑉 ∩ 𝐸,
𝑡 ≠ 𝑥.
Несложное рассуждение показывает, что это определение совпадает с определением 4.1, когда
𝐴 и 𝑥 вещественны.
Аналог теоремы 4.4 справедлив и в этом случае, и в его доказательстве не появляется ничего
нового. Мы сформулируем его ради полноты.
4.34 Теорема. Пусть 𝑓 и 𝑔 определены на 𝐸 ⊂ 𝑅. Допустим, что
𝑓 (𝑡) → 𝐴,
𝑔(𝑡) → 𝐵
при 𝑡 → 𝑥.
Тогда
(a)
(b)
(c)
(d)
если 𝑓 (𝑡) → 𝐴′ , то 𝐴′ = 𝐴,
(𝑓 + 𝑔)(𝑡) → 𝐴 + 𝐵,
(𝑓 𝑔)(𝑡) → 𝐴𝐵,
(𝑓 /𝑔)(𝑡) → 𝐴/𝐵,
если правые части в (b), (c) и (d) определены.
Напомним, что ∞ − ∞, 0 ⋅ ∞, ∞/∞, 𝐴/0 не определены (см. определение 1.23).
73
Упражнения
1. Пусть 𝑓 — вещественная функция, определенная на 𝑅1 , удовлетворяющая соотношению
lim[𝑓 (𝑥 + ℎ) − 𝑓 (𝑥 − ℎ)] = 0
ℎ→0
при любом 𝑥 ∈ 𝑅1 . Следует ли из этого, что 𝑓 непрерывна?
2. Пусть 𝑓 — непрерывное отображение метрического пространства 𝑋 в метрическое пространство 𝑌 . Доказать, что
𝑓 (𝐸)̄ ⊂ 𝑓 (𝐸)
3.
4.
5.
6.
7.
8.
9.
10.
11.
12.
13.
для любого множества 𝐸 ⊂ 𝑋 (𝐸̄ обозначает замыкание множества 𝐸). Показать на примере, что 𝑓 (𝐸)̄
может быть собственным подмножеством множества 𝑓 (𝐸).
Пусть 𝑓 — непрерывная вещественная функция на метрическом пространстве 𝑋. Пусть 𝑍(𝑓 ) (нульмножество функции 𝑓 ) есть множество всех 𝑝 ∈ 𝑋, в которых 𝑓 (𝑝) = 0. Доказать, что 𝑍(𝑓 ) замкнуто.
Пусть 𝑓 и 𝑔 — непрерывные отображения метрического пространства 𝑋 в метрическое пространство 𝑌 ,
и пусть 𝐸 — всюду плотное подмножество пространства 𝑋. Доказать, что 𝑓 (𝐸) всюду плотно в 𝑓 (𝑋).
Если 𝑔(𝑝) = 𝑓 (𝑝) при всех 𝑝 ∈ 𝐸, доказать, что 𝑔(𝑝) = 𝑓 (𝑝) при всех 𝑝 ∈ 𝑋. (Иными словами, непрерывное отображение определяется своими значениями на всюду плотном подмножестве своей области
определения.)
Пусть 𝑓 — вещественная непрерывная функция, заданная на замкнутом множестве 𝐸 ⊂ 𝑅1 . Доказать,
что существует непрерывная вещественная функция 𝑔 на 𝑅1 , такая, что 𝑔(𝑥) = 𝑓 (𝑥) при всех 𝑥 ∈ 𝐸 (такая
функция называется непрерывным продолжением функции 𝑓 с множества 𝐸 на 𝑅1 ). Показать, что
результат перестает быть верным, если опустить предположение замкнутости. Распространить результат
на векторнозначные функции. Указание. Сделать график функции 𝑔 прямой линией на каждом из
интервалов, составляющих дополнение множества 𝐸 (ср. с упражнением 29, гл. 2). Результат остается
верным, если 𝑅1 заменить любым метрическим пространством, но доказательство уже не так просто.
Если функция 𝑓 определена на 𝐸, то графиком 𝑓 называется множество точек (𝑥, 𝑓 (𝑥)), где 𝑥 ∈ 𝐸. В
частности, если 𝐸 — множество вещественных чисел, а 𝑓 вещественна, то графиком функции 𝑓 служит
некоторое подмножество плоскости.
Допустим, что 𝐸 компактно. Доказать, что 𝑓 непрерывна на 𝐸 в том и только в том случае, когда ее
график компактен.
Если 𝐸 ⊂ 𝑋, а 𝑓 — функция, заданная на 𝑋, то сужением 𝑓 на 𝐸 называется функция 𝑔 с областью
определения 𝐸, такая, что 𝑔(𝑝) = 𝑓 (𝑝) при 𝑝 ∈ 𝐸. Зададим в 𝑅2 функции 𝑓 и 𝑔: 𝑓 (0, 0) = 𝑔(0, 0) = 0,
𝑓 (𝑥, 𝑦) = 𝑥𝑦2 /(𝑥2 + 𝑦4 ), 𝑔(𝑥, 𝑦) = 𝑥𝑦2 /(𝑥2 + 𝑦6 ) при (𝑥, 𝑦) ≠ (0, 0). Доказать, что 𝑓 ограничена на 𝑅2 , что 𝑔 не
ограничена в любой окрестности точки (0, 0) и что 𝑓 разрывна в (0, 0); тем не менее сужения каждой из
функций 𝑓 и 𝑔 на любую прямую в 𝑅2 непрерывны!
Пусть 𝑓 — вещественная равномерно непрерывная функция на ограниченном множестве 𝐸 в 𝑅1 . Доказать, что 𝑓 ограничена на 𝐸.
Доказать, что заключение ложно, если опустить условие ограниченности множества 𝐸.
Показать, что определение равномерной непрерывности можно следующим образом сформулировать,
используя понятие диаметра множества: для каждого 𝜀 > 0 существует 𝛿 > 0, такое, что diam 𝑓 (𝐸) < 𝜀
для любого 𝐸 ⊂ 𝑋, для которого diam 𝐸 < 𝛿.
Провести подробно следующий вариант доказательства теоремы 4.19: если 𝑓 не является равномерно непрерывной, то для некоторого 𝜀 > 0 существуют последовательности {𝑝𝑛 }, {𝑞𝑛 } в 𝑋, такие, что
𝑑𝑋 (𝑝𝑛 , 𝑞𝑛 ) → 0, но 𝑑𝑌 (𝑓 (𝑝𝑛 ), 𝑓 (𝑞𝑛 )) > 𝜀. Воспользоваться теоремой 2.37, чтобы получить противоречие.
Пусть 𝑓 — равномерно непрерывное отображение метрического пространства 𝑋 в метрическое пространство 𝑌 . Доказать, что {𝑓 (𝑥𝑛 )} — последовательность Коши в 𝑌 для каждой последовательности Коши
{𝑥𝑛 } в 𝑋. Использовать этот результат для другого варианта доказательства теоремы из упражнения
13.
Равномерно непрерывная функция от равномерно непрерывной функции является равномерно непрерывной.
Сформулировать это утверждение более точно и доказать его.
Пусть 𝐸 — всюду плотное подмножество метрического пространства 𝑋, и пусть 𝑓 — равномерно непрерывная вещественная функция, заданная на 𝐸. Доказать, что 𝑓 имеет непрерывное продолжение с 𝐸 на
𝑋 (по поводу терминологии см. упражнение 5). (Единственность следует из упражнения 4.) Указание.
Для каждого 𝑝 ∈ 𝑋 и каждого положительного целого 𝑛 пусть 𝑉𝑛 (𝑝) — множество всех 𝑞 ∈ 𝐸, таких,
что 𝑑(𝑝, 𝑞) < 1/𝑛. Воспользоваться упражнением 9 для доказательства того, что пересечение замыканий
множеств 𝑓 (𝑉1 (𝑝)), 𝑓 (𝑉2 (𝑝)), … состоит из единственной точки пространства 𝑅1 , которую мы обозначим
через 𝑔(𝑝). Доказать, что функция 𝑔, определенная таким образом на 𝑋, и есть требуемое продолжение
функции 𝑓 .
74
14.
15.
16.
17.
Можно ли пространство значений 𝑅1 заменить здесь пространством 𝑅𝑘 ? Любым компактным метрическим пространством? Любым полным метрическим пространством? Любым метрическим пространством?
Пусть 𝐼 = [0, 1] — замкнутый единичный сегмент. Пусть 𝑓 — непрерывное отображение сегмента 𝐼 в 𝐼.
Доказать, что 𝑓 (𝑥) = 𝑥 как минимум для одного 𝑥 ∈ 𝐼.
Назовем отображение пространства 𝑋 в 𝑌 открытым, если 𝑓 (𝑉 ) открыто в 𝑌 , каково бы ни было
открытое множество 𝑉 в 𝑋.
Доказать, что каждое непрерывное открытое отображение пространства 𝑅1 в 𝑅1 монотонно.
Пусть [𝑥] обозначает наибольшее из целых чисел, не превосходящих 𝑥, т. е. [𝑥] — такое целое число, что
𝑥 − 1 < [𝑥] ≤ 𝑥; пусть (𝑥) = 𝑥 − [𝑥] обозначает дробную часть числа 𝑥. Какие разрывы имеют функции
[𝑥] и (𝑥)?
Пусть 𝑓 — вещественная функция, заданная на (𝑎, 𝑏). Доказать, что множество точек, в которых 𝑓
имеет простой разрыв, не более чем счетно. Указание. Пусть 𝐸 — множество, на котором 𝑓 (𝑥−) < 𝑓 (𝑥+).
Каждой точке 𝑥 множества 𝐸 сопоставим тройку (𝑝, 𝑞, 𝑟) рациональных чисел, таких, что
(a) 𝑓 (𝑥−) < 𝑝 < 𝑓 (𝑥+),
(b) при 𝑎 < 𝑞 < 𝑡 < 𝑥 имеем 𝑓 (𝑡) < 𝑝,
(c) при 𝑥 < 𝑡 < 𝑟 < 𝑏 имеем 𝑓 (𝑡) > 𝑝.
Множество всех таких троек не более чем счетно. Показать, что каждой такой тройке отвечает не более
одной точки множества 𝐸. Поступить подобным образом и в случае других типов простого разрыва.
18. Каждое рациональное число 𝑥 можно записать в виде 𝑥 = 𝑚/𝑛, где 𝑛 > 0, а 𝑚 и 𝑛 — целые числа, не
имеющие общих делителей. Если 𝑥 = 0, то мы полагаем 𝑛 = 1. Рассмотрим функцию 𝑓 , заданную на 𝑅1
равенством
0 (𝑥 иррационально),
𝑓 (𝑥) = 1
𝑚
{ 𝑛 (𝑥 = 𝑛 ).
Доказать, что 𝑓 непрерывна в каждой иррациональной точке и имеет простой разрыв в каждой рациональной точке.
19. Пусть 𝑓 — вещественная функция с областью определения 𝑅1 , обладающая свойством промежуточного
значения: если 𝑓 (𝑎) < 𝑐 < 𝑓 (𝑏), то 𝑓 (𝑥) = 𝑐 для некоторого 𝑥 между 𝑎 и 𝑏.
Пусть также для каждого рационального 𝑟 множество всех 𝑥, для которых 𝑓 (𝑥) = 𝑟, замкнуто.
Доказать, что 𝑓 непрерывна.
Указание. Если 𝑥𝑛 → 𝑥0 , но 𝑓 (𝑥𝑛 ) > 𝑟 > 𝑓 (𝑥0 ) для некоторого 𝑟 и всех 𝑛, то 𝑓 (𝑡𝑛 ) = 𝑟 для некоторого 𝑡𝑛
между 𝑥0 и 𝑥𝑛 ; поэтому 𝑡𝑛 → 𝑥0 . Прийти к противоречию. (N. J. Fine, Amer. Math. Monthly, vol. 73, 1966,
p. 782.)
20. Пусть 𝐸 — непустое подмножество метрического пространства 𝑋, Определим расстояние от 𝑥 ∈ 𝑋 до 𝐸
как
𝜌𝐸 (𝑥) = inf 𝑑(𝑥, 𝑧).
𝑧∈𝐸
(a) Доказать, что 𝜌𝐸 (𝑥) = 0 тогда и только тогда, когда 𝑥 ∈ 𝐸.̄
(b) Доказать, что 𝜌𝐸 — равномерно непрерывная функция на 𝑋, показав, что
|𝜌𝐸 (𝑥) − 𝜌𝐸 (𝑦)| ≤ 𝑑(𝑥, 𝑦)
для всех 𝑥 ∈ 𝑋, 𝑦 ∈ 𝑋.
Указание. Мы имеем 𝜌𝐸 (𝑥) ≤ 𝑑(𝑥, 𝑧) ≤ 𝑑(𝑥, 𝑦) + 𝑑(𝑦, 𝑧), так что
𝜌𝐸 (𝑥) ≤ 𝑑(𝑥, 𝑦) + 𝜌𝐸 (𝑦).
21. Пусть 𝐾 и 𝐹 — непересекающиеся подмножества метрического пространства 𝑋, причем 𝐾 компактно, 𝐹
замкнуто. Доказать, что существует 𝛿 > 0, такое, что 𝑑(𝑝, 𝑞) > 𝛿, если 𝑝 ∈ 𝐾, 𝑞 ∈ 𝐹 . Указание. Функция
𝜌𝐹 — положительная непрерывная функция на 𝐾.
Показать, что это утверждение может оказаться неверным, если ни одно из двух непересекающихся
замкнутых множеств не компактно.
22. Пусть 𝐴 и 𝐵 — непустые непересекающиеся замкнутые множества в метрическом пространстве 𝑋. Положим
𝜌𝐴 (𝑝)
𝑓 (𝑝) =
(𝑝 ∈ 𝑋).
𝜌𝐴 (𝑝) + 𝜌𝐵 (𝑝)
Показать, что 𝑓 — непрерывная функция на 𝑋, множество значений которой содержится в сегменте
[0, 1], что 𝑓 (𝑝) = 0 в точности на 𝐴 и 𝑓 (𝑝) = 1 в точности на 𝐵. Тем самым установлено утверждение,
обратное к теореме упражнения 3: каждое замкнутое множество 𝐴 ⊂ 𝑋 является нуль–множеством 𝑍(𝑓 )
для некоторой непрерывной вещественной функции 𝑓 на 𝑋. Полагая
𝑉 = 𝑓 −1 ([0, 21 )),
75
𝑊 = 𝑓 −1 (( 21 , 1]),
показать, что 𝑉 и 𝑊 открыты, не пересекаются и что 𝐴 ⊂ 𝑉 , 𝐵 ⊂ 𝑊 . (Таким образом, пара непересекающихся замкнутых множеств в метрическом пространстве может быть накрыта парой непересекающихся
открытых множеств. Это свойство метрических пространств называют нормальностью.)
23. Вещественная функция 𝑓 , заданная на (𝑎, 𝑏), называется выпуклой, если
𝑓 (𝜆𝑥 + (1 − 𝜆)𝑦) ≤ 𝜆𝑓 (𝑥) + (1 − 𝜆)𝑓 (𝑦)
при любых 𝑎 < 𝑥 < 𝑏, 𝑎 < 𝑦 < 𝑏, 0 < 𝜆 < 1. Доказать, что каждая выпуклая функция непрерывна.
Доказать, что каждая возрастающая выпуклая функция от выпуклой функции выпукла (например,
если 𝑓 выпукла, то 𝑒𝑓 тоже выпукла).
Если 𝑓 выпукла на (𝑎, 𝑏) и если 𝑎 < 𝑠 < 𝑡 < 𝑢 < 𝑏, показать, что
𝑓 (𝑡) − 𝑓 (𝑠)
𝑓 (𝑢) − 𝑓 (𝑠)
𝑓 (𝑢) − 𝑓 (𝑡)
≤
≤
.
𝑡−𝑠
𝑢−𝑠
𝑢−𝑡
24. Пусть 𝑓 — непрерывная вещественная функция, заданная на (𝑎, 𝑏), такая, что
𝑓(
𝑓 (𝑥) + 𝑓 (𝑦)
𝑥+𝑦
≤
2 )
2
для всех 𝑥, 𝑦 ∈ (𝑎, 𝑏). Доказать, что 𝑓 выпукла.
25. Пусть 𝐴 ⊂ 𝑅𝑘 и 𝐵 ⊂ 𝑅𝑘 . Определим 𝐴 + 𝐵 как множество всех сумм x + y при x ∈ 𝐴, y ∈ 𝐵.
(a) Пусть 𝐾 компактно, а 𝐶 замкнуто в 𝑅𝑘 . Доказать, что 𝐾 + 𝐶 замкнуто.
Указание. Возьмем z ∉ 𝐾 + 𝐶, положим 𝐹 = z − 𝐶 (множество всех z − y при y ∈ 𝐶). Тогда 𝐾 и
𝐹 не пересекаются. Выберем 𝛿, как в упражнении 21. Показать, что открытый шар с центром z и
радиусом 𝛿 не пересекается с 𝐾 + 𝐶.
(b) Пусть 𝛼 — иррациональное вещественное число. Пусть 𝐶1 — множество всех целых чисел, а 𝐶2 —
множество всех 𝑛𝛼 при 𝑛 ∈ 𝐶1 . Показать, что 𝐶1 и 𝐶2 — замкнутые подмножества пространства
𝑅1 , сумма которых 𝐶1 + 𝐶2 не является замкнутой, показав, что 𝐶1 + 𝐶2 — счетное всюду плотное
подмножество пространства 𝑅1 .
26. Пусть 𝑋, 𝑌 , 𝑍 — метрические пространства, а 𝑌 компактно. Пусть 𝑓 отображает 𝑋 в 𝑌 , а 𝑔 — непрерывное взаимно однозначное отображение пространства 𝑌 в 𝑍. Положим ℎ(𝑥) = 𝑔(𝑓 (𝑥)) для 𝑥 ∈ 𝑋.
Доказать, что 𝑓 равномерно непрерывно, если ℎ равномерно непрерывно.
Указание. 𝑔 −1 имеет компактную область определения 𝑔(𝑌 ), и 𝑓 (𝑥) = 𝑔 −1 (ℎ(𝑥)).
Доказать также, что 𝑓 непрерывно, если ℎ непрерывно.
Показать (изменив пример 4.21 или найдя другой пример), что условие компактности 𝑌 нельзя опустить,
даже если 𝑋 и 𝑍 компактны.
76
Глава 5
Дифференцирование
В этой главе (за исключением последнего раздела) мы будем заниматься только вещественными
функциями, определенными на сегментах или интервалах. Это вызвано не просто соображениями
удобства — при переходе от вещественных к векторнозначным функциям возникают значительные
трудности. Дифференцирование функций, определенных на 𝑅𝑘 , будет рассматриваться в гл. 9.
Производная вещественной функции
5.1 Определение. Пусть 𝑓 — вещественная функция, определенная на [𝑎, 𝑏]. Взяв произвольное
число 𝑥 ∈ [𝑎, 𝑏], составим отношение
(1)
𝜑(𝑡) =
𝑓 (𝑡) − 𝑓 (𝑥)
𝑡−𝑥
(𝑎 < 𝑡 < 𝑏, 𝑡 ≠ 𝑥)
и положим
𝑓 ′ (𝑥) = lim 𝜑(𝑡),
(2)
𝑡→𝑥
если этот предел существует в соответствии с определением 4.1.
Таким образом, с функцией 𝑓 связана функция 𝑓 ′ , область определения которой — множество
всех точек 𝑥, в которых существует предел (2); 𝑓 ′ называется производной функции 𝑓 .
Если 𝑓 ′ определена в точке 𝑥, то мы будем говорить, что 𝑓 дифференцируема в 𝑥. Если 𝑓 ′
определена в каждой точке некоторого множества 𝐸 ⊂ [𝑎, 𝑏], то мы будем говорить, что 𝑓 дифференцируема на 𝐸.
В формуле (2) можно брать правосторонний или левосторонний предел; это приводит к определению правосторонней и левосторонней производной. В частности, в концах сегмента 𝑎 и 𝑏 производная, если она существует, является соответственно правосторонней и левосторонней. Мы,
однако, не будем подробно рассматривать такие производные.
Если 𝑓 определена на интервале (𝑎, 𝑏) и если 𝑎 < 𝑥 < 𝑏, то 𝑓 ′ (𝑥) определяется равенствами (1) и
(2), как и выше. Но 𝑓 ′ (𝑎) и 𝑓 ′ (𝑏) в этом случае не определены.
5.2 Теорема. Пусть 𝑓 определена на [𝑎, 𝑏]. Если 𝑓 дифференцируема в точке 𝑥 ∈ [𝑎, 𝑏], то она
непрерывна в этой точке.
Доказательство. По теореме 4.4 при 𝑡 → 𝑥 мы имеем
𝑓 (𝑡) − 𝑓 (𝑥) =
𝑓 (𝑡) − 𝑓 (𝑥)
⋅ (𝑡 − 𝑥) → 𝑓 ′ (𝑥) ⋅ 0 = 0.
𝑡−𝑥
Теорема, обратная к только что доказанной, неверна. Легко построить непрерывные функции,
не дифференцируемые в изолированных точках. В гл. 7 мы познакомимся даже с такой функцией,
которая, будучи непрерывной на всей прямой, не дифференцируема ни в одной точке!
5.3 Теорема. Пусть 𝑓 и 𝑔 определены на [𝑎, 𝑏] и дифференцируемы в точке 𝑥 ∈ [𝑎, 𝑏]. Тогда 𝑓 + 𝑔,
𝑓 𝑔 и 𝑓 /𝑔 дифференцируемы в точке 𝑥 и
77
(a) (𝑓 + 𝑔)′ (𝑥) = 𝑓 ′ (𝑥) + 𝑔 ′ (𝑥);
(b) (𝑓 𝑔)′ (𝑥) = 𝑓 ′ (𝑥)𝑔(𝑥) + 𝑓 (𝑥)𝑔 ′ (𝑥);
′
𝑓
𝑔(𝑥)𝑓 ′ (𝑥) − 𝑔 ′ (𝑥)𝑓 (𝑥)
(c)
(𝑥) =
.
(𝑔)
𝑔 2 (𝑥)
В (c) мы, разумеется, предполагаем, что 𝑔(𝑥) ≠ 0.
Доказательство. Утверждение (a) следует из теоремы 4.4. Пусть ℎ = 𝑓 𝑔. Тогда
ℎ(𝑡) − ℎ(𝑥) = 𝑓 (𝑡)[𝑔(𝑡) − 𝑔(𝑥)] + 𝑔(𝑥)[𝑓 (𝑡) − 𝑓 (𝑥)].
Если мы разделим это равенство на 𝑡 − 𝑥 и заметим, что 𝑓 (𝑡) → 𝑓 (𝑥) при 𝑡 → 𝑥 (теорема 5.2), то
получим (b). Наконец, пусть ℎ = 𝑓 /𝑔. Тогда
𝑓 (𝑡) − 𝑓 (𝑥)
𝑔(𝑡) − 𝑔(𝑥)
ℎ(𝑡) − ℎ(𝑥)
1
=
− 𝑓 (𝑥)
𝑔(𝑥)
.
𝑡−𝑥
𝑔(𝑡)𝑔(𝑥) [
𝑡−𝑥
𝑡−𝑥 ]
Устремляя 𝑡 к 𝑥 и применяя теоремы 4.4 и 5.2, мы получаем (c).
5.4 Примеры. Производная каждой постоянной функции равна, очевидно, нулю. Если 𝑓 определена как 𝑓 (𝑥) = 𝑥, то 𝑓 ′ (𝑥) = 1. Повторное применение утверждений (b) и (c) показывает, что 𝑥𝑛
дифференцируема и что ее производная равна 𝑛𝑥𝑛−1 , каково бы ни было целое 𝑛 (если 𝑛 < 0, то мы
должны ограничиться 𝑥 ≠ 0). Таким образом, любой многочлен дифференцируем. То же верно в
отношении любой рациональной функции, за исключением точек, в которых обращается в нуль ее
знаменатель.
Следующая теорема известна как правило дифференцирования сложной функции. Она позволяет дифференцировать композиции функций и, возможно, является наиболее важной теоремой,
касающейся производных. Более общие ее варианты встретятся нам в гл. 9.
5.5 Теорема. Пусть 𝑓 непрерывна на [𝑎, 𝑏], 𝑓 ′ (𝑥) существует в некоторой точке 𝑥 ∈ [𝑎, 𝑏], 𝑔
определена на сегменте 𝐼, содержащем множество значений функции 𝑓 , и 𝑔 дифференцируема в
точке 𝑓 (𝑥). Если
ℎ(𝑡) = 𝑔(𝑓 (𝑡))
(𝑎 ≤ 𝑡 ≤ 𝑏),
то ℎ дифференцируема в 𝑥 и
(3)
ℎ′ (𝑥) = 𝑔 ′ (𝑓 (𝑥))𝑓 ′ (𝑥).
Доказательство. Пусть 𝑦 = 𝑓 (𝑥). По определению производной имеем
(4)
𝑓 (𝑡) − 𝑓 (𝑥) = (𝑡 − 𝑥)[𝑓 ′ (𝑥) + 𝑢(𝑡)],
(5)
𝑔(𝑠) − 𝑔(𝑦) = (𝑠 − 𝑦)[𝑔 ′ (𝑦) + 𝑣(𝑠)],
где 𝑡 ∈ [𝑎, 𝑏], 𝑠 ∈ 𝐼 и 𝑢(𝑡) → 0 при 𝑡 → 𝑥, 𝑣(𝑠) → 0 при 𝑠 → 𝑦. Пусть 𝑠 = 𝑓 (𝑡). Используя сначала (5), а
затем (4), получим
ℎ(𝑡) − ℎ(𝑥) = 𝑔(𝑓 (𝑡)) − 𝑔(𝑓 (𝑥))
= [𝑓 (𝑡) − 𝑓 (𝑥)] ⋅ [𝑔 ′ (𝑦) + 𝑣(𝑠)]
= (𝑡 − 𝑥) ⋅ [𝑓 ′ (𝑥) + 𝑢(𝑡)] ⋅ [𝑔 ′ (𝑦) + 𝑣(𝑠)],
или, если 𝑡 ≠ 𝑥,
(6)
ℎ(𝑡) − ℎ(𝑥)
= [𝑔 ′ (𝑦) + 𝑣(𝑠)] ⋅ [𝑓 ′ (𝑥) + 𝑢(𝑡)].
𝑡−𝑥
Устремляя 𝑡 к 𝑥, мы видим, что, в силу непрерывности функции 𝑓 , 𝑠 → 𝑦, поэтому правая часть
равенства (6) стремится к 𝑔 ′ (𝑦)𝑓 ′ (𝑥), откуда и следует (3).
5.6 Примеры.
78
(a) Пусть функция 𝑓 определена как
(7)
𝑓 (𝑥) =
𝑥 sin 𝑥1
{0
(𝑥 ≠ 0),
(𝑥 = 0).
Приняв без доказательства, что производная функции sin 𝑥 равна cos 𝑥 (мы будем рассматривать тригонометрические функции в гл. 8), мы можем применить теоремы 5.3 и 5.5, если
𝑥 ≠ 0, и получить
𝑓 ′ (𝑥) = sin
(8)
1
1 1
− cos
𝑥 𝑥
𝑥
(𝑥 ≠ 0).
При 𝑥 = 0 эти теоремы уже неприменимы, так как 1/𝑥 там не определено, и мы сошлемся
непосредственно на определение: при 𝑡 ≠ 0
𝑓 (𝑡) − 𝑓 (0)
1
= sin .
𝑡−0
𝑡
При 𝑡 → 0 это отношение не стремится ни к какому пределу, так что 𝑓 ′ (0) не существует.
(b) Пусть функция 𝑓 определена как
(9)
𝑓 (𝑥) =
𝑥2 sin 𝑥1
{0
(𝑥 ≠ 0),
(𝑥 = 0).
Как и выше, мы получаем
𝑓 ′ (𝑥) = 2𝑥 sin
(10)
1
1
− cos
𝑥
𝑥
(𝑥 ≠ 0).
При 𝑥 = 0 мы вспомним определение и получим
|
𝑓 (𝑡) − 𝑓 (0)
1
= 𝑡 sin | ≤ |𝑡|
𝑡−0 | |
𝑡
(𝑡 ≠ 0);
устремляя 𝑡 к 0, мы видим, что
(11)
𝑓 ′ (0) = 0.
Таким образом, 𝑓 дифференцируема во всех точках 𝑥, но 𝑓 ′ не является непрерывной функцией, так как cos(1/𝑥) в (10) не стремится ни к какому пределу при 𝑥 → 0.
Теоремы о среднем значении
5.7 Определение. Пусть 𝑓 — вещественная функция, определенная на метрическом пространстве
𝑋. Будем говорить, что 𝑓 имеет локальный максимум в точке 𝑝 ∈ 𝑋, если существует 𝛿 > 0, такое,
что 𝑓 (𝑞) ≤ 𝑓 (𝑝) при всех 𝑞 ∈ 𝑋, таких, что 𝑑(𝑝, 𝑞) < 𝛿.
Локальные минимумы определяются сходным образом.
Наша следующая теорема лежит в основе многих применений дифференцирования.
5.8 Теорема. Пусть 𝑓 определена на [𝑎, 𝑏]; если 𝑓 имеет локальный максимум в точке 𝑥 ∈ (𝑎, 𝑏)
и если существует 𝑓 ′ (𝑥), то 𝑓 ′ (𝑥) = 0.
Аналогичное утверждение, относящееся к локальным минимумам, конечно, тоже верно.
Доказательство. Выберем 𝛿 в соответствии с определением 5.7, так что
𝑎 < 𝑥 − 𝛿 < 𝑥 < 𝑥 + 𝛿 < 𝑏.
Если 𝑥 − 𝛿 < 𝑡 < 𝑥, то
𝑓 (𝑡) − 𝑓 (𝑥)
≥ 0.
𝑡−𝑥
79
Устремляя 𝑡 к 𝑥, мы видим, что 𝑓 ′ (𝑥) ≥ 0.
Если 𝑥 < 𝑡 < 𝑥 + 𝛿, то
𝑓 (𝑡) − 𝑓 (𝑥)
≤ 0,
𝑡−𝑥
откуда следует, что 𝑓 ′ (𝑥) ≤ 0. Значит, 𝑓 ′ (𝑥) = 0.
5.9 Теорема. Если 𝑓 и 𝑔 — непрерывные вещественные функции на [𝑎, 𝑏], дифференцируемые на
(𝑎, 𝑏), то существует точка 𝑥 ∈ (𝑎, 𝑏), в которой
[𝑓 (𝑏) − 𝑓 (𝑎)]𝑔 ′ (𝑥) = [𝑔(𝑏) − 𝑔(𝑎)]𝑓 ′ (𝑥).
Заметим, что дифференцируемость на концах сегмента не требуется.
Доказательство. Положим
ℎ(𝑡) = [𝑓 (𝑏) − 𝑓 (𝑎)]𝑔(𝑡) − [𝑔(𝑏) − 𝑔(𝑎)]𝑓 (𝑡)
(𝑎 ≤ 𝑡 ≤ 𝑏).
Тогда ℎ непрерывна на [𝑎, 𝑏], дифференцируема на (𝑎, 𝑏) и
(12)
ℎ(𝑎) = 𝑓 (𝑏)𝑔(𝑎) − 𝑓 (𝑎)𝑔(𝑏) = ℎ(𝑏).
Для доказательства теоремы мы должны проверить, что ℎ′ (𝑥) = 0 при некотором 𝑥 ∈ (𝑎, 𝑏).
Если ℎ постоянна, то это выполняется при любом 𝑥 ∈ (𝑎, 𝑏). Если ℎ(𝑡) > ℎ(𝑎) при некотором
𝑡 ∈ (𝑎, 𝑏), то пусть 𝑥 — точка на [𝑎, 𝑏], в которой ℎ достигает своего максимума (теорема 4.16).
В силу (12), 𝑥 ∈ (𝑎, 𝑏), и теорема 5.8 показывает, что ℎ′ (𝑥) = 0. Если ℎ(𝑡) < ℎ(𝑎) при некотором
𝑡 ∈ (𝑎, 𝑏), то можно применить такое же рассуждение, выбрав в качестве 𝑥 точку на [𝑎, 𝑏], в которой
ℎ достигает своего минимума.
Эту теорему часто называют обобщенной теоремой о среднем значении; следующий ее частный
случай называют теоремой о среднем значении.
5.10 Теорема. Если 𝑓 — вещественная непрерывная функция на [𝑎, 𝑏], дифференцируемая на (𝑎, 𝑏),
то существует точка 𝑥 ∈ (𝑎, 𝑏), в которой
𝑓 (𝑏) − 𝑓 (𝑎) = (𝑏 − 𝑎)𝑓 ′ (𝑥).
Доказательство. Положим 𝑔(𝑥) = 𝑥 в теореме 5.9.
5.11 Теорема. Пусть 𝑓 дифференцируема на (𝑎, 𝑏).
(a) Если 𝑓 ′ (𝑥) ≥ 0 при всех 𝑥 ∈ (𝑎, 𝑏), то 𝑓 монотонно возрастает.
(b) Если 𝑓 ′ (𝑥) = 0 при всех 𝑥 ∈ (𝑎, 𝑏), то 𝑓 постоянна.
(c) Если 𝑓 ′ (𝑥) ≤ 0 при всех 𝑥 ∈ (𝑎, 𝑏), то 𝑓 монотонно убывает.
Доказательство. Все эти заключения можно усмотреть из равенства
𝑓 (𝑥2 ) − 𝑓 (𝑥1 ) = (𝑥2 − 𝑥1 )𝑓 ′ (𝑥),
верного для любой пары чисел 𝑥1 , 𝑥2 на (𝑎, 𝑏), при некотором 𝑥 между 𝑥1 и 𝑥2 .
Непрерывность производных
Мы уже видели (пример 5.6(b)), что функция 𝑓 может иметь производную 𝑓 ′ , которая существует
во всех точках, но разрывна в некоторых точках. Однако не каждая функция может быть производной. В частности, производные, которые существуют в каждой точке некоторого сегмента,
так же как и функции, непрерывные на сегменте, обладают следующим важным свойством: они
принимают промежуточные значения (ср. с теоремой 4.23). Точная формулировка такова.
5.12 Теорема. Пусть 𝑓 — вещественная дифференцируемая функция на [𝑎, 𝑏], и пусть 𝑓 ′ (𝑎) <
𝜆 < 𝑓 ′ (𝑏). Тогда существует точка 𝑥 ∈ (𝑎, 𝑏), такая, что 𝑓 ′ (𝑥) = 𝜆.
80
Подобный результат верен, конечно, и тогда, когда 𝑓 ′ (𝑎) > 𝑓 ′ (𝑏).
Доказательство. Положим 𝑔(𝑡) = 𝑓 (𝑡) − 𝜆𝑡. Тогда 𝑔 ′ (𝑎) < 0, так что 𝑔(𝑡1 ) < 𝑔(𝑎) для некоторого
𝑡1 ∈ (𝑎, 𝑏), и 𝑔 ′ (𝑏) > 0, так что 𝑔(𝑡2 ) < 𝑔(𝑏) для некоторого 𝑡2 ∈ (𝑎, 𝑏). Значит, 𝑔 достигает своего
минимума на [𝑎, 𝑏] (теорема 4.16) в некоторой точке 𝑥, такой, что 𝑎 < 𝑥 < 𝑏. По теореме 5.8 𝑔 ′ (𝑥) = 0.
Значит, 𝑓 ′ (𝑥) = 𝜆.
Следствие. Если 𝑓 дифференцируема на [𝑎, 𝑏], то 𝑓 ′ не имеет простых разрывов на [𝑎, 𝑏].
Однако 𝑓 ′ вполне может иметь разрывы второго рода.
Правило Лопиталя
Следующая теорема часто бывает полезной при вычислении пределов.
5.13 Теорема. Пусть 𝑓 и 𝑔 вещественны и дифференцируемы на (𝑎, 𝑏) и 𝑔 ′ (𝑥) ≠ 0 при всех
𝑥 ∈ (𝑎, 𝑏), где −∞ ≤ 𝑎 < 𝑏 ≤ +∞. Пусть
(13)
𝑓 ′ (𝑥)
→ 𝐴 при 𝑥 → 𝑎.
𝑔 ′ (𝑥)
Если
(14)
𝑓 (𝑥) → 0 и 𝑔(𝑥) → 0 при 𝑥 → 𝑎,
или если
(15)
𝑔(𝑥) → +∞ при 𝑥 → 𝑎,
то
(16)
𝑓 (𝑥)
→ 𝐴 при 𝑥 → 𝑎.
𝑔(𝑥)
Аналогичное утверждение, конечно, верно и тогда, когда 𝑥 → 𝑏 или когда в (15) 𝑔(𝑥) → −∞.
Заметим, что сейчас мы используем понятие предела в расширенном смысле в соответствии с
определением 4.33.
Доказательство. Сначала рассмотрим случай, когда −∞ ≤ 𝐴 < +∞. Выберем вещественное число
𝑞, такое, что 𝐴 < 𝑞, а затем выберем 𝑟 так, чтобы 𝐴 < 𝑟 < 𝑞. Согласно (13), существует точка
𝑐 ∈ (𝑎, 𝑏), такая, что при 𝑎 < 𝑥 < 𝑐 имеем
(17)
𝑓 ′ (𝑥)
< 𝑟.
𝑔 ′ (𝑥)
Если 𝑎 < 𝑥 < 𝑦 < 𝑐, то, как показывает теорема 5.9, существует точка 𝑡 ∈ (𝑥, 𝑦), такая, что
(18)
𝑓 (𝑥) − 𝑓 (𝑦) 𝑓 ′ (𝑡)
= ′ < 𝑟.
𝑔(𝑥) − 𝑔(𝑦)
𝑔 (𝑡)
Допустим, что выполнено (14). Устремляя в (18) 𝑥 к 𝑎, мы видим, что
(19)
𝑓 (𝑦)
≤𝑟<𝑞
𝑔(𝑦)
(𝑎 < 𝑦 < 𝑐).
Теперь допустим, что выполнено (15). Сохраняя 𝑦 в (18) фиксированным, мы можем выбрать
такую точку 𝑐1 ∈ (𝑎, 𝑦), что 𝑔(𝑥) > 𝑔(𝑦) и 𝑔(𝑥) > 0, если 𝑎 < 𝑥 < 𝑐1 . Умножая (18) на [𝑔(𝑥) − 𝑔(𝑦)]/𝑔(𝑥),
мы получим
(20)
𝑔(𝑦) 𝑓 (𝑦)
𝑓 (𝑥)
<𝑟−𝑟
+
𝑔(𝑥)
𝑔(𝑥) 𝑔(𝑥)
81
(𝑎 < 𝑥 < 𝑐1 ).
Если в (20) устремить 𝑥 к 𝑎, то, как показывает (15), можно найти точку 𝑐2 ∈ (𝑎, 𝑐1 ), такую, что
(21)
𝑓 (𝑥)
<𝑞
𝑔(𝑥)
(𝑎 < 𝑥 < 𝑐2 ).
Итак, (19) и (21) показывают, что для любого 𝑞, подчиненного единственному условию 𝐴 < 𝑞,
найдется точка 𝑐2 , такая, что 𝑓 (𝑥)/𝑔(𝑥) < 𝑞, если 𝑎 < 𝑥 < 𝑐2 .
Точно таким же образом, если −∞ < 𝐴 ≤ +∞, а 𝑝 выбрано так, что 𝑝 < 𝐴, мы можем найти
точку 𝑐3 ∈ (𝑎, 𝑏), такую, что
(22)
𝑝<
𝑓 (𝑥)
𝑔(𝑥)
(𝑎 < 𝑥 < 𝑐3 ).
Из этих двух утверждений следует (16).
Производные высших порядков
5.14 Определение. Если 𝑓 имеет производную 𝑓 ′ на некотором сегменте и если 𝑓 ′ в свою очередь
дифференцируема, то производную функции 𝑓 ′ мы будем обозначать через 𝑓 ″ и называть второй
производной функции 𝑓 . Продолжая таким образом, мы получим функции
𝑓 , 𝑓 ′ , 𝑓 ″ , 𝑓 (3) , … , 𝑓 (𝑛) ,
каждая из которых служит производной для предшествующей. Функция 𝑓 (𝑛) называется 𝑛-ой производной, или производной порядка 𝑛 функции 𝑓 .
Для того чтобы 𝑓 (𝑛) (𝑥) существовала в точке 𝑥, 𝑓 (𝑛−1) (𝑡) должна существовать в окрестности
точки 𝑥 (или в односторонней окрестности, если 𝑥 является концом сегмента, на котором определена
𝑓 ) и 𝑓 (𝑛−1) должна быть дифференцируемой в 𝑥. Так как 𝑓 (𝑛−1) должна существовать в некоторой
окрестности точки 𝑥, 𝑓 (𝑛−2) должна быть дифференцируемой в этой окрестности.
Теорема Тейлора
5.15 Теорема. Допустим, что 𝑓 — вещественная функция на [𝑎, 𝑏], 𝑛 — положительное целое
число, 𝑓 (𝑛−1) непрерывна на [𝑎, 𝑏], 𝑓 (𝑛) (𝑡) существует при любом 𝑡 ∈ (𝑎, 𝑏). Пусть 𝛼, 𝛽 — различные
точки сегмента [𝑎, 𝑏]. Положим
𝑛−1
(23)
𝑃 (𝑡) =
𝑓 (𝑘) (𝛼)
(𝑡 − 𝛼)𝑘 .
∑ 𝑘!
𝑘=0
Тогда существует точка 𝑥 между 𝛼 и 𝛽, такая, что
(24)
𝑓 (𝛽) = 𝑃 (𝛽) +
𝑓 (𝑛) (𝑥)
(𝛽 − 𝛼)𝑛 .
𝑛!
При 𝑛 = 1 это утверждение превращается в теорему о среднем значении. В общем случае теорема
показывает, что 𝑓 можно приблизить многочленом степени 𝑛 − 1, и что (24) позволяет оценить
погрешность, если известна верхняя граница величины |𝑓 (𝑛) (𝑥)|.
Доказательство. Пусть 𝑀 — число, определяемое равенством
(25)
𝑓 (𝛽) = 𝑃 (𝛽) + 𝑀(𝛽 − 𝛼)𝑛 ,
и пусть
(26)
𝑔(𝑡) = 𝑓 (𝑡) − 𝑃 (𝑡) − 𝑀(𝑡 − 𝛼)𝑛
(𝑎 ≤ 𝑡 ≤ 𝑏).
Мы должны показать, что 𝑛!𝑀 = 𝑓 (𝑛) (𝑥) при некотором 𝑥 между 𝛼 и 𝛽. Согласно (23) и (26),
(27)
𝑔 (𝑛) (𝑡) = 𝑓 (𝑛) (𝑡) − 𝑛!𝑀
82
(𝑎 < 𝑡 < 𝑏).
Значит, доказательство будет закончено, если мы покажем, что 𝑔 (𝑛) (𝑥) = 0 при некотором 𝑥 между
𝛼 и 𝛽.
Поскольку 𝑃 (𝑘) (𝛼) = 𝑓 (𝑘) (𝛼) при 𝑘 = 0, … , 𝑛 − 1, мы имеем
(28)
𝑔(𝛼) = 𝑔 ′ (𝛼) = ⋯ = 𝑔 (𝑛−1) (𝛼) = 0.
Число 𝑀 выбрано так, что 𝑔(𝛽) = 0, поэтому, согласно теореме о среднем, 𝑔 ′ (𝑥1 ) = 0 при некотором
𝑥1 между 𝛼 и 𝛽. Так как 𝑔 ′ (𝛼) = 0, то подобным же образом заключаем, что 𝑔 ″ (𝑥2 ) = 0 при некотором
𝑥2 между 𝛼 и 𝑥1 . После 𝑛 шагов мы приходим к выводу, что 𝑔 (𝑛) (𝑥𝑛 ) = 0 при некотором 𝑥𝑛 между 𝛼
и 𝑥𝑛−1 , т. е. между 𝛼 и 𝛽.
Дифференцирование векторнозначных функций
5.16 Замечания. Определение 5.1 без всяких изменений применимо к комплексным функциям
𝑓 , определенным на [𝑎, 𝑏], при этом теоремы 5.2 и 5.3, так же как и их доказательства, остаются
верными. Если 𝑓1 и 𝑓2 — вещественная и мнимая части функции 𝑓 , т. е. если
𝑓 (𝑡) = 𝑓1 (𝑡) + 𝑖𝑓2 (𝑡)
при 𝑎 ≤ 𝑡 ≤ 𝑏, где 𝑓1 (𝑡) и 𝑓2 (𝑡) вещественны, то, очевидно,
(29)
𝑓 ′ (𝑥) = 𝑓1′ (𝑥) + 𝑖𝑓2′ (𝑥),
причем 𝑓 дифференцируема в 𝑥 тогда и только тогда, когда обе 𝑓1 и 𝑓2 дифференцируемы в 𝑥.
Переходя к общим векторнозначным функциям, т. е. к функциям f, отображающим [𝑎, 𝑏] в
некоторое 𝑅𝑘 , мы все еще можем применить определение 5.1 для определения f′ (𝑥). Теперь 𝜑(𝑡) в
(1) при каждом 𝑡 является точкой пространства 𝑅𝑘 , а предел в (2) вычисляется по норме этого
пространства. Иными словами, f′ (𝑥) — это та точка пространства 𝑅𝑘 (если она существует), для
которой
(30)
lim
𝑡→𝑥 |
f(𝑡) − f(𝑥)
− f′ (𝑥) = 0,
|
𝑡−𝑥
и f′ — снова функция со значениями в 𝑅𝑘 .
Если 𝑓1 , … , 𝑓𝑘 — компоненты функции f, определенные в теореме 4.10, то
(31)
f′ = (𝑓1′ , … , 𝑓𝑘′ ),
и f дифференцируема в точке 𝑥 тогда и только тогда, когда каждая из функций 𝑓1 , … , 𝑓𝑘 дифференцируема в 𝑥.
Теорема 5.2 остается верной и в этом случае, то же относится и к теореме 5.3(a) и (b), если 𝑓 𝑔
заменить скалярным произведением f ⋅ g (см. определение 4.3).
Однако, когда мы обращаемся к теореме о среднем значении и к одному из ее следствий, а
именно правилу Лопиталя, положение меняется. Следующие два примера показывают, что оба эти
результата неверны для комплекснозначных функций.
5.17 Пример. Положим для вещественного 𝑥
(32)
𝑓 (𝑥) = 𝑒𝑖𝑥 = cos 𝑥 + 𝑖 sin 𝑥
(последнее выражение можно рассматривать как определение комплексной показательной функции
𝑒𝑖𝑥 ; эти функции будут подробно рассмотрены в гл. 8). Тогда
(33)
𝑓 (2𝜋) − 𝑓 (0) = 1 − 1 = 0,
но
(34)
𝑓 ′ (𝑥) = 𝑖𝑒𝑖𝑥 ,
так что |𝑓 ′ (𝑥)| = 1 при всех вещественных 𝑥.
Таким образом, в этом случае теорема 5.10 неверна.
83
5.18 Пример. На интервале (0, 1) определим 𝑓 (𝑥) = 𝑥 и
2
𝑔(𝑥) = 𝑥 + 𝑥2 𝑒𝑖/𝑥 .
(35)
Поскольку |𝑒𝑖𝑡 | = 1 при всех вещественных 𝑡, мы видим, что
𝑓 (𝑥)
= 1.
𝑥→0 𝑔(𝑥)
(36)
lim
Далее,
𝑔 ′ (𝑥) = 1 + {2𝑥 −
(37)
2𝑖 𝑖/𝑥2
𝑒
𝑥}
(0 < 𝑥 < 1),
так что
|𝑔 ′ (𝑥)| ≥ |2𝑥 −
(38)
2𝑖
2
− 1 ≥ − 1.
𝑥|
𝑥
Значит,
𝑓 ′ (𝑥)
1
𝑥
=
≤
| 𝑔 ′ (𝑥) | |𝑔 ′ (𝑥)| 2 − 𝑥
(39)
и поэтому
𝑓 ′ (𝑥)
= 0.
𝑥→0 𝑔 ′ (𝑥)
(40)
lim
Сопоставляя (36) и (40), мы видим, что правило Лопиталя в этом случае не действует. Отметим
еще, что 𝑔 ′ (𝑥) ≠ 0 на (0, 1), в силу (38).
Однако имеется одно следствие теоремы о среднем значении, которое в приложениях почти
так же полезно, как теорема 5.10, и которое остается верным для векторнозначных функций: из
теоремы 5.10 следует, что
|𝑓 (𝑏) − 𝑓 (𝑎)| ≤ (𝑏 − 𝑎) sup |𝑓 ′ (𝑥)|.
(41)
𝑎<𝑥<𝑏
5.19 Теорема. Пусть f — непрерывное отображение сегмента [𝑎, 𝑏] в 𝑅𝑘 , дифференцируемое на
(𝑎, 𝑏). Тогда существует 𝑥 ∈ (𝑎, 𝑏), такое, что
|f(𝑏) − f(𝑎)| ≤ (𝑏 − 𝑎)|f′ (𝑥)|.
Доказательство.
1
Положим z = f(𝑏) − f(𝑎) и определим
𝜑(𝑡) = z ⋅ f(𝑡)
(𝑎 ≤ 𝑡 ≤ 𝑏).
Тогда 𝜑 — вещественная непрерывная функция на [𝑎, 𝑏], дифференцируемая на (𝑎, 𝑏). Теорема о
среднем значении показывает, что
𝜑(𝑏) − 𝜑(𝑎) = (𝑏 − 𝑎)𝜑′ (𝑥) = (𝑏 − 𝑎)z ⋅ f′ (𝑥)
для некоторого 𝑥 ∈ (𝑎, 𝑏). С другой стороны,
𝜑(𝑏) − 𝜑(𝑎) = z ⋅ f(𝑏) − z ⋅ f(𝑎) = z ⋅ z = |z|2 .
Неравенство Шварца теперь дает
|z|2 = (𝑏 − 𝑎)|z ⋅ f′ (𝑥)| ≤ (𝑏 − 𝑎)|z||f′ (𝑥)|.
Значит, |z| ≤ (𝑏 − 𝑎)|f′ (𝑥)|, что и требовалось доказать.
1
В.П.Хавин, переводчик второго издания данной книги на русский язык, добавил это доказательство к оригинальному.
84
Упражнения
1. Пусть 𝑓 определена при всех вещественных 𝑥, и пусть
|𝑓 (𝑥) − 𝑓 (𝑦)| ≤ (𝑥 − 𝑦)2
при всех вещественных 𝑥 и 𝑦. Доказать, что 𝑓 постоянна.
2. Пусть 𝑓 ′ (𝑥) > 0 на (𝑎, 𝑏). Доказать, что 𝑓 строго возрастает на (𝑎, 𝑏). Пусть 𝑔 — функция, обратная к 𝑓 .
Доказать, что 𝑔 дифференцируема и что
𝑔 ′ (𝑓 (𝑥)) =
1
𝑓 ′ (𝑥)
(𝑎 < 𝑥 < 𝑏).
3. Пусть 𝑔 — вещественная функция на 𝑅1 , имеющая ограниченную производную (скажем, |𝑔 ′ | ≤ 𝑀).
Зафиксируем 𝜀 > 0 и положим 𝑓 (𝑥) = 𝑥 + 𝜀𝑔(𝑥). Доказать, что 𝑓 взаимно однозначна, если 𝜀 достаточно
мало (можно определить множество допустимых 𝜀, зависящее лишь от 𝑀).
4. Пусть
𝐶
𝐶𝑛
𝐶
𝐶0 + 1 + ⋯ + 𝑛−1 +
= 0,
2
𝑛
𝑛+1
где 𝐶0 , … , 𝐶𝑛 — вещественные постоянные. Доказать, что уравнение
𝐶0 + 𝐶1 𝑥 + ⋯ + 𝐶𝑛−1 𝑥𝑛−1 + 𝐶𝑛 𝑥𝑛 = 0
имеет хотя бы один вещественный корень между 0 и 1.
5. Пусть 𝑓 определена и дифференцируема при каждом 𝑥 > 0 и 𝑓 ′ (𝑥) → 0 при 𝑥 → +∞. Положим 𝑔(𝑥) =
𝑓 (𝑥 + 1) − 𝑓 (𝑥). Доказать, что 𝑔(𝑥) → 0 при 𝑥 → +∞.
6. Пусть
(a)
(b)
(c)
(d)
𝑓 непрерывна при 𝑥 ≥ 0,
𝑓 ′ (𝑥) существует при 𝑥 > 0,
𝑓 (0) = 0,
𝑓 ′ монотонно возрастает.
Положим
𝑔(𝑥) =
𝑓 (𝑥)
𝑥
(𝑥 > 0).
Доказать, что 𝑔 монотонно возрастает.
7. Пусть 𝑓 ′ (𝑥), 𝑔 ′ (𝑥) существуют, 𝑔 ′ (𝑥) ≠ 0 и 𝑓 (𝑥) = 𝑔(𝑥) = 0. Доказать, что
lim
𝑡→𝑥
𝑓 (𝑡)
𝑓 ′ (𝑥)
= ′
𝑔(𝑡)
𝑔 (𝑥)
(это верно и для комплексных функций).
8. Пусть 𝑓 ′ непрерывна на [𝑎, 𝑏] и 𝜀 > 0. Доказать, что существует 𝛿 > 0, такое, что
|
𝑓 (𝑡) − 𝑓 (𝑥)
− 𝑓 ′ (𝑥) < 𝜀,
|
𝑡−𝑥
если 0 < |𝑡 − 𝑥| < 𝛿, 𝑎 ≤ 𝑥 ≤ 𝑏, 𝑎 ≤ 𝑡 ≤ 𝑏 (это значит, иными словами, что 𝑓 равномерно дифференцируема
на [𝑎, 𝑏], если 𝑓 ′ непрерывна на [𝑎, 𝑏]). Верно ли это также для векторнозначных функций?
9. Пусть 𝑓 — непрерывная вещественная функция на 𝑅1 , про которую известно, что 𝑓 ′ (𝑥) существует при
всех 𝑥 ≠ 0 и что 𝑓 ′ (𝑥) → 3 при 𝑥 → 0. Следует ли из этого, что 𝑓 ′ (0) существует?
10. Пусть 𝑓 и 𝑔 — комплексные дифференцируемые функции на (0, 1), 𝑓 (𝑥) → 0, 𝑔(𝑥) → 0, 𝑓 ′ (𝑥) → 𝐴,
𝑔 ′ (𝑥) → 𝐵 при 𝑥 → 0, где 𝐴 и 𝐵 — комплексные числа, 𝐵 ≠ 0. Доказать, что
lim
𝑥→0
𝑓 (𝑥)
𝐴
= .
𝑔(𝑥)
𝐵
Сравнить с примером 5.18. Указание:
𝑓 (𝑥)
𝑓 (𝑥)
𝑥
𝑥
=
−𝐴 ⋅
+𝐴⋅
.
} 𝑔(𝑥)
𝑔(𝑥) { 𝑥
𝑔(𝑥)
Применить теорему 5.13 к вещественной и мнимой частям дробей 𝑓 (𝑥)/𝑥 и 𝑔(𝑥)/𝑥.
11. Пусть 𝑓 определена в окрестности точки 𝑥, и пусть существует 𝑓 ″ (𝑥). Показать, что
lim
ℎ→0
𝑓 (𝑥 + ℎ) + 𝑓 (𝑥 − ℎ) − 2𝑓 (𝑥)
= 𝑓 ″ (𝑥).
ℎ2
Показать на примере, что этот предел может существовать и тогда, когда 𝑓 ″ (𝑥) не существует.
Указание: Использовать теорему 5.13.
85
12. Пусть 𝑓 (𝑥) = |𝑥|3 . Вычислить 𝑓 ′ (𝑥), 𝑓 ″ (𝑥) при всех вещественных 𝑥 и показать, что 𝑓 (3) (0) не существует.
13. Пусть 𝑎 и 𝑐 — вещественные числа, 𝑐 > 0 и 𝑓 определена на [−1, 1] как
𝑓 (𝑥) =
𝑥𝑎 sin(|𝑥|−𝑐 ) (если 𝑥 ≠ 0),
(если 𝑥 = 0).
{0
Доказать следующие утверждения.
(a)
(b)
(c)
(d)
(e)
(f)
(g)
𝑓 непрерывна тогда и только тогда, когда 𝑎 > 0.
𝑓 ′ (0) существует тогда и только тогда, когда 𝑎 > 1.
𝑓 ′ ограничена тогда и только тогда, когда 𝑎 ≥ 1 + 𝑐.
𝑓 ′ непрерывна тогда и только тогда, когда 𝑎 > 1 + 𝑐.
𝑓 ″ (0) существует тогда и только тогда, когда 𝑎 > 2 + 𝑐.
𝑓 ″ ограничена тогда и только тогда, когда 𝑎 ≥ 2 + 2𝑐.
𝑓 ″ непрерывна тогда и только тогда, когда 𝑎 > 2 + 2𝑐.
14. Пусть 𝑓 — дифференцируемая вещественная функция, определенная на (𝑎, 𝑏). Доказать, что 𝑓 выпукла
тогда и только тогда, когда 𝑓 ′ монотонно возрастает. Далее в предположении, что 𝑓 ″ (𝑥) существует
при каждом 𝑥 ∈ (𝑎, 𝑏), доказать, что 𝑓 выпукла тогда и только тогда, когда 𝑓 ″ (𝑥) ≥ 0 при всех 𝑥 ∈ (𝑎, 𝑏).
15. Пусть 𝑎 ∈ 𝑅1 , 𝑓 — дважды дифференцируемая вещественная функция на (𝑎, ∞), и 𝑀0 , 𝑀1 , 𝑀2 — верхние
грани соответственно функций |𝑓 (𝑥)|, |𝑓 ′ (𝑥)|, |𝑓 ″ (𝑥)| на (𝑎, ∞). Доказать, что
𝑀12 ≤ 4𝑀0 𝑀2 .
Указание: Если ℎ > 0, то из теоремы Тейлора следует, что
𝑓 ′ (𝑥) =
1
[𝑓 (𝑥 + 2ℎ) − 𝑓 (𝑥)] − ℎ𝑓 ″ (𝜉)
2ℎ
при некотором 𝜉 ∈ (𝑥, 𝑥 + 2ℎ). Значит,
|𝑓 ′ (𝑥)| ≤ ℎ𝑀2 +
𝑀0
.
ℎ
Чтобы показать, что действительно может оказаться 𝑀12 = 4𝑀0 𝑀2 , взять 𝑎 = −1, определить
𝑓 (𝑥) =
2𝑥2 − 1
2
{ 𝑥𝑥2 −1
+1
(−1 < 𝑥 < 0),
(0 ≤ 𝑥 < ∞)
и показать, что 𝑀0 = 1, 𝑀1 = 4, 𝑀2 = 4.
Сохраняется ли неравенство 𝑀12 ≤ 4𝑀0 𝑀2 также для векторнозначных функций?
16. Пусть 𝑓 дважды дифференцируема на (0, ∞), 𝑓 ″ ограничена на (0, ∞) и 𝑓 (𝑥) → 0 при 𝑥 → ∞. Доказать,
что 𝑓 ′ (𝑥) → 0 при 𝑥 → ∞.
Указание: Устремить 𝑎 к ∞ в упражнении 15.
17. Пусть 𝑓 — вещественная трижды дифференцируемая функция на [−1, 1], такая, что
𝑓 (−1) = 0,
𝑓 (0) = 0,
𝑓 (1) = 1,
𝑓 ′ (0) = 0.
Доказать, что 𝑓 (3) (𝑥) ≥ 3 при некотором 𝑥 ∈ (−1, 1).
Заметим, что равенство имеет место для 21 (𝑥3 + 𝑥2 ).
Указание: Использовать теорему 5.15 с 𝛼 = 0 и 𝛽 = ±1, чтобы показать, что существуют 𝑠 ∈ (0, 1) и
𝑡 ∈ (−1, 0), такие, что
𝑓 (3) (𝑠) + 𝑓 (3) (𝑡) = 6.
18. Пусть 𝑓 — вещественная функция на [𝑎, 𝑏], 𝑛 — положительное целое число и 𝑓 (𝑛−1) существует при
каждом 𝑡 ∈ [𝑎, 𝑏]. Пусть 𝛼, 𝛽 и 𝑃 имеют тот же смысл, что и в теореме Тейлора (5.15). Определить
𝑄(𝑡) =
𝑓 (𝑡) − 𝑓 (𝛽)
𝑡−𝛽
при 𝑡 ∈ [𝑎, 𝑏], 𝑡 ≠ 𝛽, продифференцировать
𝑓 (𝑡) − 𝑓 (𝛽) = (𝑡 − 𝛽)𝑄(𝑡)
𝑛 − 1 раз в 𝑡 = 𝛼 и получить следующий вариант теоремы Тейлора:
𝑓 (𝛽) = 𝑃 (𝛽) +
86
𝑄(𝑛−1) (𝛼)
(𝛽 − 𝛼)𝑛 .
(𝑛 − 1)!
19. Пусть 𝑓 определена на (−1, 1) и 𝑓 ′ (0) существует. Пусть −1 < 𝛼𝑛 < 𝛽𝑛 < 1, 𝛼𝑛 → 0 и 𝛽𝑛 → 0 при 𝑛 → ∞.
Определим разностные отношения
𝑓 (𝛽𝑛 ) − 𝑓 (𝛼𝑛 )
𝐷𝑛 =
.
𝛽𝑛 − 𝛼 𝑛
Доказать следующие утверждения.
(a) Если 𝛼𝑛 < 0 < 𝛽𝑛 , то lim 𝐷𝑛 = 𝑓 ′ (0).
(b) Если 0 < 𝛼𝑛 < 𝛽𝑛 и последовательность {𝛽𝑛 /(𝛽𝑛 − 𝛼𝑛 )} ограничена, то lim 𝐷𝑛 = 𝑓 ′ (0).
(c) Если 𝑓 ′ непрерывна на (−1, 1), то lim 𝐷𝑛 = 𝑓 ′ (0).
Привести пример, в котором 𝑓 дифференцируема на (−1, 1) (но 𝑓 ′ не является непрерывной в 0) и в
котором 𝛼𝑛 , 𝛽𝑛 стремятся к 0 таким образом, что lim 𝐷𝑛 существует, но отличается от 𝑓 ′ (0).
20. Сформулировать и доказать неравенство, которое следует из теоремы Тейлора и остается верным для
векторнозначных функций.
21. Пусть 𝐸 — замкнутое подмножество прямой 𝑅1 . В упражнении 22 гл. 4 было показано, что существует
вещественная непрерывная функция 𝑓 на 𝑅1 , нуль-множество которой совпадает с 𝐸. Можно ли для
каждого замкнутого множества 𝐸 найти такую 𝑓 , которая будет дифференцируема на 𝑅1 , или которая
будет 𝑛 раз дифференцируема, или которая даже будет иметь производные всех порядков на 𝑅1 ?
22. Пусть 𝑓 — вещественная функция на (−∞, ∞). Назовем 𝑥 неподвижной точкой функции 𝑓 , если 𝑓 (𝑥) =
𝑥.
(a) Если 𝑓 дифференцируема и 𝑓 ′ (𝑡) ≠ 1 при всех вещественных 𝑡, доказать, что 𝑓 имеет не более
одной неподвижной точки.
(b) Показать, что функция, определенная как
𝑓 (𝑡) = 𝑡 + (1 + 𝑒𝑡 )−1 ,
не имеет неподвижной точки, хотя 0 < 𝑓 ′ (𝑡) < 1 при всех вещественных 𝑡.
(c) Тем не менее, если существует постоянная 𝐴 < 1, такая, что |𝑓 ′ (𝑡)| ≤ 𝐴 при всех вещественных 𝑡,
доказать, что существует неподвижная точка 𝑥 функции 𝑓 , и что 𝑥 = lim 𝑥𝑛 , где 𝑥1 — произвольное
вещественное число и
𝑥𝑛+1 = 𝑓 (𝑥𝑛 )
при 𝑛 = 1, 2, 3, … .
(d) Показать, что процесс, описанный в (c), может быть изображен наглядно в виде зигзагообразного
пути
(𝑥1 , 𝑥2 ) → (𝑥2 , 𝑥2 ) → (𝑥2 , 𝑥3 ) → (𝑥3 , 𝑥3 ) → (𝑥3 , 𝑥4 ) → ⋯ .
23. Функция 𝑓 , определенная как
𝑥3 + 1
,
3
имеет три неподвижных точки, которые мы обозначим 𝛼, 𝛽, 𝛾, причем
𝑓 (𝑥) =
−2 < 𝛼 < −1,
0 < 𝛽 < 1,
1 < 𝛾 < 2.
Произвольно выбрав 𝑥1 , определим последовательность {𝑥𝑛 }, положив 𝑥𝑛+1 = 𝑓 (𝑥𝑛 ).
(a) Если 𝑥1 < 𝛼, доказать, что 𝑥𝑛 → −∞ при 𝑛 → ∞.
(b) Если 𝛼 < 𝑥1 < 𝛾, доказать, что 𝑥𝑛 → 𝛽 при 𝑛 → ∞.
(c) Если 𝛾 < 𝑥1 , доказать, что 𝑥𝑛 → +∞ при 𝑛 → ∞.
Это означает, что 𝛽 может быть найдено данным методом, а 𝛼 и 𝛾 — нет.
24. Процесс, описанный в части (c) упражнения 22, конечно, может быть применен и к функциям, отображающим (0, ∞) в (0, ∞).
Зафиксируем 𝛼 > 1 и положим
𝑓 (𝑥) =
1
𝛼
𝑥 + ),
2(
𝑥
𝑔(𝑥) =
𝛼+𝑥
.
1+𝑥
Как 𝑓 , так и 𝑔 имеют на (0, ∞) единственную неподвижную точку √𝛼. Попытаться объяснить на основе
свойств функций 𝑓 и 𝑔, почему сходимость в упражнении 16 гл. 3 настолько быстрее, чем в упражнении
17 (сравнить 𝑓 ′ и 𝑔 ′ , нарисовать зигзаги, предложенные в упражнении 22).
Сделать то же самое при 0 < 𝛼 < 1.
25. Пусть 𝑓 дважды дифференцируема на [𝑎, 𝑏], 𝑓 (𝑎) < 0, 𝑓 (𝑏) > 0, 𝑓 ′ (𝑥) ≥ 𝛿 > 0 и 0 ≤ 𝑓 ″ (𝑥) ≤ 𝑀 при всех
𝑥 ∈ [𝑎, 𝑏]. Пусть 𝜉 — единственная точка на (𝑎, 𝑏), в которой 𝑓 (𝜉) = 0.
Завершить недостающие детали в следующем плане метода Ньютона для вычисления 𝜉.
87
(a) Выбрать 𝑥1 ∈ (𝜉, 𝑏) и определить последовательность {𝑥𝑛 } как
𝑥𝑛+1 = 𝑥𝑛 −
𝑓 (𝑥𝑛 )
.
𝑓 ′ (𝑥𝑛 )
Истолковать это геометрически в терминах касательной к графику функции 𝑓 .
(b) Доказать, что 𝑥𝑛+1 < 𝑥𝑛 и что
lim 𝑥𝑛 = 𝜉.
𝑛→∞
(c) Используя теорему Тейлора, показать, что
𝑥𝑛+1 − 𝜉 =
𝑓 ″ (𝑡𝑛 )
(𝑥 − 𝜉)2
2𝑓 ′ (𝑥𝑛 ) 𝑛
при некотором 𝑡𝑛 ∈ (𝜉, 𝑥𝑛 ).
(d) Если 𝐴 = 𝑀/2𝛿, прийти к выводу, что
0 ≤ 𝑥𝑛+1 − 𝜉 ≤
𝑛
1
[𝐴(𝑥1 − 𝜉)]2
𝐴
(ср. с упражнениями 16 и 18 гл. 3).
(e) Показать, что метод Ньютона сводится к нахождению неподвижной точки функции 𝑔, определенной как
𝑓 (𝑥)
𝑔(𝑥) = 𝑥 − ′ .
𝑓 (𝑥)
Каково поведение 𝑔 ′ (𝑥) при 𝑥, близких к 𝜉?
(f) Положить 𝑓 (𝑥) = 𝑥1/3 на (−∞, ∞) и попробовать метод Ньютона. Что происходит?
26. Пусть 𝑓 дифференцируема на [𝑎, 𝑏], 𝑓 (𝑎) = 0 и существует вещественное число 𝐴, такое, что |𝑓 ′ (𝑥)| ≤
𝐴|𝑓 (𝑥)| на [𝑎, 𝑏]. Доказать, что 𝑓 (𝑥) = 0 при всех 𝑥 ∈ [𝑎, 𝑏]. Указание: Зафиксировать 𝑥0 ∈ [𝑎, 𝑏], и пусть
𝑀0 = sup |𝑓 (𝑥)|,
𝑀1 = sup |𝑓 ′ (𝑥)|
при 𝑎 ≤ 𝑥 ≤ 𝑥0 . Для любого такого 𝑥
|𝑓 (𝑥)| ≤ 𝑀1 (𝑥0 − 𝑎) ≤ 𝐴(𝑥0 − 𝑎)𝑀0 .
Значит, 𝑀0 = 0, если 𝐴(𝑥0 − 𝑎) < 1, т. е. 𝑓 = 0 на [𝑎, 𝑥0 ]. Продолжить это рассуждение.
27. Пусть 𝜑 — вещественная функция, определенная в прямоугольнике 𝑅 на плоскости, заданном неравенствами 𝑎 ≤ 𝑥 ≤ 𝑏, 𝛼 ≤ 𝑦 ≤ 𝛽. Решением задачи с начальными условиями
𝑦′ = 𝜑(𝑥, 𝑦),
𝑦(𝑎) = 𝑐
(𝛼 ≤ 𝑐 ≤ 𝛽)
называется дифференцируемая на [𝑎, 𝑏] функция 𝑓 , такая, что 𝑓 (𝑎) = 𝑐, 𝛼 ≤ 𝑓 (𝑥) ≤ 𝛽 и
𝑓 ′ (𝑥) = 𝜑(𝑥, 𝑓 (𝑥))
(𝑎 ≤ 𝑥 ≤ 𝑏).
Доказать, что эта задача имеет не более одного решения, если существует число 𝐴, такое, что
|𝜑(𝑥, 𝑦2 ) − 𝜑(𝑥, 𝑦1 )| ≤ 𝐴|𝑦2 − 𝑦1 |
при любых (𝑥, 𝑦1 ) ∈ 𝑅, (𝑥, 𝑦2 ) ∈ 𝑅.
Указание: Применить упражнение 26 к разности двух решений. Заметим, что эта теорема единственности неприменима к задаче с начальными условиями
𝑦′ = 𝑦1/2 ,
𝑦(0) = 0,
которая имеет два решения: 𝑓 (𝑥) = 0 и 𝑓 (𝑥) = 𝑥2 /4. Найти все прочие решения.
28. Сформулировать и доказать аналогичную теорему единственности для системы дифференциальных
уравнений вида
𝑦′𝑗 = 𝜑𝑖 (𝑥, 𝑦1 , … , 𝑦𝑘 ),
𝑦𝑗 (𝑎) = 𝑐𝑗
(𝑗 = 1, … , 𝑘).
Заметим, что эту систему можно записать в виде
y′ = 𝝋(𝑥, y),
y(𝑎) = c,
где y = (𝑦1 , … , 𝑦𝑘 ) пробегает 𝑘-мерную клетку, 𝝋 — отображение некоторой (𝑘 + 1)-мерной клетки в
евклидово пространство 𝑅𝑘 , компонентами которого служат функции 𝜑1 , … , 𝜑𝑘 , а c — вектор (𝑐1 , … , 𝑐𝑘 ).
Воспользоваться упражнением 26 для векторнозначных функций.
88
29. Рассмотреть частный случай упражнения 28, перейдя к системе
𝑦′𝑗 = 𝑦𝑗+1
(𝑗 = 1, … , 𝑘 − 1),
𝑘
𝑦′𝑘 = 𝑓 (𝑥) −
∑
𝑔𝑗 (𝑥)𝑦𝑗 ,
𝑗=1
где 𝑓 , 𝑔1 , … , 𝑔𝑘 — непрерывные вещественные функции на [𝑎, 𝑏], и получить теорему единственности для
решений уравнения
𝑦(𝑘) + 𝑔𝑘 (𝑥)𝑦(𝑘−1) + ⋯ + 𝑔2 (𝑥)𝑦′ + 𝑔1 (𝑥)𝑦 = 𝑓 (𝑥),
удовлетворяющих начальным условиям
𝑦(𝑎) = 𝑐1 ,
𝑦′ (𝑎) = 𝑐2 ,
89
…,
𝑦(𝑘−1) (𝑎) = 𝑐𝑘 .
Глава 6
Интеграл Римана-Стильтьеса
Основным в этой главе является определение интеграла Римана, явным образом использующее
упорядоченность числовой прямой. В соответствии с этим мы начинаем с изучения интегрирования
вещественнозначных функций на сегментах. В последующих разделах будет изложено обобщение
на случай комплекснозначных и векторнозначных функций, заданных на сегменте. Интегрирование
по множествам, отличным от сегментов, рассматривается в гл. 10 и 11.
Определение и существование интеграла
6.1 Определение. Пусть [𝑎, 𝑏] — заданный сегмент. Разбиением 𝑃 сегмента [𝑎, 𝑏] мы называем
конечное множество точек 𝑥0 , 𝑥1 , … , 𝑥𝑛 , где
𝑎 = 𝑥0 ≤ 𝑥1 ≤ ⋯ ≤ 𝑥𝑛−1 ≤ 𝑥𝑛 = 𝑏.
Мы будем писать
Δ𝑥𝑖 = 𝑥𝑖 − 𝑥𝑖−1
(𝑖 = 1, … , 𝑛).
Пусть теперь 𝑓 — ограниченная вещественная функция, определенная на [𝑎, 𝑏]. Каждому разбиению 𝑃 сегмента [𝑎, 𝑏] соответствуют числа
𝑀𝑖 = sup 𝑓 (𝑥)
(𝑥𝑖−1 ≤ 𝑥 ≤ 𝑥𝑖 ),
𝑚𝑖 = inf 𝑓 (𝑥)
(𝑥𝑖−1 ≤ 𝑥 ≤ 𝑥𝑖 ),
𝑛
𝑈 (𝑃 , 𝑓 ) =
∑
𝑀𝑖 Δ𝑥𝑖 ,
𝑖=1
𝑛
𝐿(𝑃 , 𝑓 ) =
∑
𝑚𝑖 Δ𝑥𝑖 ,
𝑖=1
и, наконец,
𝑏
(1)
⨛
𝑎
𝑓 𝑑𝑥 = inf 𝑈 (𝑃 , 𝑓 ),
𝑏
(2)
⨜𝑎
𝑓 𝑑𝑥 = sup 𝐿(𝑃 , 𝑓 ),
где верхняя и нижняя грани берутся по всем разбиениям 𝑃 сегмента [𝑎, 𝑏]. Левые части равенств (1)
и (2) называются соответственно верхним и нижним интегралом Римана функции 𝑓 по сегменту
[𝑎, 𝑏].
Если верхний интеграл равен нижнему, то мы будем говорить, что 𝑓 интегрируема по Риману
на [𝑎, 𝑏], и писать 𝑓 ∈ ℛ (иными словами, ℛ обозначает множество всех функций, интегрируемых
по Риману), а общее значение величин (1) и (2) будем обозначать
𝑏
(3)
∫
𝑎
𝑓 𝑑𝑥,
90
или
𝑏
(4)
∫
𝑎
𝑓 (𝑥) 𝑑𝑥.
Это интеграл Римана от функции 𝑓 по сегменту [𝑎, 𝑏]. Поскольку 𝑓 ограничена, существуют
два числа 𝑚 и 𝑀, такие, что
𝑚 ≤ 𝑓 (𝑥) ≤ 𝑀
(𝑎 ≤ 𝑥 ≤ 𝑏).
Значит, при любом 𝑃
𝑚(𝑏 − 𝑎) ≤ 𝐿(𝑃 , 𝑓 ) ≤ 𝑈 (𝑃 , 𝑓 ) ≤ 𝑀(𝑏 − 𝑎),
так что числа 𝐿(𝑃 , 𝑓 ) и 𝑈 (𝑃 , 𝑓 ) образуют ограниченное множество. Это показывает, что верхний
и нижний интегралы определены для любой ограниченной функции 𝑓 . Вопрос об их совпадении
и, значит, вопрос об интегрируемости функции 𝑓 , оказывается более тонким. Вместо того чтобы
исследовать его отдельно для интеграла Римана, мы сразу же рассмотрим более общую ситуацию.
6.2 Определение. Пусть 𝛼 — монотонно возрастающая функция на [𝑎, 𝑏] (поскольку 𝛼(𝑎) и 𝛼(𝑏)
конечны, 𝛼 ограничена на [𝑎, 𝑏]). Если 𝑃 — какое-нибудь разбиение сегмента [𝑎, 𝑏], то положим
Δ𝛼𝑖 = 𝛼(𝑥𝑖 ) − 𝛼(𝑥𝑖−1 ).
Ясно, что Δ𝛼𝑖 ≥ 0. Для любой вещественной функции 𝑓 , ограниченной на [𝑎, 𝑏], положим
𝑛
𝑈 (𝑃 , 𝑓 , 𝛼) =
∑
𝑀𝑖 Δ𝛼𝑖 ,
𝑖=1
𝑛
𝐿(𝑃 , 𝑓 , 𝛼) =
∑
𝑚𝑖 Δ𝛼𝑖 ,
𝑖=1
где 𝑀𝑖 и 𝑚𝑖 имеют тот же смысл, что и в определении 6.1. Положим, по определению,
(5)
(6)
⨛
⨜
𝑓 𝑑𝛼 = inf 𝑈 (𝑃 , 𝑓 , 𝛼),
𝑓 𝑑𝛼 = sup 𝐿(𝑃 , 𝑓 , 𝛼),
где верхняя и нижняя грани берутся снова по всем разбиениям.
Если левые части равенств (5) и (6) равны между собой, то их общее значение обозначается
через
𝑏
(7)
∫
𝑎
𝑓 𝑑𝛼
или иногда через
𝑏
(8)
∫
𝑎
𝑓 (𝑥) 𝑑𝛼(𝑥).
Это интеграл Римана-Стильтьеса (или просто интеграл Стильтьеса) от функции 𝑓 относительно 𝛼 по [𝑎, 𝑏].
Если (7) существует, т. е. если (5) и (6) равны между собой, то мы будем говорить, что 𝑓
интегрируема относительно 𝛼 в смысле Римана, и писать 𝑓 ∈ ℛ(𝛼).
Полагая 𝛼(𝑥) = 𝑥, мы приходим к выводу, что интеграл Римана — это частный случай интеграла
Римана-Стильтьеса. Подчеркнем, однако, что в общем случае 𝛼 не обязана быть даже непрерывной.
Несколько слов по поводу обозначений. Мы предпочитаем обозначение (7) обозначению (8),
так как фигурирующая в (8) буква 𝑥 ничего не добавляет к содержанию записи (7). Совершенно несущественно, какую букву мы употребляем для обозначения так называемой «переменной
интегрирования». Так, например, (8) — это то же самое, что
𝑏
∫
𝑎
𝑓 (𝑦) 𝑑𝛼(𝑦).
91
Интеграл зависит от 𝑓 , 𝛼, 𝑎 и 𝑏, но не от переменной интегрирования, которую вполне можно
опустить.
Роль переменной интегрирования совершенно аналогична роли индекса суммирования: два символа
𝑛
∑
𝑛
𝑐𝑖 ,
∑
𝑖=1
𝑐𝑘
𝑘=1
означают одно и то же, так как каждый из них означает 𝑐1 + 𝑐2 + ⋯ + 𝑐𝑛 .
Разумеется, не произойдет ничего страшного, если переменная интегрирования будет написана,
а в некоторых случаях даже удобно ее писать.
Теперь мы исследуем вопрос о существовании интеграла (7). Не повторяя этого каждый раз,
мы будем считать 𝑓 вещественной и ограниченной, а 𝛼 — монотонно возрастающей на [𝑎, 𝑏], и если
𝑏
исключена возможность недоразумений, мы будем писать ∫ вместо ∫𝑎 .
6.3 Определение. Мы будем говорить, что разбиение 𝑃 ∗ является измельчением разбиения 𝑃 ,
если 𝑃 ∗ ⊃ 𝑃 (т. е. если каждая точка разбиения 𝑃 служит также точкой разбиения 𝑃 ∗ ). В случае,
когда заданы два разбиения 𝑃1 и 𝑃2 , мы будем говорить, что 𝑃 ∗ есть их общее измельчение, если
𝑃 ∗ = 𝑃 1 ∪ 𝑃2 .
6.4 Теорема. Если 𝑃 ∗ — измельчение разбиения 𝑃 , то
𝐿(𝑃 , 𝑓 , 𝛼) ≤ 𝐿(𝑃 ∗ , 𝑓 , 𝛼)
(9)
и
𝑈 (𝑃 ∗ , 𝑓 , 𝛼) ≤ 𝑈 (𝑃 , 𝑓 , 𝛼).
(10)
Доказательство. Чтобы доказать (9), допустим сначала, что 𝑃 ∗ содержит ровно на одну точку
больше, чем 𝑃 . Обозначим эту новую точку через 𝑥∗ и допустим, что 𝑥𝑖−1 < 𝑥∗ < 𝑥𝑖 , где 𝑥𝑖−1 и 𝑥𝑖 —
две последовательные точки разбиения 𝑃 . Положим
𝑤1 = inf 𝑓 (𝑥)
(𝑥𝑖−1 ≤ 𝑥 ≤ 𝑥∗ ),
𝑤2 = inf 𝑓 (𝑥)
(𝑥∗ ≤ 𝑥 ≤ 𝑥𝑖 ).
Ясно, что 𝑤1 ≥ 𝑚𝑖 и 𝑤2 ≥ 𝑚𝑖 , где, как и прежде,
𝑚𝑖 = inf 𝑓 (𝑥)
(𝑥𝑖−1 ≤ 𝑥 ≤ 𝑥𝑖 ).
Значит,
𝐿(𝑃 ∗ , 𝑓 , 𝛼) − 𝐿(𝑃 , 𝑓 , 𝛼) = 𝑤1 [𝛼(𝑥∗ ) − 𝛼(𝑥𝑖−1 )] + 𝑤2 [𝛼(𝑥𝑖 ) − 𝛼(𝑥∗ )] − 𝑚𝑖 [𝛼(𝑥𝑖 ) − 𝛼(𝑥𝑖−1 )]
= (𝑤1 − 𝑚𝑖 )[𝛼(𝑥∗ ) − 𝛼(𝑥𝑖−1 )] + (𝑤2 − 𝑚𝑖 )[𝛼(𝑥𝑖 ) − 𝛼(𝑥∗ )] ≥ 0.
Если 𝑃 ∗ содержит на 𝑘 точек больше, чем 𝑃 , то мы повторим только что проведенное рассуждение 𝑘 раз и получим (9). Доказательство неравенства (10) аналогично.
𝑏
𝑏
6.5 Теорема.
⨜𝑎
𝑓 𝑑𝛼 ≤
⨛
𝑎
𝑓 𝑑𝛼.
Доказательство. Пусть 𝑃 ∗ — общее измельчение двух разбиений 𝑃1 и 𝑃2 . По теореме 6.4
𝐿(𝑃1 , 𝑓 , 𝛼) ≤ 𝐿(𝑃 ∗ , 𝑓 , 𝛼) ≤ 𝑈 (𝑃 ∗ , 𝑓 , 𝛼) ≤ 𝑈 (𝑃2 , 𝑓 , 𝛼).
Значит,
(11)
𝐿(𝑃1 , 𝑓 , 𝛼) ≤ 𝑈 (𝑃2 , 𝑓 , 𝛼).
Считая 𝑃2 фиксированным и вычисляя верхнюю грань по всем 𝑃1 , получаем из (11)
(12)
⨜
𝑓 𝑑𝛼 ≤ 𝑈 (𝑃2 , 𝑓 , 𝛼).
Вычисляя нижнюю грань по всем 𝑃2 в (12), получаем утверждение теоремы.
92
6.6 Теорема. 𝑓 ∈ ℛ(𝛼) на [𝑎, 𝑏] тогда и только тогда, когда для любого 𝜀 > 0 существует
разбиение 𝑃 , такое, что
(13)
𝑈 (𝑃 , 𝑓 , 𝛼) − 𝐿(𝑃 , 𝑓 , 𝛼) < 𝜀.
Доказательство. При любом 𝑃 имеем
𝐿(𝑃 , 𝑓 , 𝛼) ≤
𝑓 𝑑𝛼 ≤
⨜
⨛
𝑓 𝑑𝛼 ≤ 𝑈 (𝑃 , 𝑓 , 𝛼).
Поэтому из (13) следует, что
0≤
⨛
𝑓 𝑑𝛼 −
⨜
𝑓 𝑑𝛼 < 𝜀.
Значит, если (13) может быть выполнено при любом 𝜀 > 0, то
⨛
𝑓 𝑑𝛼 =
⨜
𝑓 𝑑𝛼,
т. е. 𝑓 ∈ ℛ(𝛼).
Допустим теперь, что 𝑓 ∈ ℛ(𝛼) и задано 𝜀 > 0. Тогда существуют разбиения 𝑃1 и 𝑃2 , такие, что
𝜀
,
2
𝜀
𝑓 𝑑𝛼 − 𝐿(𝑃1 , 𝑓 , 𝛼) < .
∫
2
(14)
𝑈 (𝑃2 , 𝑓 , 𝛼) −
(15)
∫
𝑓 𝑑𝛼 <
Выберем в качестве 𝑃 общее измельчение разбиений 𝑃1 и 𝑃2 . Тогда, как показывает теорема 6.4
вместе с (14) и (15),
𝑈 (𝑃 , 𝑓 , 𝛼) ≤ 𝑈 (𝑃2 , 𝑓 , 𝛼) <
∫
𝑓 𝑑𝛼 +
𝜀
< 𝐿(𝑃1 , 𝑓 , 𝛼) + 𝜀 ≤ 𝐿(𝑃 , 𝑓 , 𝛼) + 𝜀,
2
так что для такого разбиения 𝑃 выполняется (13).
Теорема 6.6 предоставляет удобный критерий интегрируемости. Но прежде чем мы применим
его, мы сформулируем несколько связанных фактов.
6.7 Теорема.
(a) Если (13) выполняется для некоторого 𝑃 и некоторого 𝜀, то (13) выполняется (с тем же
𝜀) для каждого измельчения разбиения 𝑃 .
(b) Если (13) выполняется для 𝑃 = {𝑥0 , … , 𝑥𝑛 } и если 𝑠𝑖 , 𝑡𝑖 — произвольные точки на [𝑥𝑖−1 , 𝑥𝑖 ],
то
𝑛
∑
|𝑓 (𝑠𝑖 ) − 𝑓 (𝑡𝑖 )|Δ𝛼𝑖 < 𝜀.
𝑖=1
(c) Если 𝑓 ∈ ℛ(𝛼) и выполняются предположения условия (b), то
𝑛
|∑
𝑖=1
𝑏
𝑓 (𝑡𝑖 )Δ𝛼𝑖 −
∫
𝑎
𝑓 𝑑𝛼 < 𝜀.
|
Доказательство. Утверждение (a) вытекает из теоремы 6.4. При предположениях, сделанных в
(b), как 𝑓 (𝑠𝑖 ), так и 𝑓 (𝑡𝑖 ) лежат на [𝑚𝑖 , 𝑀𝑖 ], так что |𝑓 (𝑠𝑖 ) − 𝑓 (𝑡𝑖 )| ≤ 𝑀𝑖 − 𝑚𝑖 . Следовательно,
𝑛
∑
|𝑓 (𝑠𝑖 ) − 𝑓 (𝑡𝑖 )|Δ𝛼𝑖 ≤ 𝑈 (𝑃 , 𝑓 , 𝛼) − 𝐿(𝑃 , 𝑓 , 𝛼),
𝑖=1
так что (b) доказано. Утверждение (c) следует из очевидных неравенств
𝐿(𝑃 , 𝑓 , 𝛼) ≤
∑
𝑓 (𝑡𝑖 )Δ𝛼𝑖 ≤ 𝑈 (𝑃 , 𝑓 , 𝛼)
и
𝐿(𝑃 , 𝑓 , 𝛼) ≤ ∫ 𝑓 𝑑𝛼 ≤ 𝑈 (𝑃 , 𝑓 , 𝛼).
93
6.8 Теорема. Если 𝑓 непрерывна на [𝑎, 𝑏], то 𝑓 ∈ ℛ(𝛼) на [𝑎, 𝑏].
Доказательство. Пусть задано 𝜀 > 0. Выберем 𝜂 > 0 так, чтобы
[𝛼(𝑏) − 𝛼(𝑎)]𝜂 < 𝜀.
Поскольку 𝑓 равномерно непрерывна на [𝑎, 𝑏] (теорема 4.19), существует такое 𝛿 > 0, что
(16)
|𝑓 (𝑥) − 𝑓 (𝑡)| < 𝜂,
если 𝑥 ∈ [𝑎, 𝑏], 𝑡 ∈ [𝑎, 𝑏] и |𝑥 − 𝑡| < 𝛿.
Если 𝑃 — любое разбиение [𝑎, 𝑏], такое, что Δ𝑥𝑖 < 𝛿 при всех 𝑖, то из (16) следует, что
(17)
𝑀𝑖 − 𝑚 𝑖 ≤ 𝜂
(𝑖 = 1, … , 𝑛).
Значит,
𝑛
𝑈 (𝑃 , 𝑓 , 𝛼) − 𝐿(𝑃 , 𝑓 , 𝛼) =
∑
(𝑀𝑖 − 𝑚𝑖 )Δ𝛼𝑖
𝑖=1
𝑛
≤𝜂
∑
Δ𝛼𝑖 = 𝜂[𝛼(𝑏) − 𝛼(𝑎)] < 𝜀.
𝑖=1
По теореме 6.6 𝑓 ∈ ℛ(𝛼).
6.9 Теорема. Если 𝑓 монотонна на [𝑎, 𝑏], а 𝛼 непрерывна на [𝑎, 𝑏], то 𝑓 ∈ ℛ(𝛼) (мы по-прежнему
предполагаем, разумеется, что 𝛼 монотонна).
Доказательство. Пусть задано 𝜀 > 0. Для любого положительного 𝑛 выберем разбиение так, что
Δ𝛼𝑖 =
𝛼(𝑏) − 𝛼(𝑎)
𝑛
(𝑖 = 1, … , 𝑛).
Это возможно, так как 𝛼 непрерывна (теорема 4.23).
Предположим, что 𝑓 монотонно возрастает (в другом случае доказательство аналогично). Тогда
𝑀𝑖 = 𝑓 (𝑥𝑖 ),
𝑚𝑖 = 𝑓 (𝑥𝑖−1 )
(𝑖 = 1, … , 𝑛),
так что
𝑛
𝑈 (𝑃 , 𝑓 , 𝛼) − 𝐿(𝑃 , 𝑓 , 𝛼) =
=
𝛼(𝑏) − 𝛼(𝑎)
[𝑓 (𝑥𝑖 ) − 𝑓 (𝑥𝑖−1 )]
∑
𝑛
𝑖=1
𝛼(𝑏) − 𝛼(𝑎)
⋅ [𝑓 (𝑏) − 𝑓 (𝑎)] < 𝜀,
𝑛
если 𝑛 достаточно велико. По теореме 6.6 𝑓 ∈ ℛ(𝛼).
6.10 Теорема. Пусть 𝑓 ограничена на [𝑎, 𝑏], имеет только конечное число точек разрыва на
этом сегменте, а 𝛼 непрерывна в каждой точке, в которой 𝑓 имеет разрыв. Тогда 𝑓 ∈ ℛ(𝛼).
Доказательство. Пусть задано 𝜀 > 0. Положим 𝑀 = sup |𝑓 (𝑥)|, и пусть 𝐸 — множество точек, в
которых 𝑓 имеет разрыв. Ввиду того что 𝐸 конечно, а 𝛼 непрерывна в каждой точке множества
𝐸, мы можем покрыть 𝐸 конечным числом непересекающихся сегментов [𝑢𝑗 , 𝑣𝑗 ] ⊂ [𝑎, 𝑏] так, что
сумма соответствующих разностей 𝛼(𝑣𝑗 ) − 𝛼(𝑢𝑗 ) меньше 𝜀. Кроме того, мы можем расположить эти
сегменты таким образом, что каждая точка множества 𝐸 ∩ (𝑎, 𝑏) лежит во внутренности какогонибудь [𝑢𝑗 , 𝑣𝑗 ].
Удалим интервалы (𝑢𝑗 , 𝑣𝑗 ) из [𝑎, 𝑏]. Оставшееся множество 𝐾 компактно. Значит, 𝑓 равномерно
непрерывна на 𝐾, и существует 𝛿 > 0, такое, что |𝑓 (𝑠) − 𝑓 (𝑡)| < 𝜀, если 𝑠 ∈ 𝐾, 𝑡 ∈ 𝐾, |𝑠 − 𝑡| < 𝛿.
Теперь образуем разбиение 𝑃 = {𝑥0 , 𝑥1 , … , 𝑥𝑛 } сегмента [𝑎, 𝑏] следующим образом. Каждое 𝑢𝑗
входит в 𝑃 . Каждое 𝑣𝑗 входит в 𝑃 . Ни одна точка ни одного из интервалов (𝑢𝑗 , 𝑣𝑗 ) не входит в 𝑃 .
Если 𝑥𝑖−1 не совпадает ни с одним из 𝑢𝑗 , то Δ𝑥𝑖 < 𝛿.
94
Заметим, что 𝑀𝑖 − 𝑚𝑖 ≤ 2𝑀 при всех 𝑖, и что 𝑀𝑖 − 𝑚𝑖 ≤ 𝜀, кроме случая, когда 𝑥𝑖−1 совпадает с
одним из 𝑢𝑗 . Значит, как и в доказательстве теоремы 6.8,
𝑈 (𝑃 , 𝑓 , 𝛼) − 𝐿(𝑃 , 𝑓 , 𝛼) ≤ [𝛼(𝑏) − 𝛼(𝑎)]𝜀 + 2𝑀𝜀.
Так как 𝜀 произвольно, теорема 6.6 показывает, что 𝑓 ∈ ℛ(𝛼).
Замечание. Если 𝑓 и 𝛼 имеют общую точку разрыва, то 𝑓 может не принадлежать ℛ(𝛼). Это
показано в упражнении 3.
6.11 Теорема. Пусть 𝑓 ∈ ℛ(𝛼) на [𝑎, 𝑏], 𝑚 ≤ 𝑓 ≤ 𝑀, 𝜑 непрерывна на [𝑚, 𝑀] и ℎ(𝑥) = 𝜑(𝑓 (𝑥)) на
[𝑎, 𝑏]. Тогда ℎ ∈ ℛ(𝛼) на [𝑎, 𝑏].
Доказательство. Выберем 𝜀 > 0. Ввиду того что 𝜑 равномерно непрерывна на [𝑚, 𝑀], существует
𝛿 > 0, такое, что 𝛿 < 𝜀 и |𝜑(𝑠) − 𝜑(𝑡)| < 𝜀, если |𝑠 − 𝑡| ≤ 𝛿 и 𝑠, 𝑡 ∈ [𝑚, 𝑀].
Поскольку 𝑓 ∈ ℛ(𝛼), существует разбиение 𝑃 = {𝑥0 , 𝑥1 , … , 𝑥𝑛 } сегмента [𝑎, 𝑏], такое, что
𝑈 (𝑃 , 𝑓 , 𝛼) − 𝐿(𝑃 , 𝑓 , 𝛼) < 𝛿 2 .
(18)
Пусть 𝑀𝑖 и 𝑚𝑖 имеют тот же смысл, что и в определении 6.1, и пусть 𝑀𝑖∗ , 𝑚∗𝑖 — аналогичные
величины для функции ℎ. Разобьем числа 1, … , 𝑛 на два класса: 𝑖 ∈ 𝐴, если 𝑀𝑖 − 𝑚𝑖 < 𝛿, и 𝑖 ∈ 𝐵,
если 𝑀𝑖 − 𝑚𝑖 ≥ 𝛿.
Для 𝑖 ∈ 𝐴, в силу выбора 𝛿, оказывается, что 𝑀𝑖∗ − 𝑚∗𝑖 ≤ 𝜀.
Для 𝑖 ∈ 𝐵 имеем 𝑀𝑖∗ − 𝑚∗𝑖 ≤ 2𝐾, где 𝐾 = sup |𝜑(𝑡)|, 𝑚 ≤ 𝑡 ≤ 𝑀. Согласно (18), имеем
(19)
𝛿
∑
Δ𝛼𝑖 ≤
∑
𝑖∈𝐵
(𝑀𝑖 − 𝑚𝑖 )Δ𝛼𝑖 < 𝛿 2 ,
𝑖∈𝐵
так что ∑𝑖∈𝐵 Δ𝛼𝑖 < 𝛿. Следовательно,
𝑈 (𝑃 , ℎ, 𝛼) − 𝐿(𝑃 , ℎ, 𝛼) =
∑
(𝑀𝑖∗ − 𝑚∗𝑖 )Δ𝛼𝑖 +
∑
(𝑀𝑖∗ − 𝑚∗𝑖 )Δ𝛼𝑖
𝑖∈𝐵
𝑖∈𝐴
≤ 𝜀[𝛼(𝑏) − 𝛼(𝑎)] + 2𝐾𝛿 < 𝜀[𝛼(𝑏) − 𝛼(𝑎) + 2𝐾].
Так как 𝜀 произвольно, из теоремы 6.6 следует, что ℎ ∈ ℛ(𝛼).
Замечание. Эта теорема подсказывает такой вопрос: какие же функции интегрируемы по Риману? Ответ дается в теореме 11.33(b).
Свойства интеграла
6.12 Теорема.
(a) Если 𝑓1 ∈ ℛ(𝛼) и 𝑓2 ∈ ℛ(𝛼) на [𝑎, 𝑏], то
𝑓1 + 𝑓2 ∈ ℛ(𝛼),
𝑐𝑓 ∈ ℛ(𝛼), какова бы ни была постоянная 𝑐, и
𝑏
∫
𝑎
𝑏
(𝑓1 + 𝑓2 ) 𝑑𝛼 =
𝑏
𝑓1 𝑑𝛼 +
∫
𝑎
𝑏
∫
𝑎
(b) Если 𝑓1 (𝑥) ≤ 𝑓2 (𝑥) на [𝑎, 𝑏], то
𝑏
𝑐𝑓 𝑑𝛼 = 𝑐
𝑏
∫
𝑎
∫
𝑎
∫
𝑎
𝑓 𝑑𝛼.
𝑏
𝑓1 𝑑𝛼 ≤
95
∫
𝑎
𝑓2 𝑑𝛼.
𝑓2 𝑑𝛼,
(c) Если 𝑓 ∈ ℛ(𝛼) на [𝑎, 𝑏] и если 𝑎 < 𝑐 < 𝑏, то 𝑓 ∈ ℛ(𝛼) на [𝑎, 𝑐] и на [𝑐, 𝑏] и
𝑐
∫
𝑎
𝑏
𝑓 𝑑𝛼 +
𝑏
𝑓 𝑑𝛼 =
∫
𝑐
∫
𝑎
𝑓 𝑑𝛼.
(d) Если 𝑓 ∈ ℛ(𝛼) на [𝑎, 𝑏] и если |𝑓 (𝑥)| ≤ 𝑀 на [𝑎, 𝑏], то
𝑏
|∫
𝑎
𝑓 𝑑𝛼 ≤ 𝑀[𝛼(𝑏) − 𝛼(𝑎)].
|
(e) Если 𝑓 ∈ ℛ(𝛼1 ) и 𝑓 ∈ ℛ(𝛼2 ), то 𝑓 ∈ ℛ(𝛼1 + 𝛼2 ) и
𝑏
𝑏
𝑓 𝑑(𝛼1 + 𝛼2 ) =
∫
𝑎
∫
𝑎
𝑏
𝑓 𝑑𝛼1 +
∫
𝑎
𝑓 𝑑𝛼2 ;
если 𝑓 ∈ ℛ(𝛼) и 𝑐 — положительное число, то 𝑓 ∈ ℛ(𝑐𝛼) и
𝑏
∫
𝑎
𝑏
𝑓 𝑑(𝑐𝛼) = 𝑐
∫
𝑎
𝑓 𝑑𝛼.
Доказательство. Если 𝑓 = 𝑓1 + 𝑓2 , а 𝑃 — какое-нибудь разбиение сегмента [𝑎, 𝑏], то
(20)
𝐿(𝑃 , 𝑓1 , 𝛼) + 𝐿(𝑃 , 𝑓2 , 𝛼) ≤ 𝐿(𝑃 , 𝑓 , 𝛼) ≤ 𝑈 (𝑃 , 𝑓 , 𝛼) ≤ 𝑈 (𝑃 , 𝑓1 , 𝛼) + 𝑈 (𝑃 , 𝑓2 , 𝛼).
Если 𝑓1 ∈ ℛ(𝛼) и 𝑓2 ∈ ℛ(𝛼), то любому 𝜀 > 0 отвечают такие разбиения 𝑃𝑗 (𝑗 = 1, 2), что
𝑈 (𝑃𝑗 , 𝑓𝑗 , 𝛼) − 𝐿(𝑃𝑗 , 𝑓𝑗 , 𝛼) < 𝜀.
Эти неравенства сохранятся, если 𝑃1 и 𝑃2 заменить их общим измельчением 𝑃 . Тогда из (20) следует,
что
𝑈 (𝑃 , 𝑓 , 𝛼) − 𝐿(𝑃 , 𝑓 , 𝛼) < 2𝜀,
а это значит, что 𝑓 ∈ ℛ(𝛼).
Для этого же разбиения 𝑃 имеем
𝑈 (𝑃 , 𝑓𝑗 , 𝛼) < ∫ 𝑓𝑗 𝑑𝛼 + 𝜀
(𝑗 = 1, 2);
таким образом, из (20) следует, что
∫ 𝑓 𝑑𝛼 ≤ 𝑈 (𝑃 , 𝑓 , 𝛼) < ∫ 𝑓1 𝑑𝛼 + ∫ 𝑓2 𝑑𝛼 + 2𝜀.
Так как 𝜀 было произвольным, мы заключаем, что
(21)
∫ 𝑓 𝑑𝛼 ≤ ∫ 𝑓1 𝑑𝛼 + ∫ 𝑓2 𝑑𝛼.
Если заменить 𝑓1 и 𝑓2 в (21) на −𝑓1 и −𝑓2 , то неравенство изменится на обратное, тем самым
доказано требуемое равенство.
Доказательства остальных частей теоремы 6.12 совершенно аналогичны, и мы опускаем детали.
В части (c) все дело в том, что, переходя к измельчениям при приближении интеграла ∫ 𝑓 𝑑𝛼, мы
можем ограничиться лишь разбиениями, содержащими точку 𝑐.
6.13 Теорема. Если 𝑓 ∈ ℛ(𝛼) и 𝑔 ∈ ℛ(𝛼) на [𝑎, 𝑏], то
(a) 𝑓 𝑔 ∈ ℛ(𝛼);
𝑏
(b) |𝑓 | ∈ ℛ(𝛼) и
𝑏
|𝑓 | 𝑑𝛼.
𝑓 𝑑𝛼 ≤
| ∫
|∫
𝑎
𝑎
Доказательство. Если взять 𝜑(𝑡) = 𝑡2 , то теорема 6.11 показывает, что 𝑓 2 ∈ ℛ(𝛼), если 𝑓 ∈ ℛ(𝛼).
Тождество
4𝑓 𝑔 = (𝑓 + 𝑔)2 − (𝑓 − 𝑔)2
завершает доказательство утверждения (a).
96
Если взять 𝜑(𝑡) = |𝑡|, теорема 6.11 точно так же показывает, что |𝑓 | ∈ ℛ(𝛼). Выберем 𝑐 = ±1
так, что
𝑐 ∫ 𝑓 𝑑𝛼 ≥ 0.
Тогда
| ∫ 𝑓 𝑑𝛼| = 𝑐 ∫ 𝑓 𝑑𝛼 = ∫ 𝑐𝑓 𝑑𝛼 ≤ ∫ |𝑓 | 𝑑𝛼,
так как 𝑐𝑓 ≤ |𝑓 |.
6.14 Определение. Единичная ступенчатая функция определяется как
𝐼(𝑥) =
0
{1
(𝑥 ≤ 0),
(𝑥 > 0).
6.15 Теорема. Если 𝑎 < 𝑠 < 𝑏, 𝑓 ограничена на [𝑎, 𝑏], 𝑓 непрерывна в 𝑠 и 𝛼(𝑥) = 𝐼(𝑥 − 𝑠), то
𝑏
𝑓 𝑑𝛼 = 𝑓 (𝑠).
∫
𝑎
Доказательство. Рассмотрим разбиения 𝑃 = {𝑥0 , 𝑥1 , 𝑥2 , 𝑥3 }, где 𝑥0 = 𝑎 и 𝑥1 = 𝑠 < 𝑥2 < 𝑥3 = 𝑏.
Тогда
𝑈 (𝑃 , 𝑓 , 𝛼) = 𝑀2 ,
𝐿(𝑃 , 𝑓 , 𝛼) = 𝑚2 .
Так как 𝑓 непрерывна в 𝑠, мы видим, что 𝑀2 и 𝑚2 сходятся к 𝑓 (𝑠) при 𝑥2 → 𝑠.
6.16 Теорема. Пусть 𝑐𝑛 ≥ 0 при 𝑛 = 1, 2, 3, … , ∑ 𝑐𝑛 сходится, {𝑠𝑛 } — последовательность различных точек на (𝑎, 𝑏) и
∞
(22)
𝛼(𝑥) =
∑
𝑐𝑛 𝐼(𝑥 − 𝑠𝑛 ).
𝑛=1
Если 𝑓 непрерывна на [𝑎, 𝑏], то
∞
𝑏
(23)
∫
𝑎
𝑓 𝑑𝛼 =
∑
𝑐𝑛 𝑓 (𝑠𝑛 ).
𝑛=1
Доказательство. Признак сравнения показывает, что ряд (22) сходится при каждом 𝑥. Очевидно,
что его сумма 𝛼(𝑥) монотонна и 𝛼(𝑎) = 0, 𝛼(𝑏) = ∑ 𝑐𝑛 (это тот тип функции, который рассматривался
в замечании 4.31).
Пусть дано 𝜀 > 0. Выберем 𝑁 так, что
∞
∑
𝑐𝑛 < 𝜀.
𝑁+1
Положим
𝑁
𝛼1 (𝑥) =
∑
∞
𝑐𝑛 𝐼(𝑥 − 𝑠𝑛 ),
𝛼2 (𝑥) =
𝑛=1
∑
𝑁+1
По теоремам 6.12 и 6.15
𝑁
𝑏
(24)
∫
𝑎
𝑓 𝑑𝛼1 =
∑
𝑐𝑛 𝑓 (𝑠𝑛 ).
𝑖=1
Так как 𝛼2 (𝑏) − 𝛼2 (𝑎) < 𝜀,
𝑏
(25)
|∫
𝑎
𝑓 𝑑𝛼2 ≤ 𝑀𝜀,
|
97
𝑐𝑛 𝐼(𝑥 − 𝑠𝑛 ).
где 𝑀 = sup |𝑓 (𝑥)|. Так как 𝛼 = 𝛼1 + 𝛼2 , из (24) и (25) следует, что
𝑁
| 𝑏
|
|
𝑓 𝑑𝛼 −
𝑐𝑛 𝑓 (𝑠𝑛 )| ≤ 𝑀𝜀.
∑
||
||∫
𝑎
𝑖=1
(26)
Если устремить 𝑁 к ∞, мы получим (23).
6.17 Теорема. Пусть 𝛼 монотонно возрастает и 𝛼 ′ ∈ ℛ на [𝑎, 𝑏]. Если 𝑓 — ограниченная вещественная функция на [𝑎, 𝑏], то 𝑓 ∈ ℛ(𝛼) тогда и только тогда, когда 𝑓 𝛼 ′ ∈ ℛ. В этом случае
𝑏
(27)
∫
𝑎
𝑏
𝑓 𝑑𝛼 =
∫
𝑎
𝑓 (𝑥)𝛼 ′ (𝑥) 𝑑𝑥.
Доказательство. Пусть дано 𝜀 > 0. Применим к 𝛼 ′ теорему 6.6: существует разбиение 𝑃 =
{𝑥0 , … , 𝑥𝑛 } сегмента [𝑎, 𝑏], такое, что
𝑈 (𝑃 , 𝛼 ′ ) − 𝐿(𝑃 , 𝛼 ′ ) < 𝜀.
(28)
По теореме о среднем значении существуют точки 𝑡𝑖 ∈ [𝑥𝑖−1 , 𝑥𝑖 ], такие, что
Δ𝛼𝑖 = 𝛼 ′ (𝑡𝑖 )Δ𝑥𝑖
при 𝑖 = 1, … , 𝑛. Если 𝑠𝑖 ∈ [𝑥𝑖−1 , 𝑥𝑖 ], то
𝑛
(29)
∑
|𝛼 ′ (𝑠𝑖 ) − 𝛼 ′ (𝑡𝑖 )|Δ𝑥𝑖 < 𝜀
𝑖=1
в силу (28) и теоремы 6.7(b). Положим 𝑀 = sup |𝑓 (𝑥)|. Так как
𝑛
𝑛
∑
𝑓 (𝑠𝑖 )Δ𝛼𝑖 =
𝑖=1
∑
𝑓 (𝑠𝑖 )𝛼 ′ (𝑡𝑖 )Δ𝑥𝑖 ,
𝑖=1
из (29) следует, что
𝑛
(30)
|∑
𝑖=1
𝑛
𝑓 (𝑠𝑖 )Δ𝛼𝑖 −
𝑓 (𝑠𝑖 )𝛼 ′ (𝑠𝑖 )Δ𝑥𝑖 ≤ 𝑀𝜀.
|
𝑖=1
∑
В частности,
𝑛
∑
𝑓 (𝑠𝑖 )Δ𝛼𝑖 ≤ 𝑈 (𝑃 , 𝑓 𝛼 ′ ) + 𝑀𝜀
𝑖=1
при любом выборе 𝑠𝑖 ∈ [𝑥𝑖−1 , 𝑥𝑖 ], так что
𝑈 (𝑃 , 𝑓 , 𝛼) ≤ 𝑈 (𝑃 , 𝑓 𝛼 ′ ) + 𝑀𝜀.
Аналогичное рассуждение на основе неравенства (30) приводит к
𝑈 (𝑃 , 𝑓 𝛼 ′ ) ≤ 𝑈 (𝑃 , 𝑓 , 𝛼) + 𝑀𝜀.
Следовательно,
(31)
|𝑈 (𝑃 , 𝑓 , 𝛼) − 𝑈 (𝑃 , 𝑓 𝛼 ′ )| ≤ 𝑀𝜀.
Теперь заметим, что (28) остается верным, если заменить разбиение 𝑃 любым его измельчением.
Значит, (31) также остается верным. Мы заключаем, что
𝑏
|⨛
𝑎
𝑏
𝑓 𝑑𝛼 −
⨛
𝑎
𝑓 (𝑥)𝛼 ′ (𝑥) 𝑑𝑥 ≤ 𝑀𝜀.
|
98
Но 𝜀 произвольно, значит,
𝑏
(32)
⨛
𝑎
𝑏
𝑓 𝑑𝛼 =
⨛
𝑎
𝑓 (𝑥)𝛼 ′ (𝑥) 𝑑𝑥
для любой ограниченной функции 𝑓 . Равенство нижних интегралов следует из (30) точно таким
же образом. Теорема доказана.
6.18 Замечание. Две последние теоремы иллюстрируют общность и гибкость, присущие процессу
интегрирования Стильтьеса. Если 𝛼 — ступенчатая функция (так называются функции вида (22)),
то интеграл сводится к конечному или бесконечному ряду. Если 𝛼 имеет интегрируемую производную, то интеграл сводится к обычному интегралу Римана. Это делает возможным во многих
случаях изучать ряды и интегралы одновременно, а не раздельно.
Чтобы проиллюстрировать эту мысль, рассмотрим пример из физики. Момент инерции прямого
куска проволоки единичной длины относительно оси, проходящей через его конец под прямым
углом к проволоке, равен
1
(33)
𝑥2 𝑑𝑚,
∫
0
где 𝑚(𝑥) — масса, содержащаяся в сегменте [0, 𝑥]. Если считать проволоку имеющей непрерывную
плотность 𝜌, т. е. если 𝑚′ (𝑥) = 𝜌(𝑥), то (33) превращается в
1
(34)
∫
0
𝑥2 𝜌(𝑥) 𝑑𝑥.
С другой стороны, если проволока состоит из масс 𝑚𝑖 , сосредоточенных в точках 𝑥𝑖 , то (33)
становится равным
(35)
∑
𝑥2𝑖 𝑚𝑖 .
𝑖
Итак, выражение (33) содержит (34) и (35) как частные случаи, но оно содержит гораздо больше;
например, случай, в котором 𝑚 непрерывна, но не всюду дифференцируема.
6.19 Теорема (замена переменной). Пусть 𝜑 — строго возрастающая непрерывная функция,
отображающая сегмент [𝐴, 𝐵] на [𝑎, 𝑏]. Пусть 𝛼 монотонно возрастает на [𝑎, 𝑏] и 𝑓 ∈ ℛ(𝛼)
на [𝑎, 𝑏]. Определим 𝛽 и 𝑔 на [𝐴, 𝐵] как
(36)
𝛽(𝑦) = 𝛼(𝜑(𝑦)),
𝑔(𝑦) = 𝑓 (𝜑(𝑦)).
Тогда 𝑔 ∈ ℛ(𝛽) и
𝐵
(37)
∫
𝐴
𝑏
𝑔 𝑑𝛽 =
∫
𝑎
𝑓 𝑑𝛼.
Доказательство. Каждому разбиению 𝑃 = {𝑥0 , … , 𝑥𝑛 } сегмента [𝑎, 𝑏] соответствует разбиение 𝑄 =
{𝑦0 , … , 𝑦𝑛 } сегмента [𝐴, 𝐵], такое, что 𝑥𝑖 = 𝜑(𝑦𝑖 ). Все разбиения сегмента [𝐴, 𝐵] получаются таким
образом. Так как значения, принимаемые функцией 𝑓 на [𝑥𝑖−1 , 𝑥𝑖 ], точно те же, что и принимаемые
функцией 𝑔 на [𝑦𝑖−1 , 𝑦𝑖 ], мы видим, что
(38)
𝑈 (𝑄, 𝑔, 𝛽) = 𝑈 (𝑃 , 𝑓 , 𝛼),
𝐿(𝑄, 𝑔, 𝛽) = 𝐿(𝑃 , 𝑓 , 𝛼).
Так как 𝑓 ∈ ℛ(𝛼), 𝑃 может быть выбрано так, что 𝑈 (𝑃 , 𝑓 , 𝛼) и 𝐿(𝑃 , 𝑓 , 𝛼) оба близки к ∫ 𝑓 𝑑𝛼.
Значит, (38) вместе с теоремой 6.6 показывает, что 𝑔 ∈ ℛ(𝛽) и что выполняется (37). Доказательство
завершено.
Отметим следующий специальный случай.
Возьмем 𝛼(𝑥) = 𝑥, тогда 𝛽 = 𝜑. Допустим, что 𝜑′ ∈ ℛ на [𝐴, 𝐵]. Если применить теорему 6.17 к
левой части равенства (37), мы получим
𝑏
(39)
∫
𝑎
𝐵
𝑓 (𝑥) 𝑑𝑥 =
∫
𝐴
99
𝑓 (𝜑(𝑦))𝜑′ (𝑦) 𝑑𝑦.
Интегрирование и дифференцирование
В этом разделе мы все еще будем заниматься вещественными функциями. Мы покажем, что интегрирование и дифференцирование являются в некотором смыле взаимно обратными операциями.
6.20 Теорема. Пусть 𝑓 ∈ ℛ на [𝑎, 𝑏]. Для 𝑎 ≤ 𝑥 ≤ 𝑏 положим
𝑥
𝐹 (𝑥) =
𝑓 (𝑡) 𝑑𝑡.
∫
𝑎
Тогда 𝐹 непрерывна на [𝑎, 𝑏]; более того, если 𝑓 непрерывна в точке 𝑥0 сегмента [𝑎, 𝑏], то 𝐹
дифференцируема в 𝑥0 и
𝐹 ′ (𝑥0 ) = 𝑓 (𝑥0 ).
Доказательство. Так как 𝑓 ∈ ℛ, 𝑓 ограничена. Допустим, что |𝑓 (𝑡)| ≤ 𝑀 при 𝑎 ≤ 𝑡 ≤ 𝑏. Если
𝑎 ≤ 𝑥 < 𝑦 ≤ 𝑏, то
𝑦
|𝐹 (𝑦) − 𝐹 (𝑥)| =
|∫
𝑥
𝑓 (𝑡) 𝑑𝑡 ≤ 𝑀(𝑦 − 𝑥)
|
по теореме 6.12(c) и (d). Мы видим, что для данного 𝜀 > 0
|𝐹 (𝑦) − 𝐹 (𝑥)| < 𝜀,
если только |𝑦 − 𝑥| < 𝜀/𝑀. Этим доказана непрерывность (и, более того, равномерная непрерывность) функции 𝐹 .
Допустим теперь, что 𝑓 непрерывна в 𝑥0 . Для заданного 𝜀 > 0 выберем 𝛿 > 0 так, что
|𝑓 (𝑡) − 𝑓 (𝑥0 )| < 𝜀,
если |𝑡 − 𝑥0 | < 𝛿 и 𝑎 ≤ 𝑡 ≤ 𝑏. Тогда при
𝑥0 − 𝛿 < 𝑠 ≤ 𝑥 0 ≤ 𝑡 < 𝑥 0 + 𝛿
и
𝑎≤𝑠<𝑡≤𝑏
мы по теореме 6.12(d) имеем
𝑡
𝐹 (𝑡) − 𝐹 (𝑠)
1
− 𝑓 (𝑥0 ) =
[𝑓 (𝑢) − 𝑓 (𝑥0 )] 𝑑𝑢 < 𝜀.
| |𝑡 − 𝑠 ∫
|
|
𝑡−𝑠
𝑠
Следовательно, 𝐹 ′ (𝑥0 ) = 𝑓 (𝑥0 ).
6.21 Основная теорема интегрального исчисления. Если 𝑓 ∈ ℛ на [𝑎, 𝑏] и если существует
дифференцируемая функция 𝐹 на [𝑎, 𝑏], такая, что 𝐹 ′ = 𝑓 , то
𝑏
∫
𝑎
𝑓 (𝑥) 𝑑𝑥 = 𝐹 (𝑏) − 𝐹 (𝑎).
Доказательство. Пусть дано 𝜀 > 0. Выберем разбиение 𝑃 = {𝑥0 , … , 𝑥𝑛 } сегмента [𝑎, 𝑏] так, чтобы
𝑈 (𝑃 , 𝑓 ) − 𝐿(𝑃 , 𝑓 ) < 𝜀. По теореме о среднем значении существуют точки 𝑡𝑖 ∈ [𝑥𝑖−1 , 𝑥𝑖 ], такие, что
𝐹 (𝑥𝑖 ) − 𝐹 (𝑥𝑖−1 ) = 𝑓 (𝑡𝑖 )Δ𝑥𝑖
при 𝑖 = 1, … , 𝑛. Следовательно,
𝑛
∑
𝑓 (𝑡𝑖 )Δ𝑥𝑖 = 𝐹 (𝑏) − 𝐹 (𝑎).
𝑖=1
Из теоремы 6.7(c) теперь следует, что
𝑏
𝑓 (𝑥) 𝑑𝑥 < 𝜀.
𝐹 (𝑏) − 𝐹 (𝑎) −
|
|
∫
𝑎
Так как это выполняется при любом 𝜀 > 0, доказательство завершено.
6.22 Теорема (интегрирование по частям). Пусть 𝐹 и 𝐺 — дифференцируемые функции на [𝑎, 𝑏],
𝐹 ′ = 𝑓 ∈ ℛ и 𝐺′ = 𝑔 ∈ ℛ. Тогда
𝑏
∫
𝑎
𝑏
𝐹 (𝑥)𝑔(𝑥) 𝑑𝑥 = 𝐹 (𝑏)𝐺(𝑏) − 𝐹 (𝑎)𝐺(𝑎) −
∫
𝑎
𝑓 (𝑥)𝐺(𝑥) 𝑑𝑥.
Доказательство. Положим 𝐻(𝑥) = 𝐹 (𝑥)𝐺(𝑥) и применим теорему 6.21 к 𝐻 и ее производной.
Заметим, что 𝐻 ′ ∈ ℛ по теореме 6.13.
100
Интегрирование векторнозначных функций
6.23 Определение. Пусть 𝑓1 , … , 𝑓𝑘 — вещественные функции на [𝑎, 𝑏], и пусть f = (𝑓1 , … , 𝑓𝑘 ) —
соответствующее отображение сегмента [𝑎, 𝑏] в 𝑅𝑘 . Пусть 𝛼 монотонно возрастает на [𝑎, 𝑏]. Мы
будем говорить, что f ∈ ℛ(𝛼), если 𝑓𝑗 ∈ ℛ(𝛼) при 𝑗 = 1, … , 𝑘. В этом случае мы по определению
полагаем
𝑏
∫
𝑎
𝑏
f 𝑑𝛼 =
(∫
𝑎
𝑏
𝑓1 𝑑𝛼, … ,
∫
𝑎
𝑓𝑘 𝑑𝛼 .
)
Иными словами, ∫ f 𝑑𝛼 — это точка в 𝑅𝑘 , 𝑗-ая координата которой равна ∫ 𝑓𝑗 𝑑𝛼.
Ясно, что части (a), (c) и (e) теоремы 6.12 верны и для этих векторнозначных интегралов; нужно
просто применить прежние результаты к каждой координате. То же верно в отношении теорем 6.17,
6.20 и 6.21. Для иллюстрации сформулируем аналог теоремы 6.21.
6.24 Теорема. Если f и F отображают [𝑎, 𝑏] в 𝑅𝑘 , f ∈ ℛ на [𝑎, 𝑏] и F′ = f, то
𝑏
f(𝑡) 𝑑𝑡 = F(𝑏) − F(𝑎).
∫
𝑎
Однако аналог теоремы 6.13(b) связан с некоторыми новыми моментами, по крайней мере в
доказательстве.
6.25 Теорема. Если f отображает [𝑎, 𝑏] в 𝑅𝑘 и если f ∈ ℛ(𝛼) для какой-нибудь монотонно
возрастающей функции 𝛼 на [𝑎, 𝑏], то |f| ∈ ℛ(𝛼) и
𝑏
(40)
𝑏
f 𝑑𝛼 ≤
|f| 𝑑𝛼.
|∫
| ∫
𝑎
𝑎
Доказательство. Если 𝑓1 , … , 𝑓𝑘 — компоненты отображения f, то
|f| = (𝑓12 + ⋯ + 𝑓𝑘2 )1/2 .
(41)
По теореме 6.11 каждая из функций 𝑓𝑖2 принадлежит ℛ(𝛼), следовательно, этому множеству принадлежит и их сумма. Поскольку 𝑥2 — непрерывная функция от 𝑥, теорема 4.17 показывает, что
квадратный корень — функция, непрерывная на [0, 𝑀] при каждом вещественном 𝑀. Если мы еще
раз применим теорему 6.11, то увидим, что |f| ∈ ℛ(𝛼).
Чтобы доказать (40), положим y = (𝑦1 , … , 𝑦𝑘 ), где 𝑦𝑗 = ∫ 𝑓𝑗 𝑑𝛼. Тогда y = ∫ f 𝑑𝛼 и
|y|2 =
∑
𝑦2𝑖 =
∑
𝑦𝑖
∫
𝑓𝑗 𝑑𝛼 =
( 𝑦 𝑓 ) 𝑑𝛼.
∫∑ 𝑗 𝑗
В силу неравенства Шварца
(42)
∑
𝑦𝑗 𝑓𝑗 (𝑡) ≤ |y||f(𝑡)|
(𝑎 ≤ 𝑡 ≤ 𝑏);
значит, из теоремы 6.12(b) следует, что
(43)
|y|2 ≤ |y|
∫
|f| 𝑑𝛼.
Если y = 0, то (40) тривиально. Если y ≠ 0, то, разделив (43) на |y|, мы получим (40).
Спрямляемые кривые
В заключение этой главы мы рассмотрим одно интересное геометрическое приложение некоторой
части предшествующей теории. Случай 𝑘 = 2 (т. е. случай кривых на плоскости) особенно важен
при изучении аналитических функций комплексной переменной.
101
6.26 Определение. Непрерывное отображение 𝛾 сегмента [𝑎, 𝑏] в 𝑅𝑘 называется кривой в 𝑅𝑘 .
Чтобы подчеркнуть сегмент параметра [𝑎, 𝑏], мы можем также говорить, что 𝛾 — кривая на [𝑎, 𝑏].
Если 𝛾 взаимно однозначно, то 𝛾 называется дугой.
Если 𝛾(𝑎) = 𝛾(𝑏), то 𝛾 называется замкнутой кривой.
Следует заметить, что мы определяем кривую как отображение, а не как множество точек.
Конечно, с каждой кривой 𝛾 в 𝑅𝑘 связано некоторое множество точек в 𝑅𝑘 , а именно множество
значений отображения 𝛾, однако разные кривые могут иметь одну и ту же область значений.
Свяжем с каждым разбиением 𝑃 = {𝑥0 , … , 𝑥𝑛 } сегмента [𝑎, 𝑏] и каждой кривой 𝛾 на [𝑎, 𝑏] число
𝑛
Λ(𝑃 , 𝛾) =
|𝛾(𝑥𝑖 ) − 𝛾(𝑥𝑖−1 )|.
∑
𝑖=1
В этой сумме 𝑖-ое слагаемое — расстояние (в 𝑅𝑘 ) между точками 𝛾(𝑥𝑖−1 ) и 𝛾(𝑥𝑖 ). Значит, Λ(𝑃 , 𝛾) —
длина ломаной с вершинами в 𝛾(𝑥0 ), 𝛾(𝑥1 ), … , 𝛾(𝑥𝑛 ) в указанном порядке. По мере того как разбиение
становится все мельче, эта ломаная приближается к множеству значений отображения 𝛾. Это дает
возможность определить длину кривой 𝛾 как
Λ(𝛾) = sup Λ(𝑃 , 𝛾),
где верхняя грань берется по всем разбиениям сегмента [𝑎, 𝑏].
Если Λ(𝛾) < ∞, мы будем говорить, что 𝛾 спрямляема.
В некоторых случаях Λ(𝛾) выражается интегралом Римана. Мы докажем это для непрерывно
дифференцируемых кривых, т. е. для кривых 𝛾, производная которых 𝛾 ′ непрерывна.
6.27 Теорема. Если 𝛾 ′ непрерывна на [𝑎, 𝑏], то 𝛾 спрямляема и
𝑏
Λ(𝛾) =
∫
𝑎
|𝛾 ′ (𝑡)| 𝑑𝑡.
Доказательство. Если 𝑎 ≤ 𝑥𝑖−1 < 𝑥𝑖 ≤ 𝑏, то
𝑥𝑖
|𝛾(𝑥𝑖 ) − 𝛾(𝑥𝑖−1 )| =
|∫
𝑥
𝑖−1
𝛾 ′ (𝑡) 𝑑𝑡 ≤
| ∫
𝑥
𝑥𝑖
|𝛾 ′ (𝑡)| 𝑑𝑡.
𝑖−1
Значит,
𝑏
|𝛾 ′ (𝑡)| 𝑑𝑡
∫
𝑎
для каждого разбиения 𝑃 сегмента [𝑎, 𝑏]. Следовательно,
Λ(𝑃 , 𝛾) ≤
𝑏
Λ(𝛾) ≤
∫
𝑎
|𝛾 ′ (𝑡)| 𝑑𝑡.
Докажем обратное неравенство. Пусть дано 𝜀 > 0. Ввиду того что 𝛾 ′ равномерно непрерывна
на [𝑎, 𝑏], существует 𝛿 > 0, такое, что
|𝛾 ′ (𝑠) − 𝛾 ′ (𝑡)| < 𝜀,
если |𝑠 − 𝑡| < 𝛿.
Пусть 𝑃 = {𝑥0 , … , 𝑥𝑛 } — разбиение сегмента [𝑎, 𝑏], у которого Δ𝑥𝑖 < 𝛿 при всех 𝑖. Если 𝑥𝑖−1 ≤ 𝑡 ≤ 𝑥𝑖 ,
то
|𝛾 ′ (𝑡)| ≤ |𝛾 ′ (𝑥𝑖 )| + 𝜀,
так что
𝑥𝑖
∫
𝑥𝑖−1
|𝛾 ′ (𝑡)| 𝑑𝑡 ≤ |𝛾 ′ (𝑥𝑖 )|Δ𝑥𝑖 + 𝜀Δ𝑥𝑖
𝑥𝑖
=
|∫
𝑥
𝑖−1
𝑥𝑖
≤
|∫
𝑥
𝑖−1
[𝛾 ′ (𝑡) + 𝛾 ′ (𝑥𝑖 ) − 𝛾 ′ (𝑡)] 𝑑𝑡 + 𝜀Δ𝑥𝑖
|
𝑥𝑖
𝛾 ′ (𝑡) 𝑑𝑡 +
[𝛾 ′ (𝑥𝑖 ) − 𝛾 ′ (𝑡)] 𝑑𝑡 + 𝜀Δ𝑥𝑖
| |∫
|
𝑥
𝑖−1
≤ |𝛾(𝑥𝑖 ) − 𝛾(𝑥𝑖−1 )| + 2𝜀Δ𝑥𝑖 .
102
Суммируя эти неравенства, мы получим
𝑏
∫
𝑎
|𝛾 ′ (𝑡)| 𝑑𝑡 ≤ Λ(𝑃 , 𝛾) + 2𝜀(𝑏 − 𝑎)
≤ Λ(𝛾) + 2𝜀(𝑏 − 𝑎).
Так как 𝜀 было произвольным,
𝑏
∫
𝑎
|𝛾 ′ (𝑡)| 𝑑𝑡 ≤ Λ(𝛾).
Доказательство закончено.
Упражнения
1. Пусть 𝛼 возрастает на [𝑎, 𝑏], 𝑎 ≤ 𝑥0 ≤ 𝑏, 𝛼 непрерывна в 𝑥0 , 𝑓 (𝑥0 ) = 1 и 𝑓 (𝑥) = 0, если 𝑥 ≠ 𝑥0 . Доказать,
что 𝑓 ∈ ℛ(𝛼) и ∫ 𝑓 𝑑𝛼 = 0.
𝑏
2. Пусть 𝑓 ≥ 0, 𝑓 непрерывна на [𝑎, 𝑏] и
∫
𝑎
𝑓 (𝑥) 𝑑𝑥 = 0. Доказать, что 𝑓 (𝑥) = 0 при всех 𝑥 ∈ [𝑎, 𝑏] (ср. с
упражнением 1).
3. Определим три функции 𝛽1 , 𝛽2 , 𝛽3 следующим образом: 𝛽𝑗 (𝑥) = 0, если 𝑥 < 0, 𝛽𝑗 (𝑥) = 1, если 𝑥 > 0 при
𝑗 = 1, 2, 3; 𝛽1 (0) = 0, 𝛽2 (0) = 1, 𝛽3 (0) = 12 . Пусть 𝑓 — ограниченная функция на [−1, 1].
(a) Доказать, что 𝑓 ∈ ℛ(𝛽1 ) тогда и только тогда, когда 𝑓 (0+) = 𝑓 (0), и что в этом случае
∫
𝑓 𝑑𝛽1 = 𝑓 (0).
(b) Сформулировать и доказать аналогичный результат для 𝛽2 .
(c) Доказать, что 𝑓 ∈ ℛ(𝛽3 ) тогда и только тогда, когда 𝑓 непрерывна в 0.
(d) Пусть 𝑓 непрерывна в 0. Доказать, что
∫
𝑓 𝑑𝛽1 =
∫
𝑓 𝑑𝛽2 =
∫
𝑓 𝑑𝛽3 = 𝑓 (0).
4. Пусть 𝑓 (𝑥) = 0 при всех иррациональных 𝑥, 𝑓 (𝑥) = 1 при всех рациональных 𝑥. Доказать, что 𝑓 ∉ ℛ на
[𝑎, 𝑏] при любых 𝑎 < 𝑏.
5. Пусть 𝑓 — ограниченная вещественная функция на [𝑎, 𝑏] и 𝑓 2 ∈ ℛ на [𝑎, 𝑏]. Следует ли из этого, что
𝑓 ∈ ℛ? Изменится ли ответ, если предположить, что 𝑓 3 ∈ ℛ?
6. Пусть 𝑃 — множество Кантора, построенное в пункте 2.44. Пусть 𝑓 — ограниченная вещественная функция на [0, 1], непрерывная в каждой точке вне 𝑃 . Доказать, что 𝑓 ∈ ℛ на [0,1]. Указание. 𝑃 может быть
покрыто конечным количеством интервалов, общая длина которых может быть сделана сколь угодно
малой. Продолжить рассуждение, как в теореме 6.10.
7. Пусть 𝑓 — вещественная функция на (0, 1] и 𝑓 ∈ ℛ на [𝑐, 1] при любом 𝑐 > 0. Положим по определению
1
∫
0
1
𝑓 (𝑥) 𝑑𝑥 = lim
𝑐→0
𝑓 (𝑥) 𝑑𝑥,
∫
𝑐
если этот предел существует (и конечен).
(a) Если 𝑓 ∈ ℛ на [0, 1], показать, что это определение интеграла согласуется со старым.
(b) Построить такую функцию 𝑓 , что вышеприведенный предел существует, но перестает существовать при замене 𝑓 на |𝑓 |.
8. Пусть 𝑓 ∈ ℛ на [𝑎, 𝑏] для любого 𝑏 > 𝑎 и фиксированного 𝑎. Положим по определению
∞
∫
𝑎
𝑏
𝑓 (𝑥) 𝑑𝑥 = lim
𝑏→∞ ∫
𝑎
𝑓 (𝑥) 𝑑𝑥,
если этот предел существует (и конечен). В этом случае мы будем говорить, что интеграл в левой части
сходится. Если он также сходится при замене 𝑓 на |𝑓 |, будем говорить, что он сходится абсолютно.
Пусть 𝑓 (𝑥) ≥ 0 и 𝑓 монотонно убывает на [1, ∞). Доказать, что интеграл
∞
∫
1
𝑓 (𝑥) 𝑑𝑥
103
сходится тогда и только тогда, когда сходится сумма
∞
𝑓 (𝑛)
∑
𝑛=1
(это так называемый «интегральный признак» сходимости рядов).
9. Показать, что интегрирование по частям иногда может применяться к «несобственным» интегралам,
определенным в упражнениях 7 и 8 (определите подходящие предположения, сформулируйте теорему
и докажите ее). Для иллюстрации покажите, что
∞
∞
cos 𝑥
sin 𝑥
𝑑𝑥 =
𝑑𝑥.
∫
∫
1+𝑥
(1 + 𝑥)2
0
0
Покажите, что один из этих интегралов сходится абсолютно, а другой — нет.
10. Пусть 𝑝 и 𝑞 — положительные вещественные числа, такие, что
1 1
+ = 1.
𝑝 𝑞
Докажите следующие утверждения.
(a) Если 𝑢 ≥ 0 и 𝑣 ≥ 0, то
𝑢𝑝 𝑣𝑞
+ .
𝑝
𝑞
𝑢𝑣 ≤
Равенство имеет место тогда и только тогда, когда 𝑢𝑝 = 𝑣𝑞 .
(b) Если 𝑓 ∈ ℛ(𝛼), 𝑔 ∈ ℛ(𝛼), 𝑓 ≥ 0, 𝑔 ≥ 0 и
𝑏
∫
𝑎
𝑏
𝑓 𝑝 𝑑𝛼 = 1 =
то
𝑔 𝑞 𝑑𝛼,
∫
𝑎
𝑏
∫
𝑎
𝑓 𝑔 𝑑𝛼 ≤ 1.
(c) Если 𝑓 и 𝑔 — комплексные функции в ℛ(𝛼), то
𝑏
1/𝑝
𝑏
𝑓 𝑔 𝑑𝛼 ≤
|𝑓 |𝑝 𝑑𝛼
|∫
| {∫
}
𝑎
𝑎
1/𝑞
𝑏
{∫
𝑎
|𝑔|𝑞 𝑑𝛼
}
.
Это неравенство называется неравенством Гёльдера. При 𝑝 = 𝑞 = 2 оно обычно называется неравенством Шварца (заметим, что теорема 1.35 — очень частный случай этого неравенства).
(d) Показать, что неравенство Гёльдера также выполняется для «несобственных» интегралов, описанных в упражнениях 7 и 8.
11. Пусть 𝛼 — фиксированная возрастающая функция на [𝑎, 𝑏]. Для 𝑢 ∈ ℛ(𝛼) положим по определению
1/2
𝑏
‖𝑢‖2 =
{∫
𝑎
|𝑢|2 𝑑𝛼
}
.
Если 𝑓 , 𝑔, ℎ ∈ ℛ(𝛼) доказать неравенство треугольника
‖𝑓 − ℎ‖2 ≤ ‖𝑓 − 𝑔‖2 + ‖𝑔 − ℎ‖2
как следствие неравенства Шварца, аналогично доказательству теоремы 1.37.
12. В обозначениях упражнения 11 допустим, что 𝑓 ∈ ℛ(𝛼) и 𝜀 > 0. Доказать, что существует непрерывная
функция 𝑔 на [𝑎, 𝑏], такая, что ‖𝑓 − 𝑔‖2 < 𝜀.
Указание. Пусть 𝑃 = {𝑥0 , … , 𝑥𝑛 } — подходящее разбиение сегмента [𝑎, 𝑏]. Положить
𝑔(𝑡) =
𝑡 − 𝑥𝑖−1
𝑥𝑖 − 𝑡
𝑓 (𝑥𝑖−1 ) +
𝑓 (𝑥𝑖 ),
Δ𝑥𝑖
Δ𝑥𝑖
если 𝑥𝑖−1 ≤ 𝑡 ≤ 𝑥𝑖 .
13. Положим
𝑥+1
𝑓 (𝑥) =
∫
𝑥
104
sin(𝑡2 ) 𝑑𝑡.
(a) Доказать, что |𝑓 (𝑥)| < 1/𝑥 при 𝑥 > 0.
Указание. Положить 𝑡2 = 𝑢 и проинтегрировать по частям, чтобы показать, что 𝑓 (𝑥) равно
2
(𝑥+1)
cos(𝑥2 ) cos[(𝑥 + 1)2 ]
cos 𝑢
−
−
𝑑𝑢.
∫
2𝑥
2(𝑥 + 1)
4𝑢3/2
2
𝑥
Заменить cos 𝑢 на −1.
(b) Доказать, что
2𝑥𝑓 (𝑥) = cos(𝑥2 ) − cos[(𝑥 + 1)2 ] + 𝑟(𝑥),
где |𝑟(𝑥)| < 𝑐/𝑥 при некоторой постоянной 𝑐.
(c) Найти верхний и нижний пределы функции 𝑥𝑓 (𝑥) при 𝑥 → ∞.
∞
(d) Сходится ли
∫
0
sin(𝑡2 ) 𝑑𝑡?
14. Поступить подобным образом с функцией
𝑥+1
𝑓 (𝑥) =
sin(𝑒𝑡 ) 𝑑𝑡.
∫
𝑥
Показать, что
𝑒𝑥 |𝑓 (𝑥)| < 2
и что
𝑒𝑥 𝑓 (𝑥) = cos(𝑒𝑥 ) − 𝑒−1 cos(𝑒𝑥+1 ) + 𝑟(𝑥),
где |𝑟(𝑥)| < 𝐶𝑒−𝑥 при некоторой постоянной 𝐶.
15. Пусть 𝑓 — вещественная непрерывно дифференцируемая функция на [𝑎, 𝑏], 𝑓 (𝑎) = 𝑓 (𝑏) = 0 и
𝑏
∫
𝑎
𝑓 2 (𝑥) 𝑑𝑥 = 1.
Доказать, что
𝑏
∫
𝑎
и что
𝑥𝑓 (𝑥)𝑓 ′ (𝑥) 𝑑𝑥 = − 12
𝑏
∫
𝑎
𝑏
[𝑓 ′ (𝑥)]2 𝑑𝑥 ⋅
∫
𝑎
𝑥2 𝑓 2 (𝑥) 𝑑𝑥 > 14 .
16. При 1 < 𝑠 < ∞ определим
∞
𝜁(𝑠) =
1
∑ 𝑛𝑠
𝑛=1
(это дзета-функция Римана, имеющая большую важность при изучении распределения простых чисел).
Доказать, что
(a) 𝜁(𝑠) = 𝑠
∞
[𝑥]
𝑑𝑥
∫
𝑥𝑠+1
1
и что
(b) 𝜁 (𝑠) =
∞
𝑥 − [𝑥]
𝑠
−𝑠
𝑑𝑥,
∫
𝑠−1
𝑥𝑠+1
1
где [𝑥] обозначает наибольшее из целых чисел, не превосходящих 𝑥.
Доказать, что интеграл в (b) сходится при всех 𝑠 > 0.
Указание. Для доказательства утверждения (a) вычислите разность между интервалом по [1, 𝑁] и 𝑁-ой
частной суммой ряда, определяющего 𝜁 (𝑠).
17. Пусть 𝛼 монотонно возрастает на [𝑎, 𝑏], 𝑔 непрерывна и 𝑔(𝑥) = 𝐺′ (𝑥) при 𝑎 ≤ 𝑥 ≤ 𝑏. Доказать, что
𝑏
𝑏
∫
𝑎
𝛼(𝑥)𝑔(𝑥) 𝑑𝑥 = 𝐺(𝑏)𝛼(𝑏) − 𝐺(𝑎)𝛼(𝑎) −
∫
𝑎
𝐺 𝑑𝛼.
Указание. Без потери общности можно считать 𝑔 вещественной. Если дано 𝑃 = {𝑥0 , 𝑥1 , … , 𝑥𝑛 }, выберем
𝑡𝑖 ∈ (𝑥𝑖−1 , 𝑥𝑖 ) так, чтобы 𝑔(𝑡𝑖 )Δ𝑥𝑖 = 𝐺(𝑥𝑖 ) − 𝐺(𝑥𝑖−1 ). Показать, что
𝑛
∑
𝑛
𝛼(𝑥𝑖 )𝑔(𝑡𝑖 )Δ𝑥𝑖 = 𝐺(𝑏)𝛼(𝑏) − 𝐺(𝑎)𝛼(𝑎) −
𝑖=1
∑
𝑖=1
105
𝐺(𝑥𝑖−1 )Δ𝛼𝑖 .
18. Пусть 𝛾1 , 𝛾2 , 𝛾3 — кривые на комплексной плоскости, определенные на [0, 2𝜋] как
𝛾1 (𝑡) = 𝑒𝑖𝑡 ,
𝛾2 (𝑡) = 𝑒2𝑖𝑡 ,
𝛾3 (𝑡) = 𝑒2𝜋𝑖𝑡 sin(1/𝑡) .
Показать, что эти три кривые имеют одно и то же множество значений, длина кривой 𝛾1 равна 2𝜋, длина
кривой 𝛾2 равна 4𝜋, а 𝛾3 не спрямляема.
19. Пусть 𝛾1 — кривая в 𝑅𝑘 , заданная на [𝑎, 𝑏]; пусть 𝜑 — непрерывное взаимно однозначное отображение
сегмента [𝑐, 𝑑] на [𝑎, 𝑏], такое, что 𝜑(𝑐) = 𝑎. Положим, по определению, 𝛾2 (𝑠) = 𝛾1 (𝜑(𝑠)). Доказать, что
𝛾2 — дуга, замкнутая кривая или спрямляемая кривая тогда и только тогда, когда то же самое верно в
отношении кривой 𝛾1 . Доказать, что 𝛾2 и 𝛾1 имеют одну и ту же длину.
106
Глава 7
Последовательности и ряды
функций
В этой главе мы уделим основное внимание комплекснозначным (включая, конечно, вещественнозначные) функциям, хотя многие теоремы и доказательства без труда переносятся на случай
векторнозначных функций и даже отображений в общие метрические пространства. Мы предпочитаем ограничиться более простым классом функций для того, чтобы сконцентрировать внимание
на наиболее важных вопросах, возникающих при изменении порядка предельных переходов.
Обсуждение главной проблемы
7.1 Определение. Пусть {𝑓𝑛 }, 𝑛 = 1, 2, 3, … , — последовательность функций, определенных на
множестве 𝐸, и пусть последовательность чисел {𝑓𝑛 (𝑥)} сходится при каждом 𝑥 ∈ 𝐸. Тогда мы
можем определить функцию 𝑓 , полагая
(1)
𝑓 (𝑥) = lim 𝑓𝑛 (𝑥)
𝑛→∞
(𝑥 ∈ 𝐸).
В подобных случаях мы будем говорить, что {𝑓𝑛 } сходится на 𝐸 и что 𝑓 — предел, или предельная функция, последовательности {𝑓𝑛 }. Иногда мы будем использовать более наглядную терминологию и говорить, что {𝑓𝑛 } сходится к 𝑓 поточечно на 𝐸, если выполнено (1). Аналогично, если
ряд ∑ 𝑓𝑛 (𝑥) сходится при любом 𝑥 ∈ 𝐸, то мы полагаем, по определению,
∞
(2)
𝑓 (𝑥) =
∑
𝑓𝑛 (𝑥)
(𝑥 ∈ 𝐸),
𝑛=1
причем функция 𝑓 называется суммой ряда ∑ 𝑓𝑛 .
Главная из возникающих проблем такова: выяснить, сохраняются ли важнейшие свойства функций при выполнении предельных операций (1) и (2). Например, если функции 𝑓𝑛 непрерывны, или
дифференцируемы, или интегрируемы, то верно ли то же самое в отношении предельной функции?
Каковы соотношения между, скажем, 𝑓𝑛′ и 𝑓 ′ или между интегралами от 𝑓𝑛 и от 𝑓 ?
Непрерывность функции 𝑓 в предельной точке 𝑥 означает, что
lim 𝑓 (𝑡) = 𝑓 (𝑥).
𝑡→𝑥
Значит, спросить, будет ли предел последовательности непрерывных функций непрерывной функцией, — это все равно, что спросить, верно ли равенство
(3)
lim lim 𝑓𝑛 (𝑡) = lim lim 𝑓𝑛 (𝑡),
𝑡→𝑥 𝑛→∞
𝑛→∞ 𝑡→𝑥
т. е. важен ли порядок, в котором осуществляются предельные переходы. В левой части равенства
(3) мы сначала устремляем 𝑛 к ∞, затем 𝑡 к 𝑥; в правой части — сначала 𝑡 к 𝑥, а затем 𝑛 к ∞.
107
Мы покажем сейчас на нескольких примерах, что, вообще говоря, два предельных перехода
нельзя переставить без того, чтобы это не повлияло на результат. Затем мы докажем, что при
некоторых условиях порядок, в котором выполняются операции предельного перехода, не имеет
значения.
Наш первый и простейший пример связан с «двойной последовательностью».
7.2 Пример. Для 𝑚 = 1, 2, 3, … , 𝑛 = 1, 2, 3, … положим
𝑠𝑚,𝑛 =
𝑚
.
𝑚+𝑛
Тогда при каждом фиксированном 𝑛
lim 𝑠
𝑚→∞ 𝑚,𝑛
= 1,
так что
(4)
lim lim 𝑠
𝑛→∞ 𝑚→∞ 𝑚,𝑛
= 1.
С другой стороны, при каждом фиксированном 𝑚
lim 𝑠
𝑛→∞ 𝑚,𝑛
= 0,
так что
(5)
lim lim 𝑠
𝑚→∞ 𝑛→∞ 𝑚,𝑛
= 0.
7.3 Пример. Пусть
𝑓𝑛 (𝑥) =
𝑥2
(1 + 𝑥2 )𝑛
(𝑥 вещественно, 𝑛 = 0, 1, 2, … ),
и пусть
∞
(6)
𝑓 (𝑥) =
∞
∑
𝑓𝑛 (𝑥) =
𝑛=0
𝑥2
.
∑ (1 + 𝑥2 )𝑛
𝑛=0
Так как 𝑓𝑛 (0) = 0, то и 𝑓 (0) = 0. При 𝑥 ≠ 0 последний ряд в (6) представляет сходящуюся геометрическую прогрессию с суммой 1 + 𝑥2 (теорема 3.26). Значит,
(7)
𝑓 (𝑥) =
0
{1 + 𝑥 2
(𝑥 = 0),
(𝑥 ≠ 0),
так что сходящийся ряд непрерывных функций может иметь разрывную сумму.
7.4 Пример. При 𝑚 = 1, 2, 3, … положим
𝑓𝑚 (𝑥) = lim (cos 𝑚!𝜋𝑥)2𝑛 .
𝑛→∞
Если 𝑚!𝑥 — целое число, то 𝑓𝑚 (𝑥) = 1. При всех остальных значениях 𝑥 имеем 𝑓𝑚 (𝑥) = 0. Теперь
положим
𝑓 (𝑥) = lim 𝑓𝑚 (𝑥).
𝑚→∞
При иррациональных 𝑥 имеем 𝑓𝑚 (𝑥) = 0 при всех 𝑚, значит, 𝑓 (𝑥) = 0. При рациональных 𝑥, скажем,
𝑥 = 𝑝/𝑞, где 𝑝 и 𝑞 — целые числа, 𝑚!𝑥 — целое число, если 𝑚 ≥ 𝑞, так что 𝑓 (𝑥) = 1. Значит,
(8)
lim lim (cos 𝑚!𝜋𝑥)2𝑛 =
𝑚→∞ 𝑛→∞
0 (𝑥 иррационально),
{1 (𝑥 рационально).
Таким образом, мы получили всюду разрывную предельную функцию, которая не интегрируема
по Риману (упражнение 4, гл. 6).
108
7.5 Пример. Пусть
(9)
sin 𝑛𝑥
𝑓𝑛 (𝑥) =
(𝑥 вещественно, 𝑛 = 1, 2, 3, … )
√𝑛
и
𝑓 (𝑥) = lim 𝑓𝑛 (𝑥) = 0.
𝑛→∞
Тогда 𝑓 ′ (𝑥) = 0, но
𝑓𝑛′ (𝑥) = √𝑛 cos 𝑛𝑥,
так что {𝑓𝑛′ } не сходится к 𝑓 ′ . Например,
𝑓𝑛′ (0) = √𝑛 → +∞
при 𝑛 → ∞, тогда как 𝑓 ′ (0) = 0.
7.6 Пример. Пусть
(10)
𝑓𝑛 (𝑥) = 𝑛2 𝑥(1 − 𝑥2 )𝑛
(0 ≤ 𝑥 ≤ 1, 𝑛 = 1, 2, 3, … ).
Если 0 < 𝑥 ≤ 1, то
lim 𝑓 (𝑥)
𝑛→∞ 𝑛
=0
по теореме 3.20(d). Ввиду того что 𝑓𝑛 (0) = 0,
(11)
lim 𝑓 (𝑥)
𝑛→∞ 𝑛
=0
(0 ≤ 𝑥 ≤ 1).
Простое вычисление показывает, что
1
∫
0
𝑥(1 − 𝑥2 )𝑛 𝑑𝑥 =
1
.
2𝑛 + 2
Таким образом, несмотря на (11),
1
𝑓𝑛 (𝑥) 𝑑𝑥 =
∫
0
𝑛2
→ +∞
2𝑛 + 2
при 𝑛 → ∞.
Если в (10) заменить 𝑛2 на 𝑛, то (11) все еще выполняется, но теперь
1
lim
𝑛→∞ ∫
0
𝑓𝑛 (𝑥) 𝑑𝑥 = lim
𝑛→∞
1
𝑛
= ,
2𝑛 + 2 2
тогда как
1
lim 𝑓𝑛 (𝑥) 𝑑𝑥 = 0.
]
∫
0 [𝑛→∞
Таким образом, предел интеграла не обязан совпадать с интегралом от предела, даже если оба они
конечны.
После этих примеров, показывающих, что беззаботная перестановка порядка предельных переходов может привести к ошибке, мы определим новый вид сходимости, более сильный, чем поточечная сходимость, описанная в определении 7.1, что позволит нам прийти к положительным
результатам.
109
Равномерная сходимость
7.7 Определение. Мы будем говорить, что последовательность функций {𝑓𝑛 }, 𝑛 = 1, 2, 3, … , сходится равномерно на 𝐸 к функции 𝑓 , если для любого 𝜀 > 0 существует целое число 𝑁, такое, что
при 𝑛 ≥ 𝑁 имеем
(12)
|𝑓𝑛 (𝑥) − 𝑓 (𝑥)| ≤ 𝜀
для всех 𝑥 ∈ 𝐸.
Ясно, что каждая равномерно сходящаяся последовательность сходится поточечно. Говоря подробно, разница между этими двумя понятиями заключается в следующем: если {𝑓𝑛 } сходится
поточечно на 𝐸, то существует функция 𝑓 , такая, что для любого 𝜀 > 0 и для любого 𝑥 ∈ 𝐸 существует целое число 𝑁, зависящее от 𝜀 и от 𝑥, такое, что (12) выполняется при 𝑛 ≥ 𝑁; если же {𝑓𝑛 }
сходится равномерно на 𝐸, то можно при каждом 𝜀 > 0 найти одно целое число 𝑁, которое будет
годиться при всех 𝑥 ∈ 𝐸.
Мы будем говорить, что ряд ∑ 𝑓𝑛 (𝑥) сходится равномерно на 𝐸, если последовательность частных сумм {𝑠𝑛 }, определенных равенством
𝑛
∑
𝑓𝑖 (𝑥) = 𝑠𝑛 (𝑥),
𝑖=1
равномерно сходится на 𝐸.
Следующая теорема представляет собой критерий Коши равномерной сходимости.
7.8 Теорема. Последовательность функций {𝑓𝑛 }, определенных на 𝐸, сходится равномерно на
𝐸 тогда и только тогда, когда для любого 𝜀 > 0 существует целое 𝑁, такое, что при 𝑚 ≥ 𝑁,
𝑛 ≥ 𝑁, 𝑥 ∈ 𝐸 имеем
(13)
|𝑓𝑛 (𝑥) − 𝑓𝑚 (𝑥)| ≤ 𝜀.
Доказательство. Допустим, что {𝑓𝑛 } сходится равномерно на 𝐸, и пусть 𝑓 — предельная функция. Тогда существует целое число 𝑁, такое, что из 𝑛 ≥ 𝑁, 𝑥 ∈ 𝐸 следует
|𝑓𝑛 (𝑥) − 𝑓 (𝑥)| ≤
𝜀
,
2
так что
|𝑓𝑛 (𝑥) − 𝑓𝑚 (𝑥)| ≤ |𝑓𝑛 (𝑥) − 𝑓 (𝑥)| + |𝑓 (𝑥) − 𝑓𝑚 (𝑥)| ≤ 𝜀,
если 𝑛 ≥ 𝑁, 𝑚 ≥ 𝑁, 𝑥 ∈ 𝐸.
Обратно, пусть выполняется условие Коши. По теореме 3.11 последовательность {𝑓𝑛 (𝑥)} сходится при каждом 𝑥 к пределу, который мы обозначим через 𝑓 (𝑥). Это значит, что последовательность
{𝑓𝑛 } сходится на 𝐸 к 𝑓 . Мы должны доказать, что сходимость равномерна.
Пусть задано 𝜀 > 0. Выберем 𝑁 так, чтобы выполнялось (13). Зафиксируем 𝑛 и положим 𝑚 → ∞
в (13). Ввиду того что 𝑓𝑚 (𝑥) → 𝑓 (𝑥) при 𝑚 → ∞, получаем
(14)
|𝑓𝑛 (𝑥) − 𝑓 (𝑥)| ≤ 𝜀
при любом 𝑛 ≥ 𝑁 и любом 𝑥 ∈ 𝐸. Доказательство закончено.
Иногда полезен следующий критерий.
7.9 Теорема. Пусть
lim 𝑓 (𝑥)
𝑛→∞ 𝑛
= 𝑓 (𝑥)
(𝑥 ∈ 𝐸).
Положим
𝑀𝑛 = sup |𝑓𝑛 (𝑥) − 𝑓 (𝑥)|.
𝑥∈𝐸
Тогда 𝑓𝑛 → 𝑓 равномерно на 𝐸 в том и только в том случае, когда 𝑀𝑛 → 0 при 𝑚 → ∞.
110
Мы не приводим подробного доказательства, так как это утверждение непосредственно следует
из определения 7.7.
Вейерштрассу принадлежит очент удобный признак равномерной сходимости рядов.
7.10 Теорема. Пусть {𝑓𝑛 } — последовательность функций, определенных на 𝐸, и пусть
|𝑓𝑛 (𝑥)| ≤ 𝑀𝑛
(𝑥 ∈ 𝐸, 𝑛 = 1, 2, 3, … ).
Тогда ряд ∑ 𝑓𝑛 сходится равномерно на 𝐸, если ряд ∑ 𝑀𝑛 сходится.
Заметим, что обратное не утверждается (и в действительности неверно).
Доказательство. Если ряд ∑ 𝑀𝑛 сходится, то при любом 𝜀 > 0
𝑚
𝑚
𝑓𝑖 (𝑥) ≤
𝑀𝑖 ≤ 𝜀
|∑
| ∑
𝑖=𝑛
𝑖=𝑛
(𝑥 ∈ 𝐸),
если только 𝑚 и 𝑛 достаточно велики. Равномерная сходимость вытекает теперь из теоремы 7.8.
Равномерная сходимость и непрерывность
7.11 Теорема. Пусть 𝑓𝑛 → 𝑓 равномерно на множестве 𝐸 в некотором метрическом пространстве. Пусть 𝑥 — предельная точка множества 𝐸, и пусть
(15)
lim 𝑓𝑛 (𝑡) = 𝐴𝑛
𝑡→𝑥
(𝑛 = 1, 2, 3, … ).
Тогда последовательность {𝐴𝑛 } сходится и
(16)
lim 𝑓 (𝑡) = lim 𝐴𝑛 .
𝑡→𝑥
𝑛→∞
Иными словами, в данном случае
(17)
lim lim 𝑓𝑛 (𝑡) = lim lim 𝑓𝑛 (𝑡).
𝑡→𝑥 𝑛→∞
𝑛→∞ 𝑡→𝑥
Доказательство. Пусть задано 𝜀 > 0. В силу равномерной сходимости последовательности {𝑓𝑛 }
существует такое 𝑁, что из 𝑛 ≥ 𝑁, 𝑚 ≥ 𝑁, 𝑡 ∈ 𝐸 следует
(18)
|𝑓𝑛 (𝑡) − 𝑓𝑚 (𝑡)| ≤ 𝜀.
Устремляя в (18) 𝑡 к 𝑥, получаем
|𝐴𝑛 − 𝐴𝑚 | ≤ 𝜀
при 𝑛 ≥ 𝑁, 𝑚 ≥ 𝑁, так что {𝐴𝑛 } — последовательность Коши и потому сходится. Обозначим ее
предел через 𝐴.
Далее,
(19)
|𝑓 (𝑡) − 𝐴| ≤ |𝑓 (𝑡) − 𝑓𝑛 (𝑡)| + |𝑓𝑛 (𝑡) − 𝐴𝑛 | + |𝐴𝑛 − 𝐴|.
Выберем сначала 𝑛 так, что
(20)
𝜀
3
|𝑓 (𝑡) − 𝑓𝑛 (𝑡)| ≤
при всех 𝑡 ∈ 𝐸 (это возможно в силу равномерной сходимости) и что
(21)
|𝐴𝑛 − 𝐴| ≤
𝜀
.
3
Затем для этого 𝑛 мы подберем такую окрестность 𝑉 точки 𝑥, что
(22)
|𝑓𝑛 (𝑡) − 𝐴𝑛 | ≤
111
𝜀
3
при 𝑡 ∈ 𝑉 ∩ 𝐸, 𝑡 ≠ 𝑥.
Подставляя неравенства (20)-(22) в (19), мы получим
|𝑓 (𝑡) − 𝐴| ≤ 𝜀
для всех 𝑡 ∈ 𝑉 ∩ 𝐸, 𝑡 ≠ 𝑥. Но это равносильно (16).
7.12 Теорема. Если {𝑓𝑛 } — последовательность непрерывных функций на 𝐸 и если 𝑓𝑛 → 𝑓 равномерно на 𝐸, то 𝑓 непрерывна на 𝐸.
Этот очень важный результат сразу следует из теоремы 7.11.
Обратное неверно, т. е. последовательность непрерывных функций может сходиться к непрерывной функции, даже если сходимость неравномерная. Пример 7.6 принадлежит к такому типу
(чтобы увидеть это, достаточно применить теорему 7.9). Тем не менее есть случай, когда обратное
утверждение верно.
7.13 Теорема. Пусть 𝐾 компактно, и
(a) {𝑓𝑛 } — последовательность непрерывных функций на 𝐾;
(b) {𝑓𝑛 } поточечно сходится к непрерывной функции 𝑓 на 𝐾;
(c) 𝑓𝑛 (𝑥) ≥ 𝑓𝑛+1 (𝑥) при всех 𝑥 ∈ 𝐾, 𝑛 = 1, 2, 3, … .
Тогда 𝑓𝑛 → 𝑓 равномерно на 𝐾.
Доказательство. Положим 𝑔𝑛 = 𝑓𝑛 − 𝑓 . Тогда 𝑔𝑛 непрерывна, 𝑔𝑛 → 0 поточечно и 𝑔𝑛 ≥ 𝑔𝑛+1 . Мы
должны доказать, что 𝑔𝑛 → 0 равномерно на 𝐾.
Пусть дано 𝜀 > 0. Обозначим через 𝐾𝑛 множество всех 𝑥 ∈ 𝐾, для которых 𝑔𝑛 (𝑥) ≥ 𝜀. Так
как 𝑔𝑛 непрерывна, 𝐾𝑛 замкнуто (теорема 4.8), следовательно, компактно (теорема 2.35). Так как
𝑔𝑛 ≥ 𝑔𝑛+1 , мы имеем 𝐾𝑛 ⊃ 𝐾𝑛+1 . Зафиксируем 𝑥 ∈ 𝐾. Так как 𝑔𝑛 (𝑥) → 0, мы видим, что 𝑥 ∉ 𝐾𝑛 при
достаточно большом 𝑛. Значит, 𝑥 ∉ ⋂ 𝐾𝑛 . Другими словами, ⋂ 𝐾𝑛 пусто. Следовательно, 𝐾𝑁 пусто
при некотором 𝑁 (теорема 2.36). Из этого следует, что 0 ≤ 𝑔𝑛 (𝑥) < 𝜀 при всех 𝑥 ∈ 𝐾 и при всех
𝑛 ≥ 𝑁. Теорема доказана.
Отметим, что компактность здесь существенна. Например, если
𝑓𝑛 (𝑥) =
1
𝑛𝑥 + 1
(0 < 𝑥 < 1, 𝑛 = 1, 2, 3, … ),
то 𝑓𝑛 (𝑥) → 0 монотонно на (0, 1), но сходимость неравномерна.
7.14 Определение. Пусть 𝑋 — метрическое пространство. Обозначим через 𝒞 (𝑋) множество всех
комплекснозначных непрерывных ограниченных функций с областью определения 𝑋.
(Заметим, что требование ограниченности излишне, если 𝑋 компактно (теорема 4.15). Следовательно, если 𝑋 компактно, то 𝒞 (𝑋) содержит все комплексные непрерывные функции на 𝑋.)
Свяжем с каждой функцией 𝑓 ∈ 𝒞 (𝑋) равномерную норму
‖𝑓 ‖ = sup |𝑓 (𝑥)|.
𝑥∈𝑋
Так как 𝑓 предполагается ограниченной, ‖𝑓 ‖ < ∞. Очевидно, что ‖𝑓 ‖ = 0, только если 𝑓 (𝑥) = 0
при каждом 𝑥 ∈ 𝑋, то есть только если 𝑓 = 0. Если ℎ = 𝑓 + 𝑔, то
|ℎ(𝑥)| ≤ |𝑓 (𝑥)| + |𝑔(𝑥)| ≤ ‖𝑓 ‖ + ‖𝑔‖
при всех 𝑥 ∈ 𝑋, следовательно,
‖𝑓 + 𝑔‖ ≤ ‖𝑓 ‖ + ‖𝑔‖.
Если определить расстояние между 𝑓 ∈ 𝒞 (𝑋) и 𝑔 ∈ 𝒞 (𝑋) как ‖𝑓 − 𝑔‖, то аксиомы 2.15 для
метрики выполняются.
Тем самым мы превратили 𝒞 (𝑋) в метрическое пространство.
Теорема 7.9 может быть теперь сформулирована следующим образом:
112
Последовательность {𝑓𝑛 } сходится к 𝑓 относительно метрики пространства 𝒞 (𝑋) тогда и
только тогда, когда 𝑓𝑛 → 𝑓 равномерно на 𝑋.
По аналогии замкнутые подмножества пространства 𝒞 (𝑋) иногда называются равномерно замкнутыми, замыкание множества 𝒜 ⊂ 𝒞 (𝑋) называется его равномерным замыканием и т. д.
7.15 Теорема. Определенная выше метрика превращает 𝒞 (𝑋) в полное метрическое пространство.
Доказательство. Пусть {𝑓𝑛 } — последовательность Коши в 𝒞 (𝑋). Это означает, что любому 𝜀 > 0
соответствует 𝑁, такое, что ‖𝑓𝑛 − 𝑓𝑚 ‖ < 𝜀, если 𝑛 ≥ 𝑁 и 𝑚 ≥ 𝑁. Из этого следует (по теореме 7.8),
что существует функция 𝑓 с областью определения 𝑋, к которой последовательность {𝑓𝑛 } сходится
равномерно. По теореме 7.12 𝑓 непрерывна. Более того, 𝑓 ограничена, так как существует 𝑛, такое,
что |𝑓 (𝑥) − 𝑓𝑛 (𝑥)| < 1 при всех 𝑥 ∈ 𝑋, а 𝑓𝑛 ограничена.
Следовательно, 𝑓 ∈ 𝒞 (𝑋), и так как 𝑓𝑛 → 𝑓 равномерно на 𝑋, имеем ‖𝑓 − 𝑓𝑛 ‖ → 0 при 𝑛 → ∞.
Равномерная сходимость и интегрирование
7.16 Теорема. Пусть 𝛼 монотонно возрастает на [𝑎, 𝑏]. Пусть 𝑓𝑛 ∈ ℛ(𝛼) на [𝑎, 𝑏] при 𝑛 =
1, 2, 3, … , и пусть 𝑓𝑛 → 𝑓 равномерно на [𝑎, 𝑏]. Тогда 𝑓 ∈ ℛ(𝛼) на [𝑎, 𝑏] и
𝑏
𝑏
(23)
𝑓 𝑑𝛼 = lim
𝑛→∞ ∫
𝑎
∫
𝑎
𝑓𝑛 𝑑𝛼
(существование предела является частью заключения теоремы).
Доказательство. Достаточно доказать теорему для вещественных 𝑓𝑛 . Положим
(24)
𝜀𝑛 = sup |𝑓𝑛 (𝑥) − 𝑓 (𝑥)|,
где верхняя грань берется по 𝑎 ≤ 𝑥 ≤ 𝑏. Тогда
𝑓𝑛 − 𝜀𝑛 ≤ 𝑓 ≤ 𝑓𝑛 + 𝜀𝑛 ,
так что верхний и нижний интегралы функции 𝑓 (см. определение 6.2) удовлетворяют неравенству
𝑏
(25)
∫
𝑎
𝑏
(𝑓𝑛 − 𝜀𝑛 ) 𝑑𝛼 ≤
⨜
𝑓 𝑑𝛼 ≤
⨛
𝑓 𝑑𝛼 ≤
∫
𝑎
(𝑓𝑛 + 𝜀𝑛 ) 𝑑𝛼.
Значит,
0≤
⨛
𝑓 𝑑𝛼 −
⨜
𝑓 𝑑𝛼 ≤ 2𝜀𝑛 [𝛼(𝑏) − 𝛼(𝑎)].
Так как 𝜀𝑛 → 0 при 𝑛 → ∞ (теорема 7.9), верхний и нижний интегралы функции 𝑓 равны.
Следовательно, 𝑓 ∈ ℛ(𝛼). Еще одно применение неравенства (25) теперь дает
𝑏
(26)
|∫
𝑎
𝑏
𝑓 𝑑𝛼 −
∫
𝑎
𝑓𝑛 𝑑𝛼 ≤ 𝜀𝑛 [𝛼(𝑏) − 𝛼(𝑎)].
|
Отсюда вытекает (23).
Следствие. Если 𝑓𝑛 ∈ ℛ(𝛼) на [𝑎, 𝑏] и если
∞
𝑓 (𝑥) =
∑
𝑓𝑛 (𝑥)
(𝑎 ≤ 𝑥 ≤ 𝑏),
𝑛=1
причем ряд сходится равномерно на [𝑎, 𝑏], то
∞
𝑏
∫
𝑎
𝑓 𝑑𝛼 =
∑∫
𝑎
𝑏
𝑓𝑛 𝑑𝛼.
𝑛=1
Иными словами, ряд можно интегрировать почленно.
113
Равномерная сходимость и дифференцирование
Мы уже видели в примере 7.5, что равномерная сходимость последовательности {𝑓𝑛 } не влечет
никаких выводов о последовательности {𝑓𝑛′ }. Таким образом, нужны более сильные предположения,
чтобы заключить, что 𝑓𝑛′ → 𝑓 ′ при 𝑓𝑛 → 𝑓 .
7.17 Теорема. Пусть {𝑓𝑛 } — последовательность дифференцируемых на [𝑎, 𝑏] функций, такая,
что {𝑓𝑛 (𝑥0 )} сходится в некоторой точке 𝑥0 на [𝑎, 𝑏]. Если {𝑓𝑛′ } сходится равномерно на [𝑎, 𝑏], то
{𝑓𝑛 } сходится равномерно на [𝑎, 𝑏] к некоторой функции 𝑓 , причем
𝑓 ′ (𝑥) = lim 𝑓𝑛′ (𝑥)
(27)
(𝑎 ≤ 𝑥 ≤ 𝑏).
𝑛→∞
Доказательство. Пусть дано 𝜀 > 0. Выберем такое 𝑁, что из 𝑛 ≥ 𝑁, 𝑚 ≥ 𝑁 следует
(28)
|𝑓𝑛 (𝑥0 ) − 𝑓𝑚 (𝑥0 )| <
𝜀
2
и
|𝑓𝑛′ (𝑡) − 𝑓𝑚′ (𝑡)| <
(29)
𝜀
2(𝑏 − 𝑎)
(𝑎 ≤ 𝑡 ≤ 𝑏).
Если мы применим теорему 5.19 о среднем значении к функции 𝑓𝑛 − 𝑓𝑚 , то, как показывает (29),
мы получим
(30)
|𝑓𝑛 (𝑥) − 𝑓𝑚 (𝑥) − 𝑓𝑛 (𝑡) + 𝑓𝑚 (𝑡)| ≤
|𝑥 − 𝑡|𝜀
𝜀
≤
2(𝑏 − 𝑎) 2
при любых 𝑥 и 𝑡 на [𝑎, 𝑏], если 𝑛 ≥ 𝑁, 𝑚 ≥ 𝑁. Из неравенства
|𝑓𝑛 (𝑥) − 𝑓𝑚 (𝑥)| ≤ |𝑓𝑛 (𝑥) − 𝑓𝑚 (𝑥) − 𝑓𝑛 (𝑥0 ) + 𝑓𝑚 (𝑥0 )| + |𝑓𝑛 (𝑥0 ) − 𝑓𝑚 (𝑥0 )|
следует, в силу (28) и (30), что
|𝑓𝑛 (𝑥) − 𝑓𝑚 (𝑥)| < 𝜀
(𝑎 ≤ 𝑥 ≤ 𝑏, 𝑛 ≥ 𝑁, 𝑚 ≥ 𝑁),
а потому {𝑓𝑛 } сходится равномерно на [𝑎, 𝑏]. Пусть
𝑓 (𝑥) = lim 𝑓𝑛 (𝑥)
(𝑎 ≤ 𝑥 ≤ 𝑏).
𝑛→∞
Теперь зафиксируем точку 𝑥 на [𝑎, 𝑏] и положим
(31)
𝜑𝑛 (𝑡) =
𝑓𝑛 (𝑡) − 𝑓𝑛 (𝑥)
,
𝑡−𝑥
𝜑(𝑡) =
𝑓 (𝑡) − 𝑓 (𝑥)
𝑡−𝑥
при 𝑎 ≤ 𝑡 ≤ 𝑏, 𝑡 ≠ 𝑥. Тогда
(32)
lim 𝜑𝑛 (𝑥) = 𝑓𝑛′ (𝑥)
(𝑛 = 1, 2, 3, … ).
𝑡→𝑥
Первое неравенство в (30) показывает, что
|𝜑𝑛 (𝑡) − 𝜑𝑚 (𝑡)| ≤
𝜀
2(𝑏 − 𝑎)
(𝑛 ≥ 𝑁, 𝑚 ≥ 𝑁),
поэтому {𝜑𝑛 } сходится равномерно при 𝑡 ≠ 𝑥. Поскольку {𝑓𝑛 } сходится к 𝑓 , то, в силу (31),
(33)
lim 𝜑𝑛 (𝑡) = 𝜑(𝑡)
𝑛→∞
равномерно при 𝑎 ≤ 𝑡 ≤ 𝑏, 𝑡 ≠ 𝑥.
Применяя к последовательности {𝜑𝑛 } теорему 7.11, заключаем на основании (32) и (33), что
lim 𝜑(𝑡) = lim 𝑓𝑛′ (𝑥),
𝑡→𝑥
𝑛→∞
а это, в силу определения функции 𝜑(𝑡), и есть (27).
114
Замечание. Если дополнительно предположить непрерывность функций 𝑓𝑛′ , то можно получить
гораздо более короткое доказательство равенства (27), опирающееся на теорему 7.16 и основную
теорему интегрального исчисления.
7.18 Теорема. Существует вещественная непрерывная на вещественной прямой функция, которая нигде не дифференцируема.
Доказательство. Положим
(34)
𝜑(𝑥) = |𝑥|
(−1 ≤ 𝑥 ≤ 1)
и доопределим 𝜑(𝑥) для всех вещественных 𝑥, полагая
(35)
𝜑(𝑥 + 2) = 𝜑(𝑥).
Тогда при любых 𝑠 и 𝑡
(36)
|𝜑(𝑠) − 𝜑(𝑡)| ≤ |𝑠 − 𝑡|.
В частности, 𝜑 непрерывна на 𝑅1 . Пусть
∞
(37)
𝑓 (𝑥) =
∑
𝑛=0
( 34 )𝑛 𝜑(4𝑛 𝑥).
Поскольку 0 ≤ 𝜑 ≤ 1, теорема 7.10 показывает, что ряд (37) сходится равномерно на 𝑅1 . По теореме
7.12 𝑓 непрерывна на 𝑅1 .
Зафиксируем теперь вещественное число 𝑥 и положительное целое 𝑚. Положим
𝛿𝑚 = ± 21 ⋅ 4−𝑚 ,
(38)
где знак выбран так, чтобы ни одного целого числа не лежало между 4𝑚 𝑥 и 4𝑚 (𝑥+𝛿𝑚 ). Это возможно,
так как 4𝑚 |𝛿𝑚 | = 12 . Положим
(39)
𝛾𝑛 =
𝜑(4𝑛 (𝑥 + 𝛿𝑚 )) − 𝜑(4𝑛 𝑥)
.
𝛿𝑚
Если 𝑛 > 𝑚, то 4𝑛 𝛿𝑚 — четное целое число, так что 𝛾𝑛 = 0. Если 0 ≤ 𝑛 ≤ 𝑚, то из (36) следует, что
|𝛾𝑛 | ≤ 4𝑛 .
Так как |𝛾𝑚 | = 4𝑚 , мы заключаем, что
𝑚
𝑓 (𝑥 + 𝛿𝑚 ) − 𝑓 (𝑥)
3 𝑛
=
𝛾
(
|
| |∑ 4 ) 𝑛 |
𝛿𝑚
𝑛=0
𝑚−1
≥ 3𝑚 −
∑
3𝑛
𝑛=0
=
1 𝑚
(3
2
+ 1).
Когда 𝑚 → ∞, 𝛿𝑚 → 0. Следовательно, 𝑓 не дифференцируема в 𝑥.
Равностепенно непрерывные семейства функций
В теореме 3.6 мы видели, что каждая ограниченная последовательность комплексных чисел содержит сходящуюся подпоследовательность. Возникает вопрос: верно ли что-либо подобное для
последовательностей функций? Чтобы уточнить этот вопрос, мы определим два вида ограниченности.
115
7.19 Определение. Пусть {𝑓𝑛 } — последовательность функций, определенных на множестве 𝐸.
Мы будем говорить, что {𝑓𝑛 } поточечно ограниченна на 𝐸, если последовательность {𝑓𝑛 (𝑥)}
ограничена при каждом 𝑥 ∈ 𝐸, т. е. если существует конечнозначная функция 𝜑, определенная на
𝐸, такая, что
|𝑓𝑛 (𝑥)| < 𝜑(𝑥)
(𝑥 ∈ 𝐸, 𝑛 = 1, 2, 3, … ).
Мы будем говорить, что {𝑓𝑛 } равномерно ограничена на 𝐸, если существует число 𝑀, такое, что
|𝑓𝑛 (𝑥)| < 𝑀
(𝑥 ∈ 𝐸, 𝑛 = 1, 2, 3, … ).
Если {𝑓𝑛 } поточечно ограничена на 𝐸, а 𝐸1 — счетное подмножество множества 𝐸, то всегда
можно найти подпоследовательность {𝑓𝑛𝑘 }, такую, что {𝑓𝑛𝑘 (𝑥)} сходится при каждом 𝑥 ∈ 𝐸1 . Это
можно сделать с помощью диагонального процесса, который используется ниже в доказательстве
теоремы 7.23.
Однако даже в том случае, когда {𝑓𝑛 } — равномерно ограниченная последовательность непрерывных функций на компактном множестве 𝐸, не обязательно существует подпоследовательность,
сходящаяся поточечно на 𝐸. В приводимом ниже примере было бы затруднительно доказать это с
помощью средств, которыми мы располагаем в настоящий момент, но доказательство оказывается
совсем простым, если сослаться на теорему из гл. 11.
7.20 Пример. Пусть
𝑓𝑛 (𝑥) = sin 𝑛𝑥
(0 ≤ 𝑥 ≤ 2𝜋, 𝑛 = 1, 2, 3, … ).
Допустим, что существует последовательность {𝑛𝑘 }, такая, что {sin 𝑛𝑘 𝑥} сходится при каждом 𝑥 ∈
[0, 2𝜋]. В этом случае мы должны получить
lim (sin 𝑛𝑘 𝑥 − sin 𝑛𝑘+1 𝑥) = 0
(0 ≤ 𝑥 ≤ 2𝜋);
lim (sin 𝑛𝑘 𝑥 − sin 𝑛𝑘+1 𝑥)2 = 0
(0 ≤ 𝑥 ≤ 2𝜋).
𝑘→∞
значит,
(40)
𝑘→∞
По теореме Лебега об интегрировании ограниченно сходящихся последовательностей (теорема 11.32),
из (40) следует, что
2𝜋
(41)
lim
𝑘→∞ ∫
0
(sin 𝑛𝑘 𝑥 − sin 𝑛𝑘+1 𝑥)2 𝑑𝑥 = 0.
Однако простое вычисление дает
2𝜋
∫
0
(sin 𝑛𝑘 𝑥 − sin 𝑛𝑘+1 𝑥)2 𝑑𝑥 = 2𝜋,
что противоречит равенству (41).
Другой вопрос таков: всякая ли сходящаяся последовательность содержит равномерно сходящуюся подпоследовательность? Следующий наш пример показывает, что это не обязательно имеет
место даже в том случае, когда последовательность равномерно ограничена на компактном множестве (пример 7.6 показывает, что последовательность ограниченных функций может быть сходящейся, не будучи равномерно ограниченной; но совершенно очевидно, что равномерная сходимость
последовательности ограниченных функций влечет за собой их равномерную ограниченность).
7.21 Пример. Пусть
𝑓𝑛 (𝑥) =
𝑥2
𝑥2 + (1 − 𝑛𝑥)2
(0 ≤ 𝑥 ≤ 1, 𝑛 = 1, 2, 3, … ).
Тогда |𝑓𝑛 (𝑥)| ≤ 1, так что последовательность {𝑓𝑛 } равномерно ограничена на [0, 1]. Кроме того,
lim 𝑓 (𝑥)
𝑛→∞ 𝑛
=0
но
(0 ≤ 𝑥 ≤ 1),
1
𝑓𝑛 ( ) = 1
(𝑛 = 1, 2, 3, … ),
𝑛
так что никакая подпоследовательность не сходится равномерно на [0, 1].
116
В этой связи нам понадобится понятие равностепенной непрерывности; оно дано в следующем
определении.
7.22 Определение. Семейство ℱ комплексных функций 𝑓 , определенных на множестве 𝐸 в метрическом пространстве 𝑋, называется равностепенно непрерывным на 𝐸, если для любого 𝜀 > 0
существует 𝛿 > 0, такое, что
|𝑓 (𝑥) − 𝑓 (𝑦)| < 𝜀,
когда 𝑑(𝑥, 𝑦) < 𝛿, 𝑥 ∈ 𝐸, 𝑦 ∈ 𝐸, а 𝑓 ∈ ℱ . Здесь 𝑑 обозначает расстояние в 𝑋.
Ясно, что каждая функция, входящая в состав равностепенно непрерывного семейства, равномерно непрерывна.
Последовательность в примере 7.21 не является равностепенно непрерывной.
Теоремы 7.24 и 7.25 покажут, что равностепенная непрерывность и равномерная сходимость
непрерывных функций тесно связаны друг с другом. Но вначале мы опишем процесс выбора, не
имеющий отношения к непрерывности.
7.23 Теорема. Если {𝑓𝑛 } — поточечно ограниченная последовательность комплексных функций
на счетном множестве 𝐸, то в {𝑓𝑛 } существует подпоследовательность {𝑓𝑛𝑘 }, такая, что
{𝑓𝑛𝑘 (𝑥)} сходится при каждом 𝑥 ∈ 𝐸.
Доказательство. Пусть {𝑥𝑖 }, 𝑖 = 1, 2, 3, … , — точки множества 𝐸, расположенные в последовательность. Ввиду того что последовательность {𝑓𝑛 (𝑥1 )} ограничена, существует подпоследовательность, которую мы обозначим через {𝑓1,𝑘 }, такая, что {𝑓1,𝑘 (𝑥1 )} сходится при 𝑘 → ∞.
Рассмотрим теперь последовательности 𝑆1 , 𝑆2 , 𝑆3 , … , представленные в виде таблицы
𝑆1 ∶ 𝑓1,1 𝑓1,2 𝑓1,3 𝑓1,4 ⋯
𝑆2 ∶ 𝑓2,1 𝑓2,2 𝑓2,3 𝑓2,4 ⋯
𝑆3 ∶ 𝑓3,1 𝑓3,2 𝑓3,3 𝑓3,4 ⋯
.................................
и обладающие следующими свойствами:
(a) 𝑆𝑛 — подпоследовательность последовательности 𝑆𝑛−1 при 𝑛 = 2, 3, 4, … ;
(b) {𝑓𝑛,𝑘 (𝑥𝑛 )} сходится при 𝑘 → ∞ (ограниченность последовательности {𝑓𝑛 (𝑥𝑛 )} позволяет выбрать 𝑆𝑛 с таким свойством);
(c) порядок, в котором выписываются функции, один и тот же во всех последовательностях, т.
е. если одна из функций предшествует другой в 𝑆1 , то они так же расположены во всех 𝑆𝑛
до тех пор, пока одна из этих функций не вычеркивается. Таким образом, передвигаясь из
какой-нибудь строки таблицы в следующую строку вниз, функции могут переместиться влево,
но никогда не перемещаются вправо.
Теперь мы спустимся по диагонали нашей таблицы, т. е. рассмотрим последовательность
𝑆1 ∶ 𝑓1,1
𝑓2,2
𝑓3,3
𝑓4,4
⋯ .
Согласно (c) последовательность 𝑆 (за исключением, возможно, первых 𝑛 − 1 членов) — подпоследовательность последовательности 𝑆𝑛 при 𝑛 = 1, 2, 3, … . Значит, из (b) следует, что {𝑓𝑛,𝑛 (𝑥𝑖 )}
сходится при 𝑛 → ∞ при каждом 𝑥𝑖 ∈ 𝐸.
7.24 Теорема. Пусть 𝐾 — компактное метрическое пространство. Если 𝑓𝑛 ∈ 𝒞 (𝐾) при 𝑛 =
1, 2, 3, … и последовательность {𝑓𝑛 } равномерно сходится на 𝐾, то {𝑓𝑛 } равностепенно непрерывна на 𝐾.
Доказательство. Пусть дано 𝜀 > 0. Так как {𝑓𝑛 } сходится равномерно, существует целое число
𝑁, такое, что
(42)
‖𝑓𝑛 − 𝑓𝑁 ‖ < 𝜀
(𝑛 > 𝑁)
(см. определение 7.14). Так как непрерывные функции равномерно непрерывны на компактных
множествах, существует 𝛿 > 0, такое, что
(43)
|𝑓𝑖 (𝑥) − 𝑓𝑖 (𝑦)| < 𝜀,
117
если 1 ≤ 𝑖 ≤ 𝑁 и 𝑑(𝑥, 𝑦) < 𝛿.
Если 𝑛 > 𝑁 и 𝑑(𝑥, 𝑦) < 𝛿, то
|𝑓𝑛 (𝑥) − 𝑓𝑛 (𝑦)| ≤ |𝑓𝑛 (𝑥) − 𝑓𝑁 (𝑥)| + |𝑓𝑁 (𝑥) − 𝑓𝑁 (𝑦)| + |𝑓𝑁 (𝑦) − 𝑓𝑛 (𝑦)| < 3𝜀.
С учетом (43) теорема доказана.
7.25 Теорема. Пусть 𝐾 компактно. Если 𝑓𝑛 ∈ 𝒞 (𝐾) при 𝑛 = 1, 2, 3, … и последовательность
{𝑓𝑛 } поточечно ограничена и равностепенно непрерывна на 𝐾, то
(a) {𝑓𝑛 } равномерно ограничена на 𝐾;
(b) {𝑓𝑛 } содержит равномерно сходящуюся подпоследовательность.
Доказательство.
(a) Пусть дано 𝜀 > 0. Выберем 𝛿 > 0 в соответствии с определением 7.22, так, что
(44)
|𝑓𝑛 (𝑥) − 𝑓𝑛 (𝑦)| < 𝜀
при всех 𝑛, если 𝑑(𝑥, 𝑦) < 𝛿.
Ввиду того что 𝐾 компактно, в нем существует конечный набор точек 𝑝1 , … , 𝑝𝑟 , такой, что
каждому 𝑥 ∈ 𝐾 соответствует хотя бы одно 𝑝𝑖 , такое, что 𝑑(𝑥, 𝑝𝑖 ) < 𝛿. Так как {𝑓𝑛 } поточечно ограничена, существует 𝑀𝑖 < ∞, такое, что |𝑓𝑛 (𝑝𝑖 )| < 𝑀𝑖 при всех 𝑛. Если 𝑀 =
max(𝑀1 , … , 𝑀𝑟 ), то |𝑓𝑛 (𝑥)| < 𝑀 + 𝜀 для каждого 𝑥 ∈ 𝐾. Утверждение (a) доказано.
(b) Пусть 𝐸 — счетное всюду плотное подмножество пространства 𝐾 (существование такого множества 𝐸 доказывается в упражнении 25 гл. 2). Теорема 7.23 показывает, что {𝑓𝑛 } имеет
подпоследовательность {𝑓𝑛𝑖 }, такую, что {𝑓𝑛𝑖 (𝑥)} сходится при каждом 𝑥 ∈ 𝐸.
Чтобы упростить обозначения, положим 𝑓𝑛𝑖 = 𝑔𝑖 . Мы докажем, что {𝑔𝑖 } равномерно сходится
на 𝐾.
Пусть 𝜀 > 0. Выберем 𝛿 > 0, как в начале доказательства. Пусть 𝑉 (𝑥, 𝛿) — множество всех
𝑦 ∈ 𝐾, таких, что 𝑑(𝑥, 𝑦) < 𝛿. Ввиду того что 𝐸 всюду плотно в 𝐾 и 𝐾 компактно, существует
конечный набор точек 𝑥1 , … , 𝑥𝑚 в 𝐸, такой, что
(45)
𝐾 ⊂ 𝑉 (𝑥1 , 𝛿) ∪ ⋯ ∪ 𝑉 (𝑥𝑚 , 𝛿).
Ввиду того что {𝑔𝑖 (𝑥)} сходится при любом 𝑥 ∈ 𝐸, существует целое число 𝑁, такое, что
(46)
|𝑔𝑖 (𝑥𝑠 ) − 𝑔𝑗 (𝑥𝑠 )| < 𝜀,
если 𝑖 ≥ 𝑁, 𝑗 ≥ 𝑁, 1 ≤ 𝑠 ≤ 𝑚.
Если 𝑥 ∈ 𝐾, то (45) показывает, что 𝑥 ∈ 𝑉 (𝑥𝑠 , 𝛿) при некотором 𝑠, так что
|𝑔𝑖 (𝑥) − 𝑔𝑖 (𝑥𝑠 )| < 𝜀
при каждом 𝑖. Если 𝑖 ≥ 𝑁 и 𝑗 ≥ 𝑁, из (46) следует, что
|𝑔𝑖 (𝑥) − 𝑔𝑗 (𝑥)| ≤ |𝑔𝑖 (𝑥) − 𝑔𝑖 (𝑥𝑠 )| + |𝑔𝑖 (𝑥𝑠 ) − 𝑔𝑗 (𝑥𝑠 )| + |𝑔𝑗 (𝑥𝑠 ) − 𝑔𝑗 (𝑥)|
< 3𝜀.
Теорема доказана.
Теорема Стоуна-Вейерштрасса
7.26 Теорема. Если 𝑓 — непрерывная комплексная функция на [𝑎, 𝑏], то существует такая последовательность многочленов 𝑃𝑛 , что
lim 𝑃𝑛 (𝑥) = 𝑓 (𝑥)
𝑛→∞
равномерно на [𝑎, 𝑏]. Если 𝑓 вещественна, то 𝑃𝑛 можно выбрать вещественными.
118
Именно в такой форме эта теорема была первоначально открыта Вейерштрассом.
Доказательство. Не ограничивая общности, мы можем считать, что [𝑎, 𝑏] = [0, 1]. Будем считать
также, что 𝑓 (0) = 𝑓 (1) = 0. Действительно, если теорема доказана для этого случая, то рассмотрим
𝑔(𝑥) = 𝑓 (𝑥) − 𝑓 (0) − 𝑥[𝑓 (1) − 𝑓 (0)]
(0 ≤ 𝑥 ≤ 1).
Здесь 𝑔(0) = 𝑔(1) = 0, и если 𝑔 представима в виде предела равномерно сходящейся последовательности многочленов, то очевидно, что такова и 𝑓 , так как 𝑓 − 𝑔 — многочлен.
Кроме того, будем считать, что 𝑓 (𝑥) равна нулю при 𝑥 вне сегмента [0, 1]. Тогда 𝑓 равномерно
непрерывна на всей вещественной прямой.
Положим
𝑄𝑛 (𝑥) = 𝑐𝑛 (1 − 𝑥2 )𝑛
(47)
(𝑛 = 1, 2, 3, … ),
где 𝑐𝑛 выбраны так, что
1
(48)
∫
−1
𝑄𝑛 (𝑥) 𝑑𝑥 = 1
(𝑛 = 1, 2, 3, … ).
Нам нужны некоторые сведения о порядке величины 𝑐𝑛 . Ввиду того что
1
∫
−1
(1 − 𝑥2 )𝑛 𝑑𝑥 = 2
1
∫
0
(1 − 𝑥2 )𝑛 𝑑𝑥 ≥ 2
1/√𝑛
∫
0
1/√𝑛
≥2
=
∫
0
4
(1 − 𝑥2 )𝑛 𝑑𝑥
(1 − 𝑛𝑥2 ) 𝑑𝑥
3√𝑛
1
>
,
√𝑛
из (48) следует, что
(49)
𝑐𝑛 < √𝑛.
Неравенство (1 − 𝑥2 )𝑛 ≥ 1 − 𝑛𝑥2 , которым мы воспользовались выше, легко проверяется. Для
этого нужно рассмотреть функцию
(1 − 𝑥2 )𝑛 − 1 + 𝑛𝑥2 ,
которая равна нулю при 𝑥 = 0 и имеет положительную производную на (0, 1).
При любом 𝛿 > 0 из (49) следует, что
(50)
𝑄𝑛 (𝑥) ≤ √𝑛(1 − 𝛿 2 )𝑛
(𝛿 ≤ |𝑥| ≤ 1),
так что 𝑄𝑛 → 0 равномерно при 𝛿 ≤ |𝑥| ≤ 1.
Положим теперь
1
(51)
𝑃𝑛 (𝑥) =
∫
−1
𝑓 (𝑥 + 𝑡)𝑄𝑛 (𝑡) 𝑑𝑡
(0 ≤ 𝑥 ≤ 1).
Наши предположения о функции 𝑓 показывают с помощью простой замены переменной, что
1−𝑥
𝑃𝑛 (𝑥) =
∫
−𝑥
1
𝑓 (𝑥 + 𝑡)𝑄𝑛 (𝑡) 𝑑𝑡 =
∫
0
𝑓 (𝑡)𝑄𝑛 (𝑡 − 𝑥) 𝑑𝑡,
а последний интеграл, очевидно, есть многочлен от 𝑥. Таким образом, {𝑃𝑛 } — последовательность
многочленов, причем вещественных, если 𝑓 вещественна.
119
Пусть дано 𝜀 > 0. Выберем 𝛿 > 0 так, чтобы из |𝑦 − 𝑥| < 𝛿 следовало
|𝑓 (𝑦) − 𝑓 (𝑥)| <
𝜀
.
2
Пусть 𝑀 = sup |𝑓 (𝑥)|. Используя (48), (50) и тот факт, что 𝑄𝑛 (𝑥) ≥ 0, мы видим, что при 0 ≤ 𝑥 ≤ 1
1
|𝑃𝑛 (𝑥) − 𝑓 (𝑥)| =
|∫
−1
[𝑓 (𝑥 + 𝑡) − 𝑓 (𝑥)]𝑄𝑛 (𝑡) 𝑑𝑡
|
1
≤
∫
−1
|𝑓 (𝑥 + 𝑡) − 𝑓 (𝑥)|𝑄𝑛 (𝑡) 𝑑𝑡
−𝛿
≤ 2𝑀
∫
−1
𝛿
𝑄𝑛 (𝑡) 𝑑𝑡 +
≤ 4𝑀√𝑛(1 − 𝛿 2 )𝑛 +
𝜀
2
1
𝜀
𝑄𝑛 (𝑡) 𝑑𝑡 + 2𝑀
𝑄𝑛 (𝑡) 𝑑𝑡
∫
2∫
−𝛿
𝛿
<𝜀
при всех достаточно больших 𝑛. Теорема доказана.
Поучительно набросать графики функций 𝑄𝑛 для нескольких значений 𝑛; отметим еще, что для
доказательства равномерной сходимости последовательности {𝑃𝑛 } нам потребовалась равномерная
непрерывность функции 𝑓 .
При доказательстве теоремы 7.32 нам не потребуется теорема 7.26 в полном объеме, а потребуется только один ее частный случай, который мы сформулируем в виде следствия.
7.27 Следствие. Для любого сегмента [−𝑎, 𝑎] существует последовательность вещественных
многочленов 𝑃𝑛 , такая, что 𝑃𝑛 (0) = 0 и
lim 𝑃𝑛 (𝑥) = |𝑥|
𝑛→∞
равномерно на [−𝑎, 𝑎].
Доказательство. По теореме 7.26 существует последовательность {𝑃𝑛∗ } вещественных многочленов, сходящаяся к |𝑥| равномерно на [−𝑎, 𝑎]. В частности, 𝑃𝑛∗ (0) → 0 при 𝑛 → ∞. Многочлены
𝑃𝑛 (𝑥) = 𝑃𝑛∗ (𝑥) − 𝑃𝑛∗ (0)
(𝑛 = 1, 2, 3, … )
обладают нужными свойствами.
Теперь мы выделим те свойства многочленов, на которых основана теорема Вейерштрасса.
7.28 Определение. Семейство 𝒜 комплексных функций, определенных на множестве 𝐸, называется алгеброй, если (i) 𝑓 + 𝑔 ∈ 𝒜 , (ii) 𝑓 𝑔 ∈ 𝒜 и (iii) 𝑐𝑓 ∈ 𝒜 при всех 𝑓 ∈ 𝒜 , 𝑔 ∈ 𝒜 и всех
комплексных постоянных 𝑐, иными словами, если 𝒜 замкнуто относительно сложения, умножения
и умножения на скаляры. Мы будем рассматривать также алгебры вещественных функций; в этом
случае в (iii) речь идет, конечно, об умножении лишь на вещественные 𝑐.
Если множество 𝒜 обладает тем свойством, что 𝑓 ∈ 𝒜 , когда 𝑓𝑛 ∈ 𝒜 (𝑛 = 1, 2, 3, … ) и 𝑓𝑛 → 𝑓
равномерно на 𝐸, то 𝒜 называется равномерно замкнутым.
Пусть ℬ — множество всех функций, которые служат пределами равномерно сходящихся последовательностей элементов множества 𝒜 . Тогда ℬ называется равномерным замыканием множества
𝒜 (см. определение 7.14).
Например, множество всех многочленов — алгебра, и теорему Вейерштрасса можно сформулировать так: множество непрерывных функций на [𝑎, 𝑏] есть равномерное замыкание множества
многочленов на [𝑎, 𝑏].
7.29 Теорема. Пусть ℬ — равномерное замыкание алгебры 𝒜 , состоящей из ограниченных функций. Тогда ℬ — равномерно замкнутая алгебра.
120
Доказательство. Если 𝑓 ∈ ℬ и 𝑔 ∈ ℬ, то существуют равномерно сходящиеся последовательности {𝑓𝑛 }, {𝑔𝑛 }, такие, что 𝑓𝑛 → 𝑓 , 𝑔𝑛 → 𝑔 и 𝑓𝑛 ∈ 𝒜 , 𝑔𝑛 ∈ 𝒜 . Пользуясь тем, что мы имеем дело с
ограниченными функциями, легко показать, что
𝑓𝑛 + 𝑔𝑛 → 𝑓 + 𝑔,
𝑓𝑛 𝑔𝑛 → 𝑓 𝑔,
𝑐𝑓𝑛 → 𝑐𝑓 ,
где 𝑐 — любая постоянная, причем сходимость в каждом случае равномерна.
Значит, 𝑓 + 𝑔 ∈ ℬ, 𝑓 𝑔 ∈ ℬ и 𝑐𝑓 ∈ ℬ, так что ℬ — алгебра.
По теореме 2.27 ℬ (равномерно) замкнута.
7.30 Определение. Пусть 𝒜 — семейство функций на множестве 𝐸. Тогда говорят, что 𝒜 разделяет точки множества 𝐸, если для каждой пары различных точек 𝑥1 , 𝑥2 ∈ 𝐸 найдется функция
𝑓 ∈ 𝒜 , такая, что 𝑓 (𝑥1 ) ≠ 𝑓 (𝑥2 ).
Если для каждой точки 𝑥 ∈ 𝐸 найдется функция 𝑔 ∈ 𝒜 , такая, что 𝑔(𝑥) ≠ 0, то мы будем
говорить, что 𝒜 не исчезает ни в одной точке множества 𝐸.
Алгебра всех многочленов от одной переменной, очевидно, обладает этими свойствами на 𝑅1 .
Примером алгебры, не разделяющей точек, служит множество всех четных многочленов, рассматриваемых, скажем, на [−1, 1], так как 𝑓 (−𝑥) = 𝑓 (𝑥) для каждой функции 𝑓 .
Следующая теорема иллюстрирует эти понятия.
7.31 Теорема. Пусть 𝒜 — алгебра функций на множестве 𝐸, 𝒜 разделяет точки множества
𝐸 и не исчезает ни в одной точке множества 𝐸. Пусть 𝑥1 , 𝑥2 — различные точки множества
𝐸, а 𝑐1 , 𝑐2 — постоянные (вещественные, если 𝒜 — вещественная алгебра). Тогда 𝒜 содержит
функцию 𝑓 , такую, что
𝑓 (𝑥1 ) = 𝑐1 ,
𝑓 (𝑥2 ) = 𝑐2 .
Доказательство. Наши предположения показывают, что 𝒜 содержит функции 𝑔, ℎ и 𝑘, такие,
что
𝑔(𝑥1 ) ≠ 𝑔(𝑥2 ),
ℎ(𝑥1 ) ≠ 0,
𝑘(𝑥2 ) ≠ 0.
Положим
𝑢 = 𝑔𝑘 − 𝑔(𝑥1 )𝑘,
𝑣 = 𝑔ℎ − 𝑔(𝑥2 )ℎ.
Тогда 𝑢 ∈ 𝒜 , 𝑣 ∈ 𝒜 , 𝑢(𝑥1 ) = 𝑣(𝑥2 ) = 0, 𝑢(𝑥2 ) ≠ 0 и 𝑣(𝑥1 ) ≠ 0. Следовательно, функция
𝑓=
𝑐 𝑢
𝑐1 𝑣
+ 2
𝑣(𝑥1 ) 𝑢(𝑥2 )
обладает нужными свойствами.
Теперь мы докажем теорему Стоуна, обобщающую теорему Вейерштрасса.
7.32 Теорема. Пусть 𝒜 — алгебра вещественных непрерывных функций на компактном множестве 𝐾. Если 𝒜 разделяет точки множества 𝐾 и не исчезает ни в одной точке множества 𝐾,
то равномерное замыкание ℬ алгебры 𝒜 состоит из всех вещественных непрерывных функций
на 𝐾.
Мы разобьем доказательство на четыре шага.
Шаг 1. Если 𝑓 ∈ ℬ, то |𝑓 | ∈ ℬ.
Доказательство. Пусть
(52)
𝑎 = sup |𝑓 (𝑥)|
(𝑥 ∈ 𝐾),
и пусть задано 𝜀 > 0. Согласно следствию 7.27, существуют вещественные числа 𝑐1 , … , 𝑐𝑛 , такие,
что
𝑛
(53)
|∑
𝑖=1
𝑐𝑖 𝑦𝑖 − |𝑦| < 𝜀
|
121
(−𝑎 ≤ 𝑦 ≤ 𝑎).
Так как ℬ — алгебра, функция
𝑛
𝑔=
∑
𝑐𝑖 𝑓 𝑖
𝑖=1
входит в состав множества ℬ. Согласно (52) и (53), мы имеем
|𝑔(𝑥) − |𝑓 (𝑥)|| < 𝜀
(𝑥 ∈ 𝐾).
Поскольку алгебра ℬ равномерно замкнута, то отсюда следует, что |𝑓 | ∈ ℬ.
Шаг 2. Если 𝑓 ∈ ℬ и 𝑔 ∈ ℬ, то max(𝑓 , 𝑔) ∈ ℬ и min(𝑓 , 𝑔) ∈ ℬ.
При этом ℎ = max(𝑓 , 𝑔) означает, что
ℎ(𝑥) =
𝑓 (𝑥),
{𝑔(𝑥),
если 𝑓 (𝑥) ≥ 𝑔(𝑥),
если 𝑓 (𝑥) < 𝑔(𝑥),
min(𝑓 , 𝑔) определяется аналогично.
Доказательство. Шаг 2 следует из шага 1 и тождеств
𝑓 + 𝑔 |𝑓 − 𝑔|
+
,
2
2
𝑓 + 𝑔 |𝑓 − 𝑔|
min(𝑓 , 𝑔) =
−
.
2
2
max(𝑓 , 𝑔) =
Разумеется, этот результат можно по индукции распространить на любое конечное множество
функций: если 𝑓1 , … , 𝑓𝑛 ∈ ℬ, то max(𝑓1 , … , 𝑓𝑛 ) ∈ ℬ и min(𝑓1 , … , 𝑓𝑛 ) ∈ ℬ.
Шаг 3. Пусть заданы вещественная функция 𝑓 , непрерывная на 𝐾, точка 𝑥 ∈ 𝐾 и 𝜀 > 0. Тогда
найдется функция 𝑔𝑥 ∈ ℬ, такая, что 𝑔𝑥 (𝑥) = 𝑓 (𝑥) и
(54)
𝑔𝑥 (𝑡) > 𝑓 (𝑡) − 𝜀
(𝑡 ∈ 𝐾).
Доказательство. Поскольку 𝒜 ⊂ ℬ и 𝒜 удовлетворяет условиям теоремы 7.31, то и ℬ удовлетворяет этим условиям. Значит, для любого 𝑦 ∈ 𝐾 можно найти функцию ℎ𝑦 ∈ ℬ, такую, что
(55)
ℎ𝑦 (𝑥) = 𝑓 (𝑥),
ℎ𝑦 (𝑦) = 𝑓 (𝑦).
В силу непрерывности функции ℎ𝑦 , существует открытое множество 𝐽𝑦 , содержащее 𝑦, такое,
что
(56)
ℎ𝑦 (𝑡) > 𝑓 (𝑡) − 𝜀
(𝑡 ∈ 𝐽𝑦 ).
Ввиду того, что 𝐾 компактно, имеется конечное множество точек 𝑦1 , … , 𝑦𝑛 , таких, что
(57)
𝐾 ⊂ 𝐽𝑦1 ∪ ⋯ ∪ 𝐽𝑦𝑛 .
Положим
𝑔𝑥 = max(ℎ𝑦1 , … , ℎ𝑦𝑛 ).
Согласно установленному на шаге 2, 𝑔𝑥 ∈ ℬ, а из соотношений (55)-(57) следует, что 𝑔𝑥 обладает
остальными нужными свойствами.
Шаг 4. Пусть заданы вещественная функция 𝑓 , непрерывная на 𝐾, и 𝜀 > 0. Существует функция
ℎ ∈ ℬ, такая, что
(58)
|ℎ(𝑥) − 𝑓 (𝑥)| < 𝜀
(𝑥 ∈ 𝐾).
Это утверждение равносильно утверждению теоремы, так как ℬ равномерно замкнута.
122
Доказательство. Рассмотрим функции 𝑔𝑥 для каждого 𝑥 ∈ 𝐾, построенные на шаге 3. В силу
непрерывности функций 𝑔𝑥 , существуют открытые множества 𝑉𝑥 , содержащие 𝑥, такие, что
(59)
𝑔𝑥 (𝑡) < 𝑓 (𝑡) + 𝜀
(𝑡 ∈ 𝑉𝑥 ).
Ввиду того что 𝐾 компактно, существует конечное множество точек 𝑥1 , … , 𝑥𝑚 , таких, что
(60)
𝐾 ⊂ 𝑉𝑥1 ∪ ⋯ ∪ 𝑉𝑥𝑚 .
Положим
ℎ = min(𝑔𝑥1 , … , 𝑔𝑥𝑚 ).
Как было установлено на шаге 2, ℎ ∈ ℬ, и из (54) следует, что
(61)
ℎ(𝑡) > 𝑓 (𝑡) − 𝜀
(𝑡 ∈ 𝐾),
ℎ(𝑡) < 𝑓 (𝑡) + 𝜀
(𝑡 ∈ 𝐾).
тогда как из (59) и (60) следует, что
(62)
Наконец, (58) вытекает из (61) и (62).
Теорема 7.32 не выполняется для комплексных алгебр. Контрпример дан в упражнении 21.
Однако утверждение теоремы остается верным даже для комплексных алгебр, если на 𝒜 наложить
еще одно условие, а именно если потребовать, чтобы 𝒜 была самосопряженной. Это означает, что
для любой функции 𝑓 ∈ 𝒜 комплексно сопряженная с ней функция 𝑓 ̄ тоже должна принадлежать
̄ = 𝑓 (𝑥).
𝒜 ; 𝑓 ̄ определяется как 𝑓 (𝑥)
7.33 Теорема. Пусть 𝒜 — самосопряженная алгебра комплексных непрерывных функций на компактном множестве 𝐾, 𝒜 разделяет точки множества 𝐾 и не исчезает ни в одной точке множества 𝐾. Тогда равномерное замыкание ℬ алгебры 𝒜 состоит из всех комплексных непрерывных
функций на 𝐾. Иными словами, 𝒜 всюду плотна в 𝒞 (𝐾).
Доказательство. Пусть 𝒜𝑅 — множество всех вещественных функций на 𝐾, принадлежащих 𝒜 .
Если 𝑓 ∈ 𝒜 и 𝑓 = 𝑢 + 𝑖𝑣, где 𝑢, 𝑣 вещественны, то 2𝑢 = 𝑓 + 𝑓 ,̄ а так как 𝒜 самосопряжена, то 𝑢 ∈
𝒜𝑅 . Если 𝑥1 ≠ 𝑥2 , то существует 𝑓 ∈ 𝒜 , такая, что 𝑓 (𝑥1 ) = 1, 𝑓 (𝑥2 ) = 0; значит, 0 = 𝑢(𝑥2 ) ≠ 𝑢(𝑥1 ) = 1,
откуда следует, что 𝒜𝑅 разделяет точки множества 𝐾. Если 𝑥 ∈ 𝐾, то 𝑔(𝑥) ≠ 0 при некотором 𝑔 ∈ 𝒜
и имеется комплексное число 𝜆, такое, что 𝜆𝑔(𝑥) > 0; если 𝑓 = 𝜆𝑔, 𝑓 = 𝑢 + 𝑖𝑣, то отсюда следует, что
𝑢(𝑥) > 0; значит, 𝒜𝑅 не исчезает ни в одной точке множества 𝐾.
Таким образом, 𝒜𝑅 удовлетворяет условиям теоремы 7.32. Следовательно, каждая вещественная
непрерывная функция на 𝐾 принадлежит равномерному замыканию алгебры 𝒜𝑅 , т. е. принадлежит
ℬ. Если 𝑓 — комплексная непрерывная функция на 𝐾, 𝑓 = 𝑢 + 𝑖𝑣, то 𝑢 ∈ ℬ, 𝑣 ∈ ℬ, значит, 𝑓 ∈ ℬ.
Доказательство закончено.
Упражнения
1. Доказать, что всякая равномерно сходящаяся последовательность ограниченных функций равномерно
ограничена.
2. Пусть последовательности {𝑓𝑛 } и {𝑔𝑛 } сходятся равномерно на множестве 𝐸. Доказать, что {𝑓𝑛 + 𝑔𝑛 }
сходится равномерно на 𝐸. Пусть, кроме того, {𝑓𝑛 } и {𝑔𝑛 } — последовательности ограниченных функций.
Доказать, что {𝑓𝑛 𝑔𝑛 } сходится равномерно на 𝐸.
3. Построить последовательности {𝑓𝑛 }, {𝑔𝑛 }, которые сходятся равномерно на некотором множестве 𝐸, но
{𝑓𝑛 𝑔𝑛 } не сходится равномерно на 𝐸 (разумеется, {𝑓𝑛 𝑔𝑛 } сходится на 𝐸).
4. Рассмотрим
∞
1
𝑓 (𝑥) =
.
∑ 1 + 𝑛2 𝑥
𝑛=1
При каких значениях 𝑥 этот ряд сходится абсолютно? На каких сегментах он сходится равномерно? На
каких сегментах он перестает быть равномерно сходящимся? Непрерывна ли 𝑓 в тех точках, где ряд
сходится? Ограничена ли 𝑓 ?
123
5. Пусть
1
⎧
(𝑥 < 𝑛+1 ) ,
⎪0
⎪
1
𝑓𝑛 (𝑥) = ⎨sin2 𝜋𝑥 ( 𝑛+1
≤ 𝑥 ≤ 1𝑛 ) ,
⎪
1
⎪
( 𝑛 < 𝑥) .
⎩0
Показать, что {𝑓𝑛 } сходится к непрерывной функции, но неравномерно. Воспользоваться рядом ∑ 𝑓𝑛
для доказательства того, что абсолютная сходимость, даже при всех 𝑥, не влечет за собой равномерной
сходимости.
6. Доказать, что ряд
∞
𝑥2 + 𝑛
(−1)𝑛
∑
𝑛2
𝑛=1
сходится равномерно на каждом ограниченном сегменте, но не сходится абсолютно ни при одном значении 𝑥.
7. Для 𝑛 = 1, 2, 3, … и вещественного 𝑥 положим
𝑥
𝑓𝑛 (𝑥) =
.
1 + 𝑛𝑥2
Показать, что {𝑓𝑛 } сходится равномерно к некоторой функции 𝑓 и что равенство
𝑓 ′ (𝑥) = lim 𝑓𝑛′ (𝑥)
𝑛→∞
верно, если 𝑥 ≠ 0, но неверно, если 𝑥 = 0.
8. Пусть
𝐼(𝑥) =
0 (𝑥 ≤ 0),
{1 (𝑥 > 0).
Пусть {𝑥𝑛 } — последовательность различных точек интервала (𝑎, 𝑏), и пусть ряд ∑ |𝑐𝑛 | сходится. Доказать, что ряд
∞
𝑓 (𝑥) =
∑
𝑐𝑛 𝐼(𝑥 − 𝑥𝑛 )
(𝑎 ≤ 𝑥 ≤ 𝑏)
𝑛=1
сходится равномерно и что 𝑓 непрерывна при любом 𝑥 ≠ 𝑥𝑛 .
9. Пусть {𝑓𝑛 } — последовательность непрерывных функций, сходящаяся равномерно к функции 𝑓 на множестве 𝐸. Доказать, что
lim 𝑓𝑛 (𝑥𝑛 ) = 𝑓 (𝑥)
𝑛→∞
для каждой последовательности точек 𝑥𝑛 ∈ 𝐸, такой, что 𝑥𝑛 → 𝑥 и 𝑥 ∈ 𝐸. Верно ли обратное?
10. Пусть (𝑥) обозначает дробную часть вещественного числа 𝑥 (см. определение в упражнении 16 гл. 4).
Рассмотрим функцию
∞
(𝑛𝑥)
(𝑥 вещественно).
𝑓 (𝑥) =
∑ 𝑛2
𝑛=1
Найти все точки разрыва функции 𝑓 и показать, что они образуют счетное всюду плотное множество.
Показать, что 𝑓 тем не менее интегрируема по Риману на каждом ограниченном сегменте.
11. Пусть {𝑓𝑛 }, {𝑔𝑛 } — последовательности функций, определенных на 𝐸, и
(a) частные суммы ряда ∑ 𝑓𝑛 равномерно ограничены;
(b) 𝑔𝑛 → 0 равномерно на 𝐸;
(c) 𝑔1 (𝑥) ≥ 𝑔2 (𝑥) ≥ 𝑔3 (𝑥) ≥ ⋯ при любом 𝑥 ∈ 𝐸.
Доказать, что ряд ∑ 𝑓𝑛 𝑔𝑛 сходится равномерно на 𝐸. Указание. Сравнить с теоремой 3.42.
12. Пусть 𝑔 и 𝑓𝑛 (𝑛 = 1, 2, 3, … ) определены на (0, ∞), интегрируемы по Риману на [𝑡, 𝑇 ], если 0 < 𝑡 < 𝑇 < ∞,
|𝑓𝑛 | ≤ 𝑔, 𝑓𝑛 → 𝑓 равномерно на каждом компактном подмножестве интервала (0, ∞) и
∞
∫
0
Доказать, что
𝑔(𝑥) 𝑑𝑥 < ∞.
∞
lim
∞
𝑓 (𝑥) 𝑑𝑥
∫
0
(соответствующие определения см. в упражнениях 7 и 8 гл. 6).
Это довольно слабая форма теоремы Лебега о мажорируемой сходимости (теорема 11.32). Даже в случае интеграла Римана равномерная сходимость может быть заменена поточечной сходимостью, если
предположить, что 𝑓 ∈ ℛ. (См. статьи F. Cunningham в Math. Mag., vol. 40, 1967, pp. 179-186 и H.
Kestelman в Amer. Math. Monthly, vol. 77, 1970, pp. 182-187.)
𝑛→∞ ∫
0
𝑓𝑛 (𝑥) 𝑑𝑥 =
124
13. Пусть {𝑓𝑛 } — последовательность монотонно возрастающих функций на 𝑅1 , причем 0 ≤ 𝑓𝑛 (𝑥) ≤ 1 при
всех 𝑥 и всех 𝑛.
(a) Доказать, что существуют функция 𝑓 и последовательность {𝑛𝑘 }, такие, что
𝑓 (𝑥) = lim 𝑓𝑛𝑘 (𝑥)
𝑘→∞
при всех 𝑥 ∈ 𝑅1 (утверждение о существовании такой поточечно сходящейся последовательности
обычно называется теоремой выбора Хелли).
(b) Если, кроме того, 𝑓 непрерывна, доказать, что 𝑓𝑛𝑘 → 𝑓 равномерно на компактных множествах.
Указание. (i) Некоторая подпоследовательность {𝑓𝑛𝑖 } сходится во всех рациональных точках 𝑟, обозначим этот предел 𝑓 (𝑟). (ii) Определим 𝑓 (𝑥) при всех точках 𝑥 ∈ 𝑅1 равным sup 𝑓 (𝑟), где верхняя грань
берется по всем 𝑟 ≤ 𝑥. (iii) Показать, что 𝑓𝑛𝑖 (𝑥) → 𝑓 (𝑥) при каждом 𝑥, в котором 𝑓 непрерывна (здесь
существенным образом используется монотонность). (iv) Подпоследовательность последовательности
{𝑓𝑛𝑖 } сходится в каждой точке разрыва функции 𝑓 , так как множество таких точек не более чем счетно. Утверждение (a) доказано. Чтобы доказать (b), измените доказательство части (iii) подходящим
образом.
14. Пусть 𝑓 — непрерывная вещественная функция на 𝑅1 со следующими свойствами: 0 ≤ 𝑓 (𝑡) ≤ 1, 𝑓 (𝑡+2) =
𝑓 (𝑡) при любом 𝑡 и
0 (0 ≤ 𝑡 ≤ 13 ,
𝑓 (𝑡) =
{1 ( 23 ≤ 𝑡 ≤ 1).
Положим Φ(𝑡) = (𝑥(𝑡), 𝑦(𝑡)), где
∞
𝑥(𝑡) =
∑
∞
2−𝑛 𝑓 (32𝑛−1 𝑡),
𝑦(𝑡) =
𝑛=1
∑
2−𝑛 𝑓 (32𝑛 𝑡).
𝑛=1
Доказать, что Φ непрерывна и отображает 𝐼 = [0, 1] на единичный квадрат 𝐼 2 ⊂ 𝑅2 . В действительности
показать, что Φ отображает множество Кантора на 𝐼 2 .
Указание. Каждая точка (𝑥0 , 𝑦0 ) ∈ 𝐼 2 имеет вид
∞
𝑥0 =
∑
∞
2−𝑛 𝑎2𝑛−1 ,
𝑦0 =
𝑛=1
∑
2−𝑛 𝑎2𝑛 ,
𝑛=1
где каждое 𝑎𝑖 равно 0 или 1. Если
∞
𝑡0 =
∑
3−𝑖−1 (2𝑎𝑖 ),
𝑖=1
15.
16.
17.
18.
показать, что 𝑓 (3𝑘 𝑡0 ) = 𝑎𝑘 , следовательно, 𝑥(𝑡0 ) = 𝑥0 , 𝑦(𝑡0 ) = 𝑦0 .
(Этот простой пример «кривой, заполняющей пространство» принадлежит I. J. Schoenberg, Bull. A.M.S.,
vol. 44, 1938, pp. 519.)
Пусть 𝑓 — вещественная непрерывная функция на 𝑅1 , 𝑓𝑛 (𝑡) = 𝑓 (𝑛𝑡) при 𝑛 = 1, 2, 3, … и последовательность {𝑓𝑛 } равностепенно непрерывна на [0, 1]. Какие выводы можно сделать о 𝑓 ?
Пусть {𝑓𝑛 } — равностепенно непрерывная последовательность функций на компактном множестве 𝐾 и
{𝑓𝑛 } сходится поточечно на 𝐾. Доказать, что {𝑓𝑛 } сходится равномерно на 𝐾.
Определить понятия равномерной сходимости и равностепенной непрерывности для отображений в любое метрическое пространство. Показать, что теоремы 7.9 и 7.12 верны для отображений в любое метрическое пространство, теоремы 7.8 и 7.11 верны для отображений в любое полное метрическое пространство и теоремы 7.10, 7.16, 7.17, 7.24 и 7.25 верны для векторнозначных функций, т. е. для отображений
в любое 𝑅𝑘 .
Пусть {𝑓𝑛 } — равномерно ограниченная последовательность функций, интегрируемых по Риману на
[𝑎, 𝑏]. Положим
𝑥
𝐹𝑛 (𝑥) =
∫
𝑎
𝑓𝑛 (𝑡) 𝑑𝑡
(𝑎 ≤ 𝑥 ≤ 𝑏).
Доказать, что существует подпоследовательность {𝐹𝑛𝑘 }, равномерно сходящаяся на [𝑎, 𝑏].
19. Пусть 𝐾 — компактное метрическое пространство, а 𝑆 — подмножество пространства 𝒞 (𝐾). Доказать,
что 𝑆 компактно (по отношению к метрике, определенной в пункте 7.14) тогда и только тогда, когда 𝑆
равномерно замкнуто, поточечно ограничено и равностепенно непрерывно (если 𝑆 не является равностепенно непрерывным, то 𝑆 содержит последовательность, не имеющую равностепенно непрерывной
подпоследовательности, значит, не имеющую подпоследовательности, равномерно сходящейся на 𝐾).
125
20. Если функция 𝑓 непрерывна на [0, 1] и если
1
∫
0
𝑓 (𝑥)𝑥𝑛 𝑑𝑥 = 0
(𝑛 = 0, 1, 2, … ),
доказать, что 𝑓 (𝑥) = 0 на [0, 1]. Указание. Интеграл от произведения функции 𝑓 на любой многочлен
1
𝑓 2 (𝑥) 𝑑𝑥 = 0.
∫
0
21. Пусть 𝐾 — единичная окружность на комплексной плоскости (т. е. множество всех 𝑧, таких, что |𝑧| = 1),
и пусть 𝒜 — алгебра всех функций вида
равен нулю. Воспользоваться теоремой Вейерштрасса и показать, что
𝑁
𝑓 (𝑒𝑖𝜃 ) =
∑
𝑐𝑛 𝑒𝑖𝑛𝜃
(𝜃 вещественно).
𝑛=0
Тогда 𝒜 разделяет точки множества 𝐾 и не исчезает ни в одной точке множества 𝐾, но тем не менее существуют непрерывные на 𝐾 функции, не содержащиеся в равномерном замыкании алгебры 𝒜 .
Указание. Для любой функции 𝑓 ∈ 𝒜
2𝜋
∫
0
𝑓 (𝑒𝑖𝜃 )𝑒𝑖𝜃 𝑑𝜃 = 0,
и это остается верным для каждой 𝑓 , принадлежащей замыканию алгебры 𝒜 .
22. Пусть 𝑓 ∈ ℛ(𝛼) на [𝑎, 𝑏]. Доказать, что существуют многочлены 𝑃𝑛 , для которых
𝑏
lim
𝑛→∞ ∫
𝑎
|𝑓 − 𝑃𝑛 |2 𝑑𝛼 = 0
(ср. с упражнением 12 гл. 6).
23. Положим 𝑃0 = 0 и для 𝑛 = 0, 1, 2, …
𝑃𝑛+1 (𝑥) = 𝑃𝑛 (𝑥) +
𝑥2 − 𝑃𝑛2 (𝑥)
.
2
Доказать, что
lim 𝑃𝑛 (𝑥) = |𝑥|
𝑛→∞
равномерно на [−1, 1].
(Это делает возможным доказательство теоремы Стоуна-Вейерштрасса без использования теоремы
7.26.)
Указание. Использовать тождество
|𝑥| − 𝑃𝑛+1 (𝑥) = [|𝑥| − 𝑃𝑛 (𝑥)] 1 −
[
|𝑥| + 𝑃𝑛 (𝑥)
,
]
2
чтобы доказать, что 0 ≤ 𝑃𝑛 (𝑥) ≤ 𝑃𝑛+1 (𝑥) ≤ |𝑥|, если |𝑥| ≤ 1, и что
𝑛
|𝑥|
2
|𝑥| − 𝑃𝑛 (𝑥) ≤ |𝑥| 1 −
<
,
(
2 )
𝑛+1
если |𝑥| ≤ 1.
24. Пусть 𝑋 — метрическое пространство с метрикой 𝑑. Зафиксируем точку 𝑎 ∈ 𝑋. Поставим в соответствие
каждой точке 𝑝 ∈ 𝑋 функцию 𝑓𝑝 , определенную как
𝑓𝑝 (𝑥) = 𝑑(𝑥, 𝑝) − 𝑑(𝑥, 𝑎)
(𝑥 ∈ 𝑋).
Доказать, что |𝑓𝑝 (𝑥)| ≤ 𝑑(𝑎, 𝑝) при всех 𝑥 ∈ 𝑋, следовательно, 𝑓𝑝 ∈ 𝒞 (𝑋). Доказать, что
‖𝑓𝑝 − 𝑓𝑞 ‖ = 𝑑(𝑝, 𝑞)
при всех 𝑝, 𝑞 ∈ 𝑋.
Если положить Φ(𝑝) = 𝑓𝑝 , получим, что Φ — изометрия (отображение, сохраняющее расстояния) пространства 𝑋 на Φ(𝑋) ⊂ 𝒞 (𝑋).
Пусть 𝑌 — замыкание множества Φ(𝑋) в 𝒞 (𝑋). Доказать, что 𝑌 полно.
Вывод: 𝑋 изометрично всюду плотному подмножеству полного метрического пространства 𝑌 (упражнение 24 гл. 3 содержит другое доказательство этого факта).
126
25. Пусть 𝜑 — непрерывная ограниченная вещественная функция на полосе, определяемой неравенствами
0 ≤ 𝑥 ≤ 1, −∞ < 𝑦 < ∞. Доказать, что задача с начальными условиями
𝑦′ = 𝜑(𝑥, 𝑦),
𝑦(0) = 𝑐
имеет решение (заметим, что условия этой теоремы существования менее ограничительны, чем условия
соответствующей теоремы единственности; см. упражнение 27 гл. 5).
Указание. Зафиксируем 𝑛. При 𝑖 = 0, … , 𝑛 положим 𝑥𝑖 = 𝑖/𝑛. Пусть 𝑓𝑛 — непрерывная функция на [0, 1],
такая, что 𝑓𝑛 (0) = 𝑐,
𝑓𝑛′ (𝑡) = 𝜑(𝑥𝑖 , 𝑓𝑛 (𝑥𝑖 )),
если 𝑥𝑖 < 𝑡 < 𝑥𝑖+1 ,
и положим
Δ𝑛 (𝑡) = 𝑓𝑛′ (𝑡) − 𝜑(𝑡, 𝑓𝑛 (𝑡)),
за исключением точек 𝑥𝑖 , где положим Δ𝑛 (𝑡) = 0. Тогда
𝑥
𝑓𝑛 (𝑥) = 𝑐 +
∫
0
[𝜑(𝑡, 𝑓𝑛 (𝑡)) + Δ𝑛 (𝑡)] 𝑑𝑡.
Выбрать 𝑀 < ∞ так, чтобы |𝜑| ≤ 𝑀. Проверить следующие утверждения.
(a)
(b)
(c)
(d)
|𝑓𝑛′ | ≤ 𝑀, |Δ𝑛 | ≤ 2𝑀, Δ𝑛 ∈ ℛ и |𝑓𝑛 | ≤ |𝑐| + 𝑀 = 𝑀1 , скажем, на [0, 1], при всех 𝑛.
Последовательность {𝑓𝑛 } равностепенно непрерывна на [0, 1], так как |𝑓𝑛′ | ≤ 𝑀.
Некоторая подпоследовательность {𝑓𝑛𝑘 } сходится к некоторой функции 𝑓 равномерно на [0, 1].
Ввиду того что 𝜑 равномерно непрерывна на прямоугольнике 0 ≤ 𝑥 ≤ 1, |𝑦| ≤ 𝑀1 ,
𝜑(𝑡, 𝑓𝑛𝑘 (𝑡)) → 𝜑(𝑡, 𝑓 (𝑡))
равномерно на [0, 1].
(e) Δ𝑛 (𝑡) → 0 равномерно на [0, 1], так как
Δ𝑛 (𝑡) = 𝜑(𝑥𝑖 , 𝑓𝑛 (𝑥𝑖 )) − 𝜑(𝑡, 𝑓𝑛 (𝑡))
на (𝑥𝑖 , 𝑥𝑖+1 ).
(f) Следовательно,
𝑥
𝑓 (𝑥) = 𝑐 +
∫
0
𝜑(𝑡, 𝑓 (𝑡)) 𝑑𝑡.
Эта функция 𝑓 и есть решение данной задачи.
26. Доказать аналогичную теорему существования для задачи с начальными условиями
y′ = 𝚽(𝑥, y),
y(0) = c,
где теперь c ∈ 𝑅𝑘 , y ∈ 𝑅𝑘 , а 𝚽 — непрерывное ограниченное отображение части пространства 𝑅𝑘+1 ,
определяемой условиями 0 ≤ 𝑥 ≤ 1, y ∈ 𝑅𝑘 , в 𝑅𝑘 (сравнить с упражнением 28 гл. 5). Указание. Воспользоваться векторнозначным вариантом теоремы 7.25.
127
Глава 8
Некоторые специальные функции
Степенные ряды
В этом разделе мы изучим некоторые свойства функций, представимых в виде суммы степенного
ряда, т. е. функций вида
∞
(1)
𝑓 (𝑥) =
∑
𝑐𝑛 𝑥𝑛
𝑛=0
или, более общо,
∞
(2)
𝑓 (𝑥) =
∑
𝑐𝑛 (𝑥 − 𝑎)𝑛 .
𝑛=0
Такие функции называют аналитическими.
Мы ограничимся вещественными значениями 𝑥. Поэтому вместо кругов сходимости (см. теорему
3.39) мы будем иметь дело с промежутками сходимости.
Если ряд (1) сходится при всех 𝑥 на (−𝑅, 𝑅) для некоторого 𝑅 > 0 (𝑅 может равняться +∞),
то мы будем говорить, что 𝑓 разлагается в степенной ряд в окрестности точки 𝑥 = 0. Аналогично,
если ряд (2) сходится при |𝑥 − 𝑎| < 𝑅, то говорят, что 𝑓 разлагается в степенной ряд в окрестности
точки 𝑥 = 𝑎. Для удобства мы часто будем полагать 𝑎 = 0, не ограничивая общности.
8.1 Теорема. Пусть ряд
∞
(3)
∑
𝑐𝑛 𝑥𝑛
𝑛=0
сходится при |𝑥| < 𝑅. Обозначим
∞
(4)
𝑓 (𝑥) =
∑
𝑐𝑛 𝑥𝑛
(|𝑥| < 𝑅).
𝑛=0
Тогда ряд (3) сходится равномерно на [−𝑅 + 𝜀, 𝑅 − 𝜀], каково бы ни было 𝜀 > 0. Функция 𝑓
непрерывна и дифференцируема на (−𝑅, 𝑅) и
∞
(5)
𝑓 ′ (𝑥) =
∑
𝑛𝑐𝑛 𝑥𝑛−1
(|𝑥| < 𝑅).
𝑛=1
Доказательство. Пусть дано 𝜀 > 0. Если |𝑥| ≤ 𝑅 − 𝜀, то
|𝑐𝑛 𝑥𝑛 | ≤ |𝑐𝑛 (𝑅 − 𝜀)𝑛 |,
128
а так как ряд
∑ 𝑐𝑛 (𝑅 − 𝜀)𝑛
сходится абсолютно (каждый степенной ряд по признаку Коши сходится абсолютно внутри своего
промежутка сходимости), то из теоремы 7.10 следует равномерная сходимость ряда (3) на [−𝑅 +
𝜀, 𝑅 − 𝜀].
𝑛
Ввиду того что √
𝑛 → 1 при 𝑛 → ∞, имеем
𝑛
𝑛
lim √𝑛|𝑐𝑛 | = lim √|𝑐𝑛 |,
𝑛→∞
𝑛→∞
так что ряды (4) и (5) имеют общий промежуток сходимости.
Поскольку (5) — степенной ряд, то он сходится равномерно на [−𝑅+𝜀, 𝑅−𝜀] при каждом 𝜀 > 0, и
применима теорема 7.17 (для рядов вместо последовательностей). Таким образом, (5) выполняется,
если |𝑥| ≤ 𝑅 − 𝜀.
Но для любого 𝑥, такого, что |𝑥| < 𝑅, можно найти такое 𝜀 > 0, что |𝑥| < 𝑅 − 𝜀. Значит, (5)
выполняется при |𝑥| < 𝑅.
Непрерывность функции 𝑓 следует из существования 𝑓 ′ (теорема 5.2).
Следствие. Если выполнены условия теоремы 8.1, то 𝑓 имеет производные всех порядков на
(−𝑅, 𝑅), причем
∞
𝑓 (𝑘) (𝑥) =
(6)
∑
𝑛(𝑛 − 1) ⋯ (𝑛 − 𝑘 + 1)𝑐𝑛 𝑥𝑛−𝑘 .
𝑛=𝑘
В частности,
(7)
𝑓 (𝑘) (0) = 𝑘!𝑐𝑘
(𝑘 = 0, 1, 2, … )
(здесь 𝑓 (0) обозначает 𝑓 , а 𝑓 (𝑘) — 𝑘-ую производную функции 𝑓 при 𝑘 = 1, 2, 3, … ).
Доказательство. Равенство (6) получится, если мы последовательно применим теорему 8.1 к
𝑓 , 𝑓 ′ , 𝑓 ″ , … . Полагая 𝑥 = 0 в (6), мы получим (7).
Формула (7) очень интересна. Она показывает, с одной стороны, что коэффициенты степенного
разложения функции 𝑓 определяются значениями 𝑓 и ее производных в единственной точке. С другой стороны, если даны коэффициенты, то значения производных функции 𝑓 в центре промежутка
сходимости усматриваются непосредственно из степенного ряда.
Заметим, однако, что если даже функция 𝑓 имеет производные всех порядков, то ряд ∑ 𝑐𝑛 𝑥𝑛 ,
где 𝑐𝑛 вычислено по формуле (7), может не сходиться к 𝑓 (𝑥) ни при каком 𝑥 ≠ 0. В этом случае
𝑓 не может быть разложена в степенной ряд в окрестности точки 𝑥 = 0. Действительно, если бы
выполнялось 𝑓 (𝑥) = ∑ 𝑎𝑛 𝑥𝑛 , то мы имели бы
𝑛!𝑎𝑛 = 𝑓 (𝑛) (0);
значит, 𝑎𝑛 = 𝑐𝑛 . Пример такой ситуации дан в упражнении 1.
Если ряд (3) сходится в граничной точке, скажем, при 𝑥 = 𝑅, то 𝑓 непрерывна не только на
(−𝑅, 𝑅), но и при 𝑥 = 𝑅. Это вытекает из следующей теоремы, называемой теоремой Абеля (для
простоты мы полагаем 𝑅 = 1).
8.2 Теорема. Пусть ряд ∑ 𝑐𝑛 сходится. Положим
∞
𝑓 (𝑥) =
∑
𝑐𝑛 𝑥𝑛
(−1 < 𝑥 < 1).
𝑛=0
Тогда
∞
(8)
lim 𝑓 (𝑥) =
𝑥→1
∑
𝑛=0
129
𝑐𝑛 .
Доказательство. Пусть 𝑠𝑛 = 𝑐0 + ⋯ + 𝑐𝑛 , 𝑠−1 = 0. Тогда
𝑚
∑
𝑚
𝑛
𝑐𝑛 𝑥 =
𝑛=0
𝑚−1
𝑛
(𝑠 − 𝑠𝑛−1 )𝑥 = (1 − 𝑥)
𝑠 𝑥𝑛 + 𝑠𝑚 𝑥𝑚 .
∑ 𝑛
∑ 𝑛
𝑛=0
𝑛=0
При |𝑥| < 1 положим 𝑚 → ∞ и получим
∞
(9)
𝑓 (𝑥) = (1 − 𝑥)
∑
𝑠𝑛 𝑥𝑛 .
𝑛=0
Обозначим 𝑠 = lim 𝑠𝑛 . Пусть дано 𝜀 > 0. Выберем 𝑁, такое, что из 𝑛 > 𝑁 следует
𝑛→∞
𝜀
.
2
|𝑠 − 𝑠𝑛 | <
Тогда, ввиду того что
∞
(1 − 𝑥)
∑
𝑥𝑛 = 1
(|𝑥| < 1),
𝑛=0
мы получаем из (9)
∞
𝑁
𝜀
|𝑓 (𝑥) − 𝑠| = (1 − 𝑥) (𝑠𝑛 − 𝑠)𝑥𝑛 ≤ (1 − 𝑥)
|𝑠 − 𝑠||𝑥|𝑛 + ≤ 𝜀,
∑
∑ 𝑛
2
|
|
𝑛=0
𝑛=0
если 𝑥 > 1 − 𝛿, для подходящим образом выбранного 𝛿 > 0. Отсюда следует (8).
В качестве приложения докажем теорему 3.51, которая состоит в следующем. Если ряды ∑ 𝑎𝑛 ,
∑ 𝑏𝑛 , ∑ 𝑐𝑛 сходятся к 𝐴, 𝐵, 𝐶 и если 𝑐𝑛 = 𝑎0 𝑏𝑛 + ⋯ + 𝑎𝑛 𝑏0 , то 𝐶 = 𝐴𝐵. Положим
∞
𝑓 (𝑥) =
∑
∞
𝑎𝑛 𝑥𝑛 ,
𝑔(𝑥) =
𝑛=0
∑
∞
𝑏𝑛 𝑥𝑛 ,
ℎ(𝑥) =
𝑛=0
∑
𝑐𝑛 𝑥𝑛
𝑛=0
при 0 ≤ 𝑥 ≤ 1. Если 𝑥 < 1, то эти ряды сходятся абсолютно, и поэтому их можно перемножить в
соответствии с определением 3.48; выполнив умножение, мы увидим, что
(10)
𝑓 (𝑥) ⋅ 𝑔(𝑥) = ℎ(𝑥)
(0 ≤ 𝑥 < 1).
По теореме 8.2
(11)
𝑓 (𝑥) → 𝐴,
𝑔(𝑥) → 𝐵,
ℎ(𝑥) → 𝐶
при 𝑥 → 1. Из равенств (10) и (11) следует, что 𝐴𝐵 = 𝐶.
Далее нам понадобится теорема об изменении порядка суммирования (см. упражнения 2 и 3).
8.3 Теорема. Пусть дана двойная последовательность {𝑎𝑖𝑗 }, 𝑖 = 1, 2, 3, … , 𝑗 = 1, 2, 3, … , пусть
∞
(12)
∑
|𝑎𝑖𝑗 | = 𝑏𝑖
(𝑖 = 1, 2, 3, … )
𝑗=1
и ряд ∑ 𝑏𝑖 сходится. Тогда
∞
(13)
∞
∑∑
𝑖=1 𝑗=1
∞
𝑎𝑖𝑗 =
∞
∑∑
𝑗=1 𝑖=1
130
𝑎𝑖𝑗 .
Доказательство. Мы могли бы получить (13) непосредственно, подобно (хотя и более сложно)
методу, использованному в теореме 3.55. Однако следующий метод представляется более интересным.
Пусть 𝐸 — счетное множество, состоящее из точек 𝑥0 , 𝑥1 , 𝑥2 , … , и пусть 𝑥𝑛 → 𝑥0 при 𝑛 → ∞.
Положим
∞
(14)
𝑓𝑖 (𝑥0 ) =
∑
𝑎𝑖𝑗
(𝑖 = 1, 2, 3, … ),
𝑎𝑖𝑗
(𝑖, 𝑛 = 1, 2, 3, … ),
𝑗=1
𝑛
(15)
𝑓𝑖 (𝑥𝑛 ) =
∑
𝑗=1
∞
(16)
𝑔(𝑥) =
∑
𝑓𝑖 (𝑥)
(𝑥 ∈ 𝐸).
𝑖=1
Теперь (14) и (15) совместно с (12) показывают, что каждая 𝑓𝑖 непрерывна в 𝑥0 . Поскольку
|𝑓𝑖 (𝑥)| ≤ 𝑏𝑖 при 𝑥 ∈ 𝐸, ряд (16) сходится равномерно, так что 𝑔 непрерывна в 𝑥0 (теорема 7.11).
Следовательно,
∞
∞
∑∑
∞
𝑎𝑖𝑗 =
∑
𝑖=1 𝑗=1
𝑓𝑖 (𝑥0 ) = 𝑔(𝑥0 ) = lim 𝑔(𝑥𝑛 )
𝑛→∞
𝑖=1
∞
= lim
∞
𝑛→∞ ∑
𝑖=1
𝑛
𝑓𝑖 (𝑥𝑛 ) = lim
𝑛
𝑎
𝑛→∞ ∑ ∑ 𝑖𝑗
𝑖=1 𝑗=1
∞
= lim
𝑛→∞ ∑ ∑
𝑗=1 𝑖=1
∞
𝑎𝑖𝑗 =
∞
∑∑
𝑎𝑖𝑗 .
𝑗=1 𝑖=1
8.4 Теорема. Пусть ряд
∞
𝑓 (𝑥) =
∑
𝑐𝑛 𝑥𝑛
𝑛=0
сходится при |𝑥| < 𝑅. Если −𝑅 < 𝑎 < 𝑅, то 𝑓 может быть разложена в степенной ряд в
окрестности точки 𝑥 = 𝑎, сходящийся при |𝑥 − 𝑎| < 𝑅 − |𝑎|, и
∞
(17)
𝑓 (𝑥) =
𝑓 (𝑛) (𝑎)
(𝑥 − 𝑎)𝑛
∑ 𝑛!
𝑛=0
(|𝑥 − 𝑎| < 𝑅 − |𝑎|).
Эта теорема является обобщением теоремы 5.15 и тоже известна как теорема Тейлора.
Доказательство. Имеем
∞
𝑓 (𝑥) =
∑
𝑐𝑛 [(𝑥 − 𝑎) + 𝑎]𝑛
𝑛=0
𝑛
∞
=
∑
𝑐𝑛
𝑛=0
∞
=
𝑛 𝑛−𝑚
𝑎 (𝑥 − 𝑎)𝑚
∑ (𝑚 )
𝑛=0
∞
𝑛
𝑐 𝑎𝑛−𝑚 (𝑥 − 𝑎)𝑚 .
∑ [ ∑ (𝑚) 𝑛
]
𝑛=𝑚
𝑚=0
Это и есть нужное нам разложение в окрестности точки 𝑥 = 𝑎. Чтобы убедиться в его справедливости, мы должны обосновать изменение порядка суммирования. Теорема 8.3 показывает, что оно
возможно, если ряд
∞
(18)
∞
𝑛 𝑛−𝑚
𝑐
𝑎 (𝑥 − 𝑎)𝑚
∑ ∑ | 𝑛 (𝑚)
|
𝑛=0 𝑚=0
131
сходится. Но ряд (18), очевидно, равен
∞
(19)
∑
|𝑐𝑛 | ⋅ (|𝑥 − 𝑎| + |𝑎|)𝑛 ,
𝑛=0
а (19) сходится, если |𝑥 − 𝑎| + |𝑎| < 𝑅.
Наконец, вид коэффициентов в (17) следует из (7).
Нужно заметить, что ряд (17) в действительности может сходиться в промежутке, более широком, чем интервал, определяемый неравенством |𝑥 − 𝑎| < 𝑅 − |𝑎|.
Если два степенных ряда сходятся на (−𝑅, 𝑅) к одной и той же функции, то (7) показывает, что
они должны быть тождественны, т. е. иметь одинаковые коэффициенты. Интересно, что тот же
вывод верен и при более слабых предположениях.
8.5 Теорема. Пусть ряды ∑ 𝑎𝑛 𝑥𝑛 и ∑ 𝑏𝑛 𝑥𝑛 сходятся на интервале 𝑆 = (−𝑅, 𝑅). Пусть 𝐸 —
множество всех 𝑥 ∈ 𝑆, в которых
∞
(20)
∞
∑
𝑎𝑛 𝑥𝑛 =
𝑛=0
∑
𝑏𝑛 𝑥𝑛 .
𝑛=0
Если 𝐸 имеет предельную точку в 𝑆, то 𝑎𝑛 = 𝑏𝑛 при 𝑛 = 0, 1, 2, … . Значит, (20) выполняется при
всех 𝑥 ∈ 𝑆.
Доказательство. Пусть 𝑐𝑛 = 𝑎𝑛 − 𝑏𝑛 и
∞
(21)
𝑓 (𝑥) =
∑
𝑐𝑛 𝑥𝑛
(𝑥 ∈ 𝑆).
𝑛=0
Тогда 𝑓 (𝑥) = 0 на 𝐸.
Пусть 𝐴 — множество всех предельных точек множества 𝐸 в 𝑆, а 𝐵 состоит из всех прочих
точек множества 𝑆. Из определения предельной точки ясно, что 𝐵 открыто. Допустим, что нам
удалось доказать, что 𝐴 открыто. Тогда 𝐴 и 𝐵 — непересекающиеся открытые множества, значит,
они отделены друг от друга (определение 2.45). Ввиду того что 𝑆 = 𝐴 ∪ 𝐵 и 𝑆 связно, одно из
множеств 𝐴 и 𝐵 должно быть пустым. По предположению 𝐴 непусто. Значит, 𝐵 пусто, и 𝐴 = 𝑆. Из
того, что 𝑓 непрерывна на 𝑆, следует 𝐴 ⊂ 𝐸. Таким образом, 𝐸 = 𝑆, а (7) показывает, что 𝑐𝑛 = 0
при 𝑛 = 0, 1, 2, … , что и составляет утверждение теоремы.
Таким образом, нам нужно доказать, что 𝐴 открыто. Если 𝑥0 ∈ 𝐴, то теорема 8.4 показывает,
что
∞
(22)
𝑓 (𝑥) =
∑
𝑑𝑛 (𝑥 − 𝑥0 )𝑛
(|𝑥 − 𝑥0 | < 𝑅 − |𝑥0 |).
𝑛=0
Мы утверждаем, что 𝑑𝑛 = 0 при всех 𝑛. Предполагая противное, обозначим через 𝑘 наименьшее
неотрицательное целое число, такое, что 𝑑𝑘 ≠ 0. Тогда
(23)
𝑓 (𝑥) = (𝑥 − 𝑥0 )𝑘 𝑔(𝑥)
(|𝑥 − 𝑥0 | < 𝑅 − |𝑥0 |),
где
∞
(24)
𝑔(𝑥) =
∑
𝑑𝑘+𝑚 (𝑥 − 𝑥0 )𝑚 .
𝑚=0
Поскольку 𝑔 непрерывна в 𝑥0 и
𝑔(𝑥0 ) = 𝑑𝑘 ≠ 0,
то существует 𝛿 > 0, такое, что 𝑔(𝑥) ≠ 0, если |𝑥 − 𝑥0 | < 𝛿. Из (23) следует, что 𝑓 (𝑥) ≠ 0, если
0 < |𝑥 − 𝑥0 | < 𝛿. Но это противоречит тому, что 𝑥0 — предельная точка множества 𝐸.
Таким образом, 𝑑𝑛 = 0 при всех 𝑛, так что 𝑓 (𝑥) = 0 при всех 𝑥, для которых выполняется (22),
т. е. в окрестности точки 𝑥0 . Это показывает, что 𝐴 открыто, и доказательство закончено.
132
Показательная и логарифмическая функции
Положим, по определению,
∞
(25)
𝐸(𝑧) =
𝑧𝑛
.
∑ 𝑛!
𝑛=0
Признак Даламбера показывает, что этот ряд сходится при каждом комплексном 𝑧. Применяя
теорему 3.50 об умножении абсолютно сходящихся рядов, получаем
𝐸(𝑧)𝐸(𝑤) =
=
∞
∞
∞
𝑛
∞
𝑛
𝑧𝑛
𝑤𝑚
𝑧𝑘 𝑤𝑛−𝑘
=
∑ 𝑛! ∑ 𝑚!
∑ ∑ 𝑘!(𝑛 − 𝑘)!
𝑛=0
𝑚=0
𝑛=0 𝑘=0
∞
(𝑧 + 𝑤)𝑛
1
𝑛 𝑘 𝑛−𝑘
𝑧 𝑤
=
,
∑ 𝑛! ∑ (𝑘)
∑
𝑛!
𝑛=0
𝑘=0
𝑛=0
откуда вытекает важная формула сложения
(26)
(𝑧, 𝑤 комплексные).
𝐸(𝑧 + 𝑤) = 𝐸(𝑧)𝐸(𝑤)
Одно из ее следствий таково:
(27)
𝐸(𝑧)𝐸(−𝑧) = 𝐸(𝑧 − 𝑧) = 𝐸(0) = 1
(𝑧 комплексное).
Это показывает, что 𝐸(𝑧) ≠ 0 при всех 𝑧. Согласно (25), 𝐸(𝑥) > 0, если 𝑥 > 0; а (27) показывает, что
𝐸(𝑥) > 0 при всех вещественных 𝑥. В силу (25), 𝐸(𝑥) → +∞ при 𝑥 → +∞, а (27) показывает, что
𝐸(𝑥) → 0 при 𝑥 → −∞ вдоль вещественной оси. В силу (25), из 0 < 𝑥 < 𝑦 следует, что 𝐸(𝑥) < 𝐸(𝑦);
из (27) следует 𝐸(−𝑦) < 𝐸(−𝑥); значит, 𝐸 строго возрастает на всей вещественной оси.
Формула сложения показывает также, что
(28)
lim
ℎ→0
𝐸(𝑧 + ℎ) − 𝐸(𝑧)
𝐸(ℎ) − 1
= 𝐸(𝑧) lim
= 𝐸(𝑧);
ℎ→0
ℎ
ℎ
последнее равенство следует прямо из (25).
Повторяя (26), получаем
(29)
𝐸(𝑧1 + ⋯ + 𝑧𝑛 ) = 𝐸(𝑧1 ) ⋯ 𝐸(𝑧𝑛 ).
Положим здесь 𝑧1 = ⋯ = 𝑧𝑛 = 1. Поскольку 𝐸(1) = 𝑒, где 𝑒 — число, введенное в определении 3.30,
то мы получаем
(30)
𝐸(𝑛) = 𝑒𝑛
(𝑛 = 1, 2, 3, ⋯ ).
Если 𝑝 = 𝑚/𝑛, где 𝑛, 𝑚 — положительные целые числа, то
(31)
[𝐸(𝑝)]𝑚 = 𝐸(𝑚𝑝) = 𝐸(𝑛) = 𝑒𝑛 ,
так что
(32)
𝐸(𝑝) = 𝑒𝑝
(𝑝 > 0, 𝑝 рационально).
Из (27) следует, что 𝐸(−𝑝) = 𝑒−𝑝 , если 𝑝 положительно и рационально. Таким образом, (32) выполняется при всех рациональных 𝑝.
В упражнении 6 гл. 1 мы предложили определение
𝑥𝑦 = sup 𝑥𝑝 ,
(33)
где верхняя грань берется по всем рациональным 𝑝, таким, что 𝑝 < 𝑦, для любого вещественного 𝑦
и 𝑥 > 1. Таким образом, если мы определим при любом вещественном 𝑥
(34)
𝑒𝑥 = sup 𝑒𝑝
(𝑝 < 𝑥, 𝑝 рационально),
133
то из непрерывности и монотонности функции 𝐸, а также из равенства (32) получится
𝐸(𝑥) = 𝑒𝑥
(35)
при всех вещественных 𝑥. Равенство (35) объясняет, почему функцию 𝐸 называют показательной.
Вместо 𝑒𝑥 часто используется обозначение exp(𝑥), особенно в тех случаях, когда 𝑥 — сложное
выражение.
На самом деле вместо (34) вполне можно воспользоваться (35) для определения 𝑒𝑥 ; (35) — гораздо более удобная отправная точка для исследования свойств функции 𝑒𝑥 . Скоро мы увидим,
что и (33) можно заменить более удобным определением (см. (43)).
Вернемся к обычному обозначению 𝑒𝑥 вместо 𝐸(𝑥) и подытожим то, что было доказано до сих
пор.
8.6 Теорема. Пусть 𝑒𝑥 определяется на 𝑅1 равенствами (35) и (25). Тогда
(a)
(b)
(c)
(d)
(e)
(f )
𝑒𝑥 непрерывна и дифференцируема при всех 𝑥;
(𝑒𝑥 )′ = 𝑒𝑥 ;
𝑒𝑥 — строго возрастающая функция от 𝑥 и 𝑒𝑥 > 0;
𝑒𝑥+𝑦 = 𝑒𝑥 𝑒𝑦 ;
𝑒𝑥 → +∞ при 𝑥 → +∞, 𝑒𝑥 → 0 при 𝑥 → −∞;
lim𝑥→+∞ 𝑥𝑛 𝑒−𝑥 = 0 при всех 𝑛.
Доказательство. Мы уже доказали (a)-(e); (25) показывает, что
𝑥𝑛+1
(𝑛 + 1)!
𝑒𝑥 >
при 𝑥 > 0, так что
(𝑛 + 1)!
,
𝑥
откуда следует (f). Утверждение (f) означает, что 𝑒𝑥 стремится к +∞ при 𝑥 → +∞ «быстрее», чем
любая степень 𝑥.
𝑥𝑛 𝑒−𝑥 <
Так как 𝐸 строго возрастает и дифференцируема на 𝑅1 , она имеет обратную функцию 𝐿, которая тоже строго возрастает и дифференцируема и область определения которой совпадает с 𝐸(𝑅1 ),
т. е. с множеством всех положительных чисел. Функция 𝐿 определяется из равенства
(36)
𝐸(𝐿(𝑦)) = 𝑦
(𝑦 > 0)
или, что то же самое, из равенства
(37)
(𝑥 вещественно).
𝐿(𝐸(𝑥)) = 𝑥
Дифференцируя (37), получаем (ср. с теоремой 5.5)
𝐿′ (𝐸(𝑥)) ⋅ 𝐸(𝑥) = 1.
Записывая 𝑦 = 𝐸(𝑥), имеем
(38)
𝐿′ (𝑦) =
1
𝑦
(𝑦 > 0).
Полагая 𝑥 = 0 в (37), мы видим, что 𝐿(1) = 0. Значит, из (38) следует
𝑦
(39)
𝐿(𝑦) =
∫
1
𝑑𝑥
.
𝑥
Очень часто (39) принимают за отправную точку в теории логарифма и показательной функции.
Полагая 𝑢 = 𝐸(𝑥), 𝑣 = 𝐸(𝑦), получаем из (26)
𝐿(𝑢𝑣) = 𝐿(𝐸(𝑥) ⋅ 𝐸(𝑦)) = 𝐿(𝐸(𝑥 + 𝑦)) = 𝑥 + 𝑦,
134
так что
(40)
𝐿(𝑢𝑣) = 𝐿(𝑢) + 𝐿(𝑣)
(𝑢 > 0, 𝑣 > 0).
Это показывает, что 𝐿 обладает известным свойством, которое делает логарифмы средством,
полезных для вычислений. Обычное обозначение для 𝐿(𝑥), конечно, log 𝑥.
Что касается поведения log 𝑥 при 𝑥 → +∞ и при 𝑥 → 0, теорема 8.6(e) показывает, что
log 𝑥 → +∞
при 𝑥 → +∞,
log 𝑥 → −∞
при 𝑥 → 0.
Легко видеть, что
𝑥𝑛 = 𝐸(𝑛𝐿(𝑥)),
(41)
если 𝑥 > 0, а 𝑛 — целое. Подобным же образом, если 𝑚 — положительное целое, то
1
𝑥1/𝑚 = 𝐸 ( 𝐿(𝑥)) ,
𝑚
(42)
так как каждая из частей (42) после возведения в 𝑚-ую степень превращается в соответствующую
часть равенства (36). Объединяя (41) и (42), получаем
𝑥𝛼 = 𝐸(𝛼𝐿(𝑥)) = 𝑒𝛼 log 𝑥
(43)
при любом рациональном 𝛼.
Определим теперь 𝑥𝛼 при любом вещественном 𝛼 и любом 𝑥 > 0 равенством (43). Непрерывность
и монотонность функций 𝐸 и 𝐿 показывают, что это определение приводит к тому же результату,
что и предложенное ранее. Утверждения, сформулированные в упражнении 6 гл. 1, — тривиальные
следствия (43).
Продифференцировав (43), получаем по теореме 5.5
(𝑥𝛼 )′ = 𝐸(𝛼𝐿(𝑥)) ⋅
(44)
𝛼
= 𝛼𝑥𝛼−1 .
𝑥
Заметим, что раньше мы использовали (44) только для целых значений 𝛼, а в этом случае (44) легко
следует из теоремы 5.3(b). Доказать (44), исходя непосредственно из определения производной, если
𝑥𝛼 определено, как в (33) и 𝛼 иррационально, весьма затруднительно.
Хорошо известная формула интегрирования для 𝑥𝛼 следует из (44), если 𝛼 ≠ −1, и из (38), если
𝛼 = −1. Мы хотим доказать еще одно свойство функции log 𝑥, а именно
lim 𝑥−𝛼 log 𝑥 = 0
(45)
𝑥→+∞
при каждом 𝛼 > 0. Иначе говоря, log 𝑥 → +∞ при 𝑥 → +∞ «медленнее», чем любая положительная
степень 𝑥.
Действительно, если 0 < 𝜀 < 𝛼, а 𝑥 > 1, то
𝑥−𝛼 log 𝑥 = 𝑥−𝛼
𝑥
𝑡−1 𝑑𝑡 < 𝑥−𝛼
∫
∫
1
1
𝜀
𝜀−𝛼
𝑥
−
1
𝑥
−𝛼
=𝑥 ⋅
<
,
𝜀
𝜀
𝑥
𝑡𝜀−1 𝑑𝑡
откуда и следует (45). Мы могли бы также воспользоваться теоремой 8.6(f) для вывода (45).
Тригонометрические функции
Положим, по определению,
(46)
𝐶(𝑥) =
1
[𝐸(𝑖𝑥) + 𝐸(−𝑖𝑥)],
2
𝑆(𝑥) =
135
1
[𝐸(𝑖𝑥) − 𝐸(−𝑖𝑥)].
2𝑖
Мы покажем, что 𝐶(𝑥) и 𝑆(𝑥) совпадают с функциями cos 𝑥 и sin 𝑥, определение которых обычно
основывается на геометрических соображениях. Согласно (25), 𝐸(𝑧)̄ = 𝐸(𝑧). Значит, как показывает
(46), 𝐶(𝑥) и 𝑆(𝑥) вещественны при вещественном 𝑥. Кроме того,
(47)
𝐸(𝑖𝑥) = 𝐶(𝑥) + 𝑖𝑆(𝑥).
Таким образом, 𝐶(𝑥) и 𝑆(𝑥) равны соответственно вещественной и мнимой части числа 𝐸(𝑖𝑥), если
𝑥 вещественно. Согласно (27),
|𝐸(𝑖𝑥)|2 = 𝐸(𝑖𝑥)𝐸(𝑖𝑥) = 𝐸(𝑖𝑥)𝐸(−𝑖𝑥) = 1,
так что
(48)
(𝑥 вещественно).
|𝐸(𝑖𝑥)| = 1
Из (46) можно усмотреть, что 𝐶(0) = 1, 𝑆(0) = 0, а (28) показывает, что
𝐶 ′ (𝑥) = −𝑆(𝑥),
(49)
𝑆 ′ (𝑥) = 𝐶(𝑥).
Мы утверждаем, что существуют положительные числа 𝑥, такие, что 𝐶(𝑥) = 0. Действительно,
пусть это не так. Из того, что 𝐶(0) = 1, следует тогда, что 𝐶(𝑥) > 0 при всех 𝑥 > 0, значит, 𝑆 ′ (𝑥) > 0,
согласно (49), и, значит, 𝑆 строго возрастает; а так как 𝑆(0) = 0, то 𝑆(𝑥) > 0 при 𝑥 > 0. Значит,
если 0 < 𝑥 < 𝑦, то
𝑦
(50)
𝑆(𝑥)(𝑦 − 𝑥) <
∫
𝑥
𝑆(𝑡) 𝑑𝑡 = 𝐶(𝑥) − 𝐶(𝑦) ≤ 2.
Последнее неравенство следует из (48) и (47). Так как 𝑆(𝑥) > 0, то (50) не может выполняться при
больших 𝑦, и мы получили противоречие.
Пусть 𝑥0 — наименьшее положительное число, такое, что 𝐶(𝑥0 ) = 0. Оно существует, так как
множество нулей непрерывной функции замкнуто, а 𝐶(0) ≠ 0. Определим число 𝜋 равенством
(51)
𝜋 = 2𝑥0 .
Тогда 𝐶(𝜋/2) = 0 и, как показывает (48), 𝑆(𝜋/2) = ±1. Так как 𝐶(𝑥) > 0 на (0, 𝜋/2), то 𝑆 возрастает
на (0, 𝜋/2); значит, 𝑆(𝜋/2) = 1. Таким образом,
𝐸(
𝜋𝑖
= 𝑖,
2)
и формула сложения показывает, что
(52)
𝐸(𝜋𝑖) = −1,
𝐸(2𝜋𝑖) = 1;
значит,
(53)
(𝑧 комплексное).
𝐸(𝑧 + 2𝜋𝑖) = 𝐸(𝑧)
8.7 Теорема.
(a)
(b)
(c)
(d)
Функция 𝐸 периодична с периодом 2𝜋𝑖.
Функции 𝐶 и 𝑆 периодичны с периодом 2𝜋.
Если 0 < 𝑡 < 2𝜋, то 𝐸(𝑖𝑡) ≠ 1.
Если 𝑧 — комплексное число с |𝑧| = 1, то существует единственное 𝑡 на [0, 2𝜋), такое, что
𝐸(𝑖𝑡) = 𝑧.
Доказательство. Утверждение (a) следует из (53), а (b) следует из (a) и (46).
Пусть 0 < 𝑡 < 𝜋/2, а 𝐸(𝑖𝑡) = 𝑥 + 𝑖𝑦, где 𝑥, 𝑦 вещественны. Сделанное нами ранее показывает, что
0 < 𝑥 < 1, 0 < 𝑦 < 1. Заметим, что
𝐸(4𝑖𝑡) = (𝑥 + 𝑖𝑦)4 = 𝑥4 − 6𝑥2 𝑦2 + 𝑦4 + 4𝑖𝑥𝑦(𝑥2 − 𝑦2 ).
136
Если 𝐸(4𝑖𝑡) вещественно, то 𝑥2 − 𝑦2 = 0, а так как, согласно (48), 𝑥2 + 𝑦2 = 1, то 𝑥2 = 𝑦2 = 12 , значит,
𝐸(4𝑖𝑡) = −1. Тем самым (c) доказано.
Если 0 ≤ 𝑡1 < 𝑡2 < 2𝜋, то
𝐸(𝑖𝑡2 )[𝐸(𝑖𝑡1 )]−1 = 𝐸(𝑖𝑡2 − 𝑖𝑡1 ) ≠ 1,
согласно (c). Тем самым доказано утверждение о единственности, содержащееся в (d).
Чтобы доказать утверждение о существовании в (d), зафиксируем 𝑧, такое, что |𝑧| = 1. Запишем
𝑧 = 𝑥 + 𝑖𝑦, где 𝑥 и 𝑦 вещественны. Допустим сначала, что 𝑥 ≥ 0 и 𝑦 ≥ 0. На [0, 𝜋/2] 𝐶 убывает от 1 до
0. Значит, 𝐶(𝑡) = 𝑥 для некоторого 𝑡 ∈ [0, 𝜋/2]. Из того, что 𝐶 2 + 𝑆 2 = 1 и 𝑆 ≥ 0 на [0, 𝜋/2], следует,
что 𝑧 = 𝐸(𝑖𝑡).
Если 𝑥 < 0 и 𝑦 ≥ 0, то предыдущим условиям удовлетворяет −𝑖𝑧. Значит, −𝑖𝑧 = 𝐸(𝑖𝑡) для
некторого 𝑡 ∈ [0, 𝜋/2], а так как 𝑖 = 𝐸(𝜋𝑖/2), то мы получим 𝑧 = 𝐸(𝑖(𝑡 + 𝜋/2)). Наконец, если 𝑦 < 0, то
два предыдущих случая показывают, что −𝑧 = 𝐸(𝑖𝑡) для некоторого 𝑡 ∈ (0, 𝜋). Значит, 𝑧 = −𝐸(𝑖𝑡) =
𝐸(𝑖(𝑡 + 𝜋)).
Утверждение (d) доказано, а с ним и вся теорема.
Из (d) и (48) следует, что кривая 𝛾, определяемая равенством
(54)
𝛾(𝑡) = 𝐸(𝑖𝑡)
(0 ≤ 𝑡 ≤ 2𝜋),
— простая замкнутая кривая1 , множество значений которой — единичная окружность на плоскости. Ввиду того что 𝛾 ′ (𝑡) = 𝑖𝐸(𝑖𝑡), длина кривой 𝛾 равна
2𝜋
∫
0
|𝛾 ′ (𝑡)| 𝑑𝑡 = 2𝜋
по теореме 6.27. Такого результата, конечно, и следовало ожидать для окружности длиной 1. Он
показывает, что 𝜋, определенное в (51), имеет обычный геометрический смысл.
Таким же точно образом мы увидим, что при возрастании 𝑡 от 0 до 𝑡0 точка 𝛾(𝑡) описывает дугу
окружности длиной 𝑡0 . Рассмотрение треугольника с вершинами
𝑧1 = 0,
𝑧2 = 𝛾(𝑡0 ),
𝑧3 = 𝐶(𝑡0 )
показывает, что 𝐶(𝑡) и 𝑆(𝑡) на самом деле совпадают с cos 𝑡 и sin 𝑡, если эти последние определены
обычным способом как отношения сторон прямоугольного треугольника.
Следует подчеркнуть, что мы вывели основные свойства тригонометрических функций из (46) и
(25), не привлекая геометрического понятия угла. Существуют и другие негеометрические подходы
к этим функциям. Этим вопросам посвящены статьи W. F. Eberlein (Amer. Math. Monthly, vol. 74,
1967, pp. 1223-1225) и G. B. Robison(Math. Mag., vol. 41, 1968, pp. 66-70).
Алгебраическая замкнутость поля комплексных чисел
Теперь мы в состоянии дать простое доказательство того факта, что поле комплексных чисел
алгебраически замкнуто, т. е. каждый отличный от постоянной многочлен с комплексными коэффициентами имеет комплексный корень.
8.8 Теорема. Пусть 𝑎0 , … , 𝑎𝑛 — комплексные числа, 𝑛 ≥ 1, 𝑎𝑛 ≠ 0,
𝑛
𝑃 (𝑧) =
∑
𝑎𝑘 𝑧𝑘 .
0
Тогда 𝑃 (𝑧) = 0 для некоторого комплексного числа 𝑧.
1
Простой замкнутой кривой называется кривая 𝛾 на [𝑎, 𝑏], для которой 𝛾(𝑎) = 𝛾(𝑏), но 𝛾(𝑡1 ) ≠ 𝛾(𝑡2 ) для любой другой
пары различных точек 𝑡1 , 𝑡2 ∈ [𝑎, 𝑏]. — Прим. перев.
137
Доказательство. Не ограничивая общности, допустим, что 𝑎𝑛 = 1. Положим
(55)
𝜇 = inf |𝑃 (𝑧)|
(𝑧 комплексное).
Если |𝑧| = 𝑅, то
(56)
|𝑃 (𝑧)| ≥ 𝑅𝑛 [1 − |𝑎𝑛−1 |𝑅−1 − ⋯ − |𝑎0 |𝑅−𝑛 ].
Правая часть (56) стремится к ∞ при 𝑅 → ∞. Значит, существует такое 𝑅0 , что |𝑃 (𝑧)| > 𝜇, если
|𝑧| > 𝑅0 . Ввиду того что функция |𝑃 | непрерывна на замкнутом круге с центром в 0 и радиусом
𝑅0 , мы заключаем на основании теоремы 4.16, что |𝑃 (𝑧0 )| = 𝜇 при некотором 𝑧0 .
Мы утверждаем, что 𝜇 = 0.
Если это не так, то положим 𝑄(𝑧) = 𝑃 (𝑧 + 𝑧0 )/𝑃 (𝑧0 ). Тогда 𝑄 — многочлен, отличный от постоянной, 𝑄(0) = 1 и |𝑄(𝑧)| ≥ 1 при всех 𝑧. Существует наименьшее целое 𝑘, 1 ≤ 𝑘 ≤ 𝑛, такое,
что
(57)
𝑄(𝑧) = 1 + 𝑏𝑘 𝑧𝑘 + ⋯ + 𝑏𝑛 𝑧𝑛 ,
𝑏𝑘 ≠ 0.
По теореме 8.7(d) существует вещественное 𝜃, такое, что
𝑒𝑖𝑘𝜃 𝑏𝑘 = −|𝑏𝑘 |.
(58)
Если 𝑟 > 0 и 𝑟𝑘 |𝑏𝑘 | < 1, то из (58) следует
|1 + 𝑏𝑘 𝑟𝑘 𝑒𝑖𝑘𝜃 | = 1 − 𝑟𝑘 |𝑏𝑘 |,
так что
|𝑄(𝑟𝑒𝑖𝜃 )| ≤ 1 − 𝑟𝑘 {|𝑏𝑘 | − 𝑟|𝑏𝑘+1 | − ⋯ − 𝑟𝑛−𝑘 |𝑏𝑛 |} .
При достаточно малом 𝑟 выражение в фигурных скобках положительно, значит, |𝑄(𝑟𝑒𝑖𝜃 )| < 1, и мы
пришли к противоречию.
Таким образом, 𝜇 = 0, т. е. 𝑃 (𝑧0 ) = 0.
Упражнение 27 содержит более общий результат.
Ряды Фурье
8.9 Определение. Тригонометрическим многочленом называется конечная сумма вида
𝑁
(59)
𝑓 (𝑥) = 𝑎0 +
∑
(𝑎𝑛 cos 𝑛𝑥 + 𝑏𝑛 sin 𝑛𝑥)
(𝑥 вещественно),
𝑛=1
где 𝑎0 , … , 𝑎𝑁 , 𝑏1 , … , 𝑏𝑁 — комплексные числа. Учитывая тождества (46), (59) можно также записать в виде
𝑁
(60)
𝑓 (𝑥) =
∑
𝑐𝑛 𝑒𝑖𝑛𝑥
(𝑥 вещественно),
−𝑁
который для многих целей более удобен. Ясно, что каждый тригонометрический многочлен — периодическая функция с периодом 2𝜋.
Если 𝑛 — отличное от нуля целое число, то 𝑒𝑖𝑛𝑥 — производная функции 𝑒𝑖𝑛𝑥 /𝑖𝑛, которая также
имеет период 2𝜋. Поэтому
(61)
𝜋
1
1
𝑒𝑖𝑛𝑥 𝑑𝑥 =
2𝜋 ∫
{0
−𝜋
(если 𝑛 = 0),
(если 𝑛 = ±1, ±2, … ).
Умножим (60) на 𝑒−𝑖𝑚𝑥 , где 𝑚 — целое число; интегрируя это произведение, получим на основании (61)
𝜋
(62)
𝑐𝑚 =
1
𝑓 (𝑥)𝑒−𝑖𝑚𝑥 𝑑𝑥
2𝜋 ∫
−𝜋
138
при |𝑚| ≤ 𝑁. Если |𝑚| > 𝑁, то интеграл в (62) равен 0.
Из (60) и (62) видно, что тригонометрический многочлен, заданный (60), оказывается вещественным в том и только в том случае, когда 𝑐−𝑛 = 𝑐𝑛 при 𝑛 = 0, … , 𝑁.
В соответствии с (60) мы определяем тригонометрический ряд как ряд вида
∞
(63)
∑
𝑐𝑛 𝑒𝑖𝑛𝑥
(𝑥 вещественно);
−∞
𝑁-ая частная сумма ряда (63) по определению равна правой части (60).
Если 𝑓 — интегрируемая функция на [−𝜋, 𝜋], то числа 𝑐𝑚 , заданные (62) для всех целых чисел
𝑚, называются коэффициентами Фурье функции 𝑓 , а ряд (63), составленный при помощи этих
коэффициентов, — рядом Фурье функции 𝑓 .
Теперь возникает естественный вопрос: сходится ли ряд Фурье функции 𝑓 к 𝑓 , или, более
общо, определяется ли 𝑓 своим рядом Фурье. Иначе говоря, если мы знаем коэффициенты Фурье
функции, то можем ли мы найти эту функцию, и если можем, то как?
Изучение таких рядов и, в частности, проблема представления заданной функции тригонометрическим рядом, имеет своим источником такие разделы физики, как теория колебаний и теория
распространения тепла (книга Фурье «Аналитическая теория теплоты» была опубликована в 1822
г.). Многочисленные трудные и тонкие проблемы, возникшие при этом изучении, вызвали основательный пересмотр и перестройку всей теории функций вещественной переменной. Многие выдающиеся имена, и среди них имена Римана, Кантора и Лебега, тесно связаны с этой областью, о
которой вполне можно сказать, что в наши дни она вместе со всеми ее обобщениями и ответвлениями занимает центральное положение в анализе.
Мы ограничимся несколькими основными теоремами, для доказательства которых достаточны
методы, развитые в предшествующих главах. Для более основательного исследования естественным
и необходимым средством служит интеграл Лебега.
Сначала мы изучим более общие системы функций, обладающие свойством, аналогичным (61).
8.10 Определение. Пусть {𝜑𝑛 } (𝑛 = 1, 2, 3, … ) — последовательность комплексных функий на
[𝑎, 𝑏], такая, что
𝑏
(64)
∫
𝑎
𝜑𝑛 (𝑥)𝜑𝑚 (𝑥) 𝑑𝑥 = 0
(𝑛 ≠ 𝑚).
Тогда {𝜑𝑛 } называется ортогональной системой функций на [𝑎, 𝑏]. Если, кроме того,
𝑏
(65)
∫
𝑎
|𝜑𝑛 (𝑥)|2 𝑑𝑥 = 1
при всех 𝑛, то система {𝜑𝑛 } называется ортонормальной.
Например, функции (2𝜋)
щественные функции
−
1
2 𝑒𝑖𝑛𝑥
образуют ортонормальную систему на [−𝜋, 𝜋]. Таковы же и ве-
cos 𝑥 sin 𝑥 cos 2𝑥 sin 2𝑥
,
,
,
,⋯ .
√2𝜋 √𝜋 √𝜋 √𝜋
√𝜋
1
,
Если {𝜑𝑛 } — ортонормальная система на [𝑎, 𝑏] и если
𝑏
(66)
𝑐𝑛 =
∫
𝑎
𝑓 (𝑡)𝜑𝑛 (𝑡) 𝑑𝑡
(𝑛 = 1, 2, 3, … ),
то мы будем называть 𝑐𝑛 𝑛-ым коэффициентом Фурье функции 𝑓 относительно системы {𝜑𝑛 }. Мы
будем писать
∞
(67)
𝑓 (𝑥) ∼
∑
𝑐𝑛 𝜑𝑛 (𝑥)
1
и будем называть этот ряд рядом Фурье функции 𝑓 (относительно системы {𝜑𝑛 }).
139
Заметим, что, употребляя в (67) символ ∼, мы ничего не предполагаем о сходимости ряда; этот
символ означает только, что коэффициенты задаются (66).
Следующие теоремы показывают, что частные суммы ряда Фурье функции 𝑓 обладают некоторым свойством минимальности. Мы будем предполагать здесь, как и на потяжении всей остальной
части главы, что 𝑓 ∈ ℛ, хотя это условие может быть ослаблено.
8.11 Теорема. Пусть система {𝜑𝑛 } ортонормальна на [𝑎, 𝑏]. Пусть
𝑛
(68)
𝑠𝑛 (𝑥) =
∑
𝑐𝑚 𝜑𝑚 (𝑥)
𝑚=1
есть 𝑛-ая частная сумма ряда Фурье функции 𝑓 , и пусть
𝑛
(69)
𝑡𝑛 (𝑥) =
∑
𝛾𝑚 𝜑𝑚 (𝑥).
𝑚=1
Тогда
𝑏
(70)
∫
𝑎
|𝑓 − 𝑠𝑛 |2 𝑑𝑥 ≤
𝑏
∫
𝑎
|𝑓 − 𝑡𝑛 |2 𝑑𝑥,
и равенство имеет место тогда и только тогда, когда
(71)
𝛾𝑚 = 𝑐 𝑚
(𝑚 = 1, … , 𝑛).
Иначе говоря, среди всех функций 𝑡𝑛 функция 𝑠𝑛 дает наилучшее среднеквадратичное приближение к 𝑓 .
Доказательство. Пусть ∫ обозначает интеграл по [𝑎, 𝑏], ∑ — сумму от 1 до 𝑛. Тогда
∫
𝑓 𝑡𝑛̄ =
∫
𝑓
𝛾𝑚̄ 𝜑̄ 𝑚 =
∑
𝛾𝑚 𝜑 𝑚
∑
𝛾𝑘̄ 𝜑̄ 𝑘 =
∫
𝑓 𝑡𝑛̄ −
∫
𝑓 𝑡̄ 𝑛 +
∑
𝑐𝑚 𝛾𝑚̄ −
∑
|𝑐𝑚 |2 +
∑
𝑐𝑚 𝛾𝑚̄
по определению {𝑐𝑚 },
∫
|𝑡𝑛 |2 =
∫
𝑡𝑛 𝑡𝑛̄ =
∫∑
∑
|𝛾𝑚 |2 ,
так как система {𝜑𝑚 } ортонормальна, и поэтому
∫
|𝑓 − 𝑡𝑛 |2 =
=
=
∫
∫
∫
|𝑓 |2 −
|𝑓 |2 −
|𝑓 |2 −
∫
|𝑡𝑛 |2
∑
𝑐𝑚̄ 𝛾𝑚 +
∑
∑
|𝛾𝑚 − 𝑐𝑚 |2 ,
𝛾𝑚 𝛾𝑚̄
что, очевидно, достигает минимума тогда и только тогда, когда 𝛾𝑚 = 𝑐𝑚 .
Если в этой выкладке считать 𝛾𝑚 = 𝑐𝑚 , то мы получим
𝑏
(72)
∫
𝑎
𝑛
|𝑠𝑛 (𝑥)|2 𝑑𝑥 =
∑
|𝑐𝑚 |2 ≤
1
𝑏
∫
𝑎
|𝑓 (𝑥)|2 𝑑𝑥,
так как ∫ |𝑓 − 𝑡𝑛 |2 ≥ 0.
8.12 Теорема. Если {𝜑𝑛 } — ортонормальная система на [𝑎, 𝑏] и если
∞
𝑓 (𝑥) ∼
∑
𝑐𝑛 𝜑𝑛 (𝑥),
𝑛=1
140
то
∞
(73)
∑
|𝑐𝑛 |2 ≤
𝑛=1
𝑏
∫
𝑎
|𝑓 (𝑥)|2 𝑑𝑥.
В частности,
(74)
lim 𝑐
𝑛→∞ 𝑛
= 0.
Доказательство. Устремляя 𝑛 к ∞ в (72), мы получаем (73) — так называемое «неравенство Бесселя».
8.13 Тригонометрические ряды. Начиная с этого места мы будем иметь дело только с тригонометрической системой. Мы будем рассматривать функции 𝑓 , имеющие период 2𝜋 и интегрируемые
по Риману на [−𝜋, 𝜋] (значит, и на любом ограниченном сегменте). В этом случае ряд Фурье функции 𝑓 — ряд (63), коэффициенты которого 𝑐𝑛 определяются интегралами (62), и
𝑁
(75)
𝑠𝑁 (𝑥) = 𝑠𝑁 (𝑓 ; 𝑥) =
∑
𝑐𝑛 𝑒𝑖𝑛𝑥
−𝑁
есть 𝑁-ая частная сумма ряда Фурье функции 𝑓 . Неравенство (72) теперь принимает вид
𝑁
𝜋
𝜋
1
1
|𝑐 |2 ≤
|𝑠𝑁 (𝑥)|2 𝑑𝑥 =
|𝑓 (𝑥)|2 𝑑𝑥.
∑ 𝑛
∫
2𝜋 ∫
2𝜋
−𝜋
−𝜋
−𝑁
(76)
Чтобы получить выражение для 𝑠𝑁 , более удобное, чем (75), мы определим ядро Дирихле как
𝑁
(77)
𝐷𝑁 (𝑥) =
∑
𝑒
𝑖𝑛𝑥
=
𝑛=−𝑁
sin(𝑁 + 12 )𝑥
sin(𝑥/2)
.
Первое из этих равенств — определение 𝐷𝑁 (𝑥). Второе получается, если обе части тождества
(𝑒𝑖𝑥 − 1)𝐷𝑁 (𝑥) = 𝑒𝑖(𝑁+1)𝑥 − 𝑒−𝑖𝑁𝑥
умножить на 𝑒−𝑖𝑥/2 .
Из (62) и (75) следует
𝑁
𝑠𝑁 (𝑓 ; 𝑥) =
𝜋
1
𝑓 (𝑡)𝑒−𝑖𝑛𝑡 𝑑𝑡 𝑒𝑖𝑛𝑥
∑ 2𝜋 ∫
−𝜋
−𝑁
𝜋
=
𝑁
1
𝑓 (𝑡)
𝑒𝑖𝑛(𝑥−𝑡) 𝑑𝑡
∑
2𝜋 ∫
−𝜋
−𝑁
так что
𝜋
(78)
𝑠𝑁 (𝑓 ; 𝑥) =
𝜋
1
1
𝑓 (𝑡)𝐷𝑁 (𝑥 − 𝑡) 𝑑𝑡 =
𝑓 (𝑥 − 𝑡)𝐷𝑁 (𝑡) 𝑑𝑡.
2𝜋 ∫
2𝜋 ∫
−𝜋
−𝜋
Вследствие периодичности всех входящих функций безразлично, по какому интервалу мы будем
интегрировать, лишь бы его длина была равна 2𝜋. Это показывает, что два интеграла в (78) равны.
Мы докажем всего одну теорему о поточечной сходимости ряда Фурье.
8.14 Теорема. Если при некотором 𝑥 существуют постоянные 𝛿 > 0 и 𝑀 < ∞, такие, что
(79)
|𝑓 (𝑥 + 𝑡) − 𝑓 (𝑥)| ≤ 𝑀|𝑡|
при всех 𝑡 ∈ (−𝛿, 𝛿), то
(80)
lim 𝑠𝑁 (𝑓 ; 𝑥) = 𝑓 (𝑥).
𝑁→∞
141
Доказательство. Положим
(81)
𝑔(𝑡) =
𝑓 (𝑥 − 𝑡) − 𝑓 (𝑥)
sin(𝑡/2)
при 0 < |𝑡| ≤ 𝜋 и 𝑔(0) = 0. По определению (77)
𝜋
1
𝐷𝑁 (𝑥) 𝑑𝑥 = 1.
2𝜋 ∫
−𝜋
Значит, (78) показывает, что
𝜋
1
1
𝑔(𝑡) sin (𝑁 + ) 𝑡 𝑑𝑡
2𝜋 ∫
2
−𝜋
𝜋
𝜋
1
1
𝑡
𝑡
=
𝑔(𝑡) cos ] sin 𝑁𝑡 𝑑𝑡 +
[
[𝑔(𝑡) sin 2 ] cos 𝑁𝑡 𝑑𝑡.
∫
2𝜋 ∫
2
2𝜋
−𝜋
−𝜋
𝑠𝑁 (𝑓 ; 𝑥) − 𝑓 (𝑥) =
Ввиду (79) и (81), 𝑔(𝑡) cos(𝑡/2) и 𝑔(𝑡) sin(𝑡/2) ограничены. Последние два интеграла стремятся к 0 при
𝑁 → ∞ вследствие (74). Утверждение (80) доказано.
Следствие. Если 𝑓 (𝑥) = 0 при всех 𝑥 на некотором интервале 𝐽 , то lim 𝑠𝑁 (𝑓 ; 𝑥) = 0 при каждом
𝑥 ∈ 𝐽.
Вот еще одна формулировка этого следствия.
Если 𝑓 (𝑡) = 𝑔(𝑡) при всех 𝑡 в некоторой окрестности 𝑥, то
𝑠𝑁 (𝑓 ; 𝑥) − 𝑠𝑁 (𝑔; 𝑥) = 𝑠𝑁 (𝑓 − 𝑔, 𝑥) → 0 при 𝑁 → ∞.
Обычно это утверждение называют теоремой о локализации. Оно показывает, что поведение
последовательности {𝑠𝑁 (𝑓 ; 𝑥)}, когда речь идет о сходимости, зависит только от значений функции
𝑓 в некоторой (произвольно малой) окрестности точки 𝑥. Таким образом, два ряда Фурье могут
вести себя одинаково на одном промежутке, а на некотором другом промежутке вести себя совершенно по-разному. Мы сталкиваемся здесь с замечательным контрастом между рядами Фурье и
степенными рядами (теорема 8.5).
Мы завершим раздел еще двумя теоремами о приближении.
8.15 Теорема. Если 𝑓 непрерывна (с периодом 2𝜋) и если 𝜀 > 0, то существует тригонометрический многочлен 𝑃 , такой, что
|𝑃 (𝑥) − 𝑓 (𝑥)| < 𝜀
при всех вещественных 𝑥.
Доказательство. Отождествляя 𝑥 и 𝑥 + 2𝜋, мы можем считать 2𝜋-периодические функции на 𝑅1
функциями на единичной окружности 𝑇 с помощью отображения 𝑥 → 𝑒𝑖𝑥 . Тригонометрические
многочлены, т. е. функции вида (60), образуют самосопряженную алгебру 𝒜 , которая разделяет
точки на множестве 𝑇 и не исчезает ни в одной точке множества 𝑇 . Ввиду того что 𝑇 компактно,
теорема 7.33 показывает, что 𝒜 всюду плотна в 𝒞 (𝑇 ). Это в точности утверждение теоремы.
Более точная форма этой теоремы доказывается в упражнении 15.
8.16 Теорема Парсеваля. Пусть 𝑓 и 𝑔 — интегрируемые по Риману функции с периодом 2𝜋 и
∞
(82)
𝑓 (𝑥) ∼
∑
∞
𝑐𝑛 𝑒𝑖𝑛𝑥 ,
𝑔(𝑥) ∼
−∞
∑
𝛾𝑛 𝑒𝑖𝑛𝑥 .
−∞
Тогда
𝜋
(83)
1
|𝑓 (𝑥) − 𝑠𝑁 (𝑓 ; 𝑥)|2 𝑑𝑥 = 0,
𝑁→∞ 2𝜋 ∫
−𝜋
lim
∞
𝜋
(84)
1
𝑓 (𝑥)𝑔(𝑥) 𝑑𝑥 =
𝑐 𝛾̄ ,
∑ 𝑛 𝑛
2𝜋 ∫
−𝜋
−∞
(85)
1
|𝑓 (𝑥)|2 𝑑𝑥 =
|𝑐 |2 .
∑ 𝑛
2𝜋 ∫
−𝜋
−∞
𝜋
142
∞
Доказательство. Мы будем использовать обозначение
𝜋
(86)
‖ℎ‖2 =
1
|ℎ(𝑥)|2 𝑑𝑥
}
{ 2𝜋 ∫
−𝜋
1/2
.
Пусть дано 𝜀 > 0. Так как 𝑓 ∈ ℛ и 𝑓 (𝜋) = 𝑓 (−𝜋), то построение, описанное в упражнении 12 гл.
6, дает непрерывную 2𝜋-периодическую функцию ℎ, такую, что
(87)
‖𝑓 − ℎ‖2 < 𝜀.
По теореме 8.15 существует тригонометрический многочлен 𝑃 , такой, что |ℎ(𝑥) − 𝑃 (𝑥)| < 𝜀 при
всех 𝑥. Значит, ‖ℎ − 𝑃 ‖2 < 𝜀. Если 𝑃 имеет степень 𝑁0 , то теорема 8.11 показывает, что
(88)
‖ℎ − 𝑠𝑁 (ℎ)‖2 ≤ ‖ℎ − 𝑃 ‖2 < 𝜀
при всех 𝑁 ≥ 𝑁0 . Ввиду (72) с заменой 𝑓 на ℎ − 𝑓 ,
(89)
‖𝑠𝑁 (ℎ) − 𝑠𝑁 (𝑓 )‖2 = ‖𝑠𝑁 (ℎ − 𝑓 )‖2 ≤ ‖ℎ − 𝑓 ‖2 < 𝜀.
Теперь неравенство треугольника (упражнение 11 гл. 6) вместе с (87), (88) и (89) показывает,
что
(90)
‖𝑓 − 𝑠𝑁 (𝑓 )‖2 < 3𝜀
(𝑁 ≥ 𝑁0 ).
Утверждение (83) доказано. Далее,
𝑁
𝜋
𝜋
𝑁
1
1
𝑠𝑁 (𝑓 )𝑔 ̄ 𝑑𝑥 =
𝑐
𝑒𝑖𝑛𝑥 𝑔(𝑥) 𝑑𝑥 =
𝑐 𝛾̄ ,
∑ 𝑛 2𝜋 ∫
∑ 𝑛 𝑛
2𝜋 ∫
−𝜋
−𝜋
−𝑁
−𝑁
(91)
и неравенство Шварца показывает, что
1/2
(92)
|∫
𝑓 𝑔̄ −
𝑠 (𝑓 )𝑔 ̄ ≤
|𝑓 − 𝑠𝑁 (𝑓 )||𝑔| ≤
|𝑓 − 𝑠𝑁 |2
|𝑔|2
}
| ∫
{∫
∫ 𝑁
∫
,
что стремится к 0 при 𝑁 → ∞, согласно (83). Сравнивая (91) и (92), получаем (84). Наконец, (85) —
это частный случай (84) при 𝑔 = 𝑓 .
В главе 11 будет доказано обобщение теоремы 8.16.
Гамма-функция
Эта функция тесно связана с факториалом и неожиданно обнаруживается во многих разделах
математического анализа. Ее происхождение, история и развитие очень хорошо описаны в интересной статье P. J. Davis (Amer. Math. Monthly, vol. 66, 1959, pp. 849-869). Книга Артина (см. список
литературы) — еще одно хорошее элементарное введение.
Наше изложение будет очень сжатым, с минимумом комментариев к каждой теореме. Этот раздел, таким образом, можно рассматривать как большое упражнение и как возможность применить
часть рассмотренного до сих пор материала.
8.17 Определение. При 0 < 𝑥 < ∞
∞
(93)
Γ(𝑥) =
∫
0
𝑡𝑥−1 𝑒−𝑡 𝑑𝑡.
Этот интеграл сходится при данных 𝑥 (если 𝑥 < 1, то внимания требуют как 0, так и ∞).
8.18 Теорема.
(a) Функциональное уравнение
Γ(𝑥 + 1) = 𝑥Γ(𝑥)
выполняется при 0 < 𝑥 < ∞.
143
(b) Γ(𝑛 + 1) = 𝑛! при 𝑛 = 1, 2, 3, … .
(c) Функция log Γ выпукла на (0, ∞).
Доказательство. Утверждение (a) доказывается интегрированием по частям. Ввиду того что
Γ(1) = 1, из (a) по индукции следует (b). Если 1 < 𝑝 < ∞ и (1/𝑝) + (1/𝑞) = 1, то, применяя неравенство
Гельдера (упражнение 10 гл. 6) к (93), получаем
Γ
𝑥 𝑦
+
≤ Γ(𝑥)1/𝑝 Γ(𝑦)1/𝑞 .
(𝑝 𝑞)
Это равносильно (c).
Бором и Моллерупом открыт удивительный факт, заключающийся в том, что эти три свойства
полностью характеризуют функцию Γ.
8.19 Теорема. Если 𝑓 — положительная функция на (0, ∞), такая, что
(a) 𝑓 (𝑥 + 1) = 𝑥𝑓 (𝑥),
(b) 𝑓 (1) = 1,
(c) функция log 𝑓 выпукла,
то 𝑓 (𝑥) = Γ(𝑥).
Доказательство. Ввиду того что Γ удовлетворяет свойствам (a), (b) и (c), достаточно доказать,
что 𝑓 (𝑥) однозначно определяется условиями (a), (b), (c) при всех 𝑥 > 0. Благодаря (a), это достаточно сделать при 𝑥 ∈ (0, 1).
Положим 𝜑 = log 𝑓 . Тогда
(94)
𝜑(𝑥 + 1) = 𝜑(𝑥) + log 𝑥
(0 < 𝑥 < ∞),
𝜑(1) = 0 и 𝜑 выпукла. Пусть 0 < 𝑥 < 1, а 𝑛 — положительное целое число. Ввиду (94), 𝜑(𝑛 + 1) =
log(𝑛!). Рассмотрим разностные отношения функции 𝜑 на сегментах [𝑛, 𝑛 + 1], [𝑛 + 1, 𝑛 + 1 + 𝑥],
[𝑛 + 1, 𝑛 + 2]. Так как 𝜑 выпукла,
log 𝑛 ≤
𝜑(𝑛 + 1 + 𝑥) − 𝜑(𝑛 + 1)
≤ log(𝑛 + 1).
𝑥
Повторное применение (94) дает
𝜑(𝑛 + 1 + 𝑥) = 𝜑(𝑥) + log[𝑥(𝑥 + 1) ⋯ (𝑥 + 𝑛)].
Следовательно,
0 ≤ 𝜑(𝑥) − log
𝑛!𝑛𝑥
1
≤ 𝑥 log (1 + ) .
[ 𝑥(𝑥 + 1) ⋯ (𝑥 + 𝑛) ]
𝑛
Последнее выражение стремится к 0 при 𝑛 → ∞. Значит, 𝜑(𝑥) определена, и теорема доказана.
Как побочный результат мы получили соотношение
(95)
𝑛!𝑛𝑥
,
𝑛→∞ 𝑥(𝑥 + 1) ⋯ (𝑥 + 𝑛)
Γ(𝑥) = lim
по крайней мере если 0 < 𝑥 < 1; из него можно сделать вывод о том, что (95) выполняется при всех
𝑥 > 0, так как Γ(𝑥 + 1) = 𝑥Γ(𝑥).
8.20 Теорема. Если 𝑥 > 0 и 𝑦 > 0, то
1
(96)
∫
0
𝑡𝑥−1 (1 − 𝑡)𝑦−1 𝑑𝑡 =
Γ(𝑥)Γ(𝑦)
.
Γ(𝑥 + 𝑦)
Этот интеграл — так называемая бета-функция 𝐵(𝑥, 𝑦).
144
Доказательство. Отметим, что 𝐵(1, 𝑦) = 1/𝑦, что log 𝐵(𝑥, 𝑦) — выпуклая функция от 𝑥 при каждом
фиксированном 𝑦 (это следует из неравенства Гёльдера, как в теореме 8.18), и что
(97)
𝐵(𝑥 + 1, 𝑦) =
𝑥
𝐵(𝑥, 𝑦).
𝑥+𝑦
Чтобы доказать (97), достаточно проинтегрировать по частям
1
𝐵(𝑥 + 1, 𝑦) =
𝑥
𝑡
(1 − 𝑡)𝑥+𝑦−1 𝑑𝑡.
(
)
∫
1
−
𝑡
0
Эти три свойства функции 𝐵(𝑥, 𝑦) показывают, что при каждом 𝑦 теорема 8.19 применима к функции 𝑓 , определяемой выражением
Γ(𝑥 + 𝑦)
𝑓 (𝑥) =
𝐵(𝑥, 𝑦).
Γ(𝑦)
Следовательно, 𝑓 (𝑥) = Γ(𝑥).
8.21 Некоторые следствия. Подстановка 𝑡 = sin2 𝜃 превращает (96) в
𝜋/2
(98)
2
Специальный случай 𝑥 = 𝑦 =
1
2
∫
0
(sin 𝜃)2𝑥−1 (cos 𝜃)2𝑦−1 𝑑𝜃 =
Γ(𝑥)Γ(𝑦)
.
Γ(𝑥 + 𝑦)
дает
Γ( 12 ) = √𝜋.
(99)
Подстановка 𝑡 = 𝑠2 превращает (93) в
∞
(100)
Специальный случай 𝑥 =
Γ(𝑥) = 2
1
2
∫
0
2
𝑠2𝑥−1 𝑒−𝑠 𝑑𝑠
(0 < 𝑥 < ∞).
дает
∞
(101)
∫
−∞
2
𝑒−𝑠 𝑑𝑠 = √𝜋.
В силу (99) тождество
(102)
Γ(𝑥) =
2𝑥−1
√𝜋
𝑥
𝑥+1
Γ( )Γ(
2
2 )
следует непосредственно из теоремы 8.19.
8.22 Формула Стирлинга. Из вышеизложенного следует простое приближенное выражение для
Γ(𝑥 + 1) при больших 𝑥 (следовательно, и для 𝑛! при больших 𝑛). Формула такова:
(103)
lim
𝑥→∞
Γ(𝑥 + 1)
(𝑥/𝑒)𝑥 √2𝜋𝑥
= 1.
Доказательство следующее. Положим 𝑡 = 𝑥(1 + 𝑢) в (93). Это приводит к
(104)
Γ(𝑥 + 1) = 𝑥𝑥+1 𝑒−𝑥
∞
∫
−1
[(1 + 𝑢)𝑒−𝑢 ]𝑥 𝑑𝑢.
Определим ℎ(𝑢) так, чтобы ℎ(0) = 1 и
(105)
𝑢2
(1 + 𝑢)𝑒−𝑢 = exp − ℎ(𝑢) ,
]
[ 2
145
если −1 < 𝑢 < ∞, 𝑢 ≠ 0. Тогда
(106)
2
[𝑢 − log(1 + 𝑢)].
𝑢2
ℎ(𝑢) =
Из этого следует, что ℎ непрерывна и что ℎ(𝑢) монотонно убывает от ∞ до 0 при возрастании 𝑢 от
−1 до ∞.
Подстановка 𝑢 = 𝑠√2/𝑥 превращает (104) в
∞
Γ(𝑥 + 1) = 𝑥𝑥 𝑒−𝑥 √2𝑥
(107)
∫
−∞
𝜓𝑥 (𝑠) 𝑑𝑠,
где
𝜓𝑥 (𝑠) =
exp[−𝑠2 ℎ(𝑠√2/𝑥)]
(−√𝑥/2 < 𝑠 < ∞),
{0
(𝑠 ≤ −√𝑥/2).
Отметим следующие факты о 𝜓𝑥 (𝑠):
(a)
(b)
(c)
(d)
(e)
2
Для каждого 𝑠 имеем 𝜓𝑥 (𝑠) → 𝑒−𝑠 при 𝑥 → ∞.
Сходимость в (a) равномерная на [−𝐴, 𝐴] при любом 𝐴 < ∞.
2
Если 𝑠 < 0, то 0 < 𝜓𝑥 (𝑠) < 𝑒−𝑥 .
Если 𝑠 > 0 и 𝑥 > 1, то 0 < 𝜓𝑥 (𝑠) < 𝜓1 (𝑠).
∞
∫0 𝜓1 (𝑠) 𝑑𝑠 < ∞.
Следовательно, к интегралу (107) может быть применена теорема о сходимости, сформулированная в упражнении 12 гл. 7, и она показывает, что этот интеграл сходится к √𝜋 при 𝑥 → ∞ в
силу (101). Тем самым доказано (103).
Более детальная версия этого доказательства может быть найдена в книге Р. К. Бака «Advanced
Calculus», стр. 216-218. Два других, полностью различных, доказательства см. в статье W. Feller
в Amer. Math. Monthly, vol. 74, 1967, pp. 1223-1225 (с исправлением в vol. 75, 1968, p. 518) и стр.
20-24 книги Артина.
Упражнение 20 содержит более простое доказательство менее точного результата.
Упражнения
1. Пусть
𝑓 (𝑥) =
𝑒−1/𝑥
{0
2
(𝑥 ≠ 0),
(𝑥 = 0).
Доказать, что 𝑓 имеет производные всех порядков в 𝑥 = 0 и что 𝑓 (𝑛) (0) = 0 при 𝑛 = 1, 2, 3, … .
2. Пусть 𝑎𝑖𝑗 — число, стоящее в 𝑖-ой строке и 𝑗-ом столбце таблицы
−1
0
1
2
1
4
1
8
0
0
⋯
−1
0
0
⋯
1
2
1
4
−1
0
⋯
1
2
−1 ⋯
....................... ,
так что
⎧0
⎪
𝑎𝑖𝑗 = ⎨−1
⎪2𝑗−𝑖
⎩
(𝑖 < 𝑗),
(𝑖 = 𝑗),
(𝑖 > 𝑗).
𝑎𝑖𝑗 = −2,
∑∑
Доказать, что
∑∑
𝑖
𝑗
𝑗
𝑎𝑖𝑗 = 0.
𝑖
3. Доказать, что
∑∑
𝑖
𝑎𝑖𝑗 =
𝑗
∑∑
𝑗
𝑎𝑖𝑗 ,
𝑖
если 𝑎𝑖𝑗 ≥ 0 при всех 𝑖 и 𝑗 (случай +∞ = +∞ не исключается).
146
4. Доказать следующие предельные соотношения:
𝑏𝑥 − 1
(a) lim
= log 𝑏
(𝑏 > 0);
𝑥→0
𝑥
log(1 + 𝑥)
(b) lim
= 1;
𝑥→0
𝑥
1/𝑥
(c) lim(1 + 𝑥) = 𝑒;
𝑥→0
𝑥 𝑛
(d) lim (1 + ) = 𝑒𝑥 .
𝑛→∞
𝑛
5. Найти следующие пределы:
𝑒 − (1 + 𝑥)1/𝑥
;
𝑥→0
𝑥
𝑛
1/𝑛
(b) lim
[𝑛 − 1];
𝑥→0 log 𝑛
tg 𝑥 − 𝑥
(c) lim
;
𝑥→0 𝑥(1 − cos 𝑥)
𝑥 − sin 𝑥
(d) lim
.
𝑥→0 tg 𝑥 − 𝑥
6. Пусть 𝑓 (𝑥)𝑓 (𝑦) = 𝑓 (𝑥 + 𝑦) при всех вещественных 𝑥 и 𝑦.
(a) lim
(a) Предполагая, что 𝑓 дифференцируема и не равна нулю, доказать, что
𝑓 (𝑥) = 𝑒𝑐𝑥 ,
где 𝑐 — некоторая постоянная.
(b) Доказать то же самое, предполагая только, что 𝑓 непрерывна.
𝜋
7. Пусть 0 < 𝑥 < . Доказать, что
2
2
sin 𝑥
<
< 1.
𝜋
𝑥
8. Доказать, что при 𝑛 = 0, 1, 2, … и вещественном 𝑥
| sin 𝑛𝑥| ≤ 𝑛| sin 𝑥|.
Заметим, что это неравенство может быть неверным при других значениях 𝑛. Например,
| sin 12 𝜋| > 12 | sin 𝜋|.
9.
(a) Положим 𝑠𝑁 = 1 + ( 12 ) + ⋯ + (1/𝑁). Доказать, что предел
lim (𝑠𝑁 − log 𝑁)
𝑁→∞
существует. (Этот предел, часто обозначаемый 𝛾, называется постоянной Эйлера. Его численное
значение равно 0,5772 … . Неизвестно, рационально 𝛾 или нет.)
(b) Какой примерно величины должно быть 𝑚, чтобы 𝑁 = 10𝑚 удовлетворяло неравенству 𝑠𝑁 > 100?
10. Доказать, что ряд ∑ 1/𝑝, где суммирование идет по всем простым числам, расходится.
(Это показывает, что простые числа образуют весьма существенное подмножество положительных целых чисел.)
Указание. Пусть дано 𝑁. Пусть 𝑝1 , … , 𝑝𝑘 — те простые числа, которые делят хотя бы одно целое число
≤ 𝑁. Тогда
𝑘
𝑁
1
1
1
≤
1+
+
+⋯
∑ 𝑛 ∏(
𝑝𝑗 𝑝2𝑗
)
𝑛=1
𝑗=1
𝑘
=
∏(
1−
𝑗=1
1
𝑝𝑗 )
−1
𝑘
≤ exp
2
.
∑ 𝑝𝑗
𝑗=1
Последнее неравенство выполняется, так как
(1 − 𝑥)−1 ≤ 𝑒2𝑥 ,
если 0 ≤ 𝑥 ≤ 12 .
(Существует множество доказательств этого утверждения. См., например, статью I. Niven в Amer. Math.
Monthly, vol. 78, 1971, pp. 272-273, и статью R. Bellman в Amer. Math. Monthly, vol. 50, 1943, pp. 318-319.)
147
11. Пусть 𝑓 ∈ ℛ на [0, 𝐴] при всех 𝐴 < ∞ и 𝑓 (𝑥) → 1 при 𝑥 → +∞. Доказать, что
∞
lim 𝑡
𝑡→0
∫
0
𝑒−𝑡𝑥 𝑓 (𝑥) 𝑑𝑥 = 1
(𝑡 > 0).
12. Пусть 0 < 𝛿 < 𝜋, 𝑓 (𝑥) = 1, если |𝑥| ≤ 𝛿, 𝑓 (𝑥) = 0, если 𝛿 < |𝑥| ≤ 𝜋, и 𝑓 (𝑥 + 2𝜋) = 𝑓 (𝑥) при всех 𝑥.
(a) Вычислить коэффициенты Фурье функции 𝑓 .
(b) Заключить, что
∞
sin(𝑛𝛿)
𝜋−𝛿
=
∑ 𝑛
2
𝑛=1
(0 < 𝛿 < 𝜋).
(c) На основании теоремы Парсеваля прийти к выводу, что
∞
sin2 (𝑛𝛿)
𝜋−𝛿
=
.
∑ 𝑛2 𝛿
2
𝑛=1
(d) Устремить 𝛿 к 0 и доказать, что
∞
sin 𝑥 2
𝜋
( 𝑥 ) 𝑑𝑥 = 2 .
∫
0
(e) Положить 𝛿 = 𝜋/2 в (c). Что получается?
13. Положим 𝑓 (𝑥) = 𝑥, если 0 ≤ 𝑥 < 2𝜋. Применив теорему Парсеваля, заключить, что
∞
1
𝜋2
=
.
∑ 𝑛2
6
𝑛=1
14. Пусть 𝑓 (𝑥) = (𝜋 − |𝑥|)2 на [−𝜋, 𝜋]. Доказать, что
∞
𝑓 (𝑥) =
𝜋2
4
cos 𝑛𝑥
+
∑ 𝑛2
3
𝑛=1
и прийти к выводу, что
∞
∞
1
𝜋2
=
,
2
∑𝑛
6
𝑛=1
𝜋4
1
=
.
4
∑𝑛
90
𝑛=1
(Недавняя статья E. L. Stark содержит много ссылок на ряды вида ∑ 𝑛−𝑠 , где 𝑠 — положительное целое
число. См. Math. Mag., vol. 47, 1974, pp. 197-202.)
15. Используя определение (77) для 𝐷𝑛 , положим
𝑁
𝐾𝑁 (𝑥) =
Доказать, что
𝐾𝑁 (𝑥) =
и что
1
𝐷𝑛 (𝑥).
𝑁 +1 ∑
𝑛=0
1 − cos(𝑁 + 1)𝑥
1
⋅
𝑁 +1
1 − cos 𝑥
(a) 𝐾𝑁 ≥ 0,
𝜋
1
𝐾𝑁 (𝑥) 𝑑𝑥 = 1,
(b)
2𝜋 ∫
−𝜋
1
2
(c) 𝐾𝑁 (𝑥) ≤
⋅
,
если 0 < 𝛿 ≤ |𝑥| ≤ 𝜋.
𝑁 + 1 1 − cos 𝛿
Пусть 𝑠𝑁 = 𝑠𝑁 (𝑓 ; 𝑥) — 𝑁-ая частная сумма ряда Фурье функции 𝑓 . Рассмотрим средние арифметические
𝜎𝑁 =
𝑠0 + 𝑠 1 + ⋯ + 𝑠 𝑁
.
𝑁 +1
Доказать, что
𝜋
𝜎𝑁 (𝑓 ; 𝑥) =
1
𝑓 (𝑥 − 𝑡)𝐾𝑁 (𝑡) 𝑑𝑡,
2𝜋 ∫
−𝜋
и тем самым доказать теорему Фейера:
Если 𝑓 непрерывна и имеет период 2𝜋, то 𝜎𝑁 (𝑓 ; 𝑥) → 𝑓 (𝑥) равномерно на [−𝜋, 𝜋].
Указание. Использовать свойства (a), (b), (c) и рассуждать аналогично доказательству теоремы 7.26.
16. Доказать поточечный вариант теоремы Фейера:
148
Если 𝑓 ∈ ℛ и 𝑓 (𝑥+), 𝑓 (𝑥−) существуют при некотором 𝑥, то
lim 𝜎𝑁 (𝑓 ; 𝑥) = 12 [𝑓 (𝑥+) + 𝑓 (𝑥−)].
𝑁→∞
17. Пусть 𝑓 ограничена и монотонна на [−𝜋, 𝜋), и ее коэффициенты Фурье 𝑐𝑛 задаются равенством (62).
(a) Используя упражнение 17 гл. 6, доказать, что {𝑛𝑐𝑛 } — ограниченная последовательность.
(b) Используя (a) вместе с упражнениями 16 и 14(e) гл. 3, заключить, что
lim 𝑠𝑁 (𝑓 ; 𝑥) = 12 [𝑓 (𝑥+) + 𝑓 (𝑥−)]
𝑁→∞
при каждом 𝑥.
(c) Предполагая только то, что 𝑓 ∈ ℛ на [−𝜋, 𝜋] и что 𝑓 монотонна лишь на некотором интервале
(𝛼, 𝛽) ⊂ [−𝜋, 𝜋], доказать, что заключение части (b) верно при каждом 𝑥 ∈ (𝛼, 𝛽).
(Это применение теоремы о локализации.)
18. Положим
𝑓 (𝑥) = 𝑥3 − sin2 𝑥 tg 𝑥,
𝑔(𝑥) = 2𝑥2 − sin2 𝑥 − 𝑥 tg 𝑥.
Для каждой из этих двух функций определить, положительна ли она при всех 𝑥 ∈ (0, 𝜋/2), отрицательна
или меняет знак. Обосновать свой ответ.
19. Пусть 𝑓 — непрерывная функция на 𝑅1 , 𝑓 (𝑥 + 2𝜋) = 𝑓 (𝑥), а 𝛼/𝜋 иррационально. Доказать, что
𝑁
lim
𝑁→∞
𝜋
1
1
𝑓 (𝑥 + 𝑛𝛼) =
𝑓 (𝑡) 𝑑𝑡
𝑁∑
2𝜋 ∫
−𝜋
𝑛=1
при каждом 𝑥. Указание. Провести доказательство сначала для 𝑓 (𝑥) = 𝑒𝑖𝑘𝑥 .
20. Следующее простое вычисление дает хорошее приближение к формуле Стирлинга.
При 𝑚 = 1, 2, 3, … положим
𝑓 (𝑥) = (𝑚 + 1 − 𝑥) log 𝑚 + (𝑥 − 𝑚) log(𝑚 + 1),
если 𝑚 ≤ 𝑥 ≤ 𝑚 + 1, и положим
𝑔(𝑥) =
если 𝑚 −
и что
1
2
𝑥
− 1 + log 𝑚,
𝑚
≤ 𝑥 < 𝑚 + 12 . Нарисовать графики функций 𝑓 и 𝑔. Отметить, что 𝑓 (𝑥) ≤ log 𝑥 ≤ 𝑔(𝑥) при 𝑥 ≥ 1
𝑛
∫
1
𝑓 (𝑥) 𝑑𝑥 = log(𝑛!) − 12 log 𝑛 > − 18 +
𝑛
∫
1
𝑔(𝑥) 𝑑𝑥.
Проинтегрировать log 𝑥 по [1, 𝑛]. Заключить, что
7
8
< log(𝑛!) − (𝑛 + 21 ) log 𝑛 + 𝑛 < 1
при 𝑛 = 2, 3, 4, … (заметим: log √2𝜋 ∼ 0,918 … ). Таким образом,
𝑒7/8 <
𝑛!
(𝑛/𝑒)𝑛 √𝑛
< 𝑒.
21. Пусть
𝜋
𝐿𝑛 =
1
|𝐷𝑛 (𝑡)| 𝑑𝑡
2𝜋 ∫
−𝜋
(𝑛 = 1, 2, 3, … ).
Доказать, что существует постоянная 𝐶 > 0, такая, что
𝐿𝑛 > 𝐶 log 𝑛
(𝑛 = 1, 2, 3, … ),
или, точнее, что последовательность
4
{𝐿𝑛 − 𝜋 2 log 𝑛}
ограничена.
149
22. Пусть 𝛼 вещественно и −1 < 𝑥 < 1. Доказать биномиальную теорему Ньютона
∞
(1 + 𝑥)𝛼 = 1 +
∑
𝑛=1
𝛼(𝛼 − 1) ⋯ (𝛼 − 𝑛 + 1) 𝑛
𝑥.
𝑛!
Указание. Обозначим правую часть через 𝑓 (𝑥). Доказать, что ряд сходится. Доказать, что
(1 + 𝑥)𝑓 ′ (𝑥) = 𝛼𝑓 (𝑥)
и решить это дифференциальное уравнение.
Показать также, что
∞
(1 − 𝑥)−𝛼 =
Γ(𝑛 + 𝛼) 𝑛
𝑥,
∑ 𝑛!Γ(𝛼)
𝑛=0
если −1 < 𝑥 < 1 и 𝛼 > 0.
23. Пусть 𝛾 — непрерывно дифференцируемая замкнутая кривая на комплексной плоскости с сегментом
параметра [𝑎, 𝑏]. Допустим, что 𝛾(𝑡) ≠ 0 при всех 𝑡 ∈ [𝑎, 𝑏]. Положим по определению индекс кривой 𝛾
равным
𝑏 ′
𝛾 (𝑡)
1
𝑑𝑡.
Ind(𝛾) =
2𝜋𝑖 ∫
𝛾(𝑡)
𝑎
Доказать, что 𝐼𝑛𝑑(𝛾) всегда является целым числом.
Указание. Существует 𝜑 на [𝑎, 𝑏] с 𝜑′ = 𝛾 ′ /𝛾, 𝜑(𝑎) = 0. Значит, 𝛾 exp(−𝜑) постоянно. Ввиду того, что
𝛾(𝑎) = 𝛾(𝑏), из этого следует, что exp 𝜑(𝑏) = exp 𝜑(𝑎) = 1. Отметить, что 𝜑(𝑏) = 2𝜋𝑖 Ind(𝛾).
Вычислить Ind(𝛾), если 𝛾(𝑡) = 𝑒𝑖𝑛𝑡 , 𝑎 = 0, 𝑏 = 2𝜋.
Объяснить, почему Ind(𝛾) часто называют числом зацепления кривой 𝛾 относительно точки 0.
24. Пусть 𝛾 — кривая вида, описанного в упражнении 23. Допустим, кроме того, что область значений
кривой 𝛾 не пересекает отрицательную часть вещественной оси. Доказать, что Ind(𝛾) = 0. Указание.
При 0 ≤ 𝑐 < ∞ Ind(𝛾 + 𝑐) — непрерывная функция от 𝑐, принимающая лишь целочисленные значения.
Также Ind(𝛾 + 𝑐) → 0 при 𝑐 → ∞.
25. Пусть 𝛾1 и 𝛾2 — кривые вида, описанного в упражнении 23, и
|𝛾1 (𝑡) − 𝛾2 (𝑡)| < |𝛾1 (𝑡)|
(𝑎 ≤ 𝑡 ≤ 𝑏).
Доказать, что Ind(𝛾1 ) = Ind(𝛾2 ).
Указание. Положим 𝛾 = 𝛾2 /𝛾1 . Тогда |1 − 𝛾| < 1, значит, из упражнения 24 следует, что Ind(𝛾) = 0. Также
𝛾′ 𝛾′
𝛾′
= 2 − 1.
𝛾
𝛾2
𝛾1
26. Пусть 𝛾 — замкнутая кривая на комплексной плоскости (не обязательно дифференцируемая) с сегментом параметра [0, 2𝜋], такая, что 𝛾(𝑡) ≠ 0 при всех 𝑡 ∈ [0, 2𝜋].
Выбрать 𝛿 > 0, такое, что |𝛾(𝑡)| > 𝛿 при всех 𝑡 ∈ [0, 2𝜋]. Пусть 𝑃1 и 𝑃2 — тригонометрические многочлены,
такие, что |𝑃𝑗 (𝑡) − 𝛾(𝑡)| < 𝛿/4 при всех 𝑡 ∈ [0, 2𝜋] (их существование гарантируется теоремой 8.15).
Доказать, что
Ind(𝑃1 ) = Ind(𝑃2 ),
применив упражнение 25.
Положить по определению Ind(𝛾) равным их общему значению.
Доказать, что утверждения упражнений 24 и 25 сохраняются без какого-либо условия дифференцируемости.
27. Пусть 𝑓 — непрерывная комплексная функция, определенная на комплексной плоскости. Допустим, что
существуют положительное целое число 𝑛 и комплексное число 𝑐 ≠ 0, такие, что
lim 𝑧−𝑛 𝑓 (𝑧) = 𝑐.
|𝑧|→∞
Доказать, что 𝑓 (𝑧) = 0 хотя бы при одном комплексном значении 𝑧.
Отметим, что это обобщение теоремы 8.8.
Указание. Предположим, что 𝑓 (𝑧) ≠ 0 при всех 𝑧. Положим
𝛾𝑟 (𝑡) = 𝑓 (𝑟𝑒𝑖𝑡 )
при 0 ≤ 𝑟 < ∞, 0 ≤ 𝑡 ≤ 2𝜋. Доказать следующие утверждения о кривых 𝛾𝑟 :
(a) Ind(𝛾0 ) = 0;
(b) Ind(𝛾𝑟 ) = 𝑛 при всех достаточно больших 𝑟;
150
(c) Ind(𝛾𝑟 ) — непрерывная функция от 𝑟 на [0, ∞)
(в (b) и (c) использовать последнюю часть упражнения 26).
Показать, что (a), (b) и (c) противоречат друг другу, так как 𝑛 > 0.
28. Пусть 𝐷̄ — замкнутый единичный диск на комплексной плоскости (т. е. 𝑧 ∈ 𝐷̄ тогда и только тогда,
когда |𝑧| ≤ 1). Пусть 𝑔 — непрерывное отображение диска 𝐷̄ в единичную окружность 𝑇 (т. е. |𝑔(𝑧)| = 1
̄
при каждом 𝑧 ∈ 𝐷).
Доказать, что 𝑔(𝑧) = −𝑧 хотя бы при одном 𝑧 ∈ 𝑇 .
Указание. При 0 ≤ 𝑟 ≤ 1, 0 ≤ 𝑡 ≤ 2𝜋 положим
𝛾𝑟 (𝑡) = 𝑔(𝑟𝑒𝑖𝑡 )
и положим 𝜓(𝑡) = 𝑒−𝑖𝑡 𝛾1 (𝑡). Если 𝑔(𝑧) ≠ −𝑧 при каждом 𝑧 ∈ 𝑇 , то 𝜓(𝑡) ≠ −1 при каждом 𝑡 ∈ [0, 2𝜋]. Значит,
Ind(𝜓) = 0, что следует из упражнений 24 и 26. Следовательно, Ind(𝛾𝑖 ) = 1. Но Ind(𝛾0 ) = 0. Прийти к
противоречию, как в упражнении 27.
29. Доказать, что каждое непрерывное отображение 𝑓 из 𝐷̄ в 𝐷̄ имеет неподвижную точку на 𝐷.̄
(Это двумерный случай теоремы Брауэра о неподвижной точке.)
Указание. Предположим, что 𝑓 (𝑧) ≠ 𝑧 при каждом 𝑧 ∈ 𝐷.̄ Свяжем с каждой точкой 𝑧 ∈ 𝐷̄ точку 𝑔(𝑧) ∈ 𝑇 ,
которая лежит на луче, начинающемся в 𝑓 (𝑧) и проходящем через 𝑧. Тогда 𝑔 отображает 𝐷̄ в 𝑇 , 𝑔(𝑧) = 𝑧,
если 𝑧 ∈ 𝑇 , и 𝑔 непрерывно, так как
𝑔(𝑧) = 𝑧 − 𝑠(𝑧)[𝑓 (𝑧) − 𝑧],
где 𝑠(𝑧) — единственный неотрицательный корень некоторого квадратного уравнения, коэффициенты
которого — непрерывные функции от 𝑓 и 𝑧. Применить упражнение 28.
30. Используя формулу Стирлинга, доказать, что
Γ(𝑥 + 𝑐)
=1
𝑥𝑐 Γ(𝑥)
lim
𝑥→∞
при каждой вещественной постоянной 𝑐.
31. В доказательстве теоремы 7.26 было показано, что
1
∫
−1
(1 − 𝑥2 )𝑛 𝑑𝑥 ≥
4
3√𝑛
при 𝑛 = 1, 2, 3, … . Использовать теорему 8.20 и упражнение 30, чтобы получить более точный результат
1
lim √𝑛
𝑛→∞
∫
−1
(1 − 𝑥2 )𝑛 𝑑𝑥 = √𝜋.
151
Глава 9
Функции нескольких переменных
Линейные преобразования
Эту главу мы начнем с рассмотрения множеств векторов в евклидовом 𝑛-мерном пространстве 𝑅𝑛 .
Излагаемые здесь алгебраические факты без изменений переносятся на конечномерные векторные
пространства над любым полем скаляров. Однако для наших целей мы вполне можем оставаться
в привычных рамках евклидовых пространств.
9.1 Определения.
(a) Непустое множество 𝑋 ⊂ 𝑅𝑛 называется векторным пространством, если x + y ∈ 𝑋 и 𝑐x ∈ 𝑋
при любых x ∈ 𝑋, y ∈ 𝑋 и любых скалярах 𝑐.
(b) Если x1 , … , x𝑘 ∈ 𝑅𝑛 , а 𝑐1 , … , 𝑐𝑘 — скаляры, то вектор
𝑐1 x1 + ⋯ + 𝑐𝑘 x𝑘
называется линейной комбинацией векторов x1 , … , x𝑘 . Если 𝑆 ⊂ 𝑅𝑛 и если 𝐸 — множество
всех линейных комбинаций элементов множества 𝑆, то мы будем говорить, что 𝐸 натянуто
на 𝑆 или что 𝐸 — оболочка множества 𝑆.
Заметим, что каждая оболочка есть векторное пространство.
(c) Множество, состоящее из векторов x1 , … , x𝑘 (для такого множества мы будем использовать
обозначение {x1 , … , x𝑘 }), называется независимым, если из соотношения 𝑐1 x1 + ⋯ + 𝑐𝑘 x𝑘 = 0
следует, что 𝑐1 = ⋯ = 𝑐𝑘 = 0. В противном случае это множество называется зависимым.
Заметим, что независимое множество не может содержать нулевого вектора.
(d) Если векторное пространство 𝑋 содержит независимое множество, состоящее из 𝑟 векторов,
но не содержит никакого независимого множества из 𝑟 + 1 векторов, то говорят, что 𝑋 имеет
размерность 𝑟, и пишут dim 𝑋 = 𝑟.
Множество, состоящее из одного элемента 0, — векторное пространство; его размерность равна 0.
(e) Независимое подмножество векторного пространства 𝑋, на которое натянуто 𝑋, называется
базисом пространства 𝑋.
Заметим, что если 𝐵 = {x1 , … , x𝑟 } — базис пространства 𝑋, то каждый элемент x ∈ 𝑋 допускает единственное представление вида x = ∑ 𝑐𝑗 x𝑗 . Такое представление существует, так как
𝑋 натянуто на 𝐵, и единственно, так как 𝐵 независимо. Числа 𝑐1 , … , 𝑐𝑟 называются координатами вектора x относительно базиса 𝐵.
Наиболее известным примером базиса служит множество {e1 , … , e𝑛 }, где e𝑗 — вектор в 𝑅𝑛 ,
𝑗-ая координата которого равна 1, а прочие координаты равны 0. Если x ∈ 𝑅𝑛 , x = (𝑥1 , … , 𝑥𝑛 ),
то x = ∑ 𝑥𝑗 e𝑗 . Мы будем называть множество
{e1 , … , e𝑛 }
стандартным базисом пространства 𝑅𝑛 .
9.2 Теорема. Пусть 𝑟 — положительное целое число. Если векторное пространство 𝑋 натянуто
на множество, состоящее из 𝑟 векторов, то dim 𝑋 ≤ 𝑟.
152
Доказательство. Если это неверно, то существует векторное пространство 𝑋, содержащее независимое множество 𝑄 = {y1 , … , y𝑟+1 } и натянутое на множество 𝑆0 , состоящее из 𝑟 векторов.
Пусть 0 ≤ 𝑖 < 𝑟, и допустим, что построено множество 𝑆𝑖 , на которое натянуто 𝑋 и которое
состоит из всех y𝑗 с 1 ≤ 𝑗 ≤ 𝑖 и из некоторого набора 𝑟 − 𝑖 векторов множества 𝑆0 , скажем,
x1 , … , x𝑟−𝑖 (иными словами, 𝑆𝑖 получается из 𝑆0 заменой 𝑖 из его элементов элементами множества
𝑄 без изменения оболочки). Поскольку 𝑋 натянуто на 𝑆𝑖 , то y𝑖+1 принадлежит оболочке множества
𝑆𝑖 ; значит, существуют скаляры 𝑎1 , … , 𝑎𝑖+1 , 𝑏1 , … , 𝑏𝑟−𝑖 , где 𝑎𝑖+1 = 1, такие, что
𝑖+1
∑
𝑗=1
𝑟−𝑖
𝑎𝑗 y𝑗 +
∑
𝑏𝑘 x𝑘 = 0.
𝑘=1
Если бы все 𝑏𝑘 были равны 0, то, в силу независимости множества 𝑄, и все 𝑎𝑗 были бы нулями —
противоречие. Следовательно, некоторый вектор x𝑘 ∈ 𝑆𝑖 есть линейная комбинация других элементов множества 𝑇𝑖 = 𝑆𝑖 ∪ {y𝑖+1 }. Удалим этот вектор x𝑘 из 𝑇𝑖 и обозначим оставшееся множество
через 𝑆𝑖+1 . Тогда на 𝑆𝑖+1 натянуто то же пространство, что и на 𝑇𝑖 , то есть 𝑋, так что 𝑆𝑖+1 обладает
теми же свойствами, что и 𝑆𝑖 , с заменой 𝑖 на 𝑖 + 1.
Начиная с 𝑆0 , мы построим таким образом множества 𝑆1 , … , 𝑆𝑟 . Последнее из них состоит из
y1 , … , y𝑟 , и наше построение показывает, что его оболочка равна 𝑋. Но 𝑄 независимо, значит, y𝑟+1
не принадлежит оболочке множества 𝑆𝑟 . Это противоречие доказывает теорему.
Следствие. dim 𝑅𝑛 = 𝑛.
Доказательство. Ввиду того что 𝑅𝑛 натянуто на множество {e1 , … , e𝑛 }, теорема показывает, что
dim 𝑅𝑛 ≤ 𝑛. Но множество {e1 , … , e𝑛 } независимое, и потому dim 𝑅𝑛 ≥ 𝑛.
9.3 Теорема. Пусть 𝑋 — векторное пространство и dim 𝑋 = 𝑛.
(a) Пространство 𝑋 натянуто на множество 𝐸, состоящее из 𝑛 векторов, в том и только в
том случае, когда 𝐸 независимо.
(b) 𝑋 имеет базис, и каждый базис состоит из 𝑛 векторов.
(c) Если 1 ≤ 𝑟 ≤ 𝑛 и {y1 , … , y𝑟 } — независимое множество в 𝑋, то 𝑋 имеет базис, содержащий
{y1 , … , y𝑟 }.
Доказательство. Пусть 𝐸 = {x1 , … , x𝑛 }. Ввиду того что dim 𝑋 = 𝑛, множество {x1 , … , x𝑛 , y}
зависимо при любом y ∈ 𝑋. Если 𝐸 независимо, то y принадлежит оболочке множества 𝐸; значит,
𝑋 натянуто на 𝐸. Обратно, если 𝐸 зависимо, то один из его элементов можно удалить, не меняя
оболочки. Значит, 𝑋 не может быть натянуто на 𝐸 по теореме 9.2. Тем самым (a) доказано.
Ввиду того что dim 𝑋 = 𝑛, 𝑋 содержит независимое множество, состоящее из 𝑛 векторов, и,
согласно (a), каждое такое множество образует базис пространства 𝑋; теперь (b) вытекает из 9.1(d)
и 9.2.
Чтобы доказать (c), допустим, что {x1 , … , x𝑛 } — базис пространства 𝑋. 𝑋 натянуто на множество
𝑆 = {y1 , … , y𝑟 , x1 , … , x𝑛 },
причем 𝑆 зависимо, так как оно содержит больше, чем 𝑛 векторов. Рассуждение, использованное
при доказательстве теоремы 9.2, показывает, что один из векторов x𝑖 равен линейной комбинации
остальных элементов множества 𝑆. Удалив этот вектор x𝑖 из 𝑆, мы получим множество, оболочка
которого все еще совпадает с 𝑋. Повторяя этот процесс 𝑟 раз, мы получим базис пространства 𝑋,
содержащий {y1 , … , y𝑟 } согласно (a).
9.4 Определения. Отображение 𝐴 векторного пространства 𝑋 в векторное пространство 𝑌 называется линейным преобразованием, если
𝐴(x1 + x2 ) = 𝐴x1 + 𝐴x2 ,
𝐴(𝑐x) = 𝑐𝐴x
при любых x, x1 , x2 ∈ 𝑋 и любых скалярах 𝑐. Отметим, что часто пишут 𝐴x вместо 𝐴(x), если 𝐴
линейно.
153
Заметим, что 𝐴0 = 0, если 𝐴 линейно. Заметим еще, что линейное преобразование 𝐴 пространства 𝑋 в 𝑌 полностью определяется своим действием на любом базисе: если {x1 , … , x𝑛 } — базис
пространства 𝑋, то каждое x ∈ 𝑋 имеет единственное представление вида
𝑛
x=
∑
𝑐𝑖 x𝑖 ,
𝑖=1
и линейность преобразования 𝐴 позволяет найти 𝐴x, зная векторы 𝐴x1 , … , 𝐴x𝑛 и координаты
𝑐1 , … , 𝑐𝑛 , по формуле
𝑛
𝐴x =
∑
𝑐𝑖 𝐴x𝑖 .
𝑖=1
Линейные преобразования пространства 𝑋 в 𝑋 часто называют линейными операторами на 𝑋.
Если 𝐴 — линейный оператор на 𝑋, который (i) взаимно однозначен и (ii) отображает 𝑋 на 𝑋, то 𝐴
называют обратимым. В этом случае можно определить оператор 𝐴−1 на 𝑋, положив 𝐴−1 (𝐴x) = x
при всех x ∈ 𝑋. Очевидно, что в этом случае также 𝐴(𝐴−1 x) = x при всех x ∈ 𝑋 и 𝐴−1 линеен.
Важное свойство линейных операторов на конечномерных векторных пространствах состоит в
том, что каждое из условий (i) и (ii) влечет за собой другое.
9.5 Теорема. Линейный оператор 𝐴 на конечномерном векторном пространстве 𝑋 взаимно
однозначен в том и только в том случае, когда множество его значений совпадает со всем 𝑋.
Доказательство. Пусть {x1 , … , x𝑛 } — базис пространства 𝑋. Из линейности оператора 𝐴 следует,
что его множество значений ℛ(𝐴) натянуто на множество 𝑄 = {𝐴x1 , … , 𝐴x𝑛 }. Из теоремы 9.3(a)
мы заключаем, что ℛ(𝐴) = 𝑋 тогда и только тогда, когда 𝑄 независимо. Мы должны показать,
что это происходит тогда и только тогда, когда 𝐴 взаимно однозначен.
Допустим, что 𝐴 взаимно однозначен и ∑ 𝑐𝑖 𝐴x𝑖 = 0. Тогда 𝐴(∑ 𝑐𝑖 x𝑖 ) = 0, значит, ∑ 𝑐𝑖 x𝑖 = 0,
значит, 𝑐1 = ⋯ = 𝑐𝑛 = 0, и мы заключаем, что 𝑄 независимо.
Пусть, обратно, 𝑄 независимо и 𝐴(∑ 𝑐𝑖 x𝑖 ) = 0. Тогда ∑ 𝑐𝑖 𝐴x𝑖 = 0, значит, 𝑐1 = ⋯ = 𝑐𝑛 = 0, и мы
заключаем, что 𝐴x = 0 только тогда, когда x = 0. Если теперь 𝐴x = 𝐴y, то 𝐴(x − y) = 𝐴x − 𝐴y = 0,
так что x − y = 0, и потому 𝐴 взаимно однозначен.
9.6 Определения.
(a) Обозначим через 𝐿(𝑋, 𝑌 ) множество всех линейных отображений векторного пространства 𝑋
в векторное пространство 𝑌 . Вместо 𝐿(𝑋, 𝑋) мы будем писать просто 𝐿(𝑋). Если 𝐴1 , 𝐴2 ∈
𝐿(𝑋, 𝑌 ) и если 𝑐1 , 𝑐2 — скаляры, то определим 𝑐1 𝐴1 + 𝑐2 𝐴2 равенством
(𝑐! 𝐴1 + 𝑐2 𝐴2 )x = 𝑐1 𝐴1 x + 𝑐2 𝐴2 x
(x ∈ 𝑋).
Ясно, что тогда 𝑐1 𝐴1 + 𝑐2 𝐴2 ∈ 𝐿(𝑋, 𝑌 ).
(b) Если 𝑋, 𝑌 , 𝑍 — векторные пространства, 𝐴 ∈ 𝐿(𝑋, 𝑌 ) и 𝐵 ∈ 𝐿(𝑌 , 𝑍), то мы определим их
произведение 𝐵𝐴 равным композиции отображений 𝐴 и 𝐵:
(𝐵𝐴)x = 𝐵(𝐴x)
(x ∈ 𝑋).
Тогда 𝐵𝐴 ∈ 𝐿(𝑋, 𝑍).
Отметим, что 𝐵𝐴 может отличаться от 𝐴𝐵 даже в том случае, когда 𝑋 = 𝑌 = 𝑍.
(c) Нормой ‖𝐴‖ оператора 𝐴 ∈ 𝐿(𝑅𝑛 , 𝑅𝑚 ) называется верхняя грань всех чисел |𝐴x|, где x пробегает все векторы пространства 𝑅𝑛 , такие, что |x| ≤ 1.
Заметим, что неравенство
|𝐴x| ≤ ‖𝐴‖|x|
выполняется при всех x ∈ 𝑅𝑛 . Кроме того, если 𝜆 таково, что |𝐴x| ≤ 𝜆|x| при всех x ∈ 𝑅𝑛 , то
‖𝐴‖ ≤ 𝜆.
9.7 Теорема.
(a) Если 𝐴 ∈ 𝐿(𝑅𝑛 , 𝑅𝑚 ), то ‖𝐴‖ < ∞ и 𝐴 — равномерно непрерывное отображение пространства
𝑅𝑛 в 𝑅𝑚 .
154
(b) Если 𝐴, 𝐵 ∈ 𝐿(𝑅𝑛 , 𝑅𝑚 ), а 𝑐 — скаляр, то
‖𝐴 + 𝐵‖ ≤ ‖𝐴‖ + ‖𝐵‖,
‖𝑐𝐴‖ = |𝑐|‖𝐴‖.
Если расстояние между 𝐴 и 𝐵 определить как ‖𝐴 − 𝐵‖, то 𝐿(𝑅𝑛 , 𝑅𝑚 ) становится метрическим пространством.
(c) Если 𝐴 ∈ 𝐿(𝑅𝑛 , 𝑅𝑚 ) и 𝐵 ∈ 𝐿(𝑅𝑚 , 𝑅𝑘 ), то
‖𝐵𝐴‖ ≤ ‖𝐵‖‖𝐴‖.
Доказательство.
(a) Пусть {e1 , … , e𝑛 } — стандартный базис в 𝑅𝑛 , и пусть x = ∑ 𝑐𝑖 e𝑖 , |x| ≤ 1, так что |𝑐𝑖 | ≤ 1 при
𝑖 = 1, … , 𝑛. Тогда
|𝐴x| = | 𝑐𝑖 𝐴e𝑖 | ≤
|𝑐 ||𝐴e𝑖 | ≤
|𝐴e𝑖 |,
∑
∑ 𝑖
∑
так что
𝑛
‖𝐴‖ ≤
∑
|𝐴e𝑖 | < ∞.
𝑖=1
Ввиду того что |𝐴x−𝐴y| ≤ ‖𝐴‖|x−y| при x, y ∈ 𝑅𝑛 , мы видим, что 𝐴 равномерно непрерывно.
(b) Неравенство в (b) следует из неравенства
|(𝐴 + 𝐵)x| = |𝐴x + 𝐵x| ≤ |𝐴x| + |𝐵x| ≤ (‖𝐴‖ + ‖𝐵‖)|x|.
Вторая часть (b) проверяется тем же способом. Если
𝐴, 𝐵, 𝐶 ∈ 𝐿(𝑅𝑛 , 𝑅𝑚 ),
то выполняется неравенство треугольника
‖𝐴 − 𝐶‖ = ‖(𝐴 − 𝐵) + (𝐵 − 𝐶)‖ ≤ ‖𝐴 − 𝐵‖ + ‖𝐵 − 𝐶‖,
и легко проверить, что ‖𝐴 − 𝐵‖ обладает остальными свойствами метрики (определение 2.15).
(c) Наконец, (c) следует из неравенства
|(𝐵𝐴)x| = |𝐵(𝐴x)| ≤ ‖𝐵‖|𝐴x| ≤ ‖𝐵‖‖𝐴‖|x|.
Имея в пространствах 𝐿(𝑅𝑛 , 𝑅𝑚 ) метрику, мы можем перенести на эти пространства такие понятия, как открытое множество, непрерывность и т. д. Наша следующая теорема использует эти
понятия.
9.8 Теорема. Пусть Ω — множество всех обратимых линейных операторов на 𝑅𝑛 .
(a) Если 𝐴 ∈ Ω, 𝐵 ∈ 𝐿(𝑅𝑛 ) и
‖𝐵 − 𝐴‖ ⋅ ‖𝐴−1 ‖ < 1,
то 𝐵 ∈ Ω.
(b) Ω — открытое подмножество пространства 𝐿(𝑅𝑛 ), и отображение 𝐴 → 𝐴−1 непрерывно на
Ω (это отображение, очевидно, взаимно однозначно отображает Ω на Ω и является обратным
к самому себе).
Доказательство.
(a) Положим ‖𝐴−1 ‖ = 1/𝛼 и положим ‖𝐵 − 𝐴‖ = 𝛽. Тогда 𝛽 < 𝛼. При любом x ∈ 𝑅𝑛 выполняется
𝛼|x| = 𝛼|𝐴−1 𝐴x| ≤ 𝛼‖𝐴−1 ‖ ⋅ |𝐴x|
= |𝐴x| ≤ |(𝐴 − 𝐵)x| + |𝐵x| ≤ 𝛽|x| + |𝐵x|,
так что
(1)
(𝛼 − 𝛽)|x| ≤ |𝐵x|
(x ∈ 𝑅𝑛 ).
Ввиду того что 𝛼 − 𝛽 > 0, (1) показывает, что 𝐵x ≠ 0, если x ≠ 0. Значит, 𝐵 взаимно
однозначен. По теореме 9.5 имеем 𝐵 ∈ Ω. Это верно при любом 𝐵, для которого ‖𝐵 − 𝐴‖ < 𝛼.
Следовательно, доказано (a) и открытость множества Ω.
155
(b) Далее заменим x в (1) на 𝐵 −1 y. Полученное неравенство
(2)
(𝛼 − 𝛽)|𝐵 −1 y| ≤ |𝐵𝐵 −1 y| = |y|
(𝑦 ∈ 𝑅𝑛 )
показывает, что ‖𝐵 −1 ‖ ≤ (𝛼 − 𝛽)−1 . Тождество
𝐵 −1 − 𝐴−1 = 𝐵 −1 (𝐴 − 𝐵)𝐴−1
и теорема 9.7(c) показывают теперь, что
‖𝐵 −1 − 𝐴−1 ‖ ≤ ‖𝐵 −1 ‖‖𝐴 − 𝐵‖‖𝐴−1 ‖ ≤
𝛽
.
𝛼(𝛼 − 𝛽)
Тем самым доказано утверждение о непрерывности в (b), так как 𝛽 → 0 при 𝐵 → 𝐴.
9.9 Матрицы. Пусть {x1 , … , x𝑛 } и {y1 , … , y𝑚 } — базисы векторных пространств 𝑋 и 𝑌 соответственно. Тогда любое 𝐴 ∈ 𝐿(𝑋, 𝑌 ) определяет множество чисел 𝑎𝑖𝑗 , таких, что
𝑚
(3)
𝐴x𝑗 =
∑
𝑎𝑖𝑗 y𝑖
(1 ≤ 𝑗 ≤ 𝑛).
𝑖=1
Удобно представлять себе эти числа расположенными в прямоугольную таблицу из 𝑚 строк и 𝑛
столбцов, называемую матрицей 𝑚 на 𝑛:
⎡ 𝑎11 𝑎12 ⋯ 𝑎1𝑛 ⎤
𝑎22 ⋯ 𝑎2𝑛 ⎥
⎢𝑎
[𝐴] = ⎢ 21
.
.
.
.
.
.
.
. . . . . . . . . . . . . .⎥⎥
⎢
⎣𝑎𝑚1 𝑎𝑚2 ⋯ 𝑎𝑚𝑛 ⎦
Заметим, что координаты 𝑎𝑖𝑗 вектора 𝐴x𝑗 (относительно базиса {y1 , … , y𝑚 }) находятся в 𝑗-ом столбце матрицы [𝐴]. Векторы 𝐴x𝑗 потому иногда называют векторами-столбцами матрицы [𝐴]. Используя эту терминологию, можно сказать, что множество значений преобразования 𝐴 натянуто
на векторы-столбцы матрицы [𝐴].
Если x = ∑ 𝑐𝑗 x𝑗 , то, в силу линейности преобразования 𝐴 и (3) получаем
⎞
⎛ 𝑛
⎜ 𝑎 𝑐 ⎟y .
∑ ⎜∑ 𝑖𝑗 𝑗 ⎟ 𝑖
𝑖=1 𝑗=1
⎠
⎝
𝑚
(4)
𝐴x =
Таким образом, координатами вектора 𝐴x являются ∑𝑗 𝑎𝑖𝑗 𝑐𝑗 . Заметим, что в (3) суммирование
производится по первому индексу элемента 𝑎𝑖𝑗 , тогда как, вычисляя координаты, мы суммируем
по второму индексу.
Пусть теперь дана матрица 𝑚 на 𝑛 с вещественными элементами 𝑎𝑖𝑗 . Если определить 𝐴 равенством (4), то ясно, что 𝐴 ∈ 𝐿(𝑋, 𝑌 ) и что [𝐴] — исходная матрица. Таким образом, имеется
естественное взаимно однозначное соответствие между 𝐿(𝑋, 𝑌 ) и множеством всех вещественных
матриц 𝑚 на 𝑛. Подчеркнем, однако, что [𝐴] зависит не только от 𝐴, но и от выбора базисов в
𝑋 и 𝑌 . Одно и то же 𝐴 порождает различные матрицы, если менять базисы, и обратно. Мы не
станем развивать эту мысль, так как мы обычно будем иметь дело с фиксированными базисами
(некоторые замечания по этому поводу можно найти в п. 9.37).
Если 𝑍 — третье векторное пространство с базисом {z1 , … , z𝑝 }, если 𝐴 задано равенством (3) и
если
𝑐 z ,
𝑏 z ,
(𝐵𝐴)x𝑗 =
𝐵y𝑖 =
∑ 𝑘𝑗 𝑘
∑ 𝑘𝑖 𝑘
𝑘
𝑘
то 𝐴 ∈ 𝐿(𝑋, 𝑌 ), 𝐵 ∈ 𝐿(𝑌 , 𝑍), 𝐵𝐴 ∈ 𝐿(𝑋, 𝑍), и так как
𝐵(𝐴x𝑗 ) = 𝐵
∑
𝑎𝑖𝑗 y𝑖 =
=
∑
𝑎𝑖𝑗
∑
𝑎𝑖𝑗 𝐵y𝑖
𝑖
𝑖
∑
𝑏𝑘𝑖 z𝑘 =
𝑘
∑ (∑
𝑘
156
𝑖
𝑏𝑘𝑖 𝑎𝑖𝑗
)
z𝑘 ,
то из независимости множества {z1 , … , z𝑝 } следует, что
(5)
𝑐𝑘𝑗 =
∑
𝑏𝑘𝑖 𝑎𝑖𝑗
(1 ≤ 𝑘 ≤ 𝑝, 1 ≤ 𝑗 ≤ 𝑛).
𝑖
Это показывает, как вычислить матрицу 𝑝 на 𝑛 [𝐵𝐴] из матриц [𝐵] и [𝐴]. Если мы определим
произведение [𝐵][𝐴] как [𝐵𝐴], то (5) описывает обычное правило перемножения матриц.
Наконец, допустим, что {x1 , … , x𝑛 } и {y1 , … , y𝑚 } — стандартные базисы пространств 𝑅𝑛 и 𝑅𝑚
и что 𝐴 задано равенством (4). Неравенство Шварца показывает, что
2
2
|𝐴x| =
∑ (∑
𝑖
𝑗
𝑎𝑖𝑗 𝑐𝑗
)
≤
∑ (∑
𝑖
𝑎2𝑖𝑗 ⋅
𝑗
∑
𝑗
𝑐𝑗2
)
=
∑
𝑎2𝑖𝑗 |x|2 .
𝑖,𝑗
Таким образом,
1/2
(6)
‖𝐴‖ ≤
{∑
𝑖,𝑗
𝑎2𝑖𝑗
}
.
Применяя (6) к 𝐵 − 𝐴 вместо 𝐴, где 𝐴, 𝐵 ∈ 𝐿(𝑅𝑛 , 𝑅𝑚 ), мы видим, что если элементы 𝑎𝑖𝑗 —
непрерывные функции некоторого параметра, то то же верно в отношении 𝐴. Более точно это
можно сформулировать следующим образом.
Если 𝑆 — метрическое пространство, 𝑎11 , … , 𝑎𝑚𝑛 — вещественные непрерывные функции на 𝑆, и
если при любом 𝑝 ∈ 𝑆 𝐴𝑝 — линейное преобразование пространства 𝑅𝑛 в 𝑅𝑚 , матрица которого
составлена из элементов 𝑎𝑖𝑗 (𝑝), то отображение 𝑝 → 𝐴𝑝 — непрерывное отображение пространства 𝑆 в 𝐿(𝑅𝑛 , 𝑅𝑚 ).
Дифференцирование
9.10 Предварительные соображения. Прежде чем прийти к определению производной от функции, определенной на 𝑅𝑛 (или на открытом множестве пространства 𝑅𝑛 ), еще раз взглянем на знакомый случай 𝑛 = 1 и посмотрим, каким способом можно интерпретировать производную, чтобы
естественным путем расширить этот способ на случай 𝑛 > 1.
Если 𝑓 — вещественная функция с областью определения (𝑎, 𝑏) ⊂ 𝑅1 и если 𝑥 ∈ (𝑎, 𝑏), то 𝑓 ′ (𝑥)
обычно определяется как вещественное число
(7)
lim
ℎ→0
𝑓 (𝑥 + ℎ) − 𝑓 (𝑥)
,
ℎ
конечно, в том случае, когда этот предел существует. Таким образом,
(8)
𝑓 (𝑥 + ℎ) − 𝑓 (𝑥) = 𝑓 ′ (𝑥)ℎ + 𝑟(ℎ),
где «остаток» 𝑟(ℎ) мал в том смысле, что
(9)
lim
ℎ→0
𝑟(ℎ)
= 0.
ℎ
Отметим, что (8) выражает разность 𝑓 (𝑥 + ℎ) − 𝑓 (𝑥) в виде суммы линейной функции, преобразующей ℎ в 𝑓 ′ (𝑥)ℎ, и малого остатка.
Тем самым мы можем рассматривать производную функции 𝑓 в точке 𝑥 не как вещественное
число, а как линейный оператор на 𝑅1 , преобразующий ℎ в 𝑓 ′ (𝑥)ℎ.
(Заметим, что каждому вещественному числу 𝛼 соответствует линейный оператор на 𝑅1 ; этот
оператор представляет собой просто умножение на 𝛼. Обратно, каждая линейная функция, преобразующая 𝑅1 в 𝑅1 , есть умножение на некоторое вещественное число. Это естественное взаимно
однозначное соответствие между 𝑅1 и 𝐿(𝑅1 ) объясняет предыдущие утверждения.)
157
Рассмотрим далее функцию f, отображающую (𝑎, 𝑏) ⊂ 𝑅1 в 𝑅𝑚 . В этом случае производная f′ (𝑥)
определена как вектор y ∈ 𝑅𝑚 (если он существует), для которого
(10)
lim
ℎ→0 {
f(𝑥 + ℎ) − f(𝑥)
− y = 0.
}
ℎ
Мы снова можем записать это в виде
(11)
f(𝑥 + ℎ) − f(𝑥) = ℎy + r(ℎ),
где r(ℎ)/ℎ → 0 при ℎ → 0. Главный член в правой части (11) снова является линейной функцией от
ℎ. Каждому y ∈ 𝑅𝑚 соответствует линейное преобразование пространства 𝑅1 в 𝑅𝑚 , связывающее с
каждым ℎ ∈ 𝑅1 вектор ℎy ∈ 𝑅𝑚 . Это отождествление пространства 𝑅𝑚 с 𝐿(𝑅1 , 𝑅𝑚 ) позволяет нам
рассматривать f′ (𝑥) как элемент пространства 𝐿(𝑅1 , 𝑅𝑚 ).
Таким образом, если f — дифференцируемое отображение интервала (𝑎, 𝑏) ⊂ 𝑅1 в 𝑅𝑚 и если
𝑥 ∈ (𝑎, 𝑏), то f′ (𝑥) — линейное преобразование пространства 𝑅1 в 𝑅𝑚 , удовлетворяющее условию
f(𝑥 + ℎ) − f(𝑥) − f′ (𝑥)ℎ
= 0,
ℎ→0
ℎ
(12)
lim
или, что то же самое,
(13)
|f(𝑥 + ℎ) − f(𝑥) − f′ (𝑥)ℎ|
= 0.
ℎ→0
|ℎ|
lim
Теперь мы готовы к случаю 𝑛 > 1.
9.11 Определение. Пусть 𝐸 — открытое множество в 𝑅𝑛 , f отображает 𝐸 в 𝑅𝑚 и x ∈ 𝐸. Если
существует линейное преобразование 𝐴 пространства 𝑅𝑛 в 𝑅𝑚 , такое, что
(14)
lim
h→0
|f(x + h) − f(x) − 𝐴h|
=0
|h|
то говорят, что f дифференцируемо в x, и пишут
(15)
f′ (x) = 𝐴.
Если f дифференцируемо в каждой точке x ∈ 𝐸, то говорят, что f дифференцируемо на 𝐸.
В (14), конечно, имеется в виду, что h ∈ 𝑅𝑛 . Если |h| достаточно мало, то x + h ∈ 𝐸, так как 𝐸
открыто. Таким образом, f(x + h) имеет смысл, f(x + h) ∈ 𝑅𝑚 , и так как 𝐴 ∈ 𝐿(𝑅𝑛 , 𝑅𝑚 ), то 𝐴h ∈ 𝑅𝑚 .
Таким образом,
f(x + h) − f(x) − 𝐴h ∈ 𝑅𝑚 .
Норма в числителе дроби (14) — норма пространства 𝑅𝑚 , а в знаменателе — 𝑅𝑛 -норма вектора h.
Прежде чем двинуться дальше, мы решим очевидно возникающий вопрос о единственности.
9.12 Теорема. Пусть 𝐸 и f — те же, что в определении 9.11, и (14) выполняется при 𝐴 = 𝐴1 и
при 𝐴 = 𝐴2 . Тогда 𝐴1 = 𝐴2 .
Доказательство. Если 𝐵 = 𝐴1 − 𝐴2 , то неравенство
|𝐵h| ≤ |f(x + h) − f(x) − 𝐴1 h| + |f(x + h) − f(x) − 𝐴2 h|
показывает, что |𝐵h|/|h| → 0 при h → 0. Отсюда следует, что при фиксированном h ≠ 0
(16)
|𝐵(𝑡h)|
→0
|𝑡h|
при 𝑡 → 0.
Ввиду того что 𝐵 линейно, левая часть в (16) не зависит от 𝑡. Поэтому 𝐵h = 0 при всех h ∈ 𝑅𝑛 .
Значит, 𝐵 = 0.
9.13 Замечания.
158
(a) Соотношение (14) можно переписать в виде
(17)
f(x + h) − f(x) = f′ (x)h + r(h),
где остаток r(h) удовлетворяет условию
(18)
lim
h→0
|r(h)|
= 0.
|h|
Можно истолковать (17), как в п. 9.10, сказав, что при фиксированном x и малом h левая
часть (17) приближенно равна f′ (x)h, т. е. значению линейного преобразования в точке h.
(b) Пусть f и 𝐸 — те же, что и в определении 9.11, и f дифференцируемо на 𝐸. При каждом
x ∈ 𝐸 f′ (x) — функция, а именно линейное преобразование пространства 𝑅𝑛 в 𝑅𝑚 . С другой
стороны, f′ — тоже функция: f′ отображает 𝐸 в 𝐿(𝑅𝑛 , 𝑅𝑚 ).
(c) Достаточно взглянуть на (17), чтобы убедиться, что f непрерывно в любой точке, в которой
оно дифференцируемо.
(d) Производную, определенную в (14) или (17), часто называют дифференциалом отображения
f в точке x или полной производной отображения f в точке x, чтобы отличить ее от частных
производных, которые будут введены позже.
9.14 Пример. Мы определили производные функций, отображающих 𝑅𝑛 в 𝑅𝑚 , как линейные
преобразования пространства 𝑅𝑛 в 𝑅𝑚 . Чему равна производная такого линейного преобразования?
Ответ очень прост.
Если 𝐴 ∈ 𝐿(𝑅𝑛 , 𝑅𝑚 ) и если x ∈ 𝑅𝑛 , то
(19)
𝐴′ (x) = 𝐴.
Отметим, что x появляется в левой части (19), но отсутствует в правой. Обе части (19) —
элементы пространства 𝐿(𝑅𝑛 , 𝑅𝑚 ), в то время как 𝐴x ∈ 𝑅𝑚 .
Доказательство (19) тривиально, так как
(20)
𝐴(x + h) − 𝐴x = 𝐴h
в силу линейности 𝐴. Если положить f(x) = 𝐴x, то числитель в (14) равен 0 при каждом h ∈ 𝑅𝑛 . В
(17) r(h) = 0.
Теперь мы распространим правило дифференцирования сложной функции (теорема 5.5) на
данную ситуацию.
9.15 Теорема. Пусть 𝐸 — открытое множество в 𝑅𝑛 , f отображает 𝐸 в 𝑅𝑚 , f дифференцируемо в x0 ∈ 𝐸, g отображает некоторое открытое множество, содержащее f(𝐸), в 𝑅𝑘 , и g
дифференцируемо в f(x0 ). Тогда отображение F множества 𝐸 в 𝑅𝑘 , определенное равенством
F(x) = g(f(x)),
дифференцируемо в x0 , и
(21)
F′ (x0 ) = g′ (f(x0 ))f′ (x0 ).
В правой части (21) стоит произведение двух линейных преобразований, определенное в п. 9.6.
Доказательство. Пусть y0 = f(x0 ), 𝐴 = f′ (x0 ), 𝐵 = g′ (y0 ), и пусть
u(h) = f(x0 + h) − f(x0 ) − 𝐴h,
v(k) = g(y0 + k) − g(y0 ) − 𝐵k
при всех h ∈ 𝑅𝑛 и k ∈ 𝑅𝑚 , для которых определены f(x0 + h) и g(y0 + k). Тогда
(22)
|u(h)| = 𝜀(h)|h|,
|v(k)| = 𝜂(k)|k|,
где 𝜀(h) → 0 при h → 0 и 𝜂(k) → 0 при k → 0.
159
При данном h положим k = f(x0 + h) − f(x0 ). Тогда
(23)
|k| = |𝐴h + u(h)| ≤ [‖𝐴‖ + 𝜀(h)] |h|
и
F(x0 + h) − F(x0 ) − 𝐵𝐴h = g(y0 + k) − g(y0 ) − 𝐵𝐴h
= 𝐵(k − 𝐴h) + v(k)
= 𝐵u(h) + v(k).
Значит, из (22) и (23) при h ≠ 0 следует, что
|F(x0 + h) − F(x0 ) − 𝐵𝐴h|
≤ ‖𝐵‖𝜀(h) + [‖𝐴‖ + 𝜀(h)] 𝜂(k).
|h|
Положим h → 0. Тогда 𝜀(h) → 0. Также из (23) следует, что k → 0, так что 𝜂(k) → 0. Следовательно, F′ (x0 ) = 𝐵𝐴, что и утверждает (21).
9.16 Частные производные. Снова рассмотрим функцию f, отображающую открытое множество 𝐸 ⊂ 𝑅𝑛 в 𝑅𝑚 . Пусть {e1 , … , e𝑛 } и {u1 , … , u𝑚 } — стандартные базисы пространств 𝑅𝑛 и 𝑅𝑚 .
Компоненты функции f — вещественные функции 𝑓1 , … , 𝑓𝑚 , определяемые равенством
𝑚
(24)
f(x) =
∑
𝑓𝑖 (x)u𝑖
(x ∈ 𝐸),
𝑖=1
или, что равносильно, выражениями 𝑓𝑖 (x) = f(x) ⋅ u𝑖 , 1 ≤ 𝑖 ≤ 𝑚.
При x ∈ 𝐸, 1 ≤ 𝑖 ≤ 𝑚, 1 ≤ 𝑗 ≤ 𝑛 положим, по определению,
(25)
(𝐷𝑗 𝑓𝑖 )(x) = lim
𝑓𝑖 (x + 𝑡e𝑗 ) − 𝑓𝑖 (x)
𝑡
𝑡→0
,
если этот предел существует. Записывая 𝑓𝑖 (𝑥1 , … , 𝑥𝑛 ) вместо 𝑓𝑖 (x), мы видим, что 𝐷𝑗 𝑓𝑖 есть производная функции 𝑓𝑖 по 𝑥𝑗 при фиксированных значениях остальных переменных. Поэтому часто
вместо 𝐷𝑗 𝑓𝑖 используется обозначение
𝜕𝑓𝑖
,
𝜕𝑥𝑗
(26)
и 𝐷𝑗 𝑓𝑖 называют частной производной.
Во многих случаях, где при работе с функциями одной переменной достаточно существования
производной, для функций нескольких переменных требуется непрерывность или хотя бы ограниченность частных производных. Например, функции 𝑓 и 𝑔, описанные в упражнении 7 гл. 4,
не являются непрерывными, хотя их частные производные существуют в каждой точке плоскости 𝑅2 . Даже для непрерывных функций из существования всех частных производных не следует
дифференцируемость в смысле определения 9.11; см. упражнения 6 и 14 и теорему 9.21.
Однако если известно, что отображение f дифференцируемо в точке x, то его частные производные существуют в x, и они полностью определяют линейное преобразование f′ (x).
9.17 Теорема. Пусть f отображает открытое множество 𝐸 ⊂ 𝑅𝑛 в 𝑅𝑚 и f дифференцируемо в
точке x ∈ 𝐸. Тогда частные производные (𝐷𝑗 𝑓𝑖 )(x) существуют и
𝑚
(27)
f′ (x)e𝑗 =
(𝐷 𝑓 )(x)u𝑖
∑ 𝑗 𝑖
(1 ≤ 𝑗 ≤ 𝑛).
𝑖=1
Здесь, как и в п. 9.16, {e1 , … , e𝑛 } и {u1 , … , u𝑚 } — стандартные базисы пространств 𝑅𝑛 и 𝑅𝑚 .
160
Доказательство. Зафиксируем 𝑗. Ввиду того что f дифференцируемо в x,
f(x + 𝑡e𝑗 ) − f(x) = f′ (x)(𝑡e𝑗 ) + r(𝑡e𝑗 ),
где |r(𝑡e𝑗 )|/𝑡 → 0 при 𝑡 → 0. Из линейности отображения f′ (x) тогда следует, что
(28)
lim
f(x + 𝑡e𝑗 ) − f(x)
𝑡
𝑡→0
= f′ (x)e𝑗 .
Если теперь записать f покомпонентно, как в (24), то (28) превратится в
𝑚
(29)
lim
𝑓𝑖 (x + 𝑡e𝑗 ) − 𝑓𝑖 (x)
𝑡→0 ∑
𝑖=1
𝑡
u𝑖 = f′ (x)e𝑗 .
Следовательно, каждая дробь в этой сумме имеет предел при 𝑡 → 0 (см. теорему 4.10), так что
каждая (𝐷𝑗 𝑓𝑖 )(x) существует, и тогда (27) следует из (29).
Вот некоторые следствия из теоремы 9.17.
Пусть [f′ (x)] — матрица, представляющая f′ (x) относительно стандартных базисов, как в п. 9.9.
Тогда f′ (x)e𝑗 — 𝑗-ый вектор-столбец матрицы [f′ (x)], и (27) показывает, что число (𝐷𝑗 𝑓𝑖 )(x) находится в 𝑖-ой строке и 𝑗-ом столбце матрицы [f′ (x)]. Таким образом,
⎡ (𝐷1 𝑓1 )(x) ⋯ (𝐷𝑛 𝑓1 )(x) ⎤
[f′ (x)] = ⎢. . . . . . . . . . . . . . . . . . . . . . . . . . . .⎥ .
⎢
⎥
⎣(𝐷1 𝑓𝑚 )(x) ⋯ (𝐷𝑛 𝑓𝑚 )(x)⎦
Если h = ∑ ℎ𝑗 e𝑗 — произвольный вектор в 𝑅𝑛 , то из (27) следует, что
𝑚
(30)
f′ (x)h =
𝑛
⎧
⎫
⎪
⎪
(𝐷𝑗 𝑓𝑖 )(x)ℎ𝑗 ⎬ u𝑖 .
⎨
∑ ∑
⎪
𝑖=1 ⎪
⎩ 𝑗=1
⎭
9.18 Пример. Пусть 𝛾 — дифференцируемое отображение интервала (𝑎, 𝑏) ⊂ 𝑅1 в открытое множество 𝐸 ⊂ 𝑅𝑛 , иными словами, 𝛾 — дифференцируемая кривая в 𝐸. Пусть 𝑓 — вещественнозначная
дифференцируемая функция с областью определения 𝐸. Таким образом, 𝑓 — дифференцируемое
отображение множества 𝐸 в 𝑅1 . Положим
(31)
𝑔(𝑡) = 𝑓 (𝛾(𝑡))
(𝑎 < 𝑡 < 𝑏).
Тогда по правилу дифференцирования сложной функции
(32)
𝑔 ′ (𝑡) = 𝑓 ′ (𝛾(𝑡))𝛾 ′ (𝑡)
(𝑎 < 𝑡 < 𝑏).
Ввиду того что 𝛾 ′ (𝑡) ∈ 𝐿(𝑅1 , 𝑅𝑛 ) и 𝑓 ′ (𝛾(𝑡)) ∈ 𝐿(𝑅𝑛 , 𝑅1 ), (32) определяет 𝑔 ′ (𝑡) как линейный оператор
на 𝑅1 . Это согласуется с тем, что 𝑔 отображает (𝑎, 𝑏) в 𝑅1 . С другой стороны, 𝑔 ′ (𝑡) можно также рассматривать как вещественное число (это обсуждалось в п. 9.10). Как мы сейчас увидим, это число
можно вычислить из частных производных функции 𝑓 и производных от компонент отображения
𝛾.
Относительно стандартного базиса {e1 , … , e𝑛 } пространства 𝑅𝑛 [𝛾 ′ (𝑡)] — матрица 𝑛 на 1 («матрицастолбец»), в 𝑖-ой строке которой находится 𝛾𝑖′ (𝑡), где 𝛾1 , … , 𝛾𝑛 — компоненты отображения 𝛾. При
каждом x ∈ 𝐸 [𝑓 ′ (x)] — матрица 1 на 𝑛 («матрица-строка»), в 𝑗-ом столбце которой находится
(𝐷𝑗 𝑓 )(x). Следовательно, [𝑔 ′ (𝑡)] — матрица 1 на 1, единственный элемент которой — вещественное
число
𝑛
(33)
𝑔 ′ (𝑡) =
∑
(𝐷𝑖 𝑓 )(𝛾(𝑡))𝛾𝑖′ (𝑡).
𝑖=1
Это часто встречающийся специальный случай правила дифференцирования сложной функции.
Он может быть описан следующим образом.
161
Свяжем с каждым x ∈ 𝐸 вектор, так называемый «градиент» функции 𝑓 в x, равный по
определению
𝑛
(34)
(∇𝑓 )(x) =
∑
(𝐷𝑖 𝑓 )(x)e𝑖 .
𝑖=1
Ввиду того что
𝑛
𝛾 ′ (𝑡) =
(35)
∑
𝛾𝑖′ (𝑡)e𝑖 ,
𝑖=1
(33) может быть записано в виде
𝑔 ′ (𝑡) = (∇𝑓 )(𝛾(𝑡)) ⋅ 𝛾 ′ (𝑡),
(36)
т. е. скалярного произведения векторов (∇𝑓 )(𝛾(𝑡)) и 𝛾 ′ (𝑡).
Зафиксируем теперь x ∈ 𝐸, пусть u ∈ 𝑅𝑛 — единичный вектор (т. е. |u| = 1), и выберем 𝛾 так,
что
(37)
𝛾(𝑡) = x + 𝑡u
(−∞ < 𝑡 < ∞).
Тогда 𝛾 ′ (𝑡) = u при каждом 𝑡. Следовательно, (36) показывает, что
𝑔 ′ (0) = (∇𝑓 )(x) ⋅ u.
(38)
С другой стороны, (37) показывает, что
𝑔(𝑡) − 𝑔(0) = 𝑓 (x + 𝑡u) − 𝑓 (x).
Следовательно, (38) дает
(39)
lim
𝑡→0
𝑓 (x + 𝑡u) − 𝑓 (x)
= (∇𝑓 )(x) ⋅ u.
𝑡
Предел в (39) обычно называется производной функции 𝑓 в точке x по направлению единичного
вектора u и может обозначаться через (𝐷u 𝑓 )(x).
Если 𝑓 и x фиксированы, а u меняется, то (39) показывает, что (𝐷u 𝑓 )(x) достигает максимума в
том случае, когда u — градиент (∇𝑓 )(x), умноженный на положительный скаляр (случай (∇𝑓 )(x) =
0 здесь исключается).
Если u = ∑ 𝑢𝑖 e𝑖 , то (39) показывает, что (𝐷u 𝑓 )(x) можно выразить через частные производные
функции 𝑓 в x формулой
𝑛
(40)
(𝐷u 𝑓 )(x) =
∑
(𝐷𝑖 𝑓 )(x)𝑢𝑖 .
𝑖=1
Некоторые из этих идей используются в следующей теореме.
9.19 Теорема. Пусть f отображает выпуклое открытое множество 𝐸 ⊂ 𝑅𝑛 в 𝑅𝑚 , f дифференцируемо на 𝐸 и существует вещественное число 𝑀, такое, что
‖f′ (x)‖ ≤ 𝑀
при каждом x ∈ 𝐸. Тогда
|f(b) − f(a)| ≤ 𝑀|b − a|
при всех a ∈ 𝐸, b ∈ 𝐸.
162
Доказательство. Зафиксируем a ∈ 𝐸, b ∈ 𝐸. Положим
𝛾(𝑡) = (1 − 𝑡)a + 𝑡b
при всех 𝑡 ∈ 𝑅1 , таких, что 𝛾(𝑡) ∈ 𝐸. Ввиду того что 𝐸 выпукло, 𝛾(𝑡) ∈ 𝐸, если 0 ≤ 𝑡 ≤ 1. Положим
g(𝑡) = f(𝛾(𝑡)).
Тогда
g′ (𝑡) = f′ (𝛾(𝑡))𝛾 ′ (𝑡) = f′ (𝛾(𝑡))(b − a),
так что
|g′ (𝑡)| ≤ ‖f′ (𝛾(𝑡))‖|b − a| ≤ 𝑀|b − a|
при всех 𝑡 ∈ [0, 1]. По теореме 5.19
|g(1) − g(0)| ≤ 𝑀|b − a|.
Но g(0) = f(a) и g(1) = f(b). Доказательство закончено.
Следствие. Если, кроме того, f′ (x) = 0 при всех x ∈ 𝐸, то f постоянна.
Доказательство. Чтобы доказать это утверждение, отметим, что условия теоремы выполняются
при 𝑀 = 0.
9.20 Определение. Дифференцируемое отображение f открытого множества 𝐸 ⊂ 𝑅𝑛 в 𝑅𝑚 называется непрерывно дифференцируемым на 𝐸, если f′ — непрерывное отображение множества 𝐸 в
𝐿(𝑅𝑛 , 𝑅𝑚 ).
Говоря более отчетливо, в этом определении требуется, чтобы для любого x ∈ 𝐸 и любого 𝜀 > 0
существовало 𝛿 > 0, такое, что
‖f′ (y) − f′ (x)‖ < 𝜀,
если y ∈ 𝐸 и |y − x| < 𝛿.
Если это выполняется, то мы также будем говорить, что f является 𝒞 ′ -отображением или что
𝑓 ∈ 𝒞 ′ (𝐸).
9.21 Теорема. Пусть f отображает открытое множество 𝐸 ⊂ 𝑅𝑛 в 𝑅𝑚 . Тогда f ∈ 𝒞 ′ (𝐸) в
том и только том случае, когда частные производные 𝐷𝑗 𝑓𝑖 существуют и непрерывны на 𝐸 при
1 ≤ 𝑖 ≤ 𝑚, 1 ≤ 𝑗 ≤ 𝑛.
Доказательство. Предположим сначала, что f ∈ 𝒞 ′ (𝐸). В силу (27),
(𝐷𝑗 𝑓𝑖 )(x) = (f′ (x)e𝑗 ) ⋅ u𝑖
при всех 𝑖, 𝑗 и при всех x ∈ 𝐸. Следовательно,
(𝐷𝑗 𝑓𝑖 )(y) − (𝐷𝑗 𝑓𝑖 )(x) = {[f′ (y) − f′ (x)]e𝑗 } ⋅ u𝑖 ,
и так как |u𝑖 | = |e𝑗 | = 1, то из этого следует, что
|(𝐷𝑗 𝑓𝑖 )(y) − (𝐷𝑗 𝑓𝑖 )(x)| ≤ |[f′ (y) − f′ (x)]e𝑗 |
≤ ‖f′ (y) − f′ (x)‖.
Следовательно, 𝐷𝑗 𝑓𝑖 непрерывна.
Для доказательства обратного достаточно рассмотреть случай 𝑚 = 1 (почему?). Зафиксируем
x ∈ 𝐸 и 𝜀 > 0. Ввиду того что 𝐸 открыто, существует открытый шар 𝑆 ⊂ 𝐸 с центром в x и
радиусом 𝑟, и из непрерывности функций 𝐷𝑗 𝑓 следует, что 𝑟 можно выбрать таким, что
(41)
|(𝐷𝑗 𝑓 )(y) − (𝐷𝑗 𝑓 )(x)| <
𝜀
𝑛
163
(y ∈ 𝑆, 1 ≤ 𝑗 ≤ 𝑛).
Пусть h = ∑ ℎ𝑗 e𝑗 , |h| < 𝑟. Положим v0 = 0 и v𝑘 = ℎ1 e1 + ⋯ + ℎ𝑘 e𝑘 при 1 ≤ 𝑘 ≤ 𝑛. Тогда
𝑛
(42)
𝑓 (x + h) − 𝑓 (x) =
∑
[𝑓 (x + v𝑗 ) − 𝑓 (x + v𝑗−1 )].
𝑗=1
Ввиду того что |v𝑘 | < 𝑟 при 1 ≤ 𝑘 ≤ 𝑛 и 𝑆 выпукло, отрезки с концами x + v𝑗−1 и x + v𝑗 лежат в 𝑆.
Поскольку v𝑗 = v𝑗−1 + ℎ𝑗 e𝑗 , то по теореме о среднем значении (5.10) 𝑗-ое слагаемое в (42) равно
ℎ𝑗 (𝐷𝑗 𝑓 )(x + v𝑗−1 + 𝜃𝑗 ℎ𝑗 e𝑗 )
при некотором 𝜃𝑗 ∈ (0, 1), и в силу (41) отличается от ℎ𝑗 (𝐷𝑗 𝑓 )(x) менее, чем на |ℎ𝑗 |𝜀/𝑛. Согласно
(42), отсюда следует, что
𝑛
𝑛
|
|
1
|𝑓 (x + h) − 𝑓 (x) −
|
|ℎ𝑗 |𝜀 ≤ |h|𝜀
ℎ (𝐷 𝑓 )(x) ≤
∑ 𝑗 𝑗
||
|| 𝑛 ∑
𝑗=1
𝑗=1
при всех h, таких, что |h| < 𝑟.
Это означает, что 𝑓 дифференцируема в x и что 𝑓 ′ (x) — линейная функция, которая ставит в соответствие число ∑ ℎ𝑗 (𝐷𝑗 𝑓 )(x) вектору h = ∑ ℎ𝑗 e𝑗 . Матрица [𝑓 ′ (x)] состоит из строки
(𝐷1 𝑓 )(x), … , (𝐷𝑛 𝑓 )(x), а так как 𝐷1 𝑓 , … , 𝐷𝑛 𝑓 — непрерывные функции на 𝐸, то, как показывает
заключительное замечание п. 9.9, 𝑓 ∈ 𝒞 ′ (𝐸).
Принцип сжимающих отображений
Сейчас мы прервем наше обсуждение дифференцирования, чтобы доказать теорему о неподвижной
точке, справедливую в произвольных полных метрических пространствах. Она будет использоваться при доказательстве теоремы об обратной функции.
9.22 Определение. Пусть 𝑋 — метрическое пространство с метрикой 𝑑. Если 𝜑 отображает 𝑋 в
𝑋 и если существует число 𝑐 < 1, такое, что
(43)
𝑑(𝜑(𝑥), 𝜑(𝑦)) ≤ 𝑐 𝑑(𝑥, 𝑦)
при всех 𝑥, 𝑦 ∈ 𝑋, то 𝜑 называют сжимающим отображением пространства 𝑋 в 𝑋.
9.23 Теорема. Если 𝑋 — полное метрическое пространство и если 𝜑 — сжимающее отображение пространства 𝑋 в 𝑋, то существует одна и только одна точка 𝑥 ∈ 𝑋, такая, что 𝜑(𝑥) = 𝑥.
Иными словами, 𝜑 имеет единственную неподвижную точку. Единственность очевидна, так как
если 𝜑(𝑥) = 𝑥 и 𝜑(𝑦) = 𝑦, то из (43) следует 𝑑(𝑥, 𝑦) ≤ 𝑐 𝑑(𝑥, 𝑦), что может случиться, только когда
𝑑(𝑥, 𝑦) = 0.
Существование неподвижной точки отображения 𝜑 является основной частью теоремы. Доказательство в действительности предоставляет конструктивный метод нахождения неподвижной
точки.
Доказательство. Выберем произвольно 𝑥0 ∈ 𝑋 и определим последовательность {𝑥𝑛 } рекуррентно, положив
(44)
𝑥𝑛+1 = 𝜑(𝑥𝑛 )
(𝑛 = 0, 1, 2, … ).
Выберем 𝑐 < 1 так, чтобы выполнялось (43). Тогда при 𝑛 ≥ 1 имеем
𝑑(𝑥𝑛+1 , 𝑥𝑛 ) = 𝑑(𝜑(𝑥𝑛 ), 𝜑(𝑥𝑛−1 )) ≤ 𝑐 𝑑(𝑥𝑛 , 𝑥𝑛−1 ).
Следовательно, по индукции получаем
(45)
𝑑(𝑥𝑛+1 , 𝑥𝑛 ) ≤ 𝑐 𝑛 𝑑(𝑥1 , 𝑥0 )
164
(𝑛 = 0, 1, 2, … ).
Если 𝑛 < 𝑚, то из этого следует, что
𝑚
𝑑(𝑥𝑛 , 𝑥𝑚 ) ≤
∑
𝑑(𝑥𝑖 , 𝑥𝑖−1 )
𝑖=𝑛+1
≤ (𝑐 𝑛 + 𝑐 𝑛+1 + ⋯ + 𝑐 𝑚−1 ) 𝑑(𝑥1 , 𝑥0 )
≤ [(1 − 𝑐)−1 𝑑(𝑥1 , 𝑥0 )]𝑐 𝑛 .
Следовательно, {𝑥𝑛 } — последовательность Коши. Ввиду того что 𝑋 полно, lim 𝑥𝑛 = 𝑥 при некотором 𝑥 ∈ 𝑋.
Ввиду того что 𝜑 — сжимающее отображение, 𝜑 непрерывно (даже равномерно непрерывно) на
𝑋. Значит,
𝜑(𝑥) = lim 𝜑(𝑥𝑛 ) = lim 𝑥𝑛+1 = 𝑥.
𝑛→∞
𝑛→∞
Теорема об обратной функции
Теорема об обратной функции утверждает, грубо говоря, что непрерывно дифференцируемое отображение f обратимо в окрестности любой точки x, в которой обратимо линейное преобразование
f′ (x).
9.24 Теорема. Пусть f — 𝒞 ′ -отображение открытого множества 𝐸 ⊂ 𝑅𝑛 в 𝑅𝑛 , f′ (a) обратимо
при некотором a ∈ 𝐸 и b = f(a). Тогда
(a) существуют открытые множества 𝑈 и 𝑉 в 𝑅𝑛 , такие, что a ∈ 𝑈 , b ∈ 𝑉 , f взаимно
однозначно на 𝑈 и f(𝑈 ) = 𝑉 ;
(b) если g — отображение, обратное к f (оно существует согласно (a)), заданное на 𝑉 равенством
g(f(x)) = x
(x ∈ 𝑈 ),
то g ∈ 𝒞 ′ (𝑉 ).
Записывая равенство y = f(x) через компоненты, мы приходим к следующей интерпретации
заключения теоремы: систему из 𝑛 уравнений
𝑦𝑖 = 𝑓𝑖 (𝑥1 , … , 𝑥𝑛 )
(1 ≤ 𝑖 ≤ 𝑛)
можно разрешить относительно 𝑥1 , … , 𝑥𝑛 , выразив их через 𝑦1 , … , 𝑦𝑛 , если ограничить x и y достаточно малыми окрестностями точек a и b; решения единственны и непрерывно дифференцируемы.
Доказательство.
(a) Пусть f′ (a) = 𝐴. Выберем 𝜆 так, чтобы
(46)
2𝜆‖𝐴−1 ‖ = 1.
Ввиду того что f′ непрерывно в a, существует такой открытый шар 𝑈 ⊂ 𝐸 с центром в a, что
(47)
‖f′ (x) − 𝐴‖ < 𝜆
(x ∈ 𝑈 ).
Свяжем с каждой точкой y ∈ 𝑅𝑛 функцию 𝜑, определенную равенством
(48)
𝜑(x) = x + 𝐴−1 (y − f(x))
(x ∈ 𝐸).
Отметим, что f(x) = y в том и только в том случае, когда x — неподвижная точка
отображения 𝜑.
Ввиду того что 𝜑′ (x) = 𝐼 − 𝐴−1 f′ (x) = 𝐴−1 (𝐴 − f′ (x)), из (46) и (47) следует, что
(49)
‖𝜑′ (x)‖ <
1
2
165
(x ∈ 𝑈 ).
Следовательно,
(50)
|𝜑(x1 ) − 𝜑(x2 )| ≤ 12 |x1 − x2 |
(x1 , x2 ∈ 𝑈 )
по теореме 9.19. Значит, 𝜑 имеет не более одной неподвижной точки в 𝑈 , так что f(x) = y не
более чем для одного x ∈ 𝑈 .
Таким образом, f взаимно однозначно на 𝑈 .
Далее положим 𝑉 = f(𝑈 ) и выберем y0 ∈ 𝑉 . Тогда y0 = f(x0 ) при некотором x0 ∈ 𝑈 . Пусть
𝐵 — открытый шар с центром в x0 и радиусом 𝑟 > 0, достаточно малым, чтобы его замыкание
𝐵̄ лежало в 𝑈 . Мы покажем, что y ∈ 𝑉 , если |y − y0 | < 𝜆𝑟. Это, очевидно, доказывает, что 𝑉
открыто.
Зафиксируем y, |y − y0 | < 𝜆𝑟. Используя 𝜑 из (48), получим
|𝜑(x0 ) − x0 | = |𝐴−1 (y − y0 )| < ‖𝐴−1 ‖𝜆𝑟 =
𝑟
.
2
Если x ∈ 𝐵,̄ то из (50) следует, что
|𝜑(x) − x0 | ≤ |𝜑(x) − 𝜑(x0 )| + |𝜑(x0 ) − x0 |
𝑟
1
< |x − x0 | + ≤ 𝑟;
2
2
значит, 𝜑(x) ∈ 𝐵. Отметим, что (50) выполняется, если x1 ∈ 𝐵,̄ x2 ∈ 𝐵.̄
Следовательно, 𝜑 — сжимающее отображение множества 𝐵̄ в 𝐵.̄ Будучи замкнутым подмножеством пространства 𝑅𝑛 , 𝐵̄ полно. Из теоремы 9.23 тогда следует, что 𝜑 имеет неподвижную
точку x ∈ 𝐵.̄ Для этой точки x 𝑓 (x) = y. Значит, y ∈ f(𝐵)̄ ⊂ f(𝑈 ) = 𝑉 .
Часть (a) теоремы доказана.
(b) Выберем y ∈ 𝑉 , y + k ∈ 𝑉 . Тогда существует точка x ∈ 𝑈 , x + h ∈ 𝑈 , такая, что y = f(x),
y + k = f(x + h). Используя 𝜑 из (48), получим
𝜑(x + h) − 𝜑(x) = h + 𝐴−1 [f(x) − f(x + h)] = h − 𝐴−1 k.
Из (50) следует, что |h − 𝐴−1 k| ≤ 12 |h|. Значит, |𝐴−1 k| ≥ 12 |h|, и
(51)
|h| ≤ 2‖𝐴−1 ‖|k| = 𝜆−1 |k|.
Из (46), (47) и теоремы 9.8 следует, что отображение f′ (x) имеет обратное, которое мы обозначим через 𝑇 . Ввиду того что
g(y + k) − g(y) − 𝑇 k = h − 𝑇 k = −𝑇 [f(x + h) − f(x) − f′ (x)h],
из (51) следует
|g(y + k) − g(y) − 𝑇 k| ‖𝑇 ‖ |f(x + h) − f(x) − f′ (x)h|
≤
⋅
.
|k|
𝜆
|h|
При k → 0 (51) показывает, что h → 0. Следовательно, правая часть последнего неравенства
стремится к 0. Значит, то же самое верно для левой части. Таким образом, мы доказали, что
g′ (y) = 𝑇 . Но 𝑇 является обратным к f′ (x) = f′ (g(y)). Следовательно,
(52)
g′ (y) = {f′ (g(y))}−1
(y ∈ 𝑉 ).
Наконец, отметим, что g — непрерывное отображение множества 𝑉 на 𝑈 (так как g дифференцируемо), что f′ — непрерывное отображение множества 𝑈 во множество Ω всех обратимых
элементов пространства 𝐿(𝑅𝑛 ), и что обращение — непрерывное отображение множества Ω на
Ω по теореме 9.8. Эти утверждения совместно с (52) показывают, что g ∈ 𝒞 ′ (𝑉 ). Теорема
доказана.
166
Замечание. Условие f ∈ 𝒞 ′ (𝐸) было полностью использовано лишь в последнем абзаце доказательства. Все остальные утверждения до (52) включительно были сделаны на основе существования
производной f′ (x) при x ∈ 𝐸, обратимости преобразования f′ (a) и непрерывности производной f′
лишь в точке a. В этой связи мы отсылаем к статье A. Nijenhuis в Amer. Math. Monthly, vol. 81,
1974, pp. 969-980.
Следующее утверждение вытекает непосредственно из части (a) теоремы об обратной функции.
9.25 Теорема. Если f — 𝒞 ′ -отображение открытого множества 𝐸 ⊂ 𝑅𝑛 в 𝑅𝑛 и если f′ (x) обратимо при каждом x ∈ 𝐸, то f(𝑊 ) — открытое множество пространства 𝑅𝑛 , каково бы ни было
открытое множество 𝑊 ⊂ 𝐸.
Иными словами, f — открытое отображение множества 𝐸 в 𝑅𝑛 .
Предположения, сделанные в этой теореме, обеспечивают для каждой точки x ∈ 𝐸 существование такой окрестности, на которой f взаимно однозначно. Можно сказать, что f локально взаимно
однозначно на 𝐸. Однако f не обязано при этом быть взаимно однозначным на 𝐸. Пример приведен
в упражнении 17.
Теорема о неявной функции
Если 𝑓 — непрерывно дифференцируемая вещественная функция на плоскости, то уравнение 𝑓 (𝑥, 𝑦) =
0 можно разрешить относительно 𝑦, выразив его через 𝑥, в окрестности любой точки (𝑎, 𝑏), в которой
𝑓 (𝑎, 𝑏) = 0 и 𝜕𝑓 /𝜕𝑦 ≠ 0. Подобным же образом можно разрешить это уравнение относительно 𝑥, выразив его через 𝑦, рядом с точкой (𝑎, 𝑏), если 𝜕𝑓 /𝜕𝑥 ≠ 0 в (𝑎, 𝑏). Простым примером, показывающим
необходимость предположения 𝜕𝑓 /𝜕𝑦 ≠ 0, служит 𝑓 (𝑥, 𝑦) = 𝑥2 + 𝑦2 − 1.
Предшествующее неформальное утверждение — простейший случай (случай 𝑚 = 𝑛 = 1 теоремы
9.28) так называемой «теоремы о неявной функции». Ее доказательство существенным образом
использует тот факт, что непрерывно дифференцируемые преобразования локально ведут себя
подобно своим производным. В связи с этим мы сначала докажем теорему 9.27 — линейный вариант
теоремы 9.28.
9.26 Обозначение. Если x = (𝑥1 , … , 𝑥𝑛 ) ∈ 𝑅𝑛 и y = (𝑦1 , … , 𝑦𝑛 ) ∈ 𝑅𝑚 , то через (x, y) мы будем
обозначать точку (или вектор)
(𝑥1 , … , 𝑥𝑛 , 𝑦1 , … , 𝑦𝑚 ) ∈ 𝑅𝑛+𝑚 .
В этом разделе первый элемент в (x, y) или в подобном символе всегда будет обозначать вектор в
𝑅𝑛 , а второй — вектор в 𝑅𝑚 .
Каждое 𝐴 ∈ 𝐿(𝑅𝑛+𝑚 , 𝑅𝑛 ) может быть разделено на два линейных преобразования 𝐴𝑥 и 𝐴𝑦 ,
определяемых равенствами
(53)
𝐴𝑥 h = 𝐴(h, 0),
𝐴𝑦 k = 𝐴(0, k)
при любых h ∈ 𝑅𝑛 , k ∈ 𝑅𝑚 . Тогда 𝐴𝑥 ∈ 𝐿(𝑅𝑛 ), 𝐴𝑦 ∈ 𝐿(𝑅𝑚 , 𝑅𝑛 ) и
(54)
𝐴(h, k) = 𝐴𝑥 h + 𝐴𝑦 k.
Теперь линейный вариант теоремы о неявной функции почти очевиден.
9.27 Теорема. Если 𝐴 ∈ 𝐿(𝑅𝑛+𝑚 , 𝑅𝑛 ) и если 𝐴𝑥 обратимо, то каждому вектору k ∈ 𝑅𝑚 соответствует единственный вектор h ∈ 𝑅𝑛 , такой, что 𝐴(h, k) = 0.
Этот вектор h может быть вычислен из k по формуле
(55)
h = −(𝐴𝑥 )−1 𝐴𝑦 k.
Доказательство. В силу (54) 𝐴(h, k) = 0 в том и только в том случае, когда
𝐴𝑥 h + 𝐴𝑦 k = 0,
что равносильно (55), если 𝐴𝑥 обратимо.
167
Говоря другими словами, вывод теоремы 9.27 означает, что уравнение 𝐴(h, k) = 0 может быть
разрешено (единственным образом) относительно h при данном k, и что решение h — линейная
функция вектора k. Читатели, знакомые с линейной алгеброй, найдут сходство с известным утверждением о системах линейных уравнений.
9.28 Теорема. Пусть f — 𝒞 ′ -отображение открытого множества 𝐸 ∈ 𝑅𝑛+𝑚 в 𝑅𝑛 , такое, что
f(a, b) = 0 для некоторой точки (a, b) ∈ 𝐸.
Положим 𝐴 = f′ (a, b) и допустим, что 𝐴𝑥 обратимо.
Тогда существуют открытые множества 𝑈 ∈ 𝑅𝑛+𝑚 и 𝑊 ∈ 𝑅𝑚 с (a, b) ∈ 𝑈 и b ∈ 𝑊 , обладающие следующим свойством.
Каждой точке y ∈ 𝑊 соответствует единственная точка x, такая, что
(56)
(x, y) ∈ 𝑈
и
f(x, y) = 0.
Если эту точку x обозначить через g(y), то g — 𝒞 ′ -отображение множества 𝑊 в пространство 𝑅𝑛 , g(b) = a,
(57)
f(g(y), y) = 0
(y ∈ 𝑊 )
и
(58)
g′ (b) = −(𝐴𝑥 )−1 𝐴𝑦 .
Функция g «неявно» определена равенством (57), отсюда и название теоремы.
Уравнение f(x, y) = 0 можно записать в виде системы 𝑛 уравнений с 𝑛 + 𝑚 неизвестными:
(59)
𝑓1 (𝑥1 , … , 𝑥𝑛 , 𝑦1 , … , 𝑦𝑚 ) = 0,
.............................
𝑓𝑛 (𝑥1 , … , 𝑥𝑛 , 𝑦1 , … , 𝑦𝑚 ) = 0.
Предположение об обратимости отображения 𝐴𝑥 означает, что матрица 𝑛 на 𝑛
⎡𝐷1 𝑓1 ⋯ 𝐷𝑛 𝑓1 ⎤
⎢. . . . . . . . . . . . . . . . . .⎥ ,
⎥
⎢
⎣𝐷1 𝑓𝑛 ⋯ 𝐷𝑛 𝑓𝑛 ⎦
вычисленная в точке (a, b), задает обратимый линейный оператор на 𝑅𝑛 ; другими словами, ее
векторы-столбцы должны быть независимыми или, что то же самое, ее определитель должен быть
отличен от нуля (см. теорему 9.36). Если, кроме того, (59) выполняется при x = a и y = b, то
заключение теоремы состоит в том, что (59) можно разрешить относительно 𝑥1 , … , 𝑥𝑛 , выразив их
через 𝑦1 , … , 𝑦𝑚 , при каждом y, близком к b, и что эти решения являются непрерывно дифференцируемыми функциями точки y.
Доказательство. Определим F равенством
(60)
F(x, y) = (f(x, y), y)
((x, y) ∈ 𝐸).
Тогда F — 𝒞 ′ -отображение множества 𝐸 в 𝑅𝑛+𝑚 . Мы утверждаем, что F′ (a, b) является обратимым
элементом пространства 𝐿(𝑅𝑛+𝑚 ).
Поскольку f(a, b) = 0, то
f(a + h, b + k) = 𝐴(h, k) + r(h, k),
где r — остаток, входящий в определение производной f′ (a, b). Из того, что
F(a + h, b + k) − F(a, b) = (f(a + h, b + k), k)
= (𝐴(h, k), k) + (r(h, k), 0),
следует, что F′ (a, b) — линейный оператор на 𝑅𝑛+𝑚 , который ставит в соответствие вектору (h, k)
вектор (𝐴(h, k), k). Если этот образ равен 0, то 𝐴(h, k) = 0 и k = 0, а потому 𝐴(h, 0) = 0 и из
теоремы 9.27 следует, что h = 0. Это значит, что F′ (a, b) взаимно однозначен и, следовательно,
обратим (теорема 9.5).
168
Поэтому к F применима теорема об обратной функции. Она показывает, что существуют открытые множества 𝑈 и 𝑉 в 𝑅𝑛+𝑚 с (a, b) ∈ 𝑈 , (0, b) ∈ 𝑉 , такие, что F — взаимно однозначное
отображение множества 𝑈 на 𝑉 .
Пусть 𝑊 — множество всех y ∈ 𝑅𝑚 , таких, что (0, y) ∈ 𝑉 . Отметим, что b ∈ 𝑊 .
Очевидно, что 𝑊 открыто, так как 𝑉 открыто.
Если y ∈ 𝑊 , то (0, y) = F(x, y) при некотором (x, y) ∈ 𝑈 . В силу (60) f(x, y) = 0 при таких x.
Предположим при том же y, что (x′ , y) ∈ 𝑈 и f(x′ , y) = 0. Тогда
F(x′ , y) = (f(x′ , y), y) = (f(x, y), y) = F(x, y).
Из того, что F взаимно однозначно на 𝑈 , следует, что x′ = x.
Первая часть теоремы доказана.
Для доказательства второй части определим g(y) при y ∈ 𝑊 так, чтобы (g(y), y) ∈ 𝑈 и выполнялось (57). Тогда
(61)
F(g(y), y) = (0, y)
(y ∈ 𝑊 ).
Если G — отображение множества 𝑉 в 𝑈 , обратное к F, то G ∈ 𝒞 ′ по теореме об обратной функции,
и из (61) получаем
(62)
(g(y), y) = G(0, y)
(y ∈ 𝑊 ).
Ввиду того что G ∈ 𝒞 ′ , (62) показывает, что g ∈ 𝒞 ′ .
Наконец, чтобы получить выражение для g′ (b), положим (g(y), y) = Φ(y). Тогда
(63)
Φ′ (y)k = (g′ (y)k, k)
(y ∈ 𝑊 , k ∈ 𝑅𝑚 ).
В силу (57) f(Φ(y)) = 0 на 𝑊 . По правилу дифференцирования сложной функции получаем
f′ (Φ(y))Φ′ (y) = 0.
Если y = b, то Φ(y) = (a, b) и f′ (Φ(y)) = 𝐴. Таким образом,
𝐴Φ′ (b) = 0.
(64)
Из (64), (63) и (54) теперь следует, что
𝐴𝑥 g′ (b)k + 𝐴𝑦 k = 𝐴(g′ (b)k, k) = 𝐴Φ′ (b)k = 0
при каждом k ∈ 𝑅𝑚 . Таким образом,
𝐴𝑥 g′ (b) + 𝐴𝑦 = 0.
(65)
Это равносильно (58), поэтому теорема доказана.
Замечание. Если выразить (65) через компоненты отображений f и g, получаем
𝑛
∑
(𝐷𝑗 𝑓𝑖 )(a, b)(𝐷𝑘 𝑔𝑗 )(b) = −(𝐷𝑛+𝑘 𝑓𝑖 )(a, b)
𝑗=1
или
𝑛
𝜕𝑔𝑗
𝜕𝑓𝑖
𝜕𝑓𝑖
=−
,
∑ ( 𝜕𝑥𝑗 ) ( 𝜕𝑦𝑘 )
( 𝜕𝑦𝑘 )
𝑗=1
где 1 ≤ 𝑖 ≤ 𝑛, 1 ≤ 𝑘 ≤ 𝑚.
При каждом 𝑘 это система из 𝑛 линейных уравнений, в которой неизвестными являются производные 𝜕𝑔𝑗 /𝜕𝑦𝑘 (1 ≤ 𝑗 ≤ 𝑛).
169
9.29 Пример. Возьмем 𝑛 = 2, 𝑚 = 3. Рассмотрим отображение f = (𝑓1 , 𝑓2 ) пространства 𝑅5 в 𝑅2 ,
заданное равенствами
𝑓1 (𝑥1 , 𝑥2 , 𝑦1 , 𝑦2 , 𝑦3 ) = 2𝑒𝑥1 + 𝑥2 𝑦1 − 4𝑦2 + 3,
𝑓2 (𝑥1 , 𝑥2 , 𝑦1 , 𝑦2 , 𝑦3 ) = 𝑥2 cos 𝑥1 − 6𝑥1 + 2𝑦1 − 𝑦3 .
Если a = (0, 1) и b = (3, 2, 7), то f(a, b) = 0.
Относительно стандартных базисов матрица преобразования 𝐴 = f′ (a, b) равна
[𝐴] =
2 3 1
[−6 1 2
−4
0
.
0 −1]
Значит,
[𝐴𝑥 ] =
2 3
,
[−6 1]
[𝐴𝑦 ] =
1
[2
−4
0
0
.
−1]
Мы видим, что векторы-столбцы матрицы [𝐴𝑥 ] независимы. Значит, 𝐴𝑥 обратим, и из теоремы о
неявной функции следует существование 𝒞 ′ -отображения g, определенного в окрестности точки
(3, 2, 7), такого, что g(3, 2, 7) = (0, 1) и f(g(y), y) = 0.
Для нахождения g′ (3, 2, 7) можно использовать (58): ввиду того что
[(𝐴𝑥 )−1 ] = [𝐴𝑥 ]−1 =
1 1
20 [6
−3
,
2]
из (58) следует, что
[g′ (3, 2, 7)] = −
1 1
20 [6
−3 1
2] [2
−4
0
1
0
4
=
−1] [− 1
2
1
5
6
5
3
− 20
.
1
]
10
Записывая это равенство через частные производные, приходим к выводу, что
𝐷1 𝑔1 =
1
4
𝐷1 𝑔2 = − 12
𝐷 2 𝑔1 =
𝐷 2 𝑔2 =
1
5
6
5
3
𝐷3 𝑔1 = − 20
𝐷3 𝑔2 =
1
10
в точке (3, 2, 7).
Теорема о ранге
Хотя данная теорема не настолько важна, как теорема об обратной функции или теорема о неявной
функции, мы приводим ее как еще одну интересную иллюстрацию общего принципа: локальное
поведение непрерывно дифференцируемого отображения F вблизи точки x подобно поведению
линейного преобразования F′ (x).
Для формулировки этой теоремы нам понадобятся некоторые дополнительные сведения о линейных преобразованиях.
9.30 Определения. Пусть 𝑋 и 𝑌 — векторные пространства и 𝐴 ∈ 𝐿(𝑋, 𝑌 ), как в определении 9.6.
Нуль-пространство преобразования 𝐴, обозначаемое 𝒩 (𝐴), — множество всех x ∈ 𝑋, в которых
𝐴x = 0. Очевидно, что 𝒩 (𝐴) — векторное пространство в 𝑋.
Подобным образом образ преобразования 𝐴, обозначаемый ℛ(𝐴), является векторным пространством в 𝑌 .
Рангом преобразования 𝐴 (обозначается rank 𝐴) называется размерность пространства ℛ(𝐴).
Например, обратимые элементы пространства 𝐿(𝑅𝑛 ) — в точности те элементы, которые имеют
ранг 𝑛. Это следует из теоремы 9.5.
Если 𝐴 ∈ 𝐿(𝑋, 𝑌 ) и 𝐴 имеет ранг 0, то 𝐴x = 0 при всех 𝑥 ∈ 𝐴, значит, 𝒩 (𝐴) = 𝑋. В этой связи
см. упражнение 25.
170
9.31 Проекции. Пусть 𝑋 — векторное пространство. Оператор 𝑃 ∈ 𝐿(𝑋) называется проекцией в
𝑋, если 𝑃 2 = 𝑃 .
Говоря более отчетливо, требование заключается в том, что 𝑃 (𝑃 x) = 𝑃 (x) при каждом x ∈ 𝑋.
Другими словами, 𝑃 оставляет неподвижным каждый вектор в своем образе ℛ(𝑃 ).
Вот некоторые элементарные свойства проекций.
(a) Если 𝑃 — проекция в 𝑋, то каждый элемент x ∈ 𝑋 имеет единственное представление
вида
x = x1 + x2 ,
где x1 ∈ ℛ(𝑃 ), x2 ∈ 𝒩 (𝑃 ).
Чтобы получить такое представление, положим x1 = 𝑃 x, x2 = x − x1 . Тогда 𝑃 x2 = 𝑃 x − 𝑃 x1 =
𝑃 x − 𝑃 2 x = 0. Что касается единственности, применим 𝑃 к уравнению x = x1 + x2 . Так как
x1 ∈ ℛ(𝑃 ), то 𝑃 x1 = x1 ; из того, что 𝑃 x2 = 0, следует, что x1 = 𝑃 x.
(b) Если 𝑋 — конечномерное векторное пространство и если 𝑋1 — векторное пространство в
𝑋, то существует проекция 𝑃 в 𝑋 с ℛ(𝑃 ) = 𝑋1 .
Если 𝑋1 содержит только 0, то это утверждение тривиально: положим 𝑃 x = 0 при всех x ∈ 𝑋.
Пусть dim 𝑋1 = 𝑘 > 0. По теореме 9.3 𝑋 имеет базис {u1 , … , u𝑛 }, такой, что {u1 , … , u𝑘 }
является базисом пространства 𝑋1 . Положим
𝑃 (𝑐1 u1 + ⋯ + 𝑐𝑛 u𝑛 ) = 𝑐1 u1 + ⋯ + 𝑐𝑘 u𝑘
при любых значениях скаляров 𝑐1 , … , 𝑐𝑛 .
Тогда 𝑃 x = x при любом x ∈ 𝑋1 , и 𝑋1 = ℛ(𝑃 ).
Отметим, что {u𝑘+1 , … , u𝑛 } — базис пространства 𝒩 (𝑃 ). Отметим также, что при 0 < dim 𝑋1 <
dim 𝑋 существует бесконечно много проекций в 𝑋 с образом 𝑋1 .
9.32 Теорема. Пусть 𝑚, 𝑛, 𝑟 — неотрицательные целые числа, 𝑚 ≥ 𝑟, 𝑛 ≥ 𝑟, F — 𝒞 ′ -отображение
открытого множества 𝐸 ⊂ 𝑅𝑛 в 𝑅𝑚 и F′ (x) имеет ранг 𝑟 при всех x ∈ 𝐸.
Зафиксируем a ∈ 𝐸, положим 𝐴 = F′ (a). Пусть 𝑌1 — образ преобразования 𝐴, и пусть 𝑃 —
проекция в 𝑅𝑚 , образом которой является 𝑌1 . Пусть 𝑌2 — нуль-пространство проекции 𝑃 .
Тогда существуют открытые множества 𝑈 и 𝑉 в 𝑅𝑛 с a ∈ 𝑈 , 𝑈 ⊂ 𝐸 и взаимно однозначное
′
𝒞 -отображение H пространства 𝑉 на 𝑈 (обратное к которому также принадлежит к классу
𝒞 ′ ), такие, что
(66)
F(H(x)) = 𝐴x + 𝜑(𝐴x)
(x ∈ 𝑉 ),
где 𝜑 — 𝒞 ′ -отображение открытого множества 𝐴(𝑉 ) ⊂ 𝑌1 в 𝑌2 .
После доказательства мы дадим более геометрическое объяснение сведений, содержащихся в
(66).
Доказательство. Если 𝑟 = 0, то теорема 9.19 показывает, что F(x) постоянно в окрестности 𝑈
точки a, и (66) тривиально выполняется, если взять 𝑉 = 𝑈 , H(x) = x, 𝜑(0) = F(a).
Начиная с этого места, мы предполагаем, что 𝑟 > 0. Ввиду того что dim 𝑌1 = 𝑟, 𝑌1 имеет базис
{y1 , … , y𝑟 }. Выберем z𝑖 ∈ 𝑅𝑛 так, что 𝐴z𝑖 = y𝑖 (1 ≤ 𝑖 ≤ 𝑟), и определим линейное отображение 𝑆
пространства 𝑌1 в 𝑅𝑛 , положив
(67)
𝑆(𝑐1 y1 + ⋯ + 𝑐𝑟 y𝑟 ) = 𝑐1 z1 + ⋯ + 𝑐𝑟 z𝑟
при любых значениях скаляров 𝑐1 , … , 𝑐𝑟 .
Тогда 𝐴𝑆y𝑖 = 𝐴z𝑖 = y𝑖 при 1 ≤ 𝑖 ≤ 𝑟. Таким образом,
(68)
𝐴𝑆y = y
(y ∈ 𝑌1 ).
Определим отображение G множества 𝐸 в 𝑅𝑛 , положив
(69)
G(x) = x + 𝑆𝑃 [F(x) − 𝐴x]
(x ∈ 𝐸).
Ввиду того что F′ (a) = 𝐴, дифференцирование равенства (69) показывает, что G′ (a) = 𝐼, тождественному оператору на 𝑅𝑛 . По теореме об обратной функции существуют открытые множества 𝑈
171
и 𝑉 в 𝑅𝑛 с a ∈ 𝑈 , такие, что G — взаимно однозначное отображение множества 𝑈 на 𝑉 , обратное
к которому H также принадлежит к классу 𝒞 ′ . Более того, уменьшая при необходимости 𝑈 и 𝑉 ,
мы можем добиться того, чтобы 𝑉 было выпуклым, а H′ (x) — обратимым при каждом x ∈ 𝑉 .
Отметим, что 𝐴𝑆𝑃 𝐴 = 𝐴, так как 𝑃 𝐴 = 𝐴 и выполняется (68). Следовательно, из (69) получаем
(70)
𝐴G(x) = 𝑃 F(x)
(x ∈ 𝐸).
В частности, (70) выполняется при x ∈ 𝑈 . Если заменить x на H(x), то мы получим
(71)
𝑃 F(H(x)) = 𝐴x
(x ∈ 𝑉 ).
Положим
(72)
𝜓(x) = F(H(x)) − 𝐴x
(x ∈ 𝑉 ).
Ввиду того что 𝑃 𝐴 = 𝐴, из (71) следует, что 𝑃 𝜓(x) = 0 при всех x ∈ 𝑉 . Следовательно, 𝜓 —
𝒞 ′ -отображение множества 𝑉 в 𝑌2 .
Ввиду того что 𝑉 открыто, очевидно, что 𝐴(𝑉 ) — открытое подмножество образа ℛ(𝐴) = 𝑌1 .
Чтобы завершить доказательство, т. е. перейти от (72) к (66), мы должны показать, что существует 𝒞 ′ -отображение 𝜑 множества 𝐴(𝑉 ) в 𝑌2 , удовлетворяющее равенству
(73)
𝜑(𝐴x) = 𝜓(x)
(x ∈ 𝑉 ).
В качестве шага к (73) мы вначале докажем, что
(74)
𝜓(x1 ) = 𝜓(x2 ),
если x1 ∈ 𝑉 , x2 ∈ 𝑉 , 𝐴x1 = 𝐴x2 .
Положим Φ(x) = F(H(x)) при x ∈ 𝑉 . Так как H′ (x) имеет ранг 𝑛 при всех x ∈ 𝑉 и F′ (x) имеет
ранг 𝑟 при всех x ∈ 𝑈 , то
(75)
rank Φ′ (x) = rank F′ (H(x))H′ (x) = 𝑟
(x ∈ 𝑉 ).
Зафиксируем x ∈ 𝑉 . Пусть 𝑀 — образ преобразования Φ′ (x). Тогда 𝑀 ⊂ 𝑅𝑚 , dim 𝑀 = 𝑟. Вследствие (71),
(76)
𝑃 Φ′ (x) = 𝐴.
Таким образом, 𝑃 отображает 𝑀 на ℛ(𝐴) = 𝑌1 . Так как 𝑀 и 𝑌1 имеют одинаковую размерность,
то 𝑃 (суженное на 𝑀) взаимно однозначно.
Допустим теперь, что 𝐴h = 0. Тогда 𝑃 Φ′ (x)h = 0 вследствие (76). Но Φ′ (x)h ∈ 𝑀, а 𝑃 взаимно однозначно на 𝑀. Значит, Φ′ (x)h = 0. Взгляд на (72) теперь показывает, что мы доказали
следующее.
Если x ∈ 𝑉 и 𝐴h = 0, то 𝜓 ′ (x)h = 0.
Теперь мы можем доказать (74). Пусть x1 ∈ 𝑉 , x2 ∈ 𝑉 , 𝐴x1 = 𝐴x2 . Положим h = x2 − x1 и
(77)
g(𝑡) = 𝜓(x1 + 𝑡h)
(0 ≤ 𝑡 ≤ 1).
Из выпуклости множества 𝑉 следует, что x1 + 𝑡h ∈ 𝑉 при этих 𝑡. Значит,
(78)
g′ (𝑡) = 𝜓 ′ (x1 + 𝑡h)h = 0
(0 ≤ 𝑡 ≤ 1),
так что g(1) = g(0). Но g(1) = 𝜓(x2 ), а g(0) = 𝜓(x1 ). Тем самым доказано (74).
Ввиду (74), 𝜓(x) зависит только от 𝐴x при x ∈ 𝑉 . Значит, (73) однозначно определяет 𝜑 на
𝐴(𝑉 ). Остается только доказать, что 𝜑 ∈ 𝒞 ′ .
Зафиксируем y0 ∈ 𝐴(𝑉 ) и зафиксируем x0 ∈ 𝑉 так, чтобы 𝐴x0 = y0 . Ввиду того что 𝑉 открыто,
y0 имеет окрестность 𝑊 в 𝑌1 , такую, что вектор
(79)
x = x0 + 𝑆(y − y0 )
172
лежит в 𝑉 при всех y ∈ 𝑊 . Ввиду (68),
𝐴x = 𝐴x0 + y − y0 = y.
Тогда из (73) и (79) следует
(80)
𝜑(y) = 𝜓(x0 − 𝑆y0 + 𝑆y)
(y ∈ 𝑊 ).
′
Эта формула показывает, что 𝜑 ∈ 𝒞 на 𝑊 , а значит, и на 𝐴(𝑉 ), так как y0 было выбрано произвольно на 𝐴(𝑉 ).
Теорема доказана.
О геометрии отображения F данная теорема говорит следующее.
Если y ∈ F(𝑈 ), то y = F(H(x)) при некотором x ∈ 𝑉 , и (66) показывает, что 𝑃 y = 𝐴x. Следовательно,
(81)
y = 𝑃 y + 𝜑(𝑃 y)
(y ∈ F(𝑈 )).
Это равенство показывает, что y определяется его проекцией 𝑃 y и что 𝑃 , суженное на F(𝑈 ), —
взаимно однозначное отображение множества F(𝑈 ) на 𝐴(𝑉 ). Следовательно, F(𝑈 ) — «𝑟-мерная поверхность», имеющая ровно одну точку «над» каждой точкой множества 𝐴(𝑉 ). Мы можем также
рассматривать F(𝑈 ) как график отображения 𝜑.
Если Φ(x) = F(H(x)), как в доказательстве, то (66) показывает, что множества уровня отображения Φ (множества, на которых Φ принимает постоянное значение) — это в точности множества
уровня преобразования 𝐴 на 𝑉 . Они «плоские», так как они являются пересечениями с 𝑉 параллельных переносов векторного пространства 𝒩 (𝐴). Отметим, что dim 𝒩 (𝐴) = 𝑛 − 𝑟 (упражнение
25).
Множества уровня отображения F в 𝑈 — это образы под действием отображения H плоских
множеств уровня отображения Φ в 𝑉 . Следовательно, это «(𝑛 − 𝑟)-мерные поверхности» в 𝑈 .
Определители
Определители — это числа, связанные с квадратными матрицами, а значит, и с операторами, представляемыми этими матрицами. Они равны 0 в том и только в том случае, когда соответствующий
оператор не является обратимым. Следовательно, они могут использоваться для того, чтобы решить, выполняются ли условия предыдущих теорем. Они играют еще более важную роль в гл.
10.
9.33 Определение. Если (𝑗1 , … , 𝑗𝑛 ) — упорядоченная 𝑛-ка целых чисел, то положим по определению
(82)
𝑠(𝑗1 , … , 𝑗𝑛 ) =
∏
sgn(𝑗𝑞 − 𝑗𝑝 ),
𝑝<𝑞
где sgn 𝑥 = 1, если 𝑥 > 0, sgn 𝑥 = −1, если 𝑥 < 0, sgn 𝑥 = 0, если 𝑥 = 0. Тогда 𝑠(𝑗1 , … , 𝑗𝑛 ) = 1, −1 или
0 и меняет знак, если какие-нибудь два из чисел 𝑗 меняются местами.
Пусть [𝐴] — матрица линейного оператора 𝐴 на 𝑅𝑛 относительно стандартного базиса {e1 , … , e𝑛 },
с элементами 𝑎(𝑖, 𝑗) в 𝑖-ой строке и 𝑗-ом столбце. Определителем матрицы [𝐴] называется число
(83)
det[𝐴] =
𝑠(𝑗 , … , 𝑗𝑛 )𝑎(1, 𝑗1 )𝑎(2, 𝑗2 ) ⋯ 𝑎(𝑛, 𝑗𝑛 ).
∑ 1
Суммирование в (83) производится по всем упорядоченным 𝑛-кам целых чисел (𝑗1 , … , 𝑗𝑛 ), таким,
что 1 ≤ 𝑗𝑟 ≤ 𝑛.
Векторы-столбцы x𝑗 матрицы [𝐴] равны
𝑛
(84)
x𝑗 =
∑
𝑎(𝑖, 𝑗)e𝑖
(1 ≤ 𝑗 ≤ 𝑛).
𝑖=1
Удобно представлять себе det[𝐴] как функцию векторов-столбцов матрицы [𝐴]. Если мы запишем
det(x1 , … , x𝑛 ) = det[𝐴],
то теперь det — вещественная функция на множестве всех упорядоченных 𝑛-ок векторов в 𝑅𝑛 .
173
9.34 Теорема.
(a) Если 𝐼 — тождественный оператор на 𝑅𝑛 , то
det[𝐼] = det(e1 , … , e𝑛 ) = 1.
(b) det — линейная функция каждого из векторов-столбцов x𝑗 , если остальные фиксированы.
(c) Если [𝐴]1 получается из [𝐴] перестановкой двух столбцов, то det[𝐴]1 = − det[𝐴].
(d) Если [𝐴] имеет два равных столбца, то det[𝐴] = 0.
Доказательство. Если 𝐴 = 𝐼, то 𝑎(𝑖, 𝑖) = 1, а 𝑎(𝑖, 𝑗) = 0 при 𝑖 ≠ 𝑗. Значит,
det[𝐼] = 𝑠(1, 2, … , 𝑛) = 1,
и (a) доказано. Согласно (82), 𝑠(𝑗1 , … , 𝑗𝑛 ) = 0, если какие-нибудь два из чисел 𝑗 равны. Каждое из
𝑛! остальных произведений в (83) содержит ровно по одному множителю из каждого столбца. Тем
самым доказано (b). Утверждение (c) немедленно следует из того, что 𝑠(𝑗1 , … , 𝑗𝑛 ) меняет знак, если
переставить какие-нибудь два из чисел 𝑗, а (d) следует из (c).
9.35 Теорема. Если [𝐴] и [𝐵] — матрицы 𝑛 на 𝑛, то
det([𝐵][𝐴]) = det[𝐵] det[𝐴].
Доказательство. Если x1 , … , x𝑛 — столбцы матрицы [𝐴], то положим
(85)
Δ𝐵 (x1 , … , x𝑛 ) = Δ𝐵 [𝐴] = det([𝐵][𝐴]).
Столбцы матрицы [𝐵][𝐴] — это векторы 𝐵x1 , … , 𝐵x𝑛 . Таким образом,
(86)
Δ𝐵 (x1 , … , x𝑛 ) = det(𝐵x1 , … , 𝐵x𝑛 ).
Согласно (86) и теореме 9.34, Δ𝐵 тоже обладает свойствами 9.34(b)-(d). Согласно (b) и (84),
Δ𝐵 [𝐴] = Δ𝐵
(∑
𝑖
𝑎(𝑖, 1)e𝑖 , x2 , … , x𝑚
)
=
∑
𝑎(𝑖, 1)Δ𝐵 (e𝑖 , x2 , … , x𝑚 ).
𝑖
Повторяя эту процедуру с x2 , … , x𝑛 , мы получаем
(87)
Δ𝐵 [𝐴] =
∑
𝑎(𝑖1 , 1)𝑎(𝑖2 , 2) ⋯ 𝑎(𝑖𝑛 , 𝑛)Δ𝐵 (e𝑖1 , … , e𝑖𝑛 ),
где суммирование распространяется на все упорядоченные 𝑛-ки (𝑖1 , … , 𝑖𝑛 ) с 1 ≤ 𝑖𝑟 ≤ 𝑛. Согласно (c)
и (d),
(88)
Δ𝐵 (e𝑖1 , … , e𝑖𝑛 ) = 𝑡(𝑖1 , … , 𝑖𝑛 )Δ𝐵 (e1 , … , e𝑛 ),
где 𝑡 = 1, 0 или −1, а так как [𝐵][𝐼] = [𝐵], то из (85) следует, что
(89)
Δ𝐵 (e1 , … , e𝑛 ) = det[𝐵].
Подставляя (89) и (88) в (87), мы получаем
det([𝐵][𝐴]) = { 𝑎(𝑖1 , 1) ⋯ 𝑎(𝑖𝑛 , 𝑛)𝑡(𝑖1 , … , 𝑖𝑛 )} det[𝐵],
∑
каковы бы ни были матрицы 𝑛 на 𝑛 [𝐴] и [𝐵]. Полагая 𝐵 = 𝐼, мы видим, что сумма в фигурных
скобках равна det[𝐴]. Теорема доказана.
9.36 Теорема. Линейный оператор 𝐴 на 𝑅𝑛 обратим тогда и только тогда, когда det[𝐴] ≠ 0.
174
Доказательство. Если 𝐴 обратим, то, как показывает теорема 9.35,
det[𝐴] det[𝐴−1 ] = det[𝐴𝐴−1 ] = det[𝐼] = 1,
так что det[𝐴] ≠ 0.
Если 𝐴 не обратим, то столбцы x1 , … , x𝑛 матрицы [𝐴] зависимы (теорема 9.5); следовательно,
имеется столбец, скажем, x𝑘 , такой, что
(90)
x𝑘 +
∑
𝑐𝑗 x𝑗 = 0
𝑗≠𝑘
при некоторых значениях скаляров 𝑐𝑗 . Согласно 9.34(b) и (d), столбец x𝑘 можно заменить на x𝑘 +
𝑐𝑗 x𝑗 , не меняя определителя, если 𝑗 ≠ 𝑘. Повторяя этот процесс, столбец x𝑘 можно заменить на
левую часть (90), то есть на 0, не меняя определителя. Но матрица, имеющая 0 в одном из столбцов,
имеет определитель 0. Поэтому det[𝐴] = 0.
9.37 Замечание. Пусть {e1 , … , e𝑛 } и {u1 , … , u𝑛 } — базисы в 𝑅𝑛 . Каждому линейному оператору
𝐴 на 𝑅𝑛 соответствуют матрицы [𝐴] и [𝐴]𝑈 с элементами 𝑎𝑖𝑗 и 𝛼𝑖𝑗 , задаваемыми равенствами
𝐴e𝑗 =
∑
𝑎𝑖𝑗 e𝑖 ,
𝐴u𝑗 =
𝑖
∑
𝛼𝑖𝑗 u𝑖 .
𝑖
Если u𝑗 = 𝐵e𝑗 = ∑ 𝑏𝑖𝑗 e𝑖 , то 𝐴u𝑗 равно
∑
𝑘
𝛼𝑘𝑗 𝐵e𝑘 =
∑
𝛼𝑘𝑗
∑
𝑏𝑖𝑘 e𝑖 =
𝑖
𝑘
∑ (∑
𝑖
𝑏𝑖𝑘 𝛼𝑘𝑗
𝑘
)
e𝑖 ,
а также
𝐴𝐵e𝑗 = 𝐴
∑
𝑘
𝑏𝑘𝑗 e𝑘 =
∑ (∑
𝑖
𝑘
𝑎𝑖𝑘 𝑏𝑘𝑗
)
e𝑖 .
Таким образом, ∑ 𝑏𝑖𝑘 𝛼𝑘𝑗 = ∑ 𝑎𝑖𝑘 𝑏𝑘𝑗 , или
(91)
[𝐵][𝐴]𝑈 = [𝐴][𝐵].
Поскольку 𝐵 обратим, то det[𝐵] ≠ 0. Комбинируя (91) с теоремой 9.35, получаем
(92)
det[𝐴]𝑈 = det[𝐴].
Поэтому определитель матрицы линейного оператора не зависит от базиса, который был использован при построении матрицы. Таким образом, имеет смысл говорить об определителе линейного
оператора, не имея при этом в виду никакого базиса.
9.38 Якобианы. Если f отображает открытое множество 𝐸 ⊂ 𝑅𝑛 в 𝑅𝑛 и если f дифференцируемо
в точке x ∈ 𝐸, то определитель линейного оператора f′ (x) называется якобианом отображения f в
точке x и обозначается символом
(93)
𝐽f (x) = det f′ (x).
Мы также будем использовать для 𝐽f (x) обозначение
(94)
𝜕(𝑦1 , … , 𝑦𝑛 )
,
𝜕(𝑥1 , … , 𝑥𝑛 )
если (𝑦1 , … , 𝑦𝑛 ) = f(𝑥1 , … , 𝑥𝑛 ).
Используя понятие якобиана, основное условие теоремы об обратной функции можно записать
так: 𝐽f (a) ≠ 0 (ср. с теоремой 9.36). Если теорему о неявной функции сформулировать в терминах
функций (59), то условие этой теоремы сводится к неравенству
𝜕(𝑓1 , … , 𝑓𝑛 )
≠ 0.
𝜕(𝑥1 , … , 𝑥𝑛 )
175
Производные высшего порядка
9.39 Определение. Пусть 𝑓 — вещественная функция, определенная на открытом множестве
𝐸 ⊂ 𝑅𝑛 , с частными производными 𝐷1 𝑓 , … , 𝐷𝑛 𝑓 . Если функции 𝐷𝑗 𝑓 сами дифференцируемы, то
частные производные второго порядка функции 𝑓 определяются равенствами
𝐷𝑖𝑗 𝑓 = 𝐷𝑖 𝐷𝑗 𝑓
(𝑖, 𝑗 = 1, … , 𝑛).
Если все эти функции 𝐷𝑖𝑗 𝑓 непрерывны на 𝐸, то говорят, что 𝑓 — функция класса 𝒞 ″ на 𝐸 или
что 𝑓 ∈ 𝒞 ″ (𝐸).
Говорят, что отображение 𝑓 множества 𝐸 в 𝑅𝑚 принадлежит классу 𝒞 ″ , если каждая его компонента принадлежит классу 𝒞 ″ .
Может оказаться, что в некоторой точке 𝐷𝑖𝑗 𝑓 ≠ 𝐷𝑗𝑖 𝑓 , хотя обе производные существуют (см.
упражнение 27). Однако мы увидим ниже, что 𝐷𝑖𝑗 𝑓 = 𝐷𝑗𝑖 𝑓 , если эти производные непрерывны.
Для простоты (и не умаляя общности) мы сформулируем две наши следующие теоремы для
функций двух переменных. Первая — теорема о среднем значении.
9.40 Теорема. Пусть 𝑓 определена на открытом множестве 𝐸 ⊂ 𝑅2 и 𝐷1 𝑓 и 𝐷21 𝑓 существуют в каждой точке множества 𝐸. Пусть 𝑄 ⊂ 𝐸 — замкнутый прямоугольник со сторонами,
параллельными осям координат, и противоположными вершинами (𝑎, 𝑏) и (𝑎 + ℎ, 𝑏 + 𝑘) (ℎ ≠ 0,
𝑘 ≠ 0). Положим
Δ(𝑓 , 𝑄) = 𝑓 (𝑎 + ℎ, 𝑏 + 𝑘) − 𝑓 (𝑎 + ℎ, 𝑏) − 𝑓 (𝑎, 𝑏 + 𝑘) + 𝑓 (𝑎, 𝑏).
Тогда во внутренности прямоугольника 𝑄 существует точка (𝑥, 𝑦), такая, что
(95)
Δ(𝑓 , 𝑄) = ℎ𝑘(𝐷21 𝑓 )(𝑥, 𝑦).
Отметим аналогию между (95) и теоремой 5.10; ℎ𝑘 — площадь прямоугольника 𝑄.
Доказательство. Положим 𝑢(𝑡) = 𝑓 (𝑡, 𝑏 + 𝑘) − 𝑓 (𝑡, 𝑏). Два применения теоремы 5.10 показывают,
что существует 𝑥 между 𝑎 и 𝑎 + ℎ, а также 𝑦 между 𝑏 и 𝑏 + 𝑘, такие, что
Δ(𝑓 , 𝑄) = 𝑢(𝑎 + ℎ) − 𝑢(𝑎)
= ℎ𝑢′ (𝑥)
= ℎ[(𝐷1 𝑓 )(𝑥, 𝑏 + 𝑘) − (𝐷1 𝑓 )(𝑥, 𝑏)]
= ℎ𝑘(𝐷21 𝑓 )(𝑥, 𝑦).
9.41 Теорема. Пусть 𝑓 определена на открытом множестве 𝐸 ⊂ 𝑅2 , пусть 𝐷1 𝑓 , 𝐷21 𝑓 и 𝐷2 𝑓
существуют в каждой точке множества 𝐸 и 𝐷21 𝑓 непрерывна в некоторой точке (𝑎, 𝑏) ∈ 𝐸.
Тогда 𝐷12 𝑓 существует в точке (𝑎, 𝑏) и
(96)
(𝐷12 𝑓 )(𝑎, 𝑏) = (𝐷21 𝑓 )(𝑎, 𝑏).
Следствие. 𝐷21 𝑓 = 𝐷12 𝑓 , если 𝑓 ∈ 𝒞 ″ (𝐸).
Доказательство. Положим 𝐴 = (𝐷21 𝑓 )(𝑎, 𝑏). Выберем 𝜀 > 0. Если 𝑄 — прямоугольник, как в
теореме 9.40, а ℎ и 𝑘 достаточно малы, то мы имеем
|𝐴 − (𝐷21 𝑓 )(𝑥, 𝑦)| < 𝜀
при всех (𝑥, 𝑦) ∈ 𝑄. Таким образом,
|
Δ(𝑓 , 𝑄)
−𝐴 <𝜀
|
ℎ𝑘
в силу (95). Зафиксируем ℎ и положим 𝑘 → 0. Ввиду того что 𝐷2 𝑓 существует на 𝐸, из последнего
неравенства следует, что
(97)
(𝐷2 𝑓 )(𝑎 + ℎ, 𝑏) − (𝐷2 𝑓 )(𝑎, 𝑏)
− 𝐴 ≤ 𝜀.
|
|
ℎ
Так как 𝜀 было произвольным и так как (97) выполняется при всех достаточно малых ℎ ≠ 0, то
(𝐷12 𝑓 )(𝑎, 𝑏) = 𝐴. Из этого следует (96).
176
Дифференцирование под знаком интеграла
Пусть 𝜑 — функция двух переменных, которую можно интегрировать по одной переменной и дифференцировать по другой. При каких условиях результат останется тем же, если операции предельного перехода будут выполнены в другом порядке? Более точная формулировка: при каких
условиях на 𝜑 можно доказать, что верно равенство
𝑏
𝑏
𝜕𝜑
𝑑
𝜑(𝑥, 𝑡) 𝑑𝑥 =
(𝑥, 𝑡) 𝑑𝑥
∫
𝑑𝑡 ∫
𝑎
𝑎 𝜕𝑡
(98)
(контрпример представлен в упражнении 28)?
Будет удобно использовать обозначение
𝜑𝑡 (𝑥) = 𝜑(𝑥, 𝑡).
(99)
Таким образом, 𝜑𝑡 при каждом 𝑡 — функция одной переменной.
9.42 Теорема. Пусть
(a)
(b)
(c)
(d)
𝜑(𝑥, 𝑡) определена при 𝑎 ≤ 𝑥 ≤ 𝑏, 𝑐 ≤ 𝑡 ≤ 𝑑;
𝛼 — возрастающая функция на [𝑎, 𝑏];
𝜑𝑡 ∈ ℛ(𝛼) при каждом 𝑡 ∈ [𝑐, 𝑑];
𝑐 < 𝑠 < 𝑑 и каждому 𝜀 > 0 соответствует 𝛿 > 0, такое, что
|(𝐷2 𝜑)(𝑥, 𝑡) − (𝐷2 𝜑)(𝑥, 𝑠)| < 𝜀
при всех 𝑥 ∈ [𝑎, 𝑏] и при всех 𝑡 ∈ (𝑠 − 𝛿, 𝑠 + 𝛿).
Положим
𝑏
(100)
𝑓 (𝑡) =
∫
𝑎
𝜑(𝑥, 𝑡) 𝑑𝛼(𝑥)
(𝑐 ≤ 𝑡 ≤ 𝑑).
Тогда (𝐷2 𝜑)𝑠 ∈ ℛ(𝛼), 𝑓 ′ (𝑠) существует и
𝑓 ′ (𝑠) =
(101)
𝑏
∫
𝑎
(𝐷2 𝜑)(𝑥, 𝑠) 𝑑𝛼(𝑥).
Отметим, что (c) просто утверждает существование интегралов (100) при всех 𝑡 ∈ [𝑐, 𝑑]. Отметим также, что (d) наверняка выполняется, если 𝐷2 𝜑 непрерывна на прямоугольнике, на котором
определена 𝜑.
Доказательство. Рассмотрим разностные отношения
𝜓(𝑥, 𝑡) =
𝜑(𝑥, 𝑡) − 𝜑(𝑥, 𝑠)
𝑡−𝑠
при 0 < |𝑡 − 𝑠| < 𝛿. По теореме 5.10 каждой точке (𝑥, 𝑡) соответствует число 𝑢 между 𝑠 и 𝑡, такое, что
𝜓(𝑥, 𝑡) = (𝐷2 𝜑)(𝑥, 𝑢).
Таким образом, из (d) следует, что
(102)
|𝜓(𝑥, 𝑡) − (𝐷2 𝜑)(𝑥, 𝑠)| < 𝜀
(𝑎 ≤ 𝑥 ≤ 𝑏,
0 < |𝑡 − 𝑠| < 𝛿).
Отметим, что
(103)
𝑏
𝑓 (𝑡) − 𝑓 (𝑠)
𝜓(𝑥, 𝑡) 𝑑𝛼(𝑥).
=
∫
𝑡−𝑠
𝑎
В силу (102) 𝜓 𝑡 → (𝐷2 𝜑)𝑠 равномерно на [𝑎, 𝑏] при 𝑡 → 𝑠. Ввиду того что каждая 𝜓 𝑡 ∈ ℛ(𝛼),
требуемый вывод следует из (103) и теоремы 7.16.
177
9.43 Пример. Конечно, можно доказать аналоги теоремы 9.42 на (−∞, ∞) вместо [𝑎, 𝑏]. Вместо
этого мы просто разберем пример. Положим
∞
(104)
𝑓 (𝑡) =
2
𝑒−𝑥 cos(𝑥𝑡) 𝑑𝑥
∫
−∞
и
∞
(105)
𝑔(𝑡) = −
∫
−∞
2
𝑥𝑒−𝑥 sin(𝑥𝑡) 𝑑𝑥
при −∞ < 𝑡 < ∞. Оба интеграла существуют (они сходятся абсолютно), так как абсолютные значения подынтегральных выражений не превышают соответственно exp(−𝑥2 ) и |𝑥| exp(−𝑥2 ).
Отметим, что 𝑔 получена из 𝑓 дифференцированием подынтегрального выражения по 𝑡. Мы
утверждаем, что 𝑓 дифференцируема и что
𝑓 ′ (𝑡) = 𝑔(𝑡)
(106)
(−∞ < 𝑡 < ∞).
Чтобы доказать это, вначале оценим разностные отношения косинуса: если 𝛽 > 0, то
(107)
𝛼+𝛽
cos(𝛼 + 𝛽) − cos 𝛼
1
+ sin 𝛼 =
(sin 𝛼 − sin 𝑡) 𝑑𝑡.
𝛽
𝛽∫
𝛼
Ввиду того что | sin 𝛼 − sin 𝑡| ≤ |𝑡 − 𝛼|, правая часть (107) не превышает 𝛽/2 по абсолютной величине;
случай 𝛽 < 0 аналогичен. Таким образом,
(108)
|
cos(𝛼 + 𝛽) − cos 𝛼
+ sin 𝛼 ≤ |𝛽|
|
𝛽
при всех 𝛽 (если считать левую сторону равной 0, когда 𝛽 = 0).
Теперь зафиксируем 𝑡 и зафиксируем ℎ ≠ 0. Применим (108) с 𝛼 = 𝑥𝑡, 𝛽 = 𝑥ℎ; из (104) и (105)
следует, что
∞
2
𝑓 (𝑡 + ℎ) − 𝑓 (𝑡)
− 𝑔(𝑡) ≤ |ℎ|
𝑥2 𝑒−𝑥 𝑑𝑥.
|
|
∫
ℎ
−∞
Если ℎ → 0, то мы получаем (106).
Сделаем еще один шаг: проинтегрировав (104) по частям, получим
∞
(109)
𝑓 (𝑡) = 2
∫
−∞
𝑥𝑒−𝑥
2
sin(𝑥𝑡)
𝑑𝑥.
𝑡
Таким образом, 𝑡𝑓 (𝑡) = −2𝑔(𝑡), и из (106) теперь следует, что 𝑓 удовлетворяет дифференциальному
уравнению
(110)
2𝑓 ′ (𝑡) + 𝑡𝑓 (𝑡) = 0.
Если мы решим это дифференциальное уравнение и используем тот факт, что 𝑓 (0) = √𝜋 (см. п.
8.21), мы получим, что
(111)
𝑡2
𝑓 (𝑡) = √𝜋 exp −
.
( 4)
Таким образом, интеграл (104) вычислен в явном виде.
Упражнения
1. Пусть 𝑆 — непустое подмножество векторного пространства 𝑋. Доказать (как утверждается в п. 9.1),
что оболочка множества 𝑆 является векторным пространством.
2. Доказать (как утверждается в п. 9.6), что 𝐵𝐴 линейно, если 𝐴 и 𝐵 — линейные преобразования.
Доказать также, что 𝐴−1 линейно и обратимо.
3. Пусть 𝐴 ∈ 𝐿(𝑋, 𝑌 ) и 𝐴x = 0, только если x = 0. Доказать, что 𝐴 взаимно однозначно.
178
4. Доказать (как утверждается в п. 9.30), что нуль-пространства и образы линейных преобразований являются векторными пространствами.
5. Доказать, что каждому 𝐴 ∈ 𝐿(𝑅𝑛 , 𝑅1 ) соответствует единственный вектор y, такой, что 𝐴x = x ⋅ y.
Доказать также, что ‖𝐴‖ = |y|.
Указание. При некоторых условиях неравенство Шварца превращается в равенство.
6. Пусть 𝑓 (0, 0) = 0 и
𝑥𝑦
𝑓 (𝑥, 𝑦) = 2
,
если (𝑥, 𝑦) ≠ (0, 0).
𝑥 + 𝑦2
7.
8.
9.
10.
11.
Доказать, что (𝐷1 𝑓 )(𝑥, 𝑦) и (𝐷2 𝑓 )(𝑥, 𝑦) существуют в каждой точке пространства 𝑅2 , хотя 𝑓 не является
непрерывной в (0, 0).
Пусть 𝑓 — вещественнозначная функция, определенная на открытом множестве 𝐸 ⊂ 𝑅𝑛 , и пусть частные
производные 𝐷1 𝑓 , … , 𝐷𝑛 𝑓 ограничены на 𝐸. Доказать, что 𝑓 непрерывна на 𝐸.
Указание. Действовать так же, как при доказательстве теоремы 9.21.
Пусть 𝑓 — дифференцируемая вещественная функция на открытом множестве 𝐸 ⊂ 𝑅𝑛 , и пусть 𝑓 имеет
локальный максимум в точке x ∈ 𝐸. Доказать, что 𝑓 ′ (x) = 0.
Пусть f — дифференцируемое отображение связного открытого множества 𝐸 ⊂ 𝑅𝑛 в 𝑅𝑚 , и пусть f′ (x) = 0
при всех x ∈ 𝐸. Доказать, что f постоянна на 𝐸.
Пусть 𝑓 — вещественная функция, определенная на выпуклом открытом множестве 𝐸 ⊂ 𝑅𝑛 , такая, что
(𝐷1 𝑓 )(x) = 0 при каждом x ∈ 𝐸. Доказать, что 𝑓 (x) зависит только от 𝑥2 , … , 𝑥𝑛 .
Показать, что выпуклость множества 𝐸 можно заменить более слабым условием, но что какое-то условие
все-таки требуется. Например, если 𝑛 = 2, а 𝐸 имеет форму подковы, то утверждение может и не быть
верным.
Пусть 𝑓 и 𝑔 — дифференцируемые вещественные функции на 𝑅𝑛 . Доказать, что
∇(𝑓 𝑔) = 𝑓 ∇𝑔 + 𝑔∇𝑓
и что ∇(1/𝑓 ) = −𝑓 −2 ∇𝑓 , если 𝑓 ≠ 0.
12. Зафиксируем два вещественных числа 𝑎 и 𝑏, 0 < 𝑎 < 𝑏. Определим отображение f = (𝑓1 , 𝑓2 , 𝑓3 ) плоскости
𝑅2 в 𝑅3 равенствами
𝑓1 (𝑠, 𝑡) = (𝑏 + 𝑎 cos 𝑠) cos 𝑡,
𝑓2 (𝑠, 𝑡) = (𝑏 + 𝑎 cos 𝑠) sin 𝑡,
𝑓3 (𝑠, 𝑡) = 𝑎 sin 𝑠.
Описать множество значений 𝐾 отображения f (это определенное компактное подмножество пространства 𝑅3 ).
(a) Показать, что существует ровно 4 точки p ∈ 𝐾, таких, что
(∇𝑓1 )(f−1 (p)) = 0.
Найти эти точки.
(b) Определить множество всех q ∈ 𝐾, таких, что
(∇𝑓3 )(f−1 (q)) = 0.
(c) Показать, что одна из точек p, найденных в части (a), соответствует локальному максимуму функции 𝑓1 , одна — локальному минимуму, а две оставшиеся — ни тому, ни другому (это так называемые «седловые точки»).
Какие из точек q, найденных в части (b), соответствуют максимумам или минимумам?
(d) Пусть 𝜆 — иррациональное вещественное число. Положим g(𝑡) = f(𝑡, 𝜆𝑡). Доказать, что g — взаимно
однозначное отображение прямой 𝑅1 на всюду плотное подмножество множества 𝐾. Доказать, что
|g′ (𝑡)|2 = 𝑎2 + 𝜆2 (𝑏 + 𝑎 cos 𝑡)2 .
13. Пусть f — дифференцируемое отображение прямой 𝑅1 в 𝑅3 , такое, что |f(𝑡)| = 1 при каждом 𝑡. Доказать,
что f′ (𝑡) ⋅ f(𝑡) = 0.
Истолковать этот результат геометрически.
14. Положим 𝑓 (0, 0) = 0 и
𝑥3
𝑓 (𝑥, 𝑦) = 2
,
если (𝑥, 𝑦) ≠ (0, 0).
𝑥 + 𝑦2
(a) Доказать, что 𝐷1 𝑓 и 𝐷2 𝑓 — ограниченные функции на 𝑅2 (значит, 𝑓 непрерывна).
(b) Пусть u — произвольный единичный вектор в 𝑅2 . Показать, что производная по направлению
(𝐷u 𝑓 )(0, 0) существует и что ее абсолютная величина не превышает 1.
179
(c) Пусть 𝛾 — дифференцируемое отображение прямой 𝑅1 в 𝑅2 (другими словами, 𝛾 — дифференцируемая кривая в 𝑅2 ) с 𝛾(0) = (0, 0) и |𝛾 ′ (0)| > 0. Положим 𝑔(𝑡) = 𝑓 (𝛾(𝑡)). Доказать, что 𝑔 дифференцируема при любом 𝑡 ∈ 𝑅1 .
Если 𝛾 ∈ 𝒞 ′ , доказать, что 𝑔 ∈ 𝒞 ′ .
(d) Доказать, что, несмотря на это, 𝑓 не дифференцируема в (0, 0).
Указание. Формула (40) не выполняется.
15. Положим 𝑓 (0, 0) = 0 и
𝑓 (𝑥, 𝑦) = 𝑥2 + 𝑦2 − 2𝑥2 𝑦 −
4𝑥6 𝑦2
,
(𝑥4 + 𝑦2 )2
если (𝑥, 𝑦) ≠ (0, 0).
(a) Доказать, что для любой точки (𝑥, 𝑦) ∈ 𝑅2 верно неравенство
4𝑥4 𝑦2 ≤ (𝑥4 + 𝑦2 )2 .
Сделать вывод о том, что 𝑓 непрерывна.
(b) При 0 ≤ 𝜃 ≤ 2𝜋, −∞ < 𝑡 < ∞ положим
𝑔𝜃 (𝑡) = 𝑓 (𝑡 cos 𝜃, 𝑡 sin 𝜃).
Показать, что 𝑔𝜃 (0) = 0, 𝑔𝜃′ (0) = 0, 𝑔𝜃″ (0) = 2. Следовательно, каждая 𝑔𝜃 имеет строгий локальный
минимум при 𝑡 = 0.
Другими словами, сужение функции 𝑓 на каждую прямую, проходящую через (0, 0), имеет строгий
локальный минимум в (0, 0).
(c) Показать, что (0, 0) тем не менее не является локальным минимумом функции 𝑓 , т. к. 𝑓 (𝑥, 𝑥2 ) = −𝑥4 .
16. Показать, что непрерывность производной f′ в точке a существенна в теореме об обратной функции,
даже в том случае, когда 𝑛 = 1. Если
1
𝑓 (𝑡) = 𝑡 + 2𝑡2 sin ( )
𝑡
при 𝑡 ≠ 0 и 𝑓 (0) = 0, то 𝑓 ′ (0) = 1, 𝑓 ′ ограничена на (−1, 1), но 𝑓 не является взаимно однозначной ни в
какой окрестности точки 0.
17. Пусть f = (𝑓1 , 𝑓2 ) — отображение плоскости 𝑅2 в 𝑅2 , заданное равенствами
𝑓1 (𝑥, 𝑦) = 𝑒𝑥 cos 𝑦,
𝑓2 (𝑥, 𝑦) = 𝑒𝑥 sin 𝑦.
(a) Каково множество значений отображения f?
(b) Показать, что якобиан отображения f отличен от нуля во всех точках плоскости 𝑅2 . Таким образом,
каждая точка плоскости 𝑅2 обладает окрестностью, на которой f взаимно однозначно. Тем не менее
f не является взаимно однозначным на 𝑅2 .
(c) Положим a = (0, 𝜋/3), b = f(a). Пусть g — непрерывная обратная функция к f, определенная в
окрестности точки b, такая, что g(b) = a. Найти явную формулу для g, вычислить f′ (a) и g′ (b) и
проверить формулу (52).
(d) Каковы образы прямых, параллельных осям координат, при отображении f?
18. Исследовать аналогичным образом отображение, заданное равенствами
𝑢 = 𝑥 2 − 𝑦2 ,
𝑣 = 2𝑥𝑦.
19. Показать, что из системы уравнений
3𝑥 + 𝑦 − 𝑧 + 𝑢2 = 0,
𝑥 − 𝑦 + 2𝑧 + 𝑢 = 0,
2𝑥 + 2𝑦 − 3𝑧 + 2𝑢 = 0
можно выразить 𝑥, 𝑦, 𝑢 через 𝑧; 𝑥, 𝑧, 𝑢 через 𝑦; 𝑦, 𝑧, 𝑢 через 𝑥; но нельзя выразить 𝑥, 𝑦, 𝑧 через 𝑢.
20. Положить 𝑛 = 𝑚 = 1 в теореме о неявной функции и истолковать эту теорему (а также ее доказательство)
графически.
21. Положим функцию 𝑓 на 𝑅2 равной
𝑓 (𝑥, 𝑦) = 2𝑥3 − 3𝑥2 + 2𝑦3 + 3𝑦2 .
(a) Найти четыре точки в 𝑅2 , в которых градиент функции 𝑓 равен нулю. Показать, что 𝑓 имеет
ровно один локальный максимум и один локальный минимум на 𝑅2 .
180
(b) Пусть 𝑆 — множество всех (𝑥, 𝑦) ∈ 𝑅2 , в которых 𝑓 (𝑥, 𝑦) = 0. Найти точки множества 𝑆, не имеющие
окрестностей, в которых уравнение 𝑓 (𝑥, 𝑦) = 0 можно решить, выразив 𝑦 через 𝑥 (или 𝑥 через 𝑦).
Описать 𝑆 так точно, насколько это возможно.
22. Исследовать аналогичным образом функцию
𝑓 (𝑥, 𝑦) = 2𝑥3 + 6𝑥𝑦2 − 3𝑥2 + 3𝑦2 .
23. Положим функцию 𝑓 на 𝑅3 равной
𝑓 (𝑥, 𝑦1 , 𝑦2 ) = 𝑥2 𝑦1 + 𝑒𝑥 + 𝑦2 .
Показать, что 𝑓 (0, 1, −1) = 0, (𝐷1 𝑓 )(0, 1, −1) ≠ 0 и что, следовательно, существует дифференцируемая
функция 𝑔 в некоторой окрестности точки (1, −1) в 𝑅2 , такая, что 𝑔(1, −1) = 0 и
𝑓 (𝑔(𝑦1 , 𝑦2 ), 𝑦1 , 𝑦2 ) = 0.
Найти (𝐷1 𝑔)(1, −1) и (𝐷2 𝑔)(1, −1).
24. При (𝑥, 𝑦) ≠ (0, 0) положим f = (𝑓1 , 𝑓2 ), где
𝑓1 (𝑥, 𝑦) =
𝑥2 − 𝑦 2
,
𝑥2 + 𝑦 2
𝑓2 (𝑥, 𝑦) =
𝑥𝑦
.
𝑥2 + 𝑦2
Вычислить ранг преобразования f′ (𝑥, 𝑦) и найти множество значений отображения f.
25. Пусть 𝐴 ∈ 𝐿(𝑅𝑛 , 𝑅𝑚 ) и 𝑟 — ранг преобразования 𝐴.
(a) Определим 𝑆, как в доказательстве теоремы 9.32. Показать, что 𝑆𝐴 — проекция в 𝑅𝑛 , нуль-пространство
которой совпадает с 𝒩 (𝐴), а множество значений — с ℛ(𝑆). Указание. В силу (68) 𝑆𝐴𝑆𝐴 = 𝑆𝐴.
(b) Используя (a), доказать, что
dim 𝒩 (𝐴) + dim ℛ(𝐴) = 𝑛.
26. Показать, что из существования (и даже из непрерывности) производной 𝐷12 𝑓 не следует существование
производной 𝐷1 𝑓 . Например, пусть 𝑓 (𝑥, 𝑦) = 𝑔(𝑥), где 𝑔 нигде не дифференцируема.
27. Пусть 𝑓 (0, 0) = 0 и
𝑥𝑦(𝑥2 − 𝑦2 )
,
𝑓 (𝑥, 𝑦) =
𝑥2 + 𝑦2
если (𝑥, 𝑦) ≠ (0, 0). Доказать, что
(a) 𝑓 , 𝐷1 𝑓 и 𝐷2 𝑓 непрерывны на 𝑅2 ;
(b) 𝐷12 𝑓 и 𝐷21 𝑓 существуют в каждой точке плоскости 𝑅2 и непрерывны всюду, кроме точки (0, 0);
(c) (𝐷12 𝑓 )(0, 0) = 1, а (𝐷21 𝑓 )(0, 0) = −1.
28. При 𝑡 ≥ 0 положим
(0 ≤ 𝑥 ≤ √𝑡),
⎧𝑥
⎪
𝜑(𝑥, 𝑡) = ⎨−𝑥 + 2√𝑡 (√𝑡 ≤ 𝑥 ≤ 2√𝑡),
⎪0
(в прочих случаях),
⎩
и положим 𝜑(𝑥, 𝑡) = −𝜑(𝑥, |𝑡|), если 𝑡 < 0.
Показать, что 𝜑 непрерывна на 𝑅2 и
(𝐷2 𝜑)(𝑥, 0) = 0
при всех 𝑥. Положим
1
𝑓 (𝑡) =
∫
−1
𝜑(𝑥, 𝑡) 𝑑𝑥.
Показать, что 𝑓 (𝑡) = 𝑡, если |𝑡| < 14 . Таким образом,
1
𝑓 ′ (0) ≠
∫
−1
(𝐷2 𝜑)(𝑥, 0) 𝑑𝑥.
29. Пусть 𝐸 — открытое множество в 𝑅𝑛 . Классы 𝒞 ′ (𝐸) и 𝒞 ″ (𝐸) определены в тексте. По индукции можно
определить 𝒞 (𝑘) (𝐸) для положительных целых чисел 𝑘 следующим образом: утверждение 𝑓 ∈ 𝒞 (𝑘) (𝐸)
означает, что частные производные 𝐷1 𝑓 , … , 𝐷𝑛 𝑓 принадлежат к 𝒞 (𝑘−1) (𝐸).
Пусть 𝑓 ∈ 𝒞 (𝑘) (𝐸). Показать (повторным применением теоремы 9.41), что производная 𝑘-го порядка
𝐷𝑖1 𝑖2 …𝑖𝑘 𝑓 = 𝐷𝑖1 𝐷𝑖2 … 𝐷𝑖𝑘 𝑓
не меняется при перестановке индексов 𝑖1 , … , 𝑖𝑘 .
В частности, если 𝑛 ≥ 3, то
𝐷1213 𝑓 = 𝐷3112 𝑓
для каждой функции 𝑓 ∈ 𝒞 (4) .
181
30. Пусть 𝑓 ∈ 𝒞 (𝑚) (𝐸), где 𝐸 — открытое подмножество пространства 𝑅𝑛 . Зафиксируем a ∈ 𝐸. Пусть точка
x ∈ 𝑅𝑛 достаточно близка к 0 для того, чтобы точки
p(𝑡) = a + 𝑡x
лежали в 𝐸, если 0 ≤ 𝑡 ≤ 1. Положим
ℎ(𝑡) = 𝑓 (p(𝑡))
1
при всех 𝑡 ∈ 𝑅 , для которых p(𝑡) ∈ 𝐸.
(a) При 1 ≤ 𝑘 ≤ 𝑚 показать (повторным применением правила дифференцирования сложной функции), что
ℎ(𝑘) (𝑡) =
(𝐷
𝑓 )(p(𝑡))𝑥𝑖1 ⋯ 𝑥𝑖𝑘 .
∑ 𝑖1 …𝑖𝑘
Суммирование производится по всем упорядоченным наборам из 𝑘 чисел (𝑖1 , … , 𝑖𝑘 ), в которых
каждое 𝑖𝑗 — одно из чисел 1, … , 𝑛.
(b) По теореме Тейлора (5.15)
𝑚−1
ℎ(1) =
ℎ(𝑘) (0) ℎ(𝑚) (𝑡)
+
∑ 𝑘!
𝑚!
𝑘=0
при некотором 𝑡 ∈ (0, 1). Использовать это для доказательства теоремы Тейлора для 𝑛 переменных,
показав, что формула
𝑚−1
𝑓 (a + x) =
1
(𝐷
𝑓 )(a)𝑥𝑖1 ⋯ 𝑥𝑖𝑘 + 𝑟(x)
∑ 𝑘! ∑ 𝑖1 …𝑖𝑘
𝑘=0
представляет 𝑓 (a+x) в виде суммы так называемого «многочлена Тейлора степени 𝑚−1» и остатка,
удовлетворяющего условию
𝑟(x)
= 0.
lim
x→0 |x|𝑚−1
Суммирование каждой из внутренних сумм производится по всем упорядоченным 𝑘-ам (𝑖1 , … , 𝑖𝑘 ),
как в части (a); как обычно, производная нулевого порядка функции 𝑓 — сама 𝑓 , так что свободный
член многочлена Тейлора функции 𝑓 в a равен 𝑓 (a).
(c) Упражнение 29 показывает, что в многочлене Тейлора, выписанном в части (b), встречаются повторения. В частности, 𝐷113 встречается три раза, в виде 𝐷113 , 𝐷131 , 𝐷311 . Сумма трех соответствующих
членов может быть записана в виде
3(𝐷12 𝐷3 𝑓 )(a)𝑥21 𝑥3 .
Доказать (подсчитав, сколько раз встречается каждая производная), что многочлен Тейлора в
части (b) может быть записан в виде
𝑠
𝑠
(𝐷11 ⋯ 𝐷𝑛𝑛 𝑓 )(a) 𝑠1
𝑠
𝑥1 ⋯ 𝑥𝑛𝑛 .
∑
𝑠1 ! ⋯ 𝑠𝑛 !
Здесь суммирование производится по всем упорядоченным 𝑛-кам (𝑠1 , … , 𝑠𝑛 ), таким, что каждое
𝑠𝑖 — неотрицательное целое число и 𝑠1 + ⋯ + 𝑠𝑛 ≤ 𝑚 − 1.
31. Пусть 𝑓 ∈ 𝒞 (3) в некоторой окрестности точки a ∈ 𝑅2 , градиент функции 𝑓 равен 0 в a, но не все
производные второго порядка функции 𝑓 равны 0 в a. Показать, как можно определить по многочлену
Тейлора функции 𝑓 в a (степени 2), имеет ли 𝑓 локальный максимум, локальный минимум либо не
имеет ни того, ни другого в точке a.
Обобщить данное наблюдение на 𝑅𝑛 вместо 𝑅2 .
182
Глава 10
Интегрирование
дифференциальных форм
Интегрирование можно изучать на многих уровнях. В гл. 6 была разработана теория для разумно
ведущих себя функций на сегментах вещественной прямой. В гл. 11 мы встретимся с хорошо разработанной теорией интегрирования, которая может применяться к гораздо более широким классам
функций, определенных на более или менее произвольных множествах, необязательно являющихся
подмножествами пространств 𝑅𝑛 . Настоящая глава посвящена вопросам теории интегрирования,
тесно связанным с геометрией евклидовых пространств, таким, как формула замены переменных,
криволинейные интегралы и механизм дифференциальных форм, используемый в условии и доказательстве 𝑛-мерного аналога основной теоремы интегрального исчисления, называемого теоремой
Стокса.
Интегрирование
10.1 Определение. Пусть 𝐼 𝑘 — 𝑘-клетка в 𝑅𝑘 , состоящая из всех
x = (𝑥1 , … , 𝑥𝑘 ),
таких, что
(1)
𝑎𝑖 ≤ 𝑥𝑖 ≤ 𝑏𝑖
(𝑖 = 1, … , 𝑘),
𝐼 𝑗 — 𝑗-клетка в 𝑅𝑗 , определяемая первыми 𝑗 из неравенств (1), а 𝑓 — вещественная непрерывная
функция на 𝐼 𝑘 .
Положим 𝑓 = 𝑓𝑘 и определим 𝑓𝑘−1 на 𝐼 𝑘−1 равенством
𝑏𝑘
𝑓𝑘−1 (𝑥1 , … , 𝑥𝑘−1 ) =
∫
𝑎𝑘
𝑓𝑘 (𝑥1 , … , 𝑥𝑘−1 , 𝑥𝑘 ) 𝑑𝑥𝑘 .
Из равномерной непрерывности функции 𝑓𝑘 на 𝐼𝑘 следует, что 𝑓𝑘−1 непрерывна на 𝐼 𝑗 . Поэтому
мы можем продолжить процесс и получим функции 𝑓𝑗 , непрерывные на 𝐼 𝑗 , такие, что 𝑓𝑗−1 есть
интеграл от 𝑓𝑗 относительно 𝑥𝑗 по [𝑎𝑗 , 𝑏𝑗 ]. После 𝑘 шагов мы придем к числу 𝑓0 , которое называется
интегралом функции 𝑓 по 𝑘-клетке 𝐼 𝑘 и записывается в виде
(2)
∫
𝐼𝑘
𝑓 (x) 𝑑x
или
∫
𝐼𝑘
𝑓.
На первый взгляд это определение интеграла зависит от порядка, в котором производятся 𝑘 интегрирований. Однако это только кажущаяся зависимость. Чтобы доказать это, введем временно
обозначение 𝐿(𝑓 ) для интеграла (2) и 𝐿′ (𝑓 ) для результата, получаемого выполнением 𝑘 интегрирований в каком-либо ином порядке.
183
10.2 Теорема. Какова бы ни была функция 𝑓 ∈ 𝒞 (𝐼 𝑘 ), имеем 𝐿(𝑓 ) = 𝐿′ (𝑓 ).
Доказательство. Если ℎ(x) = ℎ1 (𝑥1 ) ⋯ ℎ𝑘 (𝑥𝑘 ), где ℎ𝑗 ∈ 𝒞 ([𝑎𝑗 , 𝑏𝑗 ]), то
𝑘
𝐿(ℎ) =
𝑏𝑖
∏∫
𝑎
𝑖=1
ℎ𝑖 (𝑥𝑖 ) 𝑑𝑥𝑖 = 𝐿′ (ℎ).
𝑖
Если 𝒜 — множество всех конечных сумм таких функций ℎ, то 𝐿(𝑔) = 𝐿′ (𝑔) при всех 𝑔 ∈ 𝒜 . Кроме
того, 𝒜 — алгебра функций на 𝐼 𝑘 , к которой применима теорема Стоуна-Вейерштрасса.
Положим 𝑉 = ∏𝑘1 (𝑏𝑖 − 𝑎𝑖 ). Если 𝑓 ∈ 𝒞 (𝐼 𝑘 ) и 𝜀 > 0, то существует функция 𝑔 ∈ 𝒜 , такая, что
‖𝑓 − 𝑔‖ < 𝜀/𝑉 , где ‖𝑓 ‖ определяется как max |𝑓 (x)| (x ∈ 𝐼 𝑘 ). Тогда |𝐿(𝑓 − 𝑔)| < 𝜀, |𝐿′ (𝑓 − 𝑔)| < 𝜀, и
так как
𝐿(𝑓 ) − 𝐿′ (𝑓 ) = 𝐿(𝑓 − 𝑔) + 𝐿′ (𝑔 − 𝑓 ),
то мы заключаем, что |𝐿(𝑓 ) − 𝐿′ (𝑓 )| < 2𝜀.
С этим пунктом связано упражнение 2.
10.3 Определение. Носителем (вещественной или комплексной) функции 𝑓 на 𝑅𝑘 называется
замыкание множества всех точек x ∈ 𝑅𝑘 , в которых 𝑓 (x) ≠ 0. Если 𝑓 — непрерывная функция с
компактным носителем, а 𝐼 𝑘 — какая-нибудь 𝑘-клетка, содержащая носитель функции 𝑓 , то положим
(3)
∫
𝑅𝑘
𝑓=
∫
𝐼𝑘
𝑓.
Так определенный интеграл, очевидно, не зависит от выбора 𝐼 𝑘 , лишь бы клетка 𝐼 𝑘 содержала
носитель функции 𝑓 .
Может показаться заманчивым распространить это определение интеграла по 𝑅𝑘 на функции,
являющиеся пределами (в некотором смысле) непрерывных функций с компактным носителем. Мы
не хотим обсуждать условия, при которых это можно сделать; этот вопрос уместно решать с помощью интеграла Лебега. Мы лишь опишем один очень простой пример, который будет использован
при доказательстве теоремы Стокса.
10.4 Пример. Пусть 𝑄𝑘 — 𝑘-симплекс, состоящий из всех точек x = (𝑥1 , … , 𝑥𝑘 ) в 𝑅𝑘 , для которых
𝑥1 + ⋯ + 𝑥𝑘 ≤ 1 и 𝑥𝑖 ≥ 0 при 𝑖 = 1, … , 𝑘. Если 𝑘 = 3, например, то 𝑄𝑘 — тетраэдр с вершинами 0, e1 ,
e2 , e3 . Если 𝑓 ∈ 𝒞 (𝑄𝑘 ), то продолжим 𝑓 до функции на 𝐼 𝑘 , полагая 𝑓 (x) = 0 вне 𝑄𝑘 , и положим
(4)
∫
𝑄𝑘
𝑓=
∫
𝐼𝑘
𝑓.
Здесь 𝐼 𝑘 — «единичный куб», определяемый неравенствами
0 ≤ 𝑥𝑖 ≤ 1
(1 ≤ 𝑖 ≤ 𝑘).
Ввиду того что 𝑓 может быть разрывной на 𝐼 𝑘 , существование интеграла в правой части (4)
требует доказательства. Мы также хотим показать, что этот интеграл не зависит от порядка, в
котором выполняются 𝑘 однократных интегрирований.
Чтобы сделать это, выберем 0 < 𝛿 < 1, положим
(5)
⎧1
⎪
𝜑(𝑡) = ⎨ (1−𝑡)
𝛿
⎪0
⎩
(𝑡 ≤ 1 − 𝛿),
(1 − 𝛿 < 𝑡 ≤ 1),
(1 < 𝑡)
и определим
(6)
𝐹 (x) = 𝜑(𝑥1 + ⋯ + 𝑥𝑘 )𝑓 (x)
Тогда 𝐹 ∈ 𝒞 (𝐼 𝑘 ).
184
(x ∈ 𝐼 𝑘 ).
Положим y = (𝑥1 , … , 𝑥𝑘−1 ), x = (y, 𝑥𝑘 ). При каждом y ∈ 𝐼 𝑘−1 множество всех 𝑥𝑘 , таких, что
𝐹 (y, 𝑥𝑘 ) ≠ 𝑓 (y, 𝑥𝑘 ), либо пусто, либо является интервалом, длина которого не превосходит 𝛿. Так
как 0 ≤ 𝜑 ≤ 1, то
(7)
|𝐹𝑘−1 (y) − 𝑓𝑘−1 (y)| ≤ 𝛿‖𝑓 ‖
(y ∈ 𝐼 𝑘−1 ),
где ‖𝑓 ‖ имеет тот же смысл, что и в доказательстве теоремы 10.2, а 𝐹𝑘−1 и 𝑓𝑘−1 те же, что и в
определении 10.1.
При 𝛿 → 0 (7) показывает, что 𝑓𝑘−1 — равномерный предел последовательности непрерывных
функций. Следовательно, 𝑓𝑘−1 ∈ 𝒞 (𝐼 𝑘−1 ), и дальнейшие интегрирования не представляют проблем.
Существование интеграла (4) доказано. Далее, (7) показывает, что
(8)
|∫
𝐼𝑘
𝐹 (x) 𝑑x −
∫
𝐼𝑘
𝑓 (x) 𝑑x ≤ 𝛿‖𝑓 ‖.
|
Отметим, что (8) справедливо вне зависимости от порядка, в котором производятся 𝑘 однократных
интегрирований. Ввиду того что 𝐹 ∈ 𝒞 (𝐼 𝑘 ), на ∫ 𝐹 не сказывается никакое изменение этого порядка.
Следовательно, (8) показывает, что то же верно и в отношении ∫ 𝑓 .
Утверждение доказано.
Наша следующая цель — формула замены переменных, сформулированная в теореме 10.9. Для
облегчения доказательства мы вначале обсудим так называемые простые отображения и разбиения
единицы. Простые отображения позволят нам получить более ясную картину локального действия
𝒞 ′ -отображения с обратимой производной, а разбиения единицы — очень полезное средство, делающее возможным использование локальных сведений в глобальной ситуации.
Простые отображения
10.5 Определение. Если G отображает открытое множество 𝐸 ⊂ 𝑅𝑛 в 𝑅𝑛 и если существуют
целое число 𝑚 и вещественная функция 𝑔 с областью определения 𝐸, такие, что
(9)
G(x) =
∑
𝑥𝑖 e𝑖 + 𝑔(x)e𝑚
(x ∈ 𝐸),
𝑖≠𝑚
то G называется простым. Простое отображение, тем самым, изменяет самое большее одну координату. Отметим, что (9) также можно записать в виде
(10)
G(x) = x + [𝑔(x) − 𝑥𝑚 ]e𝑚 .
Если 𝑔 дифференцируема в некоторой точке a ∈ 𝐸, то же самое верно и в отношении G. Матрица
[𝛼𝑖𝑗 ] оператора G′ (a) имеет в 𝑚-ой строке элементы
(11)
(𝐷1 𝑔)(a), … , (𝐷𝑚 𝑔)(a), … , (𝐷𝑛 𝑔)(a).
При 𝑗 ≠ 𝑚 имеем 𝛼𝑗𝑗 = 1, а при 𝑖 ≠ 𝑗 имеем 𝛼𝑖𝑗 = 0. Следовательно, якобиан оператора G в a равен
(12)
𝐽G (a) = det[G′ (a)] = (𝐷𝑚 𝑔)(a),
и из теоремы 9.36 следует, что G′ (a) обратимо тогда и только тогда, когда (𝐷𝑚 𝑔)(a) ≠ 0.
10.6 Определение. Линейный оператор 𝐵 на 𝑅𝑘 , меняющий местами некоторую пару элементов
стандартного базиса и оставляющий прочие на месте, называется переворотом.
Например, переворот 𝐵 на 𝑅4 , меняющий местами e2 и e4 , имеет вид
(13)
𝐵(𝑥1 e1 + 𝑥2 e2 + 𝑥3 e3 + 𝑥4 e4 ) = 𝑥1 e1 + 𝑥2 e4 + 𝑥3 e3 + 𝑥4 e2
или, что то же самое,
(14)
𝐵(𝑥1 e1 + 𝑥2 e2 + 𝑥3 e3 + 𝑥4 e4 ) = 𝑥1 e1 + 𝑥4 e2 + 𝑥3 e3 + 𝑥2 e4 .
Значит, 𝐵 можно рассматривать и как перестановку двух координат, а не двух базисных векторов.
185
В следующем доказательстве мы будем пользоваться проекциями 𝑃0 , … , 𝑃𝑛 в 𝑅𝑛 , определяемыми
равенствами 𝑃0 x = 0 и
(15)
𝑃𝑚 x = 𝑥1 e1 + ⋯ 𝑥𝑚 e𝑚
при 1 ≤ 𝑚 ≤ 𝑛. Таким образом, 𝑃𝑚 — проекция, множество значений и нуль-пространство которой
натянуты соответственно на {e1 , … , e𝑚 } и {e𝑚+1 , … , e𝑛 }.
10.7 Теорема. Пусть F — 𝒞 ′ -отображение открытого множества 𝐸 ⊂ 𝑅𝑛 в 𝑅𝑛 , 0 ∈ 𝐸, F(0) = 0,
а F′ (0) обратимо.
Тогда существует такая окрестность точки 0 в 𝑅𝑛 , в которой имеет место представление
(16)
F(x) = 𝐵1 ⋯ 𝐵𝑛−1 G𝑛 ∘ ⋯ ∘ G1 (x).
В (16) каждое G𝑖 — простое 𝒞 ′ -отображение в некоторой окрестности точки 0, G𝑖 (0) = 0,
обратимо, а каждое 𝐵𝑖 — либо переворот, либо тождественный оператор.
G′𝑖 (0)
Вкратце, (16) представляет F локально как композицию простых отображений и переворотов.
Доказательство. Положим F = F1 , и пусть 1 ≤ 𝑚 ≤ 𝑛 − 1. Примем следующее индуктивное
предположение (очевидно, справедливое при 𝑚 = 1):
𝑉𝑚 — окрестность точки 0, F𝑚 ∈ 𝒞 ′ (𝑉𝑚 ), F𝑚 (0) = 0, F′𝑚 (0) обратимо и
(17)
𝑃𝑚−1 F𝑚 (x) = 𝑃𝑚−1 x
(x ∈ 𝑉𝑚 ).
Ввиду (17) имеем
𝑛
(18)
F𝑚 (x) = 𝑃𝑚−1 x +
∑
𝛼𝑖 (x)e𝑖 ,
𝑖=𝑚
где 𝛼𝑚 , … , 𝛼𝑛 — вещественные 𝒞 ′ -функции на 𝑉𝑚 . Значит,
𝑛
(19)
F′𝑚 (0)e𝑚 =
(𝐷 𝛼 )(0)e𝑖 .
∑ 𝑚 𝑖
𝑖=𝑚
Так как F′𝑚 (0) обратимо, то левая часть равенства (19) не равна 0, и, следовательно, существует 𝑘,
такое, что 𝑚 ≤ 𝑘 ≤ 𝑛 и (𝐷𝑚 𝛼𝑘 )(0) ≠ 0.
Обозначим через 𝐵𝑚 переворот, меняющий местами 𝑚 и это 𝑘 (если 𝑘 = 𝑚, то 𝐵𝑚 — тождественный оператор), и положим
(20)
G𝑚 (x) = x + [𝛼𝑘 (x) − 𝑥𝑚 ]e𝑚
(x ∈ 𝑉𝑚 ).
Тогда G𝑚 ∈ 𝒞 ′ (𝑉𝑚 ), G𝑚 простое и G′𝑚 (0) обратимо, так как (𝐷𝑚 𝛼𝑘 )(0) ≠ 0.
Стало быть, из теоремы об обратной функции следует, что существует открытое множество 𝑈𝑚
с 0 ∈ 𝑈𝑚 ⊂ 𝑉𝑚 , такое, что G𝑚 — взаимно однозначное отображение множества 𝑈𝑚 на окрестность
𝑉𝑚+1 точки 0, на которой G−1
𝑚 непрерывно дифференцируемо. Положим F𝑚+1 равным
(21)
F𝑚+1 (y) = 𝐵𝑚 F𝑚 ∘ G−1
𝑚 (y)
(y ∈ 𝑉𝑚+1 ).
Тогда F𝑚+1 ∈ 𝒞 ′ (𝑉𝑚+1 ), F𝑚+1 (0) = 0 и F′𝑚+1 (0) обратимо (по правилу дифференцирования сложной функции). Также при x ∈ 𝑈𝑚
(22)
𝑃𝑚 F𝑚+1 (G𝑚 (x)) = 𝑃𝑚 𝐵𝑚 F𝑚 (x)
= 𝑃𝑚 [𝑃𝑚−1 x + 𝛼𝑘 (x)e𝑚 + ⋯ ]
= 𝑃𝑚−1 x + 𝛼𝑘 (x)e𝑚
= 𝑃𝑚 G𝑚 (x),
так что
(23)
𝑃𝑚 F𝑚+1 (y) = 𝑃𝑚 y
186
(y ∈ 𝑉𝑚+1 ).
Следовательно, предположение индукции выполняется для 𝑚 + 1 на месте 𝑚 (в (22) мы вначале
использовали (21), затем (18) и определение множества 𝐵𝑚 , затем определение множества 𝑃𝑚 и,
наконец, (20)).
Ввиду того что 𝐵𝑚 𝐵𝑚 = 𝐼, (21) при y = G𝑚 (x) эквивалентно
(24)
F𝑚 (x) = 𝐵𝑚 F𝑚+1 (G𝑚 (x))
(x ∈ 𝑈𝑚 ).
Если мы применим это равенство при 𝑚 = 1, … , 𝑛 − 1, то мы последовательно получим
F = F1 = 𝐵1 F2 ∘ G1
= 𝐵1 𝐵2 F3 ∘ G2 ∘ G1 = ⋯
= 𝐵1 ⋯ 𝐵𝑛−1 F𝑛 ∘ G𝑛−1 ∘ ⋯ ∘ G1
в некоторой окрестности точки 0. В силу (17) F𝑛 простое. Доказательство закончено.
Разбиения единицы
10.8 Теорема. Пусть 𝐾 — компактное подмножество пространства 𝑅𝑛 , а {𝑉𝛼 } — открытое
покрытие множества 𝐾. Тогда существуют функции 𝜓1 , … , 𝜓𝑠 ∈ 𝒞 (𝑅𝑛 ), такие, что
(a) 0 ≤ 𝜓𝑖 ≤ 1 при 1 ≤ 𝑖 ≤ 𝑠;
(b) носитель каждой функции 𝜓𝑖 содержится в некотором 𝑉𝛼 и
(c) 𝜓1 (x) + ⋯ + 𝜓𝑠 (x) = 1 при каждом x ∈ 𝐾.
Ввиду (c) семейство {𝜓𝑖 } называют разбиением единицы, а свойство (b) иногда выражают словами «разбиение {𝜓𝑖 } подчинено покрытию {𝑉𝛼 }».
Следствие. Если 𝑓 ∈ 𝒞 (𝑅𝑛 ) и носитель функции 𝑓 содержится в 𝐾, то
𝑠
(25)
𝑓=
∑
𝜓𝑖 𝑓 .
𝑖=1
Носитель каждой функции 𝜓𝑖 𝑓 содержится в некотором 𝑉𝛼 .
Смысл равенства (25) в том, что оно представляет 𝑓 в виде суммы непрерывных функций 𝜓𝑖 𝑓
с «небольшими» носителями.
Доказательство. Поставим в соответствие каждой точке x ∈ 𝐾 индекс 𝛼(x), такой, что x ∈ 𝑉𝛼(x) .
Тогда существуют открытые шары 𝐵(x) и 𝑊 (x) с центром в x, такие, что
(26)
𝐵(x) ⊂ 𝑊 (x) ⊂ 𝑊 (x) ⊂ 𝑉𝛼(x) .
Ввиду того что 𝐾 компактно, в нем существуют точки x1 , … , x𝑠 , такие, что
(27)
𝐾 ⊂ 𝐵(x1 ) ∪ ⋯ ∪ 𝐵(x𝑠 ).
В силу (26) существуют функции 𝜑1 , … , 𝜑𝑠 ∈ 𝒞 (𝑅𝑛 ), такие, что 𝜑1 (x) = 1 на 𝐵(x𝑖 ), 𝜑𝑖 (x) = 0 вне
𝑊 (x𝑖 ) и 0 ≤ 𝜑𝑖 (x) ≤ 1 на 𝑅𝑛 . Положим 𝜓1 = 𝜑1 и
(28)
𝜓𝑖+1 = (1 − 𝜑1 ) ⋯ (1 − 𝜑𝑖 )𝜑𝑖+1
при 𝑖 = 1, … , 𝑠 − 1.
Свойства (a) и (b) очевидны. Соотношение
(29)
𝜓1 + ⋯ + 𝜓𝑖 = 1 − (1 − 𝜑1 ) ⋯ (1 − 𝜑𝑖 )
тривиально при 𝑖 = 1. Если (29) выполняется при некотором 𝑖 < 𝑠, то сложение (28) и (29) дает (29)
с 𝑖 + 1 на месте 𝑖. Следовательно,
𝑠
𝑠
(30)
∑
𝑖=1
𝜓𝑖 (x) = 1 −
[1 − 𝜑𝑖 (x)]
∏
(x ∈ 𝑅𝑛 ).
𝑖=1
Если x ∈ 𝐾, то x ∈ 𝐵(x𝑖 ) при некотором 𝑖, значит, 𝜑𝑖 (x) = 1 и произведение в (30) равно 0. Тем
самым доказано (c).
187
Замена переменных
Теперь мы можем описать, как на кратный интеграл действует замена переменных. Для простоты
мы ограничимся непрерывными функциями с компактным носителем, хотя это излишне ограничивает нас во многих приложениях. Это иллюстрируется упражнениями 9-13.
10.9 Теорема. Пусть 𝑇 — взаимно однозначное 𝒞 ′ -отображение открытого множества 𝐸 ⊂ 𝑅𝑘
в 𝑅𝑘 , такое, что 𝐽𝑇 (x) ≠ 0 при всех x ∈ 𝐸. Если 𝑓 — непрерывная функция на 𝑅𝑘 с компактным
носителем, содержащимся в 𝑇 (𝐸), то
(31)
∫
𝑅𝑘
𝑓 (y) 𝑑y =
∫
𝑅𝑘
𝑓 (𝑇 (x))|𝐽𝑇 (x)| 𝑑x.
Напомним, что 𝐽𝑇 — якобиан отображения 𝑇 . Так как 𝐽𝑇 (x) ≠ 0, то по теореме об обратной
функции 𝑇 −1 непрерывно на 𝑇 (𝐸), и тем самым подынтегральная функция в правой части (31)
имеет компактный носитель, содержащийся в 𝐸 (теорема 4.14).
Поясним появление абсолютной величины якобиана 𝐽𝑇 (x) в (31). Пусть 𝑘 = 1. Предположим,
что 𝑇 — взаимно однозначное 𝒞 ′ -отображение пространства 𝑅1 на 𝑅1 . Тогда 𝐽𝑇 (𝑥) = 𝑇 ′ (𝑥). Если 𝑇
возрастает, то
(32)
∫
𝑅1
𝑓 (𝑦) 𝑑𝑦 =
∫
𝑅1
𝑓 (𝑇 (𝑥))𝑇 ′ (𝑥) 𝑑𝑥
по теоремам 6.19 и 6.17, какова бы ни была непрерывная функция 𝑓 с компактным носителем. Но
если 𝑇 убывает, то 𝑇 ′ (𝑥) < 0, и если 𝑓 положительна на внутренности своего носителя, то левая
часть (32) положительна, а правая отрицательна. Верное равенство получится, если в (32) заменить
𝑇 ′ на |𝑇 ′ |.
Заметим, что интегралы, которые мы сейчас рассматриваем, — это интегралы от функций по
подмножествам пространства 𝑅𝑘 , и с этими подмножествами мы не связываем никакой ориентации
или направления. Мы встанем на другую точку зрения, когда будем заниматься интегрированием
дифференциальных форм по поверхностям.
Доказательство. Из только что сделанных замечаний следует, что (31) справедливо, если 𝑇 —
простое 𝒞 ′ -отображение (см. определение 10.5), а теорема 10.2 показывает, что (31) верно и в том
случае, когда 𝑇 — линейное преобразование, просто меняющее местами две координаты.
Если теорема верна для отображений 𝑃 , 𝑄 и если 𝑆(x) = 𝑃 (𝑄(x)), то
∫
𝑓 (z) 𝑑z =
=
=
∫
𝑓 (𝑃 (y))|𝐽𝑃 (y)| 𝑑y
∫
𝑓 (𝑃 (𝑄(x)))|𝐽𝑃 (𝑄(x))||𝐽𝑄 (x)| 𝑑x
∫
𝑓 (𝑆(x))|𝐽𝑆 (x)| 𝑑x,
так как
𝐽𝑃 (𝑄(x))𝐽𝑄 (x) = det 𝑃 ′ (𝑄(x)) det 𝑄′ (x)
= det 𝑃 ′ (𝑄(x))𝑄′ (x) = det 𝑆 ′ (x) = 𝐽𝑆 (x),
согласно теореме об умножении определителей и правилу дифференцирования сложной функции.
Таким образом, теорема верна и для 𝑆.
Каждая точка a ∈ 𝐸 имеет окрестность 𝑈 ⊂ 𝐸, в которой
(33)
𝑇 (x) = 𝑇 (a) + 𝐵1 ⋯ 𝐵𝑘−1 G𝑘 ∘ G𝑘−1 ∘ ⋯ ∘ G1 (x − a),
где G𝑖 и 𝐵𝑖 те же, что в теореме 10.7. Полагая 𝑉 = 𝑇 (𝑈 ), получаем, что (31) справедливо, если
носитель функции 𝑓 содержится в 𝑉 . Итак:
Каждая точка y ∈ 𝑇 (𝐸) лежит в открытом множестве 𝑉y ⊂ 𝑇 (𝐸), таком, что (31) справедливо
для любой непрерывной функции с носителем, содержащимся в 𝑉y .
188
Теперь пусть 𝑓 — непрерывная функция с компактным носителем 𝐾 ⊂ 𝑇 (𝐸). Ввиду того что
{𝑉y } покрывает 𝐾, следствие из теоремы 10.8 показывает, что 𝑓 = ∑ 𝜓𝑖 𝑓 , где каждая 𝜓𝑖 непрерывна
и носитель каждой 𝜓𝑖 содержится в некотором 𝑉y . Следовательно, (31) выполняется для каждой
функции 𝜓𝑖 𝑓 , значит, и для их суммы 𝑓 .
Дифференциальные формы
В этом разделе мы частично разработаем аппарат, необходимый для 𝑛-мерного варианта основной
теоремы интегрального исчисления, обычно называемого теоремой Стокса. Исходный вид теоремы
Стокса возник в приложениях векторного анализа к электромагнетизму и был сформулирован в
терминах вихря векторного поля. Другими специальными случаями являются теорема Грина и
теорема о дивергенции. Краткому изложению этих вопросов посвящен конец данной главы.
Любопытной особенностью теоремы Стокса является то, что единственная ее трудность состоит
в замысловатой структуре определений, требуемых для ее формулировки. Эти определения касаются дифференциальных форм, их производных, границ и ориентации. Как только эти понятия
объяснены, утверждение теоремы формулируется очень кратко и сжато, а доказательство почти
не представляет проблем.
До сих пор мы рассматривали производные функций нескольких переменных только для функций, определенных на открытых множествах. Это позволило нам избежать трудностей, которые
могут встретиться в граничных точках. Однако теперь нам будет удобно рассматривать дифференцируемые функции на компактных множествах. Поэтому мы примем следующее соглашение.
Говоря, что f есть 𝒞 ′ -отображение (или 𝒞 ″ -отображение) компактного множества 𝐷 ⊂ 𝑅𝑘 в
𝑛
𝑅 , мы будем иметь в виду, что существует 𝒞 ′ -отображение (или 𝒞 ″ -отображение) g некоторого
открытого множества 𝑊 ⊂ 𝑅𝑘 в 𝑅𝑛 , такое, что 𝐷 ⊂ 𝑊 и что g(x) = f(x) при всех x ∈ 𝐷.
10.10 Определение. Пусть 𝐸 — открытое множество в 𝑅𝑛 ; 𝑘-поверхностью в 𝐸 мы будем называть 𝒞 ′ -отображение Φ некоторого компактного множества 𝐷 ⊂ 𝑅𝑘 в 𝐸.
Множество 𝐷 называется множеством параметров поверхности Φ. Точки множества 𝐷 мы
будем обозначать через u = (𝑢1 , … , 𝑢𝑘 ).
Мы будем иметь дело только с простейшей ситуацией, когда 𝐷 — или 𝑘-клетка, или 𝑘-симплекс
𝑄𝑘 , описанный в примере 10.4. Причина этого в том, что нам предстоит интегрировать по 𝐷, а
мы еще не развили теории интегрирования по более сложным подмножествам пространства 𝑅𝑘 .
Мы увидим, что это ограничение, наложенное на 𝐷 (молчаливо предполагаемое в дальнейшем) не
влечет существенной потери общности в теории дифференциальных форм.
Мы подчеркиваем, что 𝑘-поверхности в 𝐸 определяются как отображения в 𝐸, а не как подмножества множества 𝐸. Это согласуется с нашим определением кривых (определение 6.26). В
действительности 1-поверхности — это в точности непрерывно дифференцируемые кривые.
10.11 Определение. Пусть 𝐸 — открытое множество в 𝑅𝑛 . Дифференциальной формой порядка
𝑘 ≥ 1 в 𝐸 (кратко, 𝑘-формой в 𝐸) называется функция 𝜔, символически записываемая в виде суммы
(34)
𝜔=
∑
𝑎𝑖1 …𝑖𝑘 (x) 𝑑𝑥𝑖1 ∧ ⋯ ∧ 𝑑𝑥𝑖𝑘
(индексы 𝑖1 , … , 𝑖𝑘 независимо пробегают от 1 до 𝑛), сопоставляющая каждой 𝑘-поверхности Φ в 𝐸
число 𝜔(Φ) = ∫Φ 𝜔 в соответствии с правилом
(35)
∫
Φ
𝜔=
∑
∫
𝐷
𝑎𝑖1 …𝑖𝑘 (Φ(u))
𝜕(𝑥𝑖1 , … , 𝑥𝑖𝑘 )
𝜕(𝑢1 , … , 𝑢𝑘 )
𝑑u,
где 𝐷 — множество параметров поверхности Φ.
Функции 𝑎𝑖1 …𝑖𝑘 предполагаются вещественными и непрерывными на 𝐸. Если 𝜑1 , … , 𝜑𝑛 — компоненты отображения Φ, то якобиан в (35) — это якобиан отображения
(𝑢1 , … , 𝑢𝑘 ) → (𝜑𝑖1 (u), … , 𝜑𝑖𝑘 (u)).
Заметим, что правая часть (35) — интеграл по 𝐷, описанный в определении 10.1 (или примере
10.4), и что (35) — определение символа ∫Φ 𝜔.
189
Говорят, что 𝑘-форма 𝜔 принадлежит классу 𝒞 ′ или 𝒞 ″ , если все функции 𝑎𝑖1 …𝑖𝑘 в (34) принадлежат классу 𝒞 ′ или 𝒞 ″ .
0-формой в 𝐸 называют функцию, непрерывную на 𝐸.
10.12 Примеры.
(a) Пусть 𝛾 — 1-поверхность (кривая класса 𝒞 ′ ) в 𝑅3 с множеством параметров [0, 1].
Будем записывать (𝑥, 𝑦, 𝑧) вместо (𝑥1 , 𝑥2 , 𝑥3 ) и положим
𝜔 = 𝑥 𝑑𝑦 + 𝑦 𝑑𝑥.
Тогда
1
∫
𝛾
𝜔=
∫
0
[𝛾1 (𝑡)𝛾2′ (𝑡) + 𝛾2 (𝑡)𝛾1′ (𝑡)] 𝑑𝑡 = 𝛾1 (1)𝛾2 (1) − 𝛾1 (0)𝛾2 (0).
Отметим, что в данном примере ∫𝛾 𝜔 зависит только от начальной точки 𝛾(0) и конечной
точки 𝛾(1) кривой 𝛾. В частности, ∫𝛾 𝜔 = 0 для каждой замкнутой кривой 𝛾 (как мы увидим
позднее, это справедливо для каждой 1-формы 𝜔, являющейся точной).
Интегралы от 1-форм часто называют криволинейными интегралами.
(b) Зафиксируем 𝑎 > 0, 𝑏 > 0 и положим
𝛾(𝑡) = (𝑎 cos 𝑡, 𝑏 sin 𝑡)
(0 ≤ 𝑡 ≤ 2𝜋),
так что 𝛾 — замкнутая кривая в 𝑅2 (ее область значений — эллипс). Тогда
2𝜋
∫
𝛾
𝑥 𝑑𝑦 =
∫
0
𝑎𝑏 cos2 𝑡 𝑑𝑡 = 𝜋𝑎𝑏,
в то время как
2𝜋
∫
𝛾
𝑦 𝑑𝑥 = −
∫
0
𝑎𝑏 sin2 𝑡 𝑑𝑡 = −𝜋𝑎𝑏.
Отметим, что ∫𝛾 𝑥 𝑑𝑦 — площадь области, ограниченной кривой 𝛾. Это частный случай теоремы Грина.
(c) Пусть 𝐷 — 3-клетка, определяемая неравенствами
0 ≤ 𝑟 ≤ 1,
0 ≤ 𝜃 ≤ 𝜋,
0 ≤ 𝜑 ≤ 2𝜋.
Положим Φ(𝑟, 𝜃, 𝜑) = (𝑥, 𝑦, 𝑧), где
𝑥 = 𝑟 sin 𝜃 cos 𝜑,
𝑦 = 𝑟 sin 𝜃 sin 𝜑,
𝑧 = 𝑟 cos 𝜃.
Тогда
𝐽Φ (𝑟, 𝜃, 𝜑) =
𝜕(𝑥, 𝑦, 𝑧)
= 𝑟2 sin 𝜃.
𝜕(𝑟, 𝜃, 𝜑)
Значит,
(36)
∫
Φ
𝑑𝑥 ∧ 𝑑𝑦 ∧ 𝑑𝑧 =
∫
𝐷
𝐽Φ =
4𝜋
.
3
Отметим, что Φ отображает 𝐷 на замкнутый единичный шар пространства 𝑅3 , отображение
взаимно однозначно во внутренности множества 𝐷 (но некоторые точки границы отождествляются отображением Φ), а интеграл (36) равен объему шара Φ(𝐷).
10.13 Элементарные свойства. Пусть 𝜔, 𝜔1 , 𝜔2 — 𝑘-формы в 𝐸. Мы будем писать 𝜔1 = 𝜔2 тогда
и только тогда, когда 𝜔1 (Φ) = 𝜔2 (Φ) для всех 𝑘-поверхностей Φ в 𝐸. В частности, 𝜔 = 0 означает,
190
что 𝜔(Φ) = 0 для всех 𝑘-поверхностей Φ в 𝐸. Если 𝑐 — вещественное число, то 𝑐𝜔 — 𝑘-форма,
определенная как
(37)
∫
Φ
𝑐𝜔 = 𝑐
∫
Φ
𝜔,
а 𝜔 = 𝜔1 + 𝜔2 означает, что
(38)
∫
Φ
𝜔=
∫
Φ
𝜔1 +
∫
Φ
𝜔2
для любой 𝑘-поверхности Φ в 𝐸. Как частный случай (37) отметим, что −𝜔 определяется так, что
(39)
∫
Φ
(−𝜔) = −
∫
Φ
𝜔.
Рассмотрим 𝑘-форму
(40)
𝜔 = 𝑎(x) 𝑑𝑥𝑖1 ∧ ⋯ ∧ 𝑑𝑥𝑖𝑘 .
Пусть 𝜔̄ — 𝑘-форма, полученная перестановкой некоторой пары индексов в (40). Согласно (35) и
(39), из того, что определитель меняет знак при перестановке двух его строк, следует, что
(41)
𝜔̄ = −𝜔.
Как частный случай отметим, что антикоммутативный закон
(42)
𝑑𝑥𝑖 ∧ 𝑑𝑥𝑗 = −𝑑𝑥𝑗 ∧ 𝑑𝑥𝑖
выполняется при всех 𝑖 и 𝑗. В частности,
(43)
𝑑𝑥𝑖 ∧ 𝑑𝑥𝑗 = 0
(𝑖 = 1, … , 𝑛).
Более общо, вернемся к (40) и положим 𝑖𝑟 = 𝑖𝑠 при некоторых 𝑟 ≠ 𝑠. Если переставить эти два
индекса, то 𝜔̄ = 𝜔, значит, 𝜔 = 0 в силу (41).
Другими словами, если 𝜔 задана равенством (40), то 𝜔 = 0, кроме случаев, когда все индексы
𝑖1 , … , 𝑖𝑘 различны.
Следовательно, если 𝜔 задана равенством (34), то слагаемые с повторяющимися индексами
можно опустить, не изменяя 𝜔.
Это означает, что 0 — единственная 𝑘-форма в любом открытом множестве пространства 𝑅𝑛 ,
если 𝑘 > 𝑛.
Антикоммутативность, выраженная равенством (42), заставляет проявлять пристальное внимание к знакам минуса при изучении дифференциальных форм.
10.14 Базисные 𝑘-формы. Если 𝑖1 , … , 𝑖𝑘 — целые числа, такие, что 1 ≤ 𝑖1 < 𝑖2 < ⋯ < 𝑖𝑘 ≤ 𝑛, а
𝐼 — упорядоченный набор 𝑘 чисел {𝑖1 , … , 𝑖𝑘 }, то мы будем называть 𝐼 возрастающим 𝑘-индексом
и использовать краткое обозначение
(44)
𝑑𝑥𝐼 = 𝑑𝑥𝑖1 ∧ ⋯ ∧ 𝑑𝑥𝑖𝑘 .
Эти формы 𝑑𝑥𝐼 называются базисными 𝑘-формами в 𝑅𝑛 .
Нетрудно проверить, что существует ровно 𝑛!/𝑘!(𝑛 − 𝑘)! базисных 𝑘-форм в 𝑅𝑛 , однако мы не
будем этим пользоваться.
Гораздо важнее тот факт, что каждая 𝑘-форма может быть выражена через базисные 𝑘-формы.
Чтобы увидеть это, отметим, что каждый упорядоченный набор 𝑘 различных целых чисел {𝑗1 , … , 𝑗𝑘 }
может быть превращен в возрастающий 𝑘-индекс 𝐽 конечным числом перестановок пар; каждая
из этих перестановок приводит к умножению на −1, как мы видели в п. 10.13; значит,
(45)
𝑑𝑥𝑗1 ∧ ⋯ ∧ 𝑑𝑥𝑗𝑘 = 𝜀(𝑗1 , … , 𝑗𝑘 ) 𝑑𝑥𝐽 ,
191
где 𝜀(𝑗1 , … , 𝑗𝑘 ) равно 1 или −1 в зависимости от количества требуемых перестановок. В действительности легко видеть, что
(46)
𝜀(𝑗1 , … , 𝑗𝑘 ) = 𝑠(𝑗1 , … , 𝑗𝑘 ),
где 𝑠 то же, что и в определении 9.33.
Например,
𝑑𝑥1 ∧ 𝑑𝑥5 ∧ 𝑑𝑥3 ∧ 𝑑𝑥2 = −𝑑𝑥1 ∧ 𝑑𝑥2 ∧ 𝑑𝑥3 ∧ 𝑑𝑥5
и
𝑑𝑥4 ∧ 𝑑𝑥2 ∧ 𝑑𝑥3 = 𝑑𝑥2 ∧ 𝑑𝑥3 ∧ 𝑑𝑥4 .
Если каждый набор 𝑘 чисел в (34) преобразован в возрастающий 𝑘-индекс, то мы получаем так
называемый стандартный вид формы 𝜔:
(47)
𝜔=
𝑏𝐼 (x) 𝑑𝑥𝐼 .
∑
𝐼
Суммирование в (47) производится по всем возрастающим 𝑘-индексам 𝐼 (конечно, каждый возрастающий 𝑘-индекс получается из многих (если быть точным, то из 𝑘!) наборов, поэтому каждое 𝑏𝐼
в (47) может быть суммой нескольких коэффициентов, входящих в (34)).
Например,
𝑥1 𝑑𝑥2 ∧ 𝑑𝑥1 − 𝑥2 𝑑𝑥3 ∧ 𝑑𝑥2 + 𝑥3 𝑑𝑥2 ∧ 𝑑𝑥3 + 𝑑𝑥1 ∧ 𝑑𝑥2
является 2-формой в 𝑅3 , стандартный вид которой равен
(1 − 𝑥1 ) 𝑑𝑥1 ∧ 𝑑𝑥2 + (𝑥2 + 𝑥3 ) 𝑑𝑥2 ∧ 𝑑𝑥3 .
Следующая теорема единственности является одной из главных причин для введения понятия
стандартного вида 𝑘-формы.
10.15 Теорема. Пусть
(48)
𝜔=
∑
𝑏𝐼 (x) 𝑑𝑥𝐼
𝐼
является стандартным видом 𝑘-формы 𝜔 в открытом множестве 𝐸 ⊂ 𝑅𝑛 . Если 𝜔 = 0 в 𝐸, то
𝑏𝐼 (x) = 0 для любого возрастающего 𝑘-индекса 𝐼 и любого x ∈ 𝐸.
Отметим, что аналогичное утверждение не выполняется для сумм вида (34), так как, например,
𝑑𝑥1 ∧ 𝑑𝑥2 + 𝑑𝑥2 ∧ 𝑑𝑥1 = 0.
Доказательство. Чтобы прийти к противоречию, допустим, что 𝑏𝐽 (v) > 0 для некоторого v ∈ 𝐸
и некоторого возрастающего 𝑘-индекса 𝐽 = {𝑗1 , … , 𝑗𝑘 }. Так как 𝑏𝐽 непрерывна, то существует ℎ > 0,
такое, что 𝑏𝐽 (x) > 0 для всех x ∈ 𝑅𝑛 , координаты которых удовлетворяют условию |𝑥𝑖 − 𝑣𝑖 | ≤ ℎ.
Пусть 𝐷 — 𝑘-клетка в 𝑅𝑘 , такая, что u ∈ 𝐷 тогда и только тогда, когда |𝑢𝑟 | ≤ ℎ при 𝑟 = 1, … , 𝑘.
Положим
𝑘
(49)
Φ(u) = v +
∑
𝑟=1
𝑢𝑟 e𝑗𝑟
(u ∈ 𝐷).
Тогда Φ — 𝑘-поверхность в 𝐸 с множеством параметров 𝐷, и 𝑏𝐽 (Φ(u)) > 0 при всех u ∈ 𝐷.
Мы утверждаем, что
(50)
∫
Φ
𝜔=
∫
𝐷
𝑏𝐽 (Φ(u)) 𝑑u.
Из того, что правая часть (50) положительна, следует, что 𝜔(Φ) ≠ 0. Значит, (50) приводит к
противоречию.
Чтобы доказать (50), применим (35) к выражению (48). Конкретнее, вычислим якобиан, присутствующий в (35). В силу (49),
𝜕(𝑥𝑗1 , … , 𝑥𝑗𝑘 )
= 1.
𝜕(𝑢1 , … , 𝑢𝑘 )
Для любого возрастающего 𝑘-индекса 𝐼 ≠ 𝐽 якобиан равен 0, так как он является определителем
матрицы со строкой нулей.
192
10.16 Произведение базисных 𝑘-форм. Пусть
(51)
𝐼 = {𝑖1 , … , 𝑖𝑝 },
𝐽 = {𝑗1 , … , 𝑗𝑞 },
где 1 ≤ 𝑖1 < ⋯ < 𝑖𝑝 ≤ 𝑛 и 1 ≤ 𝑗1 < ⋯ < 𝑗𝑞 ≤ 𝑛. Произведение соответствующих базисных форм 𝑑𝑥𝐼 и
𝑑𝑥𝐽 в 𝑅𝑛 — (𝑝 + 𝑞)-форма в 𝑅𝑛 , обозначаемая символом 𝑑𝑥𝐼 ∧ 𝑑𝑥𝐽 и определяемая как
(52)
𝑑𝑥𝐼 ∧ 𝑑𝑥𝐽 = 𝑑𝑥𝑖1 ∧ ⋯ ∧ 𝑑𝑥𝑖𝑝 ∧ 𝑑𝑥𝑗1 ∧ ⋯ ∧ 𝑑𝑥𝑗𝑞 .
Если 𝐼 и 𝐽 имеют общий элемент, то обсуждение в п. 10.13 показывает, что 𝑑𝑥𝐼 ∧ 𝑑𝑥𝐽 = 0.
Если 𝐼 и 𝐽 не имеют общих элементов, то будем записывать в виде [𝐼, 𝐽 ] возрастающий (𝑝 + 𝑞)индекс, получаемый расположением элементов множества 𝐼 ∪ 𝐽 в возрастающем порядке. Тогда
𝑑𝑥[𝐼,𝐽 ] — базисная (𝑝 + 𝑞)-форма. Мы утверждаем, что
𝑑𝑥𝐼 ∧ 𝑑𝑥𝐽 = (−1)𝛼 𝑑𝑥[𝐼,𝐽 ] ,
(53)
где 𝛼 — количество отрицательных разностей 𝑗𝑡 − 𝑖𝑠 (тем самым количество положительных разностей равно 𝑝𝑞 − 𝛼).
Чтобы доказать (53), произведем следующую операцию над числами
(54)
𝑖 1 , … , 𝑖𝑝 ; 𝑗 1 , … , 𝑗 𝑞 .
Будем шаг за шагом перемещать 𝑖𝑝 вправо, пока его правый сосед не станет больше 𝑖𝑝 . Число шагов
равно числу индексов 𝑡, таких, что 𝑖𝑝 < 𝑗𝑡 (отметим, что возможен случай 0 шагов). Повторим то
же самое для 𝑖𝑝−1 , … , 𝑖1 . Общее число произведенных шагов равно 𝛼. В результате получается
расположение [𝐼, 𝐽 ]. Применение каждого шага к правой части (52) умножает 𝑑𝑥𝐼 ∧ 𝑑𝑥𝐽 на −1.
Значит, (53) выполняется.
Отметим, что правая часть (53) — стандартный вид формы 𝑑𝑥𝐼 ∧ 𝑑𝑥𝐽 .
Далее, пусть 𝐾 = (𝑘1 , … , 𝑘𝑟 ) — возрастающий 𝑟-индекс в {1, … , 𝑛}. Мы используем (53) для
доказательства того, что
(55)
(𝑑𝑥𝐼 ∧ 𝑑𝑥𝐽 ) ∧ 𝑑𝑥𝐾 = 𝑑𝑥𝐼 ∧ (𝑑𝑥𝐽 ∧ 𝑑𝑥𝐾 ).
Если любые два из множеств 𝐼, 𝐽 , 𝐾 имеют общий элемент, то обе части (55) равны 0, значит,
они равны.
Поэтому предположим, что 𝐼, 𝐽 , 𝐾 попарно не пересекаются. Обозначим через [𝐼, 𝐽 , 𝐾] возрастающий (𝑝 + 𝑞 + 𝑟)-индекс, полученный из их объединения. Свяжем 𝛽 с упорядоченной парой (𝐽 , 𝐾)
и 𝛾 с упорядоченной парой (𝐼, 𝐾) тем же способом, что и 𝛼 связан с (𝐼, 𝐽 ) в (53). Левая часть (55)
тогда равна
(−1)𝛼 𝑑𝑥[𝐼,𝐽 ] ∧ 𝑑𝑥𝐾 = (−1)𝛼 (−1)𝛽+𝛾 𝑑𝑥[𝐼,𝐽 ,𝐾]
после двукратного применения (53), а правая часть (55) равна
(−1)𝛽 𝑑𝑥𝐼 ∧ 𝑑𝑥[𝐽 ,𝐾] = (−1)𝛽 (−1)𝛼+𝛾 𝑑𝑥[𝐼,𝐽 ,𝐾] .
Значит, (55) выполняется.
10.17 Умножение. Пусть 𝜔 и 𝜆 — соответственно 𝑝- и 𝑞-формы в некотором открытом множестве
𝐸 ∈ 𝑅𝑛 со стандартными видами
(56)
𝜔=
∑
𝑏𝐼 (x) 𝑑𝑥𝐼 ,
𝜆=
𝐼
∑
𝑐𝐽 (x) 𝑑𝑥𝐽 ,
𝐽
где 𝐼 и 𝐽 пробегают соответственно все возрастающие 𝑝-индексы и все возрастающие 𝑞-индексы,
выбранные из множества {1, … , 𝑛}.
Их произведение, обозначаемое символом 𝜔 ∧ 𝜆, определяется как
(57)
𝜔∧𝜆=
∑
𝑏𝐼 (x)𝑐𝐽 (x) 𝑑𝑥𝐼 ∧ 𝑑𝑥𝐽 .
𝐼,𝐽
В этой сумме 𝐼 и 𝐽 независимо пробегают свои допустимые значения, а 𝑑𝑥𝐼 ∧ 𝑑𝑥𝐽 имеет смысл, как
в п. 10.16. Таким образом, 𝜔 ∧ 𝜆 — (𝑝 + 𝑞)-форма в 𝐸.
193
Легко видеть (мы оставляем детали в качестве упражнения), что законы дистрибутивности
(𝜔1 + 𝜔2 ) ∧ 𝜆 = (𝜔1 ∧ 𝜆) + (𝜔2 ∧ 𝜆)
и
𝜔 ∧ (𝜆1 + 𝜆2 ) = (𝜔 ∧ 𝜆1 ) + (𝜔 ∧ 𝜆2 )
выполняются по отношению к сложению, определенному в п. 10.13. Если соединить эти законы
дистрибутивности с (55), мы получим закон ассоциативности
(58)
(𝜔 ∧ 𝜆) ∧ 𝜎 = 𝜔 ∧ (𝜆 ∧ 𝜎)
для произвольных форм 𝜔, 𝜆, 𝜎 в 𝐸.
В этом обсуждении молчаливо подразумевалось, что 𝑝 ≥ 1 и 𝑞 ≥ 1. Произведение 0-формы 𝑓 и
𝑝-формы 𝜔, заданной (56), определяется просто как 𝑝-форма
𝑓 𝜔 = 𝜔𝑓 =
∑
𝑓 (x)𝑏𝐼 (x) 𝑑𝑥𝐼 .
𝐼
Принято писать 𝑓 𝜔 вместо 𝑓 ∧ 𝜔, если 𝑓 — 0-форма.
10.18 Дифференцирование. Мы определим теперь оператор дифференцирования 𝑑, который
ставит в соответствие (𝑘 + 1)-форму 𝑑𝜔 каждой 𝑘-форме 𝜔 класса 𝒞 ′ в некотором открытом множестве 𝐸 ⊂ 𝑅𝑛 .
0-форма класса 𝒞 ′ в 𝐸 — это просто вещественная функция 𝑓 ∈ 𝒞 ′ (𝐸), и мы полагаем по
определению
𝑛
(59)
𝑑𝑓 =
∑
(𝐷𝑖 𝑓 )(x) 𝑑𝑥𝑖 .
𝑖=1
Если 𝜔 = ∑ 𝑏𝐼 (x) 𝑑𝑥𝐼 — стандартный вид 𝑘-формы 𝜔 и 𝑏𝐼 ∈ 𝒞 ′ (𝐸) для каждого возрастающего
𝑘-индекса 𝐼, то мы полагаем по определению
(60)
𝑑𝜔 =
(𝑑𝑏 ) ∧ 𝑑𝑥𝐼 .
∑ 𝐼
𝐼
10.19 Пример. Пусть 𝐸 открыто в 𝑅𝑛 , 𝑓 ∈ 𝒞 ′ (𝐸), а 𝛾 — непрерывно дифференцируемая кривая
в 𝐸 с областью определения [0, 1]. В силу (59) и (35),
1 𝑛
(61)
∫
𝛾
𝑑𝑓 =
∑
∫
0
(𝐷𝑖 𝑓 )(𝛾(𝑡))𝛾𝑖′ (𝑡) 𝑑𝑡.
𝑖=1
По правилу дифференцирования сложной функции последнее подынтегральное выражение равно
(𝑓 ∘ 𝛾)′ (𝑡). Значит,
(62)
∫
𝛾
𝑑𝑓 = 𝑓 (𝛾(1)) − 𝑓 (𝛾(0)),
и мы видим, что ∫𝛾 𝑑𝑓 имеет одно и то же значение для всех кривых 𝛾 с одинаковой начальной и
одинаковой конечной точками, как в пункте (a) примера 10.12.
Сравнение с примером 10.12(b) теперь показывает, что 1-форма 𝑥 𝑑𝑦 не является производной
никакой 0-формы 𝑓 . Это также можно вывести из части (b) следующей теоремы, так как
𝑑(𝑥 𝑑𝑦) = 𝑑𝑥 ∧ 𝑑𝑦 ≠ 0.
10.20 Теорема.
(a) Если 𝜔 и 𝜆 — соответственно 𝑘- и 𝑚-формы класса 𝒞 ′ в 𝐸, то
(63)
𝑑(𝜔 ∧ 𝜆) = (𝑑𝜔) ∧ 𝜆 + (−1)𝑘 𝜔 ∧ 𝑑𝜆.
194
(b) Если 𝜔 — класса 𝒞 ″ в 𝐸, то 𝑑 2 𝜔 = 0.
Здесь 𝑑 2 𝜔 обозначает, разумеется, 𝑑(𝑑𝜔).
Доказательство. Ввиду (57) и (60), для доказательства (a) достаточно доказать (63) в частном
случае
(64)
𝜔 = 𝑓 𝑑𝑥𝐼 ,
𝜆 = 𝑔 𝑑𝑥𝐽 ,
где 𝑓 , 𝑔 ∈ 𝒞 ′ (𝐸), 𝑑𝑥𝐼 — базисная 𝑘-форма, а 𝑑𝑥𝐽 — базисная 𝑚-форма (если 𝑘, 𝑚 или оба равны 0,
то просто опустим 𝑑𝑥𝐼 или 𝑑𝑥𝐽 в (64); следующее доказательство останется неизменным). Тогда
𝜔 ∧ 𝜆 = 𝑓 𝑔 𝑑𝑥𝐼 ∧ 𝑑𝑥𝐽 .
Будем считать, что 𝐼 и 𝐽 не имеют общих элементов (в противном случае обе части (63) равны 0).
Тогда, согласно (53),
𝑑(𝜔 ∧ 𝜆) = 𝑑(𝑓 𝑔 𝑑𝑥𝐼 ∧ 𝑑𝑥𝐽 ) = (−1)𝛼 𝑑(𝑓 𝑔 𝑑𝑥[𝐼,𝐽 ] ).
Согласно (59), 𝑑(𝑓 𝑔) = 𝑓 𝑑𝑔 + 𝑔 𝑑𝑓 . Значит, (60) дает
𝑑(𝜔 ∧ 𝜆) = (−1)𝛼 (𝑓 𝑑𝑔 + 𝑔 𝑑𝑓 ) ∧ 𝑑𝑥[𝐼,𝐽 ]
= (𝑔 𝑑𝑓 + 𝑓 𝑑𝑔) ∧ 𝑑𝑥𝐼 ∧ 𝑑𝑥𝐽 .
Ввиду того что 𝑑𝑔 — 1-форма, а 𝑑𝑥𝐼 — 𝑘-форма, мы имеем
𝑑𝑔 ∧ 𝑑𝑥𝐼 = (−1)𝑘 𝑑𝑥𝐼 ∧ 𝑑𝑔
в силу (42). Значит,
𝑑(𝜔 ∧ 𝜆) = (𝑑𝑓 ∧ 𝑑𝑥𝐼 ) ∧ (𝑔 𝑑𝑥𝐽 ) + (−1)𝑘 (𝑓 𝑑𝑥𝐼 ) ∧ (𝑑𝑔 ∧ 𝑑𝑥𝐽 )
= (𝑑𝜔) ∧ 𝜆 + (−1)𝑘 𝜔 ∧ 𝑑𝜆,
из чего следует (a).
Отметим, что мы без упоминания использовали закон ассоциативности (58).
Докажем (b) вначале для 0-формы 𝑓 ∈ 𝒞 ″ :
⎛ 𝑛
⎞
𝑑 2 𝑓 = 𝑑 ⎜ (𝐷𝑗 𝑓 )(x) 𝑑𝑥𝑗 ⎟
⎜∑
⎟
⎝ 𝑗=1
⎠
𝑛
=
∑
𝑑(𝐷𝑗 𝑓 ) ∧ 𝑑𝑥𝑗
𝑗=1
𝑛
=
∑
(𝐷𝑖𝑗 𝑓 )(x) 𝑑𝑥𝑖 ∧ 𝑑𝑥𝑗 .
𝑖,𝑗=1
Ввиду того что 𝐷𝑖𝑗 𝑓 = 𝐷𝑗𝑖 𝑓 (теорема 9.41) и 𝑑𝑥𝑖 ∧ 𝑑𝑥𝑗 = −𝑑𝑥𝑗 ∧ 𝑑𝑥𝑖 , мы имеем 𝑑 2 𝑓 = 0.
Если 𝜔 = 𝑓 𝑑𝑥𝐼 , как в (64), то 𝑑𝜔 = (𝑑𝑓 ) ∧ 𝑑𝑥𝐼 . В силу (60) имеем 𝑑(𝑑𝑥𝐼 ) = 0. Значит, из (63)
следует, что
𝑑 2 𝜔 = (𝑑 2 𝑓 ) ∧ 𝑑𝑥𝐼 = 0.
10.21 Замена переменных. Пусть 𝐸 — открытое множество в 𝑅𝑛 , 𝑇 — 𝒞 ′ -отображение множества 𝐸 в открытое множество 𝑉 ⊂ 𝑅𝑚 , а 𝜔 — 𝑘-форма в 𝑉 , стандартное представление которой
имеет вид
(65)
𝜔=
∑
𝑏𝐼 (y) 𝑑𝑦𝐼
𝐼
(мы используем обозначение y для точек множества 𝑉 и x для точек множества 𝐸).
195
Пусть 𝑡1 , … , 𝑡𝑚 — компоненты отображения 𝑇 ; другими словами, если
y = (𝑦1 , … , 𝑦𝑚 ) = 𝑇 (x),
то 𝑦𝑖 = 𝑡𝑖 (x). В соответствии с (59),
𝑛
(66)
𝑑𝑡𝑖 =
∑
(𝐷𝑗 𝑡𝑖 )(x) 𝑑𝑥𝑗
(1 ≤ 𝑖 ≤ 𝑚).
𝑗=1
Таким образом, каждая 𝑑𝑡𝑖 — 1-форма в 𝐸.
Отображение 𝑇 преобразует 𝜔 в 𝑘-форму 𝜔𝑇 в 𝐸, определяемую равенством
(67)
𝜔𝑇 =
∑
𝐼
𝑏𝐼 (𝑇 (x)) 𝑑𝑡𝑖1 ∧ ⋯ ∧ 𝑑𝑡𝑖𝑘 .
В каждом слагаемом (67) 𝐼 = {𝑖1 , … , 𝑖𝑘 } — возрастающий 𝑘-индекс.
Наша следующая теорема показывает, что сложение, умножение и дифференцирование форм
определены так, что они перестановочны с заменой переменных.
10.22 Теорема. Пусть 𝐸 и 𝑇 — те же, что и в определении 10.21, а 𝜔 и 𝜆 — соответственно
𝑘- и 𝑚-формы в 𝑉 . Тогда
(a) (𝜔 + 𝜆)𝑇 = 𝜔𝑇 + 𝜆𝑇 , если 𝑘 = 𝑚;
(b) (𝜔 ∧ 𝜆)𝑇 = 𝜔𝑇 ∧ 𝜆𝑇 ;
(c) 𝑑(𝜔𝑇 ) = (𝑑𝜔)𝑇 , если 𝜔 — класса 𝒞 ′ , а 𝑇 — класса 𝒞 ″ .
Доказательство. Часть (a) вытекает непосредствено из определений. Часть (b) почти так же
очевидна, как только мы заметим, что
(68)
(𝑑𝑦𝑖1 ∧ ⋯ ∧ 𝑑𝑦𝑖𝑟 )𝑇 = 𝑑𝑡𝑖1 ∧ ⋯ ∧ 𝑑𝑡𝑖𝑟
независимо от того, является последовательность {𝑖1 , … , 𝑖𝑟 } возрастающей или нет; (68) выполняется, так как обе части этого равенства требуют одинакового количества перемен знака для
получения возрастающих индексов.
Мы переходим к доказательству (c). Если 𝑓 — 0-форма класса 𝒞 ′ в 𝑉 , то
𝑓𝑇 (x) = 𝑓 (𝑇 (x)),
𝑑𝑓 =
∑
(𝐷𝑖 𝑓 )(y) 𝑑𝑦𝑖 .
𝑖
Из правила дифференцирования сложной функции следует, что
(69)
𝑑(𝑓𝑇 ) =
(𝐷 𝑓 )(x) 𝑑𝑥𝑗
∑ 𝑗 𝑇
𝑗
=
∑∑
𝑗
=
(𝐷𝑖 𝑓 )(𝑇 (x))(𝐷𝑗 𝑡𝑖 )(x) 𝑑𝑥𝑗
𝑖
(𝐷 𝑓 )(𝑇 (x)) 𝑑𝑡𝑖
∑ 𝑖
𝑖
= (𝑑𝑓 )𝑇 .
Если 𝑑𝑦𝐼 = 𝑑𝑦𝑖1 ∧ ⋯ ∧ 𝑑𝑦𝑖𝑘 , то (𝑑𝑦𝐼 )𝑇 = 𝑑𝑡𝑖1 ∧ ⋯ ∧ 𝑑𝑡𝑖𝑘 , и теорема 10.20 показывает, что
(70)
𝑑((𝑑𝑦𝐼 )𝑇 ) = 0
(именно здесь используется предположение о том, что 𝑇 ∈ 𝒞 ″ ).
Пусть теперь 𝜔 = 𝑓 𝑑𝑦𝐼 . Тогда
𝜔𝑇 = 𝑓𝑇 (x)(𝑑𝑦𝐼 )𝑇
и предшествующие вычисления приводят к
𝑑(𝜔𝑇 ) = 𝑑(𝑓𝑇 ) ∧ (𝑑𝑦𝐼 )𝑇 = (𝑑𝑓 )𝑇 ∧ (𝑑𝑦𝐼 )𝑇
= ((𝑑𝑓 ) ∧ 𝑑𝑦𝐼 )𝑇 = (𝑑𝜔)𝑇 .
196
Первое равенство выполняется в силу (63) и (70), второе — в силу (69), третье — в силу (b), а
последнее — по определению 𝑑𝜔.
Общий случай (c) следует из только что доказанного частного случая, если применить (a).
Доказательство закончено.
Наша следующая цель — теорема 10.25. Вначале мы докажем два других важных свойства
преобразований дифференциальных форм, из которых она непосредственно следует.
10.23 Теорема. Пусть 𝑇 — 𝒞 ′ -отображение открытого множества 𝐸 ⊂ 𝑅𝑛 в открытое множество 𝑉 ⊂ 𝑅𝑚 , 𝑆 — 𝒞 ′ -отображение множества 𝑉 в открытое множество 𝑊 ⊂ 𝑅𝑝 , а 𝜔 —
𝑘-форма в 𝑊 , причем 𝜔𝑆 — 𝑘-форма в 𝑉 и обе формы (𝜔𝑆 )𝑇 и 𝜔𝑆𝑇 — 𝑘-формы в 𝐸, где 𝑆𝑇 определяется равенством (𝑆𝑇 )(x) = 𝑆(𝑇 (x)). Тогда
(71)
(𝜔𝑆 )𝑇 = 𝜔𝑆𝑇 .
Доказательство. Если 𝜔 и 𝜆 — формы в 𝑊 , то, как показывает теорема 10.22,
((𝜔 ∧ 𝜆)𝑆 )𝑇 = (𝜔𝑆 ∧ 𝜆𝑆 )𝑇 = (𝜔𝑆 )𝑇 ∧ (𝜆𝑆 )𝑇
и
(𝜔 ∧ 𝜆)𝑆𝑇 = 𝜔𝑆𝑇 ∧ 𝜆𝑆𝑇 .
Таким образом, если (71) выполняется для 𝜔 и для 𝜆, то (71) выполняется и для 𝜔 ∧ 𝜆. Поскольку
каждую форму можно построить из 0-форм и 1-форм с помощью операций сложения и умножения
и поскольку (71) выполняется тривиальным образом для 0-форм, то достаточно доказать (71) в
том случае, когда 𝜔 = 𝑑𝑧𝑞 , 𝑞 = 1, … , 𝑝 (мы обозначаем точки множеств 𝐸, 𝑉 , 𝑊 соответственно
через x, y, z).
Пусть 𝑡1 , … , 𝑡𝑚 — компоненты отображения 𝑇 , пусть 𝑠1 , … , 𝑠𝑝 — компоненты отображения 𝑆, и
пусть 𝑟1 , … , 𝑟𝑝 — компоненты отображения 𝑆𝑇 . Если 𝜔 = 𝑑𝑧𝑞 , то
𝜔𝑆 = 𝑑𝑠𝑞 =
∑
(𝐷𝑗 𝑠𝑞 )(y) 𝑑𝑦𝑗 ,
𝑗
так что по правилу дифференцирования сложной функции имеем
(𝜔𝑆 )𝑇 =
∑
(𝐷𝑗 𝑠𝑞 )(𝑇 (x)) 𝑑𝑡𝑗
∑
(𝐷𝑗 𝑠𝑞 )(𝑇 (x))
∑
(𝐷𝑖 𝑟𝑞 )(x) 𝑑𝑥𝑖 = 𝑑𝑟𝑞 = 𝜔𝑆𝑇 .
𝑗
=
𝑗
=
∑
(𝐷𝑖 𝑡𝑗 )(x) 𝑑𝑥𝑖
𝑖
𝑖
10.24 Теорема. Пусть 𝜔 — 𝑘-форма в открытом множестве 𝐸 ⊂ 𝑅𝑛 , Φ — 𝑘-поверхность в 𝐸
с множеством параметров 𝐷 ⊂ 𝑅𝑘 , а Δ — 𝑘-поверхность в 𝑅𝑘 с множеством параметров 𝐷,
заданная равенством Δ(u) = u (u ∈ 𝐷). Тогда
∫
Φ
𝜔=
∫
Δ
𝜔Φ .
Доказательство. Нам достаточно рассмотреть только случай
𝜔 = 𝑎(x) 𝑑𝑥𝑖1 ∧ ⋯ ∧ 𝑑𝑥𝑖𝑘 .
Если 𝜑1 , … , 𝜑𝑛 — компоненты отображения Φ, то
𝜔Φ = 𝑎(Φ(u)) 𝑑𝜑𝑖1 ∧ ⋯ ∧ 𝑑𝜑𝑖𝑘 .
Теорема будет доказана, если мы установим, что
(72)
𝑑𝜑𝑖1 ∧ ⋯ ∧ 𝑑𝜑𝑖𝑘 = 𝐽 (u) 𝑑𝑢1 ∧ ⋯ ∧ 𝑑𝑢𝑘 ,
197
где
𝐽 (u) =
𝜕(𝑥𝑖1 , … , 𝑥𝑖𝑘 )
𝜕(𝑢1 , … , 𝑢𝑘 )
,
поскольку из (72) следует, что
∫
Φ
𝜔=
=
∫
𝐷
∫
Δ
𝑎(Φ(u))𝐽 (u) 𝑑u
𝑎(Φ(u))𝐽 (u) 𝑑𝑢1 ∧ ⋯ ∧ 𝑑𝑢𝑘 =
∫
Δ
𝜔Φ .
Пусть [𝐴] — матрица 𝑘 на 𝑘 с элементами
𝛼(𝑝, 𝑞) = (𝐷𝑞 𝜑𝑖𝑝 )(u)
(𝑝, 𝑞 = 1, … , 𝑘).
Тогда
𝑑𝜑𝑖𝑝 =
∑
𝛼(𝑝, 𝑞) 𝑑𝑢𝑞 ,
𝑞
так что
𝑑𝜑𝑖1 ∧ ⋯ ∧ 𝑑𝜑𝑖𝑘 =
𝛼(1, 𝑞1 ) ⋯ 𝛼(𝑘, 𝑞𝑘 ) 𝑑𝑢𝑞1 ∧ ⋯ ∧ 𝑑𝑢𝑞𝑘 .
∑
В последней сумме 𝑞1 , … , 𝑞𝑘 независимо пробегают от 1 до 𝑘. Из антикоммутативного закона (42)
следует, что
𝑑𝑢𝑞1 ∧ ⋯ ∧ 𝑑𝑢𝑞𝑘 = 𝑠(𝑞1 , … , 𝑞𝑘 ) 𝑑𝑢1 ∧ ⋯ ∧ 𝑑𝑢𝑘 ,
где 𝑠 — то же, что в определении 9.33; применяя это определение, мы видим, что
𝑑𝜑𝑖1 ∧ ⋯ ∧ 𝑑𝜑𝑖𝑘 = det[𝐴] 𝑑𝑢1 ∧ ⋯ ∧ 𝑑𝑢𝑘 ;
а так как 𝐽 (u) = det[𝐴], то (72) доказано.
Комбинируя две последние теоремы, получаем заключительный результат этого раздела.
10.25 Теорема. Пусть 𝑇 — 𝒞 ′ -отображение открытого множества 𝐸 ⊂ 𝑅𝑛 в открытое множество 𝑉 ⊂ 𝑅𝑚 , Φ — 𝑘-поверхность в 𝐸, а 𝜔 — 𝑘-форма в 𝑉 .
Тогда
∫
𝑇Φ
𝜔=
∫
Φ
𝜔𝑇 .
Доказательство. Пусть 𝐷 — множество параметров поверхности Φ (и тем самым поверхности
𝑇 Φ). Определим Δ, как в теореме 10.24.
Тогда
∫
𝑇Φ
𝜔=
∫
Δ
𝜔𝑇 Φ =
∫
Δ
(𝜔𝑇 )Φ =
∫
Φ
𝜔𝑇 .
Первое из этих равенств — теорема 10.24, примененная к 𝑇 Φ вместо Φ. Второе следует из теоремы
10.23. Третье — теорема 10.24, примененная к 𝜔𝑇 вместо 𝜔.
Симплексы и цепи
10.26 Аффинные симплексы. Отображение f векторного пространства 𝑋 в векторное пространство 𝑌 называют аффинным, если f − f(0) линейно. Другими словами, требование состоит в том,
что
(73)
f(x) = f(0) + 𝐴x
при некотором 𝐴 ∈ 𝐿(𝑋, 𝑌 ).
Аффинное отображение пространства 𝑅𝑘 в 𝑅𝑛 тем самым определено, если нам известно f(0) и
f(e𝑖 ) при 1 ≤ 𝑖 ≤ 𝑘; как обычно, {e1 , … , e𝑘 } — стандартный базис пространства 𝑅𝑘 .
198
Стандартным симплексом 𝑄𝑘 называется множество всех u ∈ 𝑅𝑘 вида
𝑘
(74)
u=
∑
𝛼𝑖 e𝑖 ,
𝑖=1
таких, что 𝛼𝑖 ≥ 0 при 𝑖 = 1, … , 𝑘 и ∑ 𝛼𝑖 ≤ 1.
Положим теперь, что p0 , p1 , … , p𝑘 — точки пространства 𝑅𝑛 . Ориентированным аффинным 𝑘симплексом
(75)
𝜎 = [p0 , p1 , … , p𝑘 ]
называется 𝑘-поверхность в 𝑅𝑛 с множеством параметров 𝑄𝑘 , заданная аффинным отображением
𝑘
(76)
𝜎(𝛼1 e1 + ⋯ + 𝛼𝑘 e𝑘 ) = p0 +
∑
𝛼𝑖 (p𝑖 − p0 ).
𝑖=1
Заметим, что 𝜎 характеризуется тем, что
(77)
𝜎(0) = p0 ,
𝜎(e𝑖 ) = p𝑖
(при 1 ≤ 𝑖 ≤ 𝑘),
и что
(78)
𝜎(u) = p0 + 𝐴u
(u ∈ 𝑄𝑘 ),
где 𝐴 ∈ 𝐿(𝑅𝑘 , 𝑅𝑛 ) и 𝐴e𝑖 = p𝑖 − p0 при 1 ≤ 𝑖 ≤ 𝑘.
Мы называем 𝜎 ориентированным, чтобы подчеркнуть, что порядок вершин p0 , … , p𝑘 следует
учитывать. Если
(79)
𝜎̄ = [p𝑖0 , p𝑖1 , … , p𝑖𝑘 ],
где {𝑖0 , 𝑖1 , … , 𝑖𝑘 } — перестановка упорядоченного множества {0, 1, … , 𝑘}, то мы будем писать
(80)
𝜎̄ = 𝑠(𝑖0 , 𝑖1 , … , 𝑖𝑘 )𝜎,
где 𝑠 — функция из определения 9.33. Таким образом, 𝜎̄ = ±𝜎 в зависимости от того, 𝑠 = 1 или
𝑠 = −1. Строго говоря, считая (75) и (76) определением симплекса 𝜎, мы не имеем права писать
𝜎̄ = 𝜎, кроме случая, когда 𝑖0 = 0, … , 𝑖𝑘 = 𝑘, даже если 𝑠(𝑖0 , … , 𝑖𝑘 ) = 1; здесь мы имеем дело
не с равенством, а с отношением эквивалентности. Однако для наших целей такое обозначение
оправдано, как показывает теорема 10.27.
Если 𝜎̄ = 𝜀𝜎 (следуя только что принятому соглашению) и если 𝜀 = 1, то говорят, что 𝜎̄ и 𝜎 имеют
одинаковую ориентацию; если 𝜀 = −1, то говорят, что 𝜎̄ и 𝜎 имеют противоположные ориентации.
Заметим, что мы не определяем, что такое «ориентация симплекса». То, что мы определили, —
это отношение между парами симплексов, имеющих одно и то же множество вершин, отношение
«иметь одинаковую ориентацию».
Существует, однако, ситуация, в которой ориентация симплекса может быть определена естественным способом. Это происходит, если 𝑛 = 𝑘 и если векторы p𝑖 − p0 (1 ≤ 𝑖 ≤ 𝑘) независимы. В
этом случае линейное преобразование 𝐴, входящее в (78), обратимо, и его определитель (равный
якобиану симплекса 𝜎) отличен от 0. Тогда 𝜎 называют положительно (отрицательно) ориентированным, если det 𝐴 положителен (отрицателен). В частности, симплекс [0, e1 , … , e𝑘 ] в 𝑅𝑘 , заданный
тождественным отображением, имеет положительную ориентацию.
До сих пор мы предполагали, что 𝑘 ≥ 1. Ориентированным 0-симплексом называется точка,
которой приписан некоторый знак. Мы пишем 𝜎 = +p0 или 𝜎 = −p0 . Если 𝜎 = 𝜀p0 (𝜀 = ±1) и если
𝑓 — 0-форма (т. е. вещественная функция), то мы полагаем по определению
∫
𝜎
𝑓 = 𝜀𝑓 (p0 ).
199
10.27 Теорема. Если 𝜎 — ориентированный аффинный 𝑘-симплекс в открытом множестве 𝐸 ⊂
𝑅𝑛 и если 𝜎̄ = 𝜀𝜎, то
(81)
∫
𝜎̄
𝜔=𝜀
∫
𝜎
𝜔
для любой 𝑘-формы 𝜔 в 𝐸.
Доказательство. Если 𝑘 = 0, то (81) следует из предыдущего определения. Итак, будем считать,
что 𝑘 ≥ 1 и что 𝜎 — симплекс (75).
Пусть 1 ≤ 𝑗 ≤ 𝑘, и пусть 𝜎̄ получается из 𝜎 перестановкой вершин p0 и p𝑗 . Тогда 𝜀 = −1 и
(u ∈ 𝑄𝑘 ),
𝜎(u)
̄
= p𝑗 + 𝐵u
где 𝐵 — линейное отображение пространства 𝑅𝑘 в 𝑅𝑛 , определенное равенствами 𝐵e𝑗 = p0 − p𝑗 ,
𝐵e𝑖 = p𝑖 − p𝑗 , если 𝑖 ≠ 𝑗. Обозначая 𝐴e𝑖 = x𝑖 (1 ≤ 𝑖 ≤ 𝑘), где 𝐴 задано равенствами (78), мы видим,
что векторы-столбцы отображения 𝐵 (т. е. векторы 𝐵e𝑖 ) равны
x1 − x𝑗 , … , x𝑗−1 − x𝑗 , −x𝑗 , x𝑗+1 − x𝑗 , … , x𝑘 − x𝑗 .
Если вычесть 𝑗-й столбец из каждого из остальных, то ни один из определителей в (35) не изменится, и мы получим столбцы x1 , … , x𝑗−1 , −x𝑗 , x𝑗+1 , … , x𝑘 . Они отличаются от столбцов преобразования
𝐴 только знаком 𝑗-го столбца. Значит, в этом случае (81) выполняется.
Пусть теперь 0 < 𝑖 < 𝑗 ≤ 𝑘, и пусть 𝜎̄ получается из 𝜎 перестановкой вершин p𝑖 и p𝑗 . Тогда 𝜎(u)
̄
=
p0 + 𝐶u, где 𝐶 имеет те же столбцы, что и 𝐴, за исключением 𝑖-го и 𝑗-го, которые переставлены.
Отсюда снова следует, что выполняется (81), так как 𝜀 = −1.
Общий случай также выполняется, так как каждая перестановка множества {0, 1, … , 𝑘} является композицией частных случаев, с которыми мы уже имели дело.
10.28 Аффинные цепи. Аффинной 𝑘-цепью Γ в открытом множестве 𝐸 ⊂ 𝑅𝑛 называется семейство, состоящее из конечного числа ориентированных аффинных 𝑘-симплексов 𝜎1 , … , 𝜎𝑟 в 𝐸. Они
необязательно различны, то есть один и тот же симплекс может встретиться в Γ несколько раз.
Если Γ — такая цепь и если 𝜔 — 𝑘-форма в 𝐸, то по определению полагаем
𝑟
(82)
∫
Γ
𝜔=
∑∫
𝜎
𝑖=1
𝜔.
𝑖
Мы можем рассматривать 𝑘-поверхность Φ в 𝐸 как функцию, определенную на множестве
всех 𝑘-форм в 𝐸 и сопоставляющую форме 𝜔 число ∫Φ 𝜔. Вещественнозначные функции можно
складывать (см. определение 4.3), что подсказывает использование обозначения
(83)
Γ = 𝜎1 + ⋯ + 𝜎𝑟
или, более сжато,
𝑟
(84)
Γ=
∑
𝜎𝑖 ,
𝑖=1
подчеркивающего тот факт, что (82) выполняется для каждой 𝑘-формы 𝜔 в 𝐸.
Чтобы избежать непонимания, особо отметим, что введенные в (83) и (80) обозначения необходимо использовать с осторожностью. Дело в том, что каждый ориентированный аффинный 𝑘симплекс 𝜎 в 𝑅𝑛 является функцией двояким образом, с различными областями определения и
множествами значений, и, следовательно, возможны две совершенно разные операции сложения.
Изначально симплекс 𝜎 был определен как функция с областью определения 𝑄𝑘 и значениями в
𝑅𝑛 ; соответственно, сумма 𝜎1 + 𝜎2 могла бы определяться как функция 𝜎, ставящая в соответствие
каждому u ∈ 𝑄𝑘 вектор 𝜎1 (u) + 𝜎2 (u); отметим, что тогда 𝜎 — снова ориентированный аффинный
𝑘-симплекс в 𝑅𝑛 ! Это не то, что имелось в виду в (83).
Например, если 𝜎2 = −𝜎1 в смысле (80) (то есть если 𝜎1 и 𝜎2 имеют одно и то же множество
вершин, но противоположную ориентацию) и если Γ = 𝜎1 + 𝜎2 , то ∫Γ 𝜔 = 0 для всех 𝜔, и мы можем
выразить это, написав Γ = 0 или 𝜎1 + 𝜎2 = 0. Это не означает, что 𝜎1 (u) + 𝜎2 (u) является нулевым
вектором пространства 𝑅𝑛 .
200
10.29 Границы. При 𝑘 ≥ 1 границей ориентированного аффинного 𝑘-симплекса
𝜎 = [p0 , p1 , … , p𝑘 ]
называется аффинная (𝑘 − 1)-цепь
𝑘
(85)
𝜕𝜎 =
∑
(−1)𝑗 [p0 , … , p𝑗−1 , p𝑗+1 , … , p𝑘 ].
𝑗=0
Например, если 𝜎 = [p0 , p1 , p2 ], то
𝜕𝜎 = [p1 , p2 ] − [p0 , p2 ] + [p0 , p1 ] = [p0 , p1 ] + [p1 , p2 ] + [p2 , p0 ],
что совпадает с обычным определением ориентированной границы треугольника.
Заметим, что если 1 ≤ 𝑗 ≤ 𝑘, то симплекс 𝜎𝑗 = [p0 , … , p𝑗−1 , p𝑗+1 , … , p𝑘 ], входящий в (85), имеет
𝑄𝑘−1 своим множеством параметров и определяется равенством
(86)
(u ∈ 𝑄𝑘−1 ),
𝜎𝑗 (u) = p0 + 𝐵u
где 𝐵 — линейное отображение из 𝑅𝑘−1 в 𝑅𝑛 , определяемое равенствами
𝐵e𝑖 = p𝑖 − p0
(если 1 ≤ 𝑖 ≤ 𝑗 − 1),
𝐵e𝑖 = p𝑖+1 − p0
(если 𝑗 ≤ 𝑖 ≤ 𝑘 − 1).
Симплекс
𝜎0 = [p1 , p2 , … , p𝑘 ],
который тоже входит в (85), задается отображением
𝜎0 (u) = p1 + 𝐵u,
где 𝐵e𝑖 = p𝑖+1 − p𝑖 при 1 ≤ 𝑖 ≤ 𝑘 − 1.
10.30 Дифференцируемые симплексы и цепи. Пусть 𝑇 — 𝒞 ″ -отображение открытого множества 𝐸 ⊂ 𝑅𝑛 в открытое множество 𝑉 ⊂ 𝑅𝑚 , не обязательно взаимно однозначное. Если 𝜎 —
ориентируемый аффинный 𝑘-симплекс в 𝐸, то композиция отображений Φ = 𝑇 ∘ 𝜎 (мы иногда будем писать просто 𝑇 𝜎) — 𝑘-поверхность в 𝑉 с множеством параметров 𝑄𝑘 . Мы будем называть Φ
ориентированным 𝑘-симплексом класса 𝒞 ″ .
Конечное семейство Ψ ориентированных 𝑘-симплексов Φ1 , … , Φ𝑟 класса 𝒞 ″ в 𝑉 называется 𝑘цепью класса 𝒞 ″ в 𝑉 . Если 𝜔 — 𝑘-форма в 𝑉 , то по определению полагаем
𝑟
(87)
∫
Ψ
𝜔=
∑∫
Φ
𝑖=1
𝜔
𝑖
и используем соответствующее обозначение Ψ = ∑ Φ𝑖 .
Если Γ = ∑ 𝜎𝑖 — аффинная цепь и Φ𝑖 = 𝑇 ∘ 𝜎𝑖 , то мы будем писать Ψ = 𝑇 ∘ Γ или
(88)
𝑇(
∑
𝜎𝑖 ) =
∑
𝑇 𝜎𝑖 .
Граница 𝜕Φ ориентированного 𝑘-симплекса Φ = 𝑇 ∘ 𝜎 — это по определению (𝑘 − 1)-цепь
(89)
𝜕Φ = 𝑇 (𝜕𝜎).
В обоснование (89) заметим, что если 𝑇 аффинно, то Φ = 𝑇 ∘𝜎 — ориентированный аффинный 𝑘симплекс, и в этом случае (89) не просто определение, а является следствием (85). Таким образом,
(89) обобщает этот частный случай.
Очевидно, что 𝜕Φ принадлежит классу 𝒞 ″ , если Φ принадлежит этому классу.
Наконец, мы определяем границу 𝜕Ψ 𝑘-цепи Ψ = ∑ Φ𝑖 как (𝑘 − 1)-цепь
(90)
𝜕Ψ =
∑
201
𝜕Φ𝑖 .
10.31 Положительно ориентированные границы. До сих пор мы связывали границы с цепями, а не с подмножествами пространства 𝑅𝑛 . Такое понятие границы более всего подходит для
условия и доказательства теоремы Стокса. Однако в приложениях, особенно в 𝑅2 или 𝑅3 , привычно
и удобно говорить также о «ориентированных границах» различных множеств. Мы кратко опишем
это.
Пусть 𝑄𝑛 — стандартный симплекс в 𝑅𝑛 , и пусть 𝜎0 — тождественное отображение с областью
определения 𝑄𝑛 . Как мы видели в п. 10.26, 𝜎0 можно рассматривать как положительно ориентированный 𝑛-симплекс в 𝑅𝑛 . Его граница 𝜕𝜎0 — аффинная (𝑛 − 1)-цепь. Эта цепь называется положительно ориентированной границей множества 𝑄𝑛 .
Например, положительно ориентированной границей множества 𝑄3 является
[e1 , e2 , e3 ] − [0, e2 , e3 ] + [0, e1 , e3 ] − [0, e1 , e2 ].
Далее, пусть 𝑇 — взаимно однозначное отображение симплекса 𝑄𝑛 в 𝑅𝑛 класса 𝒞 ″ , якобиан которого положителен (по крайней мере во внутренности симплекса 𝑄𝑛 ). Обозначим 𝐸 = 𝑇 (𝑄𝑛 ). По
теореме об обратной функции 𝐸 является замыканием некоторого открытого подмножества пространства 𝑅𝑛 . Положительно ориентированной границей множества 𝐸 по определению называется
(𝑛 − 1)-цепь
𝜕𝑇 = 𝑇 (𝜕𝜎0 ),
которая может обозначаться через 𝜕𝐸.
Возникает очевидный вопрос: если 𝐸 = 𝑇1 (𝑄𝑛 ) = 𝑇2 (𝑄𝑛 ), а 𝑇1 и 𝑇2 оба имеют положительный
якобиан, верно ли, что 𝜕𝑇1 = 𝜕𝑇2 ? Другими словами, выполняется ли равенство
∫
𝜕𝑇1
𝜔=
∫
𝜕𝑇2
𝜔
для любой (𝑛 − 1)-формы 𝜔? Ответ положителен, но мы опустим доказательство (пример можно
увидеть, сравнив конец этого раздела с упражнением 17).
Конструкцию можно продолжить. Пусть
Ω = 𝐸1 ∪ ⋯ ∪ 𝐸𝑟 ,
где 𝐸𝑖 = 𝑇𝑖 (𝑄𝑛 ), каждое 𝑇𝑖 имеет те же свойства, что и 𝑇 выше, а внутренности множеств 𝐸𝑖 попарно
не пересекаются. Тогда (𝑛 − 1)-цепь
𝜕𝑇1 + ⋯ + 𝜕𝑇𝑟 = 𝜕Ω
называется положительно ориентированной границей множества Ω.
Например, единичный квадрат 𝐼 2 в 𝑅2 является объединением симплексов 𝜎1 (𝑄2 ) и 𝜎2 (𝑄2 ), где
𝜎1 (u) = u,
𝜎2 (u) = e1 + e2 − u.
Как 𝜎1 , так и 𝜎2 имеют якобиан 1 > 0. Так как
𝜎1 = [0, e1 , e2 ],
𝜎2 = [e1 + e2 , e2 , e1 ],
то мы имеем
𝜕𝜎1 = [e1 , e2 ] − [0, e2 ] + [0, e1 ],
𝜕𝜎2 = [e2 , e1 ] − [e1 + e2 , e1 ] + [e1 + e2 , e2 ];
сумма этих двух границ равна
𝜕𝐼 2 = [0, e1 ] + [e1 , e1 + e2 ] + [e1 + e2 , e2 ] + [e2 , 0],
то есть положительно ориентированной границе квадрата 𝐼 2 . Отметим, что слагаемые [e1 , e2 ] и
[e2 , e1 ] взаимно уничтожились.
Если Φ — 2-поверхность в 𝑅𝑚 с множеством параметров 𝐼 2 , то Φ (рассматриваемая как функция
на 2-формах) совпадает с 2-цепью
Φ ∘ 𝜎1 + Φ ∘ 𝜎 2 .
202
Таким образом,
𝜕Φ = 𝜕(Φ ∘ 𝜎1 ) + 𝜕(Φ ∘ 𝜎2 )
= Φ(𝜕𝜎1 ) + Φ(𝜕𝜎2 ) = Φ(𝜕𝐼 2 ).
Другими словами, если множество параметров поверхности Φ — квадрат 𝐼 2 , то мы можем, не
возвращаясь к симплексу 𝑄2 , получить 𝜕Φ напрямую из 𝜕𝐼 2 .
Другие примеры можно найти в упражнениях 17-19.
10.32 Пример. При 0 ≤ 𝑢 ≤ 𝜋, 0 ≤ 𝑣 ≤ 2𝜋 положим
Σ(𝑢, 𝑣) = (sin 𝑢 cos 𝑣, sin 𝑢 sin 𝑣, cos 𝑢).
Тогда Σ — 2-поверхность в 𝑅3 , множество параметров которой — прямоугольник 𝐷 ⊂ 𝑅2 , а область
значений — единичная сфера в 𝑅3 . Ее граница равна
𝜕Σ = Σ(𝜕𝐷) = 𝛾1 + 𝛾2 + 𝛾3 + 𝛾4 ,
где
𝛾1 (𝑢) = Σ(𝑢, 0) = (sin 𝑢, 0, cos 𝑢),
𝛾2 (𝑣) = Σ(𝜋, 𝑣) = (0, 0, −1),
𝛾3 (𝑢) = Σ(𝜋 − 𝑢, 2𝜋) = (sin 𝑢, 0, − cos 𝑢),
𝛾4 (𝑣) = Σ(0, 2𝜋 − 𝑣) = (0, 0, 1),
а сегменты параметров для 𝑢 и 𝑣 соответственно равны [0, 𝜋] и [0, 2𝜋].
Ввиду того что 𝛾2 и 𝛾4 постоянны, их производные равны 0, значит, интеграл от любой 1-формы
по 𝛾2 или 𝛾4 равен 0 (см. пример 10.12(a)).
Ввиду того что 𝛾3 (𝑢) = 𝛾1 (𝜋 − 𝑢), непосредственное применение (35) показывает, что
𝜔=−
∫
𝛾3
∫
𝛾1
𝜔
для каждой 1-формы 𝜔. Значит, ∫𝜕Σ 𝜔 = 0, и мы заключаем, что 𝜕Σ = 0.
(Используя географическую терминологию, 𝜕Σ начинается на северном полюсе 𝑁, движется к
южному полюсу 𝑆 по меридиану, останавливается в 𝑆, возвращается в 𝑁 по тому же меридиану и,
наконец, останавливается в 𝑁. Два прохода по меридиану происходят в противоположных направлениях. Следовательно, два соответствующих криволинейных интеграла взаимно уничтожаются.
В упражнении 32 приведена кривая, также дважды проходящая по границе, но без взаимного
уничтожения.)
Теорема Стокса
10.33 Теорема. Если Ψ — 𝑘-цепь класса 𝒞 ″ в открытом множестве 𝑉 ⊂ 𝑅𝑚 и если 𝜔 — (𝑘 − 1)форма класса 𝒞 ′ в 𝑉 , то
(91)
∫
Ψ
𝑑𝜔 =
∫
𝜕Ψ
𝜔.
Случай 𝑘 = 𝑚 = 1 — не что иное, как основная теорема интегрального исчисления (с дополнительным условием дифференцируемости). Случай 𝑘 = 𝑚 = 2 — теорема Грина, а 𝑘 = 𝑚 = 3 — так
называемая «теорема о дивергенции» Гаусса. Случай 𝑘 = 2, 𝑚 = 3 первоначально открыт Стоксом
(в книге Спивака описаны исторические обстоятельства). Эти частные случаи будут подробнее
рассмотрены в конце настоящей главы.
Доказательство. Достаточно доказать, что
(92)
∫
Φ
𝑑𝜔 =
203
∫
𝜕Φ
𝜔
для каждого ориентированного 𝑘-симплекса Φ класса 𝒞 ″ в 𝑉 , так как если доказано (92), а Ψ =
∑ Φ𝑖 , то из (87) и (89) следует (91).
Зафиксируем такой симплекс Φ и положим
(93)
𝜎 = [0, e1 , … , e𝑘 ].
Тогда 𝜎 — ориентированный аффинный 𝑘-симплекс с множеством параметров 𝑄𝑘 , заданный тождественным отображением. Ввиду того что Φ также определен на 𝑄𝑘 (см. определение 10.30) и
Φ ∈ 𝒞 ″ , существует открытое множество 𝐸 ⊂ 𝑅𝑘 , содержащее 𝑄𝑘 , и 𝒞 ″ -отображение 𝑇 множества
𝐸 в 𝑉 , такое, что Φ = 𝑇 ∘ 𝜎. Согласно теоремам 10.25 и 10.22(c) левая часть (92) равна
∫
𝑇𝜎
𝑑𝜔 =
(𝑑𝜔)𝑇 =
𝑑(𝜔𝑇 ).
∫
∫
𝜎
𝜎
Еще одно применение теоремы 10.25 с учетом (89) показывает, что правая часть (92) равна
∫
𝜕(𝑇 𝜎)
𝜔=
∫
𝑇 (𝜕𝜎)
𝜔=
∫
𝜕𝜎
𝜔𝑇 .
Так как 𝜔𝑇 — (𝑘 − 1)-форма в 𝐸, то мы видим, что для доказательства (92) достаточно показать, что
(94)
∫
𝜎
𝑑𝜆 =
𝜆
∫
𝜕𝜎
для специального симплекса (93) и для каждой (𝑘 − 1)-формы 𝜆 класса 𝒞 ′ в 𝐸.
Если 𝑘 = 1, то из определения ориентированного 0-симплекса вытекает, что в (94) утверждается
всего лишь то, что
1
(95)
∫
0
𝑓 ′ (𝑢) 𝑑𝑢 = 𝑓 (1) − 𝑓 (0)
для любой непрерывно дифференцируемой функции 𝑓 на [0, 1], что справедливо согласно основной
теореме интегрального исчисления.
С этого момента мы будем полагать, что 𝑘 > 1. Зафиксируем целое 𝑟 (1 ≤ 𝑟 ≤ 𝑘) и выберем
𝑓 ∈ 𝒞 ′ (𝐸). Достаточно доказать (94) для случая
(96)
𝜆 = 𝑓 (x) 𝑑𝑥1 ∧ ⋯ ∧ 𝑑𝑥𝑟−1 ∧ 𝑑𝑥𝑟+1 ∧ ⋯ ∧ 𝑑𝑥𝑘 ,
так как каждая (𝑘 − 1)-форма равна сумме таких специальных форм при 𝑟 = 1, … , 𝑘.
Ввиду (85) граница симплекса (93) равна
𝑘
𝜕𝜎 = [e1 , … , e𝑘 ] +
∑
(−1)𝑖 𝜏𝑖 ,
𝑖=1
где
𝜏𝑖 = [0, e1 , … , e𝑖−1 , e𝑖+1 , … , e𝑘 ]
при 𝑖 = 1, … , 𝑘. Положим
𝜏0 = [e𝑟 , e1 , … , e𝑟−1 , e𝑟+1 , … , e𝑘 ].
Отметим, что 𝜏0 получается из [e1 , … , e𝑘 ] с помощью 𝑟 − 1 последовательных перестановок e𝑟 с
соседями слева. Таким образом,
𝑘
(97)
𝜕𝜎 = (−1)𝑟−1 𝜏0 +
∑
(−1)𝑖 𝜏𝑖 .
𝑖=1
Каждый симплекс 𝜏𝑖 имеет множество параметров 𝑄𝑘−1 .
204
Если x = 𝜏0 (u) и u ∈ 𝑄𝑘−1 , то
(98)
(1 ≤ 𝑗 < 𝑟),
⎧𝑢𝑗
⎪
𝑥𝑗 = ⎨1 − (𝑢1 + ⋯ + 𝑢𝑘−1 ) (𝑗 = 𝑟),
⎪𝑢
(𝑟 < 𝑗 ≤ 𝑘).
⎩ 𝑗−1
Если 1 ≤ 𝑖 ≤ 𝑘, u ∈ 𝑄𝑘−1 , а x = 𝜏𝑖 (u), то
⎧𝑢𝑗
⎪
𝑥𝑗 = ⎨0
⎪𝑢
⎩ 𝑗−1
(99)
(1 ≤ 𝑗 < 𝑖),
(𝑗 = 𝑖),
(𝑖 < 𝑗 ≤ 𝑘).
При 0 ≤ 𝑖 ≤ 𝑘 обозначим через 𝐽𝑖 якобиан отображения
(100)
(𝑢1 , … , 𝑢𝑘−1 ) → (𝑥1 , … , 𝑥𝑟−1 , 𝑥𝑟+1 , … , 𝑥𝑘 ),
задаваемого симплексом 𝜏𝑖 . Если 𝑖 = 0 или 𝑖 = 𝑟, то (98) и (99) показывают, что (100) — тождественное отображение. Таким образом, 𝐽0 = 1, 𝐽𝑟 = 1. Для остальных 𝑖 из того, что 𝑥𝑖 = 0 в (99),
следует, что 𝐽𝑖 имеет строку нулей, значит, 𝐽𝑖 = 0. Таким образом,
(101)
∫
𝜏𝑖
𝜆=0
(𝑖 ≠ 0, 𝑖 ≠ 𝑟)
в силу (35) и (96). В результате (97) дает
(102)
∫
𝜕𝜎
𝜆 = (−1)𝑟−1
= (−1)𝑟−1
∫
𝜏0
∫
𝜆 + (−1)𝑟
∫
𝜏𝑟
𝜆
[𝑓 (𝜏0 (u)) − 𝑓 (𝜏𝑟 (u))] 𝑑u.
С другой стороны,
𝑑𝜆 = (𝐷𝑟 𝑓 )(x) 𝑑𝑥𝑟 ∧ 𝑑𝑥1 ∧ ⋯ ∧ 𝑑𝑥𝑟−1 ∧ 𝑑𝑥𝑟+1 ∧ ⋯ ∧ 𝑑𝑥𝑘
= (−1)𝑟−1 (𝐷𝑟 𝑓 )(x) 𝑑𝑥1 ∧ ⋯ ∧ 𝑑𝑥𝑘 ,
так что
(103)
∫
𝜎
𝑑𝜆 = (−1)𝑟−1
∫
𝑄𝑘
(𝐷𝑟 𝑓 )(x) 𝑑x.
Вычислим (103), проинтегрировав вначале по 𝑥𝑟 по сегменту
[0, 1 − (𝑥1 + ⋯ + 𝑥𝑟−1 + 𝑥𝑟+1 + ⋯ + 𝑥𝑘 )],
положим (𝑥1 , … , 𝑥𝑟−1 , 𝑥𝑟+1 , … , 𝑥𝑘 ) = (𝑢1 , … , 𝑢𝑘−1 ), после чего с помощью (98) мы видим, что интеграл
по 𝑄𝑘 в (103) равен интегралу по 𝑄𝑘−1 в (102). Таким образом, (94) выполняется, и доказательство
закончено.
Замкнутые формы и точные формы
10.34 Определение. Пусть 𝜔 — 𝑘-форма в открытом множестве 𝐸 ⊂ 𝑅𝑛 . Если существует (𝑘 − 1)форма 𝜆 в 𝐸, такая, что 𝜔 = 𝑑𝜆, то 𝜔 называют точной в 𝐸.
Если 𝜔 — класса 𝒞 ′ и 𝑑𝜔 = 0, то 𝜔 называют замкнутой.
Теорема 10.20(b) показывает, что каждая точная форма класса 𝒞 ′ замкнута.
В некоторых множествах 𝐸, например, в выпуклых, верно обратное; это доказывается в теореме
10.39 (известной как лемма Пуанкаре) и теореме 10.40. Однако в примерах 10.36 и 10.37 показаны
замкнутые формы, не являющиеся точными.
205
10.35 Замечания.
(a) Является ли данная 𝑘-форма 𝜔 замкнутой, можно определить, просто продифференцировав
коэффициенты ее стандартного вида. Например, 1-форма
𝑛
(104)
𝜔=
∑
𝑓𝑖 (x) 𝑑𝑥𝑖 ,
𝑖=1
где 𝑓𝑖 ∈ 𝒞 ′ (𝐸) для некоторого открытого множества 𝐸 ⊂ 𝑅𝑛 , замкнута тогда и только тогда,
когда уравнения
(105)
(𝐷𝑗 𝑓𝑖 )(x) = (𝐷𝑖 𝑓𝑗 )(x)
выполняются при всех 𝑖, 𝑗 в {1, … , 𝑛} и при всех x ∈ 𝐸.
Отметим, что (105) — «поточечное» условие; оно не включает в себя никаких глобальных
свойств, зависящих от формы множества 𝐸.
С другой стороны, чтобы показать, что 𝜔 точна в 𝐸, необходимо доказать существование
формы 𝜆, определенной в 𝐸, такой, что 𝑑𝜆 = 𝜔. Это сводится к решению системы дифференциальных уравнений в частных производных, не просто локально, а во всем 𝐸. Например,
чтобы показать, что форма (104) точна во множестве 𝐸, надо найти функцию (0-форму)
𝑔 ∈ 𝒞 ′ (𝐸), такую, что
(106)
(𝐷𝑖 𝑔)(x) = 𝑓𝑖 (x)
(x ∈ 𝐸, 1 ≤ 𝑖 ≤ 𝑛).
Естественно, (105) — необходимое условие разрешимости системы (106).
(b) Пусть 𝜔 — точная 𝑘-форма в 𝐸. Тогда существует (𝑘 − 1)-форма 𝜆 в 𝐸 с 𝑑𝜆 = 𝜔, и теорема
Стокса утверждает, что
(107)
∫
Ψ
𝜔=
∫
Ψ
𝑑𝜆 =
∫
𝜕Ψ
𝜆
для каждой 𝑘-цепи Ψ класса 𝒞 ″ в 𝐸.
Если Ψ1 и Ψ2 — такие цепи и если они имеют одинаковую границу, то
∫
Ψ1
𝜔=
∫
Ψ2
𝜔.
В частности, интеграл от точной 𝑘-формы в 𝐸 равен 0 по каждой 𝑘 цепи в 𝐸, граница
которой равна 0.
Как важный частный случай отметим, что интегралы точных 1-форм в 𝐸 равны 0 по замкнутым (дифференцируемым) кривым в 𝐸.
(c) Пусть 𝜔 — замкнутая 𝑘-форма в 𝐸. Тогда 𝑑𝜔 = 0, и теорема Стокса утверждает, что
(108)
∫
𝜕Ψ
𝜔=
∫
Ψ
𝑑𝜔 = 0
для любой (𝑘 + 1)-цепи Ψ класса 𝒞 ″ в 𝐸.
Другими словами, интегралы от замкнутых 𝑘-форм в 𝐸 равны 0 по 𝑘-цепям, являющихся
границами (𝑘 + 1)-цепей в 𝐸.
(d) Пусть Ψ — (𝑘 + 1)-цепь в 𝐸 и пусть 𝜆 — (𝑘 − 1)-форма в 𝐸, обе класса 𝒞 ″ . Ввиду того что
𝑑 2 𝜆 = 0, два применения теоремы Стокса показывают, что
(109)
∫
𝜕𝜕Ψ
𝜆=
∫
𝜕Ψ
𝑑𝜆 =
∫
Ψ
𝑑 2 𝜆 = 0.
Мы заключаем, что 𝜕 2 Ψ = 0. Другими словами, граница границы равна 0.
См. упражнение 16 для более прямого доказательства этого факта.
206
10.36 Пример. Пусть 𝐸 = 𝑅2 − {0}, плоскость с выколотым началом. 1-форма
(110)
𝜂=
𝑥 𝑑𝑦 − 𝑦 𝑑𝑥
𝑥2 + 𝑦 2
замкнута в 𝑅2 − {0}. Это легко проверить дифференцированием. Зафиксируем 𝑟 > 0 и положим
(111)
𝛾(𝑡) = (𝑟 cos 𝑡, 𝑟 sin 𝑡)
(0 ≤ 𝑡 ≤ 2𝜋).
Тогда 𝛾 — кривая («ориентированный 1-симплекс») в 𝑅2 − {0}. Ввиду того что 𝛾(0) = 𝛾(2𝜋), имеем
(112)
𝜕𝛾 = 0.
Прямое вычисление показывает, что
(113)
∫
𝛾
𝜂 = 2𝜋 ≠ 0.
Обсуждение в замечаниях 10.35(b) и (c) показывает, что из (113) можно сделать два вывода.
Во-первых, 𝜂 не является точной в 𝑅2 − {0}, так как в противном случае (112) сделало бы
интеграл (113) равным 0.
Во-вторых, 𝛾 не является границей никакой 2-цепи в 𝑅2 − {0} (класса 𝒞 ″ ), так как в противном
случае замкнутость формы 𝜂 сделала бы интеграл (113) равным 0.
10.37 Пример. Пусть 𝐸 = 𝑅3 − {0}, трехмерное пространство с выколотым началом. Положим
(114)
𝜁=
𝑥 𝑑𝑦 ∧ 𝑑𝑧 + 𝑦 𝑑𝑧 ∧ 𝑑𝑥 + 𝑧 𝑑𝑥 ∧ 𝑥𝑦
,
(𝑥2 + 𝑦2 + 𝑧2 )3/2
где мы написали (𝑥, 𝑦, 𝑧) вместо (𝑥1 , 𝑥2 , 𝑥3 ). Дифференцирование показывает, что 𝑑𝜁 = 0, так что
𝜁 — замкнутая 2-форма в 𝑅3 − {0}.
Пусть Σ — 2-цепь в 𝑅3 − {0}, построенная в примере 10.32; напомним, что Σ — параметризация единичной сферы в 𝑅3 . Используя прямоугольник 𝐷 из примера 10.32 в качестве множества
параметров, легко вычислить, что
(115)
∫
Σ
𝜁=
∫
𝐷
sin 𝑢 𝑑𝑢 𝑑𝑣 = 4𝜋 ≠ 0.
Как и в предыдущем примере, мы можем сделать вывод, что 𝜁 не является точной в 𝑅3 − {0}
(ввиду того, что 𝜕Σ = 0, как показано в примере 10.32) и что сфера Σ не является границей никакой
3-цепи в 𝑅3 − {0} (класса 𝒞 ″ ), хотя 𝜕Σ = 0.
Следующий результат будет использоваться в доказательстве теоремы 10.39.
10.38 Теорема. Пусть 𝐸 — выпуклое открытое множество в 𝑅𝑛 , 𝑓 ∈ 𝒞 ′ (𝐸), 𝑝 целое, 1 ≤ 𝑝 ≤ 𝑛
и
(116)
(𝑝 < 𝑗 ≤ 𝑛, x ∈ 𝐸).
(𝐷𝑗 𝑓 )(x) = 0
Тогда существует функция 𝐹 ∈ 𝒞 ′ (𝐸), такая, что
(117)
(𝐷𝑝 𝐹 )(x) = 𝑓 (x),
(𝐷𝑗 𝐹 )(x) = 0
(𝑝 < 𝑗 ≤ 𝑛, x ∈ 𝐸).
Доказательство. Запишем x = (x′ , 𝑥𝑝 , x″ ), где
x′ = (𝑥1 , … , 𝑥𝑝−1 ), x″ = (𝑥𝑝+1 , … , 𝑥𝑛 )
(если 𝑝 = 1, то x′ отсутствует; если 𝑝 = 𝑛, то x″ отсутствует). Пусть 𝑉 — множество всех (x′ , 𝑥𝑝 ) ∈
𝑅𝑝 , таких, что (x′ , 𝑥𝑝 , x″ ) ∈ 𝐸 при некотором x″ . Будучи проекцией множества 𝐸, 𝑉 является
выпуклым открытым множеством в 𝑅𝑝 . Ввиду того что 𝐸 выпукло и выполняется (116), 𝑓 (x) не
зависит от x″ . Таким образом, существует функция 𝜑 с областью определения 𝑉 , такая, что
𝑓 (x) = 𝜑(x′ , 𝑥𝑝 )
207
при всех x ∈ 𝐸.
Если 𝑝 = 1, то 𝑉 — интервал в 𝑅1 (возможно, неограниченный). Выберем 𝑐 ∈ 𝑉 и положим
𝑥1
𝐹 (x) =
𝜑(𝑡) 𝑑𝑡
∫
𝑐
(x ∈ 𝐸).
Если 𝑝 > 1, то обозначим через 𝑈 множество всех x′ ∈ 𝑅𝑝−1 , таких, что (x′ , 𝑥𝑝 ) ∈ 𝑉 при
некотором 𝑥𝑝 . Тогда 𝑈 — выпуклое открытое множество в 𝑅𝑝−1 , и существует функция 𝛼 ∈ 𝒞 ′ (𝑈 ),
такая, что (x′ , 𝛼(x′ )) ∈ 𝑉 при каждом x′ ∈ 𝑈 ; другими словами, график функции 𝛼 лежит в 𝑉
(упражнение 29). Положим
𝑥𝑝
𝐹 (x) =
𝜑(x′ , 𝑡) 𝑑𝑡
∫
𝛼(x′ )
(x ∈ 𝐸).
В обоих случаях 𝐹 удовлетворяет (117).
𝑏
𝑎
(Замечание. Напомним обычное соглашение, что ∫𝑎 означает − ∫𝑏 , если 𝑏 < 𝑎.)
10.39 Теорема. Если 𝐸 ⊂ 𝑅𝑛 выпукло и открыто, если 𝑘 ≥ 1, если 𝜔 — 𝑘-форма класса 𝒞 ′ в 𝐸 и
если 𝑑𝜔 = 0, то существует (𝑘 − 1)-форма 𝜆 в 𝐸, такая, что 𝜔 = 𝑑𝜆.
Короче говоря, замкнутые формы являются точными в выпуклых множествах.
Доказательство. При 𝑝 = 1, … , 𝑛 обозначим через 𝑌𝑝 множество всех 𝑘-форм 𝜔 класса 𝒞 ′ в 𝐸,
стандартный вид которых
(118)
𝜔=
∑
𝑓𝐼 (x) 𝑑𝑥𝐼
𝐼
не включает 𝑑𝑥𝑝+1 , … , 𝑑𝑥𝑛 . Другими словами, 𝐼 ⊂ {1, … , 𝑝}, если 𝑓𝐼 (x) ≠ 0 при некотором x ∈ 𝐸.
Проведем индукцию по 𝑝.
Пусть вначале 𝜔 ∈ 𝑌1 . Тогда 𝜔 = 𝑓 (x) 𝑑𝑥1 . Так как 𝑑𝜔 = 0, то (𝐷𝑗 𝑓 )(x) = 0 при 1 < 𝑗 ≤ 𝑛, x ∈ 𝐸.
По теореме 10.38 существует функция 𝐹 ∈ 𝒞 ′ (𝐸), такая, что 𝐷1 𝐹 = 𝑓 и 𝐷𝑗 𝐹 = 0 при 1 < 𝑗 ≤ 𝑛.
Тогда
𝑑𝐹 = (𝐷1 𝐹 )(x) 𝑑𝑥1 = 𝑓 (x) 𝑑𝑥1 = 𝜔.
Теперь возьмем 𝑝 > 1 и сделаем следующее предположение индукции: каждая замкнутая 𝑘форма, принадлежащая 𝑌𝑝−1 , точна в 𝐸.
Выберем 𝜔 ∈ 𝑌𝑝 так, что 𝑑𝜔 = 0. В силу (118)
𝑛
(119)
∑∑
(𝐷𝑗 𝑓𝐼 )(x) 𝑑𝑥𝑗 ∧ 𝑑𝑥𝐼 = 𝑑𝜔 = 0.
𝐼 𝑗=1
Рассмотрим фиксированное 𝑗, где 𝑝 < 𝑗 ≤ 𝑛. Каждый 𝐼, встречающийся в (118), лежит в {1, … , 𝑝}.
Если 𝐼1 , 𝐼2 — пара таких 𝑘-индексов и если 𝐼1 ≠ 𝐼2 , то (𝑘+1)-индексы (𝐼1 , 𝑗), (𝐼2 , 𝑗) различны. Таким
образом, взаимного уничтожения не происходит, и мы получаем из (119), что каждый коэффициент
в (118) удовлетворяет
(120)
(𝐷𝑗 𝑓𝐼 )(x) = 0
(x ∈ 𝐸, 𝑝 < 𝑗 ≤ 𝑛).
Теперь соберем все слагаемые в (118), содержащие 𝑑𝑥𝑝 , и перепишем 𝜔 в виде
(121)
𝜔=𝛼+
∑
𝐼0
𝑓𝐼 (x) 𝑑𝑥𝐼0 ∧ 𝑑𝑥𝑝 ,
где 𝛼 ∈ 𝑌𝑝−1 , каждый 𝐼0 — возрастающий (𝑘 − 1)-индекс в {1, … , 𝑝 − 1}, а 𝐼 = (𝐼0 , 𝑝). В силу (120)
теорема 10.38 предоставляет функции 𝐹𝐼 ∈ 𝒞 ′ (𝐸), такие, что
(122)
𝐷𝑝 𝐹𝐼 = 𝑓𝐼 ,
𝐷 𝑗 𝐹𝐼 = 0
Положим
(123)
𝛽=
∑
𝐼0
𝐹𝐼 (x) 𝑑𝑥𝐼0
208
(𝑝 < 𝑗 ≤ 𝑛).
и обозначим 𝛾 = 𝜔 − (−1)𝑘−1 𝑑𝛽. Из того, что 𝛽 — (𝑘 − 1)-форма, следует, что
𝑝
𝛾 =𝜔−
∑∑
𝐼0 𝑗=1
(𝐷𝑗 𝐹𝐼 )(x) 𝑑𝑥𝐼0 ∧ 𝑑𝑥𝑗
𝑝−1
=𝛼−
∑∑
𝐼0 𝑗=1
(𝐷𝑗 𝐹𝐼 )(x) 𝑑𝑥𝐼0 ∧ 𝑑𝑥𝑗 ,
что, очевидно, лежит в 𝑌𝑝−1 . Ввиду того что 𝑑𝜔 = 0 и 𝑑 2 𝛽 = 0, мы имеем 𝑑𝛾 = 0. Предположение
индукции поэтому показывает, что 𝛾 = 𝑑𝜇 для некоторой (𝑘−1)-формы 𝜇 в 𝐸. Если 𝜆 = 𝜇 +(−1)𝑘−1 𝛽,
то мы приходим к выводу, что 𝜔 = 𝑑𝜆.
Утверждение теоремы доказано по индукции.
10.40 Теорема. Зафиксируем 𝑘, 1 ≤ 𝑘 ≤ 𝑛. Пусть 𝐸 ⊂ 𝑅𝑛 — открытое множество, в котором
каждая замкнутая 𝑘-форма точна. Пусть 𝑇 — взаимно однозначное 𝒞 ″ -отображение множества 𝐸 на открытое множество 𝑈 ⊂ 𝑅𝑛 , обратное к которому 𝑆 также класса 𝒞 ″ .
Тогда каждая замкнутая 𝑘-форма в 𝑈 точна в 𝑈 .
Отметим, что каждое выпуклое открытое множество 𝐸 удовлетворяет данному условию по
теореме 10.39. Соотношение между 𝐸 и 𝑈 можно выразить, сказав, что они 𝒞 ″ -эквивалентны.
Таким образом, каждая замкнутая форма точна в любом множестве, 𝒞 ″ -эквивалентном выпуклому открытому множеству.
Доказательство. Пусть 𝜔 — 𝑘-форма в 𝑈 с 𝑑𝜔 = 0. По теореме 10.22(c) 𝜔𝑇 — 𝑘-форма в 𝐸, для
которой 𝑑(𝜔𝑇 ) = 0. Значит, 𝜔𝑇 = 𝑑𝜆 для некоторой (𝑘 − 1)-формы 𝜆 в 𝐸. Применив теорему 10.23 и
еще раз теорему 10.22(c), получим
𝜔 = (𝜔𝑇 )𝑆 = (𝑑𝜆)𝑆 = 𝑑(𝜆𝑆 ).
Так как 𝜆𝑆 — (𝑘 − 1)-форма в 𝑈 , то 𝜔 точна в 𝑈 .
10.41 Замечание. В приложениях клетки (см. определение 2.17) часто являются более удобными
множествами параметров, чем симплексы. Если бы все наше построение было основано на клетках,
а не на симплексах, то вычисления в доказательстве теоремы Стокса были бы даже проще (в книге
Спивака так и сделано). Мы предпочли симплексы, потому что определение границы ориентированного симплекса кажется легче и естественнее, чем для клетки (см. упражнение 19). Кроме того,
разбиение множеств на симплексы (называемое «триангуляцией») играет важную роль в топологии, и существуют тесные взаимосвязи между некоторыми аспектами топологии, с одной стороны,
и дифференциальными формами, с другой. Намеки на них содержатся в разделе 10.35. Хорошее
введение в этот вопрос содержит книга Зингера и Торпа.
Так как каждая клетка может быть триангулирована, то мы мы можем рассматривать ее как
цепь. Для двух измерений это сделано в примере 10.32; для трех измерений см. упражнение 18.
Лемма Пуанкаре (теорема 10.39) может быть доказана различными способами. См., например,
страницу 94 книги Спивака или страницу 280 книги Флеминга. Два простых доказательства некоторых частных случаев показаны в упражнениях 24 и 27.
Векторный анализ
Мы завершаем эту главу некоторыми приложениями предыдущего материала к теоремам, относящимся к векторному анализу в 𝑅3 . Это частные случаи теорем о дифференциальных формах,
но обычно они формулируются в других обозначениях. Таким образом, перед нами стоит задача
перевода с одного языка на другой.
10.42 Векторные поля. Пусть F = 𝐹1 e1 + 𝐹2 e2 + 𝐹3 e3 — непрерывное отображение открытого
множества 𝐸 ⊂ 𝑅3 в 𝑅3 . Ввиду того что F ставит в соответствие каждой точке множества 𝐸
209
некоторый вектор, F иногда называют векторным полем, особенно в физике. С каждым таким F
связана 1-форма
(124)
𝜆F = 𝐹1 𝑑𝑥 + 𝐹2 𝑑𝑦 + 𝐹3 𝑑𝑧
и 2-форма
(125)
𝜔F = 𝐹1 𝑑𝑦 ∧ 𝑑𝑧 + 𝐹2 𝑑𝑧 ∧ 𝑑𝑥 + 𝐹3 𝑑𝑥 ∧ 𝑑𝑦.
Здесь и в оставшейся части главы мы используем привычное обозначение (𝑥, 𝑦, 𝑧) вместо (𝑥1 , 𝑥2 , 𝑥3 ).
Обратно, ясно, что каждая 1-форма 𝜆 в 𝐸 является 𝜆F для некоторого векторного поля F в 𝐸,
и что каждая 2-форма 𝜔 является 𝜔F для некоторого F. В 𝑅3 , таким образом, изучение 1-форм и
2-форм равнообъемно изучению векторных полей.
Если 𝑢 ∈ 𝒞 ′ (𝐸) — вещественная функция, то ее градиент
∇𝑢 = (𝐷1 𝑢)e1 + (𝐷2 𝑢)e2 + (𝐷3 𝑢)e3
является примером векторного поля в 𝐸.
Пусть теперь F — векторное поле в 𝐸 класса 𝒞 ′ . Его вихрь (ротор) ∇ × F — векторное поле,
определенное в 𝐸 равенством
∇ × F = (𝐷2 𝐹3 − 𝐷3 𝐹2 )e1 + (𝐷3 𝐹1 − 𝐷1 𝐹3 )e2 + (𝐷1 𝐹2 − 𝐷2 𝐹1 )e3 ,
а его дивергенция — вещественная функция ∇ ⋅ F, определенная в 𝐸 равенством
∇ ⋅ F = 𝐷1 𝐹1 + 𝐷2 𝐹2 + 𝐷3 𝐹3 .
Эти величины имеют разнообразные физические интерпретации. За подробностями мы отсылаем к книге Келлога.
Далее следуют некоторые соотношения между градиентом, ротором и дивергенцией.
10.43 Теорема. Пусть 𝐸 — открытое множество в 𝑅3 , 𝑢 ∈ 𝒞 ″ (𝐸), а G — векторное поле в 𝐸
класса 𝒞 ″ .
(a) Если F = ∇𝑢, то ∇ × F = 0.
(b) Если F = ∇ × G, то ∇ ⋅ F = 0.
Далее, если 𝐸 𝒞 ″ -эквивалентно выпуклому множеству, то справедливы обратные утверждения
к (a) и (b), в которых мы предполагаем, что F — векторное поле в 𝐸 класса 𝒞 ′ .
(a’) Если ∇ × F = 0, то F = ∇𝑢 для некоторой функции 𝑢 ∈ 𝒞 ″ (𝐸).
(b’) Если ∇ ⋅ F = 0, то F = ∇ × G для некоторого векторного поля G в 𝐸 класса 𝒞 ″ .
Доказательство. Если мы сравним определения выражений ∇𝑢, ∇×F и ∇⋅F с дифференциальными формами 𝜆F и 𝜔F , определенными в (124) и (125), то получим четыре следующих утверждения.
F = ∇𝑢,
∇ × F = 0,
F = ∇ × G,
∇ ⋅ F = 0,
если и только если
если и только если
если и только если
если и только если
𝜆F = 𝑑𝑢.
𝑑𝜆F = 0.
𝜔F = 𝑑𝜆G .
𝑑𝜔F = 0.
Теперь, если F = ∇𝑢, то 𝜆F = 𝑑𝑢, значит, 𝑑𝜆F = 𝑑 2 𝑢 = 0 (теорема 10.20), что означает, что
∇ × F = 0. Таким образом, (a) доказано.
Что касается (a’), предположение сводится к тому, что 𝑑𝜆F = 0 в 𝐸. По теореме 10.40 имеем
𝜆F = 𝑑𝑢 для некоторой 0-формы 𝑢. Значит, F = ∇𝑢.
Доказательства (b) и (b’) проходят по точно такому же образцу.
10.44 Элементы объема. Элементом объема в 𝑅𝑘 называется 𝑘-форма
𝑑𝑥1 ∧ ⋯ ∧ 𝑑𝑥𝑘 .
210
Она часто обозначается через 𝑑𝑉 (или через 𝑑𝑉𝑘 , если желательно явным образом указать размерность), и используется обозначение
(126)
∫
Φ
𝑓 (x) 𝑑𝑥1 ∧ ⋯ ∧ 𝑑𝑥𝑘 =
∫
Φ
𝑓 𝑑𝑉 ,
если Φ — положительно ориентированная 𝑘-поверхность в 𝑅𝑘 , а 𝑓 — непрерывная функция на множестве значений поверхности Φ.
Причина использования этих обозначений очень проста: если 𝐷 — множество параметров в 𝑅𝑘 ,
а Φ — взаимно однозначное 𝒞 ′ -отображение множества 𝐷 в 𝑅𝑘 с положительным якобианом 𝐽Φ ,
то левая часть (126) равна
∫
𝐷
𝑓 (Φ(u))𝐽Φ (u) 𝑑u =
∫
Φ(𝐷)
𝑓 (x) 𝑑x
ввиду (35) и теоремы 10.9.
В частности, если 𝑓 = 1, то (126) называют объемом поверхности Φ. Мы уже видели частный
случай этого определения в (36).
Обычно 𝑑𝑉2 обозначают через 𝑑𝐴.
10.45 Теорема Грина. Пусть 𝐸 — открытое множество в 𝑅2 , 𝛼 ∈ 𝒞 ′ (𝐸), 𝛽 ∈ 𝒞 ′ (𝐸), а Ω —
замкнутое подмножество множества 𝐸 с положительно ориентированной границей 𝜕Ω, как
описано в п. 10.31. Тогда
(127)
∫
𝜕Ω
(𝛼 𝑑𝑥 + 𝛽 𝑑𝑦) =
𝜕𝛽 𝜕𝛼
−
𝑑𝐴.
(
∫
𝜕𝑦 )
Ω 𝜕𝑥
Доказательство. Положим 𝜆 = 𝛼 𝑑𝑥 + 𝛽 𝑑𝑦. Тогда
𝑑𝜆 = (𝐷2 𝛼) 𝑑𝑦 ∧ 𝑑𝑥 + (𝐷1 𝛽) 𝑑𝑥 ∧ 𝑑𝑦
= (𝐷1 𝛽 − 𝐷2 𝛼) 𝑑𝐴,
и (127) превращается в
∫
𝜕Ω
𝜆=
∫
Ω
𝑑𝜆,
что выполняется в силу теоремы 10.33.
При подстановке 𝛼(𝑥, 𝑦) = −𝑦 и 𝛽(𝑥, 𝑦) = 𝑥 (127) становится равным
1
2
(128)
∫
𝜕Ω
(𝑥 𝑑𝑦 − 𝑦 𝑑𝑥) = 𝐴(Ω),
площади множества Ω.
При 𝛼 = 0, 𝛽 = 𝑥 получается аналогичная формула. В примере 10.12(b) был рассмотрен частный
случай этого равенства.
10.46 Элементы площади в 𝑅3 . Пусть Φ — 2-поверхность в 𝑅3 класса 𝒞 ′ с множеством параметров 𝐷 ⊂ 𝑅2 . Свяжем с каждой точкой (𝑢, 𝑣) ∈ 𝐷 вектор
(129)
N(𝑢, 𝑣) =
𝜕(𝑦, 𝑧)
𝜕(𝑥, 𝑦)
𝜕(𝑧, 𝑥)
e +
e +
e .
𝜕(𝑢, 𝑣) 1 𝜕(𝑢, 𝑣) 2 𝜕(𝑢, 𝑣) 3
Якобианы в (129) соответствуют уравнению
(130)
(𝑥, 𝑦, 𝑧) = Φ(𝑢, 𝑣).
Если 𝑓 — непрерывная функция на Φ(𝐷), то поверхностный интеграл функции 𝑓 по Φ определяется как
(131)
∫
Φ
𝑓 𝑑𝐴 =
∫
𝐷
𝑓 (Φ(𝑢, 𝑣))|N(𝑢, 𝑣)| 𝑑𝑢 𝑑𝑣.
211
В частности, при 𝑓 = 1 мы получаем площадь поверхности Φ, а именно
(132)
𝐴(Φ) =
∫
𝐷
|N(𝑢, 𝑣)| 𝑑𝑢 𝑑𝑣.
Нижеследующее обсуждение покажет, что (131) и его частный случай (132) являются разумными определениями. Оно также опишет геометрические свойства вектора N.
Запишем Φ = 𝜑1 e1 + 𝜑2 e2 + 𝜑3 e3 , зафиксируем точку p0 = (𝑢0 , 𝑣0 ) ∈ 𝐷, положим N = N(p0 ),
положим
(133)
𝛼𝑖 = (𝐷1 𝜑𝑖 )(p0 ),
𝛽𝑖 = (𝐷2 𝜑𝑖 )(p0 )
(𝑖 = 1, 2, 3),
и пусть 𝑇 ∈ 𝐿(𝑅2 , 𝑅3 ) — линейное преобразование, заданное равенством
3
(134)
𝑇 (𝑢, 𝑣) =
∑
(𝛼𝑖 𝑢 + 𝛽𝑖 𝑣)e𝑖 .
𝑖=1
Отметим, что 𝑇 = Φ′ (p0 ) в соответствии с определением 9.11.
С этого момента будем полагать, что ранг преобразования 𝑇 равен 2 (если он равен 1 или 0,
то N = 0, и нижеупомянутая касательная плоскость вырождается в прямую или в точку). Тогда
множество значений аффинного отображения
(𝑢, 𝑣) → Φ(p0 ) + 𝑇 (𝑢, 𝑣)
является плоскостью Π, называемой касательной плоскостью к Φ в p0 (можно было бы назвать
Π касательной плоскостью в Φ(p0 ), а не в p0 , но это приводит к проблемам, если Φ не является
взаимно однозначным).
Если мы используем (133) в (129), то получим
(135)
N = (𝛼2 𝛽3 − 𝛼3 𝛽2 )e1 + (𝛼3 𝛽1 − 𝛼1 𝛽3 )e2 + (𝛼1 𝛽2 − 𝛼2 𝛽1 )e3 ,
и (134) показывает, что
3
(136)
𝑇 e1 =
3
∑
𝛼𝑖 e𝑖 ,
𝑇 e2 =
𝑖=1
∑
𝛽𝑖 e𝑖 .
𝑖=1
Простое вычисление теперь приводит к
(137)
N ⋅ (𝑇 e1 ) = 0 = N ⋅ (𝑇 e2 ).
Отсюда следует, что вектор N перпендикулярен к Π. По этой причине он называется нормалью к
Φ в p0 .
Второе свойство вектора N, также проверяемое прямым вычислением на основе (135) и (136), состоит в том, что определитель линейного преобразования пространства 𝑅3 , переводящего {e1 , e2 , e3 }
в {𝑇 e1 , 𝑇 e2 , N}, равен |N|2 > 0 (упражнение 30). Таким образом, 3-симплекс
(138)
[0, 𝑇 e1 , 𝑇 e2 , N]
положительно ориентирован.
Третье свойство вектора N, которым мы будем пользоваться, является следствием первых двух.
Вышеупомянутый определитель, равный |N|2 , равен объему параллелепипеда с ребрами [0, 𝑇 e1 ],
[0, 𝑇 e2 ], [0, N]. В силу (137) ребро [0, N] перпендикулярно другим двум ребрам. Площадь параллелограмма с вершинами
(139)
0, 𝑇 e1 , 𝑇 e2 , 𝑇 (e1 + e2 ),
следовательно, равна |N|.
212
Этот параллелограмм является образом единичного квадрата в 𝑅2 при отображении 𝑇 . Если
𝐸 — любой прямоугольник в 𝑅2 , из этого следует (в силу линейности отображения 𝑇 ), что площадь
параллелограмма 𝑇 (𝐸) равна
(140)
𝐴(𝑇 (𝐸)) = |N|𝐴(𝐸) =
∫
𝐸
|N(𝑢0 , 𝑣0 )| 𝑑𝑢 𝑑𝑣.
Мы приходим к выводу, что (132) выполняется, когда Φ аффинно. Чтобы обосновать определение (132) в общем случае, разделим 𝐷 на маленькие прямоугольники, выберем в каждом точку
(𝑢0 , 𝑣0 ) и заменим Φ в каждом прямоугольнике соответствующей касательной плоскостью. Сумма
площадей получившихся параллелограммов, вычисленных с помощью (140), является приближением к 𝐴(Φ). Наконец, (131) можно получить из (132), приближая 𝑓 ступенчатыми функциями.
10.47 Пример. Пусть заданы 0 < 𝑎 < 𝑏. Пусть 𝐾 — 3-клетка, определяемая неравенствами
0 ≤ 𝑡 ≤ 𝑎,
0 ≤ 𝑢 ≤ 2𝜋,
0 ≤ 𝑣 ≤ 2𝜋.
Уравнения
𝑥 = 𝑡 cos 𝑢,
(141)
𝑦 = (𝑏 + 𝑡 sin 𝑢) cos 𝑣,
𝑧 = (𝑏 + 𝑡 sin 𝑢) sin 𝑣
описывают отображение Ψ пространства 𝑅3 в 𝑅3 , взаимно однозначное во внутренности клетки 𝐾,
такое, что Ψ(𝐾) является полноторием. Его якобиан равен
𝐽Ψ =
𝜕(𝑥, 𝑦, 𝑧)
= 𝑡(𝑏 + 𝑡 sin 𝑢)
𝜕(𝑡, 𝑢, 𝑣)
и положителен на 𝐾, кроме грани 𝑡 = 0. Если проинтегрировать 𝐽Ψ по 𝐾, то получим
vol(Ψ(𝐾)) = 2𝜋 2 𝑎2 𝑏,
объем нашего полнотория.
Теперь рассмотрим 2-цепь Φ = 𝜕Ψ (см. упражнение 19). Ψ отображает грани 𝑢 = 0 и 𝑢 = 2𝜋
клетки 𝐾 на одну и ту же боковую поверхность цилиндра, но с противоположными ориентациями.
Ψ отображает грани 𝑣 = 0 и 𝑣 = 2𝜋 на один и тот же круг, но с противоположными ориентациями.
Ψ отображает грань 𝑡 = 0 на окружность, дающую нулевой вклад в 2-цепь 𝜕Ψ (соответствующие
якобианы равны 0). Таким образом, Φ — просто 2-поверхность, получаемая подстановкой 𝑡 = 𝑎 в
(141), с множеством параметров 𝐷 — квадратом, определяемым неравенствами 0 ≤ 𝑢 ≤ 2𝜋, 0 ≤ 𝑣 ≤
2𝜋.
Таким образом, согласно (129) и (141), нормаль к Φ в (𝑢, 𝑣) ∈ 𝐷 — вектор
N(𝑢, 𝑣) = 𝑎(𝑏 + 𝑎 sin 𝑢)n(𝑢, 𝑣),
где
n(𝑢, 𝑣) = (cos 𝑢)e1 + (sin 𝑢 cos 𝑣)e2 + (sin 𝑢 sin 𝑣)e3 .
Так как |n(𝑢, 𝑣)| = 1, то мы имеем |N(𝑢, 𝑣)| = 𝑎(𝑏 + 𝑎 sin 𝑢), и если проинтегрировать это выражение
по 𝐷, то с учетом (131) получается выражение
𝐴(Φ) = 4𝜋 2 𝑎𝑏
для площади поверхности нашего тора.
Если рассматривать N = N(𝑢, 𝑣) как направленный отрезок, указывающий из Φ(𝑢, 𝑣) в Φ(𝑢, 𝑣) +
N(𝑢, 𝑣), то N показывает наружу, то есть от Ψ(𝐾). Причина этого в том, что 𝐽Ψ > 0 при 𝑡 = 𝑎.
Например, возьмем 𝑢 = 𝑣 = 𝜋/2, 𝑡 = 𝑎. Это дает наибольшее значение 𝑧 на Ψ(𝐾), и N = 𝑎(𝑏 + 𝑎)e3
показывает «вверх» при таком выборе точки (𝑢, 𝑣).
213
10.48 Интегралы от 1-форм в 𝑅3 . Пусть 𝛾 — 𝒞 ′ -кривая в открытом множестве 𝐸 ⊂ 𝑅3 с сегментом параметров [0, 1], а F — векторное поле в 𝐸, как в п. 10.42. Определим 𝜆F с помощью (124).
Интеграл от 𝜆F по 𝛾 можно записать в виде, который мы сейчас опишем.
Для любого 𝑢 ∈ [0, 1] вектор
𝛾 ′ (𝑢) = 𝛾1′ (𝑢)e1 + 𝛾2′ (𝑢)e2 + 𝛾3′ (𝑢)e3
называется касательным вектором к 𝛾 в 𝑢. Обозначим через t = t(𝑢) единичный вектор в направлении 𝛾 ′ (𝑢). Таким образом,
𝛾 ′ (𝑢) = |𝛾 ′ (𝑢)|t(𝑢)
(если 𝛾 ′ (𝑢) = 0 при некотором 𝑢, то положим t(𝑢) = e1 ; любой другой выбор также подойдет). В
силу (35) имеем
3
∫
𝛾
(142)
𝜆F =
1
∑∫
0
𝑖=1
1
=
∫
0
1
=
∫
0
𝐹𝑖 (𝛾(𝑢))𝛾𝑖′ (𝑢) 𝑑𝑢
F(𝛾(𝑢)) ⋅ 𝛾 ′ (𝑢) 𝑑𝑢
F(𝛾(𝑢)) ⋅ t(𝑢)|𝛾 ′ (𝑢)| 𝑑𝑢.
Теорема 6.27 позволяет назвать |𝛾 ′ (𝑢)| 𝑑𝑢 элементом длины дуги вдоль 𝛾. Удобным обозначением
для этого является 𝑑𝑠, и (142) можно переписать в виде
(143)
∫
𝛾
𝜆F =
(F ⋅ t) 𝑑𝑠.
∫
𝛾
Ввиду того что t — единичный касательный вектор к 𝛾, F ⋅ t называют касательной составляющей векторного поля F вдоль 𝛾.
Правую часть (143) следует рассматривать просто как сокращение последнего интеграла в (142).
Дело в том, что F определено на области значений кривой 𝛾, но t определено на [0, 1]; таким
образом, F ⋅ t следует правильно понимать. Естественно, если 𝛾 взаимно однозначна, то t(𝑢) можно
заменить на t(𝛾(𝑢)), и данная трудность пропадает.
10.49 Интегралы от 2-форм в 𝑅3 . Пусть Φ — 2-поверхность в открытом множестве 𝐸 ⊂ 𝑅3
класса 𝒞 ′ с множеством параметров 𝐷 ⊂ 𝑅2 , а F — векторное поле в 𝐸. Определим 𝜔F с помощью
(125). Как и в предыдущем пункте, мы получим другое выражение для интеграла от 𝜔F по Φ.
В силу (35) и (129) имеем
∫
Φ
𝜔F =
=
=
∫
Φ
(𝐹1 𝑑𝑦 ∧ 𝑑𝑧 + 𝐹2 𝑑𝑧 ∧ 𝑑𝑥 + 𝐹3 𝑑𝑥 ∧ 𝑑𝑦)
𝜕(𝑦, 𝑧)
𝜕(𝑥, 𝑦)
𝜕(𝑧, 𝑥)
(𝐹1 ∘ Φ)
𝑑𝑢 𝑑𝑣
+ (𝐹2 ∘ Φ)
+ (𝐹3 ∘ Φ)
{
∫
𝜕(𝑢,
𝑣)
𝜕(𝑢,
𝑣)
𝜕(𝑢, 𝑣) }
𝐷
∫
𝐷
F(Φ(𝑢, 𝑣)) ⋅ N(𝑢, 𝑣) 𝑑𝑢 𝑑𝑣.
Теперь обозначим через n = n(𝑢, 𝑣) единичный вектор в направлении N(𝑢, 𝑣) (если N(𝑢, 𝑣) = 0
при некотором (𝑢, 𝑣) ∈ 𝐷, то положим n(𝑢, 𝑣) = e1 ). Тогда N = |N|n, и поэтому последний интеграл
становится равным
∫
𝐷
F(Φ(𝑢, 𝑣)) ⋅ n(𝑢, 𝑣)|N(𝑢, 𝑣)| 𝑑𝑢 𝑑𝑣.
В силу (131) мы, наконец, можем записать его в виде
(144)
∫
Φ
𝜔F =
∫
Φ
(F ⋅ n) 𝑑𝐴.
Замечание относительно смысла выражения F⋅n, сделанное в конце п. 10.48, применимо и в данной
ситуации.
Теперь мы можем перейти к классической форме теоремы Стокса.
214
10.50 Формула Стокса. Если F — векторное поле класса 𝒞 ′ в открытом множестве 𝐸 ⊂ 𝑅3 , а
Φ — 2-поверхность класса 𝒞 ″ в 𝐸, то
(145)
∫
Φ
(∇ × F) ⋅ n 𝑑𝐴 =
∫
𝜕Φ
(F ⋅ t) 𝑑𝑠.
Доказательство. Положим H = ∇ × F. Тогда, как в доказательстве теоремы 10.43, имеем
(146)
𝜔H = 𝑑𝜆F .
Отсюда
∫
Φ
(∇ × F) ⋅ n 𝑑𝐴 =
=
∫
Φ
∫
Φ
(H ⋅ n) 𝑑𝐴 =
𝑑𝜆F =
∫
𝜕Φ
∫
Φ
𝜆F =
𝜔H
∫
𝜕Φ
(F ⋅ t) 𝑑𝑠.
Здесь мы воспользовались определением векторного поля H, затем (144) с H вместо F, затем
(146), затем — главный шаг — теоремой 10.33, и, наконец, (143), очевидным способом обобщенным
с кривых на 1-цепи.
10.51 Теорема о дивергенции. Если F — векторное поле класса 𝒞 ′ в открытом множестве 𝐸 ⊂
𝑅3 , а Ω — замкнутое подмножество множества 𝐸 с положительно ориентированной границей
𝜕Ω (как описано в п. 10.31), то
(147)
∫
Ω
(∇ ⋅ F) 𝑑𝑉 =
∫
𝜕Ω
(F ⋅ n) 𝑑𝐴.
Доказательство. В силу (125) имеем
𝑑𝜔F = (∇ ⋅ F) 𝑑𝑥 ∧ 𝑑𝑦 ∧ 𝑑𝑧 = (∇ ⋅ F) 𝑑𝑉 .
Отсюда
∫
Ω
(∇ ⋅ F) 𝑑𝑉 =
∫
Ω
𝑑𝜔F =
∫
𝜕Ω
𝜔F =
∫
𝜕Ω
(F ⋅ n) 𝑑𝐴
в силу теоремы 10.33, примененной к 2-форме 𝜔F , и (144).
Упражнения
1. Пусть 𝐻 — компактное выпуклое множество в 𝑅𝑘 с непустой внутренностью. Пусть 𝑓 ∈ 𝒞 (𝐻), положим
𝑓 (x) = 0 в дополнении 𝐻 и определим ∫𝐻 𝑓 , как в определении 10.3.
Доказать, что ∫𝐻 𝑓 не зависит от порядка, в котором производятся 𝑘 интегрирований.
Указание. Приблизить 𝑓 функциями, непрерывными на 𝑅𝑘 , носители которых содержатся в 𝐻, как
было сделано в примере 10.4.
2. При 𝑖 = 1, 2, 3, … пусть 𝜑𝑖 ∈ 𝒞 (𝑅1 ) имеет носитель, содержащийся в (2−𝑖 , 21−𝑖 ), и ∫ 𝜑𝑖 = 1. Положим
∞
𝑓 (𝑥, 𝑦) =
∑
[𝜑𝑖 (𝑥) − 𝜑𝑖+1 (𝑥)]𝜑𝑖 (𝑦).
𝑖=1
Тогда 𝑓 имеет компактный носитель в 𝑅2 , непрерывна всюду, кроме точки (0, 0), и
∫
3.
𝑑𝑦
∫
но
𝑓 (𝑥, 𝑦) 𝑑𝑥 = 0,
∫
𝑑𝑥
∫
𝑓 (𝑥, 𝑦) 𝑑𝑦 = 1.
Заметить, что 𝑓 не ограничена в каждой окрестности точки (0, 0).
(a) Если F то же, что и в теореме 10.7, то положим 𝐴 = F′ (0), F1 (x) = 𝐴−1 F(x). Тогда F′1 (0) = 𝐼.
Показать, что
F1 (x) = G𝑛 ∘ G𝑛−1 ∘ ⋯ ∘ G1 (x)
в некоторой окрестности точки 0 для некоторых простых отображений G1 , … , G𝑛 . Мы получаем
другой вариант теоремы 10.7:
F(x) = F′ (0)G𝑛 ∘ G𝑛−1 ∘ ⋯ ∘ G1 (x).
215
(b) Доказать, что отображение (𝑥, 𝑦) → (𝑦, 𝑥) пространства 𝑅2 на 𝑅2 не является композицией никаких
двух простых отображений ни в какой окрестности начала (это показывает, что перевороты 𝐵𝑖 не
могут быть опущены из условия теоремы 10.7).
4. Для (𝑥, 𝑦) ∈ 𝑅2 положим
F(𝑥, 𝑦) = (𝑒𝑥 cos 𝑦 − 1, 𝑒𝑥 sin 𝑦).
Показать, что F = G2 ∘ G1 , где отображения
G1 (𝑥, 𝑦) = (𝑒𝑥 cos 𝑦 − 1, 𝑦),
G2 (𝑥, 𝑦) = (𝑢, (1 + 𝑢) tg 𝑣)
являются простыми в некоторой окрестности точки (0, 0).
Вычислить якобианы отображений G1 , G2 , F в (0, 0). Обозначим
H2 (𝑥, 𝑦) = (𝑥, 𝑒𝑥 sin 𝑦).
Найти такое
H1 (𝑢, 𝑣) = (ℎ(𝑢, 𝑣), 𝑣),
чтобы выполнялось F = H1 ∘ H2 в некоторой окрестности точки (0, 0).
5. Сформулировать и доказать аналог теоремы 10.8, в котором 𝐾 — компактное подмножество произвольного метрического пространства (заменить функции 𝜑𝑖 , участвующие в доказательстве теоремы 10.8,
функциями вида, описанного в упражнении 22 главы 4).
6. Усилить заключение теоремы 10.8, показав, что функции 𝜓𝑖 могут быть выбраны дифференцируемыми
и даже бесконечно дифференцируемыми (использовать упражнение 1 главы 8 при построении вспомогательных функций 𝜑𝑖 ).
7. (a) Показать, что симплекс 𝑄𝑘 является наименьшим выпуклым подмножеством пространства 𝑅𝑘 ,
содержащим 0, e1 , … , e𝑘 .
(b) Показать, что аффинные отображения переводят выпуклые множества в выпуклые множества.
8. Пусть 𝐻 — параллелограмм в 𝑅2 с вершинами (1, 1), (3, 2), (4, 5), (2, 4). Найти аффинное отображение
𝑇 , переводящее (0, 0) в (1, 1), (1, 0) в (3, 2), (0, 1) в (2, 4). Показать, что 𝐽𝑇 = 5. Использовать 𝑇 , чтобы
преобразовать интеграл
𝛼=
∫
𝐻
𝑒𝑥−𝑦 𝑑𝑥 𝑑𝑦
к интегралу по 𝐼 2 и таким образом вычислить 𝛼.
9. Зададим (𝑥, 𝑦) = 𝑇 (𝑟, 𝜃) на прямоугольнике
0 ≤ 𝑟 ≤ 𝑎,
0 ≤ 𝜃 ≤ 2𝜋
𝑥 = 𝑟 cos 𝜃,
𝑦 = 𝑟 sin 𝜃.
уравнениями
Показать, что 𝑇 отображает этот прямоугольник на замкнутый круг 𝐷 с центром в (0, 0) и радиусом 𝑎,
что 𝑇 взаимно однозначно во внутренности прямоугольника и что 𝐽𝑇 (𝑟, 𝜃) = 𝑟. Пусть 𝑓 ∈ 𝒞 (𝐷). Доказать
формулу интегрирования в полярных координатах:
𝑎
∫
𝐷
𝑓 (𝑥, 𝑦) 𝑑𝑥 𝑑𝑦 =
2𝜋
∫
0 ∫
0
𝑓 (𝑇 (𝑟, 𝜃))𝑟 𝑑𝑟 𝑑𝜃.
Указание. Пусть 𝐷0 — внутренность круга 𝐷 без отрезка из (0, 0) в (0, 𝑎). В текущем виде теорема 10.9
применима к непрерывным функциям 𝑓 , носитель которых лежит в 𝐷0 . Чтобы избавиться от этого
ограничения, поступить аналогично примеру 10.4.
10. Устремить 𝑎 к ∞ в упражнении 9 и доказать, что
∞
∫
𝑅2
𝑓 (𝑥, 𝑦) 𝑑𝑥 𝑑𝑦 =
∫
∫
0
0
2𝜋
𝑓 (𝑇 (𝑟, 𝜃))𝑟 𝑑𝑟 𝑑𝜃
для непрерывных функций 𝑓 , убывающих достаточно быстро при |𝑥| + |𝑦| → ∞ (найти более точную
формулировку). Применить этот вывод к функции
𝑓 (𝑥, 𝑦) = exp(−𝑥2 − 𝑦2 ),
чтобы вывести формулу (101) из главы 8.
216
11. Зададим (𝑢, 𝑣) = 𝑇 (𝑠, 𝑡) на полосе
0 < 𝑠 < ∞,
0 < 𝑡 < 1,
положив 𝑢 = 𝑠 − 𝑠𝑡, 𝑣 = 𝑠𝑡. Показать, что 𝑇 — взаимно однозначное отображение полосы на первый
квадрант 𝑄 в 𝑅2 . Показать, что 𝐽𝑇 (𝑠, 𝑡) = 𝑠.
Для 𝑥 > 0, 𝑦 > 0 проинтегрировать
𝑢𝑥−1 𝑒−𝑢 𝑣𝑦−1 𝑒−𝑣
по 𝑄, использовать теорему 10.9 для преобразования интеграла по квадранту в интеграл по полосе и
вывести таким образом формулу (96) из главы 8.
(Для этого применения теорема 10.9 должна быть обобщена на некоторые несобственные интегралы.
Провести это обобщение.)
12. Пусть 𝐼 𝑘 — множество всех u = (𝑢1 , … , 𝑢𝑘 ) ∈ 𝑅𝑘 с 0 ≤ 𝑢𝑖 ≤ 1 для всех 𝑖; пусть 𝑄𝑘 — множество всех
x = (𝑥1 , … , 𝑥𝑘 ) ∈ 𝑅𝑘 с 𝑥𝑖 ≥ 0, ∑ 𝑥𝑖 ≤ 1 (𝐼 𝑘 — единичный куб; 𝑄𝑘 — стандартный симплекс в 𝑅𝑘 ). Зададим
x = 𝑇 (u) уравнениями
𝑥1 = 𝑢 1 ,
𝑥2 = (1 − 𝑢1 )𝑢2 ,
⋯
𝑥𝑘 = (1 − 𝑢1 ) ⋯ (1 − 𝑢𝑘−1 )𝑢𝑘 .
Показать, что
𝑘
𝑘
∑
𝑥𝑖 = 1 −
𝑖=1
𝑘
(1 − 𝑢𝑖 ).
∏
𝑖=1
𝑘
Показать, что 𝑇 отображает 𝐼 на 𝑄 , что 𝑇 взаимно однозначно во внутренности куба 𝐼 𝑘 и что обратное
к нему отображение 𝑆 определено во внутренности симплекса 𝑄𝑘 как 𝑢1 = 𝑥1 и
𝑢𝑖 =
𝑥𝑖
1 − 𝑥1 − ⋯ − 𝑥𝑖−1
при 𝑖 = 2, … , 𝑘. Показать, что
𝐽𝑇 (u) = (1 − 𝑢1 )𝑘−1 (1 − 𝑢2 )𝑘−2 ⋯ (1 − 𝑢𝑘−1 )
и
𝐽𝑆 (x) = [(1 − 𝑥1 )(1 − 𝑥1 − 𝑥2 ) ⋯ (1 − 𝑥1 − ⋯ − 𝑥𝑘−1 )]−1 .
13. Пусть 𝑟1 , … , 𝑟𝑘 — неотрицательные целые числа. Доказать, что
𝑟
∫
𝑄𝑘
𝑟
𝑥11 ⋯ 𝑥𝑘𝑘 𝑑𝑥 =
𝑟1 ! ⋯ 𝑟𝑘 !
.
(𝑘 + 𝑟1 + ⋯ + 𝑟𝑘 )!
Указание. Использовать упражнение 12, теоремы 10.9 и 8.20.
Отметим, что частный случай 𝑟1 = ⋯ = 𝑟𝑘 = 0 показывает, что объем симплекса 𝑄𝑘 равен 1/𝑘!.
14. Доказать формулу (46).
15. Пусть 𝜔 и 𝜆 — соответственно 𝑘- и 𝑚-формы. Доказать, что
𝜔 ∧ 𝜆 = (−1)𝑘𝑚 𝜆 ∧ 𝜔.
16. Пусть 𝑘 ≥ 2 и 𝜎 = [p0 , p1 , … , p𝑘 ] — ориентированный аффинный 𝑘-симплекс. Доказать, что 𝜕 2 𝜎 = 0,
исходя непосредственно из определения граничного оператора 𝜕. Вывести из этого, что 𝜕 2 Ψ = 0 для
любой цепи Ψ.
Указание. Для ознакомления сделать это вначале для 𝑘 = 2, 𝑘 = 3. В общем случае при 𝑖 < 𝑗 пусть
𝜎𝑖𝑗 — (𝑘 − 2)-симплекс, полученный удалением p𝑖 и p𝑗 из 𝜎. Показать, что каждый 𝜎𝑖𝑗 встречается в 𝜕 2 𝜎
дважды, с противоположными знаками.
17. Положим 𝐽 2 = 𝜏1 + 𝜏2 , где
𝜏1 = [0, e1 , e1 + e2 ],
𝜏2 = −[0, e2 , e2 + e1 ].
Объяснить, почему можно назвать 𝐽 2 положительно ориентированным единичным квадратом в 𝑅2 .
Показать, что 𝜕𝐽 2 является суммой 4 ориентированных аффинных 1-симплексов. Найти их. Чему равно
𝜕(𝜏1 − 𝜏2 )?
18. Рассмотрим ориентированный аффинный 3-симплекс
𝜎1 = [0, e1 , e1 + e2 , e1 + e2 + e3 ]
217
в 𝑅3 . Показать, что 𝜎1 (рассматриваемый как линейное преобразование) имеет определитель 1. Таким
образом, 𝜎1 положительно ориентирован.
Пусть 𝜎2 , … , 𝜎6 — пять других ориентированных 3-симплексов, полученных следующим образом. Существует пять перестановок (𝑖1 , 𝑖2 , 𝑖3 ) чисел (1, 2, 3), отличных от (1, 2, 3). Свяжем с каждой (𝑖1 , 𝑖2 , 𝑖3 )
симплекс
𝑠(𝑖1 , 𝑖2 , 𝑖3 )[0, e𝑖1 , e𝑖1 + e𝑖2 , e𝑖1 + e𝑖2 + e𝑖3 ],
где 𝑠 — знак, входящий в определение определителя (так 𝜏2 был получен из 𝜏1 в упражнении 17).
Показать, что 𝜎2 , … , 𝜎6 положительно ориентированы.
Положим 𝐽 3 = 𝜎1 + ⋯ + 𝜎6 . Тогда 𝐽 3 можно назвать положительно ориентированным единичным кубом
в 𝑅3 .
Показать, что 𝜕𝐽 3 является суммой 12 ориентированных аффинных 2-симплексов (эти 12 треугольников
покрывают поверхность единичного куба 𝐼 3 ).
Показать, что x = (𝑥1 , 𝑥2 , 𝑥3 ) лежит во множестве значений симплекса 𝜎1 , если и только если 0 ≤ 𝑥3 ≤
𝑥2 ≤ 𝑥1 ≤ 1.
Показать, что области значений симплексов 𝜎1 , … , 𝜎6 имеют непересекающиеся внутренности и что их
объединение покрывает 𝐼 3 (ср. с упражнением 13; отметим, что 3! = 6).
19. Пусть 𝐽 2 и 𝐽 3 — те же, что и в упражнениях 17 и 18. Определим отображения
𝐵01 (𝑢, 𝑣) = (0, 𝑢, 𝑣),
𝐵11 (𝑢, 𝑣) = (1, 𝑢, 𝑣),
𝐵02 (𝑢, 𝑣) = (𝑢, 0, 𝑣),
𝐵12 (𝑢, 𝑣) = (𝑢, 1, 𝑣),
𝐵03 (𝑢, 𝑣) = (𝑢, 𝑣, 0),
𝐵13 (𝑢, 𝑣) = (𝑢, 𝑣, 1).
Они аффинны и отображают 𝑅2 в 𝑅3 .
Положим 𝛽𝑟𝑖 = 𝐵𝑟𝑖 (𝐽 2 ) при 𝑟 = 0, 1, 𝑖 = 1, 2, 3. Каждое 𝛽𝑟𝑖 — аффинная ориентированная 2-цепь (см. п.
10.30). Проверить, что
3
𝜕𝐽 3 =
∑
(−1)𝑖 (𝛽0𝑖 − 𝛽1𝑖 )
𝑖=1
в соответствии с упражнением 18.
20. Сформулировать условия, при которых формула
∫
Φ
𝑓 𝑑𝜔 =
∫
𝜕Φ
𝑓𝜔 −
∫
Φ
(𝑑𝑓 ) ∧ 𝜔
верна, и показать, что она обобщает формулу интегрирования по частям.
Указание: 𝑑(𝑓 𝜔) = (𝑑𝑓 ) ∧ 𝜔 + 𝑓 𝑑𝜔.
21. Как в примере 10.36, рассмотрим 1-форму
𝜂=
𝑥 𝑑𝑦 − 𝑦 𝑑𝑥
𝑥2 + 𝑦2
в 𝑅2 − {0}.
(a) Выполнить вычисления, приводящие к формуле (113), и доказать. что 𝑑𝜂 = 0.
(b) Пусть 𝛾(𝑡) = (𝑟 cos 𝑡, 𝑟 sin 𝑡) при некотором 𝑟 > 0, и пусть Γ — 𝒞 ″ -кривая в 𝑅2 − {0} с сегментом
параметров [0, 2𝜋] с Γ(0) = Γ(2𝜋), такая, что отрезки [𝛾(𝑡), Γ(𝑡)] не содержат 0 ни при каком 𝑡 ∈ [0, 2𝜋].
Доказать, что
∫
Γ
𝜂 = 2𝜋.
Указание. При 0 ≤ 𝑡 ≤ 2𝜋, 0 ≤ 𝑢 ≤ 1 положим
Φ(𝑡, 𝑢) = (1 − 𝑢)Γ(𝑡) + 𝑢𝛾(𝑡).
Тогда Φ — 2-поверхность в 𝑅2 − {0}, множеством параметров которой является указанный прямоугольник. Благодаря взаимному уничтожению (как в примере 10.32),
𝜕Φ = Γ − 𝛾.
Использовать теорему Стокса, чтобы заключить, что
∫
Γ
так как 𝑑𝜂 = 0.
218
𝜂=
∫
𝛾
𝜂,
(c) Возьмем Γ(𝑡) = (𝑎 cos 𝑡, 𝑏 sin 𝑡), где 𝑎 > 0, 𝑏 > 0 фиксированы. Использовать часть (b), чтобы показать, что
2𝜋
𝑎𝑏
𝑑𝑡 = 2𝜋.
∫
𝑎2 cos2 𝑡 + 𝑏2 sin2 𝑡
0
(d) Показать, что
𝑦
𝜂 = 𝑑(arctg )
𝑥
в любом открытом множестве, в котором 𝑥 ≠ 0, и что
𝑥
𝜂 = 𝑑(− arctg )
𝑦
в любом открытом множестве, в котором 𝑦 ≠ 0.
Объяснить, почему это обосновывает обозначение 𝜂 = 𝑑𝜃, несмотря на то, что 𝜂 не является точной
в 𝑅2 − {0}.
(e) Показать, что (b) может быть получено из (d).
(f) Пусть Γ — любая замкнутая 𝒞 ′ -кривая в 𝑅2 − {0}. Доказать, что
1
𝜂 = Ind(Γ)
2𝜋 ∫
Γ
(определение индекса кривой см. в упражнении 23 из главы 8).
22. Как в примере 10.37, определим 𝜁 в 𝑅3 − {0} равенством
𝜁=
𝑥 𝑑𝑦 ∧ 𝑑𝑧 + 𝑦 𝑑𝑧 ∧ 𝑑𝑥 + 𝑧 𝑑𝑥 ∧ 𝑑𝑦
,
𝑟3
где 𝑟 = (𝑥2 + 𝑦2 + 𝑧2 )1/2 . Пусть 𝐷 — прямоугольник, заданный неравенствами 0 ≤ 𝑢 ≤ 𝜋, 0 ≤ 𝑣 ≤ 2𝜋, и
пусть Σ — 2-поверхность в 𝑅3 с множеством параметров 𝐷, заданная уравнениями
𝑥 = sin 𝑢 cos 𝑣,
𝑦 = sin 𝑢 sin 𝑣,
𝑧 = cos 𝑢.
(a) Доказать, что 𝑑𝜁 = 0 в 𝑅3 − {0}.
(b) Обозначим через 𝑆 сужение поверхности Σ на множество параметров 𝐸 ⊂ 𝐷. Доказать, что
∫
𝑆
𝜁=
∫
𝐸
sin 𝑢 𝑑𝑢 𝑑𝑣 = 𝐴(𝑆),
где 𝐴 обозначает площадь, как в п. 10.43. Отметим, что это содержит (115) как частный случай.
(c) Пусть 𝑔, ℎ1 , ℎ2 , ℎ3 — 𝒞 ″ -функции на [0, 1], 𝑔 > 0. Пусть (𝑥, 𝑦, 𝑧) = Φ(𝑠, 𝑡) задает 2-поверхность Φ с
множеством параметров 𝐼 2 уравнениями
𝑥 = 𝑔(𝑡)ℎ1 (𝑠),
𝑦 = 𝑔(𝑡)ℎ2 (𝑠),
𝑧 = 𝑔(𝑡)ℎ3 (𝑠).
Доказать, что
∫
Φ
𝜁 = 0,
непосредственно из (35).
Обратим внимание на форму множества значений поверхности Φ: при фиксированном 𝑠 точка
Φ(𝑠, 𝑡) пробегает отрезок на прямой, проходящей через 0. Множество значений поверхности Φ,
таким образом, лежит на «конусе» с вершиной в начале.
(d) Пусть 𝐸 — замкнутый прямоугольник в 𝐷 со сторонами, параллельными сторонам прямоугольника
𝐷. Пусть 𝑓 ∈ 𝒞 ″ (𝐷), 𝑓 > 0. Пусть Ω — 2-поверхность с множеством параметров 𝐸, заданная
уравнением
Ω(𝑢, 𝑣) = 𝑓 (𝑢, 𝑣)Σ(𝑢, 𝑣).
Обозначим через 𝑆 то же, что в (b). Доказать, что
∫
Ω
𝜁=
∫
𝑆
𝜁 = 𝐴(𝑆)
(ввиду того что 𝑆 — «радиальная проекция» поверхности Ω на единичную сферу, этот результат позволяет называть ∫Ω 𝜁 «телесным углом» с вершиной в начале, стягиваемым множеством
значений поверхности Ω).
Указание. Рассмотреть 3-поверхность Ψ, заданную уравнением
Ψ(𝑡, 𝑢, 𝑣) = [1 − 𝑡 + 𝑡𝑓 (𝑢, 𝑣)]Σ(𝑢, 𝑣),
219
где (𝑢, 𝑣) ∈ 𝐸, 0 ≤ 𝑡 ≤ 1. При фиксированном 𝑣 отображение (𝑡, 𝑢) → Ψ(𝑡, 𝑢, 𝑣) является 2-поверхностью
Φ, к которой можно применить (c), чтобы показать, что ∫Φ 𝜁 = 0. То же верно при фиксированном
𝑢. В силу (a) и теоремы Стокса имеем
∫
𝜕Ψ
𝜁=
∫
Ψ
𝑑𝜁 = 0.
(e) Положим 𝜆 = −(𝑧/𝑟)𝜂, где
𝜂=
𝑥 𝑑𝑦 − 𝑦 𝑑𝑥
,
𝑥2 + 𝑦2
как в упражнении 21. Тогда 𝜆 — 1-форма в открытом множестве 𝑉 ⊂ 𝑅3 , в котором 𝑥2 + 𝑦2 > 0.
Показать, что 𝜁 точна в 𝑉 , показав, что
𝜁 = 𝑑𝜆.
(f) Вывести (d) из (e), не используя (c).
Указание. Для начала пусть 0 < 𝑢 < 𝜋 на 𝐸. В силу (e) имеем
∫
Ω
𝜁=
∫
𝜕Ω
и
𝜆
∫
𝑆
𝜁=
∫
𝜕𝑆
𝜆.
Показать, что интегралы от 𝜆 равны, используя часть (d) упражнения 21 и заметив, что 𝑧/𝑟 одно
и то же как в Σ(𝑢, 𝑣), так и в Ω(𝑢, 𝑣).
(g) Точна ли 𝜁 в дополнении каждой прямой, проходящей через начало?
23. Зафиксируем 𝑛. Обозначим 𝑟𝑘 = (𝑥21 + ⋯ + 𝑥2𝑘 )1/2 при 1 ≤ 𝑘 ≤ 𝑛. Пусть 𝐸𝑘 — множество всех x ∈ 𝑅𝑛 , при
которых 𝑟𝑘 > 0, и пусть 𝜔𝑘 — (𝑘 − 1)-форма, определенная в 𝐸𝑘 равенством
𝑘
𝜔𝑘 = (𝑟𝑘 )−𝑘
(−1)𝑖−1 𝑥𝑖 𝑑𝑥1 ∧ ⋯ ∧ 𝑑𝑥𝑖−1 ∧ 𝑑𝑥𝑖+1 ∧ ⋯ ∧ 𝑑𝑥𝑘 .
∑
𝑖=1
Отметим, что 𝜔2 = 𝜂, 𝜔3 = 𝜁 в обозначениях упражнений 21 и 22. Отметим также, что
𝐸1 ⊂ 𝐸2 ⊂ ⋯ ⊂ 𝐸𝑛 = 𝑅𝑛 − {0}.
(a) Доказать, что 𝑑𝜔𝑘 = 0 в 𝐸𝑘 .
(b) При 𝑘 = 2, … , 𝑛 доказать, что 𝜔𝑘 точна в 𝐸𝑘−1 , показав, что
𝜔𝑘 = 𝑑(𝑓𝑘 𝜔𝑘−1 ) = (𝑑𝑓𝑘 ) ∧ 𝜔𝑘−1 ,
где 𝑓𝑘 (x) = (−1)𝑘 𝑔𝑘 (𝑥𝑘 /𝑟𝑘 ) и
𝑡
𝑔𝑘 (𝑡) =
∫
−1
(1 − 𝑠2 )(𝑘−3)/2 𝑑𝑠
(−1 < 𝑡 < 1).
Указание: 𝑓𝑘 удовлетворяет дифференциальным уравнениям
x ⋅ (∇𝑓𝑘 )(x) = 0
и
(𝐷𝑘 𝑓𝑘 )(x) =
(−1)𝑘 (𝑟𝑘−1 )𝑘−1
.
(𝑟𝑘 )𝑘
(c) Точна ли 𝜔𝑛 в 𝐸𝑛 ?
(d) Отметим, что (b) является обобщением части (e) упражнения 22. Попытаться обобщить некоторые
другие утверждения упражнений 21 и 22 на 𝜔𝑛 при произвольном 𝑛.
24. Пусть 𝜔 = ∑ 𝑎𝑖 (x) 𝑑𝑥𝑖 — 1-форма класса 𝒞 ″ в выпуклом открытом множестве 𝐸 ⊂ 𝑅𝑛 . Пусть 𝑑𝜔 = 0.
Доказать, что 𝜔 точна в 𝐸, пользуясь следующим планом.
Зафиксируем p ∈ 𝐸. Положим
𝑓 (x) =
∫
[p,x]
𝜔
(x ∈ 𝐸).
Применить теорему Стокса к аффинному ориентированному 2-симплексу [p, x, y] в 𝐸. Вывести, что
𝑛
𝑓 (y) − 𝑓 (x) =
∑
1
(𝑦𝑖 − 𝑥𝑖 )
𝑖=1
при x ∈ 𝐸, y ∈ 𝐸. Значит, (𝐷𝑖 𝑓 )(x) = 𝑎𝑖 (x).
220
∫
0
𝑎𝑖 ((1 − 𝑡)x + 𝑡y) 𝑑𝑡
25. Пусть 𝜔 — 1-форма в открытом множестве 𝐸 ⊂ 𝑅𝑛 , такая, что
∫
𝛾
𝜔=0
для любой замкнутой кривой 𝛾 в 𝐸 класса 𝒞 ′ . Доказать, что 𝜔 точна в 𝐸, частично следуя методу,
кратко описанному в упражнении 24.
26. Пусть 𝜔 — 1-форма в 𝑅3 − {0} класса 𝒞 ′ и 𝑑𝜔 = 0. Доказать, что 𝜔 точна в 𝑅3 − {0}.
Указание. Каждая замкнутая непрерывно дифференцируемая кривая в 𝑅3 − {0} является границей
некоторой 2-поверхности в 𝑅3 − {0}. Применить теорему Стокса и упражнение 25.
27. Пусть 𝐸 — открытая 3-клетка в 𝑅3 с ребрами, параллельными осям координат. Пусть (𝑎, 𝑏, 𝑐) ∈ 𝐸,
𝑓𝑖 ∈ 𝒞 ′ (𝐸) при 𝑖 = 1, 2, 3,
𝜔 = 𝑓1 𝑑𝑦 ∧ 𝑑𝑧 + 𝑓2 𝑑𝑧 ∧ 𝑑𝑥 + 𝑓3 𝑑𝑥 ∧ 𝑑𝑦
и пусть 𝑑𝜔 = 0 в 𝐸. Положим
𝜆 = 𝑔1 𝑑𝑥 + 𝑔2 𝑑𝑦,
где
𝑧
𝑔1 (𝑥, 𝑦, 𝑧) =
∫
𝑐
𝑦
𝑓2 (𝑥, 𝑦, 𝑠) 𝑑𝑠 −
∫
𝑏
𝑓3 (𝑥, 𝑡, 𝑐) 𝑑𝑡,
𝑧
𝑔2 (𝑥, 𝑦, 𝑧) = −
∫
𝑐
𝑓1 (𝑥, 𝑦, 𝑠) 𝑑𝑠
при (𝑥, 𝑦, 𝑧) ∈ 𝐸. Доказать, что 𝑑𝜆 = 𝜔 в 𝐸.
Вычислить эти интегралы при 𝜔 = 𝜁 и таким образом найти форму 𝜆, входящую в часть (e) упражнения
22.
28. Зафиксируем 𝑏 > 𝑎 > 0, положим
Φ(𝑟, 𝜃) = (𝑟 cos 𝜃, 𝑟 sin 𝜃)
при 𝑎 ≤ 𝑟 ≤ 𝑏, 0 ≤ 𝜃 ≤ 2𝜋 (множеством значений поверхности Φ является кольцо в 𝑅2 ). Положим
𝜔 = 𝑥3 𝑑𝑦. Вычислить интегралы
∫
Φ
и
𝑑𝜔
∫
𝜕Φ
𝜔,
чтобы убедиться в том, что они равны.
29. Доказать существование функции 𝛼 со свойствами, необходимыми для доказательства теоремы 10.38,
и доказать, что получающаяся функция 𝐹 — класса 𝒞 ′ (оба утверждения становятся тривиальными,
если 𝐸 — открытая клетка или открытый шар, так как 𝛼 тогда можно взять постоянной; сослаться на
теорему 9.42).
30. Пусть N — вектор, полученный в (135). Доказать, что
⎡𝛼1
det ⎢𝛼2
⎢
⎣𝛼3
𝛽1
𝛽2
𝛽3
𝛼2 𝛽3 − 𝛼3 𝛽2 ⎤
𝛼3 𝛽1 − 𝛼1 𝛽3 ⎥ = |N|2 .
⎥
𝛼1 𝛽2 − 𝛼2 𝛽1 ⎦
Также проверить уравнение (137).
31. Пусть 𝐸 ⊂ 𝑅3 открыто, пусть 𝑔 ∈ 𝒞 ″ (𝐸), ℎ ∈ 𝒞 ″ (𝐸). Рассмотрим векторное поле
F = 𝑔∇ℎ.
(a) Доказать, что
∇ ⋅ F = 𝑔∇2 ℎ + (∇𝑔) ⋅ (∇ℎ),
2
2
ℎ/𝜕𝑥2𝑖
где ∇ ℎ = ∇ ⋅ (∇ℎ) = ∑ 𝜕
— так называемый «лапласиан» функции ℎ.
(b) Пусть Ω — замкнутое подмножество множества 𝐸 с положительно ориентированной границей 𝜕Ω
(как в теореме 10.51). Доказать, что
∫
Ω
[𝑔∇2 ℎ + (∇𝑔) ⋅ (∇ℎ)] 𝑑𝑉 =
𝜕ℎ
𝑑𝐴,
𝑔
∫
𝜕Ω 𝜕𝑛
где (для удобства) мы написали 𝜕ℎ/𝜕𝑛 вместо (∇ℎ)⋅n (таким образом, 𝜕ℎ/𝜕𝑛 — производная функции
ℎ по направлению внешней нормали к 𝜕Ω, так называемая нормальная производная функции ℎ).
Поменять местами 𝑔 и ℎ, вычесть получившуюся формулу из предыдущей, чтобы получить
∫
Ω
(𝑔∇2 ℎ − ℎ∇2 𝑔) 𝑑𝑉 =
𝜕𝑔
𝜕ℎ
𝑔
−ℎ
𝑑𝐴.
(
∫
𝜕𝑛
𝜕𝑛 )
𝜕Ω
Эти две формулы обычно называют тождествами Грина.
221
(c) Пусть ℎ является гармонической в 𝐸; это означает, что ∇2 ℎ = 0. Взять 𝑔 = 1 и прийти к выводу,
что
𝜕ℎ
𝑑𝐴 = 0.
∫
𝜕Ω 𝜕𝑛
Взять 𝑔 = ℎ и прийти к выводу, что ℎ = 0 в Ω, если ℎ = 0 на 𝜕Ω.
(d) Показать, что тождества Грина также верны в 𝑅2 .
32. Зафиксируем 𝛿, 0 < 𝛿 < 1. Пусть 𝐷 — множество всех (𝜃, 𝑡) ∈ 𝑅2 , таких, что 0 ≤ 𝜃 ≤ 𝜋, −𝛿 ≤ 𝑡 ≤ 𝛿. Пусть
Φ — 2-поверхность в 𝑅3 с множеством параметров 𝐷, заданная уравнениями
𝑥 = (1 − 𝑡 sin 𝜃) cos 2𝜃,
𝑦 = (1 − 𝑡 sin 𝜃) sin 2𝜃,
𝑧 = 𝑡 cos 𝜃,
где (𝑥, 𝑦, 𝑧) = Φ(𝜃, 𝑡). Отметим, что Φ(𝜋, 𝑡) = Φ(0, −𝑡), и что Φ взаимно однозначна на остальной части
множества 𝐷.
Множество значений 𝑀 = Φ(𝐷) поверхности Φ известно как лента Мебиуса. Это простейший пример
неориентируемой поверхности.
Доказать утверждения в следующем описании. Положим p1 = (0, −𝛿), p2 = (𝜋, −𝛿), p3 = (𝜋, 𝛿), p4 = (0, 𝛿),
p5 = p1 . Положим 𝛾𝑖 = [p𝑖 , p𝑖+1 ], 𝑖 = 1, … , 4, и положим Γ𝑖 = Φ ∘ 𝛾𝑖 . Тогда
𝜕Φ = Γ1 + Γ2 + Γ3 + Γ4 .
Положим a = (1, 0, −𝛿), b = (1, 0, 𝛿). Тогда
Φ(p1 ) = Φ(p3 ) = a,
Φ(p2 ) = Φ(p4 ) = b,
а 𝜕Φ может быть описана следующим образом.
Γ1 поднимается по спирали от a к b; ее проекция на плоскость (𝑥, 𝑦) имеет число зацепления +1 относительно начала (см. упражнение 23, гл. 8).
Γ2 = [b, a].
Γ3 поднимается по спирали от a к b; ее проекция на плоскость (𝑥, 𝑦) имеет число зацепления −1 относительно начала.
Γ4 = [b, a].
Таким образом, 𝜕Φ = Γ1 + Γ3 + 2Γ2 .
Если мы пойдем из a в b вдоль Γ1 и продолжим по «ребру» множества 𝑀, пока не вернемся в a,
пройденная кривая будет равна
Γ = Γ1 − Γ3 ,
она может также быть представлена на сегменте параметра [0, 2𝜋] уравнениями
𝑥 = (1 + 𝛿 sin 𝜃) cos 2𝜃,
𝑦 = (1 + 𝛿 sin 𝜃) sin 2𝜃,
𝑧 = −𝛿 cos 𝜃.
Стоит подчеркнуть, что Γ ≠ 𝜕Φ. Пусть 𝜂 — 1-форма, обсуждаемая в упражнениях 21 и 22. Ввиду того
что 𝑑𝜂 = 0, теорема Стокса показывает, что
∫
𝜕Φ
𝜂 = 0.
Но, хотя Γ — «геометрическая» граница множества 𝑀, мы имеем
∫
Γ
𝜂 = 4𝜋.
Чтобы избежать этой возможной путаницы, формулу Стокса (теорема 10.50) часто формулируют только
для ориентируемых поверхностей Φ.
222
Глава 11
Теория Лебега
Цель этой главы — изложить основные понятия лебеговой теории меры и интеграла и в довольно общем виде доказать некоторые важнейшие теоремы, не затемняя основных идей множеством
сравнительно тривиальных деталей. Поэтому в некоторых случаях доказательства лишь намечаются, а некоторые более легкие предложения формулируются без доказательства. Однако читателю,
овладевшему техникой, применявшейся в предыдущих главах, нетрудно будет восстановить пропущенные этапы рассуждений.
Теорию интеграла Лебега можно излагать несколькими различными способами. Из них здесь
будет обсуждаться лишь один. Что касается других возможных способов, то за ними мы отсылаем
читателя к специальным курсам по теории интеграла, перечисленным в библиографии.
Функции множеств
Пусть 𝐴 и 𝐵 — какие-нибудь множества. Символом 𝐴 − 𝐵 мы будем обозначать множество всех
элементов 𝑥, таких, что 𝑥 ∈ 𝐴, 𝑥 ∉ 𝐵. Обозначение 𝐴 − 𝐵 применяется не только тогда, когда
𝐵 ⊂ 𝐴. Пустое множество будет обозначаться символом 0, и мы будем говорить, что множества 𝐴
и 𝐵 не пересекаются, если 𝐴 ∩ 𝐵 = 0.
11.1 Определение. Семейство множеств ℛ называется кольцом, если из того, что 𝐴 ∈ ℛ и 𝐵 ∈ ℛ,
следует, что
(1)
𝐴 ∪ 𝐵 ∈ ℛ,
𝐴 − 𝐵 ∈ ℛ.
Ввиду того что 𝐴 ∩ 𝐵 = 𝐴 − (𝐴 − 𝐵), верно также, что 𝐴 ∩ 𝐵 ∈ ℛ, если ℛ — кольцо.
Кольцо ℛ называется 𝜎-кольцом, если
∞
(2)
⋃
𝐴𝑛 ∈ ℛ,
𝑛=1
каковы бы ни были множества 𝐴𝑛 ∈ ℛ (𝑛 = 1, 2, 3, … ). Поскольку
∞
⋂
∞
𝐴𝑛 = 𝐴 1 −
𝑛=1
то мы имеем также
⋃
(𝐴1 − 𝐴𝑛 ),
𝑛=1
∞
⋂
𝐴𝑛 ∈ ℛ,
𝑛=1
если ℛ есть 𝜎-кольцо.
11.2 Определение. Мы будем говорить, что 𝜑 — функция множества, определенная на ℛ, если
𝜑 каждому множеству 𝐴 ∈ ℛ сопоставляет число 𝜑(𝐴), принадлежащее расширенной системе
вещественных чисел. Функция 𝜑 называется аддитивной, если из того, что 𝐴 ∩ 𝐵 = 0, следует
(3)
𝜑(𝐴 ∪ 𝐵) = 𝜑(𝐴) + 𝜑(𝐵),
223
и 𝜑 называется счетно-аддитивной, если из того, что 𝐴𝑖 ∩ 𝐴𝑗 = 0 (𝑖 ≠ 𝑗), следует
∞
(4)
𝜑
∞
(⋃
𝑛=1
𝐴𝑛
)
=
∑
𝜑(𝐴𝑛 ).
𝑛=1
Мы всегда будем предполагать, что не более чем одно из чисел +∞ и −∞ принадлежит множеству значений функции 𝜑; если бы это было не так, то правая часть равенства (3) могла бы
не иметь смысла. Кроме того, мы исключим из рассмотрения функции множества, единственным
значением которых служит +∞ или −∞.
Интересно отметить, что левая часть равенства (4) не зависит от порядка, в котором расположены множества 𝐴𝑛 . Значит, из теоремы о перестановках ряда следует, что ряд в правой части
равенства (4) сходится абсолютно, если вообще сходится; если же он расходится, то его частные
суммы стремятся к +∞ или к −∞.
Если функция 𝜑 аддитивна, то, как легко видеть, она обладает следующими свойствами.
(5)
𝜑(0) = 0.
(6)
𝜑(𝐴1 ∪ ⋯ ∪ 𝐴𝑛 ) = 𝜑(𝐴1 ) + ⋯ + 𝜑(𝐴𝑛 ),
если 𝐴𝑖 ∩ 𝐴𝑗 = 0 при 𝑖 ≠ 𝑗.
(7)
𝜑(𝐴1 ∪ 𝐴2 ) + 𝜑(𝐴1 ∩ 𝐴2 ) = 𝜑(𝐴1 ) + 𝜑(𝐴2 ).
Если 𝜑(𝐴) ≥ 0 при всех 𝐴 и 𝐴1 ⊂ 𝐴2 , то
(8)
𝜑(𝐴1 ) ≤ 𝜑(𝐴2 ).
Имея в виду неравенство (8), неотрицательные функции множества часто называют монотонными.
(9)
𝜑(𝐴 − 𝐵) = 𝜑(𝐴) − 𝜑(𝐵),
если 𝐵 ⊂ 𝐴 и |𝜑(𝐵)| < +∞.
11.3 Теорема. Пусть 𝜑 — счетно-аддитивная функция множества, определенная на кольце ℛ.
Пусть 𝐴𝑛 ∈ ℛ (𝑛 = 1, 2, 3, … ), 𝐴1 ⊂ 𝐴2 ⊂ 𝐴3 ⊂ ⋯ , 𝐴 ∈ ℛ и
∞
𝐴=
⋃
𝐴𝑛 .
𝑛=1
Тогда при 𝑛 → ∞
𝜑(𝐴𝑛 ) → 𝜑(𝐴).
Доказательство. Пусть 𝐵1 = 𝐴1 и
𝐵𝑛 = 𝐴𝑛 − 𝐴𝑛−1
(𝑛 = 2, 3, … ).
Тогда 𝐵𝑖 ∩ 𝐵𝑗 = 0 при 𝑖 ≠ 𝑗, 𝐴𝑛 = 𝐵1 ∪ ⋯ ∪ 𝐵𝑛 и 𝐴 = ⋃ 𝐵𝑛 . Значит,
𝑛
𝜑(𝐴𝑛 ) =
∑
𝜑(𝐵𝑖 )
𝑖=1
и
∞
𝜑(𝐴) =
∑
𝑖=1
224
𝜑(𝐵𝑖 ).
Построение меры Лебега
11.4 Определение. Пусть 𝑅𝑝 обозначает 𝑝-мерное евклидово пространство. Прямоугольником в
𝑅𝑝 мы называем множество всех точек x = (𝑥1 , … , 𝑥𝑝 ), таких, что
(10)
𝑎𝑖 ≤ 𝑥𝑖 ≤ 𝑏𝑖
(𝑖 = 1, … , 𝑝),
или же множество точек, определяемое неравенствами (10), в которых некоторые (или все) знаки
≤ заменены на <. Возможность равенства 𝑎𝑖 = 𝑏𝑖 при каком-нибудь значении 𝑖 не исключается; в
частности, пустое множество — тоже прямоугольник.
Если 𝐴 — объединение конечного числа прямоугольников, то говорят, что 𝐴 — элементарное
множество.
Пусть 𝐼 — прямоугольник; положим по определению
𝑝
𝑚(𝐼) =
(𝑏 − 𝑎𝑖 ),
∏ 𝑖
𝑖=1
вне зависимости от того, включается или нет знак равенства в неравенства (10).
Если 𝐴 = 𝐼1 ∪ ⋯ ∪ 𝐼𝑛 и эти прямоугольники попарно не пересекаются, то мы полагаем
(11)
𝑚(𝐴) = 𝑚(𝐼1 ) + ⋯ + 𝑚(𝐼𝑛 ).
Обозначим буквой ℰ семейство всех элементарных подмножеств пространства 𝑅𝑝 .
Теперь следует убедиться в том, что
(12) ℰ — кольцо, но не 𝜎-кольцо;
(13) если 𝐴 ∈ ℰ , то 𝐴 представимо в виде объединения конечного числа непересекающихся прямоугольников;
(14) если 𝐴 ∈ ℰ , то 𝑚(𝐴) корректно определено равенством (11); это значит, что, исходя из двух
различных представлений множества 𝐴 в виде объединения непересекающихся прямоугольников, мы получим одно и то же значение 𝑚(𝐴);
(15) функция 𝑚 аддитивна на ℰ .
Заметим, что если 𝑝 = 1, 2, 3, то 𝑚 — это соответственно длина, площадь и объем.
11.5 Определение. Неотрицательная аддитивная функция множества 𝜑, определенная на ℰ ,
называется регулярной, если верно следующее: для любого 𝐴 ∈ ℰ и любого числа 𝜀 > 0 существуют
множества 𝐹 ∈ ℰ , 𝐺 ∈ ℰ , такие, что 𝐹 замкнуто, 𝐺 открыто, 𝐹 ⊂ 𝐴 ⊂ 𝐺 и
(16)
𝜑(𝐺) − 𝜀 ≤ 𝜑(𝐴) ≤ 𝜑(𝐹 ) + 𝜀.
11.6 Примеры.
(a) Функция множества 𝑚 регулярна.
Если 𝐴 — прямоугольник, то требование определения 11.5 выполняется тривиальным образом. Общий случай следует из (13).
(b) Положим 𝑅𝑝 = 𝑅1 , и пусть 𝛼 — монотонно возрастающая функция, определенная для всех
вещественных 𝑥. Положим
𝜇([𝑎, 𝑏)) = 𝛼(𝑏−) − 𝛼(𝑎−),
𝜇([𝑎, 𝑏]) = 𝛼(𝑏+) − 𝛼(𝑎−),
𝜇((𝑎, 𝑏]) = 𝛼(𝑏+) − 𝛼(𝑎+),
𝜇((𝑎, 𝑏)) = 𝛼(𝑏−) − 𝛼(𝑎+).
Здесь [𝑎, 𝑏) — множество 𝑎 ≤ 𝑥 < 𝑏, и т. д. Эти случаи следует различать из-за возможных
разрывов функции 𝛼. Если 𝜇 определить на элементарных множествах, как в (11), то функция
𝜇 оказывается регулярной на ℰ . Доказательство точно такое же, как в (a).
Наша следующая цель состоит в доказательстве того, что каждая функция множества, регулярная на ℰ , может быть продолжена до счетно-аддитивной функции множества, определенной
на 𝜎-кольце, содержащем ℰ .
225
11.7 Определение. Пусть функция 𝜇 аддитивна, регулярна, неотрицательна и конечна на ℰ .
Рассмотрим счетные покрытия какого-нибудь множества 𝐸 ⊂ 𝑅𝑝 открытыми элементарными множествами 𝐴𝑛 :
∞
𝐸⊂
⋃
𝐴𝑛 .
𝑛=1
Положим, по определению,
∞
𝜇∗ (𝐸) = inf
(17)
∑
𝜇(𝐴𝑛 ),
𝑛=1
где нижняя грань берется по всем счетным покрытиям множества 𝐸 открытыми элементарными
множествами. Число 𝜇∗ (𝐸) называется внешней мерой множества 𝐸, соответствующей функции 𝜇.
Ясно, что 𝜇∗ (𝐸) ≥ 0 при всех 𝐸 и что
𝜇∗ (𝐸1 ) ≤ 𝜇 ∗ (𝐸2 ),
(18)
если 𝐸1 ⊂ 𝐸2 .
11.8 Теорема.
(a) Для любого 𝐴 ∈ ℰ имеем 𝜇∗ (𝐴) = 𝜇(𝐴).
(b) Если 𝐸 = ⋃∞
1 𝐸𝑛 , то
∞
𝜇 ∗ (𝐸) ≤
(19)
∑
𝜇∗ (𝐸𝑛 ).
𝑛=1
Отметим, что (a) означает, что функция 𝜇∗ — продолжение функции 𝜇 с кольца ℰ на семейство
всех подмножеств пространства 𝑅𝑝 . Свойство (19) называется субаддитивностью.
Доказательство. Пусть 𝐴 ∈ ℰ и 𝜀 > 0.
Из регулярности меры 𝜇 следует, что множество 𝐴 содержится в некотором открытом элементарном множестве 𝐺, таком, что 𝜇(𝐺) ≤ 𝜇(𝐴) + 𝜀. Поскольку 𝜇∗ (𝐴) ≤ 𝜇(𝐺) и поскольку число 𝜀
произвольно, то
𝜇∗ (𝐴) ≤ 𝜇(𝐴).
(20)
По определению 𝜇 ∗ существует такая последовательность {𝐴𝑛 } открытых элементарных множеств, объединение которых содержит множество 𝐴, что
∞
∑
𝜇(𝐴𝑛 ) ≤ 𝜇 ∗ (𝐴) + 𝜀.
𝑛=1
Регулярность функции 𝜇 показывает, что множество 𝐴 содержит замкнутое элементарное множество 𝐹 , такое, что 𝜇(𝐹 ) ≥ 𝜇(𝐴) − 𝜀, а так как множество 𝐹 компактно, то
𝐹 ⊂ 𝐴1 ∪ ⋯ ∪ 𝐴 𝑁
при некотором 𝑁. Значит,
𝑁
𝜇(𝐴) ≤ 𝜇(𝐹 ) + 𝜀 ≤ 𝜇(𝐴1 ∪ ⋯ ∪ 𝐴𝑛 ) + 𝜀 ≤
∑
𝜇(𝐴𝑛 ) + 𝜀 ≤ 𝜇 ∗ (𝐴) + 2𝜀.
1
Сопоставляя это неравенство с неравенством (20), получаем (a).
Пусть теперь 𝐸 = ⋃ 𝐸𝑛 , и пусть 𝜇∗ (𝐸𝑛 ) < +∞ при всех 𝑛. Данному числу 𝜀 > 0 отвечают
покрытия {𝐴𝑛𝑘 }, 𝑘 = 1, 2, 3, … , множеств 𝐸𝑛 открытыми элементарными множествами, такие, что
∞
(21)
∑
𝜇(𝐴𝑛𝑘 ) ≤ 𝜇 ∗ (𝐸𝑛 ) + 2−𝑛 𝜀.
𝑘=1
226
Тогда
∞
𝜇∗ (𝐸) ≤
∞
∞
∑∑
𝜇(𝐴𝑛𝑘 ) ≤
𝑛=1 𝑘=1
∑
𝜇∗ (𝐸𝑛 ) + 𝜀,
𝑛=1
∗
откуда следует (19). Если же 𝜇 (𝐸𝑛 ) = +∞ при некотором 𝑛, то неравенство (19), разумеется,
тривиально.
11.9 Определение. Для любых множеств 𝐴 ⊂ 𝑅𝑝 , 𝐵 ⊂ 𝑅𝑝 положим
(22)
𝑆(𝐴, 𝐵) = (𝐴 − 𝐵) ∪ (𝐵 − 𝐴),
(23)
𝑑(𝐴, 𝐵) = 𝜇 ∗ (𝑆(𝐴, 𝐵)).
Будем писать 𝐴𝑛 → 𝐴, если
lim 𝑑(𝐴, 𝐴𝑛 ) = 0.
𝑛→∞
Если существует такая последовательность {𝐴𝑛 } элементарных множеств, что 𝐴𝑛 → 𝐴, то мы
будем говорить, что множество 𝐴 конечно 𝜇-измеримо, и будем писать 𝐴 ∈ 𝔐𝐹 (𝜇).
Если множество 𝐴 равно объединению счетного семейства конечно 𝜇-измеримых множеств, то
мы будем говорить, что 𝐴 𝜇-измеримо, и будем писать 𝐴 ∈ 𝔐(𝜇).
Множество 𝑆(𝐴, 𝐵) — это так называемая «симметрическая разность» множеств 𝐴 и 𝐵. Мы
увидим, что 𝑑(𝐴, 𝐵) в сущности является функцией расстояния.
Следующая теорема позволит нам получить нужное продолжение функции 𝜇.
11.10 Теорема. Множество 𝔐(𝜇) является 𝜎-кольцом, а функция 𝜇 ∗ счетно-аддитивна на 𝔐(𝜇).
Прежде чем обратиться к доказательству этой теоремы, мы изучим некоторые свойства множества 𝑆(𝐴, 𝐵) и числа 𝑑(𝐴, 𝐵). Имеем:
(24)
𝑆(𝐴, 𝐵) = 𝑆(𝐵, 𝐴),
𝑆(𝐴, 𝐴) = 0,
(25)
𝑆(𝐴, 𝐵) ⊂ 𝑆(𝐴, 𝐶) ∪ 𝑆(𝐶, 𝐵),
(26)
𝑆(𝐴1 ∪ 𝐴2 , 𝐵1 ∪ 𝐵2 )⎫
⎪
𝑆(𝐴1 ∩ 𝐴2 , 𝐵1 ∩ 𝐵2 )⎬ ⊂ 𝑆(𝐴1 , 𝐵1 ) ∪ 𝑆(𝐴2 , 𝐵2 ).
𝑆(𝐴1 − 𝐴2 , 𝐵1 − 𝐵2 )⎪
⎭
Утверждение (24) очевидно, а (25) следует из того, что
(𝐴 − 𝐵) ⊂ (𝐴 − 𝐶) ∪ (𝐶 − 𝐵),
(𝐵 − 𝐴) ⊂ (𝐶 − 𝐴) ∪ (𝐵 − 𝐶).
Первая из формул (26) следует из того, что
(𝐴1 ∪ 𝐴2 ) − (𝐵1 ∪ 𝐵2 ) ⊂ (𝐴1 − 𝐵1 ) ∪ (𝐴2 − 𝐵2 ).
Наконец, обозначая через 𝐸 𝑐 дополнение множества 𝐸, получаем
𝑆(𝐴1 ∩ 𝐴2 , 𝐵1 ∩ 𝐵2 ) = 𝑆(𝐴𝑐1 ∪ 𝐴𝑐2 , 𝐵1𝑐 ∪ 𝐵2𝑐 )
⊂ 𝑆(𝐴𝑐1 , 𝐵1𝑐 ) ∪ 𝑆(𝐴𝑐2 , 𝐵2𝑐 ) = 𝑆(𝐴1 , 𝐵1 ) ∪ 𝑆(𝐴2 , 𝐵2 ),
и последняя из формул (26) получится, если заметить, что
𝐴1 − 𝐴2 = 𝐴1 ∩ 𝐴𝑐2 .
Согласно (23), (19) и (18), из этих свойств множества 𝑆(𝐴, 𝐵) следует, что
(27)
𝑑(𝐴, 𝐵) = 𝑑(𝐵, 𝐴),
𝑑(𝐴, 𝐴) = 0,
(28)
𝑑(𝐴, 𝐵) ≤ 𝑑(𝐴, 𝐶) + 𝑑(𝐶, 𝐵),
(29)
𝑑(𝐴1 ∪ 𝐴2 , 𝐵1 ∪ 𝐵2 )⎫
⎪
𝑑(𝐴1 ∩ 𝐴2 , 𝐵1 ∩ 𝐵2 )⎬ ≤ 𝑑(𝐴1 , 𝐵1 ) + 𝑑(𝐴2 , 𝐵2 ).
𝑑(𝐴1 − 𝐴2 , 𝐵1 − 𝐵2 )⎪
⎭
227
Соотношения (27) и (28) показывают, что 𝑑(𝐴, 𝐵) удовлетворяет требованиям определения 2.15,
за исключением того, что из 𝑑(𝐴, 𝐵) = 0 не следует 𝐴 = 𝐵. Например, если 𝜇 = 𝑚, множество 𝐴
счетно, а 𝐵 пусто, то
𝑑(𝐴, 𝐵) = 𝑚∗ (𝐴) = 0;
чтобы убедиться в этом, покроем 𝑛-ю точку множества 𝐴 прямоугольником 𝐼𝑛 , таким, что
𝑚(𝐼𝑛 ) < 2−𝑛 𝜀.
Но если мы будем считать два множества 𝐴 и 𝐵 эквивалентными при условии
𝑑(𝐴, 𝐵) = 0,
то подмножества пространства 𝑅𝑝 разобьются на классы эквивалентности, и 𝑑(𝐴, 𝐵) превращает
множество этих классов эквивалентности в метрическое пространство. Тогда 𝔐𝐹 (𝜇) оказывается
замыканием множества ℰ . Эта интерпретация несущественна для доказательства, но она объясняет
идею, лежащую в его основе.
Нам потребуется еще одно свойство числа 𝑑(𝐴, 𝐵), а именно
(30)
|𝜇∗ (𝐴) − 𝜇 ∗ (𝐵)| ≤ 𝑑(𝐴, 𝐵),
если по крайней мере одно из чисел 𝜇∗ (𝐴), 𝜇∗ (𝐵) конечно. Действительно, пусть 0 ≤ 𝜇 ∗ (𝐵) ≤ 𝜇 ∗ (𝐴).
Тогда (28) показывает, что
𝑑(𝐴, 0) ≤ 𝑑(𝐴, 𝐵) + 𝑑(𝐵, 0),
то есть
𝜇∗ (𝐴) ≤ 𝑑(𝐴, 𝐵) + 𝜇 ∗ (𝐵).
Но так как 𝜇∗ (𝐵) конечно, то
𝜇∗ (𝐴) − 𝜇 ∗ (𝐵) ≤ 𝑑(𝐴, 𝐵).
Доказательство теоремы 11.10. Пусть 𝐴 ∈ 𝔐𝐹 (𝜇), 𝐵 ∈ 𝔐𝐹 (𝜇). Выберем последовательности
{𝐴𝑛 }, {𝐵𝑛 } так, чтобы 𝐴𝑛 ∈ ℰ , 𝐵𝑛 ∈ ℰ , 𝐴𝑛 → 𝐴, 𝐵𝑛 → 𝐵. Тогда (29) и (30) показывают, что
(31)
𝐴𝑛 ∪ 𝐵𝑛 → 𝐴 ∪ 𝐵,
(32)
𝐴𝑛 ∩ 𝐵𝑛 → 𝐴 ∩ 𝐵,
(33)
𝐴𝑛 − 𝐵𝑛 → 𝐴 − 𝐵,
(34)
𝜇 ∗ (𝐴𝑛 ) → 𝜇 ∗ (𝐴),
и 𝜇∗ (𝐴) < +∞, так как 𝑑(𝐴𝑛 , 𝐴) → 0. Согласно (31) и (33), 𝔐𝐹 (𝜇) — кольцо. Согласно (7),
𝜇(𝐴𝑛 ) + 𝜇(𝐵𝑛 ) = 𝜇(𝐴𝑛 ∪ 𝐵𝑛 ) + 𝜇(𝐴𝑛 ∩ 𝐵𝑛 ).
Полагая 𝑛 → ∞, получаем из (34) и теоремы 11.8(a)
𝜇∗ (𝐴) + 𝜇 ∗ (𝐵) = 𝜇 ∗ (𝐴 ∪ 𝐵) + 𝜇 ∗ (𝐴 ∩ 𝐵).
Если 𝐴 ∩ 𝐵 = 0, то 𝜇 ∗ (𝐴 ∩ 𝐵) = 0.
Следовательно, функция 𝜇∗ аддитивна на 𝔐𝐹 (𝜇).
Пусть теперь 𝐴 ∈ 𝔐(𝜇). Тогда 𝐴 можно представить в виде объединения счетного семейства
непересекающихся множеств из 𝔐𝐹 (𝜇). Действительно, если 𝐴 = ⋃ 𝐴′𝑛 , где 𝐴′𝑛 ∈ 𝔐𝐹 (𝜇), то положим
𝐴1 = 𝐴′1 и
𝐴𝑛 = (𝐴′1 ∪ ⋯ ∪ 𝐴′𝑛 ) − (𝐴′1 ∪ ⋯ ∪ 𝐴′𝑛−1 )
(𝑛 = 2, 3, 4, … ).
Тогда
∞
(35)
𝐴=
⋃
𝑛=1
228
𝐴𝑛
— требуемое представление. Согласно (19),
∞
𝜇∗ (𝐴) ≤
(36)
∑
𝜇∗ (𝐴𝑛 ).
𝑛=1
С другой стороны, 𝐴 ⊃ 𝐴1 ∪ ⋯ ∪ 𝐴𝑛 , и, в силу аддитивности функции 𝜇 ∗ на 𝔐𝐹 (𝜇), получаем
(37)
𝜇∗ (𝐴) ≥ 𝜇 ∗ (𝐴1 ∪ ⋯ ∪ 𝐴𝑛 ) = 𝜇 ∗ (𝐴1 ) + ⋯ + 𝜇 ∗ (𝐴𝑛 ).
Из (36) и (37) следует, что
∞
𝜇∗ (𝐴) =
(38)
∑
𝜇∗ (𝐴𝑛 ).
𝑛=1
Допустим, что число 𝜇∗ (𝐴) конечно. Положим 𝐵𝑛 = 𝐴1 ∪ ⋯ ∪ 𝐴𝑛 . Тогда, как показывает (38),
∞
𝑑(𝐴, 𝐵𝑛 ) = 𝜇 ∗ (
∞
𝐴𝑖 ) =
⋃
∑
𝜇∗ (𝐴𝑖 ) → 0
𝑖=𝑛+1
𝑖=𝑛+1
при 𝑛 → ∞. Значит, 𝐵𝑛 → 𝐴, а так как 𝐵𝑛 ∈ 𝔐𝐹 (𝜇), то легко видеть, что 𝐴 ∈ 𝔐𝐹 (𝜇).
Таким образом, мы показали, что 𝐴 ∈ 𝔐𝐹 (𝜇), если 𝐴 ∈ 𝔐(𝜇) и 𝜇∗ (𝐴) < +∞.
Теперь уже ясно, что функция 𝜇∗ счетно-аддитивна на 𝔐(𝜇). Действительно, если
𝐴=
⋃
𝐴𝑛 ,
где {𝐴𝑛 } — последовательность непересекающихся множеств из 𝔐(𝜇), то, как мы показали, (38)
выполняется, если 𝜇 ∗ (𝐴𝑛 ) < +∞ при всех 𝑛, а в противном случае равенство (38) тривиально.
Наконец, мы должны показать, что 𝔐(𝜇) — это 𝜎-кольцо. Если 𝐴𝑛 ∈ 𝔐(𝜇), 𝑛 = 1, 2, 3, … , то
ясно, что ⋃ 𝐴𝑛 ∈ 𝔐(𝜇) (теорема 2.12). Пусть 𝐴 ∈ 𝔐(𝜇), 𝐵 ∈ 𝔐(𝜇) и
∞
𝐴=
⋃
∞
𝐴𝑛 ,
𝐵=
𝑛=1
⋃
𝐵𝑛 ,
𝑛=1
где 𝐴𝑛 , 𝐵𝑛 ∈ 𝔐𝐹 (𝜇). Тогда тождество
∞
𝐴𝑛 ∩ 𝐵 =
⋃
(𝐴𝑛 ∩ 𝐵𝑖 )
𝑖=1
показывает, что 𝐴𝑛 ∩ 𝐵 ∈ 𝔐(𝜇), а так как
𝜇∗ (𝐴𝑛 ∩ 𝐵) ≤ 𝜇 ∗ (𝐴𝑛 ) < +∞,
то 𝐴𝑛 ∩ 𝐵 ∈ 𝔐𝐹 (𝜇). Значит, 𝐴𝑛 − 𝐵 ∈ 𝔐𝐹 (𝜇), и 𝐴 − 𝐵 ∈ 𝔐(𝜇), так как 𝐴 − 𝐵 = ⋃∞
𝑛=1 (𝐴𝑛 − 𝐵).
Теперь мы заменим 𝜇∗ (𝐴) на 𝜇(𝐴), если 𝐴 ∈ 𝔐(𝜇). Таким образом, функция 𝜇, определенная
первоначально только на ℰ , продолжена до счетно-аддитивной функции множества на 𝜎-кольце
𝔐(𝜇). Эта продолженная функция множества называется мерой. В том частном случае, когда
𝜇 = 𝑚, она называется лебеговой мерой в пространстве 𝑅𝑝 .
11.11 Замечания.
(a) Если множество 𝐴 открыто, то 𝐴 ∈ 𝔐(𝜇). Действительно, каждое открытое множество в 𝑅𝑝
равно объединению счетного семейства открытых прямоугольников. Чтобы в этом убедиться,
достаточно построить счетную базу, элементами которой служат открытые прямоугольники.
Переходя к дополнениям, получаем, что каждое замкнутое множество лежит в 𝔐(𝜇).
229
(b) Если 𝐴 ∈ 𝔐(𝜇) и 𝜀 > 0, то существуют множества 𝐹 и 𝐺, такие, что
𝐹 ⊂ 𝐴 ⊂ 𝐺,
𝐹 замкнуто, 𝐺 открыто и
(39)
𝜇(𝐺 − 𝐴) < 𝜀,
𝜇(𝐴 − 𝐹 ) < 𝜀.
Первое неравенство выполняется потому, что мера 𝜇∗ была определена с помощью покрытий
открытыми элементарными множествами. Второе неравенство получится, если перейти к
дополнениям.
(c) Мы говорим, что 𝐸 — борелевское множество, если 𝐸 может быть получено с помощью счетного множества операций, исходя из открытых множеств, причем каждая операция — это
либо взятие объединения, либо взятие пересечения, либо переход к дополнению. Множество
ℬ всех борелевских подмножеств пространства 𝑅𝑝 образует 𝜎-кольцо; в действительности
это наименьшее из 𝜎-колец, содержащих все открытые множества. Согласно замечанию (a),
𝐸 ∈ 𝔐(𝜇), если 𝐸 ∈ ℬ.
(d) Если 𝐴 ∈ 𝔐(𝜇), то существуют борелевские множества 𝐹 и 𝐺, такие, что 𝐹 ⊂ 𝐴 ⊂ 𝐺 и
(40)
𝜇(𝐺 − 𝐴) = 𝜇(𝐴 − 𝐹 ) = 0.
Это следует из (b), если взять 𝜀 = 1/𝑛 и положить 𝑛 → ∞.
Поскольку 𝐴 = 𝐹 ∪ (𝐴 − 𝐹 ), мы видим, что каждое 𝐴 ∈ 𝔐(𝜇) представляет собой объединение
борелевского множества и множества нулевой меры.
Борелевские множества 𝜇-измеримы при каждом 𝜇. Но множества меры нуль (т. е. множества
𝐸, для которых 𝜇∗ (𝐸) = 0) могут быть различными для различных 𝜇.
(e) Какова бы ни была функция 𝜇, множества меры нуль образуют 𝜎-кольцо.
(f) В случае меры Лебега всякое счетное множество имеет меру нуль. Но существуют и несчетные
(и даже совершенные) множества меры нуль. Примером может служить множество Кантора:
используя обозначения из п. 2.44, легко показать, что
𝑚(𝐸𝑛 ) = ( 32 )𝑛
(𝑛 = 1, 2, 3, … ),
а так как 𝑃 = ⋂ 𝐸𝑛 , то 𝑃 ⊂ 𝐸𝑛 при любом 𝑛, так что 𝑚(𝑃 ) = 0.
Пространства с мерой
11.12 Определение. Пусть 𝑋 — множество, не обязательно являющееся подмножеством евклидова пространства или вообще какого-нибудь метрического пространства. 𝑋 называется пространством с мерой, если существует 𝜎-кольцо 𝔐 подмножеств множества 𝑋 (называемых измеримыми
множествами) и неотрицательная счетно-аддитивная функция множества 𝜇 (называемая мерой),
определенная на 𝔐.
Если, кроме того, 𝑋 ∈ 𝔐, то 𝑋 называется измеримым пространством.
Например, мы можем взять в качестве 𝑋 пространство 𝑅𝑝 , в качестве 𝔐 — множество всех
измеримых по Лебегу подмножеств пространства 𝑅𝑝 , а в качестве 𝜇 — меру Лебега.
Или в качестве 𝑋 можно взять множество всех положительных целых чисел, в качестве 𝔐 —
множество всех подмножеств множества 𝑋, а в качестве 𝜇(𝐸) — число элементов множества 𝐸.
Другой пример дает теория вероятностей, в которой события можно рассматривать как множества, а вероятность наступления события — это аддитивная (или счетно-аддитивная) функция
множества.
В следующих разделах мы всегда будем иметь дело с измеримыми пространствами. Следует
подчеркнуть, что теория интеграла, к которой мы вскоре перейдем, ни в каком отношении не
стала бы проще, если бы мы пожертвовали той степенью общности, которой мы сейчас достигли, и
ограничились, скажем, мерой Лебега на промежутке вещественной прямой. На самом деле основные
черты теории с гораздо большей ясностью проявляются именно в общей ситуации, когда хорошо
видно, что все зависит только от счетной аддитивности меры 𝜇 на некотором 𝜎-кольце.
Нам будет удобно ввести обозначение
(41)
{𝑥|𝑃 }
для множества всех элементов 𝑥, обладающих свойством 𝑃 .
230
Измеримые функции
11.13 Определение. Пусть 𝑓 — функция, определенная на измеримом пространстве 𝑋 со значениями в расширенной системе вещественных чисел. Функция 𝑓 называется измеримой, если
множество
(42)
{𝑥|𝑓 (𝑥) > 𝑎}
измеримо при каждом вещественном 𝑎.
11.14 Пример. Если 𝑋 = 𝑅𝑝 , а 𝔐 = 𝔐(𝜇) в соответствии с определением 11.9, то каждая непрерывная функция 𝑓 измерима, так как в этом случае множество (42) открыто.
11.15 Теорема. Следующие условия эквивалентны:
(43)
{𝑥|𝑓 (𝑥) > 𝑎} измеримо при каждом вещественном 𝑎;
(44)
{𝑥|𝑓 (𝑥) ≥ 𝑎} измеримо при каждом вещественном 𝑎;
(45)
{𝑥|𝑓 (𝑥) < 𝑎} измеримо при каждом вещественном 𝑎;
(46)
{𝑥|𝑓 (𝑥) ≤ 𝑎} измеримо при каждом вещественном 𝑎.
Доказательство. Отношения
∞
{𝑥|𝑓 (𝑥) ≥ 𝑎} =
1
𝑥|𝑓 (𝑥) > 𝑎 − } ,
⋂{
𝑛
𝑛=1
{𝑥|𝑓 (𝑥) < 𝑎} = 𝑋 − {𝑥|𝑓 (𝑥) ≥ 𝑎},
∞
{𝑥|𝑓 (𝑥) ≤ 𝑎} =
1
𝑥|𝑓 (𝑥) < 𝑎 + } ,
⋂{
𝑛
𝑛=1
{𝑥|𝑓 (𝑥) > 𝑎} = 𝑋 − {𝑥|𝑓 (𝑥) ≤ 𝑎}
последовательно показывают, что из (43) следует (44), из (44) следует (45), из (45) следует (46), а
из (46) следует (43).
Значит, каждое из этих условий можно использовать вместо (42) для определения измеримости.
11.16 Теорема. Если 𝑓 измерима, то |𝑓 | измерима.
Доказательство.
{𝑥||𝑓 (𝑥)| < 𝑎} = {𝑥|𝑓 (𝑥) < 𝑎} ∩ {𝑥|𝑓 (𝑥) > −𝑎}.
11.17 Теорема. Пусть {𝑓𝑛 } — последовательность измеримых функций. При 𝑥 ∈ 𝑋 положим
𝑔(𝑥) = sup 𝑓𝑛 (𝑥)
(𝑛 = 1, 2, 3, … ),
ℎ(𝑥) = lim 𝑓𝑛 (𝑥).
𝑛→∞
Тогда 𝑔 и ℎ измеримы.
Конечно, то же верно и в отношении нижней грани и нижнего предела.
Доказательство.
∞
{𝑥|𝑔(𝑥) > 𝑎} =
{𝑥|𝑓𝑛 (𝑥) > 𝑎},
⋃
𝑛=1
ℎ(𝑥) = inf 𝑔𝑚 (𝑥),
где 𝑔𝑚 (𝑥) = sup 𝑓𝑛 (𝑥) (𝑛 ≥ 𝑚).
Следствия.
231
(a) Если 𝑓 и 𝑔 измеримы, то max(𝑓 , 𝑔) и min(𝑓 , 𝑔) измеримы. Если
𝑓 + = max(𝑓 , 0),
(47)
𝑓 − = − min(𝑓 , 0),
то, в частности, 𝑓 + и 𝑓 − измеримы.
(b) Предел сходящейся последовательности измеримых функций — измеримая функция.
11.18 Теорема. Пусть 𝑓 и 𝑔 — измеримые вещественные функции, определенные на множестве
𝑋, пусть функция 𝐹 вещественна и непрерывна на 𝑅2 , и пусть
ℎ(𝑥) = 𝐹 (𝑓 (𝑥), 𝑔(𝑥))
(𝑥 ∈ 𝑋).
Тогда функция ℎ измерима.
В частности, функции 𝑓 + 𝑔 и 𝑓 𝑔 измеримы.
Доказательство. Пусть
𝐺𝑎 = {(𝑢, 𝑣)|𝐹 (𝑢, 𝑣) > 𝑎}.
Тогда 𝐺𝑎 — открытое подмножество пространства 𝑅2 и
∞
𝐺𝑎 =
⋃
𝐼𝑛 ,
𝑛=1
где {𝐼𝑛 } — последовательность открытых прямоугольников:
𝐼𝑛 = {(𝑢, 𝑣)|𝑎𝑛 < 𝑢 < 𝑏𝑛 , 𝑐𝑛 < 𝑣 < 𝑑𝑛 }.
Поскольку множество
{𝑥|𝑎𝑛 < 𝑓 (𝑥) < 𝑏𝑛 } = {𝑥|𝑓 (𝑥) > 𝑎𝑛 } ∩ {𝑥|𝑓 (𝑥) < 𝑏𝑛 }
измеримо, то множество
{𝑥|(𝑓 (𝑥), 𝑔(𝑥)) ∈ 𝐼𝑛 } = {𝑥|𝑎𝑛 < 𝑓 (𝑥) < 𝑏𝑛 } ∩ {𝑥|𝑐𝑛 < 𝑔(𝑥) < 𝑑𝑛 }
измеримо. Значит, то же верно и в отношении множества
{𝑥|ℎ(𝑥) > 𝑎} = {𝑥|(𝑓 (𝑥), 𝑔(𝑥)) ∈ 𝐺𝑛 }
∞
=
⋃
{𝑥|(𝑓 (𝑥), 𝑔(𝑥)) ∈ 𝐼𝑛 }.
𝑛=1
Подводя итоги, мы можем сказать, что все обычные операции анализа, включая операции, связанные с предельным переходом, будучи примененными к измеримым функциям, приводят снова к
измеримым функциям; иными словами, все функции, с которыми обычно встречаются, измеримы.
То, что эта формулировка тем не менее довольно груба, видно из следующего примера (основанного на лебеговой мере на вещественной прямой): если ℎ(𝑥) = 𝑓 (𝑔(𝑥)), где функция 𝑓 измерима,
а 𝑔 непрерывна, то функция ℎ не обязательно измерима (подробности см. МакШейн, с. 241).
Читатель, возможно, заметил, что в нашем обсуждении измеримых функций нигде не упоминалась мера. В самом деле, класс функций, измеримых на 𝑋, зависит только от 𝜎-кольца 𝔐
(обозначения те же, что в п. 11.12). Например, можно говорить о функциях, измеримых по Борелю
на 𝑅𝑝 , т. е. о функциях 𝑓 , для которых множество
{𝑥|𝑓 (𝑥) > 𝑎}
всегда борелевское, не упоминая при этом никакой конкретной меры.
232
Простые функции
11.19 Определение. Пусть 𝑠 — вещественная функция, определенная на множестве 𝑋. Если множество значений функции 𝑠 конечно, то мы будем говорить, что 𝑠 — простая функция.
Пусть 𝐸 ⊂ 𝑋, и пусть
(48)
𝐾𝐸 (𝑥) =
1
{0
(𝑥 ∈ 𝐸),
(𝑥 ∉ 𝐸).
𝐾𝐸 называется характеристической функцией множества 𝐸.
Пусть множество значений функции 𝑠 состоит из различных чисел 𝑐1 , … , 𝑐𝑛 . Пусть
𝐸𝑖 = {𝑥|𝑠(𝑥) = 𝑐𝑖 }
(𝑖 = 1, … , 𝑛).
Тогда
𝑛
(49)
𝑠=
∑
𝑖=1
𝑐𝑖 𝐾𝐸𝑖 ,
т. е. каждая простая функция представляет собой конечную линейную комбинацию характеристических функций. Ясно, что 𝑠 измерима тогда и только тогда, когда множества 𝐸1 , … , 𝐸𝑛 измеримы.
Оказывается, любую функцию можно приблизить простыми функциями.
11.20 Теорема. Пусть 𝑓 — вещественная функция, определенная на множестве 𝑋. Тогда существует последовательность {𝑠𝑛 } простых функций, такая, что 𝑠𝑛 (𝑥) → 𝑓 (𝑥) при 𝑛 → ∞ для
всякого 𝑥 ∈ 𝑋. Если функция 𝑓 измерима, то можно выбрать последовательность {𝑠𝑛 } так,
чтобы все функции 𝑠𝑛 тоже были измеримы. Если 𝑓 ≥ 0, то последовательность {𝑠𝑛 } можно
выбрать монотонно возрастающей.
Доказательство. Если 𝑓 ≥ 0, то положим
𝑖−1
𝑖
𝐸𝑛𝑖 = {𝑥| 𝑛 ≤ 𝑓 (𝑥) < 𝑛 } ,
2
2
𝐹𝑛 = {𝑥|𝑓 (𝑥) ≥ 𝑛}
при 𝑛 = 1, 2, 3, … , 𝑖 = 1, 2, … , 𝑛2𝑛 . Пусть
𝑛2𝑛
(50)
𝑠𝑛 =
𝑖−1
𝐾 + 𝑛𝐾𝐹𝑛 .
∑ 2𝑛 𝐸𝑛𝑖
𝑖=1
В общем случае запишем 𝑓 = 𝑓 + − 𝑓 − и применим предыдущую конструкцию к 𝑓 + и 𝑓 − .
Заметим, что последовательность {𝑠𝑛 }, заданная равенством (50), сходится к 𝑓 равномерно, если
𝑓 ограничена.
Интегрирование
Мы определим интегрирование на измеримом пространстве 𝑋 с 𝜎-кольцом 𝔐 измеримых множеств и с мерой 𝜇. Читатель, желающий иметь перед глазами более конкретную ситуацию, может
представлять себе 𝑋 как вещественную прямую или как прямоугольник, а 𝜇 — как меру Лебега.
11.21 Определение. Допустим, что функция
𝑛
(51)
𝑠(𝑥) =
∑
𝑖=1
𝑐𝑖 𝐾𝐸𝑖 (𝑥)
(𝑥 ∈ 𝑋, 𝑐𝑖 > 0)
измерима, и пусть 𝐸 ∈ 𝔐. Положим
𝑛
(52)
𝐼𝐸 (𝑠) =
∑
𝑐𝑖 𝜇(𝐸 ∩ 𝐸𝑖 ).
𝑖=1
233
Если функция 𝑓 измерима и неотрицательна, то мы определим
(53)
∫
𝐸
𝑓 𝑑𝜇 = sup 𝐼𝐸 (𝑠),
где верхняя грань берется по всем измеримым простым функциям 𝑠, таким, что 0 ≤ 𝑠 ≤ 𝑓 .
Левая часть равенства (53) называется интегралом Лебега функции 𝑓 относительно меры 𝜇 по
множеству 𝐸. Заметим, что интеграл может быть равным +∞.
Легко проверить, что
(54)
∫
𝐸
𝑠 𝑑𝜇 = 𝐼𝐸 (𝑠)
для любой неотрицательной простой измеримой функции 𝑠.
11.22 Определение. Пусть функция 𝑓 измерима. Рассмотрим два интеграла
(55)
∫
𝐸
𝑓 + 𝑑𝜇,
∫
𝐸
𝑓 − 𝑑𝜇,
где 𝑓 + и 𝑓 − определены, как в (47).
Если хотя бы один из интегралов (55) конечен, то мы полагаем по определению
(56)
∫
𝐸
𝑓 𝑑𝜇 =
∫
𝐸
𝑓 + 𝑑𝜇 −
∫
𝐸
𝑓 − 𝑑𝜇.
Если оба интеграла (55) конечны, то и разность (56) конечна, и мы говорим, что функция 𝑓
интегрируема (или суммируема) на множестве 𝐸 в смысле Лебега по отношению к мере 𝜇; мы
пишем 𝑓 ∈ ℒ (𝜇) на 𝐸. Если 𝜇 = 𝑚, то обычное обозначение таково: 𝑓 ∈ ℒ на 𝐸.
Эта терминология может вызвать небольшую путаницу: если (56) равно +∞ или −∞, то интеграл функции 𝑓 по множеству 𝐸 определен, хотя функция 𝑓 и не интегрируема в только что
разъясненном смысле слова; 𝑓 интегрируема на 𝐸 только тогда, когда ее интеграл по 𝐸 конечен.
Мы главным образом будем заниматься интегрируемыми функциями, хотя в некоторых случаях
предпочтительно рассматривать более общую ситуацию.
11.23 Замечания. Следующие свойства очевидны.
(a) Если 𝑓 измерима и ограничена на 𝐸 и 𝜇(𝐸) < +∞, то 𝑓 ∈ ℒ (𝜇) на 𝐸.
(b) Если 𝑎 ≤ 𝑓 (𝑥) ≤ 𝑏 при 𝑥 ∈ 𝐸, а 𝜇(𝐸) < +∞, то
𝑎𝜇(𝐸) ≤
∫
𝐸
𝑓 𝑑𝜇 ≤ 𝑏𝜇(𝐸).
(c) Если 𝑓 и 𝑔 ∈ ℒ (𝜇) на 𝐸 и если 𝑓 (𝑥) ≤ 𝑔(𝑥) при 𝑥 ∈ 𝐸, то
∫
𝐸
𝑓 𝑑𝜇 ≤
∫
𝐸
𝑔 𝑑𝜇.
(d) Если 𝑓 ∈ ℒ (𝜇) на 𝐸, то 𝑐𝑓 ∈ ℒ (𝜇) на 𝐸, каково бы ни было конечное число 𝑐, и
∫
𝐸
𝑐𝑓 𝑑𝜇 = 𝑐
∫
𝐸
𝑓 𝑑𝜇.
(e) Если 𝜇(𝐸) = 0, а 𝑓 измерима, то
∫
𝐸
𝑓 𝑑𝜇 = 0.
(f) Если 𝑓 ∈ ℒ (𝜇) на 𝐸, 𝐴 ∈ 𝔐 и 𝐴 ⊂ 𝐸, то 𝑓 ∈ ℒ (𝜇) на 𝐴.
11.24 Теорема.
234
(a) Пусть 𝑓 измерима и неотрицательна на 𝑋. Для 𝐴 ∈ 𝔐 положим
(57)
𝜑(𝐴) =
∫
𝐴
𝑓 𝑑𝜇.
Тогда функция 𝜑 счетно-аддитивна на 𝔐.
(b) То же верно, если 𝑓 ∈ ℒ (𝜇) на 𝑋.
Доказательство. Ясно, что (b) следует из (a), если мы запишем 𝑓 = 𝑓 + − 𝑓 − и применим (a) к
𝑓 + и 𝑓 −.
Чтобы доказать (a), мы должны показать, что
∞
(58)
𝜑(𝐴) =
∑
𝜑(𝐴𝑛 ),
𝑛=1
если 𝐴𝑛 ∈ 𝔐 (𝑛 = 1, 2, 3, … ), 𝐴𝑖 ∩ 𝐴𝑗 = 0 при 𝑖 ≠ 𝑗 и 𝐴 = ⋃∞
1 𝐴𝑛 .
Если 𝑓 — характеристическая функция, то счетная аддитивность функции 𝜑 — то же самое,
что счетная аддитивность функции 𝜇, так как
∫
𝐴
𝐾𝐸 𝑑𝜇 = 𝜇(𝐴 ∩ 𝐸).
Если 𝑓 — простая функция, то 𝑓 имеет вид (51) и утверждение теоремы также выполняется.
В общем случае для каждой измеримой простой функции 𝑠, такой, что 0 ≤ 𝑠 ≤ 𝑓 , имеем
∞
∫
𝐴
𝑠 𝑑𝜇 =
∞
∑∫
𝐴
𝑛=1
𝑠 𝑑𝜇 ≤
∑
𝜑(𝐴𝑛 ).
𝑛=1
𝑛
Поэтому, согласно (53),
∞
(59)
𝜑(𝐴) ≤
∑
𝜑(𝐴𝑛 ).
𝑛=1
Заметим теперь, что если 𝜑(𝐴𝑛 ) = +∞ при каком-нибудь 𝑛, то (58) тривиально, так как 𝜑(𝐴) ≥
𝜑(𝐴𝑛 ). Поэтому пусть 𝜑(𝐴𝑛 ) < +∞ при всех 𝑛.
Для заданного 𝜀 > 0 выберем измеримую функцию 𝑠 так, что 0 ≤ 𝑠 ≤ 𝑓 и
(60)
∫
𝐴1
𝑠 𝑑𝜇 ≥
∫
𝐴1
𝑓 𝑑𝜇 − 𝜀,
∫
𝐴2
𝑠 𝑑𝜇 ≥
∫
𝐴2
𝑓 𝑑𝜇 − 𝜀.
Ясно, что
𝜑(𝐴1 ∪ 𝐴2 ) ≥
∫
𝐴1 ∪𝐴2
𝑠 𝑑𝜇 =
∫
𝐴1
𝑠 𝑑𝜇 +
∫
𝐴2
𝑠 𝑑𝜇 ≥ 𝜑(𝐴1 ) + 𝜑(𝐴2 ) − 2𝜀,
так что
𝜑(𝐴1 ∪ 𝐴2 ) ≥ 𝜑(𝐴1 ) + 𝜑(𝐴2 ).
Следовательно, при каждом 𝑛
(61)
𝜑(𝐴1 ∪ ⋯ ∪ 𝐴𝑛 ) ≥ 𝜑(𝐴1 ) + ⋯ + 𝜑(𝐴𝑛 ).
Поскольку 𝐴 ⊃ 𝐴1 ∪ ⋯ ∪ 𝐴𝑛 , то из (61) следует, что
∞
(62)
𝜑(𝐴) ≥
∑
𝑛=1
и (58) вытекает из (59) и (62).
235
𝜑(𝐴𝑛 ),
Следствие. Если 𝐴 ∈ 𝔐, 𝐵 ∈ 𝔐, 𝐵 ⊂ 𝐴 и 𝜇(𝐴 − 𝐵) = 0, то
∫
𝐴
𝑓 𝑑𝜇 =
∫
𝐵
𝑓 𝑑𝜇.
Поскольку 𝐴 = 𝐵 ∪ (𝐴 − 𝐵), это следует из замечания 11.23(e).
11.25 Замечания. Приведенное выше следствие показывает, что множествами меры нуль при
интегрировании можно пренебречь.
Мы будем писать 𝑓 ∼ 𝑔 на 𝐸, если множество
{𝑥|𝑓 (𝑥) ≠ 𝑔(𝑥)} ∩ 𝐸
имеет меру нуль.
Тогда 𝑓 ∼ 𝑓 ; из 𝑓 ∼ 𝑔 следует, что 𝑔 ∼ 𝑓 ; и из 𝑓 ∼ 𝑔, 𝑔 ∼ ℎ следует, что 𝑓 ∼ ℎ. Это значит, что
отношение ∼ есть отношение эквивалентности.
Если 𝑓 ∼ 𝑔 на 𝐸, то, очевидно, для любого измеримого подмножества 𝐴 множества 𝐸 имеем
∫
𝐴
𝑓 𝑑𝜇 =
∫
𝐴
𝑔 𝑑𝜇,
при условии, что эти интегралы существуют.
Если свойство 𝑃 выполняется для каждого 𝑥 ∈ 𝐸 − 𝐴 и если 𝜇(𝐴) = 0, то обычно говорят, что 𝑃
выполняется для почти всех 𝑥 ∈ 𝐸 или что 𝑃 выполняется почти всюду на 𝐸. (Смысл этого «почти
всюду» зависит, разумеется, от той конкретной меры, которую мы рассматриваем. В литературе,
если не оговорено противное, обычно имеют в виду меру Лебега.)
Если 𝑓 ∈ ℒ (𝜇) на 𝐸, то ясно, что 𝑓 (𝑥) должно быть конечным почти всюду на 𝐸. Поэтому в
большинстве случаев мы можем, не умаляя общности, с самого начала предполагать, что функции,
с которыми мы имеем дело, принимают только конечные значения.
11.26 Теорема. Если 𝑓 ∈ ℒ (𝜇) на 𝐸, то |𝑓 | ∈ ℒ (𝜇) на 𝐸 и
(63)
|∫
𝐸
𝑓 𝑑𝜇 ≤
|𝑓 | 𝑑𝜇.
| ∫
𝐸
Доказательство. Запишем 𝐸 = 𝐴 ∪ 𝐵, где 𝑓 (𝑥) ≥ 0 на 𝐴 и 𝑓 (𝑥) < 0 на 𝐵. По теореме 11.24
∫
𝐸
|𝑓 | 𝑑𝜇 =
∫
𝐴
|𝑓 | 𝑑𝜇 +
∫
𝐵
|𝑓 | 𝑑𝜇 =
∫
𝐴
𝑓 + 𝑑𝜇 +
∫
𝐵
𝑓 − 𝑑𝜇 < +∞,
так что |𝑓 | ∈ ℒ (𝜇). Поскольку 𝑓 ≤ |𝑓 | и −𝑓 ≤ |𝑓 |, мы видим, что
∫
𝐸
𝑓 𝑑𝜇 ≤
∫
𝐸
|𝑓 | 𝑑𝜇,
−
∫
𝐸
𝑓 𝑑𝜇 ≤
∫
𝐸
|𝑓 | 𝑑𝜇,
откуда и следует (63).
Поскольку из интегрируемости функции 𝑓 следует интегрируемость функции |𝑓 |, то интеграл
Лебега часто называют абсолютно сходящимся. Конечно, можно определить и неабсолютно сходящиеся интегралы, и при изучении некоторых проблем это даже существенно. Но у этих интегралов
отсутствуют наиболее полезные свойства интеграла Лебега, и они играют в анализе несколько менее
важную роль.
11.27 Теорема. Пусть 𝑓 измерима на 𝐸, |𝑓 | ≤ 𝑔 и 𝑔 ∈ ℒ (𝜇) на 𝐸. Тогда 𝑓 ∈ ℒ (𝜇) на 𝐸.
Доказательство. Имеем 𝑓 + ≤ 𝑔 и 𝑓 − ≤ 𝑔.
11.28 Теорема Лебега о монотонной сходимости. Пусть 𝐸 ∈ 𝔐. Пусть {𝑓𝑛 } — такая последовательность измеримых функций, что
(64)
0 ≤ 𝑓1 (𝑥) ≤ 𝑓2 (𝑥) ≤ ⋯
236
(𝑥 ∈ 𝐸).
Пусть функция 𝑓 определена как
(65)
𝑓𝑛 (𝑥) → 𝑓 (𝑥)
(𝑥 ∈ 𝐸)
при 𝑛 → ∞. Тогда
(66)
∫
𝐸
𝑓𝑛 𝑑𝜇 →
𝑓 𝑑𝜇
∫
𝐸
(𝑛 → ∞).
Доказательство. Согласно (64), существует такое 𝛼, что
(67)
∫
𝐸
𝑓𝑛 𝑑𝜇 → 𝛼
при 𝑛 → ∞, а так как ∫ 𝑓𝑛 ≤ ∫ 𝑓 , то
(68)
𝛼≤
∫
𝐸
𝑓 𝑑𝜇.
Выберем 𝑐 так, чтобы 0 < 𝑐 < 1, и пусть 𝑠 — простая измеримая функция, такая, что 0 ≤ 𝑠 ≤ 𝑓 .
Положим
𝐸𝑛 = {𝑥|𝑓𝑛 (𝑥) ≥ 𝑐𝑠(𝑥)}
(𝑛 = 1, 2, 3, … ).
Согласно (64), 𝐸1 ⊂ 𝐸2 ⊂ 𝐸3 ⊂ ⋯ , а в силу (65)
∞
(69)
𝐸=
⋃
𝐸𝑛 .
𝑛=1
При любом 𝑛
(70)
∫
𝐸
𝑓𝑛 𝑑𝜇 ≥
𝑓𝑛 𝑑𝜇 ≥ 𝑐
∫
𝐸𝑛
∫
𝐸𝑛
𝑠 𝑑𝜇.
Положим 𝑛 → ∞ в (70). Поскольку интеграл — счетно-аддитивная функция множества (теорема
11.24), то, как показывает (69), можно применить теорему 11.3 к последнему интегралу в (70), и
мы получим
(71)
𝛼≥𝑐
∫
𝐸
𝑠 𝑑𝜇.
Положив 𝑐 → 1, мы видим, что
𝛼≥
∫
𝐸
𝑠 𝑑𝜇,
а из (53) следует, что
(72)
𝛼≥
∫
𝐸
𝑓 𝑑𝜇.
Теорема следует теперь из (67), (68) и (72).
11.29 Теорема. Пусть 𝑓 = 𝑓1 + 𝑓2 , где 𝑓𝑖 ∈ ℒ (𝜇) на 𝐸 (𝑖 = 1, 2). Тогда 𝑓 ∈ ℒ (𝜇) на 𝐸 и
(73)
∫
𝐸
𝑓 𝑑𝜇 =
∫
𝐸
𝑓1 𝑑𝜇 +
∫
𝐸
𝑓2 𝑑𝜇.
Доказательство. Сначала допустим, что 𝑓1 ≥ 0, 𝑓2 ≥ 0. Если 𝑓1 и 𝑓2 — простые функции, то (73)
тривиально следует из (52) и (54). В общем случае выберем монотонно возрастающие последовательности {𝑠′𝑛 }, {𝑠″𝑛 } неотрицательных измеримых простых функций, сходящихся к 𝑓1 , 𝑓2 . Теорема
11.20 показывает, что это возможно. Положим 𝑠𝑛 = 𝑠′𝑛 + 𝑠″𝑛 . Тогда
∫
𝐸
𝑠𝑛 𝑑𝜇 =
∫
𝐸
𝑠′𝑛 𝑑𝜇 +
237
∫
𝐸
𝑠″𝑛 𝑑𝜇,
и (73) получится, если мы положим 𝑛 → ∞ и применим теорему 11.28.
Теперь допустим, что 𝑓1 ≥ 0, 𝑓2 ≤ 0. Положим
𝐴 = {𝑥|𝑓 (𝑥) ≥ 0},
𝐵 = {𝑥|𝑓 (𝑥) < 0}.
Тогда функции 𝑓 , 𝑓1 и −𝑓2 неотрицательны на 𝐴. Значит,
(74)
∫
𝐴
𝑓1 𝑑𝜇 =
∫
𝐴
𝑓 𝑑𝜇 +
∫
𝐴
(−𝑓2 ) 𝑑𝜇 =
∫
𝐴
𝑓 𝑑𝜇 −
∫
𝐴
𝑓2 𝑑𝜇.
Аналогично функции −𝑓 , 𝑓1 и −𝑓2 неотрицательны на 𝐵, так что
∫
𝐵
(−𝑓2 ) 𝑑𝜇 =
∫
𝐵
𝑓1 𝑑𝜇 +
∫
𝐵
(−𝑓 ) 𝑑𝜇,
или
(75)
∫
𝐵
𝑓1 𝑑𝜇 =
∫
𝐵
𝑓 𝑑𝜇 −
∫
𝐵
𝑓2 𝑑𝜇,
и (73) получается, если сложить (74) и (75).
В общем случае 𝐸 можно разложить на четыре множества 𝐸𝑖 , на каждом из которых 𝑓1 (𝑥) и
𝑓2 (𝑥) сохраняют знак. Из двух доказанных случаев следует, что
∫
𝐸𝑖
𝑓 𝑑𝜇 =
∫
𝐸𝑖
𝑓1 𝑑𝜇 +
∫
𝐸𝑖
𝑓2 𝑑𝜇
(𝑖 = 1, 2, 3, 4),
и (73) получается, если сложить эти четыре равенства.
Теорему 11.28 можно следующим образом переформулировать в терминах рядов функций.
11.30 Теорема. Пусть 𝐸 ∈ 𝔐. Если {𝑓𝑛 } — последовательность неотрицательных измеримых
функций и
∞
(76)
𝑓 (𝑥) =
∑
𝑓𝑛 (𝑥)
(𝑥 ∈ 𝐸),
𝑛=1
то
∞
∫
𝐸
𝑓 𝑑𝜇 =
∑∫
𝐸
𝑓𝑛 𝑑𝜇.
𝑛=1
Доказательство. Частные суммы ряда (76) образуют монотонно возрастающую последовательность.
11.31 Теорема Фату. Пусть 𝐸 ∈ 𝔐. Если {𝑓𝑛 } — последовательность неотрицательных измеримых функций и
𝑓 (𝑥) = lim 𝑓𝑛 (𝑥)
(𝑥 ∈ 𝐸),
𝑛→∞
то
(77)
∫
𝐸
𝑓 𝑑𝜇 ≤ lim
𝐸
𝑛→∞ ∫
𝑓𝑛 𝑑𝜇.
В (77) может иметь место строгое неравенство. Пример указан в упражнении 5.
Доказательство. При 𝑛 = 1, 2, 3, … и 𝑥 ∈ 𝐸 положим
𝑔𝑛 (𝑥) = inf 𝑓𝑖 (𝑥)
(𝑖 ≥ 𝑛).
Тогда 𝑔𝑛 измеримы на 𝐸 и
(78)
0 ≤ 𝑔1 (𝑥) ≤ 𝑔2 (𝑥) ≤ ⋯ ,
(79)
𝑔𝑛 (𝑥) ≤ 𝑓𝑛 (𝑥),
(80)
𝑔𝑛 (𝑥) → 𝑓 (𝑥)
238
(𝑛 → ∞).
Согласно (78), (80) и теореме 11.28,
(81)
∫
𝐸
𝑔𝑛 𝑑𝜇 →
∫
𝐸
𝑓 𝑑𝜇,
так что (77) следует из (79) и (81).
11.32 Теорема Лебега об ограниченной сходимости. Пусть 𝐸 ∈ 𝔐. Пусть {𝑓𝑛 } — такая
последовательность измеримых функций, что
(82)
𝑓𝑛 (𝑥) → 𝑓 (𝑥)
(𝑥 ∈ 𝐸)
при 𝑛 → ∞. Если существует функция 𝑔 ∈ ℒ (𝜇), такая, что
(83)
|𝑓𝑛 (𝑥)| ≤ 𝑔(𝑥)
(𝑛 = 1, 2, 3, … , 𝑥 ∈ 𝐸),
то
(84)
lim
𝑛→∞ ∫
𝐸
𝑓𝑛 𝑑𝜇 =
∫
𝐸
𝑓 𝑑𝜇.
Неравенство (83) означает, что функция 𝑔 ограничивает последовательность {𝑓𝑛 }, этим объясняется название теоремы. В силу замечания 11.25, утверждение теоремы остается верным, если
(82) выполняется почти всюду на 𝐸.
Доказательство. Заметим сначала, что из (83) и теоремы 11.27 следует, что 𝑓𝑛 ∈ ℒ (𝜇) и 𝑓 ∈ ℒ (𝜇)
на 𝐸.
Ввиду того что 𝑓𝑛 + 𝑔 ≥ 0, теорема Фату показывает, что
∫
𝐸
(𝑓 + 𝑔) 𝑑𝜇 ≤ lim
𝐸
𝑛→∞ ∫
(𝑓𝑛 + 𝑔) 𝑑𝜇,
иначе говоря,
(85)
∫
𝐸
𝑓 𝑑𝜇 ≤ lim
𝐸
𝑛→∞ ∫
𝑓𝑛 𝑑𝜇.
Аналогично, поскольку 𝑔 − 𝑓𝑛 ≥ 0, то
∫
𝐸
(𝑔 − 𝑓 ) 𝑑𝜇 ≤ lim
𝐸
𝑛→∞ ∫
(𝑔 − 𝑓𝑛 ) 𝑑𝜇,
так что
−
∫
𝐸
𝑓 𝑑𝜇 ≤ lim −
𝑓𝑛 𝑑𝜇 ,
]
𝐸
𝑛→∞ [ ∫
другими словами,
(86)
∫
𝐸
𝑓 𝑑𝜇 ≥ lim
𝑛→∞ ∫
𝐸
𝑓 𝑑𝜇.
Существование предела в (84) и равенство (84) теперь следуют из (85) и (86).
Следствие. Если 𝜇(𝐸) < +∞, последовательность {𝑓𝑛 } равномерно ограничена на 𝐸 и 𝑓𝑛 (𝑥) → 𝑓 (𝑥)
на 𝐸, то выполняется (84).
Равномерно ограниченные сходящиеся последовательности часто называют ограниченно сходящимися.
239
Сравнение с интегралом Римана
Наша следующая теорема показывает, что каждая функция, интегрируемая по Риману на некотором сегменте, интегрируема на этом сегменте и по Лебегу, и что функции, интегрируемые по
Риману, подчиняются довольно ограничительным условиям непрерывности. Теория Лебега позволяет интегрировать функции гораздо более широкого класса. Однако самое значительное ее
преимущество состоит, вероятно, в той свободе, с которой в интегралах Лебега оказывается возможным производить операции предельного перехода; с этой точки зрения теоремы о сходимости
составляют суть лебеговской теории.
Одна из трудностей, встречающихся в теории Римана, заключается в том, что предел последовательности функций, интегрируемых по Риману (или даже непрерывных), может уже не быть
интегрируемым по Риману. В теории Лебега эта трудность почти исключается, так как предел
последовательности измеримых функций снова является измеримой функцией.
Пусть пространством с мерой 𝑋 служит сегмент [𝑎, 𝑏] вещественной прямой с 𝜇 = 𝑚 (мера
Лебега), а 𝔐 — семейство измеримых по Лебегу подмножеств сегмента [𝑎, 𝑏]. Вместо
∫
𝑋
𝑓 𝑑𝑚
для интеграла Лебега функции 𝑓 по сегменту [𝑎, 𝑏] принято употреблять привычное обозначение
𝑏
∫
𝑎
𝑓 𝑑𝑥.
Чтобы отличить интеграл Лебега от интеграла Римана, мы будем этот последний обозначать как
𝑏
ℛ
∫
𝑎
𝑓 𝑑𝑥.
11.33 Теорема.
(a) Если 𝑓 ∈ ℛ на [𝑎, 𝑏], то 𝑓 ∈ ℒ на [𝑎, 𝑏] и
𝑏
(87)
∫
𝑎
𝑏
𝑓 𝑑𝑥 = ℛ
∫
𝑎
𝑓 𝑑𝑥.
(b) Пусть 𝑓 ограничена на [𝑎, 𝑏]. Тогда 𝑓 ∈ ℛ на [𝑎, 𝑏] тогда и только тогда, когда 𝑓 непрерывна
почти всюду на [𝑎, 𝑏].
Доказательство. Пусть 𝑓 ограничена. По определению 6.1 и теореме 6.4 существует последовательность {𝑃𝑘 } разбиений сегмента [𝑎, 𝑏], такая, что 𝑃𝑘+1 — измельчение разбиения 𝑃𝑘 , что расстояние между соседними точками разбиения 𝑃𝑘 менее 1/𝑘, и что
(88)
lim 𝐿(𝑃𝑘 , 𝑓 ) = ℛ
𝑘→∞
⨜
lim 𝑈 (𝑃𝑘 , 𝑓 ) = ℛ
𝑓 𝑑𝑥,
𝑘→∞
⨛
𝑓 𝑑𝑥
(в данном доказательстве все интегралы берутся по [𝑎, 𝑏]).
Если 𝑃𝑘 = {𝑥0 , 𝑥1 , … , 𝑥𝑛 } с 𝑥0 = 𝑎, 𝑥𝑛 = 𝑏, положим
𝑈𝑘 (𝑎) = 𝐿𝑘 (𝑎) = 𝑓 (𝑎);
положим 𝑈𝑘 (𝑥) = 𝑀𝑖 и 𝐿𝑘 (𝑥) = 𝑚𝑖 при 𝑥𝑖−1 < 𝑥 ≤ 𝑥𝑖 , 1 ≤ 𝑖 ≤ 𝑛 (обозначения те же, что и в п. 6.1).
Тогда
(89)
𝐿(𝑃𝑘 , 𝑓 ) =
∫
𝐿𝑘 𝑑𝑥,
𝑈 (𝑃𝑘 , 𝑓 ) =
∫
𝑈𝑘 𝑑𝑥
и
(90)
𝐿1 (𝑥) ≤ 𝐿2 (𝑥) ≤ ⋯ ≤ 𝑓 (𝑥) ≤ ⋯ ≤ 𝑈2 (𝑥) ≤ 𝑈1 (𝑥)
240
для всех 𝑥 ∈ [𝑎, 𝑏], поскольку 𝑃𝑘+1 — измельчение разбиения 𝑃𝑘 . В силу (90) существуют
(91)
𝐿(𝑥) = lim 𝐿𝑘 (𝑥),
𝑈 (𝑥) = lim 𝑈𝑘 (𝑥).
𝑘→∞
𝑘→∞
Заметим, что 𝐿 и 𝑈 — ограниченные измеримые функции на [𝑎, 𝑏], что
(92)
𝐿(𝑥) ≤ 𝑓 (𝑥) ≤ 𝑈 (𝑥)
(𝑎 ≤ 𝑥 ≤ 𝑏)
и что
(93)
∫
𝐿 𝑑𝑥 = ℛ
⨜
𝑓 𝑑𝑥,
∫
𝑈 𝑑𝑥 = ℛ
⨛
𝑓 𝑑𝑥
в силу (88), (90) и теоремы о монотонной сходимости.
До этого момента относительно 𝑓 не делалось никаких предположений, кроме того, что 𝑓 —
ограниченная вещественная функция на [𝑎, 𝑏].
Чтобы завершить доказательство, отметим, что 𝑓 ∈ ℛ тогда и только тогда, когда верхний и
нижний интегралы Римана совпадают, то есть тогда и только тогда, когда
(94)
∫
𝐿 𝑑𝑥 =
∫
𝑈 𝑑𝑥;
ввиду того, чо 𝐿 ≤ 𝑈 , (94) выполняется тогда и только тогда, когда 𝐿(𝑥) = 𝑈 (𝑥) для почти всех
𝑥 ∈ [𝑎, 𝑏] (упражнение 1).
В этом случае (92) показывает, что
(95)
𝐿(𝑥) = 𝑓 (𝑥) = 𝑈 (𝑥)
почти всюду на [𝑎, 𝑏], так что 𝑓 измерима, и (87) следует из (93) и (95).
Кроме того, если 𝑥 не принадлежит никакому из 𝑃𝑘 , то легко видеть, что 𝑈 (𝑥) = 𝐿(𝑥) тогда и
только тогда, когда 𝑓 непрерывна в точке 𝑥. Так как объединение множеств 𝑃𝑘 счетно, то его мера
равна 0, и мы делаем вывод, что 𝑓 непрерывна почти всюду на [𝑎, 𝑏] тогда и только тогда, когда
𝐿(𝑥) = 𝑈 (𝑥) почти всюду, то есть (как мы видели выше) тогда и только тогда, когда 𝑓 ∈ ℛ.
Доказательство закончено.
Известное соотношение между интегрированием и дифференцированием в большой степени
переносится в лебеговскую теорию. Если 𝑓 ∈ ℒ на [𝑎, 𝑏] и
𝑥
(96)
𝐹 (𝑥) =
∫
𝑎
𝑓 𝑑𝑡
(𝑎 ≤ 𝑥 ≤ 𝑏),
то 𝐹 ′ (𝑥) = 𝑓 (𝑥) почти всюду на [𝑎, 𝑏].
Обратно, если 𝐹 дифференцируема в каждой точке сегмента [𝑎, 𝑏] («почти всюду» здесь недостаточно!) и если 𝐹 ′ ∈ ℒ на [𝑎, 𝑏], то
𝑥
𝐹 (𝑥) − 𝐹 (𝑎) =
∫
𝑎
𝐹 ′ (𝑡)
(𝑎 ≤ 𝑥 ≤ 𝑏).
Доказательства этих двух теорем можно найти в любой из книг по теории интеграла из списка
литературы.
Интегрирование комплексных функций
Пусть 𝑓 — комплекснозначная функция, определенная на пространстве с мерой 𝑋, и 𝑓 = 𝑢 + 𝑖𝑣, где
𝑢 и 𝑣 вещественны. Мы будем говорить, что 𝑓 измерима, если обе функции 𝑢 и 𝑣 измеримы.
Легко проверить, что суммы и произведения комплексных измеримых функций снова измеримы. Так как
|𝑓 | = (𝑢2 + 𝑣2 )1/2 ,
из теоремы 11.18 следует, что |𝑓 | измерима для любой комплексной измеримой функции 𝑓 .
241
Допустим, что 𝜇 — мера на 𝑋, 𝐸 — измеримое подмножество 𝑋, а 𝑓 — комплексная функция
на 𝑋. Мы будем говорить, что 𝑓 ∈ ℒ (𝜇) на 𝐸, если 𝑓 измерима и
(97)
∫
𝐸
|𝑓 | 𝑑𝜇 < +∞,
при этом мы полагаем по определению
∫
𝐸
𝑓 𝑑𝜇 =
∫
𝐸
𝑢 𝑑𝜇 + 𝑖
𝑣 𝑑𝜇,
∫
𝐸
если выполнено (97). Так как |𝑢| ≤ |𝑓 |, |𝑣| ≤ |𝑓 | и |𝑓 | ≤ |𝑢| + |𝑣|, то ясно, что (97) выполняется
тогда и только тогда, когда 𝑢 ∈ ℒ (𝜇) и 𝑣 ∈ ℒ (𝜇) на 𝐸.
Теоремы 11.23(a), (d), (e), (f), 11.24(b), 11.26, 11.27, 11.29 и 11.32 могут быть перенесены на интегралы Лебега от комплексных функций. Доказательства совсем просты, и только доказательство
теоремы 11.26 представляет некоторый интерес. Вот оно.
Если 𝑓 ∈ ℒ (𝜇) на 𝐸, то существует комплексное число 𝑐, |𝑐| = 1, такое, что
𝑐
∫
𝐸
𝑓 𝑑𝜇 ≥ 0.
Положим 𝑔 = 𝑐𝑓 = 𝑢 + 𝑖𝑣, 𝑢 и 𝑣 вещественны. Тогда
|∫
𝐸
𝑓 𝑑𝜇 = 𝑐
𝑓 𝑑𝜇 =
𝑔 𝑑𝜇 =
𝑢 𝑑𝜇 ≤
|𝑓 | 𝑑𝜇.
|
∫
∫
∫
∫
𝐸
𝐸
𝐸
𝐸
Третье из вышеприведенных равенств выполняется, так как из предыдущих равенств следует, что
∫ 𝑔 𝑑𝜇 вещественен.
Функции класса ℒ 2
В качестве приложения теории Лебега мы изложим обобщение теоремы Парсеваля (которую мы
доказали в гл. 8 лишь для функций, интегрируемых по Риману) и докажем теорему Рисса-Фишера
для ортонормальных систем функций.
11.34 Определение. Пусть 𝑋 — измеримое пространство. Мы будем говорить, что комплексная
функция 𝑓 ∈ ℒ 2 (𝜇) на 𝑋, если 𝑓 измерима и если
∫
𝑋
|𝑓 |2 𝑑𝜇 < +∞.
Если 𝜇 — мера Лебега, то мы будем писать 𝑓 ∈ ℒ 2 . Если 𝑓 ∈ ℒ 2 (𝜇) (начиная с этого места мы
будем опускать слова «на 𝑋»), то мы полагаем по определению
1/2
‖𝑓 ‖ =
{∫
𝑋
|𝑓 |2 𝑑𝜇
}
и называем ‖𝑓 ‖ ℒ 2 (𝜇)-нормой функции 𝑓 .
11.35 Теорема. Пусть 𝑓 ∈ ℒ 2 (𝜇) и 𝑔 ∈ ℒ 2 (𝜇). Тогда 𝑓 𝑔 ∈ ℒ (𝜇) и
(98)
∫
𝑋
|𝑓 𝑔| 𝑑𝜇 ≤ ‖𝑓 ‖‖𝑔‖.
Это неравенство Шварца, которое мы уже встречали в случае рядов и интеграла Римана. Оно
вытекает из неравенства
0≤
∫
𝑋
(|𝑓 | + 𝜆|𝑔|)2 𝑑𝜇 = ‖𝑓 ‖2 + 2𝜆
которое выполняется при всяком вещественном 𝜆.
242
∫
𝑋
|𝑓 𝑔| 𝑑𝜇 + 𝜆2 ‖𝑔‖2 ,
11.36 Теорема. Если 𝑓 ∈ ℒ 2 (𝜇) и 𝑔 ∈ ℒ 2 (𝜇), то 𝑓 + 𝑔 ∈ ℒ 2 (𝜇) и
‖𝑓 + 𝑔‖ ≤ ‖𝑓 ‖ + ‖𝑔‖.
Доказательство. Неравенство Шварца показывает, что
‖𝑓 + 𝑔‖2 =
∫
|𝑓 |2 +
∫
𝑓 𝑔̄ +
∫
𝑓 𝑔̄ +
∫
|𝑔|2
≤ ‖𝑓 ‖2 + 2‖𝑓 ‖‖𝑔‖ + ‖𝑔‖2
= (‖𝑓 ‖ + ‖𝑔‖)2 .
11.37 Замечание. Если определить расстояние между двумя функциями 𝑓 и 𝑔 в ℒ 2 (𝜇) равным
‖𝑓 − 𝑔‖, то мы видим, что условия определения 2.15 выполняются, кроме того, что из ‖𝑓 − 𝑔‖ = 0
не следует, что 𝑓 (𝑥) = 𝑔(𝑥) при всех 𝑥, а следует только при почти всех 𝑥. Таким образом, если
мы отождествим функции, отличающиеся только на множестве меры нуль, то ℒ 2 (𝜇) оказывается
метрическим пространством.
Рассмотрим теперь ℒ 2 на сегменте вещественной прямой с мерой Лебега.
11.38 Теорема. Непрерывные функции образуют всюду плотное подмножество пространства
ℒ 2 на [𝑎, 𝑏].
Точнее, это значит, что для любой 𝑓 ∈ ℒ 2 на [𝑎, 𝑏] и любого 𝜀 > 0 существует функция 𝑔,
непрерывная на [𝑎, 𝑏], такая, что
1/2
𝑏
‖𝑓 − 𝑔‖ =
|𝑓 − 𝑔|2 𝑑𝑥
{∫
𝑎
< 𝜀.
}
Доказательство. Мы будем говорить, что 𝑓 аппроксимируется в ℒ 2 последовательностью {𝑔𝑛 },
если ‖𝑓 − 𝑔𝑛 ‖ → 0 при 𝑛 → ∞.
Пусть 𝐴 — замкнутое подмножество сегмента [𝑎, 𝑏], а 𝐾𝐴 — его характеристическая функция.
Положим
𝑡(𝑥) = inf |𝑥 − 𝑦|
(𝑦 ∈ 𝐴)
и
𝑔𝑛 (𝑥) =
1
1 + 𝑛𝑡(𝑥)
(𝑛 = 1, 2, 3, … ).
Тогда 𝑔𝑛 непрерывна на [𝑎, 𝑏], 𝑔𝑛 (𝑥) = 1 на 𝐴 и 𝑔𝑛 (𝑥) → 0 на 𝐵, где 𝐵 = [𝑎, 𝑏] − 𝐴. Значит,
1/2
‖𝑔𝑛 − 𝐾𝐴 ‖ =
{∫
𝐵
𝑔𝑛2 𝑑𝑥
}
→0
по теореме 11.32. Итак, характеристические функции замкнутых множеств можно аппроксимировать в ℒ 2 непрерывными функциями.
Согласно (39), то же верно и в отношении характеристической функции любого измеримого
множества и, стало быть, для простых измеримых функций.
Если 𝑓 ≥ 0 и 𝑓 ∈ ℒ 2 , то пусть {𝑠𝑛 } — монотонно возрастающая последовательность простых
неотрицательных измеримых функций, такая, что 𝑠𝑛 (𝑥) → 𝑓 (𝑥). Ввиду того что |𝑓 − 𝑠𝑛 |2 ≤ 𝑓 2 ,
теорема 11.32 показывает, что ‖𝑓 − 𝑠𝑛 ‖ → 0.
Отсюда следует утвержденгие теоремы и в общем случае.
11.39 Определение. Мы будем говорить, что последовательность комплексных функций {𝜑𝑛 } —
ортонормальная система функций на измеримом пространстве 𝑋, если
∫
𝑋
𝜑𝑛 𝜑̄ 𝑚 𝑑𝜇 =
0
{1
(𝑛 ≠ 𝑚),
(𝑛 = 𝑚).
В частности, должно выполняться 𝜑𝑛 ∈ ℒ 2 (𝜇). Если 𝑓 ∈ ℒ 2 (𝜇) и если
𝑐𝑛 =
∫
𝑋
𝑓 𝜑̄ 𝑛 𝑑𝜇
(𝑛 = 1, 2, 3, … ),
243
то мы будем писать
∞
𝑓∼
∑
𝑐𝑛 𝜑𝑛 ,
𝑛=1
как в определении 8.10.
Определение тригонометрического ряда Фурье аналогично распространяется на ℒ 2 (или даже
на ℒ ) на [−𝜋, 𝜋]. Теоремы 8.11 и 8.12 (неравенство Бесселя) верны для любой 𝑓 ∈ ℒ 2 (𝜇). Доказательства дословно те же.
Теперь мы можем доказать теорему Парсеваля.
11.40 Теорема. Пусть
∞
(99)
𝑓 (𝑥) ∼
∑
𝑐𝑛 𝑒𝑖𝑛𝑥 ,
−∞
где 𝑓 ∈ ℒ 2 на [−𝜋, 𝜋]. Пусть 𝑠𝑛 — 𝑛-я частная сумма ряда (99). Тогда
(100)
lim ‖𝑓 − 𝑠𝑛 ‖ = 0,
𝑛→∞
∞
(101)
∑
|𝑐𝑛 |2 =
−∞
𝜋
1
|𝑓 |2 𝑑𝑥.
2𝜋 ∫
−𝜋
Доказательство. Пусть 𝜀 > 0. По теореме 11.38 существует непрерывная функция 𝑔, такая, что
‖𝑓 − 𝑔‖ <
𝜀
.
2
Легко видеть, что мы можем подобрать ее так, чтобы 𝑔(𝜋) = 𝑔(−𝜋). Тогда 𝑔 можно продолжить до
периодической непрерывной функции. По теореме 8.16 существует тригонометрический многочлен
𝑇 степени 𝑁, такой, что
𝜀
‖𝑔 − 𝑇 ‖ < .
2
Значит, по теореме 8.11 (расширенной на ℒ 2 ) при 𝑛 ≥ 𝑁 имеем
‖𝑠𝑛 − 𝑓 ‖ ≤ ‖𝑇 − 𝑓 ‖ < 𝜀,
откуда и следует (100). Равенство (101) можно вывести из (100) так же, как при доказательстве
теоремы 8.16.
Следствие. Если 𝑓 ∈ ℒ 2 на [−𝜋, 𝜋] и если
𝜋
∫
−𝜋
𝑓 (𝑥)𝑒−𝑖𝑛𝑥 𝑑𝑥 = 0
(𝑛 = 0, ±1, ±2, … ),
то ‖𝑓 ‖ = 0.
Таким образом, если две функции в ℒ 2 имеют одинаковые ряды Фурье, то они отличаются
самое большее на множестве меры нуль.
11.41 Определение. Пусть 𝑓 и 𝑓𝑛 ∈ ℒ 2 (𝜇) (𝑛 = 1, 2, 3, … ). Будем говорить, что последовательность {𝑓𝑛 } сходится к 𝑓 в ℒ 2 (𝜇), если ‖𝑓𝑛 −𝑓 ‖ → 0. Будем говорить, что {𝑓𝑛 } — последовательность
Коши в ℒ 2 (𝜇), если для любого 𝜀 > 0 существует целое 𝑁, такое, что из 𝑛 ≥ 𝑁, 𝑚 ≥ 𝑁 следует
‖𝑓𝑛 − 𝑓𝑚 ‖ ≤ 𝜀.
11.42 Теорема. Если {𝑓𝑛 } — последовательность Коши в ℒ 2 (𝜇), то существует функция 𝑓 ∈
ℒ 2 (𝜇), такая, что {𝑓𝑛 } сходится к 𝑓 в ℒ 2 (𝜇).
Другими словами, ℒ 2 (𝜇) — полное метрическое пространство.
244
Доказательство. Поскольку {𝑓𝑛 } — последовательность Коши, то мы можем найти такую последовательность {𝑛𝑘 }, 𝑘 = 1, 2, 3, … , что
‖𝑓𝑛𝑘 − 𝑓𝑛𝑘+1 ‖ <
1
2𝑘
(𝑘 = 1, 2, 3, … ).
Выберем функцию 𝑔 ∈ ℒ 2 (𝜇). В силу неравенства Шварца
∫
𝑋
|𝑔(𝑓𝑛𝑘 − 𝑓𝑛𝑘+1 )| 𝑑𝜇 ≤
‖𝑔‖
.
2𝑘
Значит,
∞
(102)
∑∫
𝑋
𝑘=1
|𝑔(𝑓𝑛𝑘 − 𝑓𝑛𝑘+1 )| 𝑑𝜇 ≤ ‖𝑔‖.
По теореме 11.30 мы можем поменять местами суммирование и интегрирование в (102). Следовательно,
∞
(103)
|𝑔(𝑥)|
|𝑓𝑛𝑘 (𝑥) − 𝑓𝑛𝑘+1 (𝑥)| < +∞
∑
𝑘=1
почти всюду на 𝑋. Поэтому
∞
(104)
∑
𝑘=1
|𝑓𝑛𝑘+1 (𝑥) − 𝑓𝑛𝑘 (𝑥)| < +∞
почти всюду на 𝑋. Действительно, если бы ряд (104) расходился на множестве 𝐸 положительной
меры, то мы могли бы выбрать 𝑔(𝑥) отличной от нуля на подмножестве множества 𝐸 положительной
меры и прийти к противоречию с (103).
Поскольку 𝑘-я частная сумма ряда
∞
∑
(𝑓𝑛𝑘+1 (𝑥) − 𝑓𝑛𝑘 (𝑥)),
𝑘=1
сходящегося почти всюду на 𝑋, равна
𝑓𝑛𝑘+1 (𝑥) − 𝑓𝑛1 (𝑥),
то равенство
𝑓 (𝑥) = lim 𝑓𝑛𝑘 (𝑥)
𝑘→∞
опредлеляет 𝑓 (𝑥) для почти всех 𝑥 ∈ 𝑋, и неважно, как мы определим 𝑓 (𝑥) в остальных точках
множества 𝑋.
Теперь мы покажем, что функция 𝑓 обладает нужными свойствами. Пусть 𝜀 > 0. Возьмем 𝑁,
указанное в определении 11.41. Если 𝑛𝑘 > 𝑁, то теорема Фату показывает, что
‖𝑓 − 𝑓𝑛𝑘 ‖ ≤ lim ‖𝑓𝑛𝑖 − 𝑓𝑛𝑘 ‖ ≤ 𝜀.
𝑖→∞
Таким образом, 𝑓 − 𝑓𝑛𝑘 ∈ ℒ 2 (𝜇), а так как 𝑓 = (𝑓 − 𝑓𝑛𝑘 ) + 𝑓𝑛𝑘 , то 𝑓 ∈ ℒ 2 (𝜇). Кроме того, ввиду того
что 𝜀 произвольно,
lim ‖𝑓 − 𝑓𝑛𝑘 ‖ = 0.
𝑘→∞
Наконец, из неравенства
(105)
‖𝑓 − 𝑓𝑛 ‖ ≤ ‖𝑓 − 𝑓𝑛𝑘 ‖ + ‖𝑓𝑛𝑘 − 𝑓𝑛 ‖
следует, что последовательность {𝑓𝑛 } сходится к 𝑓 в ℒ 2 (𝜇); действительно, выбирая 𝑛 и 𝑛𝑘 достаточно большими, мы можем сделать оба слагаемых в правой части неравенства (105) сколь угодно
малыми.
245
11.43 Теорема Рисса-Фишера. Пусть {𝜑𝑛 } — ортонормальная система на 𝑋. Допустим, что
ряд ∑ |𝑐𝑛 |2 сходится, и положим 𝑠𝑛 = 𝑐1 𝜑1 + ⋯ + 𝑐𝑛 𝜑𝑛 . Тогда существует функция 𝑓 ∈ ℒ 2 (𝜇),
такая, что {𝑠𝑛 } сходится к 𝑓 в ℒ 2 (𝜇), причем
∞
𝑓∼
∑
𝑐𝑛 𝜑𝑛 .
𝑛=1
Доказательство. Если 𝑛 > 𝑚, то
‖𝑠𝑛 − 𝑠𝑚 ‖2 = |𝑐𝑚+1 |2 + ⋯ + |𝑐𝑛 |2 ,
так что {𝑠𝑛 } — последовательность Коши в ℒ 2 (𝜇). По теореме 11.42 существует функция 𝑓 ∈ ℒ 2 (𝜇),
такая, что
lim ‖𝑓 − 𝑠𝑛 ‖ = 0.
𝑛→∞
Теперь при 𝑛 > 𝑘
∫
𝑋
𝑓 𝜑̄ 𝑘 𝑑𝜇 − 𝑐𝑘 =
∫
𝑋
𝑓 𝜑̄ 𝑘 𝑑𝜇 −
∫
𝑋
𝑠𝑛 𝜑̄ 𝑘 𝑑𝜇,
так что
|∫
𝑋
𝑓 𝜑̄ 𝑘 𝑑𝜇 − 𝑐𝑘 ≤ ‖𝑓 − 𝑠𝑛 ‖ ⋅ ‖𝜑𝑘 ‖ + ‖𝑓 − 𝑠𝑛 ‖.
|
Полагая 𝑛 → ∞, получаем
𝑐𝑘 =
∫
𝑋
𝑓 𝜑̄ 𝑘 𝑑𝜇
(𝑘 = 1, 2, 3, … ),
и доказательство закончено.
11.44 Определение. Ортонормальная система {𝜑𝑛 } называется полной, если для 𝑓 ∈ ℒ 2 (𝜇) из
того, что
∫
𝑋
𝑓 𝜑̄ 𝑛 𝑑𝜇 = 0
(𝑛 = 1, 2, 3, … )
следует, что ‖𝑓 ‖ = 0.
В следствии к теореме 11.40 мы показали полноту тригонометрической системы на основании
равенства Парсеваля (101). Обратно, равенство Парсеваля выполняется для любой полной ортонормальной системы.
11.45 Теорема. Пусть {𝜑𝑛 } — полная ортонормальная система. Если 𝑓 ∈ ℒ 2 (𝜇) и если
∞
(106)
𝑓∼
∑
𝑐𝑛 𝜑𝑛 ,
𝑛=1
то
∞
(107)
∫
𝑋
|𝑓 |2 𝑑𝜇 =
∑
|𝑐𝑛 |2 .
𝑛=1
Доказательство. Из неравенства Бесселя следует, что ряд ∑ |𝑐𝑛 |2 сходится. Положим
𝑠𝑛 = 𝑐1 𝜑1 + ⋯ + 𝑐𝑛 𝜑𝑛 .
В силу теоремы Рисса-Фишера существует функция 𝑔 ∈ ℒ 2 (𝜇), такая, что
∞
(108)
𝑔∼
∑
𝑐𝑛 𝜑𝑛
𝑛=1
и ‖𝑔 − 𝑠𝑛 ‖ → 0. Значит, ‖𝑠𝑛 ‖ → ‖𝑔‖. Поскольку
‖𝑠𝑛 ‖2 = |𝑐1 |2 + ⋯ + |𝑐𝑛 |2 ,
246
то
∞
(109)
∫
𝑋
|𝑔|2 𝑑𝜇 =
∑
|𝑐𝑛 |2 .
𝑛=1
Теперь из (106), (108) и полноты системы {𝜑𝑛 } следует, что ‖𝑓 − 𝑔‖ = 0, так что из (109) следует
(107).
Комбинируя теоремы 11.43 и 11.45, мы приходим к очень интересному выводу: каждая полная
ортонормальная система порождает взаимно однозначное соответствие между функциями 𝑓 ∈
ℒ 2 (𝜇) (функции, совпадающие почти всюду, отождествляются) и последовательностями {𝑐𝑛 }, для
которых сходится ряд ∑ |𝑐𝑛 |2 . Представление
∞
𝑓∼
∑
𝑐𝑛 𝜑𝑛
𝑛=1
и равенство Парсеваля показывают, что ℒ 2 (𝜇) можно рассматривать как бесконечномерное евклидово пространство (так называемое «гильбертово пространство»), в котором точка 𝑓 имеет
координаты 𝑐𝑛 , а функции 𝜑𝑛 служат координатными векторами.
Упражнения
1. Пусть 𝑓 ≥ 0 и ∫𝐸 𝑓 𝑑𝜇 = 0. Доказать, что 𝑓 (𝑥) = 0 почти всюду на 𝐸. Указание. Пусть 𝐸𝑛 — подмножество
множества 𝐸, на котором 𝑓 (𝑥) > 1/𝑛. Положим 𝐴 = ⋃ 𝐸𝑛 . Тогда 𝜇(𝐴) = 0 в том и только в том случае,
когда 𝜇(𝐸𝑛 ) = 0 при всех 𝑛.
2. Если ∫𝐴 𝑓 𝑑𝜇 = 0 для всякого измеримого подмножества 𝐴 измеримого множества 𝐸, то 𝑓 (𝑥) = 0 почти
всюду на 𝐸.
3. Пусть {𝑓𝑛 } — последовательность измеримых функций. Доказать, что множество точек 𝑥, в которых
{𝑓𝑛 (𝑥)} сходится, измеримо.
4. Если 𝑓 ∈ ℒ (𝜇) на 𝐸, а 𝑔 ограничена и измерима на 𝐸, то 𝑓 𝑔 ∈ ℒ (𝜇) на 𝐸.
5. Положим
𝑔(𝑥) =
0 (0 ≤ 𝑥 ≤ 12 ),
{1 ( 12 < 𝑥 ≤ 1),
𝑓2𝑘 (𝑥) = 𝑔(𝑥)
(0 ≤ 𝑥 ≤ 1),
𝑓2𝑘+1 (𝑥) = 𝑔(1 − 𝑥)
(0 ≤ 𝑥 ≤ 1).
Показать, что
lim 𝑓𝑛 (𝑥) = 0
(0 ≤ 𝑥 ≤ 1),
𝑛→∞
но
1
𝑓𝑛 (𝑥) 𝑑𝑥 =
∫
0
1
2
(ср. с (77)).
6. Пусть
𝑓𝑛 (𝑥) =
1
𝑛
(|𝑥| ≤ 𝑛),
{0
(|𝑥| > 𝑛).
Тогда 𝑓𝑛 (𝑥) → 0 равномерно на 𝑅1 , но
∞
∫
−∞
𝑓𝑛 𝑑𝑥 = 2
∞
(𝑛 = 1, 2, 3, … )
(мы пишем ∫−∞ вместо ∫𝑅1 ). Таким образом, из равномерной сходимости не следует ограниченная сходимость в смысле теоремы 11.32. Однако на множествах конечной меры равномерно сходящиеся последовательности ограниченных функций удовлетворяют теореме 11.32.
7. Найти необходимое и достаточное условие для того, чтобы 𝑓 ∈ ℛ(𝛼) на [𝑎, 𝑏]. Указание. Рассмотреть
пример 11.6(b) и теорему 11.33.
𝑥
8. Пусть 𝑓 ∈ ℛ на [𝑎, 𝑏] и 𝐹 (𝑥) = ∫𝑎 𝑓 (𝑡) 𝑑𝑡. Доказать, что 𝐹 ′ (𝑥) = 𝑓 (𝑥) почти всюду на [𝑎, 𝑏].
247
9. Доказать, что функция 𝐹 , задаваемая равенством (96), непрерывна на [𝑎, 𝑏].
10. Пусть 𝜇(𝑋) < +∞ и 𝑓 ∈ ℒ 2 (𝜇) на 𝑋. Доказать, что 𝑓 ∈ ℒ (𝜇) на 𝑋. Если
𝜇(𝑋) = +∞,
то это неверно. Например, если
𝑓 (𝑥) =
1
,
1 + |𝑥|
то 𝑓 ∈ ℒ 2 на 𝑅1 , но 𝑓 ∉ ℒ на 𝑅1 .
11. Пусть 𝑓 , 𝑔 ∈ ℒ (𝜇) на 𝑋. Определим расстояние между 𝑓 и 𝑔 как
∫
𝑋
|𝑓 − 𝑔| 𝑑𝜇.
Доказать, что ℒ (𝜇) — полное метрическое пространство.
12. Пусть
(a) |𝑓 (𝑥, 𝑦)| ≤ 1 при 0 ≤ 𝑥 ≤ 1, 0 ≤ 𝑦 ≤ 1,
(b) при фиксированном 𝑥 функция 𝑓 (𝑥, 𝑦) непрерывна по 𝑦,
(c) при фиксированном 𝑦 функция 𝑓 (𝑥, 𝑦) непрерывна по 𝑥.
Положим
1
𝑔(𝑥) =
𝑓 (𝑥, 𝑦) 𝑑𝑦
∫
0
(0 ≤ 𝑥 ≤ 1).
Непрерывна ли 𝑔?
13. Рассмотрим функции
𝑓𝑛 (𝑥) = sin 𝑛𝑥
(𝑛 = 1, 2, 3, … , −𝜋 ≤ 𝑥 ≤ 𝜋)
как точки пространства ℒ 2 . Доказать, что множество этих точек замкнуто и ограничено, но не компактно.
14. Доказать, что комплексная функция 𝑓 измерима тогда и только тогда, когда 𝑓 −1 (𝑉 ) измеримо для всех
открытых множеств 𝑉 на комплексной плоскости.
15. Пусть ℛ — кольцо всех элементарных подмножеств промежутка (0, 1]. Если 0 < 𝑎 ≤ 𝑏 ≤ 1, то положим
𝜑([𝑎, 𝑏]) = 𝜑([𝑎, 𝑏)) = 𝜑((𝑎, 𝑏]) = 𝜑((𝑎, 𝑏)) = 𝑏 − 𝑎,
но
𝜑((0, 𝑏)) = 𝜑((0, 𝑏]) = 1 + 𝑏,
если 0 < 𝑏 ≤ 1. Показать, что этим определена аддитивная функция множества 𝜑 на ℛ, которая не
регулярна и не может быть продолжена до функции, счетно-аддитивной на 𝜎-кольце.
16. Пусть {𝑛𝑘 } — возрастающая последовательность положительных целых чисел, а 𝐸 — множество всех
точек 𝑥 ∈ (−𝜋, 𝜋), в которых сходится последовательность {sin 𝑛𝑘 𝑥}. Доказать, что 𝑚(𝐸) = 0. Указание.
При любом 𝐴 ⊂ 𝐸
∫
𝐴
sin 𝑛𝑘 𝑥 𝑑𝑥 → 0
и
2
(sin 𝑛𝑘 𝑥)2 𝑑𝑥 = (1 − cos 2𝑛𝑘 𝑥) 𝑑𝑥 → 𝑚(𝐴)
∫
∫
𝐴
𝐴
при 𝑘 → ∞.
17. Допустим, что 𝐸 ⊂ (−𝜋, 𝜋), 𝑚(𝐸) > 0, 𝛿 > 0. Воспользоваться неравенством Бесселя для доказательства
того, что имеется не более чем конечное число таких целых 𝑛, что sin 𝑛𝑥 ≥ 𝛿 при всех 𝑥 ∈ 𝐸.
18. Пусть 𝑓 ∈ ℒ 2 (𝜇), 𝑔 ∈ ℒ 2 (𝜇). Доказать, что
2
|∫
𝑓 𝑔 ̄ 𝑑𝜇
|
=
∫
|𝑓 |2 𝑑𝜇
∫
|𝑔|2 𝑑𝜇
тогда и только тогда, когда существует такое число 𝑐, что 𝑔(𝑥) = 𝑐𝑓 (𝑥) почти всюду (ср. с теоремой
11.35).
248
Глава 1
Системы вещественных и
комплексных чисел
1.1 Допустим, что r + x рационально. Тогда x = (r + x) − r тоже рационально — противоречие.
Допустим, что rx рационально. Тогда x = (rx)/r тоже рационально — противоречие.
1.2 Пусть (m/n)2 = 12, где m и n целые, причем хотя бы одно из них не делится на 3. Избавляясь от
дробей, получаем
m2 = 12n2 .
Значит, m2 делится на 3. В свою очередь m тоже делится на 3, значит, m2 делится на 9. Значит,
правая часть тоже должна делиться на 9, что означает, что n2 делится на 3, то есть n делится на 3.
Противоречие.
1.3 Из xy = xz с помощью аксиом (M) следует
y = 1y = ((1/x) · x)y = (1/x)(xy)
= (1/x)(xz) = ((1/x) · x)z = 1z = z.
Тем самым доказано (a). Чтобы получить (b), положим z = 1 в (a). Чтобы получить (c), положим
z = 1/x в (a).
Так как (1/x) · x = 1, из (c) подстановкой 1/x вместо x получается (d).
1.4 Выберем x ∈ E (это возможно, так как E непусто). Так как α — нижняя граница E, имеем α ≤ x,
аналогично x ≤ β. Соединяя неравенства, получаем α ≤ x ≤ β.
1.5 Обозначим α = − sup(−A) (sup(−A) существует, так как −A непусто и ограничено сверху). Мы
должны доказать, что α = inf A. Проверим определение inf:
Пусть x ∈ A. Тогда −x ∈ −A, значит, −x ≤ sup(−A), следовательно, x ≥ − sup(−A), т.е. α ≤ x. Это
означает, что α является нижней границей A.
Пусть β > α. Тогда −β < sup(−A), то есть −β не является верхней границей −A, что означает
существование x ∈ −A, такого, что x > −β. Тогда −x ∈ A и −x < β, то есть β не является нижней
границей A.
1.6
1
(a) Обозначим α = (bm )1/n , β = (bp )1/q . По определению αn = bm , β q = bp . Возводя первое равенство
в степень p и второе в степень m, получаем αnp = bmp = β mq . Так как m/n = p/q, np = mq и из
единственности корня получаем α = β.
(b) Пусть r = m/n, s = p/q (n > 0, q > 0).
br+s = b(mq+np)/nq
= (bmq+np )1/nq
= (b
(согласно (a))
mq np 1/nq
b )
= (bmq )1/nq (bnp )1/nq
=b
(по следствию из теоремы 1.21)
m/n p/q
(согласно (a))
b
r s
=b b .
(c) Если r > s, то br /bs = br−s > 1, то есть br — верхняя граница B(r). Так как br ∈ B(r), никакое
x < br не является верхней границей B(r). Следовательно, br = sup B(r).
(d) Допустим, что bx+y > bx by . Тогда по определению верхней грани существует рациональное t ≤
x+y, для которого bt > bx by , а по упр. 1.7(c) существует целое n > 0, для которого b1/n < bt /(bx by ).
Обозначим u = t − 1/n, тогда u < x + y и bu > bx by . Выберем рациональное r так, чтобы
u − y < r < x. Тогда u − r < y, bu = br bu−r < bx by — противоречие.
Допустим, что bx by > bx+y . Тогда bx > bx+y /by . Выберем c так, чтобы bx > c > bx+y /by . Тогда
by > bx+y /c, и по определению верхней грани существуют рациональные r ≤ x и s ≤ y, такие, что
br > c и bs > bx+y /c. Тогда br+s = br bs > bx+y — противоречие.
1.7
bn − 1 = (bn−1 + · · · + 1)(b − 1) ≥ n(b − 1).
Подставля b1/n вместо b, получаем b − 1 ≥ n(b1/n − 1).
b − 1 ≥ n(b1/n − 1) > (b − 1)(b1/n − 1)/(t − 1), значит, b1/n − 1 < t − 1, т. е. b1/n < t.
b1/n < y · b−w , значит, bw+1/n < y.
Подставляя в (c) t = y −1 · bw , получаем b1/n < y −1 · bw , следовательно, bw−1/n > y.
Если bx < y, то из (d) следует, что x не является верхней границей A. Если bx > y, то из (e)
следует, что x не является точной верхней границей A. Значит, bx = y.
(g) Пусть x > 0. Выберем рациональное 0 < r < x. По свойству верхней грани bx ≥ br > 1.
Если y > x, то by /bx = by−x > 1.
(a)
(b)
(c)
(d)
(e)
(f)
1.8 По 1.18(d) −1 = i2 > 0, следовательно, по 1.18(a) 1 < 0 — противоречие.
1.9 Проверим свойства порядка из определения 1.5.
(i) Если a < c, то z < w; если a > c, то z > w; если a = c и b < d, то z < w; если a = c и b > d, то
z > w; если же a = c и b = d, то z = w.
(ii) Пусть x = e + f i, z < w и w < x. Очевидно, что a ≤ c ≤ e. Если a < c или c < e, то a < e, и z < x.
В случае же a = c = e мы имеем b < d < f , и снова z < x.
Пусть E — множество комплексных чисел, имеющих отрицательную вещественную часть. Его верхними границами при заданном отношении порядка являются все комплексные числа с неотрицательной
вещественной частью. Среди них нет наименьшего элемента: для любого z, являющегося верхней границей E, z − i также является верхней границей E и предшествует z при заданном отношении порядка.
Следовательно, полученное упорядоченное множество не обладает свойством верхней грани.
2
1.10
z 2 = a2 − b2 + 2abi
=
|w| + u |w| − u
−
+2
2
2
(|w| + u)(|w| − u)
4
1/2
i
= u + (u2 + v 2 − u2 )1/2 i
= u + vi, еслиv ≥ 0.
Аналогично,
z̄ 2 = a2 − b2 − 2abi
|w| + u |w| − u
=
−
−2
2
2
(|w| + u)(|w| − u)
4
1/2
i
= u − (u2 + v 2 − u2 )1/2 i
= u + vi, еслиv ≤ 0.
Итак, каждое ненулевое комплексное число имеет два комплексных квадратных корня.
1.11 Так как |z| = |rw| = r|w| = r, должно быть r = |z|, так что r определяется однозначно.
Если z ̸= 0, то r =
̸ 0, и w = z/r также определяется однозначно. Если же z = 0, w можно выбрать
произвольно, соблюдая условие |w| = 1.
1.12 По индукции: |z1 + · · · + zn + zn+1 | ≤ |z1 + · · · + zn | + |zn+1 | ≤ |z1 | + · · · + |zn | + |zn+1 |.
1.13 Если |x| ≥ |y|, то ||x| − |y|| = |x| − |y| ≤ |x − y|, так как |x| ≤ |x − y| + |y|. Если же |x| ≤ |y|, то
||x| − |y|| = |y| − |x| ≤ |x − y| = |y − x|, так как |y| ≤ |y − x| + |x|.
1.14 |1 + z|2 + |1 − z|2 = 1 + z + z̄ + z z̄ + 1 − z − z̄ + z z̄ = 4.
1.15 Неравенство Шварца обращается в равенство, если
P
|Baj − Cbj |2 = 0, то есть Baj − Cbj =0 для
всех j. Это возможно, во-первых, если B = 0, то есть все bj равны нулю. Во-вторых, если B ̸= 0, это
возможно, если aj = (C/B)bP
j для всех
Pj, то есть a и b пропорциональны друг другу. Обратно, если
aj = Kbj для всех j, то C =
aj b̄j =
Kbj b̄j = KB, и Baj − Cbj = 0 для всех j, то есть неравенство
Шварца обращается в равенство.
1.16 Положим
u=z−
x+y
,
2
v=
y−x
.
2
Тогда исходное равенство принимает вид
|u − v| = |u + v| = r,
где |v| =
d
.
2
По упр. 1.17
2r2 = |u − v|2 + |u + v|2 = 2|u|2 + 2|v|2 = 2|u|2 +
т.е.
r
|u| =
r2 −
3
d2
.
4
d2
,
2
С другой стороны,
u2 − 2u · v + v2 = |u − v|2 = |u + v|2 = u2 + 2u · v + v2 ,
т.е.
u · v = 0.
(a) Без ограничения общности будем считать, что v1 ̸= 0. Для любого λ ∈ R обозначим
r
d2 aλ
aλ = (−v2 − λv3 , v1 , λv1 , 0, · · · , 0),
uλ = r2 −
.
4 |aλ |
Легко проверить, что uλ · v = 0. Из этого следует
|uλ − v| = |uλ + v| =
p
|uλ |2 + |v|2 = r.
Кроме того, при λ ̸= µ коэффициенты aλ и aµ не являются пропорциональными, следовательно,
uλ ̸= uµ при λ ̸= µ. Итак, мы получили бесконечно много решений
x+y
.
2
Если k = 2, то условие u · v = 0 превращается в u1 v1 + u2 v2 = 0. Решая это уравнение совместно
с условием на |u|, получаем два решения:
r
4r2
x+y
u=±
− 1(v2 , −v1 ),
z=u+
.
d2
2
zλ = uλ +
Если k = 1, то u · v = 0 при v ̸= 0 означает, что u = 0, и решений нет.
(b) Если 2r = d, то u = 0, и мы имеем одно решение при любом k:
z=
x+y
.
2
(c) Если 2r < d, то |u| < 0, что невозможно. Решений нет.
1.17
|x + y|2 + |x − y|2 = x · x + 2x · y + y · y + x · x − 2x · y + y · y = 2|x|2 + 2|y|2 .
Сумма квадратов диагоналей параллелограмма равна сумме квадратов его сторон.
1.18 Если x = 0, то y может быть произвольным ненулевым вектором. Иначе без ограничения общности положим x1 ̸= 0. Искомый вектор равен
y = (−x2 , x1 , 0, · · · , 0).
Если k = 1, то x · y ̸= 0 означает, что x ̸= 0 и y ̸= 0, поэтому такое возможно только при x = 0.
1.19 Проведем цепочку равносильных преобразований:
|x − a| = 2|x − b|,
|x| − 2x · a + |a|2 = 4|x|2 − 8x · b + 4|b|2 ,
2
4b − a
·x=
3
2
4b − a
16|b | − 8a · b + |a|2
|x|2 − 2
·x+
=
3
9
4b − a 2
|x −
| =
3
4b − a
|x −
|=
3
|x|2 − 2
4
|a|2 − 4|b|2
,
3
2
2
|a| − 4|b|
16|b|2 − 8a · b + |a|2
+
,
3
9
4|b − a|2
,
9
2|b − a|
.
3
1.20 Шаг 3 и проверка аксиом (A1)–(A3) остаются без изменений, только не надо проверять свой-
ство (III). Обозначим 0∗ множество неположительных рациональных чисел. В проверке аксиомы (A4)
строгие неравенства меняются на нестрогие. Осталось найти нарушение (A5).
Пусть α — множество рациональных чисел, меньших r. Ясно, что α является сечением. Допустим,
что существует −α такое, что α + (−α) = 0∗ . В −α должен существовать элемент q > −r, иначе мы не
сможем получить 0 в виде суммы элементов из α и −α. Значит, −q < r, и мы можем выбрать p ∈ α
так, что −q < p < r. Это означает, что p ∈ α, q ∈ −α, p + q > 0, что невозможно, поскольку p + q
должно принадлежать 0∗ .
5
Глава 2
Элементы топологии
2.1 Отрицанием утверждения 0 ⊂ A является утверждение «существует x ∈ 0, такое, что x ∈/ A»,
очевидным образом неверное.
2.2 При каждом положительном целом N существует не более (2N + 1)N +2 уравнений, удовлетворяющих условию в указании. Обозначим AN множество корней этих уравнений. Оно также конечно,
так как
S∞ каждое из этих уравнений имеет не более N корней. Множество всех алгебраических чисел
A = N =1 AN — счетное объединение конечных множеств и потому не более чем счетно по следствию
из теоремы 2.12.
С другой стороны, все положительные целые числа являются элементами множества A, так как
положительное целое число a является корнем уравнения с целыми коэффициентами x − a = 0. Следовательно, A бесконечно, и поэтому счетно.
2.3 Если все вещественные числа являются алгебраическими, то их множество счетно (упр. 2), что
противоречит следствию из теоремы 2.43.
2.4 Если множество иррациональных вещественных чисел счетно, то множество всех вещественных
чисел является объединением двух счетных множеств и потому также счетно, что противоречит следствию из теоремы 2.43.
2.5 Пусть E — множество чисел вида a+1/n, где a = 1, 2, 3, а n = 2, 3, 4, . . . . Очевидно, оно ограничено,
так как является подмножеством сегмента [1, 7/2].
Тогда при x = 1, 2, 3 каждая окрестность Nr (x) содержит точку x + 1/ max(n, 2), принадлежащую
E, где n — целое положительное число, такое, что n − 1 ≤ 1/r < n. Это означает, что точки 1, 2, 3
являются предельными точками множества E.
Для прочих x определим r таким образом:
(a)
(b)
(c)
(d)
r
r
r
r
= 1 − x, если x < 1;
= min(x − 1, 2 − x)/2, если 1 < x < 2;
= min(x − 2, 3 − x)/2, если 2 < x < 3;
= (x − 3)/2, если 3 < x.
Легко убедиться, что Nr (x) при таком определении r содержит лишь конечное число точек множества E. Это означает, что прочие точки не являются предельными точками множества E.
2.6 Пусть x ∈ Ē ′ . Это означает, что для любой окрестности N (x) существует точка y ∈ N (x), такая,
что y ∈ Ē и y ̸= x. Так как N (x) — открытое множество (по теореме 2.19), существует окрестность
N ′ (y) ⊂ N (x) (определение 2.18 (ef)). Так как y ∈ Ē, то либо (i) y ∈ E, либо (ii) y ∈ E ′ и по теореме 2.20
6
существует z ∈ N ′ (y), такая, что z ∈ E и z ̸= x. В любом случае мы нашли в N (x) точку множества
E, не совпадающую с x, то есть x ∈ E ′ . Следовательно, Ē ′ ⊂ E ′ .
Из определения предельной точки следует, что если A ⊂ B, то A′ ⊂ B ′ . Комбинируя это с доказанным утверждением, получаем:
(a) E ′ ⊂ Ē, следовательно, E ′′ ⊂ Ē ′ ⊂ E ′ , и E ′ замкнуто;
(b) E ⊂ Ē, следовательно, Ē ′ ⊂ E ′ ⊂ Ē ′ , то есть E ′ = Ē ′ .
Для множества E из упражнения 2.5 E ′ = {1, 2, 3}, а E ′′ пусто.
2.7 Из определения предельной точки следует, что если Ai ⊂ B, то Āi ⊂ B̄. Следовательно,
S
i Āi ⊂ B̄
как для конечного, так и для счетного объединения.
Sn
Пусть теперь x ∈
/ i=1 Āi . Это означает, что для каждого i существует окрестность Ni (x), не
содержащая точек множества Ai . Пересечение этих окрестностей по теореме 2.24(c) является открытым
множеством и поэтому содержит
окрестность точки x, не содержащую точек множества B, то есть
Sn
x∈
/ B̄. Следовательно, B̄ ⊂ i=1 Āi .
Пусть {ri } — расположенные в последовательность
S∞ все рациональные числа. Пусть Ai для каждого
i — множество из единственной точки ri . Тогда i=1 Āi = Q, а B̄ = R.
2.8 Каждая точка x открытого множества E является его внутренней точкой, то есть Nr′ (x) ⊂ E
′
)
при некотором r′ > 0. Для каждой окрестности Nr (x) имеем, например, x + min(r,r
e1 ⊂ E ∩ Nr (x).
2
Следовательно, x является предельной точкой множества E.
Любое одноточечное подмножество R2 замкнуто, но его множество предельных точек пусто.
2.9
(a) Пусть p ∈ E ◦ . Существует окрестность N (p) ⊂ E. Так как окрестность является открытым
множеством, для любой точки q ∈ N (p) существует окрестность, целиком лежащая в N (p) и тем
более в E, то есть q — внутренняя точка E, другими словами, q ∈ E ◦ . Это означает, что p —
внутренняя точка E ◦ . Значит, все точки множества E ◦ внутренние, то есть оно открыто.
(b) Если E ◦ = E, то E открыто благодаря (a). Если же E открыто, то все его точки являются
внутренними, то есть E ⊂ E ◦ . Так как очевидно, что E ◦ ⊂ E, имеем E ◦ = E.
(c) G = G◦ ⊂ E ◦ .
(d) Из (a) и (c) следует, что E ◦ — наибольшее открытое подмножество пространства X, содержащееся в E. Переходя к дополнениям, заключаем, что E ◦c — наименьшее замкнутое подмножество
пространства X, содержащее E c . Но это совпадает с заключением теоремы 2.27 для E c . Следовательно, E ◦c = E c .
(e) На прямой Q◦ пусто, а Q̄◦ = R.
(f) На прямой Q̄ = R, а Q◦ пусто.
2.10 Пункты (a) и (b) определения 2.15 тривиально выполняются. Если в пункте (c) все три точки
p, q и r совпадают, то обе части неравенства равны нулю, в противном случае левая часть не больше
1, а правая — не меньше 1. Следовательно, пункт (c) также выполняется.
Одной из окрестностей точки x является множество N1 (x), состоящее из самой этой точки x. Объединением таких множеств можно получить любое подмножество пространства X, то есть все подмножества пространства X открыты. Если брать их дополнения, то также можно получить любое
подмножество пространства X, то есть все подмножества пространства X также замкнуты.
Для любого бесконечного подмножества пространства X из его покрытия одноэлементными множествами нельзя выделить конечное подпокрытие, поэтому компактны только все конечные подмножества пространства X.
7
2.11
(a) Нарушено условие (c): d1 (0, 2) > d1 (0, 1) + d1 (1, 2).
(b) Условия (a) и (b) очевидным образом выполняются.
(d2 (p, r) + d2 (r, q))2 = (
p
p
|p − r| + |r − q|)2
≥ |p − r| + |r − q| ≥ |p − q|
p
2
= |p − q| = (d2 (p, q))2 ,
то есть условие (c) также выполняется.
(c) Нарушено условие (a): d3 (1, −1) = 0.
(d) Нарушено условие (a): d4 (1, 1) ̸= 0.
(e) Условия (a) и (b) очевидным образом выполняются.
|p − r|
|r − q|
+
1 + |p − r| 1 + |r − q|
1 − |p − r||r − q|
=1−
1 + |p − r| + |r − q| + |p − r||r − q|
1
≥1−
1 + |p − q|
|p − q|
=
= d5 (p, q),
1 + |p − q|
d5 (p, r) + d5 (r, q) =
то есть условие (c) также выполняется.
2.12 Пусть {Gα } — открытое покрытие множества K. Существует некоторое α0 , для которого 0 ∈
Gα0 . Так как Gα0 — открытое множество, существует r > 0, такое, что (−r, r) ⊂ Gα0 . Обозначим N
наименьшее положительное целое число, такое, что N > 1/r. Тогда при i > N число 1/i лежит в Gα0 ,
а при каждом 1 < i ≤ N существует αi , для которого 1/i ∈ Gαi . В результате имеем
K ⊂ Gα 0 ∪ Gα 1 ∪ · · · Gα N .
2.13 Пусть K состоит из точек 1/n (n = 1, 2, . . . ), 1/n + 1/m (n = 1, 2, . . . ; m = n2 , n2 + 1, . . . )
и 0. Предельными точками K являются точки 1/n (n = 1, 2, . . . ) и 0 (доказательство аналогично
упражнению 2.5). Это означает, что K замкнуто. Так как K ограничено (K ⊂ [0, 2]), оно компактно.
2.14 Пусть Gn — последовательность интервалов (1/n, 1). Легко видеть, что Gn — открытое покрытие
интервала (0, 1). Но любое конечное подсемейство этой последовательности имеет своим объединением
Gk1 ∪ Gk2 ∪ · · · ∪ Gkn = Gmax(k1 ,k2 ,...,kn ) ,
то есть не является покрытием интервала (0, 1).
2.15 Пусть Fn = [n, +∞). Тогда {Fn } — последовательность непустых замкнутых вложенных множеств, имеющих пустое пересечение.
Пусть Bn = (0, 1/n). Тогда {Bn } — последовательность непустых ограниченных вложенных множеств, имеющих пустое пересечение.
8
2.16 Множество E может быть записано как подмножество R:
√
√
√ √
E = Q ∩ ((− 3, − 2) ∪ ( 2, 3)),
поэтому оно открыто в Q по теореме 2.30. Используя плотность пространства Q, легко доказать, что
из его покрытия открытыми множествами
r
Un = Q ∩ ((−
√
1 √
3 − , − 2) ∪ ( 2,
n
r
3−
1
))
n
нельзя выделить конечное подпокрытие. Дополнение
√ √
√
√
E c = Q ∩ ((−∞, − 3) ∪ (− 2, 2) ∪ ( 3, +∞))
также открыто в Q, поэтому E замкнуто в Q. Очевидно, что E ограничено, так как E ⊂ [−2, 2].
2.17 Несчетность E легко доказывается аналогично теореме 2.14 (или по теореме 2.43, после того
8
4
, 10
], оно не является всюду плотным в [0, 1].
как доказана совершенность). Так как E ⊂ [ 10
Если x ∈
/ E, то некоторая цифра xn в десятичном разложении числа x отлична от 4 и 7. Легко
проверить, что интервал
1
1
x − n+1 , x + n+1
10
10
не содержит точек множества E, следовательно, E замкнуто. Так как E ограничено, оно компактно.
Если x ∈ E и d > 0, то можно выбрать положительное целое n таким, что 3 · 10−n < d. По
определению множества E либо x + 3 · 10−n , либо x − 3 · 10−n принадлежит E (в зависимости от того,
равна n-я цифра числа x после запятой 4 или 7). Следовательно, в E нет изолированных точек и оно
совершенно.
2.18 Сдвинем множество E из предыдущей задачи на 0, 1010010001 · · · .
2.19
(a) A ∩ B̄ = A ∩ B пусто, Ā ∩ B = A ∩ B также пусто.
(b) Так как Ac замкнуто и B ⊂ Ac , имеем B̄ ⊂ Ac , то есть A ∩ B̄ пусто. Аналогично с перестановкой
A и B.
(c) Множества A и B открыты и не пересекаются, поэтому результат следует из (b).
(d) Пусть p и r — какие-нибудь две точки пространства X. Для каждого положительного δ, такого,
что δ < d(p, r), множества A и B из (c) не могут составлять покрытие X, так как они непусты
(p ∈ A, r ∈ B), а X связно. Это означает, что существует точка xδ ∈ (A ∪ B)c , для которой
d(p, xδ ) = δ. Следовательно, в пространстве X существует подмножество, эквивалентное сегменту
[0, d(p, r)] , то есть X несчетно.
2.20 Пусть Ē несвязно. Тогда Ē = A ∪ B, где A и B — непустые отделенные друг от друга множества.
Если B ⊃ E, то A ∩ B̄ ⊃ A ∩ Ē непусто, что противоречит отделенности A и B. Значит, A ∩ E непусто.
¯ E ⊂ Ā и B ∩
¯ E ⊂ B̄, то A ∩ E и B ∩ E — непустые
Аналогично с перестановкой A и B. Так как A ∩
отделенные друг от друга подмножества, объединение которых равно E, и E несвязно.
С другой стороны, внутренность связного множества не обязана быть связной. Примером служат
два касающихся круга на плоскости.
9
2.21
(a) Пусть x ∈ Ā0 , а Nr — окрестность p(x). Тогда Nr/∥b−a∥ (x) — окрестность точки x, по предположению содержащая точку y множества A0 . Имеем ∥p(y) − p(x)∥ = ∥b − a∥|y − x| < r, то есть
p(y) ∈ Nr ∩ A. Значит, p(x) ∈ Ā, то есть p(Ā0 ) ⊂ Ā. Аналогично p(B̄0 ) ⊂ B̄.
Так как p(A0 ∩ B̄0 ) ⊂ A ∩ B̄ и p(Ā0 ∩ B0 ) ⊂ Ā ∩ B пусты, то множества A0 и B0 отделены друг
от друга.
(b) Заметим, что 1 ∈ A0 и 0 ∈ B0 . Так как множество (A0 ∩ [0, 1]) ∪ (B0 ∩ [0, 1]) несвязно, по теореме
2.47 оно не может совпадать с сегментом [0, 1].
(c) При любом разбиении выпуклого множества на два непустых непересекающихся множества A и
B не выполняется (b), следовательно, A и B не могут быть отделенными друг от друга.
2.22 Пусть Nδ (x) — окрестность
точки x в Rk . Она содержит k-мерную открытую клетку, задаваемую
√
√
условиями yi ∈ (xi − δ/ k, xi + δ/ k). Каждый такой интервал содержит рациональное число ri , и эти
числа являются координатами точки r ∈ Nδ (x). Таким образом, в каждой окрестности каждой точки
пространства Rk лежит точка из счетного множества {rn } точек с рациональными координатами. Это
означает, что {rn } — счетное всюду плотное множество.
2.23 Пусть {bn } — счетное всюдуSплотное множество пространства X. Возьмем в качестве {Vα }
объединение семейств окрестностей j {Nri (bj )}, где {ri } — расположенные в последовательность все
рациональные числа. Это семейство счетно по теореме 2.12.
Заметим на будущее, что Nr (x) ⊂ Nr+d(x,y) (y).
Пусть G — открытое подмножество X. Это означает, что для любого x ∈ G существует окрестность Ns (x) ⊂ G. В силу плотности семейства {bn } в нем существует точка bj ∈ Ns/2 (x). Выберем
рациональное число ri так, чтобы d(x, bj ) < ri < s/2. Тогда x ∈ Nri (bj ) ⊂ Ns (x) ⊂ G.
2.24 Если приведенное в указании построение не завершается за конечное число шагов, мы получаем последовательность точек {xn }, для которой d(xi , xj ) > δ при любых i и j. Это означает, что для
любой точки p пространства X ее окрестность Nδ/2 (p) содержит не более одной точки этой последовательности, то есть {xn } не имеет предельных точек, что противоречит условию.
Обозначим результат
построения для δ = 1/k (k = 1, 2, 3, . . . ) через Bk . Объединение этих конечных
S
множеств B = k Bk счетно. Возьмем точку p ∈ E и какую-нибудь ее окрестность Nδ (p). Выберем
положительное целое k так, что k > 1/δ. По построению множества Bk существует точка x ∈ Bk ,
такая, что d(x, p) < 1/k < δ. Это означает, что B всюду плотно в X.
2.25 Для каждого n = 1, 2, 3, . . . рассмотрим открытое покрытие пространства K окрестностями
{N1/n (x)}, где x пробегает все точки пространства K. Так как K компактно, из этого покрытия можно
выбрать
конечное подпокрытие Bn = {N1/n (xni )}. Рассмотрим объединение этих подпокрытий B =
S
{N
(x
1/n ni )}. Очевидно, что B счетно. Докажем, что B является базой пространства K.
n
Пусть G — открытое множество в K и x ∈ G. По определению открытого множества существует
окрестность Nδ (x) ⊂ G. Возьмем положительное целое n, такое, что n > 2/δ. В покрытии Bn найдется окрестность N1/n (y), содержащая точку x. Имеем x ∈ N1/n (y) ⊂ Nδ (x) ⊂ G, что и требовалось
доказать.
Пусть теперь {Vα } — счетная база пространства K. Выберем из каждого множества базы Vα точку
xα (если база состоит из окрестностей, как в первой части задачи, можно взять центр каждой окрестности). Если N (x) — произвольная окрестность, то по свойству базы найдется некоторое α, для которого
xα ∈ Vα ⊂ N (x), то есть {xα } является счетным всюду плотным множеством пространства K. 1
1 Эту задачу можно решить и проще, если не пользоваться указанием. По теореме 2.37 к K применима задача 24, то есть K
сепарабельно, а задача 23 показывает, что оно имеет счетную базу.
10
2.26 E бесконечно, так как каждый его элемент
может принадлежать только конечному числу мноT
жеств Fn (иначе он будет принадлежать и Fn , что противоречит предположению о его пустоте).
Следовательно, по предположению E имеет предельную точку x. При каждом n множество E ∩ Fn
получается из E выбрасыванием конечного числа точек, поэтому x является также предельной
точT
кой E ∩ Fn , а значит, x ∈ Fn , так как Fn замкнуто. Но из этого следует, что x ∈ Fn , что снова
противоречит предположению о его пустоте.
2.27 W c — множество точек p, для которых каждое множество базы Vn , содержащее p, содержит
несчетное число точек множества E. Так как любое множество базы, содержащее p, содержит окрестность точки p, то P ⊂ W c . А так как любая окрестность точки p содержит множество базы, содержащее
p, то W c ⊂ P . В результате
имеем P = W c .
S
c
P ∩ E = E ∩ W = (E ∩ Vn ) — счетное объединение не более чем счетных множеств, поэтому оно
не более чем счетно.
P замкнуто, так как W открыто. Если точка p ∈ P является изолированной, то существует ее
окрестность N (p), целиком, кроме точки p, лежащая в W . Значит, E ∩ N (p) ⊂ E ∩ W ∪ {p} не более чем
счетно, то есть p не является точкой конденсации множества E, что противоречит предположению.
Следовательно, P совершенно.
Эти рассуждения не используют свойств Rk и справедливы для любого метрического пространства
со счетной базой.
2.28 Замкнутое множество содержит все свои точки конденсации, так как они являются предельными, поэтому вывод следует из упражнения 27.
Если счетное замкнутое множество в Rk не имеет изолированных точек, то оно совершенно, что
противоречит теореме 2.43.
2.29 Пусть {rn } — счетное всюду плотное подмножество данного открытого множества E, например,
расположенные в последовательность рациональные числа, лежащие в E. Обозначим In объединение
всех интервалов, лежащих в E и содержащих rn . In — открытое множество, и легко убедиться, что
In = (inf In , sup In ) также является интервалом (в смысле расширенной числовой прямой).
Если p ∈ E, то существует окрестность N (p) ⊂ E, содержащаяSточку p. В силу плотности в ней
содержится какое-нибудь rn , то есть p ∈ In . Это означает, что E = In .
Если In ∩ Im непусто, то [rn , rm ] ⊂ E (или [rm , rn ] ⊂ E, если m < n). Это означает, что In и Im либо
не пересекаются, либо совпадают.
Удалив повторяющиеся элементы, как в теореме 2.12, мы получаем представление множества E в
виде объединения не более чем счетного семейства попарно непересекающихся интервалов.
2.30 Эквивалентность утверждений очевидна, если положить Gn = Fnc .
Докажем второе утверждение.
Пусть V1 — какая-нибудь окрестность.
Поскольку множество Gn открыто и всюду плотно, существует окрестность Vn+1 , такая, что V̄n+1 ⊂
Vn ∩ Gn . Продолжаем по индукции.
T
Будучи замкнутым
T и ограниченным, V̄n компактно. По следствию из теоремы 2.36 V̄n непусто,
что означает, что
Gn также непусто (а так как окрестность V1 можно выбрать произвольно и она
T
пересекается с Gn , то последнее всюду плотно).
11
Глава 3
Числовые последовательности и
ряды
3.1 Это следует из неравенства ||sn | − |s|| ≤ |sn − s| (упр. 1.13).
Контрпримером служит последовательность sn = (−1)n .
3.2
lim (
n→∞
(limn→∞
p
p
n2 + n − n) = lim √
n→∞
1 + 1/n = 1, так как 1 <
n2
n
1
= lim p
=
n→∞
+n+n
1 + 1/n + 1
1
2
p
1 + 1/n < 1 + 1/n).
3.3 Докажем по индукции, что sn < sn+1 < 2 для всех n. Действительно, s1 < s2 < 2, а если
sk < sk+1 < 2, то
q
sk+1 =
2+
√
sk < sk+2 =
q
q
√
√
2 + sk+1 < 2 + 2 < 2.
Сходимость следует из теоремы 3.14.
3.4 s2m+1 = (1 + s2m−1 )/2, то есть 1 − s2m+1 = (1 − s2m−1 )/2 и s2m+1 = 1 − 2−m , а s2m = 1/2 − 2−m .
У последовательности всего два частичных предела, и limn→∞ sn = 1, limn→∞ sn = 1/2.
3.5 Обозначим левую часть C, а слагаемые в правой части A и B.
Если A или B (или оба) равны +∞, то доказывать нечего. Если хотя бы одно из них равно −∞, то
an + bn → −∞.
Если правая часть конечна, то для любого ε > 0 по теореме 3.17 найдется такое N1 , что an < A+ε/2
при n ≥ N1 и такое N2 , что bn < B + ε/2 при n ≥ N2 . Тогда при n ≥ max(N1 , N2 ) выполняется
an + bn < A + B + ε. Следовательно, C ≤ A + B + ε, а так как ε произвольно, an + bn ≤ A + B.
3.6
(a) Ряд расходится, так как расходится последовательность его частичных сумм sn =
(b)
1
1
an = √
√ < 3/2 ,
n
n( n + 1 + n)
и ряд сходится по теоремам 3.25(a) и 3.28.
(c) Ряд сходится по признаку Коши.
12
√
n + 1 − 1.
(d) Если |z| ≤ 1, то |an | ≥ 1/2, и ряд расходится по теореме 3.23. Если же |z| > 1, то начиная с
какого-то номера N выполняется |z|n > 2, то есть |1 + z n | ≥ |z|n − 1 > |z|n /2 и |an | < 2/|z|n , так
что ряд сходится по теоремам 3.25(a) и 3.26.
√
3.7 Из неравенства ( an − 1/n)2 ≥ 0 получаем, что
√
an
1
≤
n
2
an +
1
n2
.
Результат следует из теорем 3.25, 3.28 и 3.47.
3.8 Будем считать, что {bn } монотонно убывает (для возрастающей
последовательности
P
P
Pдоказательство получается сменой знака). Пусть bn → b. Тогда в равенстве
a n bn =
an (bn − b) + an b первая
сумма в правой части сходится по теореме 3.42, следовательно, вся правая часть сходится, а значит, и
левая.
3.9
(a) R = 1/ limn→∞
(b) R = 1/ limn→∞
(c) R = 1/ limn→∞
(d) R = 1/ limn→∞
√
n
n3 = 1.
= +∞.
q
n 2n
2 = 1/2.
qn
n n3
3n = 3.
2n+1 n!
(n+1)! 2n
3.10 При |z| > 1 ряд расходится по теореме 3.23.
3.11
(a) Если последовательность {an } ограничена, то есть an ≤ M при всех n, то
an
1
an ,
≥
1 + an
1+M
и искомый ряд расходится по признаку сравнения. В противном случае члены этого ряда не
стремятся к 0 и он также расходится.
(b)
а так как sN +k
Коши.
(c) Если n ≥ 2, то
aN +1
aN +k
aN +1
aN +k
sN
+ ··· +
≥
+ ··· +
=1−
,
sN +1
sN +k
sN +k
sN +k
sN +k
P an
→ ∞ при k → ∞, правая часть стремится к 1 и ряд
sn расходится по критерию
1
1
sn − sn−1
an
an
−
=
=
≥ 2.
sn−1
sn
sn−1 sn
sn−1 sn
sn
P∞ 1
Так как sn → ∞, ряд n=2 sn−1 − s1n сходится (а именно к a11 ), следовательно, по признаку
P an
сравнения сходится и ряд
s2n .
P an
(d) Если последовательность {nan } ограничена сверху, то ряд
1+nan расходится (доказательство
an
ε 1
аналогично (a)). P
Если nan ≥ ε > 0, тоP1+nan ≥ 1+ε n , и ряд также расходится. Но можно взять
an
сходящийся ряд P
an (очевидно, ряд P 1+na
также будет сходиться) и «испортить» некоторые
an n
члены, чтобы ряд
an расходился, а
1+nan остался сходящимся. Например:
(
1/n3 , если n не является полным кубом,
an = √
3
n,
если n является полным кубом.
13
Ряд
P
an
1+n2 an
сходится, так как
an
1+n2 an
<
1
n2 .
3.12
(a)
(b)
am
an
am
an
rm − rn+1
rn+1
rn
+ ··· +
>
+ ··· +
=
=1−
>1−
,
rm
rn
rm
rm
rm
rm
rm
P an
а так как rn → 0 при n → ∞, правая часть стремится к 1 и ряд
rn расходится по критерию
Коши.
√
an
rn − rn+1
√
2( rn − rn+1 ) = 2 √
>√ .
√
rn + rn+1
rn
P√
√
√
Так как rn → 0, ряд ( P
rn − rn+1 ) сходится (а именно к r1 ), следовательно, по признаку
an
√
сравнения сходится и ряд
rn .
3.13
n
X
|ck | = |a0 b0 | + |a0 b1 + a1 b0 | + · · · + |a0 bn + a1 bn−1 + · · · + an b0 |
k=0
≤ |a0 ||b0 | + |a0 ||b1 | + |a1 ||b0 | + · · · + |a0 ||bn | + |a1 ||bn−1 | + · · · + |an ||b0 |
n
n
X
X
=
|ak |
|bk |.
k=0
k=0
3.14
(a) Пусть задано ε > 0. Существует положительное целое L такое, что |sn −s| < ε/2 при n ≥ L. Кроме
того, последовательность {sn − s} ограничена, то есть существует M такое, что |sn − s| ≤ M для
всех n. Тогда при n ≥ max(L, 2LM
ε ) имеем
1
|σn − s| ≤
n+1
L−1
X
|si − s| +
i=0
n
X
!
|si − s|
i=L
<
ε ε ε
1 LM + (n + 1 − L)
< + = ε.
n+1
2
2 2
(b) sn = (−1)n .
(c) Возьмем в качестве {sn } последовательность {an } из упражнения 3.11(d) (положим s0 = 0).
Тогда, обозначив за m такое целое число, что m3 ≤ n < (m + 1)3 , получим
σn ≤
1 X 1
1 X
1 X 1
1
m(m + 1)
+
k
≤
+ 3
→0
n+1
k3
n+1
n+1
k3
m +1
2
n
m
n
k=1
k=1
k=1
при n → ∞.
(d)
s0 + s1 + · · · + sn
n+1
(n + 1)(s0 + a1 + · · · + an ) − s0 − (s0 + a1 ) − · · · − (s0 + a1 + · · · + an )
=
n+1
a1 + · · · + kak
=
→0
n+1
s n − σn = s n −
при n → ∞ вследствие (a), примененного к последовательности {nan }.
14
(e)
m+1
n+1
σn + s n −
σn
n−m
n−m
m+1
s0 + s1 + · · · + sn
=
σn + s n −
n−m
n−m
m+1
s0 + s1 + · · · + sm
(n − m)sn − sm+1 − · · · − sn
=
σn −
+
n−m
n−m
n−m
n
X
m+1
1
=
(σn − σm ) +
(sn − si ).
n−m
n − m i=m+1
s n − σn =
|sn − si | = |ai+1 + · · · + an |
(n − i)M
(так как |ak | ≤ M /k)
i+1
(n − m − 1)M
≤
(так как i ≥ m + 1).
m+2
≤
Если выбрать m так, что
n−ε
< m + 1,
1+ε
то, с одной стороны, (m + 1)/(n − m) ≤ 1/ε, а с другой, (n − m − 1)/(m + 2) < ε. Итого имеем
m≤
|sn − σ| ≤ |sn − σn | + |σn − σ| <
1
|σn − σm | + M ε + |σn − σ|.
ε
Переходя к пределу при n → ∞ (из второго неравенства для m следует, что при этом также
m → ∞), получаем
lim |sn − σ| ≤ M ε.
n→∞
Так как ε было произвольным, lim sn = σ.
3.15
3.22’ Теорема. Ряд
N , такое, что
P
an сходится тогда и только тогда, когда для любого ε > 0 существует целое
m
X
ak ≤ ε,
k=n
если m ≥ n ≥ N .
Эта теорема, как и прежде, является переформулировкой критерия Коши. Взяв m = n, получаем
|an − 0| ≤ ε
(n ≥ N ).
Иными словами,
3.23’ Теорема. Если ряд
P
an сходится, то limn→∞ an = 0.
Доказательства остальных теорем полностью аналогичны исходным, поэтому ограничимся формулировками.
3.25’(a) Теорема. Если P
|an | ≤ cn при n ≥ N0 , где N0 — некоторое фиксированное целое, и если ряд
P
cn сходится, то и ряд
an сходится.
15
p
P
3.33’ Теорема. Пусть задан ряд
an . Положим α = lim n |an |. Тогда
n→∞
P
(a) если α < 1, то ряд P an сходится;
(b) если α > 1, то ряд
an расходится;
(c) если α = 1, то признак не дает ответа.
3.34’ Теорема. Ряд
P
an
|an+1 |
< 1,
|an |
|an+1 |
(b) расходится, если
≥ 1 при n ≥ n0 , где n0 — некоторое фиксированное целое число.
|an |
(a) сходится, если lim
n→∞
3.42’ Теорема. Допустим, что
P
(a) частные суммы An ряда
an образуют ограниченную последовательность;
(b) b0 ≥ b1 ≥ b2 ≥ · · · ;
(c) lim bn = 0.
n→∞
P
Тогда ряд
bn an сходится.
3.45’ Теорема. Если ряд
3.47’
Теорема. Если
P
d · an = d · A.
P
P
an сходится абсолютно, то он сходится.
an = A,
3.55’ Теорема. Если ряд
одной и той же сумме.
P
P
bn = B, c ∈ R и d ∈ Rk , то
P
(an + bn ) = A + B,
P
can = cA и
an сходится абсолютно, то все его перестановки сходятся, причем к
3.16
(a) Имеем
√
√
(xn − α)2
α
xn +
−2 α =
> 0,
xn
2xn
√
из чего по индукции получаем, что xn > α при всех n. Далее,
1
α
α − x2n
− 2xn =
< 0,
xn+1 − xn =
xn +
2
xn
2xn
xn+1 −
√
α=
1
2
то есть последовательность {xn } монотонно убывает. По теореме 3.14 она имеет предел. Переходя
к пределу в исходном равенстве, получаем
1
α
lim xn =
lim xn +
,
2
lim xn
√
то есть lim xn = α.
(b)
√
ε2
(xn − α)2
< √n ,
2xn
2 α
то есть
2
εn+1
εn
<
.
β
β
Требуемое утверждение
получается по индукции.
√
√
3
3
, так как 3 > 17
3 < 2. Следовательно, ε1 /β <
(c) ε1 < 10
10 . Далее, 3 < β < 4, так как 2 <
εn+1 = xn+1 −
√
α=
1 2
ε5 < 4( 10
) < 4 · 10−16 ,
4
16
1 2
ε6 < 4( 10
) < 4 · 10−32 .
5
1
10 .
3.17 Имеем
√
√
α + xn √
α−1
− α=−
(xn − α),
1 + xn
1 + xn
√
√
из чего по индукции получаем, что x2n−1 > α, а x2n < α при всех n. Далее,
xn+1 −
√
α=
xn+2 − xn =
α+
1+
α+xn
1+xn
α+xn
1+xn
− xn =
2(α − x2n )
,
α + 2xn + 1
что дает (a) и (b).
Так как подпоследовательности {x2n−1 } и {x2n } монотонны и ограничены, они сходятся, то есть
|xn+2 − xn | → 0 как при четных, так и при нечетных n. Следовательно,
|xn −
√
α| =
|x2n − α|
α + 2xn + 1
α + 2x1 + 1
√ =
√ |xn+2 − xn | ≤
√
|xn+2 − xn | → 0,
xn + α
2(xn + α)
2 α
что доказывает (c).
√
Положим εn = |xn − α|. Тогда
√
√
α−1
α−1
εn+1
=
≥
,
εn
1 + xn
1 + x1
то есть сходимость не является такой быстрой, как в предыдущем упражнении.
3.18 Обозначим
sk (t) =
1 − tk
= 1 + t + · · · + tk−1 .
1−t
Понятно, что sk (t) ≤ k при t ≤ 1.
√
Пусть xn > p α. Имеем
√
p−1 !
√
p
p
√
p−1
α
α
p
xn+1 − α =
(xn − α) −
1−
p
p
xn
√
√
√
p
p
p
√
α
α
α
p−1
p
(xn − α) −
=
1−
sp−1
p
p
xn
xn
√
√
p
p
√
α
α
p−1
(xn − p α) −
1−
≥
(p − 1)
p
p
xn
√
p − 1 (xn − p α)2
=
> 0,
p
xn
√
из чего по индукции получаем, что xn > p α при всех n. Далее,
√
p
p−1
α
xn
xn+1 − xn =
xn + x−p+1
− xn = −
p
p n
p
√
p p
α
1−
< 0,
xn
то есть последовательность {xn } монотонно убывает. Переходя к пределу, получаем
lim xn =
то есть lim xn =
√
p
p−1
α
lim xn + (lim xn )−p+1 ,
p
p
α.
17
Положим εn = xn −
√
p
α. Имеем
εn+1
Обозначая β =
2
p−1
√
p
p−1 !
√
√
p
p
p−1
α
α
=
εn −
1−
p
p
xn
√
√
√
p
p
p
p−1
α
α
α
=
εn −
1−
sp−1
p
p
xn
xn
√
√
p
p
εn
α
α
=
p−1−
sp−1
p
xn
xn
√
p−1 !!
√
p
p
εn
α
α
1−
+ ··· + 1 −
=
p
xn
xn
√
√
√
p
p
p
εn
α
α
α
=
1−
s1
+ · · · + sp−1
p
xn
xn
xn
2
p−1 2
p−1 2
ε p(p − 1)
=
ε < √
ε .
≤ n
pxn
2
2xn n
2pα n
α, имеем
εn+1 < β
ε1
β
2 n
(n = 1, 2, 3, . . . ).
3.19 Нетрудно заметить, что 0,α1 α2 α3 . . . — троичное разложение числа x(a) (см. пункт 1.22). Для
каждого n множество En содержит те числа, для которых существует троичное разложение, первые n
разрядов которого не содержат единиц (включая «неправильные» троичные разложения с бесконечным
хвостом двоек, например, 31 = 0,1000 . . .3 , но существует и разложение без единицы: 13 = 0,0222 . . .3 ).
Множество Кантора — пересечение всех En — содержит в точности те числа, для которых существуют
троичные разложения, не содержащие единиц.
3.20 Пусть ε > 0. Возьмем M таким, чтобы при i ≥ M выполнялось d(pni , p) < ε/2, и N ≥ M таким,
чтобы при m, n ≥ N выполнялось d(pn , pm ) < ε/2. Так как nN ≥ N , при n ≥ N получаем
d(pn , p) ≤ d(pn , pnN ) + d(pnN , p) < ε.
3.21 Выберем в каждом En точку pn . Так как при n, m ≥ N выполняется d(pn , pm ) ≤ diam EN , {pn }
является последовательностью Коши. Обозначим ее предел p. Точка p является предельной точкой
каждого En , следовательно, их пересечение непусто. С другой стороны, если точка q принадлежит
пересечению всех En , то для любого n выполняется d(p, q) ≤ diam En , а так как diam En → 0, то q
совпадает с p.
3.22 Утверждение о плотности сводится к следующему:
T пусть дано произвольное открытое мно-
жество E0 , нам надо доказать, что его пересечение с Gn непусто. Построим последовательность
открытых множеств {En } следующим образом.
Пусть дано открытое множество En−1 . Так как Gn всюду плотно, его пересечение с En−1 непусто.
Выберем точку xn ∈ En−1 ∩Gn . Возьмем в качестве En такую окрестность точки xn , что (i) Ēn ⊂ En−1 ;
(ii) Ēn ⊂ Gn ; (iii) diam En < 1/n.
упражнения к последовательности {Ēn } дает точку, принадлежащую
T Применение предыдущего
T
Ēn , следовательно, и Gn , и E0 .
18
3.23 Применив формулу из указания второй раз (поменяв местами n и m) и скомбинировав результаты, получаем
|d(pn , qn ) − d(pm , qm )| ≤ d(pn , pm ) + d(qm , qn ).
Для заданного ε > 0 существует N1 такое, что d(pn , pm ) < ε/2 при n, m ≥ N1 , и N2 такое, что d(qn , qm ) <
ε/2 при n, m ≥ N2 . Получаем, что при n, m ≥ max(N1 , N2 ) выполняется
|d(pn , qn ) − d(pm , qm )| < ε,
то есть {d(pn , qn )} — последовательность Коши в R, следовательно, она сходится.
3.24
(a) Рефлексивность тривиальна, симметричность следует из симметричности функции расстояния,
а транзитивность — из неравенства треугольника.
(b) Пусть {pn } эквивалентна {p′n }, а {qn } эквивалентна {qn′ }. Тогда
lim d(p′n , qn′ ) ≤ lim (d(p′n , pn ) + d(pn , qn ) + d(qn , qn′ )) = lim d(pn , qn ),
n→∞
n→∞
n→∞
точно так же верно обратное неравенство. Проверим аксиомы расстояния:
(a) ∆(P, P ) = limn→∞ d(pn , pn ) = 0; если ∆(P, Q) = 0, то lim d(pn , qn ) = 0, значит, {pn } и {qn }
эквивалентны, то есть P = Q.
(b) ∆(P, Q) = limn→∞ d(pn , qn ) = limn→∞ d(qn , pn ) = ∆(Q, P ).
(c) ∆(P, R) + ∆(R, Q) = limn→∞ (d(pn , rn ) + d(rn , qn )) ≥ limn→∞ d(pn , qn ) = ∆(P, Q).
(c) Пусть {Pk } — последовательность Коши в X ∗ . Выберем для каждого k представитель класса
эквивалентности — последовательность Коши {pkn } ∈ Pk . Для каждого k существует Nk , такое, что при n, m ≥ Nk выполняется d(pkn , pkm ) < 1/k. Обозначим pk = pkNk . Очевидно, что
d(pk , pkn ) < 1/k при n ≥ Nk , так что limn→∞ d(pk , pkn ) ≤ 1/k (используем упражнение 23 с
постоянной последовательностью, все члены которой равны pk ).
Пусть задано ε > 0. Выберем M так, чтобы 1/M < ε/3 и чтобы ∆(Pk , Pl ) < ε/3 при k, l ≥ M .
При k, l ≥ M имеем
d(pk , pl ) ≤ lim (d(pk , pkn ) + d(pkn , pln ) + d(pln , pl )) < 1/k + ε/3 + 1/l ≤ ε.
n→∞
Следовательно, {pk } — последовательность Коши, и она является представителем некоторого
P ∈ X ∗ . Имеем
∆(Pk , P ) = lim d(pkn , pn ) ≤ lim (d(pkn , pk ) + d(pk , pn )) ≤ 1/k + diam Ek ,
n→∞
n→∞
где Ek состоит из точек pk , pk+1 , pk+2 , . . . . Следовательно, limk→∞ ∆(Pk , P ) = 0, то есть Pk → P .
Мы доказали, что X ∗ полно.
(d) ∆(Pp , Pq ) = limn→∞ d(p, q) = d(p, q).
(e) Пусть P ∈ X ∗ , ε > 0. Выберем представитель — последовательность Коши {pn } ∈ P . Выберем N
так, что d(pn , pm ) < ε/2 при n, m ≥ N . Тогда
∆(P, PpN ) = lim d(pn , p) ≤ ε/2 < ε,
n→∞
и P — предельная точка.
Пусть X полно, P ∈ X ∗ и {pn } ∈ P . Последовательность {pn } сходится к некоторой точке p, и из
определения ясно, что P = Pp . Следовательно, φ(X) = X ∗ .
3.25 Множество рациональных чисел с указанной метрикой является всюду плотным подмножеством
R, следовательно, его пополнением служит R (для полной строгости надо доказать уникальность пополнения с точностью до изометрии).
19
Глава 4
Непрерывность
4.1 Нет. Например,
(
f (x) =
1 (x = 0),
0 (x =
̸ 0).
4.2 Пусть x ∈ Ē. Чтобы доказать, что f (x) ∈ f (E), достаточно показать, что любая окрестность
точки f (x) пересекается с f (E). Действительно, пусть O — окрестность точки f (x). Тогда f −1 (O) —
открытое множество, содержащее точку x. Так как x ∈ Ē, существует точка y ∈ f −1 (O) ∩ E. Тогда
f (y) ∈ O ∩ f (E), что и требовалось доказать.
x
1
1
Для вещественной функции x → 1+x
2 и E = [1, +∞) имеем f (Ē) = (0, 2 ], но f (E) = [0, 2 ].
4.3 Z(f ) = f −1 ({0}) замкнуто как прообраз замкнутого множества при непрерывном отображении.
4.4 Из упражнения 2 следует, что f (X) = f (Ē) ⊂ f (E).
Для любой точки x ∈ X существует последовательность {xn } в E, сходящаяся к x, поэтому вторая
часть результата следует из теорем 4.6 и 4.2.
4.5 По упражнению 29 главы 2, E c =
S
i (ai , bi ), где i пробегает конечное или счетное множество
индексов, а интервалы (ai , bi ) попарно не пересекаются, причем могут быть бесконечными. Положим

f (x)
(x ∈ E),



f (a )
(x > ai , bi = +∞),
i
g(x) =

f (bi )
(x < bi , ai = −∞),


 f (bi )−f (ai )
f (ai )bi −f (bi )ai
x
+
(ai < x < bi ).
bi −ai
bi −ai
Заметим, что в последнем случае значения g на концах интервала совпадают с f , а внутри интервала
лежат между ними.
Пусть x ∈ R1 . Если x ∈
/ E, то x лежит в одном из интервалов (ai , bi ), на котором g либо постоянна
(в случаях 2, 3), либо линейна (в случае 4), поэтому непрерывна в точке x.
Если же x ∈ E, то для любого ε > 0 найдется δ > 0, такое, что |f (y)−f (x)| < ε при y ∈ (x−δ, x+δ)∩E.
Для левой границы этого интервала возможны следующие варианты:
(a) x − δ ∈ E. Возьмем δ1 = δ.
(b) ai < x − δ < bi < x. Возьмем δ1 = x − bi .
(c) ai = −∞, bi = x. Возьмем δ1 = δ.
n
(d) ai вещественно, bi = x. Возьмем δ1 = min δ,
bi −ai
f (bi )−f (ai )
20
o
ε .
Аналогично выберем δ2 в зависимости от положения точки x + δ. Легко проверить, что |g(y) − g(x)| < ε
при y ∈ (x − δ1 , x + δ2 ).
Без предположения о замкнутости заключение неверно: если f (x) = 1/x, E = (0, 1), то f нельзя
непрерывно продолжить на все пространство R1 , так как f (0+) не существует.
Для векторнозначных функций результат следует из теоремы 4.10(a).
4.6 Так как метрика на множестве пар точек произвольных метрических пространств не определялась, проведем доказательство для плоскости. Для полного доказательства надо ввести метрику и
обобщить несколько теорем.
Если f непрерывна, то отображение Γ(x) = (x, f (x)) непрерывно по теореме 4.10(a), следовательно,
Γ(E) компактно.
Обратно, предположим, что Γ(E) компактно. Пусть V — замкнутое подмножество пространства
−1
R1 . Множество φ−1
2 (V ) (φi — координатные функции) также замкнуто, следовательно, φ2 (V ) ∩ Γ(E)
−1
компактно. Следовательно, компактно (и тем самым замкнуто) и φ1 (φ2 (V ) ∩ Γ(E)). Но последнее
множество равно f −1 (V ), следовательно, f непрерывна.
4.7 1 − 2f (x) =
g(y 3 , y) =
2
1
2y
1
2,
(x−y 2 )2
x2 +y 4
≥ 0, 1 + 2f (x) =
(x+y 2 )2
x2 +y 4
≥ 0, так что |f (x)| ≤ 21 .
→ ∞ при y → 0.
f (y , y) =
но f (0, 0) = 0.
Очевидно, на любой прямой, не проходящей через начало координат, f и g непрерывны. Прямая,
проходящая через начало координат, имеет вид x = 0 или y = ax.
На прямой x = 0 имеем f (0, y) = g(0, y) = 0 и обе функции также непрерывны.
a2 x
f (x, ax) = 1+a
4 x2 → 0 при x → 0.
g(x, ax) =
a2 x
1+a6 x4
→ 0 при x → 0.
4.8 Так как E ограничено, существует M , такое, что E ⊂ (−M, M ). Так как f равномерно непрерывна,
существует δ > 0, такое, что |f (x) − f (y)| < 1 при x, y ∈ E и |x − y| < δ. Возьмем целое число N , такое,
что N > 2M /δ, и рассмотрим семейство сегментов Ik = [−M + 2(k − 1)/N, −M + 2k/N ], k = 1, . . . , N .
Для каждого k, при котором Ik ∩ E непусто, выберем точку xk ∈ Ik ∩ E. Тогда |f (x)| < |f (xk )| + 1 при
x ∈ Ik ∩ E, следовательно, |f (x)| < max |f (xk )| + 1 при x ∈ E.
Функция f (x) = x равномерно непрерывна на R1 , но не является ограниченной.
4.9 Пусть f равномерно непрерывно на X. Пусть также дано ε > 0. Выберем δ, соответствующее
ε/2 в условии равномерной непрерывности. Если diam E < δ, то для любых x, y ∈ E выполняется
dX (x, y) < δ, то есть dY (f (x), f (y)) < ε/2. Значит, diam f (E) ≤ ε/2 < ε.
Пусть теперь выполняется условие упражнения. Пусть также дано ε > 0. Выберем δ, соответствующее ε, в условии упражнения. Если dX (x, y) < δ, то диаметр множества, состоящего из двух точек
x и y, также меньше δ, из чего следует, что диаметр множества из f (x) и f (y) меньше ε, то есть
dY (f (x), f (y) < ε.
4.10 Если f не является равномерно непрерывной, то существует ε, для которого можно выбрать
последовательности {pn } и {qn }, такие, что dX (pn , qn ) < 1/n, но dY (f (pn ), f (qn )) ≥ ε. По теореме
2.37 существует подпоследовательность {pnk }, сходящаяся к какому-нибудь p ∈ X. Снова применяя
теорему 2.37, получаем подпоследовательность {qnki }, сходящуюся к какому-нибудь q ∈ X. Так как
dX (pnki , qnki ) → 0, имеем p = q. Но тогда из непрерывности f следует, что обе последовательности
{f (pnki )} и {f (qnki )} стремятся к f (p) = f (q), что противоречит неравенству dY (f (pn ), f (qn )) ≥ ε.
21
4.11 Пусть {xn } — последовательность Коши в X. Пусть также дано ε > 0. Выберем δ > 0 так,
чтобы dY (f (x), f (y)) < ε при dX (x, y) < δ. Выберем N так, чтобы dX (xn , xm ) < δ при n, m ≥ N . Тогда
dY (f (xn ), f (xm )) < ε при n, m ≥ N , что и требовалось доказать.
Пусть E — всюду плотное подмножество метрического пространства X, и пусть f — равномерно непрерывная функция из E в полное метрическое пространство Y . Если xn → x, то {xn } — последовательность Коши, следовательно, по доказанному, {f (xn )} — тоже последовательность Коши.
Определим g(x) как предел последовательности Коши {f (xn )}. Для корректности этого определения
надо доказать, что, если xn → x и yn → x, то lim f (xn ) = lim f (yn ). Рассмотрим последовательность
x1 , y1 , x2 , y2 , . . . . Она также стремится к x, значит, последовательность f (x1 ), f (y1 ), f (x2 ), f (y2 ), . . . сходится, и все ее подпоследовательности, в частности, {f (xn )} и {f (yn )}, сходятся к одному и тому же
пределу.
Если x ∈ E, то g(x) — предел постоянной последовательности {f (x)}, то есть g — продолжение f .
Доказательство равномерной непрерывности аналогично упражнению 4.13.
4.12 Пусть f — равномерно непрерывное отображение метрического пространства X в метрическое
пространство Y , а g — равномерно непрерывное отображение пространства Y в метрическое пространство Z. Показать, что g ◦ f — равномерно непрерывное отображение пространства X в Z.
Пусть задано ε > 0. Существует η > 0, такое, что dZ (g(x), g(y)) < ε при dY (x, y) < η. Существует также δ > 0, такое, что dY (f (x), f (y)) < η при dX (p, q) < δ. Итак, если dX (p, q) < δ, то
dZ (g(f (p)), g(f (q))) < ε, то есть g ◦ f равномерно непрерывно.
4.13 Пусть пространство значений — Rk или произвольное компактное метрическое пространство (в
случае произвольного полного метрического пространства можно воспользоваться методом из упражнения 11). Из упражнения 9 следует, что diam f (Vn (p)) → 0, по теореме 3.10(a) имеем diam f (Vn (p)) → 0,
и по теореме 3.10(b) их пересечение состоит из одной точки, которую мы обозначим g(p). Очевидно,
что g(p) = p при p ∈ E.
Единственность следует из упражнения 4. Проверим равномерную непрерывность. Пусть дано ε >
0. Выберем δ > 0 так, чтобы при p, q ∈ E таких, что dX (p, q) < δ, выполнялось dY (f (p), f (q)) < ε/3.
Пусть даны x, y ∈ X такие, что dX (x, y) < δ/3. Выберем p, q ∈ E так, чтобы dX (p, x) < δ/3, dX (q, y) <
δ/3, dY (g(x), f (p)) < ε/3 и dY (g(y), f (q)) < ε/3. Тогда
dX (p, q) ≤ dX (p, x) + dX (x, y) + dX (y, q) < δ,
так что dY (f (p), f (q)) < ε/3. Следовательно,
dY (g(x), g(y)) ≤ dY (g(x), f (p)) + dY (f (p), f (q)) + dY (f (q), g(y)) < ε.
В неполном метрическом пространстве теорема неверна: отображение x → x из Q в Q нельзя
непрерывно продолжить до отображения из R в Q.
4.14 Если f (0) = 0 или f (1) = 1, то доказывать нечего. В противном случае рассмотрим непрерывную
функцию g(x) = x − f (x). Так как g(0) < 0 < g(1), существует точка 0 < x < 1, такая, что g(x) = 0, т.
е. f (x) = x.
4.15 Пусть f не является монотонной, тогда существуют a < b < c, такие, что f (a) < f (b) >
f (c) (или наоборот, тогда доказательство аналогично). По теореме 4.16 имеем M = max f ([a, c]) ≥
f (b) > max(f (a), f (c)), так что максимум достигается во внутренней точке сегмента [a, b], т. е. M =
max f ((a, c)). Эта точка принадлежит f ((a, c)), но не является его внутренней точкой, то есть f ((a, c))
не является открытым в R.
4.16 В нецелых точках как целая, так и дробная часть непрерывны, в целых точках имеют разрыв
первого рода:
[n−] = n − 1,
[n+] = [n] = n,
22
(n−) = 1,
(n+) = (n) = 0.
4.17 Каждой точке x, в которой f (x−) < f (x+), сопоставим тройку (p, q, r) рациональных чисел,
такую, что
(a)
(b)
(c)
(d)
a < q < x < r < b,
f (x−) < p < f (x+),
если t ∈ (q, x), то f (t) < p,
если t ∈ (x, r), то f (t) > p.
Возможность этого следует из определения одностороннего предела.
Допустим, что двум точкам x < x′ соответствует одна и та же тройка (p, q, r). Возьмем произвольное
t ∈ (x, x′ ). Тогда из (c) следует, что f (t) < p (так как t ∈ (q, x′ )), а из (d) следует, что f (t) > p (так как
t ∈ (x, r)) — противоречие. Ситуация f (x−) > f (x+) аналогична.
Если f (x−) = f (x+) < f (x), сопоставим точке x тройку (p, q, r) рациональных чисел, такую, что
(a) a < q < x < r < b,
(b) f (x) > p,
(c) если t ∈ (q, x) ∪ (x, r), то f (t) < p.
Если двум точкам x < x′ соответствует одна и та же тройка (p, q, r), то из (c) следует f (x) < p, что
противоречит (b). Ситуация f (x−) = f (x+) > f (x) аналогична.
Итак, множество точек разрыва первого рода может быть представлено как объединение четырех
не более чем счетных множеств, то есть не более чем счетно.
4.18 Для любого x и любого ε > 0 на интервале (x − 1, x + 1) существует конечное число точек t, для
которых f (t) ≥ 1/ε. Следовательно, limt→x f (t) = 0.
4.19 Пусть xn → x0 . Если lim f (xn ) > f (x0 ), то существует подпоследовательность xnk , такая, что
f (xnk ) > r > f (x0 ) для некоторого рационального r и всех k. Значит, f (tk ) = r для некоторого
xnk > tk > x0 , поэтому tk → x0 . Так как f −1 (r) замкнуто и tk ∈ f −1 (r), имеем x0 ∈ f −1 (r), т.
е. f (x0 ) = r — противоречие. Значит, lim f (xn ) ≤ f (x0 ). Аналогичным образом lim f (xn ) ≥ f (x0 ).
Следовательно, lim f (xn ) = f (x0 ), что и требовалось доказать.
4.20
(a) Услокие ρE (x) = 0 означает, что для любого ε > 0 существует z ∈ E, для которого d(x, z) < ε.
Это условие равносильно x ∈ Ē.
(b) Из указания следует, что ρE (x) − ρE (y) ≤ d(x, y). Меняя местами x и y и комбинируя с предыдущим неравенством, получаем |ρE (x) − ρE (y)| ≤ d(x, y), и в определении равномерной непрерывности можно взять δ = ε.
4.21 Непрерывная функция ρF на компактном множестве K достигает минимума. Этот минимум не
может быть равен нулю, потому что иначе эта точка множества K лежала бы в F̄ = F .
Если F1 — множество точек плоскости (x, y), таких, что y ≤ 0, а F2 — множество точек плоскости
1
(x, y), таких, что y ≥ 1+x
2 , то infx∈F2 ρF1 (x) = 0.
4.22 Знаменатель не может быть равен нулю, так как A и B замкнуты и не пересекаются (см.
упражнение 20(a)). Непрерывность следует из упражнения 20(b). f (p) = 0 тогда и только тогда, когда
ρA (p) = 0, то есть p ∈ Ā = A, аналогично f (p) = 1 тогда и только тогда, когда ρB (p) = 0, то есть
p ∈ B̄ = B. Множества V и W — прообразы открытых непересекающихся подмножеств сегмента [0, 1],
поэтому также открыты и не пересекаются. A = f −1 (0) ⊂ V , B = f −1 (1) ⊂ W .
23
4.23 Подставляя в определение выпуклости λ =
f (t) ≤
t−s
u−s ,
имеем
u−t
t−s
f (s) +
f (u).
u−s
u−s
Вычитая из обеих частей f (s), получаем первое требуемое неравенство, вычитая обе части из f (u) —
второе.
Пусть a < s < x < y < t < b. Из вышеупомянутых неравенств следует, что
f (x) − f (s)
f (y) − f (x)
f (t) − f (x)
≤
≤
.
x−s
y−x
t−x
При y → x получаем f (x+) = f (x). Левый предел доказывается аналогично.
Если f выпукла на (a, b), а g выпукла и возрастает на области значений f , то
g(f (λx + (1 − λ)y)) ≤ g(λf (x) + (1 − λ)f (y))
≤ λg(f (x)) + (1 − λ)g(f (y)),
то есть g ◦ f выпукла на (a, b).
4.24 Докажем выпуклость для всех λ = j/2n , где j = 1, · · · , 2n − 1. Для n = 1 утверждение очевидно.
Пусть оно справедливо для некоторого n, докажем его для n + 1. Если λ = 2k/2n+1 = k/2n , то все уже
доказано. Остается случай λ = (2k + 1)/2n+1 . Тогда имеем
2k + 1
2k + 1
f (λx + (1 − λ)y) = f
x
+
1
−
y
2n+1
2n+1
k+1
!
k
k
k+1
2n x + (1 − 2n )y + 2n x + (1 − 2n )y
=f
2
k
k
k+1
f 2n x + (1 − 2n )y + f k+1
2n x + (1 − 2n )y
≤
2
k
k
k+1
f
(x)
+
(1
−
)f
(y)
+ k+1
n
n
2
2n f (x) + (1 − 2n )f (y)
≤ 2
2
= λf (x) + (1 − λ)f (y).
Так как рассмотренные точки составляют всюду плотное множество интервала (0, 1), а обе части
неравенства непрерывны, для доказательства результата при произвольном λ достаточно перейти к
пределу.
4.25
(a) Точки множества K + C имеют вид x = k + y, где k ∈ K, а y ∈ C. Так как F = z − C, это можно
записать в виде x = k + z − f, где f ∈ F . Тогда d(x, z) = |k + z − f − z| = |k − f| ≥ δ.
(b) То, что C1 и C2 замкнуты и счетны, очевидно. Из этого следует, что C1 + C2 счетно. Пусть
дано x ∈ R1 и ε > 0. Рассмотрим последовательность {an }, где an = (nα) (дробная часть).
Так как α иррационально, все an различны. Эта последовательность имеет предельную точку,
следовательно, найдутся m и n такие, что 0 < β = |am − an | < ε. Очевидно, что kβ ∈ C1 + C2
для любого целого k. Из архимедовости поля R следует существование такого целого k, что
(k − 1)β < x ≤ kβ. Тогда |x − kβ| < ε.
4.26 g(Y ) компактно, поэтому g −1 непрерывно по теореме 4.17. Если h равномерно непрерывно, то
f непрерывно по упражнению 12. Если h непрерывно, то f непрерывно по теореме 4.7.
Пусть X = Z = [0, 1], Y = {0} ∪ [1, ∞), f (x) = 1/x (x ̸= 0), f (0) = 0, g(y) = 1/y (y ̸= 0), g(0) = 0.
Тогда h(x) = x равномерно непрерывно, g непрерывно и взаимно однозначно, но f даже не непрерывно.
24
Download